id
int64
text
string
metadata
dict
line_start_n_end_idx
dict
quality_signals
dict
eai_taxonomy
dict
pid
string
5,216,442,902,267,683,000
LinuxQuestions.org LinuxQuestions.org (/questions/) -   Linux - Newbie (http://www.linuxquestions.org/questions/linux-newbie-8/) -   -   error checking using fsck always gives a strict warning (http://www.linuxquestions.org/questions/linux-newbie-8/error-checking-using-fsck-always-gives-a-strict-warning-4175417766/) rtz 07-20-2012 06:24 AM error checking using fsck always gives a strict warning   my ubuntu OS is not running well after loading.the timer keeps coming and flickers all the time. i cannot move to my home directory niether i can shutdown coz of the flickering of the timer all the time as if some process is still loading but this thing never stops to happen.moreover the look of my desktop has also changed to something like the default ubuntu desktop look.i think there is some disk error coz earlier i was warned by the os for disk checks but i skipped and now things have become worse however when i try to check for disk errors using sudo fsck -As i always get a strict warning from the os "filesystem /dev/sda is still mounted if you continue u will cause severe file system damage" what should i do should i continue or if there's any other way plzz tell. honeybadger 07-20-2012 08:36 AM Checking a mounted partion can (read will) cause data loss. See if you can unmount the partition. Else execute 'shutdown -r -F' as root. This will cause the system to reboot and there will be a file system check on the reboot. Hope this helps. rtz 07-20-2012 09:34 AM ok..system rebooted,file checks were performed sucessfully and no errors were found but the file manager is not starting properly on the destop it shows starting file manager and the icon keeps filckering seems like file manager is not executing properly.is there a way i can disable this execution of file manager on startup besides what's this file manager my os is ubuntu 10.04. rtz 07-20-2012 09:39 AM how do i backup the system generated file and what use does it make on backing up honeybadger 07-20-2012 12:13 PM Here is an idea. Create a new user with "adduser <username>. This will create an entirely new (and perhaps the correct configuration files for the desktop) profile for the user. And when you log in and then configuration files are good then you should be all good. Well as far as the backup goes that is my signature. Something that I learned out of experience. I keep backups of /etc /lib /bin and their counterparts in /usr/local so that in case the machine goes down (because of something I did) then there is a backup I can always back up from. Hope this helps All times are GMT -5. The time now is 08:02 AM.
{ "url": "http://www.linuxquestions.org/questions/linux-newbie-8/error-checking-using-fsck-always-gives-a-strict-warning-4175417766-print/", "source_domain": "www.linuxquestions.org", "snapshot_id": "crawl=CC-MAIN-2015-40", "warc_metadata": { "Content-Length": "7616", "Content-Type": "application/http; msgtype=response", "WARC-Block-Digest": "sha1:CHMA3QZDUBO5MSD4QLH5TNXSYW3ICS2Z", "WARC-Concurrent-To": "<urn:uuid:91c2082a-06c3-4088-821d-b0d8b7fc6b5f>", "WARC-Date": "2015-10-10T13:02:25Z", "WARC-IP-Address": "75.126.162.205", "WARC-Identified-Payload-Type": null, "WARC-Payload-Digest": "sha1:FKQ3GXAJCBKUYPICKFD6YPLWDDBBYMGO", "WARC-Record-ID": "<urn:uuid:08637f1f-fb59-4d6b-a13f-eff967a83332>", "WARC-Target-URI": "http://www.linuxquestions.org/questions/linux-newbie-8/error-checking-using-fsck-always-gives-a-strict-warning-4175417766-print/", "WARC-Truncated": null, "WARC-Type": "response", "WARC-Warcinfo-ID": "<urn:uuid:847ae868-c5ab-43ec-b2af-754a25842f04>" }, "warc_info": "robots: classic\r\nhostname: ip-10-137-6-227.ec2.internal\r\nsoftware: Nutch 1.6 (CC)/CC WarcExport 1.0\r\nisPartOf: CC-MAIN-2015-40\r\noperator: CommonCrawl Admin\r\ndescription: Wide crawl of the web for September 2015\r\npublisher: CommonCrawl\r\nformat: WARC File Format 1.0\r\nconformsTo: http://bibnum.bnf.fr/WARC/WARC_ISO_28500_version1_latestdraft.pdf" }
{ "line_start_idx": [ 0, 19, 20, 53, 130, 319, 320, 344, 345, 401, 403, 1183, 1184, 1216, 1217, 1315, 1444, 1461, 1462, 1486, 1487, 1869, 1870, 1894, 1895, 1977, 1978, 2010, 2011, 2276, 2560, 2576, 2577, 2578 ], "line_end_idx": [ 19, 20, 53, 130, 319, 320, 344, 345, 401, 403, 1183, 1184, 1216, 1217, 1315, 1444, 1461, 1462, 1486, 1487, 1869, 1870, 1894, 1895, 1977, 1978, 2010, 2011, 2276, 2560, 2576, 2577, 2578, 2625 ] }
{ "red_pajama_v2": { "ccnet_original_length": 2625, "ccnet_original_nlines": 33, "rps_doc_curly_bracket": 0, "rps_doc_ldnoobw_words": 0, "rps_doc_lorem_ipsum": 0, "rps_doc_stop_word_fraction": 0.4068376123905182, "rps_doc_ut1_blacklist": 0, "rps_doc_frac_all_caps_words": 0.02222222089767456, "rps_doc_frac_lines_end_with_ellipsis": 0, "rps_doc_frac_no_alph_words": 0.2290598303079605, "rps_doc_frac_unique_words": 0.4988399147987366, "rps_doc_mean_word_length": 4.777262210845947, "rps_doc_num_sentences": 29, "rps_doc_symbol_to_word_ratio": 0, "rps_doc_unigram_entropy": 5.033475399017334, "rps_doc_word_count": 431, "rps_doc_frac_chars_dupe_10grams": 0, "rps_doc_frac_chars_dupe_5grams": 0.04565323144197464, "rps_doc_frac_chars_dupe_6grams": 0.04565323144197464, "rps_doc_frac_chars_dupe_7grams": 0.04565323144197464, "rps_doc_frac_chars_dupe_8grams": 0.04565323144197464, "rps_doc_frac_chars_dupe_9grams": 0.04565323144197464, "rps_doc_frac_chars_top_2gram": 0.026712000370025635, "rps_doc_frac_chars_top_3gram": 0.020398249849677086, "rps_doc_frac_chars_top_4gram": 0.021369600668549538, "rps_doc_books_importance": -251.5267333984375, "rps_doc_books_importance_length_correction": -251.5267333984375, "rps_doc_openwebtext_importance": -130.32044982910156, "rps_doc_openwebtext_importance_length_correction": -130.32044982910156, "rps_doc_wikipedia_importance": -74.56689453125, "rps_doc_wikipedia_importance_length_correction": -74.56689453125 }, "fasttext": { "dclm": 0.10215271264314651, "english": 0.8962361812591553, "fineweb_edu_approx": 1.6919509172439575, "eai_general_math": 0.2768954634666443, "eai_open_web_math": 0.20574086904525757, "eai_web_code": 0.33949100971221924 } }
{ "free_decimal_correspondence": { "primary": { "code": "004.02854", "labels": { "level_1": "General works, books and libraries, information sciences", "level_2": "", "level_3": "Computers and Computer science" } }, "secondary": { "code": "005.456", "labels": { "level_1": "General works, books and libraries, information sciences", "level_2": "", "level_3": "Computer programming" } } }, "bloom_cognitive_process": { "primary": { "code": "3", "label": "Apply" }, "secondary": { "code": "2", "label": "Understand" } }, "bloom_knowledge_domain": { "primary": { "code": "3", "label": "Procedural" }, "secondary": { "code": "2", "label": "Conceptual" } }, "document_type_v1": { "primary": { "code": "5", "label": "Social/Forum" }, "secondary": { "code": "3", "label": "Reference/Encyclopedic/Educational" } }, "extraction_artifacts": { "primary": { "code": "3", "label": "Irrelevant Content" }, "secondary": { "code": "0", "label": "No Artifacts" } }, "missing_content": { "primary": { "code": "0", "label": "No missing content" }, "secondary": { "code": "-1", "label": "Abstain" } }, "document_type_v2": { "primary": { "code": "18", "label": "Q&A Forum" }, "secondary": { "code": "21", "label": "Customer Support" } }, "reasoning_depth": { "primary": { "code": "2", "label": "Basic Reasoning" }, "secondary": { "code": "3", "label": "Intermediate Reasoning" } }, "technical_correctness": { "primary": { "code": "3", "label": "Mostly Correct" }, "secondary": { "code": "4", "label": "Highly Correct" } }, "education_level": { "primary": { "code": "2", "label": "High School Level" }, "secondary": { "code": "3", "label": "Undergraduate Level" } } }
672f1e42c33a7f9846924a2431ea77df
7,554,906,489,138,663,000
Managed WordPress builds upon optimized WordPress hosting in a few key areas. Your website will be assigned a customer support squad that isn't just super-knowledgeable in all things WordPress, but one that also ensures that you don't have to ever worry about going into your site's back end to do anything other than create content. Managed WordPress hosts typically offer site-staging for posts and pages so that you can test them before they go live, automatic malware detection and removal, and enhanced security, too. Dedicated server is a physical server that you can lease from the hosting provider. This allows you to have full control over the server including the choice of operating system, hardware etc. You DO NOT need a dedicated server if you are just starting out. Once your site is receiving a significant amount of traffic, only then you should consider upgrading to a dedicated server. WPBeginner runs on a dedicated server by HostGator. If you do not employ a system administrator or have no experience with servers, then we recommend that you get a managed dedicated server. WordPress hosting providers who offer managed dedicated servers employ full-time system administrators that maintain your servers. Along with doing software updates, they also do server monitoring, offer phone support etc. Most top websites use clusters of dedicated servers. Domain name generators allow you to input an initial keyword or domain and then generate a list of similar domains you can purchase. While some specialize in offering an extensive list of domain extensions, others help you find domains specific to your business niche, and still others help you find currently owned domains and then negotiate their acquisition. A great domain name is a concise, easy to type, and memorable URL that reflects your brand or your website’s subject matter. Avoid using hyphens, strings of numbers, or unnecessary words to make it easy for your visitors to remember and find your website. Remember: A great domain name is one that your visitors can type correctly on their first try. Dedicated server is a physical server that you can lease from the hosting provider. This allows you to have full control over the server including the choice of operating system, hardware etc. You DO NOT need a dedicated server if you are just starting out. Once your site is receiving a significant amount of traffic, only then you should consider upgrading to a dedicated server. WPBeginner runs on a dedicated server by HostGator. If you do not employ a system administrator or have no experience with servers, then we recommend that you get a managed dedicated server. WordPress hosting providers who offer managed dedicated servers employ full-time system administrators that maintain your servers. Along with doing software updates, they also do server monitoring, offer phone support etc. Most top websites use clusters of dedicated servers. The WHOIS domain database is a listing of all registered domains, and is regularly used for various legal purposes. Network administrators use WHOIS data to identify and fix problems. For instance, WHOIS information can be used to determine the availability of domain names, identify trademark infringement, and keep domain name registrants accountable. Often overlooked, web hosting is one of the key components of every successful website. Choosing the best WordPress hosting for your needs can improve your SEO and increase sales. There are various different types of WordPress hosting options available such as Free, Shared, VPS, Dedicated, and managed WordPress hosting. In this guide, we will help you choose the best WordPress hosting for your website. The Internet Corporation for Assigned Names and Numbers (ICANN) is the organization that governs the rules and regulations for domain name registrations. ICANN requires, for various reasons including to determine ownership of a domain should a dispute transpire, that a publicly accessible database be maintained that contains the contact information of all domain registrants. In layman's terms this means your domain name will be searchable by anyone and those search results will include your full name, physical address and other contact information. In order to protect your privacy in this regard, Domain.com offers WHOIS Domain Privacy which then masks your information using our own and implements a procedure for you to control who is able to then gain access to your contact information via a WHOIS search. Whenever you buy a domain name, no matter what domain name registration service you use, you are subject to the same ICANN rules, for this reason it is important to use a reputable service who cares about your privacy. Domain.com always recommends enabling WHOIS Domain Privacy. Domain name generators allow you to input an initial keyword or domain and then generate a list of similar domains you can purchase. While some specialize in offering an extensive list of domain extensions, others help you find domains specific to your business niche, and still others help you find currently owned domains and then negotiate their acquisition. According to ICANN’s (Internet Corporation for Assigned Names and Numbers) rules and regulations, anyone can look up the WHOIS data on a domain name. Unfortunately, this became a good and free opportunity for spammers, hackers and all types of scammers to use that data for illegal activities. At the very least you can be exposed to unsolicited commercial email (SPAM), other junk email and telemarketing calls. When you build a website, you want visitors to come and see what you've done. To get them there, you need a unique domain name that connects to your sites servers. Domain name registration is required to ensure that no one else in the world can claim ownership of your web site's address and to make finding your website simple. Find your one of a kind domain name. By definition, a domain name is simply a human readable form of an IP address. In function it is the destination that you type into a web browser in order to visit a website, such a www.google.com. Metaphorically, it is very similar to how you would scroll to a contact in your cell phone rather than manually dialing the person by entering their full phone number; the phone number would be an IP address and the saved contact would be a domain name. Always buy a domain with a reputable domain registrar. An SSL connection encrypts the data that travels between your site and users' web browsers, thus safeguarding the transmission of purchasing information. All the WordPress hosting services in this roundup offer SSL certificates, but the prices vary from company to company. Some companies include a free SSL certificate when you sign up for a hosting plan, while others charge close to $100. Is there another way? Yes, there is. Businesses should be making use of a domain name registration company to centrally administer all their domains. There are lots of very professional domain name registration companies that have been in business for years and that provide fully functional control panels that give their customers full control over their domain names. Then, all they have to do is change the name server details from their old host to their new host for each domain. With only 12 predesigned themes compared to Squarespace’s 60, DreamHost’s Remixer website builder―included in both plans―isn’t as robust as the Squarespace builder. Still, both builders offer similar, fairly limited customization capabilities; you can’t drag-and-drop elements anywhere on the page. Instead, you must add them in a predefined stacked structure. However, Remixer allows you to export designs to WordPress for additional customization. It is quick and easy. If you have found a suitable domain name, enter your preferred name into the IONOS domain checker. The tool will immediately check whether your desired domain name is free or not. That's all. But, don't forget – hurry up to get your desired domain before someone else does, because you aren't the only one searching for the perfect domain name. Do you want to find a great free domain name for your site? Our domain checker help you to find one. You can check the domain names which will attracts your website to people surfing the net. Your idea for a domain name should be clear and catchy, when you use the IONOS domain checker. This will make your site easily identifiable and people will come back again and again. With the domain check you will find a good free domain name for your successful website. You also need to consider how long you'll need WordPress web hosting. If it's a short-term project—say, less than a month or two—you'll typically receive a refund should you cancel your hosting within 60 days. Some companies offer 30-day money-back guarantees, while others offer 90-day money-back guarantees. Once again, it's beneficial to do your homework. Disclaimer: Reviews on FitSmallBusiness.com are the product of independent research by our writers, researchers, and editorial team. User reviews and comments are contributions from independent users not affiliated with FitSmallBusiness.com's editorial team. Banks, issuers, credit card companies, and other product & service providers are not responsible for any content posted on FitSmallBusiness.com. As such, they do not endorse or guarantee any posted comments or reviews. Like Ask a Question A great domain name is a concise, easy to type, and memorable URL that reflects your brand or your website’s subject matter. Avoid using hyphens, strings of numbers, or unnecessary words to make it easy for your visitors to remember and find your website. Remember: A great domain name is one that your visitors can type correctly on their first try. In a metaphysics driven society where freewill is law, your choices are your own as you attend the most prestigious of magical schools. Learn to master psychic Ability, fight monsters, argue with an Angel, cut classes, befriend unusual races, chase ghosts, and navigate through an exciting city full of in depth characters in an exquisitely detailed world. In a metaphysics driven society where freewill is law, your choices are your own as you attend the most prestigious of magical schools. Learn to master psychic Ability, fight monsters, argue with an Angel, cut classes, befriend unusual races, chase ghosts, and navigate through an exciting city full of in depth characters in an exquisitely detailed world. Contact the owner. Before so much as hinting at a price, simply email to ask whether or not the domain is for sale. If you are known or can clearly be linked to a thriving business, create a generic alternate email address through which to contact them, as your success might be leveraged against you. Be aware, however, that an informal-sounding email address is more likely to be regarded as spam or junk mail. ×
{ "url": "https://puoomyweb.us/raspberry-pi-web-hosting-what-is-web-hosting-and-how-does-it-work.html", "source_domain": "puoomyweb.us", "snapshot_id": "crawl=CC-MAIN-2020-16", "warc_metadata": { "Content-Length": "18616", "Content-Type": "application/http; msgtype=response", "WARC-Block-Digest": "sha1:GHOAG22NBGN7KPNEYW6PDYEVWSX3DG62", "WARC-Concurrent-To": "<urn:uuid:d480fd69-2e51-4709-9523-9ea971ccd2dc>", "WARC-Date": "2020-04-05T09:18:14Z", "WARC-IP-Address": "104.27.132.104", "WARC-Identified-Payload-Type": "text/html", "WARC-Payload-Digest": "sha1:6J66NFH7T7LGVPHZVHCXDQ6BPA7E4YX5", "WARC-Record-ID": "<urn:uuid:bf574c1a-d7d9-4e3f-8416-0aa84249db5b>", "WARC-Target-URI": "https://puoomyweb.us/raspberry-pi-web-hosting-what-is-web-hosting-and-how-does-it-work.html", "WARC-Truncated": null, "WARC-Type": "response", "WARC-Warcinfo-ID": "<urn:uuid:455d7f5f-00c8-42db-a79d-7c5b365fd10d>" }, "warc_info": "isPartOf: CC-MAIN-2020-16\r\npublisher: Common Crawl\r\ndescription: Wide crawl of the web for March/April 2020\r\noperator: Common Crawl Admin ([email protected])\r\nhostname: ip-10-67-67-81.ec2.internal\r\nsoftware: Apache Nutch 1.16 (modified, https://github.com/commoncrawl/nutch/)\r\nrobots: checked via crawler-commons 1.1-SNAPSHOT (https://github.com/crawler-commons/crawler-commons)\r\nformat: WARC File Format 1.1\r\nconformsTo: http://iipc.github.io/warc-specifications/specifications/warc-format/warc-1.1/" }
{ "line_start_idx": [ 0, 1, 524, 1373, 1374, 1736, 2087, 2937, 2938, 3292, 3698, 3699, 4795, 4796, 4797, 5160, 5161, 5574, 5575, 5941, 5942, 5943, 6450, 6451, 6452, 6844, 6845, 6846, 7332, 7782, 8149, 8613, 8972, 9470, 9821, 9822, 10179, 10536, 10949 ], "line_end_idx": [ 1, 524, 1373, 1374, 1736, 2087, 2937, 2938, 3292, 3698, 3699, 4795, 4796, 4797, 5160, 5161, 5574, 5575, 5941, 5942, 5943, 6450, 6451, 6452, 6844, 6845, 6846, 7332, 7782, 8149, 8613, 8972, 9470, 9821, 9822, 10179, 10536, 10949, 10950 ] }
{ "red_pajama_v2": { "ccnet_original_length": 10950, "ccnet_original_nlines": 38, "rps_doc_curly_bracket": 0, "rps_doc_ldnoobw_words": 0, "rps_doc_lorem_ipsum": 0, "rps_doc_stop_word_fraction": 0.4265632629394531, "rps_doc_ut1_blacklist": 0, "rps_doc_frac_all_caps_words": 0.015026659704744816, "rps_doc_frac_lines_end_with_ellipsis": 0, "rps_doc_frac_no_alph_words": 0.12263693660497665, "rps_doc_frac_unique_words": 0.3269447684288025, "rps_doc_mean_word_length": 5.030439853668213, "rps_doc_num_sentences": 101, "rps_doc_symbol_to_word_ratio": 0, "rps_doc_unigram_entropy": 5.5959954261779785, "rps_doc_word_count": 1774, "rps_doc_frac_chars_dupe_10grams": 0.35230839252471924, "rps_doc_frac_chars_dupe_5grams": 0.3738234043121338, "rps_doc_frac_chars_dupe_6grams": 0.36911699175834656, "rps_doc_frac_chars_dupe_7grams": 0.3623935580253601, "rps_doc_frac_chars_dupe_8grams": 0.35230839252471924, "rps_doc_frac_chars_dupe_9grams": 0.35230839252471924, "rps_doc_frac_chars_top_2gram": 0.02801433950662613, "rps_doc_frac_chars_top_3gram": 0.010757509618997574, "rps_doc_frac_chars_top_4gram": 0.0076199001632630825, "rps_doc_books_importance": -921.5164184570312, "rps_doc_books_importance_length_correction": -921.5164184570312, "rps_doc_openwebtext_importance": -444.4598388671875, "rps_doc_openwebtext_importance_length_correction": -444.4598388671875, "rps_doc_wikipedia_importance": -342.8088684082031, "rps_doc_wikipedia_importance_length_correction": -342.8088684082031 }, "fasttext": { "dclm": 0.02610635943710804, "english": 0.9273988008499146, "fineweb_edu_approx": 1.4998838901519775, "eai_general_math": 0.005789520218968391, "eai_open_web_math": 0.0416719913482666, "eai_web_code": 0.007132890168577433 } }
{ "free_decimal_correspondence": { "primary": { "code": "004.67", "labels": { "level_1": "General works, books and libraries, information sciences", "level_2": "", "level_3": "Computers and Computer science" } }, "secondary": { "code": "005.1", "labels": { "level_1": "General works, books and libraries, information sciences", "level_2": "", "level_3": "Computer programming" } } }, "bloom_cognitive_process": { "primary": { "code": "2", "label": "Understand" }, "secondary": { "code": "3", "label": "Apply" } }, "bloom_knowledge_domain": { "primary": { "code": "2", "label": "Conceptual" }, "secondary": { "code": "3", "label": "Procedural" } }, "document_type_v1": { "primary": { "code": "3", "label": "Reference/Encyclopedic/Educational" }, "secondary": { "code": "6", "label": "Promotional/Advertisement" } }, "extraction_artifacts": { "primary": { "code": "3", "label": "Irrelevant Content" }, "secondary": { "code": "-1", "label": "Abstain" } }, "missing_content": { "primary": { "code": "0", "label": "No missing content" }, "secondary": { "code": "-1", "label": "Abstain" } }, "document_type_v2": { "primary": { "code": "10", "label": "Knowledge Article" }, "secondary": { "code": "17", "label": "Product Page" } }, "reasoning_depth": { "primary": { "code": "2", "label": "Basic Reasoning" }, "secondary": { "code": "3", "label": "Intermediate Reasoning" } }, "technical_correctness": { "primary": { "code": "3", "label": "Mostly Correct" }, "secondary": { "code": "4", "label": "Highly Correct" } }, "education_level": { "primary": { "code": "1", "label": "General Audience" }, "secondary": { "code": "2", "label": "High School Level" } } }
672f1e42c33a7f9846924a2431ea77df
-9,131,117,573,859,838,000
In this guide we are going to see what is project Lombok, setting up with IDE eclipse / STS,  a simple java Lombok maven example and project Lombok main features. 1. What is Project Lombok? Project Lombok is a java library that plugs into your editor or IDE like eclipse, STS, IntelliJ etc, also plugs into build tools like maven, gradle, ant etc. Lombok library simply tells to the IDE , to generate all the boilerplate code for you silently. And this is done during compile time, so you usually don’t have to care about lombok being on your runtime path at all. The main aim of project Lombok is  reduce boilerplate headaches out of the developer’s task list. It can be useful for making your code more concise, and speeding up development time. Below image illustrates how java project Lombok works: 1. Plugin Lombok java library to your IDE. 2. Use Lombok annotations to write your POJO classes. 3.  Lombok will generates the boiler plate code (gettters, setters, toString(),…etc) automatically, which happens at compile time, so developer no need to write setters, getters etc code manually. java lombok workflow 2. Lombok in Eclipse 1. If we’re using Eclipse IDE or Spring tool Suite (STS), we need to get the Lombok jar first. The latest version is located on Maven Central repository. For our example, we’re using lombok-1.18.10.jar. 2. Double click on downloaded lombok.jar file. 3. Installer UI open. 4. Specify location your IDE Eclipse.exe / STS.exe path, click on Install/Update button. lombok eclipse installer ui If the installation is successful, we can exit the installer. After installing the plugin, we need to restart the IDE and ensure that Lombok is correctly configured. We can check this in eclipse go to help option click About eclipse: lombok eclipse setup success 3. Adding Lombok to Maven The last remaining part is to ensure that Lombok binaries are on the compiler classpath. Using Maven, we can add the dependency to the pom.xml. Copy to Clipboard 4. A simple Java lombok example: You can annotate any field with @Getter and/or @Setter, to let lombok generate the default getter/setter automatically. The following example illustrates how to add the @Getter and/or @Setter Lombok annotations to the top of the class and how it reduce the boiler plate from sources file. Lets have a look into plain Java POJO class that without using Lombok and POJO with the usage of Lombok, in below example technically both right hand side and left hand side code is equal, when we use Lombok in POJO class developer no need to write boiler plate code. A no Lombok java Pojo Copy to Clipboard A Lombok java Pojo Copy to Clipboard 5. Demo Lombok Pojo class Copy to Clipboard 6. Output results Copy to Clipboard References
{ "url": "https://javabydeveloper.com/a-basic-java-lombok-maven-example-with-eclipse/", "source_domain": "javabydeveloper.com", "snapshot_id": "crawl=CC-MAIN-2020-05", "warc_metadata": { "Content-Length": "103304", "Content-Type": "application/http; msgtype=response", "WARC-Block-Digest": "sha1:TBWYXVEKGIF2DDE6IKKFMEAL5LFGXKU7", "WARC-Concurrent-To": "<urn:uuid:fc159684-6bb4-495a-bc0c-770a87cd9ef4>", "WARC-Date": "2020-01-23T09:33:22Z", "WARC-IP-Address": "104.28.1.200", "WARC-Identified-Payload-Type": "text/html", "WARC-Payload-Digest": "sha1:OU3OMZZ4MLLMUL5XXVQCDIPYM5YMTNBE", "WARC-Record-ID": "<urn:uuid:e7cdd574-0fe0-4523-aac2-4c7c417cef74>", "WARC-Target-URI": "https://javabydeveloper.com/a-basic-java-lombok-maven-example-with-eclipse/", "WARC-Truncated": null, "WARC-Type": "response", "WARC-Warcinfo-ID": "<urn:uuid:6d9a4e5d-e1f3-4b05-807e-353e52237a8a>" }, "warc_info": "isPartOf: CC-MAIN-2020-05\r\npublisher: Common Crawl\r\ndescription: Wide crawl of the web for January 2020\r\noperator: Common Crawl Admin ([email protected])\r\nhostname: ip-10-67-67-204.ec2.internal\r\nsoftware: Apache Nutch 1.16 (modified, https://github.com/commoncrawl/nutch/)\r\nrobots: checked via crawler-commons 1.1-SNAPSHOT (https://github.com/crawler-commons/crawler-commons)\r\nformat: WARC File Format 1.1\r\nconformsTo: http://iipc.github.io/warc-specifications/specifications/warc-format/warc-1.1/" }
{ "line_start_idx": [ 0, 163, 164, 193, 194, 752, 753, 808, 809, 854, 910, 1109, 1110, 1131, 1132, 1153, 1154, 1359, 1408, 1432, 1523, 1524, 1552, 1553, 1615, 1616, 1788, 1789, 1818, 1819, 1845, 1846, 1990, 1991, 2009, 2010, 2043, 2044, 2333, 2334, 2602, 2603, 2625, 2626, 2644, 2645, 2664, 2665, 2683, 2684, 2710, 2711, 2729, 2730, 2748, 2749, 2767, 2768 ], "line_end_idx": [ 163, 164, 193, 194, 752, 753, 808, 809, 854, 910, 1109, 1110, 1131, 1132, 1153, 1154, 1359, 1408, 1432, 1523, 1524, 1552, 1553, 1615, 1616, 1788, 1789, 1818, 1819, 1845, 1846, 1990, 1991, 2009, 2010, 2043, 2044, 2333, 2334, 2602, 2603, 2625, 2626, 2644, 2645, 2664, 2665, 2683, 2684, 2710, 2711, 2729, 2730, 2748, 2749, 2767, 2768, 2778 ] }
{ "red_pajama_v2": { "ccnet_original_length": 2778, "ccnet_original_nlines": 57, "rps_doc_curly_bracket": 0, "rps_doc_ldnoobw_words": 0, "rps_doc_lorem_ipsum": 0, "rps_doc_stop_word_fraction": 0.3309859335422516, "rps_doc_ut1_blacklist": 0, "rps_doc_frac_all_caps_words": 0.03345070034265518, "rps_doc_frac_lines_end_with_ellipsis": 0, "rps_doc_frac_no_alph_words": 0.18485915660858154, "rps_doc_frac_unique_words": 0.44155845046043396, "rps_doc_mean_word_length": 4.7207794189453125, "rps_doc_num_sentences": 44, "rps_doc_symbol_to_word_ratio": 0.0017605599714443088, "rps_doc_unigram_entropy": 4.855642318725586, "rps_doc_word_count": 462, "rps_doc_frac_chars_dupe_10grams": 0, "rps_doc_frac_chars_dupe_5grams": 0.04676754027605057, "rps_doc_frac_chars_dupe_6grams": 0.02659331075847149, "rps_doc_frac_chars_dupe_7grams": 0, "rps_doc_frac_chars_dupe_8grams": 0, "rps_doc_frac_chars_dupe_9grams": 0, "rps_doc_frac_chars_top_2gram": 0.03576340898871422, "rps_doc_frac_chars_top_3gram": 0.03438790142536163, "rps_doc_frac_chars_top_4gram": 0.017423199489712715, "rps_doc_books_importance": -252.11453247070312, "rps_doc_books_importance_length_correction": -252.11453247070312, "rps_doc_openwebtext_importance": -153.8848876953125, "rps_doc_openwebtext_importance_length_correction": -153.8848876953125, "rps_doc_wikipedia_importance": -95.31021118164062, "rps_doc_wikipedia_importance_length_correction": -95.31021118164062 }, "fasttext": { "dclm": 0.4123401641845703, "english": 0.7779456377029419, "fineweb_edu_approx": 2.756441354751587, "eai_general_math": 0.47113728523254395, "eai_open_web_math": 0.22130286693572998, "eai_web_code": 0.21186649799346924 } }
{ "free_decimal_correspondence": { "primary": { "code": "005.13324", "labels": { "level_1": "General works, books and libraries, information sciences", "level_2": "", "level_3": "Computer programming" } }, "secondary": { "code": "005.133", "labels": { "level_1": "General works, books and libraries, information sciences", "level_2": "", "level_3": "Computer programming" } } }, "bloom_cognitive_process": { "primary": { "code": "3", "label": "Apply" }, "secondary": { "code": "2", "label": "Understand" } }, "bloom_knowledge_domain": { "primary": { "code": "3", "label": "Procedural" }, "secondary": { "code": "2", "label": "Conceptual" } }, "document_type_v1": { "primary": { "code": "3", "label": "Reference/Encyclopedic/Educational" }, "secondary": { "code": "4", "label": "Code/Software" } }, "extraction_artifacts": { "primary": { "code": "0", "label": "No Artifacts" }, "secondary": { "code": "3", "label": "Irrelevant Content" } }, "missing_content": { "primary": { "code": "4", "label": "Missing Images or Figures" }, "secondary": { "code": "0", "label": "No missing content" } }, "document_type_v2": { "primary": { "code": "23", "label": "Tutorial" }, "secondary": { "code": "8", "label": "Documentation" } }, "reasoning_depth": { "primary": { "code": "2", "label": "Basic Reasoning" }, "secondary": { "code": "3", "label": "Intermediate Reasoning" } }, "technical_correctness": { "primary": { "code": "4", "label": "Highly Correct" }, "secondary": { "code": "3", "label": "Mostly Correct" } }, "education_level": { "primary": { "code": "3", "label": "Undergraduate Level" }, "secondary": { "code": "2", "label": "High School Level" } } }
672f1e42c33a7f9846924a2431ea77df
3,665,861,754,920,843,000
Beefy Boxes and Bandwidth Generously Provided by pair Networks Clear questions and runnable code get the best and fastest answer   PerlMonks   Re^2: parsing a bibliography by ww (Bishop) on Dec 02, 2004 at 16:48 UTC ( #411853=note: print w/replies, xml ) Need Help?? in reply to Re: parsing a bibliography in thread parsing a bibliography wonderful! Wish I could ++ your solution repeatedly! This writeup led to a "Eureka!" moment; the kind of haze-clearing that makes PM so valuable to beginners like me. request: please add to our understanding by commenting lines of regex, esp that part of line8 reading (?=[A-Z][a-z]) (grouped but non-capture??) and in line13, ( [^\[]+ ) which, as I read Owl (pocket ref), means capture one-or-more of a class including not-an-open_BRKT and close_BRKT ...which doesn't make sense to me, and -- more importantly, doesn't seem to WORK that way. Replies are listed 'Best First'. Re^3: parsing a bibliography by BrowserUk (Pope) on Dec 02, 2004 at 17:36 UTC The regex commented. my( $authors, $title, $thing, $pub, $date, $comment, $no ) = m/ ^ ## Author(s): Capture the minimum needed to satisfy that: ## a) It ends with a '. ' ## b) And the next word is not an initial ## IE: Lookahead and check the next word starts with ## 1 uppercase *and* one lowercase character. -( .*? \. ) \s(?=[A-Z][a-z]) ## Title: Greedily capture something that ends with '. ' ( .+ ) \.\s+ ## Location: Non-greedily capture ## Ends with a ': '. ## Doesn't contain a ':' ( [^:]+? ) : \s+ ## Publisher: ## Single word followed by a ', ' (\S+), \s+ ## Year: Capture Four digits ## Discard anything else upto '. ' ( \d{4} ) [^.]* \. \s+ ## Comment: Greedy capture non-'[' characters ## Ie. Stop capturing when you see a '[' ( [^\[]+ ) ## No: Capture 1 (or more) digits between '[' & ']' ## Discard any trailing space to the EOS. \[ ( \d+ ) \] \s* $ /x; Examine what is said, not who speaks. "But you should never overestimate the ingenuity of the sceptics to come up with a counter-argument." -Myles Allen "Think for yourself!" - Abigail        "Time is a poor substitute for thought"--theorbtwo         "Efficiency is intelligent laziness." -David Dunham "Memory, processor, disk in that order on the hardware side. Algorithm, algorithm, algorithm on the code side." - tachyon Log In? Username: Password: What's my password? Create A New User Node Status? node history Node Type: note [id://411853] help Chatterbox? and the web crawler heard nothing... How do I use this? | Other CB clients Other Users? Others surveying the Monastery: (6) As of 2016-09-27 03:49 GMT Sections? Information? Find Nodes? Leftovers? Voting Booth? Extraterrestrials haven't visited the Earth yet because: Results (494 votes). Check out past polls.
{ "url": "http://www.perlmonks.org/index.pl?node_id=411853", "source_domain": "www.perlmonks.org", "snapshot_id": "crawl=CC-MAIN-2016-40", "warc_metadata": { "Content-Length": "21601", "Content-Type": "application/http; msgtype=response", "WARC-Block-Digest": "sha1:UBAAD62N6UG7QHJ4RAURE3AKKXFOTUAH", "WARC-Concurrent-To": "<urn:uuid:a91b4fbf-3b74-4543-8b1d-d1ae7203629b>", "WARC-Date": "2016-09-27T03:50:01Z", "WARC-IP-Address": "216.92.34.251", "WARC-Identified-Payload-Type": null, "WARC-Payload-Digest": "sha1:GD6NZC4POCC3SXGACRFV6FXYFQBSVGZA", "WARC-Record-ID": "<urn:uuid:07040930-d0c0-449d-b09c-e7d2c1174db7>", "WARC-Target-URI": "http://www.perlmonks.org/index.pl?node_id=411853", "WARC-Truncated": "length", "WARC-Type": "response", "WARC-Warcinfo-ID": "<urn:uuid:138a73b3-a07c-4677-b4ab-2f47b171fca9>" }, "warc_info": "robots: classic\r\nhostname: ip-10-143-35-109.ec2.internal\r\nsoftware: Nutch 1.6 (CC)/CC WarcExport 1.0\r\nisPartOf: CC-MAIN-2016-40\r\noperator: CommonCrawl Admin\r\ndescription: Wide crawl of the web for September 2016\r\npublisher: CommonCrawl\r\nformat: WARC File Format 1.0\r\nconformsTo: http://bibnum.bnf.fr/WARC/WARC_ISO_28500_version1_latestdraft.pdf" }
{ "line_start_idx": [ 0, 63, 97, 129, 131, 143, 144, 173, 174, 189, 269, 270, 271, 310, 343, 344, 511, 512, 614, 615, 630, 631, 659, 660, 675, 676, 687, 688, 893, 894, 927, 956, 1005, 1006, 1031, 1032, 1885, 1886, 1928, 2047, 2201, 2327, 2328, 2336, 2346, 2356, 2357, 2377, 2395, 2408, 2421, 2451, 2456, 2468, 2505, 2506, 2544, 2557, 2593, 2620, 2630, 2643, 2655, 2666, 2684, 2745, 2746, 2747, 2748, 2749, 2750, 2751, 2752 ], "line_end_idx": [ 63, 97, 129, 131, 143, 144, 173, 174, 189, 269, 270, 271, 310, 343, 344, 511, 512, 614, 615, 630, 631, 659, 660, 675, 676, 687, 688, 893, 894, 927, 956, 1005, 1006, 1031, 1032, 1885, 1886, 1928, 2047, 2201, 2327, 2328, 2336, 2346, 2356, 2357, 2377, 2395, 2408, 2421, 2451, 2456, 2468, 2505, 2506, 2544, 2557, 2593, 2620, 2630, 2643, 2655, 2666, 2684, 2745, 2746, 2747, 2748, 2749, 2750, 2751, 2752, 2798 ] }
{ "red_pajama_v2": { "ccnet_original_length": 2798, "ccnet_original_nlines": 72, "rps_doc_curly_bracket": 0.000714799971319735, "rps_doc_ldnoobw_words": 0, "rps_doc_lorem_ipsum": 0, "rps_doc_stop_word_fraction": 0.2530120611190796, "rps_doc_ut1_blacklist": 0, "rps_doc_frac_all_caps_words": 0.02560240961611271, "rps_doc_frac_lines_end_with_ellipsis": 0.013698630034923553, "rps_doc_frac_no_alph_words": 0.3840361535549164, "rps_doc_frac_unique_words": 0.6658291220664978, "rps_doc_mean_word_length": 4.82412052154541, "rps_doc_num_sentences": 41, "rps_doc_symbol_to_word_ratio": 0.05572288855910301, "rps_doc_unigram_entropy": 5.291225433349609, "rps_doc_word_count": 398, "rps_doc_frac_chars_dupe_10grams": 0, "rps_doc_frac_chars_dupe_5grams": 0.013541669584810734, "rps_doc_frac_chars_dupe_6grams": 0, "rps_doc_frac_chars_dupe_7grams": 0, "rps_doc_frac_chars_dupe_8grams": 0, "rps_doc_frac_chars_dupe_9grams": 0, "rps_doc_frac_chars_top_2gram": 0.01666666939854622, "rps_doc_frac_chars_top_3gram": 0.0416666716337204, "rps_doc_frac_chars_top_4gram": 0.022916670888662338, "rps_doc_books_importance": -313.8157653808594, "rps_doc_books_importance_length_correction": -313.8157653808594, "rps_doc_openwebtext_importance": -186.24819946289062, "rps_doc_openwebtext_importance_length_correction": -186.24819946289062, "rps_doc_wikipedia_importance": -139.6714324951172, "rps_doc_wikipedia_importance_length_correction": -139.6714324951172 }, "fasttext": { "dclm": 0.14970827102661133, "english": 0.8639147877693176, "fineweb_edu_approx": 1.7871497869491577, "eai_general_math": 0.7543619871139526, "eai_open_web_math": 0.37698715925216675, "eai_web_code": 0.8233226537704468 } }
{ "free_decimal_correspondence": { "primary": { "code": "005.133", "labels": { "level_1": "General works, books and libraries, information sciences", "level_2": "", "level_3": "Computer programming" } }, "secondary": { "code": "025.0285", "labels": { "level_1": "General works, books and libraries, information sciences", "level_2": "Library science", "level_3": "Library administration" } } }, "bloom_cognitive_process": { "primary": { "code": "3", "label": "Apply" }, "secondary": { "code": "2", "label": "Understand" } }, "bloom_knowledge_domain": { "primary": { "code": "3", "label": "Procedural" }, "secondary": { "code": "2", "label": "Conceptual" } }, "document_type_v1": { "primary": { "code": "5", "label": "Social/Forum" }, "secondary": { "code": "3", "label": "Reference/Encyclopedic/Educational" } }, "extraction_artifacts": { "primary": { "code": "3", "label": "Irrelevant Content" }, "secondary": { "code": "0", "label": "No Artifacts" } }, "missing_content": { "primary": { "code": "0", "label": "No missing content" }, "secondary": { "code": "-1", "label": "Abstain" } }, "document_type_v2": { "primary": { "code": "18", "label": "Q&A Forum" }, "secondary": { "code": "8", "label": "Documentation" } }, "reasoning_depth": { "primary": { "code": "3", "label": "Intermediate Reasoning" }, "secondary": { "code": "2", "label": "Basic Reasoning" } }, "technical_correctness": { "primary": { "code": "4", "label": "Highly Correct" }, "secondary": { "code": "3", "label": "Mostly Correct" } }, "education_level": { "primary": { "code": "3", "label": "Undergraduate Level" }, "secondary": { "code": "2", "label": "High School Level" } } }
672f1e42c33a7f9846924a2431ea77df
-8,374,698,531,838,814,000
What are the reasons for StackOverflow failure? I want to hear different views on what are the reasons for SO(StackOverflow) failures? SO,  was created to mimic EE and be a competition for it but in my view it has failed "miserably" so far. Here are the reasons I have  found so far: 1: Extremely hostile environment for a new user. Most of the new user's questions will stuck  with downvotes and new user can even get banned by asking a "Valid" question. 2: SO's dysfunctional voting system encourages voting wars and competition groups who vote against each other no matter how is an answer valid. I want to know  your point of view about SO. Also are you aware of any other websites(I.T) which can come close to the quality of  EE. LVL 15 unknown_routineAsked: Who is Participating?   TommySzalapskiConnect With a Mentor Commented: Perhaps they don't want lots of new users. In all honesty, new users can clutter a forum by posting questions that have been answered many times already. We don't mind that here, because it's easy points for us. Experts Exchange has done a very good job of building a sense of community. Experts from here have developed friendships, met in real life, gotten jobs for each other, etc. We tend to be very forgiving of new users who do silly things and coach them instead of attacking them (unless they wander into the lounge, then they get attacked - which you may have noticed). I don't think SO has that same sense of community. I think the fact that you have to pay or answer questions to be a full member here has really helped that community feeling. People automatically have somewhat of a vested interest since they have given something (time or money) to contribute before they can even post a new question. SO does not do that so there seems to be a lot more anarchy. They don't seem to have the high quality moderator support either that we have here to keep things from getting out of hand. 0   TommySzalapskiConnect With a Mentor Commented: SO has a goal to be completely objective so if a 'valid' question is a subjective question, it will get deleted. You can't ask opinion questions there. (You can here!) Yes, the politics of voting wars can get messy. In general, there is a lot of good content there, but it can take some time to get through the jungle. Also are you aware of any other websites(I.T) which can come close to the quality of  EE. There are none, because this is the only one I am on! Oh wait, this isn't the lounge... 0   Dave BaldwinFixer of ProblemsCommented: I hear you say that there are two things that you don't like.  I don't see how that defines them as failing.  I personally don't go there unless Google takes me there. 0 The 14th Annual Expert Award Winners The results are in! Meet the top members of our 2017 Expert Awards. Congratulations to all who qualified!   unknown_routineAuthor Commented: The number of new users who run away from S.O because of it's hostile behavior is gigantic. I call this a big failure. Because users are true steam and power of a website. 0   Dave BaldwinFixer of ProblemsCommented: You can call it want you want.  They don't seem to be going out of business. 0   unknown_routineAuthor Commented: We tend to be very forgiving of new users who do silly things and coach them instead of attacking them People sholud not underestimate the above. This is a huge priviledge over S.O. They don't seem to have the high quality moderator support either that we have here to keep things from getting out of hand. They indeed do have a "moderator support" . But in S.O moderator support traslates to protecting their friends and bashing others who disagree with them. 0   COBOLdinosaurConnect With a Mentor Commented: S.O. continues to survive for 4 reasons. They have investors with deep pockets who continue to eat the finanacial loss. They have a "special relationship" with Google foster by the cross-polination of staff. They benefited from EE screwing up big time and alienating a lot of people. They benefit from all the SO links that get post to their crap content by Google Monkeys on EE The first will eventually dry up. There is nothing we can do about the second because evil seeks out evil. The third will take some time for redemption to kick in. The final one is now being dealt with by the new corp of Topic Advisors and IMO anyone who post a link to SO should get suspended. Cd& 0   TommySzalapskiCommented: The first will eventually dry up. At which point they will cry to their buddies at Google who will then take ownership of the domain and compound issue 2. 0   Eric AKA NetminderCommented: they will cry to their buddies at Google who will then take ownership of the domain and then, two to three years later, will shut it down like so many of their other acquired ventures... 0   Dave BaldwinConnect With a Mentor Fixer of ProblemsCommented: Interesting.  Are you guys aware that StackOverflow is one of 106 Q&A sites owned and run by StackExchange?  And if Q&A sites disappeared just because they had problems, Experts-Exchange would have been gone years ago. 0   mbizupConnect With a Mentor Commented: << point of view about SO. >> As an Expert, my preference and commitment is to EE for a multitude of reasons -- by a long shot. However as a professional, looking for fast solutions specifically to Web Development questions, my answers almost invariably come from SO.  I have never actually posted a question there, but wind up there from my own searches and also suggestions posted by Experts in questions on EE. The response time to my Web Development questions on EE has generally not been very fast, and I'm actively independently searching and trying solutions while my questions are posted here.  Despite major improvements in EE's search feature, I still find Google to be faster and more intuitive... and Google searches tend to lead me to solutions at SO. Further, when Experts post to my Web Dev questions, it is often in the form of links to SO, or links to questions at other forums whose comments contain links to SO.  So even EE questions directly or indirectly take me to SO. What keeps me posting questions at EE is the friendly, targeted, personalized follow-up I get when it is needed. The initiatives Cd& mentioned in the Topic Advisor program will eventually lead to more original content, and EE's search capabilities are improving.  But while my preference and time commitment are with EE, SO is undeniably a very valuable resource to me at this time. 0   unknown_routineAuthor Commented: The final one is now being dealt with by the new corp of Topic Advisors and IMO anyone who post a link to SO should get suspended. When is this go to be in effect? 0   unknown_routineAuthor Commented: @mbizup A computer Q/A websites is to be for everyone who needs knowlege and not only for power users. The question is how much each I.T Website is befecial to a typical person who ask questions. The amount of hostility to the new users in S.O is unimaginable. Yes S.O has some top contributors but in my opnion it still fails to address the average user. I am a S.O member and sometimes I answer some questions there. My S.O Avatar is Experts_Exchange  Icon :) I get downvote for my answers only because in my S.O profile  I have  mentioned I prefer EE over S.O  :) Well I can't care less about downvotes.  S.O so called repuation is "Worlthess" anyway. I personally know some people who ran away from S.O after a bad experience as a new user. 0   mbizupCommented: << Avatar is Experts_Exchange  Icon >> That is priceless....  :-) << When is this go to be in effect? >> Apart from Cd&'s suspension idea, it is already in effect.  We're making an organized effort to delete blind/unsupported links and get experts who post comments like that to include supporting original content.... to steer responses away from the style used in one of my last posted technical questions (which for the record is the ONLY 'B' grade I have ever awarded on EE): http://www.experts-exchange.com/Programming/Languages/.NET/Visual_Basic.NET/Q_28023626.html 0   COBOLdinosaurCommented: When is this go to be in effect? The Topic Advisors have been mandate to clean upthe links based on the guidelines published in the last EE newsletter, and we are actively removing links to Stack and other links that are harmful to EE quality. Cd& 0   unknown_routineAuthor Commented: Thanks all for your comments. Very interesting pointers. 0 Question has a verified solution. Are you are experiencing a similar issue? Get a personalized answer when you ask a related question. Have a better answer? Share it in a comment. All Courses From novice to tech pro — start learning today.
{ "url": "https://www.experts-exchange.com/questions/28240841/What-are-the-reasons-for-StackOverflow-failure.html", "source_domain": "www.experts-exchange.com", "snapshot_id": "crawl=CC-MAIN-2018-13", "warc_metadata": { "Content-Length": "178713", "Content-Type": "application/http; msgtype=response", "WARC-Block-Digest": "sha1:Q4ZZOON5JN6IBGWVUVXEIWFJDIONJQ5K", "WARC-Concurrent-To": "<urn:uuid:70f600c2-7ca2-4894-a67d-48b65a386874>", "WARC-Date": "2018-03-22T17:24:31Z", "WARC-IP-Address": "104.20.168.10", "WARC-Identified-Payload-Type": "text/html", "WARC-Payload-Digest": "sha1:7ACUOUIPBXFOYBWPWQGB55WBE4JQIMG4", "WARC-Record-ID": "<urn:uuid:14f7aadb-b1a5-4a91-9c41-ff30122f8f82>", "WARC-Target-URI": "https://www.experts-exchange.com/questions/28240841/What-are-the-reasons-for-StackOverflow-failure.html", "WARC-Truncated": "length", "WARC-Type": "response", "WARC-Warcinfo-ID": "<urn:uuid:d6ab1927-cf79-4986-a62e-8221fdf86db4>" }, "warc_info": "robots: classic\r\nhostname: ip-10-181-96-138.ec2.internal\r\nsoftware: Nutch 1.6 (CC)\r\nisPartOf: CC-MAIN-2018-13\r\noperator: Common Crawl Admin\r\ndescription: Wide crawl of the web for March 2018\r\npublisher: Common Crawl\r\nformat: WARC File Format 1.0\r\nconformsTo: http://bibnum.bnf.fr/WARC/WARC_ISO_28500_version1_latestdraft.pdf" }
{ "line_start_idx": [ 0, 48, 49, 136, 137, 138, 244, 245, 288, 289, 461, 462, 606, 607, 652, 653, 743, 750, 772, 794, 796, 843, 1055, 1056, 1423, 1424, 1946, 1948, 1950, 1997, 2165, 2316, 2317, 2407, 2495, 2497, 2499, 2539, 2707, 2709, 2746, 2747, 2853, 2854, 2856, 2889, 2981, 2982, 3062, 3064, 3066, 3106, 3183, 3185, 3187, 3220, 3323, 3324, 3403, 3404, 3405, 3530, 3531, 3685, 3687, 3689, 3735, 3776, 3777, 3856, 3944, 4020, 4115, 4410, 4411, 4415, 4417, 4419, 4444, 4478, 4599, 4601, 4603, 4632, 4716, 4717, 4820, 4822, 4824, 4886, 5105, 5107, 5109, 5148, 5178, 5179, 5277, 5278, 5564, 5565, 5916, 5917, 6143, 6144, 6257, 6258, 6528, 6530, 6532, 6565, 6696, 6697, 6698, 6731, 6733, 6735, 6768, 6776, 6777, 6872, 6873, 6966, 6967, 7127, 7128, 7234, 7235, 7340, 7341, 7342, 7430, 7431, 7521, 7523, 7525, 7542, 7545, 7578, 7581, 7582, 7609, 7610, 7613, 7646, 7649, 7650, 8025, 8026, 8118, 8120, 8122, 8146, 8179, 8180, 8391, 8392, 8396, 8398, 8400, 8433, 8490, 8492, 8526, 8527, 8628, 8629, 8674, 8675, 8687, 8688 ], "line_end_idx": [ 48, 49, 136, 137, 138, 244, 245, 288, 289, 461, 462, 606, 607, 652, 653, 743, 750, 772, 794, 796, 843, 1055, 1056, 1423, 1424, 1946, 1948, 1950, 1997, 2165, 2316, 2317, 2407, 2495, 2497, 2499, 2539, 2707, 2709, 2746, 2747, 2853, 2854, 2856, 2889, 2981, 2982, 3062, 3064, 3066, 3106, 3183, 3185, 3187, 3220, 3323, 3324, 3403, 3404, 3405, 3530, 3531, 3685, 3687, 3689, 3735, 3776, 3777, 3856, 3944, 4020, 4115, 4410, 4411, 4415, 4417, 4419, 4444, 4478, 4599, 4601, 4603, 4632, 4716, 4717, 4820, 4822, 4824, 4886, 5105, 5107, 5109, 5148, 5178, 5179, 5277, 5278, 5564, 5565, 5916, 5917, 6143, 6144, 6257, 6258, 6528, 6530, 6532, 6565, 6696, 6697, 6698, 6731, 6733, 6735, 6768, 6776, 6777, 6872, 6873, 6966, 6967, 7127, 7128, 7234, 7235, 7340, 7341, 7342, 7430, 7431, 7521, 7523, 7525, 7542, 7545, 7578, 7581, 7582, 7609, 7610, 7613, 7646, 7649, 7650, 8025, 8026, 8118, 8120, 8122, 8146, 8179, 8180, 8391, 8392, 8396, 8398, 8400, 8433, 8490, 8492, 8526, 8527, 8628, 8629, 8674, 8675, 8687, 8688, 8735 ] }
{ "red_pajama_v2": { "ccnet_original_length": 8735, "ccnet_original_nlines": 169, "rps_doc_curly_bracket": 0, "rps_doc_ldnoobw_words": 0, "rps_doc_lorem_ipsum": 0, "rps_doc_stop_word_fraction": 0.4384787380695343, "rps_doc_ut1_blacklist": 0, "rps_doc_frac_all_caps_words": 0.057046979665756226, "rps_doc_frac_lines_end_with_ellipsis": 0.0117647098377347, "rps_doc_frac_no_alph_words": 0.1482102870941162, "rps_doc_frac_unique_words": 0.3496644198894501, "rps_doc_mean_word_length": 4.597986698150635, "rps_doc_num_sentences": 115, "rps_doc_symbol_to_word_ratio": 0.002796419896185398, "rps_doc_unigram_entropy": 5.52806282043457, "rps_doc_word_count": 1490, "rps_doc_frac_chars_dupe_10grams": 0.12406948208808899, "rps_doc_frac_chars_dupe_5grams": 0.17953583598136902, "rps_doc_frac_chars_dupe_6grams": 0.14246095716953278, "rps_doc_frac_chars_dupe_7grams": 0.1345788836479187, "rps_doc_frac_chars_dupe_8grams": 0.1345788836479187, "rps_doc_frac_chars_dupe_9grams": 0.12406948208808899, "rps_doc_frac_chars_top_2gram": 0.006422420032322407, "rps_doc_frac_chars_top_3gram": 0.008028030395507812, "rps_doc_frac_chars_top_4gram": 0.011677130125463009, "rps_doc_books_importance": -810.5039672851562, "rps_doc_books_importance_length_correction": -810.5039672851562, "rps_doc_openwebtext_importance": -469.6815185546875, "rps_doc_openwebtext_importance_length_correction": -469.6815185546875, "rps_doc_wikipedia_importance": -256.1414794921875, "rps_doc_wikipedia_importance_length_correction": -256.1414794921875 }, "fasttext": { "dclm": 0.05624539032578468, "english": 0.9599161148071289, "fineweb_edu_approx": 1.313105583190918, "eai_general_math": 0.11263024806976318, "eai_open_web_math": 0.2879854440689087, "eai_web_code": 0.007086339872330427 } }
{ "free_decimal_correspondence": { "primary": { "code": "004.656", "labels": { "level_1": "General works, books and libraries, information sciences", "level_2": "", "level_3": "Computers and Computer science" } }, "secondary": { "code": "302.22", "labels": { "level_1": "Social sciences", "level_2": "", "level_3": "Social psychology" } } }, "bloom_cognitive_process": { "primary": { "code": "5", "label": "Evaluate" }, "secondary": { "code": "4", "label": "Analyze" } }, "bloom_knowledge_domain": { "primary": { "code": "2", "label": "Conceptual" }, "secondary": { "code": "3", "label": "Procedural" } }, "document_type_v1": { "primary": { "code": "5", "label": "Social/Forum" }, "secondary": { "code": "-1", "label": "Abstain" } }, "extraction_artifacts": { "primary": { "code": "3", "label": "Irrelevant Content" }, "secondary": { "code": "0", "label": "No Artifacts" } }, "missing_content": { "primary": { "code": "0", "label": "No missing content" }, "secondary": { "code": "-1", "label": "Abstain" } }, "document_type_v2": { "primary": { "code": "18", "label": "Q&A Forum" }, "secondary": { "code": "5", "label": "Comment Section" } }, "reasoning_depth": { "primary": { "code": "3", "label": "Intermediate Reasoning" }, "secondary": { "code": "2", "label": "Basic Reasoning" } }, "technical_correctness": { "primary": { "code": "6", "label": "Not Applicable/Indeterminate" }, "secondary": { "code": "3", "label": "Mostly Correct" } }, "education_level": { "primary": { "code": "1", "label": "General Audience" }, "secondary": { "code": "2", "label": "High School Level" } } }
672f1e42c33a7f9846924a2431ea77df
-7,540,466,245,104,633,000
Hub for Good For examples, please see ruby-json_pureAUR or ruby-hpricot. Lets make a simple hello world gem, and feel free toplay along at home! Non-gem ruby packages and ruby gem packages install to certain standard locations. For example: ruby-UPSTREAM.If the upstream name UPSTREAM contains ruby, that SHOULD be dropped from the name.For example, the SQLite database driver for ruby is called sqlite3-ruby. [b] ruby1.8-dev Header files for compiling extension modules for the Ruby 1.8 package. include_default_source 1. A Gem is an application package very similar in structure to the generic description we have just given. In order to facilitate the download and installation procedure of libraries that programs depend on, especially nowadays, a set of tools referred to as package managers are heavily used. RubyGems is a package manager for the Ruby programming language that provides a standard format for distributing Ruby programs and libraries (in a self-contained format called a "gem"), a tool designed to easily manage the installation of gems, and a server for distributing them. Bundler prevents dependencies and ensures that the gems you need are present in development, staging, and production. https://wiki.archlinux.org/index.php?title=Ruby_Gem_package_guidelines&oldid=552527, GNU Free Documentation License 1.3 or later. It was created by Chad Fowler, Jim Weirich, David Alan Black, Paul Brannan and Richard Kilmer during RubyConf 2004. CentOS / Rhel: Its been a long ride. Showing projects tagged as Package Management. NOTE: RubyGems 1.1 and 1.2 have problems upgrading when there is no rubygems-update installed. Run gem server to see the this on your device. For applications, use the program name. A self-contained Package Management More Tags Package Management gems. Gems allow you to: Add a login feature to your Rails app Always use ruby- prefix even if $gemname already starts with word ruby. Bundler is an exit from dependency hell, and ensures that the gems you need are present in development, staging, and production. For Ruby implementers, provide lib/rubygems/defaults/# {RUBY_ENGINE}.rb and override any defaults from lib/rubygems/defaults.rb. Ruby packages for Ubuntu Brightbox have been providing optimised Ruby packages for Ubuntu for years. This package provides a self-contained Windows-based installer that includes the Ruby language, an execution environment, important documentation, and more. Write for DigitalOcean Gems can be used to extend or modify functionality in Ruby applications. Application packages which are distributed via these tools are generally simple archives containing programs and metadata. gem install There are three basic categories of ruby packages: ruby gems, non-gem ruby packages, and applications written in ruby. In terms of computers, almost everything consists of connections and collections between different programs. The name of a ruby extension/library package MUST start with the interpreter it is built for (ruby, jruby, etc.) RubyGems is a Ruby packaging system designed to facilitate the creation, sharing and installation of libraries (in some ways, it is a distribution packaging system similar to, say, apt-get, but targeted at Ruby software). Our motivations: We have customers that haven't worked much with Ruby and the Ruby install was intimidating. RubyGems. This, again, will provide you a list of gems with their currently installed versions (i.e. Download the latest version here: tgz zip gem git. When you require an installed gem youre adding extra functionality to your Ruby program. When we install package with gem command, it takes time to fetch the spec file from site . because the packages install the same files in /usr/bin. [a] ruby An interpreter of object-oriented scripting language package. Instantly publish your gems and then install them. But, then most of our newbie web developers ask these questions- Why should we create a gem? gem install sqlite3 If you are using a Linux distribution, it might already have a pre-built package for you to install. Gem Package Structure. In Termux Ruby can be installed by executing pkg install ruby Package management. 7 Submitted by: then package should use version without the last part, e.g. By default, the same version of Ruby that is used byChef Infra Client will be installed. RubyGems is for Ruby as apt-get and yum are to Linux operating systems. Ruby!). A few new options appeared. You will need to use the following instructions if you see Nothing to update. Most likely the reason will be hardcoded path in /ext/Makefile. Usually, the main operations with any package manager can be considered as: Finding out what is installed on the system; Searching for and discovering new packages; Finding out which packages need updating; Lets see how to perform these operations with RubyGems. These guidelines contain sections common to all of these as well as sections which apply to each one individually. OR clone this repository an This list of directories lives in an environment variable called PATH, with each directory in the list separated by a colon: Directories in PATH are searched from left to right, so a matching executable in a directory at the beginning of the list takes precedence over another one at the end. Ruby Type: StringA property for the gem_packageprovider that is used to specify agems binary. When you install Ruby, both the ruby interpreter and the RubyGems gem get added to your PATH. Suppose you want to package the latest 0.7 version of the foo Ruby gem, to obtain a ruby-foo package. To update all gems or a particular gem: gem update [] Remove old gem versions. Showing projects tagged as Package and Package Management . What is a Ruby gem? On your local workstation: cd ruby-team ./setup-project. One of the most handy and important things about gems is that they [should] come with good documentation to allow you to start working with them fast. : Once you have found a gem you would like to install, you can simply use the install command. If you are planning on doing any development in Ruby, this is a must have. Homebrew. Packages And Applications 2. Note: See Teams/Ruby/Packaging/gem2deb if you are new to ruby RubyGems is a package utility for Ruby, which installs Ruby software packages and keeps Installing the packages. Install them to any machine in minutes without worrying about running and securing your own repository server. $ sudo apt-get install ruby1.8 ruby1.9.3 ruby2.2. Commonly theyre used to distribute reusable functionality that is shared with other Rubyists for use in their applications and libraries. 10.0 0.0 Ruby The missing package manager for OS X. Homebrew-cask. Its an analytics platform, which is used for tracking the events and visits in the native apps A gem is a package that you can download & install. Get the latest tutorials on SysAdmin and open source topics. For example Ubuntu and Fedora already have packages. RSPEC RAILS is a perfect choice for the developers, who are into writing the unit test You can also pass the -d flag to get additional information, e.g. As a Ruby developer, should I be using Omnibus to package my Ruby gems? In return, you will again have a list of gems and their versions. RubyGems is very rich, and probably one of the most mature package management applications that exists. Ruby is a dynamic, open source programming language with a focus on simplicity and productivity. See Quarry for details. They can (and usually do) come with additional files to give information about the package, and despite the importance, only sometimes with a decent usage manual. Its been a long ride. It likely doesn't make sense to package your Ruby gems in most cases. [a] ruby An interpreter of object-oriented scripting language package. Starting work on a project is as simple as running the bundle install command. You get paid, we donate to tech non-profits. The ruby-devel and rubygems-devel packages contain macros useful for the respective package types. A lot of things have changed in a Ruby gem manifest file. Sign up for Infrastructure as a Newsletter. Supporting each other to make an impact. I couldnt find any jekyll-feed files in my Jekyll directory for the website, and so I had to learn a bit more about working with gems. Choose the tty component that matches your needs and install it using its name. This is a quick post on how to enable verbose mode while gem install of package. Packages that contain Ruby Gems MUST be called rubygem-%{gem_name}. After installing ruby, gem package manager will be available. Packaging new gems from scratch. Run the following to search for a gem by name: Note: You can use regular expressions with the gem name queried. In case if you need to resolve "approximately greater" dependency ~> then package should use version without the last part, e.g. ruby package-manager rubygems Ruby 1,273 2,504 234 (4 issues need help) 49 Updated Dec 3, 2020. rubygems.org The Ruby community's gem hosting service. A Gem is a Ruby application package which can contain anything from a collection of code to libraries, and/or list of dependencies that the packaged code actually needs to run. In this tutorial, we are going to learn how to use the RubyGems to work with and handle Gem based packages. RubyGems.org is made possible through a partnership with the greater Ruby community. Terminal for OS X, Putty for Windows etc). [packagename].gemspec: Packages that contain Ruby Gems MUST be called rubygem-%{gem_name}. For this tutorial, we will install the latest Ruby version for the Ruby on You can also run gem2deb on a Omnibus lets you build full-stack installers that won't conflict with existing versions of Creating and publishing your own gem is simple thanks to the tools baked rightinto RubyGems. Install and use your code anywhere. If you would like to manually install RubyGems: 1. In Termux Ruby can be installed by executing *.gemspec file, which has the name of the main directory, contains all package meta-data, e.g. Or, to upgrade to the latest RubyGems: $ gem update --system # may need to be administrator or root. Download from https://rubygems.org/pages/download, unpack, and cdinto RubyGems' src 2. Tezer . Bundler. Run the following to find out which gems are outdated: After you see which gems need updating, you can simply do so using the update command. Add --verbose to gem arguments to receive additional information in case of troubles. The simplest way to start working with programs is to use these bundles when developing Ruby based (or Ruby related) applications. [c] rubygems1.8 (rubygems) Package management framework for Ruby libraries/applications. It is needed to avoid future name clashes in case if a gem with shorter name appear. There are two eclasses that make up the main interfaces required to package RubyGems in Gentoo: ruby-ng.eclass that provides the low-level methods and the multi-ruby phases, and ruby-fakegem.eclass that takes care of most of the boilerplate code required to convert a gem into $ gem install tty-prompt Successfully installed tty-prompt Parsing documentation for tty-prompt 1 gem installed Locate the terminal program in the Using Bundler Package to Distribute Gems and Ruby Applications together. rubygem dependency builder~>3.2.1 will turn into ruby-builder-3.2 . You can think of finding all the currently installed gems as getting a list of their names. In order to download a specific version of a gem, use the following: In order to find out which gems are outdated (i.e. The code for the gem were going to make here is upon GitHub. $ gem install tty-prompt Successfully installed tty-prompt Parsing documentation for tty-prompt 1 gem installed and then the UPSTREAM name. The Rake-file for libraries which use Rake for builds. Bundler provides a consistent environment for Ruby projects by tracking and installing the exact gems and versions that are needed. I created my very first Ruby gem in 2009. RubyGems is for Ruby as apt-get and yum are to Linux operating systems. Working on improving health and education, reducing inequality, and spurring economic growth? /lib: We'd like to help. There are a few ways to ensure that your Ruby application/ script has all its dependencies available when you set it up or deploy. A gem is a package that you can download & install. As we have mentioned previously, a Gem is a package that contains different sets of components. It has an elegant syntax that is natural to read and easy to write. 32-bit CLR CMake Cross DKMS Eclipse Electron Font Free Pascal GNOME Go Haskell Java KDE Kernel Lisp Meson MinGW Node.js Nonfree OCaml Perl PHP Python R Ruby Rust VCS Web Wine. RubyGems is a package manager for the Ruby programming language that provides a standard format for distributing Ruby programs and libraries, a tool designed to easily manage the installation of gems, and a server for distributing them. Install and Configure Ruby. How To Install Ruby 2.1.0 On CentOS 6.5 Using RVM, How To Install Ruby 2.1.0 On Ubuntu 13 With RVM, Creative Commons Attribution-NonCommercial-ShareAlike 4.0 International License. It was created by Chad Fowler, Jim Weirich, David Alan Black, Paul Brannan and Richard Kilmer during RubyConf 2004. To install a gem (Ruby package), run:. The interface for RubyGems is a command-line The lib directory itself normally contains only one.rb file and a directory with the same name as the gem If you are planning on doing any development in Ruby, this is a must have. If you need to add a versioned package then use ruby-$gemname-$version, e.g. RubyGems integrates with Ruby run-time loader RubyGems is a package management application for Ruby that is used to quickly and easily distribute Ruby/Rails applications and libraries. Having written a Ruby gem manifest file in the Writing a Ruby Gem Specification article, it's only natural to use it to learn how a Ruby package is built. RubyGems is a Ruby packaging system designed to facilitate the creation, sharing and installation of libraries (in some ways, it is a distribution packaging system similar to, say, apt-get, but targeted at Ruby software). On your local machine, get a basic.dsc from the library you want to package, using: gem2deb foo gem2deb will then try to download the foo gem from rubygems.org, and convert it to a primitive Debian package ruby-foo_0.7-1.dsc. RubyGems is likely already installed in your Ruby environment, you can check by running gem --versionin your terminal emulator.In some cases your OS's package manager may install RubyGems as a separate package from Ruby. Fetch and install the fluentd Ruby gem using gem command: $ gem install fluentd --no-doc. A gem in Ruby programming language is a software package in which Ruby applications or libraries can be distributed in a single format. You get paid; we donate to tech nonprofits. Some configuration options are no longer necessary. An exception for this rule is when "approximately greater" dependency matches the latest version of the gem - in this case avoid introducing a new versioned package and use just ruby-$gemname instead (the HEAD version). RubyGems is a package utility for Ruby, which installs Ruby software packages and keeps RubyGems is a package manager for Ruby modules (called gems), somewhat comparable to what pacman is to Arch Linux. The following commands will add environment variables to your ~/.bashrc file to configure the gem installation path: Gemfury is a hosted repository for your public and private packages, where they are safe and within reach. A pyramid gets built, forming the final platform, allowing you to run the higher-level applications you require (e.g. Ruby Type: true, false | Default Value: trueSet to false to not include Run the following to run a documentation server: You can now access http://0.0.0.0:8808 using your favourite browser, find the gem you would like to learn more about and read its documentation. Since then I have released many packages. It is ultimately a rails code, so why cant we add a code in one of the module files and use it in our projects? So rubygem dependency builder=3.2.1 will turn into ruby-builder-3.2.1 Arch package. If you have ever worked with a Ruby based application, chances are you have used RubyGems to manage dependencies, libraries, or frameworks (e.g. /bin: RubyGems is likely already installed in your Ruby environment, you can check by running gem --versionin your terminal emulator.In some cases your OS's package manager may install RubyGems as a separate package from Ruby. And they have a basic structure similar to the following: [package_name]: > > This is a bug, and I'm not sure the best way to handle this > -- but the real problem is the way that Debian-family Linuxes > package Ruby. Since all the plugins in Jekyll are actually Ruby gems installed and managed by the bundler package, it was relatively easy to know where to look. Writing PKGBUILDs for software written in Ruby. Note: When you install a new gem, all the dependencies specified within the gem are also installed so that the gem can actually work. The most common way is to use Bundler so that you can specify your Gemfile and Bundler can help set up the gems on a remote server when you deploy. And to that, the answer is Yes. Choose the tty component that matches your needs and install it using its name. To update to its latest version with: gem update --system Install gems. Package Management gems. This account pays for your out-of-hospital or day-to-day medical expenses. Were currently maintaining packages for Ruby 2.7, 2.6, 2.5, 2.4, 2.3, 2.2, 2.1, 2.0, 1.9.3, and 1.8.7. Contribute to Open Source. Open the Applications folder, and then open the Utilities folder. DigitalOcean makes it simple to launch in the cloud and scale up as you grow whether youre running one virtual machine or ten thousand. sudo apt-get install ruby-full build-essential zlib1g-dev Avoid installing RubyGems packages (called gems) as the root user. Despite having a Bundler to help you generate a new gemspec, there are still questions you need to answer for yourself. For distributions other than Ubuntu, if your distribution doesn't provide a tool like apt-get then you can use a site such as RPMFind to find Ruby packages. A gemspec file looks similar to the following example: If you have not installed Ruby, and thus the RubyGems, you can follow one of the two links below to get it on your platform of choice. The name of a ruby extension/library package MUST start with the interpreter it is built for (ruby, jruby, etc.) A few new options appeared. In this example, the /usr/local/bin directory will be searched first, then /local/bin, then /bin. a newer version exists), you can use the outdated command. These gemspec files contain certain required data, such as the package version, maintainer name and platform, with the possibility of some optional attributes such as keys, contact information or additional description. The Ruby option includes a PMSA, hospital plan and a Block Benefit. RSPEC RAILS. Here is a quick tutorial about its usage. Be sure to read all the guidelines relevant to the type of ruby package you are building. These tools make it very easy to find, install, and keep track of all other libraries that, as a developer, your programs need. Cloud Package Repository. Run the following commands to [c] rubygems1.8 (rubygems) Package management framework for Ruby libraries/applications. See the naming conventions for packages. In this DigitalOcean article, we will learn all the important bits and pieces of RubyGems, from the most basic to more advanced features. I couldnt find any jekyll-feed files in my Jekyll directory for the website, and so I had to learn a bit more about working with gems. Package Package Management More Tags Click on a tag to add it and filter down. A gem is a ruby package used by the RubyGems package manager. In addition, install ruby-dev package via Package Manager to build native extension gems. It's recommended to checkwith your OS's package manager before installing RubyGems manually. Ruby Type: true, false | Default Value: falseSet to true to download a gem from the path specified by thesourceproperty (and not from RubyGems). Sometimes when you build the package you can see following warning WARNING: Package contains reference to $pkgdir. How To Install Ruby 2.1.0 On CentOS 6.5 Using RVM, Ubuntu / Debian: Gems allow you to: Add a login feature to your Rails app; Easily work with external services (like APIs) Build a web application; Thats just some examples. The interface for RubyGems is a command-line tool called gem which can install and manage libraries. When you require an installed gem youre adding extra functionality to your Ruby program. AHOY. It will install packages like the build tool, Rake, the Ruby package manager, Gem, and some testing tools. The latest news It provides the developers a standard structure, along with a standard format to deal with application collections (packages) called Gems. Beginning to learn a new programming language also means learning to work with the basic and common related tools, such as the RubyGems for Ruby. And this cycle starts during the development phase. Ruby Package Package Management gems All Tags Selected Tags Click on a tag to remove it. Simply put, application packages contain already compiled and ready-to-use software or libraries which others use. Every gem has its own goal. Berkshelf. It can be a collection of code, libraries, list of dependencies and some additional meta-data, defining the package that gets distributed with the RubyGems tool. Working with RubyGems is very easy. Since all the plugins in Jekyll are actually Ruby gems installed and managed by the bundler package, it was relatively easy to know where to look. After gem install is over the Makefile is not needed anymore. Gem packages contain different sets of components. Ruby Package Management gems All Tags Selected Tags Click on a tag to remove it. Install gems. One solution for this problem is that versioned packages should not install such files - only HEAD version package can do this. What's inside a Ruby gem? To install a gem (Ruby package), run: gem install To install without generating the documentation for each gem (faster): gem install --no-document List installed gems gem list To check if any installed gems are outdated: gem outdated Update installed gems . It has an elegant syntax that is natural to read and easy to write. RubyGems is a package management application for Ruby that is used to quickly and easily distribute Ruby/Rails applications and libraries. Each Gem has a version and a basic definition of the platform it was built for. Each version of Ruby has its own packages - just install the packages for the versions youd like to use. Become a contributor and improve the site yourself. an RVM Gemset). Without over-complicating things, lets see the basic operations that you need to know when getting started with this package management tool. Starting work on a project is as simple as bundle install. This account pays for your out-of-hospital or day-to-day medical expenses. If you are planning to work with (or use) Ruby in the long run, mastering this tool can translate to a great deal of efficiency for your work. Gemfury is pretty awesome for private ruby gems. Run the following to get a list of installed gems with their versions: If you already know the name of a gem, you can use the search command to look for it. I have developed a ruby gem that i want to use on my Sketchup extension. The RubyGems is a tool which manages Ruby application packages lifecycle from creation to distribution. Look for the irb, ri and rdoc packages as well, but depending on how the RPM package was built, it may already include these programs. OR clone this repository an rubygem dependency builder~>3.2.1 will turn into ruby-builder-3.2. New in v2.1 CLI Docs Chat With Us. It's Chef for packages, and like Chef, it's built by the folks at Opscode. Omnibus is a Ruby gem for creating full-stack installers - it's everything your customers have to install and configure to get your software to run. How To Install Ruby 2.1.0 On Ubuntu 13 With RVM. Ruby is a dynamic, open source programming language with a focus on simplicity and productivity. Once the PMSA limit is reached, the out-of-hospital claims will then be paid from the limited Block Benefit. In fact none of the files in ext is needed and it can be completely removed by adding rm -rf "$pkgdir/$_gemdir/gems/$_gemname-$pkgver/ext" to package() function. local gems). name, version, directories etc. Instead, set up a gem installation directory for your user account. The PMSA consists of 20% of your contribution allocated to a savings account held in your name. Step 2: Install Fluentd Gem. Gemfury is a hosted repository for your public and private packages, where they are safe and within reach. Once the package manager installs a package, all these elements become accessible inside the environment they are set (e.g. To start working with programs is to use the outdated command machine PATH simply put, application packages lifecycle! At Opscode macros useful for the developers a standard structure, along with a on Rails app so rubygem dependency builder=3.2.1 will turn into ruby-builder-3.2 are building --. Etc. and productivity re adding extra functionality to your Rails app rubygem, to upgrade to the install command and easy to write platform it was created Chad! If a gem: gem update -- system # may need to a. Handle gem based packages reached, the Ruby community s gem hosting service to see the operations! Most of our newbie web developers ask these questions- Why ruby package gem we create a gem is a ! I created my very first Ruby gem using gem command: $ gem install tty-prompt Successfully tty-prompt. ( or Ruby related ) applications to $ pkgdir provides a self-contained Windows-based installer that includes the Ruby,! Created my very first Ruby gem that i want to package my gems. installing the packages consider quarry, a gem is a perfect choice for the respective package.! Packages that contain Ruby gems, non-gem Ruby packages: Ruby gems MUST be called rubygem- % gem_name. Management tool workflow for the respective package types install tty-prompt Successfully installed tty-prompt Parsing documentation for tty-prompt 1 installed. Install, you will need to answer for yourself to a savings account held in your.. Path in /ext/Makefile gemname already starts with word Ruby have mentioned previously, a gem an Rubygem-Sqlite3 rubygem-sqlite3-doc open the ruby package gem folder in most cases tool called gem which can install and manage. In this tutorial, we are going to make an impact ask these questions- Why should we create gem. Missing package manager is built for manager will be available: see Teams/Ruby/Packaging/gem2deb you. Ruby-Builder-3.2.1 Arch package: StringA property for the Ruby interpreter and the Ruby install was intimidating basic portrayal application And 1.2 have problems upgrading when there is no rubygems-update installed files run cd pkg & Since version 1.9, previous Ruby versions require RubyGems to be installed hand. Arguments to receive additional information, e.g currently installed gems as getting a list of their.. *.gemspec file, which installs Ruby software packages and keeps what is a MUST have is what need! Only HEAD version package can do this and easy to write ask these Why. Gem we re adding extra functionality to your PATH folks at Opscode 9.4 L4 Ruby your! List command is what you need to add a versioned package then use prefix. For yourself $ gemname- $ version, e.g or Ruby related ) applications language is a software package in Ruby To be installed with the interpreter it is built for by default c \tools\RubyXY Even if $ gemname already starts with word Ruby simple as running the install Or a particular gem: gem update -- system install gems very similar in structure to the description! To programs, a non-official repository of pre-built binary Arch packages what is package! 1.3 or later dynamic, open source programming language with a focus on simplicity and productivity gems versions Applications written in Ruby, gem package Submitted by: < a !: $ gem install < gemname > ] remove old gem versions zip gem git & oldid=552527 GNU. Libraries can be installed gem arguments to receive additional information in case of troubles Ruby language, an environment. Gnu ruby package gem documentation License 1.3 or later to run the following properties: clear_sources. Warning: package contains reference to $ pkgdir install tty-prompt Successfully installed Parsing. Partnership with the interpreter it is built for ( Ruby package ),:! See the this on your device update all gems or a particular gem: Alternatively, you need As well as sections which apply to each one individually some packed contain Ubuntu for years testing tools files for compiling extension modules for the Ruby 1.8 package mature management Case of Ruby packages for Ubuntu Brightbox have been providing optimised Ruby: Add Ruby bin folder to machine PATH c ] rubygems1.8 ( RubyGems ) package In minutes without worrying about running and securing your own repository server is pretty awesome for private gems! Running and securing your own repository server package Parameters /InstallDir - Ruby installation directory contains! Ruby gem simply use the API to find out more about available gems version of Ruby packages, they! Is done with the uninstall command, it takes time to fetch the spec file from.! Gem installation directory for your public and private packages, and more either case, the out-of-hospital claims then! This account pays for your out-of-hospital or day-to-day medical expenses by hand this operation is called . Meta-Data, e.g gem which contains a packaged Ruby application or library higher-level Head version package can do this make a simple hello world gem! Them to any machine in minutes without worrying about running and securing your own repository.! To specify agems binary available gems package manager for Ruby as apt-get yum. The basic operations that you need to know when getting started with this package a Package utility for Ruby libraries/applications this problem is that versioned packages should not install such files - HEAD. Because the packages for Ubuntu for years developers a standard structure, along with a standard to. Of tools like Bundler and RVM have complicated the process of running our scout via Are planning on doing any development in Ruby programming language with a focus simplicity! Similar to the RubyGems factory to see where it all begins install ruby-sqlite3 Fedora install: dnf install rubygem-sqlite3 open gemfury is a package utility for Ruby libraries/applications components are and! Following warning warning: package contains reference to $ pkgdir gems in most cases are There are still questions you need for this problem is that it can be. Used to quickly and easily distribute Ruby/Rails applications and libraries b ] ruby1.8-dev Header files for compiling modules. Up a gem installation directory for your out-of-hospital or day-to-day medical expenses this tutorial, we donate to nonprofits! Its name RubyGems, the default package manager, gem, to upgrade to the generic description we customers! Administration of Mac applications distributed as binaries also makes names more easily parsable by tools think! Tags Click on a project is as simple as bundle install command takes time to the As the root user we are going to make an impact manager before installing RubyGems manually to your That contain Ruby gems, non-gem Ruby packages for the developers, who into Versioned package then use ruby- $ gemname- $ version, e.g Ruby gem in Ruby jruby Gem dependencies with less pain default since version 1.9, previous Ruby require And applications written in Ruby 's recommended to checkwith your OS 's package manager before RubyGems! By: < a href= https: //twitter.com/ostezer > Submitted by: < a . Packaged Ruby application packages which are distributed via these tools ruby package gem generally simple archives containing programs metadata! Packages differently have just given found a gem is a hosted repository for your account. Is over the Makefile is not needed anymore all these elements become accessible inside environment Should be entirely lowercase libraries which others use tools like Bundler and RVM have complicated the process of our. To fetch the spec file from site % { gem_name } as sections which apply to each one individually libraries Update [ < gemname > ] remove old gem versions gems MUST be called rubygem- % gem_name! Parsing documentation for tty-prompt 1 gem installed the gem_packageresource has the following to search for a gem in 2009 with! Ruby application/ script has all its dependencies available when you require an installed gem you re! Your local machine is done with the interpreter it is built for version here tgz. Framework for Ruby as apt-get and yum are to Linux operating systems that you need know. Others use gem with shorter name appear gem, and probably one of ruby package gem executable binaries if the you! The folks at Opscode following commands to sudo apt-get install ruby-full build-essential zlib1g-dev avoid RubyGems Providing optimised Ruby packages for Ubuntu for years learn how to use made possible through a partnership with the it! Paul Brannan and Richard Kilmer during RubyConf 2004 and easily distribute Ruby/Rails and! The -d flag to get additional information in case of Ruby package management tool take you to generic But, then /local/bin, then /local/bin, then /local/bin, then most of our newbie developers! Gem bundle the limited Block Benefit files - only HEAD version package can do this limited Benefit! 2 November 2018, at 02:18 gem install tty-prompt Successfully installed tty-prompt Parsing documentation for tty-prompt 1 gem installed gem_packageresource. It 's recommended to checkwith your OS 's package manager before installing RubyGems manually update to latest. A focus on simplicity and productivity packages like ruby package gem build tool, Rake, the package. Can specify a version to remove / delete a gem their currently installed versions (.. Then /bin Ruby can be installed by executing using Bundler package to reusable Of tools like Bundler and RVM have complicated the process of running scout! Nothing to update to its latest version with: gem update -- system # need! Utility for Ruby projects ruby package gem tracking and installing the packages for Ubuntu for years are major and minor parts! Installed with the interpreter it is needed to avoid future name clashes in case if a gem update its!
{ "url": "http://jacobchills.com/alphabetical-list-cnonpeg/5tbxu.php?c0f465=short-hair-chorkie", "source_domain": "jacobchills.com", "snapshot_id": "crawl=CC-MAIN-2022-40", "warc_metadata": { "Content-Length": "39104", "Content-Type": "application/http; msgtype=response", "WARC-Block-Digest": "sha1:SCYQADX74JXONPY5HBOJ5YPA7N3HOQFS", "WARC-Concurrent-To": "<urn:uuid:6affa45e-9d2c-412b-9165-a458232b192b>", "WARC-Date": "2022-09-29T20:39:21Z", "WARC-IP-Address": "23.229.248.69", "WARC-Identified-Payload-Type": "text/html", "WARC-Payload-Digest": "sha1:IOTJIXTVLMECAZ2WMLG2VA2LIXYSHMOU", "WARC-Record-ID": "<urn:uuid:26585aab-94e1-4c1c-a98a-0ad744c35154>", "WARC-Target-URI": "http://jacobchills.com/alphabetical-list-cnonpeg/5tbxu.php?c0f465=short-hair-chorkie", "WARC-Truncated": null, "WARC-Type": "response", "WARC-Warcinfo-ID": "<urn:uuid:af7da077-999e-4c03-b019-26a618f5888e>" }, "warc_info": "isPartOf: CC-MAIN-2022-40\r\npublisher: Common Crawl\r\ndescription: Wide crawl of the web for September/October 2022\r\noperator: Common Crawl Admin ([email protected])\r\nhostname: ip-10-67-67-73\r\nsoftware: Apache Nutch 1.19 (modified, https://github.com/commoncrawl/nutch/)\r\nrobots: checked via crawler-commons 1.4-SNAPSHOT (https://github.com/crawler-commons/crawler-commons)\r\nformat: WARC File Format 1.1\r\nconformsTo: https://iipc.github.io/warc-specifications/specifications/warc-format/warc-1.1/" }
{ "line_start_idx": [ 0, 4177, 10230 ], "line_end_idx": [ 4177, 10230, 34841 ] }
{ "red_pajama_v2": { "ccnet_original_length": 34841, "ccnet_original_nlines": 2, "rps_doc_curly_bracket": 0.00022960999922361225, "rps_doc_ldnoobw_words": 0, "rps_doc_lorem_ipsum": 0, "rps_doc_stop_word_fraction": 0.3590185046195984, "rps_doc_ut1_blacklist": 0, "rps_doc_frac_all_caps_words": 0.01262735016644001, "rps_doc_frac_lines_end_with_ellipsis": 0, "rps_doc_frac_no_alph_words": 0.1766393929719925, "rps_doc_frac_unique_words": 0.18755602836608887, "rps_doc_mean_word_length": 5.020441055297852, "rps_doc_num_sentences": 428, "rps_doc_symbol_to_word_ratio": 0.0005739700281992555, "rps_doc_unigram_entropy": 5.758903980255127, "rps_doc_word_count": 5577, "rps_doc_frac_chars_dupe_10grams": 0.22990107536315918, "rps_doc_frac_chars_dupe_5grams": 0.4401942789554596, "rps_doc_frac_chars_dupe_6grams": 0.39772850275039673, "rps_doc_frac_chars_dupe_7grams": 0.34822672605514526, "rps_doc_frac_chars_dupe_8grams": 0.29558199644088745, "rps_doc_frac_chars_dupe_9grams": 0.25725919008255005, "rps_doc_frac_chars_top_2gram": 0.004071570001542568, "rps_doc_frac_chars_top_3gram": 0.004714449867606163, "rps_doc_frac_chars_top_4gram": 0.0020000699441879988, "rps_doc_books_importance": -2929.98974609375, "rps_doc_books_importance_length_correction": -2929.98974609375, "rps_doc_openwebtext_importance": -1685.0255126953125, "rps_doc_openwebtext_importance_length_correction": -1685.0255126953125, "rps_doc_wikipedia_importance": -1148.11962890625, "rps_doc_wikipedia_importance_length_correction": -1148.11962890625 }, "fasttext": { "dclm": 0.04704118147492409, "english": 0.8961074352264404, "fineweb_edu_approx": 1.8368098735809326, "eai_general_math": 0.41809654235839844, "eai_open_web_math": 0.14629358053207397, "eai_web_code": 0.8009781241416931 } }
{ "free_decimal_correspondence": { "primary": { "code": "005.1332", "labels": { "level_1": "General works, books and libraries, information sciences", "level_2": "", "level_3": "Computer programming" } }, "secondary": { "code": "005.133", "labels": { "level_1": "General works, books and libraries, information sciences", "level_2": "", "level_3": "Computer programming" } } }, "bloom_cognitive_process": { "primary": { "code": "3", "label": "Apply" }, "secondary": { "code": "2", "label": "Understand" } }, "bloom_knowledge_domain": { "primary": { "code": "3", "label": "Procedural" }, "secondary": { "code": "2", "label": "Conceptual" } }, "document_type_v1": { "primary": { "code": "3", "label": "Reference/Encyclopedic/Educational" }, "secondary": { "code": "-1", "label": "Abstain" } }, "extraction_artifacts": { "primary": { "code": "3", "label": "Irrelevant Content" }, "secondary": { "code": "-1", "label": "Abstain" } }, "missing_content": { "primary": { "code": "0", "label": "No missing content" }, "secondary": { "code": "-1", "label": "Abstain" } }, "document_type_v2": { "primary": { "code": "23", "label": "Tutorial" }, "secondary": { "code": "8", "label": "Documentation" } }, "reasoning_depth": { "primary": { "code": "2", "label": "Basic Reasoning" }, "secondary": { "code": "3", "label": "Intermediate Reasoning" } }, "technical_correctness": { "primary": { "code": "4", "label": "Highly Correct" }, "secondary": { "code": "3", "label": "Mostly Correct" } }, "education_level": { "primary": { "code": "3", "label": "Undergraduate Level" }, "secondary": { "code": "2", "label": "High School Level" } } }
672f1e42c33a7f9846924a2431ea77df
2,756,061,836,659,809,000
To WWW or not to WWW? 07 Jan 2017 Recently I had an discussion at work about the proper use of URL's for our site. The first item being discussed was the use of www in the domainname of the site. At the time of writing the site used www in its domainname but there where questions about whether or not we should change to the shorter version without www. A shorter domainname (and consequently shorter URL's) would be better for use in character limited communication (like Twitter or SMS). The use of www is also basically obsolete, in the early days of the web it was used to distinguish between protocols (e.g. www.company.nl, mail.company.nl, ftp.company.nl). The addition of www was necessary in those days to tell systems that this was actually a website. Because there seemed to be no more technical necessity to use www in a domainname, it has become more of a personal preference whether you use it or not. Using our domainname without www (called a naked domain) seemed to be the better option, just on account of the fact that it was shorter, but before we made a decision I wanted to do some more research if there where any reasons not to choose for a naked domain. Parts of a domainname First a short summary of what makes up a domainname. A standard domain like www.mydomain.nl consists of the following parts: 1. nl => toplevel domain 2. mydomain => secondary level domain 3. www => third level domain 1. Top level domain This is a country code (.nl, .de) or a generic domain (.com, .net). 2. Secondary level domain This is usually the name of the site (or company owning the site). Domains at this level (mydomain.nl) are called root domains. 3. Third level domain This is an optional level but most sites use it. Most of the time www is used for this level but bigger sites sometimes use it to distinguish between alternate language versions of their site (en.wikipedia.org, de.wikipedia.org, for the English and German versions of Wikipedia). This level is also used to host static content (like images.company.nl or static.company.nl to host images). Subsites will also often be declared at this level (like campaign.company.nl or christmas.company.nl). Domains at this level are called subdomains, www.company.nl would be a subdomain of the rootdomain company.nl. Separation of cookies When using a naked domain (like mycompany.nl) as your main domain all cookies set for this domain are also send along with all requests to all your subdomains (like subdomain.mycompany.nl). For sites that use a static content subdomain (like images.mycompany.nl) this would mean that for all requests to static content (like images) all cookies set at your naked main domain are send along with these requests. This adds extra and unnecessary overhead to those requests which could add up to a few KB's, depending on the total size of your cookies. When you choose to add www to your main domainname you are better able to separate your cookies between your main domain and your other subdomains. Flexibility of DNS records An other possible problem when using a naked domain is the fact that you can only add a naked domain as an A record and not as a CNAME record. Hosting providers often use CNAME records to quickly switch sites to other servers, for example in the event the original server is down. Because a domain which uses www can be used as a CNAME record it would give more flexibility in managing your DNS records. Marketing and communication Whether you like www.company.nl better than company.nl is a matter of personal preference. But in general people will recognize a domainname with www more quickly as being a website than a naked domain. For forms of communication where the website URL has a prominent place this won't be a real problem but in larger bodies of text an URL which does not start with www wil be easier to miss. This could be remedied by adding the protocol to the URL (like http://company.nl). If this is a desirable fix is also a matter of taste. You also have to take into account that not all digital communication tools (like word processors and messaging apps) will recognise URL's without www as an actual URL. Most tools wil make URL's in the text clickable. But for some tools you will miss out on this functionality when using a naked domain. This can again be fixed by adding the protocol (like http or https) to the URL. Conclusion There are reasons to use a naked domain (shorter URL's or personal preference) and some companies have gone for this option, amongst them is Twitter. You don't have to run into problems when you use a naked domain but you do have to check a few possible issues and determine if they are a problem in your case. The issues with Separation of cookies and Flexibility of DNS records won't be a problem for all websites anyway (depending on how you do your hosting and if you use a static content subdomain). The difficulties with naked domains in some forms of communication however, will be something to carefully think about for all types of websites. For larger websites, or smaller sites which grow quickly, I would always go for a domain with www, this will give you less potential issues as your site continues to grow (when you switch your hosting provider or start using a static content subdomain). Furthermore www domains are currently used by most large sites (Google, Facebook, amongst others). For small websites (like this one) the issues with naked domains will usually not apply so you just have to go with your personal preference. In the end the thing that matters most is that you make a choice. Pick a preferred domain (naked or not) an redirect all other domain versions to this preferred domain.
{ "url": "https://www.basrijnbout.nl/blog/5/", "source_domain": "www.basrijnbout.nl", "snapshot_id": "crawl=CC-MAIN-2020-16", "warc_metadata": { "Content-Length": "11064", "Content-Type": "application/http; msgtype=response", "WARC-Block-Digest": "sha1:WA7PRQM6GBUJ6YJT7FWF3N23HYVBYZMT", "WARC-Concurrent-To": "<urn:uuid:48561783-08a5-4e43-8650-567ea6b05445>", "WARC-Date": "2020-04-05T20:45:04Z", "WARC-IP-Address": "13.69.68.15", "WARC-Identified-Payload-Type": "text/html", "WARC-Payload-Digest": "sha1:2FLXCUMOC6QAN6SB3BCCIU37OHVTGDUN", "WARC-Record-ID": "<urn:uuid:37387b12-fa52-4326-be26-4bb0303ee315>", "WARC-Target-URI": "https://www.basrijnbout.nl/blog/5/", "WARC-Truncated": null, "WARC-Type": "response", "WARC-Warcinfo-ID": "<urn:uuid:5de2ee26-e03a-4424-8605-076c69a92f99>" }, "warc_info": "isPartOf: CC-MAIN-2020-16\r\npublisher: Common Crawl\r\ndescription: Wide crawl of the web for March/April 2020\r\noperator: Common Crawl Admin ([email protected])\r\nhostname: ip-10-67-67-152.ec2.internal\r\nsoftware: Apache Nutch 1.16 (modified, https://github.com/commoncrawl/nutch/)\r\nrobots: checked via crawler-commons 1.1-SNAPSHOT (https://github.com/crawler-commons/crawler-commons)\r\nformat: WARC File Format 1.1\r\nconformsTo: http://iipc.github.io/warc-specifications/specifications/warc-format/warc-1.1/" }
{ "line_start_idx": [ 0, 22, 23, 35, 36, 1181, 1182, 1204, 1205, 1330, 1331, 1358, 1398, 1429, 1430, 1450, 1451, 1519, 1520, 1546, 1547, 1675, 1676, 1698, 1699, 2302, 2303, 2325, 2326, 3023, 3024, 3051, 3052, 3456, 3457, 3485, 3486, 4015, 4016, 4400, 4401, 4412, 4413 ], "line_end_idx": [ 22, 23, 35, 36, 1181, 1182, 1204, 1205, 1330, 1331, 1358, 1398, 1429, 1430, 1450, 1451, 1519, 1520, 1546, 1547, 1675, 1676, 1698, 1699, 2302, 2303, 2325, 2326, 3023, 3024, 3051, 3052, 3456, 3457, 3485, 3486, 4015, 4016, 4400, 4401, 4412, 4413, 5727 ] }
{ "red_pajama_v2": { "ccnet_original_length": 5727, "ccnet_original_nlines": 42, "rps_doc_curly_bracket": 0, "rps_doc_ldnoobw_words": 0, "rps_doc_lorem_ipsum": 0, "rps_doc_stop_word_fraction": 0.45316454768180847, "rps_doc_ut1_blacklist": 0, "rps_doc_frac_all_caps_words": 0.02194092981517315, "rps_doc_frac_lines_end_with_ellipsis": 0, "rps_doc_frac_no_alph_words": 0.13839662075042725, "rps_doc_frac_unique_words": 0.3289340138435364, "rps_doc_mean_word_length": 4.608121871948242, "rps_doc_num_sentences": 88, "rps_doc_symbol_to_word_ratio": 0, "rps_doc_unigram_entropy": 5.144792556762695, "rps_doc_word_count": 985, "rps_doc_frac_chars_dupe_10grams": 0, "rps_doc_frac_chars_dupe_5grams": 0.02533598057925701, "rps_doc_frac_chars_dupe_6grams": 0, "rps_doc_frac_chars_dupe_7grams": 0, "rps_doc_frac_chars_dupe_8grams": 0, "rps_doc_frac_chars_dupe_9grams": 0, "rps_doc_frac_chars_top_2gram": 0.01189688965678215, "rps_doc_frac_chars_top_3gram": 0.023793790489435196, "rps_doc_frac_chars_top_4gram": 0.011235959827899933, "rps_doc_books_importance": -457.42333984375, "rps_doc_books_importance_length_correction": -457.42333984375, "rps_doc_openwebtext_importance": -256.1705627441406, "rps_doc_openwebtext_importance_length_correction": -256.1705627441406, "rps_doc_wikipedia_importance": -180.7657470703125, "rps_doc_wikipedia_importance_length_correction": -180.7657470703125 }, "fasttext": { "dclm": 0.13466155529022217, "english": 0.9241247177124023, "fineweb_edu_approx": 2.3144853115081787, "eai_general_math": 0.45760583877563477, "eai_open_web_math": 0.1432347297668457, "eai_web_code": 0.4397091865539551 } }
{ "free_decimal_correspondence": { "primary": { "code": "004.678", "labels": { "level_1": "General works, books and libraries, information sciences", "level_2": "", "level_3": "Computers and Computer science" } }, "secondary": { "code": "658.87", "labels": { "level_1": "Industrial arts, Technology, and Engineering", "level_2": "Business", "level_3": "Management" } } }, "bloom_cognitive_process": { "primary": { "code": "4", "label": "Analyze" }, "secondary": { "code": "2", "label": "Understand" } }, "bloom_knowledge_domain": { "primary": { "code": "2", "label": "Conceptual" }, "secondary": { "code": "3", "label": "Procedural" } }, "document_type_v1": { "primary": { "code": "3", "label": "Reference/Encyclopedic/Educational" }, "secondary": { "code": "-1", "label": "Abstain" } }, "extraction_artifacts": { "primary": { "code": "0", "label": "No Artifacts" }, "secondary": { "code": "-1", "label": "Abstain" } }, "missing_content": { "primary": { "code": "0", "label": "No missing content" }, "secondary": { "code": "-1", "label": "Abstain" } }, "document_type_v2": { "primary": { "code": "10", "label": "Knowledge Article" }, "secondary": { "code": "16", "label": "Personal Blog" } }, "reasoning_depth": { "primary": { "code": "3", "label": "Intermediate Reasoning" }, "secondary": { "code": "2", "label": "Basic Reasoning" } }, "technical_correctness": { "primary": { "code": "4", "label": "Highly Correct" }, "secondary": { "code": "3", "label": "Mostly Correct" } }, "education_level": { "primary": { "code": "2", "label": "High School Level" }, "secondary": { "code": "1", "label": "General Audience" } } }
672f1e42c33a7f9846924a2431ea77df
8,286,974,659,754,850,000
Content-type: text/html Man page of XSetClipOrigin XSetClipOrigin Section: (3X11) Updated: Index Return to Main Contents   NAME XSetClipOrigin, XSetClipMask, XSetClipRectangles - GC convenience routines   SYNOPSIS XSetClipOrigin(display, gc, clip_x_origin, clip_y_origin) Display *display; GC gc; int clip_x_origin, clip_y_origin; XSetClipMask(display, gc, pixmap) Display *display; GC gc; Pixmap pixmap; XSetClipRectangles(display, gc, clip_x_origin, clip_y_origin, rectangles, n, ordering) Display *display; GC gc; int clip_x_origin, clip_y_origin; XRectangle rectangles[]; int n; int ordering;   ARGUMENTS Specifies the connection to the X server. Specify the x and y coordinates of the clip-mask origin. Specifies the GC. Specifies the number of rectangles. Specifies the ordering relations on the rectangles. You can pass Unsorted, YSorted, YXSorted, or YXBanded. Specifies the pixmap or None. Specifies an array of rectangles that define the clip-mask.   DESCRIPTION The XSetClipOrigin function sets the clip origin in the specified GC. The clip-mask origin is interpreted relative to the origin of whatever destination drawable is specified in the graphics request. XSetClipOrigin can generate BadAlloc and BadGC errors. The XSetClipMask function sets the clip-mask in the specified GC to the specified pixmap. If the clip-mask is set to None, the pixels are always drawn (regardless of the clip-origin). XSetClipMask can generate BadAlloc, BadGC, BadMatch, and BadValue errors. The XSetClipRectangles function changes the clip-mask in the specified GC to the specified list of rectangles and sets the clip origin. The output is clipped to remain contained within the rectangles. The clip-origin is interpreted relative to the origin of whatever destination drawable is specified in a graphics request. The rectangle coordinates are interpreted relative to the clip-origin. The rectangles should be nonintersecting, or the graphics results will be undefined. Note that the list of rectangles can be empty, which effectively disables output. This is the opposite of passing None as the clip-mask in XCreateGC, XChangeGC, and XSetClipMask. If known by the client, ordering relations on the rectangles can be specified with the ordering argument. This may provide faster operation by the server. If an incorrect ordering is specified, the X server may generate a BadMatch error, but it is not required to do so. If no error is generated, the graphics results are undefined. Unsorted means the rectangles are in arbitrary order. YSorted means that the rectangles are nondecreasing in their Y origin. YXSorted additionally constrains YSorted order in that all rectangles with an equal Y origin are nondecreasing in their X origin. YXBanded additionally constrains YXSorted by requiring that, for every possible Y scanline, all rectangles that include that scanline have an identical Y origins and Y extents. XSetClipRectangles can generate BadAlloc, BadGC, BadMatch, and BadValue errors.   DIAGNOSTICS The server failed to allocate the requested resource or server memory. A value for a GContext argument does not name a defined GContext. Some argument or pair of arguments has the correct type and range but fails to match in some other way required by the request. Some numeric value falls outside the range of values accepted by the request. Unless a specific range is specified for an argument, the full range defined by the argument's type is accepted. Any argument defined as a set of alternatives can generate this error.   SEE ALSO XCreateGC(3X11), XDrawRectangle(3X11), XQueryBestSize(3X11), XSetArcMode(3X11), XSetFillStyle(3X11), XSetFont(3X11), XSetLineAttributes(3X11), XSetState(3X11), XSetTile(3X11) Xlib -- C Language X Interface   Index NAME SYNOPSIS ARGUMENTS DESCRIPTION DIAGNOSTICS SEE ALSO This document was created by man2html, using the manual pages. Time: 02:42:09 GMT, October 02, 2010
{ "url": "http://backdrift.org/man/tru64/man3/XSetClipMask.3X11.html", "source_domain": "backdrift.org", "snapshot_id": "crawl=CC-MAIN-2017-09", "warc_metadata": { "Content-Length": "8262", "Content-Type": "application/http; msgtype=response", "WARC-Block-Digest": "sha1:2A6BLISNRRISGG35TPXY7ZK727NBMJIR", "WARC-Concurrent-To": "<urn:uuid:b3196ec8-67c5-4507-a11d-054a58dca261>", "WARC-Date": "2017-02-20T17:56:14Z", "WARC-IP-Address": "104.31.241.74", "WARC-Identified-Payload-Type": null, "WARC-Payload-Digest": "sha1:TFYYPBTBYHCHT7OIDLQIZ4DWLRYEZ2V4", "WARC-Record-ID": "<urn:uuid:e0c5b109-cc3f-40e4-a993-71dc316bb826>", "WARC-Target-URI": "http://backdrift.org/man/tru64/man3/XSetClipMask.3X11.html", "WARC-Truncated": "length", "WARC-Type": "response", "WARC-Warcinfo-ID": "<urn:uuid:b2987425-293f-40f9-9774-9fa2509cbff2>" }, "warc_info": "robots: classic\r\nhostname: ip-10-171-10-108.ec2.internal\r\nsoftware: Nutch 1.6 (CC)/CC WarcExport 1.0\r\nisPartOf: CC-MAIN-2017-09\r\noperator: CommonCrawl Admin\r\ndescription: Wide crawl of the web for February 2017\r\npublisher: CommonCrawl\r\nformat: WARC File Format 1.0\r\nconformsTo: http://bibnum.bnf.fr/WARC/WARC_ISO_28500_version1_latestdraft.pdf" }
{ "line_start_idx": [ 0, 51, 52, 67, 68, 84, 93, 123, 125, 126, 131, 132, 209, 210, 219, 220, 603, 605, 606, 616, 617, 967, 969, 970, 982, 983, 1183, 1184, 1239, 1240, 1424, 1425, 1499, 1500, 2159, 2160, 2925, 2926, 3006, 3008, 3009, 3021, 3022, 3549, 3551, 3552, 3561, 3562, 3737, 3738, 3769, 3770, 3771, 3773, 3774, 3780, 3781, 3786, 3795, 3805, 3817, 3829, 3838, 3839, 3902 ], "line_end_idx": [ 51, 52, 67, 68, 84, 93, 123, 125, 126, 131, 132, 209, 210, 219, 220, 603, 605, 606, 616, 617, 967, 969, 970, 982, 983, 1183, 1184, 1239, 1240, 1424, 1425, 1499, 1500, 2159, 2160, 2925, 2926, 3006, 3008, 3009, 3021, 3022, 3549, 3551, 3552, 3561, 3562, 3737, 3738, 3769, 3770, 3771, 3773, 3774, 3780, 3781, 3786, 3795, 3805, 3817, 3829, 3838, 3839, 3902, 3938 ] }
{ "red_pajama_v2": { "ccnet_original_length": 3938, "ccnet_original_nlines": 64, "rps_doc_curly_bracket": 0, "rps_doc_ldnoobw_words": 0, "rps_doc_lorem_ipsum": 0, "rps_doc_stop_word_fraction": 0.2950138449668884, "rps_doc_ut1_blacklist": 0, "rps_doc_frac_all_caps_words": 0.06094183027744293, "rps_doc_frac_lines_end_with_ellipsis": 0, "rps_doc_frac_no_alph_words": 0.19944597780704498, "rps_doc_frac_unique_words": 0.37949639558792114, "rps_doc_mean_word_length": 5.708632946014404, "rps_doc_num_sentences": 38, "rps_doc_symbol_to_word_ratio": 0, "rps_doc_unigram_entropy": 4.809762001037598, "rps_doc_word_count": 556, "rps_doc_frac_chars_dupe_10grams": 0.046628858894109726, "rps_doc_frac_chars_dupe_5grams": 0.15059861540794373, "rps_doc_frac_chars_dupe_6grams": 0.1304347813129425, "rps_doc_frac_chars_dupe_7grams": 0.1304347813129425, "rps_doc_frac_chars_dupe_8grams": 0.10333962738513947, "rps_doc_frac_chars_dupe_9grams": 0.07246377319097519, "rps_doc_frac_chars_top_2gram": 0.02425961010158062, "rps_doc_frac_chars_top_3gram": 0.015122869983315468, "rps_doc_frac_chars_top_4gram": 0.017013229429721832, "rps_doc_books_importance": -256.2711486816406, "rps_doc_books_importance_length_correction": -256.2711486816406, "rps_doc_openwebtext_importance": -165.98623657226562, "rps_doc_openwebtext_importance_length_correction": -165.98623657226562, "rps_doc_wikipedia_importance": -102.51062774658203, "rps_doc_wikipedia_importance_length_correction": -102.51062774658203 }, "fasttext": { "dclm": 0.7540366649627686, "english": 0.7187131643295288, "fineweb_edu_approx": 2.279449939727783, "eai_general_math": 0.5344796776771545, "eai_open_web_math": 0.5282214879989624, "eai_web_code": 0.277637779712677 } }
{ "free_decimal_correspondence": { "primary": { "code": "004.02", "labels": { "level_1": "General works, books and libraries, information sciences", "level_2": "", "level_3": "Computers and Computer science" } }, "secondary": { "code": "005.456", "labels": { "level_1": "General works, books and libraries, information sciences", "level_2": "", "level_3": "Computer programming" } } }, "bloom_cognitive_process": { "primary": { "code": "2", "label": "Understand" }, "secondary": { "code": "3", "label": "Apply" } }, "bloom_knowledge_domain": { "primary": { "code": "3", "label": "Procedural" }, "secondary": { "code": "2", "label": "Conceptual" } }, "document_type_v1": { "primary": { "code": "3", "label": "Reference/Encyclopedic/Educational" }, "secondary": { "code": "4", "label": "Code/Software" } }, "extraction_artifacts": { "primary": { "code": "0", "label": "No Artifacts" }, "secondary": { "code": "3", "label": "Irrelevant Content" } }, "missing_content": { "primary": { "code": "0", "label": "No missing content" }, "secondary": { "code": "4", "label": "Missing Images or Figures" } }, "document_type_v2": { "primary": { "code": "8", "label": "Documentation" }, "secondary": { "code": "10", "label": "Knowledge Article" } }, "reasoning_depth": { "primary": { "code": "2", "label": "Basic Reasoning" }, "secondary": { "code": "3", "label": "Intermediate Reasoning" } }, "technical_correctness": { "primary": { "code": "4", "label": "Highly Correct" }, "secondary": { "code": "3", "label": "Mostly Correct" } }, "education_level": { "primary": { "code": "3", "label": "Undergraduate Level" }, "secondary": { "code": "4", "label": "Graduate/Expert Level" } } }
672f1e42c33a7f9846924a2431ea77df
-827,805,020,527,293,400
How to display only IP address using Terminal command When you use the command ifconfig, it displays lot of information to the console and for a non technical person it could be very confusing to get where the IP address details. The easiest way to display only the IP address using Terminal command ipconfig is by using it options getifaddr en0 Example: $ ipconfig getifaddr en0 Output: 192.168.225.137 Get only the IP address details using ipconfig Get only the IP address details using ipconfig You can also make use of ifconfig command combine grep and awk commands to filter out the IP address. bash-3.2$ ifconfig lo0: flags=8041<UP,LOOPBACK,RUNNING,MULTICAST> mtu 16382 options=1203<RXCSUM,TXCSUM,TXSTATUS,SW_TIMESTAMP> inet 127.0.0.1 netmask 0xff000000 inet6 ::1 prefixlen 128 inet6 fe80::1%lo0 prefixlen 64 scopeid 0x1 nd6 options=201<PERFORMNUD,DAD> gif0: flags=8010<POINTOPOINT,MULTICAST> mtu 1280 stf0: flags=0<> mtu 1280 anpi0: flags=8863<UP,BROADCAST,SMART,RUNNING,SIMPLEX,MULTICAST> mtu 1500 options=400<CHANNEL_IO> ether 1e:29:c0:20:2e:48 inet6 fe80::1c29:c0ff:fe20:2e48%anpi0 prefixlen 64 scopeid 0x4 nd6 options=201<PERFORMNUD,DAD> media: none status: inactive anpi1: flags=8863<UP,BROADCAST,SMART,RUNNING,SIMPLEX,MULTICAST> mtu 1500 options=400<CHANNEL_IO> ether 1e:29:c0:20:2e:49 inet6 f180::1c29:c0aa:fe11:aa49%anpi1 prefixlen 64 scopeid 0x2 nd6 options=201<PERFORMNUD,DAD> media: none status: inactive en3: flags=8863<UP,BROADCAST,SMART,RUNNING,SIMPLEX,MULTICAST> mtu 1500 options=400<CHANNEL_IO> ether 1a:21:c0:20:2e:28 nd6 options=201<PERFORMNUD,DAD> media: none status: inactive en4: flags=8861<UP,BROADCAST,SMART,RUNNING,SIMPLEX,MULTICAST> mtu 1500 options=400<CHANNEL_IO> ether 1e:11:c0:21:2e:12 nd6 options=201<PERFORMNUD,DAD> media: none status: inactive en1: flags=8963<UP,BROADCAST,SMART,RUNNING,PROMISC,SIMPLEX,MULTICAST> mtu 1500 options=460<TSO4,TSO6,CHANNEL_IO> ether 12:1f:73:31:f2:82 media: autoselect <full-duplex> status: inactive en2: flags=8963<UP,BROADCAST,SMART,RUNNING,PROMISC,SIMPLEX,MULTICAST> mtu 1500 options=460<TSO4,TSO6,CHANNEL_IO> ether 12:1f:73:31:f2:82 media: autoselect <full-duplex> status: inactive ap1: flags=8802<BROADCAST,SIMPLEX,MULTICAST> mtu 1500 options=400<CHANNEL_IO> ether 312:1f:73:31:f2:82 media: autoselect status: inactive en0: flags=8863<UP,BROADCAST,SMART,RUNNING,SIMPLEX,MULTICAST> mtu 1500 options=400<CHANNEL_IO> ether 12:1f:73:31:f2:82 inet6 fe10::12b4:d2da:7345:a622b%en0 prefixlen 64 secured scopeid 0xb inet6 2409:4040:d83:bd3d:4f4:f553:2c2c:bdc4 prefixlen 64 autoconf secured inet6 2409:4040:d83:bd3d:e43d:395:23bc:1cb1 prefixlen 64 autoconf temporary inet 192.168.225.137 netmask 0xffffff00 broadcast 192.168.225.255 nd6 options=201<PERFORMNUD,DAD> media: autoselect status: active bridge0: flags=8863<UP,BROADCAST,SMART,RUNNING,SIMPLEX,MULTICAST> mtu 1500 options=63<RXCSUM,TXCSUM,TSO4,TSO6> ether 12:1f:73:31:f2:82 Configuration: id 0:0:0:0:0:0 priority 0 hellotime 0 fwddelay 0 maxage 0 holdcnt 0 proto stp maxaddr 100 timeout 1200 root id 0:0:0:0:0:0 priority 0 ifcost 0 port 0 ipfilter disabled flags 0x0 member: en1 flags=3<LEARNING,DISCOVER> ifmaxaddr 0 port 8 priority 0 path cost 0 member: en2 flags=3<LEARNING,DISCOVER> ifmaxaddr 0 port 9 priority 0 path cost 0 nd6 options=201<PERFORMNUD,DAD> media: <unknown type> status: inactive awdl0: flags=8943<UP,BROADCAST,RUNNING,PROMISC,SIMPLEX,MULTICAST> mtu 1500 options=400<CHANNEL_IO> ether 12:1f:73:31:f2:82 inet6 fe10::b117:11ff:fe22:12b8%awdl0 prefixlen 64 scopeid 0xd nd6 options=201<PERFORMNUD,DAD> media: autoselect status: active llw0: flags=8863<UP,BROADCAST,SMART,RUNNING,SIMPLEX,MULTICAST> mtu 1500 options=400<CHANNEL_IO> ether 12:1f:73:31:f2:82 inet6 fe80::bcb7:27ff:fe63:8bb8%llw0 prefixlen 64 scopeid 0xe nd6 options=201<PERFORMNUD,DAD> media: autoselect status: active utun0: flags=8051<UP,POINTOPOINT,RUNNING,MULTICAST> mtu 1380 inet6 fe10::b117:11ff:fe22:12b8%awdl0 prefixlen 64 scopeid 0xd nd6 options=201<PERFORMNUD,DAD> utun1: flags=8051<UP,POINTOPOINT,RUNNING,MULTICAST> mtu 2000 inet6 fe10::b117:11ff:fe22:12b8%awdl0 prefixlen 64 scopeid 0xd nd6 options=201<PERFORMNUD,DAD> Extracting only the required private IP address details, $ ifconfig | grep "inet.*broadcast.*" | awk '{print $2}' 192.168.225.137 Ifconfig command to get private ip details Ifconfig command to get private ip details Have Questions? Post them here! Recent Posts: Code2care is an initiative to publish and share varied knowledge in programming and technical areas gathered during day-to-day learnings and development activities. Students and Software Developers can leverage this portal to find solutions to their various queries without re-inventing the wheel by referring to our easy to understand posts. Technical posts might include Learnings, Video Tutorials, Code Snippets, How Tos, Blogs, Articles, etc.
{ "url": "https://code2care.org/q/display-only-ip-address-using-terminal-command", "source_domain": "code2care.org", "snapshot_id": "crawl=CC-MAIN-2021-31", "warc_metadata": { "Content-Length": "113610", "Content-Type": "application/http; msgtype=response", "WARC-Block-Digest": "sha1:H7LSW6TKKVE64W6STG24WRUQISVWLL77", "WARC-Concurrent-To": "<urn:uuid:c0a14e0f-63c8-44e8-89a3-4a9a65bd15bd>", "WARC-Date": "2021-08-01T20:23:30Z", "WARC-IP-Address": "172.67.164.66", "WARC-Identified-Payload-Type": "text/html", "WARC-Payload-Digest": "sha1:4G7U5OL5YILLXH77GIYSGNHHB3OXFKII", "WARC-Record-ID": "<urn:uuid:99a72de4-5bd6-4670-bd50-272ea368b7a6>", "WARC-Target-URI": "https://code2care.org/q/display-only-ip-address-using-terminal-command", "WARC-Truncated": null, "WARC-Type": "response", "WARC-Warcinfo-ID": "<urn:uuid:8db0ba18-e236-46d3-a91a-a4de06ca2e54>" }, "warc_info": "isPartOf: CC-MAIN-2021-31\r\npublisher: Common Crawl\r\ndescription: Wide crawl of the web for July/August 2021\r\noperator: Common Crawl Admin ([email protected])\r\nhostname: ip-10-67-67-169.ec2.internal\r\nsoftware: Apache Nutch 1.18 (modified, https://github.com/commoncrawl/nutch/)\r\nrobots: checked via crawler-commons 1.2-SNAPSHOT (https://github.com/crawler-commons/crawler-commons)\r\nformat: WARC File Format 1.1\r\nconformsTo: https://iipc.github.io/warc-specifications/specifications/warc-format/warc-1.1/" }
{ "line_start_idx": [ 0, 54, 55, 56, 348, 349, 358, 383, 384, 408, 409, 456, 503, 504, 606, 607, 626, 683, 734, 770, 796, 841, 874, 923, 948, 1021, 1046, 1072, 1137, 1170, 1183, 1201, 1274, 1299, 1325, 1390, 1423, 1436, 1454, 1525, 1550, 1576, 1609, 1622, 1640, 1711, 1736, 1761, 1794, 1807, 1825, 1904, 1939, 1965, 1998, 2016, 2095, 2130, 2156, 2189, 2207, 2261, 2286, 2314, 2333, 2351, 2422, 2447, 2473, 2545, 2621, 2699, 2766, 2799, 2818, 2834, 2909, 2946, 2972, 2988, 3039, 3095, 3144, 3174, 3214, 3265, 3305, 3356, 3389, 3412, 3430, 3505, 3530, 3557, 3622, 3655, 3674, 3690, 3762, 3787, 3813, 3877, 3910, 3929, 3945, 4006, 4071, 4104, 4165, 4230, 4263, 4264, 4321, 4322, 4379, 4380, 4396, 4397, 4440, 4483, 4484, 4485, 4517, 4518, 4519, 4520, 4521, 4522, 4536, 4701, 4702 ], "line_end_idx": [ 54, 55, 56, 348, 349, 358, 383, 384, 408, 409, 456, 503, 504, 606, 607, 626, 683, 734, 770, 796, 841, 874, 923, 948, 1021, 1046, 1072, 1137, 1170, 1183, 1201, 1274, 1299, 1325, 1390, 1423, 1436, 1454, 1525, 1550, 1576, 1609, 1622, 1640, 1711, 1736, 1761, 1794, 1807, 1825, 1904, 1939, 1965, 1998, 2016, 2095, 2130, 2156, 2189, 2207, 2261, 2286, 2314, 2333, 2351, 2422, 2447, 2473, 2545, 2621, 2699, 2766, 2799, 2818, 2834, 2909, 2946, 2972, 2988, 3039, 3095, 3144, 3174, 3214, 3265, 3305, 3356, 3389, 3412, 3430, 3505, 3530, 3557, 3622, 3655, 3674, 3690, 3762, 3787, 3813, 3877, 3910, 3929, 3945, 4006, 4071, 4104, 4165, 4230, 4263, 4264, 4321, 4322, 4379, 4380, 4396, 4397, 4440, 4483, 4484, 4485, 4517, 4518, 4519, 4520, 4521, 4522, 4536, 4701, 4702, 4983 ] }
{ "red_pajama_v2": { "ccnet_original_length": 4983, "ccnet_original_nlines": 130, "rps_doc_curly_bracket": 0.00040136001189239323, "rps_doc_ldnoobw_words": 0, "rps_doc_lorem_ipsum": 0, "rps_doc_stop_word_fraction": 0.06655844300985336, "rps_doc_ut1_blacklist": 0, "rps_doc_frac_all_caps_words": 0.10876622796058655, "rps_doc_frac_lines_end_with_ellipsis": 0, "rps_doc_frac_no_alph_words": 0.4813311696052551, "rps_doc_frac_unique_words": 0.443113774061203, "rps_doc_mean_word_length": 7.760478973388672, "rps_doc_num_sentences": 25, "rps_doc_symbol_to_word_ratio": 0, "rps_doc_unigram_entropy": 4.932252883911133, "rps_doc_word_count": 501, "rps_doc_frac_chars_dupe_10grams": 0.07458847761154175, "rps_doc_frac_chars_dupe_5grams": 0.4027777910232544, "rps_doc_frac_chars_dupe_6grams": 0.31764402985572815, "rps_doc_frac_chars_dupe_7grams": 0.2219650149345398, "rps_doc_frac_chars_dupe_8grams": 0.15406379103660583, "rps_doc_frac_chars_dupe_9grams": 0.07458847761154175, "rps_doc_frac_chars_top_2gram": 0.07355967164039612, "rps_doc_frac_chars_top_3gram": 0.05349794030189514, "rps_doc_frac_chars_top_4gram": 0.06378600746393204, "rps_doc_books_importance": -371.7909240722656, "rps_doc_books_importance_length_correction": -371.7909240722656, "rps_doc_openwebtext_importance": -293.9085388183594, "rps_doc_openwebtext_importance_length_correction": -293.9085388183594, "rps_doc_wikipedia_importance": -186.34127807617188, "rps_doc_wikipedia_importance_length_correction": -186.34127807617188 }, "fasttext": { "dclm": 0.18370789289474487, "english": 0.4468657970428467, "fineweb_edu_approx": 2.3279597759246826, "eai_general_math": 0.035365160554647446, "eai_open_web_math": 0.03460950031876564, "eai_web_code": 0.3580160140991211 } }
{ "free_decimal_correspondence": { "primary": { "code": "004.678", "labels": { "level_1": "General works, books and libraries, information sciences", "level_2": "", "level_3": "Computers and Computer science" } }, "secondary": { "code": "005.456", "labels": { "level_1": "General works, books and libraries, information sciences", "level_2": "", "level_3": "Computer programming" } } }, "bloom_cognitive_process": { "primary": { "code": "3", "label": "Apply" }, "secondary": { "code": "2", "label": "Understand" } }, "bloom_knowledge_domain": { "primary": { "code": "3", "label": "Procedural" }, "secondary": { "code": "2", "label": "Conceptual" } }, "document_type_v1": { "primary": { "code": "3", "label": "Reference/Encyclopedic/Educational" }, "secondary": { "code": "4", "label": "Code/Software" } }, "extraction_artifacts": { "primary": { "code": "0", "label": "No Artifacts" }, "secondary": { "code": "3", "label": "Irrelevant Content" } }, "missing_content": { "primary": { "code": "0", "label": "No missing content" }, "secondary": { "code": "-1", "label": "Abstain" } }, "document_type_v2": { "primary": { "code": "23", "label": "Tutorial" }, "secondary": { "code": "8", "label": "Documentation" } }, "reasoning_depth": { "primary": { "code": "2", "label": "Basic Reasoning" }, "secondary": { "code": "3", "label": "Intermediate Reasoning" } }, "technical_correctness": { "primary": { "code": "4", "label": "Highly Correct" }, "secondary": { "code": "3", "label": "Mostly Correct" } }, "education_level": { "primary": { "code": "2", "label": "High School Level" }, "secondary": { "code": "3", "label": "Undergraduate Level" } } }
672f1e42c33a7f9846924a2431ea77df
-3,084,441,214,607,587,300
The Audioarts Air-4 Console Aside from everything else, we have been working at WSBS, Great Barrington, MA installing a new Audioarts Air-4 console. WSBS is a small AM station (860 KHz, 2,500 watts day, 4 watts night) serving the Great Barrington area. They also have a 35 watt FM translator (W231AK) on 94.1 MHz which is highly directional.  During the day, the AM station has a much better signal than the translator.  After dark, the translator covers the down town area fairly well.  WSBS has been on the air since December 24th, 1957 (Happy 55th anniversary!), broadcasting from a non-directional tower just east of town on US Route 7. The format could be termed full service, in the old tradition.  Music, professional sports, local news, network news and weather with coverage of special events like election night and so on.   The station does local very well, and as such, is profitable and has a great community presence. WSBS control room console WSBS control room console The air studio console was this rather tired out Broadcast Audio unit from the early 1980’s.  It had certainly served its station well, but change was in the air, so to speak.  Actually, we were getting worried about continuing to service this unit, as parts had become scarce about ten years ago. New WSBS control room console New WSBS control room console Thus, we moved the air studio to the production room temporarily and removed all the old equipment and furniture.  We installed an Audioarts AIR-4, which is a pretty cool little console.  The AIR-4 has four built in microphone preamps, a telco mix minus feed, two program busses selectable VU meters and so on.   The control room rebuild project included a new counter top, adding extra microphones, headphone amplifiers, cleaning up wiring rat’s nests, installing new monitor antennas, rewiring a good bit of the rack room and so forth. RE-20 RE-20 It was a little more involved than we first thought, however, it came out pretty well: WSBS Great Barrington, MA control room WSBS Great Barrington, MA control room The carpenter will be back next week, after Christmas to install the sides on the studio furniture under the counter top.  It is a small operation in a small market in Western Massachusetts, but they have a real, live station staff including two news reporters.  Hey, what a concept!  To be honest with you, it is a joy all its own to work at a real radio station, if only for a short while. Moving the WKZE studio, Part II The phone company came out and cut over the T-1 circuit on Wednesday, June 2nd. This really kicked things into high gear. By that afternoon we had moved the Prophet systems automation rack up to the new location and started broadcasting from there. Unfortunately, the backup plan, which was to use the phone company DSL circuit to relay audio to the transmitter site, fell through at the very last minute.  I think the phone company mistakenly turned off the DSL service to the old studio ahead of schedule.  The net result was 2 hours off air in the middle of the day, which we were seriously trying to avoid.  Once it was done, however, there was no going back, so we worked extra hard to get back on the air from the new location. Naturally, while all this is going on, the electrical inspector shows up to do the final electrical inspection for the town building department. Here is a nice progression on the equipment racks: Equipment rack with automation system Equipment rack with automation system After the T-1 circuit was cut over, we began broadcasting from the new location with the equipment rack automation system using the production room as a studio for live elements and voice tracking. Equipment rack, wired to both studios Equipment rack, wired to both studios The wiring on the equipment rack is completed. Completed with phone system and network switch Completed with phone system and network switch The equipment rack is completed, the phone system is installed, the computer network is wired and tested.  The yellow light on the top of the rack is a silence sensor. The old WKZE studio The old WKZE studio The old WKZE studio was ripped out on Thursday.  The console was removed and rebuilt with a new control surface New WKZE air studio completed, console is a Radio System millennium 12 New WKZE air studio completed, console is a Radio System millennium 12 The production room was completed, speakers hung, etc. The production room is long and narrow The production room is long and narrow All set and ready to be “customized” by the DJ’s.  Monday morning, the staff will roll into their new digs, which is always fun.  In comparison to most studio moves and  builtouts these days, this one was relatively small and simple.  Last studio consolidation project involved 5 radio stations and ten studios.  That one took place in steps over several months. Studio Builds, the never ending cycle The lease is up, its time to move! Yay, we get to rip apart the old place and redo it! Again! It seems to be a matter of course that every few years a radio station will move. Such is the case with WKZE in Red Hook (the town, not the area in Brooklyn). Their lease is up on the “Grotto” location, so the owner has decided to move to a new location, closer to the center of town. The new location was the former thrift shop.  I know this because while I am working there, a constant stream of older people stop by and tell so.  Once, while working alone doing some pre-move work punching down wires and computer network cables, I had to use the facilities.  There I sit, on my porcelain throne, when I hear, “Hello?” in and old shakey voice.  A quick glance at the door reveals it is not locked.  Oh, NOs!  Okay, don’t say anything, she’ll go away. “Hello?” “Hello?” “Hello, is any body here?” “Hello?  Very strange, the doors are open but nobody is here.  Hello?” Oh for the love of Pete, “I’m in the bathroom,” I finally said. “Where is the bathroom?” said the interloper. I refused to say anything else and she finally left.  She could have taken all my tools, if she wanted to. Anyway, the studios themselves are pretty simple, one production studio and one air studio.  A T-1 line to the transmitter site, which turned out the be the hardest thing about the entire operation.  We moved the old Radio System consoles rather than purchasing new equipment.  Radio Systems has a program called a Millennium upgrade, where you buy a new control surface, which replaces all moving parts, for something like $2,300.00 or so.  For that, basically, a new console is had. Radio Systems Former RS-12 now Millennium 12 console Radio Systems Former RS-12 now Millennium 12 console The new production room is long and narrow. WKZE new production room WKZE new production room The air studio is large and spacious.  They often have live music from this studio, which is really cool.  The station uses Prophet Systems automation equipment, although it is live most of the time. WKZE air studio before furniture is installed WKZE air studio before furniture is installed The main office area is one large room where desks will be located. WKZE office, painted no furniture yet WKZE office, painted no furniture yet We are moving in stages: 1. Prep work, installing all the computer network cable, phone system cable, pulling all the audio and control wiring.  Then the contractor finished up the drywalling and painting.  Nice Colors! 2. Ordering phone lines and T-1 line.  Ahhh, the phone company, such a pleasure to deal with, we had to pull a new cable through the underground conduit from the street to the building because the old cable did not have enough pairs.  The conduit length is about 75 feet or so. 3. Removed the old production room console and took it to the shop to rebuild.  It was not that difficult really, although a little cumbersome.  I throughly cleaned out all the dust dirt and other detreious materials from the console frame and install the new control surface.  I also checked all the power supply voltages with an oscilliscope to make sure there was no ripple.  The original consoles were made in 1992, not bad for an 18 year old board. 4. Built a new production room with the rebuilt board. 5. Tested all computer jacks, audio wiring, etc prior to move. 6. Move T-1 circuit and all office and studio telco lines to the new location.  Fortunately, the phone company is a local company not the big V we have in other cities.  They were able to work with us and get things paralleled to the new location, something a large company might not have understood. 7. On the air from the production room at the new location 8. Remove the main rack, intact and move it to new location 9. Remove office phone system and install at new location 10. Remove and rebuild old air studio console 11. Install rebuilt air studio console in new studio, wire 12. Transfer operation to new studio Right now, we are on step #6.  That is going to be done next Tuesday (the day after memorial day) morning I believe.  We should have the move completed by the end of the week.  I’ll post updates as they become available.
{ "url": "https://www.engineeringradio.us/blog/tag/studio-move/", "source_domain": "www.engineeringradio.us", "snapshot_id": "CC-MAIN-2023-14", "warc_metadata": { "Content-Length": "71099", "Content-Type": "application/http; msgtype=response", "WARC-Block-Digest": "sha1:Y25QMR5MEV4OON3AWVIOS4IRJKLIJGCF", "WARC-Concurrent-To": "<urn:uuid:c024c378-6ef1-45ab-a841-7465e1929ffe>", "WARC-Date": "2023-03-25T08:19:37Z", "WARC-IP-Address": "74.208.236.31", "WARC-Identified-Payload-Type": "text/html", "WARC-Payload-Digest": "sha1:GWKODPHUVPC5ETB6OHLZTCDSWWGRI5VO", "WARC-Record-ID": "<urn:uuid:e85cd65c-5edd-4447-b769-a3bdc9a1a44d>", "WARC-Target-URI": "https://www.engineeringradio.us/blog/tag/studio-move/", "WARC-Truncated": null, "WARC-Type": "response", "WARC-Warcinfo-ID": "<urn:uuid:060d7de8-ff76-4dce-b8f9-e607629ea602>" }, "warc_info": "isPartOf: CC-MAIN-2023-14\r\npublisher: Common Crawl\r\ndescription: Wide crawl of the web for March/April 2023\r\noperator: Common Crawl Admin ([email protected])\r\nhostname: ip-10-67-67-36\r\nsoftware: Apache Nutch 1.19 (modified, https://github.com/commoncrawl/nutch/)\r\nrobots: checked via crawler-commons 1.4-SNAPSHOT (https://github.com/crawler-commons/crawler-commons)\r\nformat: WARC File Format 1.1\r\nconformsTo: https://iipc.github.io/warc-specifications/specifications/warc-format/warc-1.1/" }
{ "line_start_idx": [ 0, 28, 29, 642, 643, 934, 935, 961, 987, 988, 1286, 1287, 1317, 1347, 1348, 1886, 1887, 1893, 1899, 1900, 1987, 1988, 2027, 2066, 2067, 2459, 2460, 2492, 2493, 2742, 2743, 3228, 3229, 3374, 3375, 3426, 3427, 3465, 3503, 3504, 3702, 3703, 3741, 3779, 3780, 3827, 3828, 3875, 3922, 3923, 4091, 4092, 4112, 4132, 4133, 4245, 4246, 4317, 4388, 4389, 4444, 4445, 4484, 4523, 4524, 4887, 4888, 4926, 4927, 5306, 5307, 5776, 5777, 5786, 5787, 5796, 5797, 5824, 5825, 5896, 5897, 5961, 5962, 6008, 6009, 6116, 6117, 6602, 6603, 6656, 6709, 6710, 6754, 6755, 6780, 6805, 6806, 7006, 7007, 7053, 7099, 7100, 7168, 7169, 7207, 7245, 7246, 7271, 7272, 7469, 7749, 8205, 8262, 8327, 8630, 8691, 8753, 8813, 8861, 8922, 8961, 8962 ], "line_end_idx": [ 28, 29, 642, 643, 934, 935, 961, 987, 988, 1286, 1287, 1317, 1347, 1348, 1886, 1887, 1893, 1899, 1900, 1987, 1988, 2027, 2066, 2067, 2459, 2460, 2492, 2493, 2742, 2743, 3228, 3229, 3374, 3375, 3426, 3427, 3465, 3503, 3504, 3702, 3703, 3741, 3779, 3780, 3827, 3828, 3875, 3922, 3923, 4091, 4092, 4112, 4132, 4133, 4245, 4246, 4317, 4388, 4389, 4444, 4445, 4484, 4523, 4524, 4887, 4888, 4926, 4927, 5306, 5307, 5776, 5777, 5786, 5787, 5796, 5797, 5824, 5825, 5896, 5897, 5961, 5962, 6008, 6009, 6116, 6117, 6602, 6603, 6656, 6709, 6710, 6754, 6755, 6780, 6805, 6806, 7006, 7007, 7053, 7099, 7100, 7168, 7169, 7207, 7245, 7246, 7271, 7272, 7469, 7749, 8205, 8262, 8327, 8630, 8691, 8753, 8813, 8861, 8922, 8961, 8962, 9182 ] }
{ "red_pajama_v2": { "ccnet_original_length": 9182, "ccnet_original_nlines": 121, "rps_doc_curly_bracket": 0, "rps_doc_ldnoobw_words": 0, "rps_doc_lorem_ipsum": 0, "rps_doc_stop_word_fraction": 0.3785220682621002, "rps_doc_ut1_blacklist": 0, "rps_doc_frac_all_caps_words": 0.03296118974685669, "rps_doc_frac_lines_end_with_ellipsis": 0, "rps_doc_frac_no_alph_words": 0.16586922109127045, "rps_doc_frac_unique_words": 0.35512256622314453, "rps_doc_mean_word_length": 4.536769390106201, "rps_doc_num_sentences": 109, "rps_doc_symbol_to_word_ratio": 0.0005316300084814429, "rps_doc_unigram_entropy": 5.536401271820068, "rps_doc_word_count": 1591, "rps_doc_frac_chars_dupe_10grams": 0.016070930287241936, "rps_doc_frac_chars_dupe_5grams": 0.12344139814376831, "rps_doc_frac_chars_dupe_6grams": 0.09573289006948471, "rps_doc_frac_chars_dupe_7grams": 0.06566915661096573, "rps_doc_frac_chars_dupe_8grams": 0.03491272032260895, "rps_doc_frac_chars_dupe_9grams": 0.016070930287241936, "rps_doc_frac_chars_top_2gram": 0.00761984009295702, "rps_doc_frac_chars_top_3gram": 0.013577169738709927, "rps_doc_frac_chars_top_4gram": 0.012191739864647388, "rps_doc_books_importance": -907.275390625, "rps_doc_books_importance_length_correction": -907.275390625, "rps_doc_openwebtext_importance": -459.98675537109375, "rps_doc_openwebtext_importance_length_correction": -459.98675537109375, "rps_doc_wikipedia_importance": -381.1850891113281, "rps_doc_wikipedia_importance_length_correction": -381.1850891113281 }, "fasttext": { "dclm": 0.04438120126724243, "english": 0.956480860710144, "fineweb_edu_approx": 1.1092697381973267, "eai_general_math": 0.029945969581604004, "eai_open_web_math": 0.06248855963349342, "eai_web_code": 0.0027522998861968517 } }
{ "free_decimal_correspondence": { "primary": { "code": "004.0285", "labels": { "level_1": "General works, books and libraries, information sciences", "level_2": "", "level_3": "Computers and Computer science" } }, "secondary": { "code": "621.384", "labels": { "level_1": "Industrial arts, Technology, and Engineering", "level_2": "Engineering", "level_3": "Mechanical engineering and Machinery" } } }, "bloom_cognitive_process": { "primary": { "code": "3", "label": "Apply" }, "secondary": { "code": "2", "label": "Understand" } }, "bloom_knowledge_domain": { "primary": { "code": "3", "label": "Procedural" }, "secondary": { "code": "2", "label": "Conceptual" } }, "document_type_v1": { "primary": { "code": "9", "label": "Personal/Misc" }, "secondary": { "code": "-1", "label": "Abstain" } }, "extraction_artifacts": { "primary": { "code": "0", "label": "No Artifacts" }, "secondary": { "code": "3", "label": "Irrelevant Content" } }, "missing_content": { "primary": { "code": "4", "label": "Missing Images or Figures" }, "secondary": { "code": "-1", "label": "Abstain" } }, "document_type_v2": { "primary": { "code": "16", "label": "Personal Blog" }, "secondary": { "code": "8", "label": "Documentation" } }, "reasoning_depth": { "primary": { "code": "2", "label": "Basic Reasoning" }, "secondary": { "code": "3", "label": "Intermediate Reasoning" } }, "technical_correctness": { "primary": { "code": "3", "label": "Mostly Correct" }, "secondary": { "code": "4", "label": "Highly Correct" } }, "education_level": { "primary": { "code": "2", "label": "High School Level" }, "secondary": { "code": "1", "label": "General Audience" } } }
672f1e42c33a7f9846924a2431ea77df
-8,850,672,658,307,157,000
Stack Overflow is a community of 4.7 million programmers, just like you, helping each other. Join them; it only takes a minute: Sign up Join the Stack Overflow community to: 1. Ask programming questions 2. Answer and help your peers 3. Get recognized for your expertise I have problem with my gem and Ruby command, I think it's because of path problem. gem install jekyll takes for a long time to run, and outputs an error message: ERROR: Could not find a valid gem 'jekyll' (>= 0) Unable to download data from https://rubygems.org/ - too many connection resets(https://s3.amazonaws.com/production.s3.rubygems.org/latest_specs.4.8.gz) I entered which ruby, which output: /usr/local/bin/ruby If I enter which gem, it prints out: gem () { typeset result ( typeset rvmrc rvm_rvmrc_files=("/etc/rvmrc" "$HOME/.rvmrc") if [[ -n "${rvm_prefix:-}" ]] && ! [[ "$HOME/.rvmrc" -ef "${rvm_prefix}/.rvmrc" ]] then rvm_rvmrc_files+=("${rvm_prefix}/.rvmrc") fi for rvmrc in "${rvm_rvmrc_files[@]}" do [[ -s "${rvmrc}" ]] && source "${rvmrc}" || true done unset rvm_rvmrc_files command gem "$@" ) || result=$? hash -r return ${result:-0} } Enter whereis gem, prints out: /usr/bin/gem Does anyone know how to fix it? I've tried for a long time and get confused. UPDATE: Enter gem env I got: RubyGems Environment: - RUBYGEMS VERSION: 2.0.3 - RUBY VERSION: 2.0.0 (2013-06-27 patchlevel 247) [x86_64-darwin13.0.0] - INSTALLATION DIRECTORY: /Users/lijung/.rvm/gems/ruby-1.9.3-p374 - RUBY EXECUTABLE: /usr/local/Cellar/ruby/2.0.0-p247/bin/ruby - EXECUTABLE DIRECTORY: /Users/lijung/.rvm/gems/ruby-1.9.3-p374/bin - RUBYGEMS PLATFORMS: - ruby - x86_64-darwin-13 - GEM PATHS: - /Users/lijung/.rvm/gems/ruby-1.9.3-p374 - /usr/local/bin/gem - GEM CONFIGURATION: - :update_sources => true - :verbose => true - :backtrace => false - :bulk_threshold => 1000 - REMOTE SOURCES: - https://rubygems.org/ enter rvm info: ruby-1.9.3-p374: system: uname: "Darwin chi.local 13.0.0 Darwin Kernel Version 13.0.0: Thu Sep 19 22:22:27 PDT 2013; root:xnu-2422.1.72~6/RELEASE_X86_64 x86_64" system: "osx/10/x86_64" bash: "/usr/local/bin/bash => GNU bash, version 4.2.37(2)-release (i386-apple-darwin12.2.0)" zsh: "/bin/zsh => zsh 5.0.2 (x86_64-apple-darwin13.0)" rvm: version: "rvm 1.18.3 (master) by Wayne E. Seguin <[email protected]>, Michal Papis <[email protected]> [https://rvm.io/]" updated: "9 months 18 days 11 hours 37 minutes 54 seconds ago" ruby: interpreter: "ruby" version: "2.0.0p247" date: "2013-06-27" platform: "x86_64-darwin13.0.0" patchlevel: "2013-06-27 revision 41674" full_version: "ruby 2.0.0p247 (2013-06-27 revision 41674) [x86_64-darwin13.0.0]" homes: gem: "/Users/lijung/.rvm/gems/ruby-1.9.3-p374" ruby: "/Users/lijung/.rvm/rubies/ruby-1.9.3-p374" binaries: ruby: "/usr/local/bin/ruby" irb: "/usr/local/bin/irb" gem: "/usr/local/bin/gem" rake: "/usr/local/bin/rake" environment: PATH: "/usr/local/bin:/usr/local/sbin:/Users/lijung/.rvm/gems/ruby-1.9.3-p374/bin:/Users/lijung/.rvm/gems/ruby-1.9.3-p374@global/bin:/Users/lijung/.rvm/rubies/ruby-1.9.3-p374/bin:/Users/lijung/.rvm/bin:/opt/local/bin:/opt/local/sbin:/usr/bin:/bin:/usr/sbin:/sbin:/opt/X11/bin:/Applications/Postgres.app/Contents/MacOS/bin" GEM_HOME: "/Users/lijung/.rvm/gems/ruby-1.9.3-p374" GEM_PATH: "/usr/local/bin/gem" MY_RUBY_HOME: "/Users/lijung/.rvm/rubies/ruby-1.9.3-p374" IRBRC: "/Users/lijung/.rvm/rubies/ruby-1.9.3-p374/.irbrc" RUBYOPT: "" gemset: "" share|improve this question      First, you need to show what gem knows about its world: Append the output of gem env to your questions. The error about connection resets means your internet connection to the rubygems server wasn't good. Usually that's a spurious problem that fixes itself (after some internet elf adjusts something). – the Tin Man Nov 20 '13 at 15:42      @theTinMan I've just update the message of gem env, I think the path is a huge mess. – LiJung Nov 20 '13 at 15:43      Your version of Rubygems is old. Run gem update --system to fix that. It looks like your Ruby environment is very confused too; I see inconsistencies where it thinks your Ruby executable is vs where RVM would have put it. How did you install your Rubies, because it's NOT what RVM would do. – the Tin Man Nov 20 '13 at 15:45 1   Append the output of rvm info please. – the Tin Man Nov 20 '13 at 15:46 1   You've crossed the beams, and this is what happens. You can't use RVM with other Ruby managers; Or, better said, you can, but YOU get to manage the mess that ensues. – the Tin Man Nov 20 '13 at 15:48 up vote 2 down vote accepted The first thing I see that is a "really bad thing" when using RVM is: PATH: "/usr/local/bin:/usr/local/sbin:/Users/lijung/.rvm/gems/ruby-1.9.3-p374/bin:/Users/lijung/.rvm/gems/ruby-1.9.3-p374@global/bin:/Users/lijung/.rvm/rubies/ruby-1.9.3-p374/bin:/Users/lijung/.rvm/bin:/opt/local/bin:/opt/local/sbin:/usr/bin:/bin:/usr/sbin:/sbin:/opt/X11/bin:/Applications/Postgres.app/Contents/MacOS/bin" Even with how Stack Overflow formats this you can see that the RVM initialization isn't correct. RVM must be the first thing in your path. Look in your ~/.bash_profile or ~/.bashrc file and adjust the PATH manipulations so it comes first, then close the terminal window and reopen it. Checking the PATH should look something like: "/Users/lijung/.rvm/gems/ruby-1.9.3-p374/bin:/Users/lijung/.rvm/gems/ruby-1.9.3-p374@global/bin:/Users/lijung/.rvm/rubies/ruby-1.9.3-p374/bin:/Users/lijung/.rvm/bin:/usr/local/bin:/usr/local/sbin:/opt/local/bin:/opt/local/sbin:/usr/bin:/bin:/usr/sbin:/sbin:/opt/X11/bin:/Applications/Postgres.app/Contents/MacOS/bin" The path is the chain of directories used by your shell when looking for a command. Your current path is telling the shell to find commands in /usr/local/bin and /usr/local/sbin first, which means it will find all Ruby-related commands in that directory first, which are NOT the ones that RVM installed for you. Fixing the RVM initializer so it runs at the right point in your ~/.bash_profile or ~/.bashrc is the first step. Your RVM is out of date too. Every couple weeks you should run rvm get stable to make sure you've got the latest settings and bug-fixes. Once your path is straightened out you can run that. The error: Unable to download data from https://rubygems.org/ - too many connection resets... is indicative of internet problems between you and your server. Usually that will be fixed on its own. If it doesn't heal itself in a reasonable time, try restarting your machine, which will give the IP stack a chance to restart and flush all its caches; Though that code is well tested, there are still times it can lose it's mind and do dumb things. The reset is a poke in its eye which gets its attention. If THAT doesn't fix it consider calling your ISP. I be able to use Ruby, that I've downloaded from brew? How can I set my gem path to the right path? Deal with things in the right order. You can't use your Ruby as it's currently configured because your gems are not correctly associated with the right Ruby version and any changes you make to the Ruby installations will only result in confusion and mayhem. Once you have the path corrected and RVM running correctly, then it's a very simple RVM command to enable other Homebrew Rubies: rvm use system --default At that point, RVM will step out of the way and your other Rubies will be accessible via the PATH chain. You have to manage which Ruby version will be executed at that point. Avoiding that hassle is why we use RVM or rbenv; They can easily install multiple Ruby versions and let you switch between them with a simple command, or even automatically when you chdir into a directory, if you set them up to do so. Brew only muddies the water, and trying to use a Brew installed Ruby was the source of your problem, so I'd recommend seriously rethinking that decision. Finally, because you don't seem to understand what you're doing, and you're on Mac OS X, I'd recommend fixing this path problem, then backing away from the computer, READ ALL THE RVM PAGES until you REALLY understand what it does, then carefully make changes. Don't trust what other sites say about installing RVM, trust what the RVM authors say first. The authors know what works best for using it. You might want to look into using JewelryBox also. share|improve this answer      Thanks a lot @theTimMan this really helps me a lot. Am I able to use Ruby, that I've downloaded from brew? How can I set my gem path to the right path? – LiJung Nov 20 '13 at 16:12      @theTimMan I've set the path export PATH=/usr/local/bin:/usr/local/sbin:/Users/lijung/.rvm/gems/ruby-1.9.3-p374/bin:‌​/Users/lijung/.rvm/gems/ruby-1.9.3-p374@global/bin:/Users/lijung/.r vm/rubies/ruby-1.9.3-p374/bin:/Users/lijung/.rvm/bin:/opt/local/bin:/opt/local/s‌​bin:/usr/bin:/bin:/usr/sbin:/sbin:/opt/X11/bin:/Applications/Postgres.app/ Contents/MacOS/bin to ~/.zshrc but I think it still run Ruby that install from brew. – LiJung Nov 20 '13 at 17:07      cause I've got ruby 2.0.0p247 (2013-06-27 revision 41674) [x86_64-darwin13.0.0] after I enter ruby -v – LiJung Nov 20 '13 at 17:08 1   "I've set the path export PATH=/usr/local/bin:/usr/local/sbin:/Users/lijung/.rvm/gems/ruby-1.9.3-p374/bin‌​". NO. Read what I said in the answer. You CAN NOT HAVE THE BREW RUBY accessed first. – the Tin Man Nov 20 '13 at 17:35 Your Answer   discard By posting your answer, you agree to the privacy policy and terms of service. Not the answer you're looking for? Browse other questions tagged or ask your own question.
{ "url": "http://stackoverflow.com/questions/20100099/ruby-gem-running-forever-and-ruby-gem-path-error", "source_domain": "stackoverflow.com", "snapshot_id": "crawl=CC-MAIN-2016-07", "warc_metadata": { "Content-Length": "92635", "Content-Type": "application/http; msgtype=response", "WARC-Block-Digest": "sha1:MJROOSJPHKA2AFJFNCXQK7JXM3K5ZD5P", "WARC-Concurrent-To": "<urn:uuid:e3852de9-f443-4154-aa21-5f4c4e32426e>", "WARC-Date": "2016-02-07T08:54:32Z", "WARC-IP-Address": "104.16.36.249", "WARC-Identified-Payload-Type": null, "WARC-Payload-Digest": "sha1:GV4JYEBE5TGGMUC6B4JWMD5X3IC2WSIG", "WARC-Record-ID": "<urn:uuid:0ff24441-9522-4b4c-9f91-9b0a5202fbce>", "WARC-Target-URI": "http://stackoverflow.com/questions/20100099/ruby-gem-running-forever-and-ruby-gem-path-error", "WARC-Truncated": null, "WARC-Type": "response", "WARC-Warcinfo-ID": "<urn:uuid:8b92b2ce-10be-46d8-a804-d4f22fb41a6a>" }, "warc_info": "robots: classic\r\nhostname: ip-10-236-182-209.ec2.internal\r\nsoftware: Nutch 1.6 (CC)/CC WarcExport 1.0\r\nisPartOf: CC-MAIN-2016-07\r\noperator: CommonCrawl Admin\r\ndescription: Wide crawl of the web for February 2016\r\npublisher: CommonCrawl\r\nformat: WARC File Format 1.0\r\nconformsTo: http://bibnum.bnf.fr/WARC/WARC_ISO_28500_version1_latestdraft.pdf" }
{ "line_start_idx": [ 0, 93, 94, 129, 130, 138, 176, 207, 239, 278, 279, 362, 363, 382, 383, 443, 444, 495, 496, 649, 650, 686, 687, 707, 708, 745, 746, 755, 772, 776, 794, 845, 932, 941, 990, 997, 1038, 1045, 1100, 1109, 1135, 1156, 1174, 1184, 1206, 1208, 1209, 1240, 1241, 1254, 1255, 1332, 1333, 1334, 1342, 1343, 1364, 1365, 1387, 1417, 1493, 1563, 1629, 1701, 1727, 1740, 1765, 1782, 1831, 1859, 1884, 1917, 1943, 1972, 2005, 2027, 2058, 2059, 2075, 2076, 2093, 2094, 2104, 2250, 2283, 2387, 2454, 2455, 2462, 2597, 2669, 2670, 2678, 2703, 2733, 2764, 2804, 2850, 2935, 2936, 2945, 3005, 3067, 3068, 3080, 3120, 3159, 3198, 3238, 3239, 3254, 3589, 3649, 3688, 3750, 3819, 3840, 3861, 3889, 3894, 4230, 4235, 4349, 4354, 4679, 4683, 4755, 4759, 4959, 4988, 4989, 5059, 5060, 5383, 5384, 5715, 5716, 6033, 6034, 6346, 6347, 6460, 6461, 6651, 6652, 6663, 6664, 6747, 6748, 7207, 7208, 7209, 7309, 7310, 7697, 7698, 7723, 7724, 7899, 7900, 8289, 8290, 8690, 8691, 8742, 8743, 8769, 8774, 8955, 8960, 9417, 9422, 9553, 9557, 9784, 9785, 9797, 9798, 9800, 9808, 9809, 9887, 9888 ], "line_end_idx": [ 93, 94, 129, 130, 138, 176, 207, 239, 278, 279, 362, 363, 382, 383, 443, 444, 495, 496, 649, 650, 686, 687, 707, 708, 745, 746, 755, 772, 776, 794, 845, 932, 941, 990, 997, 1038, 1045, 1100, 1109, 1135, 1156, 1174, 1184, 1206, 1208, 1209, 1240, 1241, 1254, 1255, 1332, 1333, 1334, 1342, 1343, 1364, 1365, 1387, 1417, 1493, 1563, 1629, 1701, 1727, 1740, 1765, 1782, 1831, 1859, 1884, 1917, 1943, 1972, 2005, 2027, 2058, 2059, 2075, 2076, 2093, 2094, 2104, 2250, 2283, 2387, 2454, 2455, 2462, 2597, 2669, 2670, 2678, 2703, 2733, 2764, 2804, 2850, 2935, 2936, 2945, 3005, 3067, 3068, 3080, 3120, 3159, 3198, 3238, 3239, 3254, 3589, 3649, 3688, 3750, 3819, 3840, 3861, 3889, 3894, 4230, 4235, 4349, 4354, 4679, 4683, 4755, 4759, 4959, 4988, 4989, 5059, 5060, 5383, 5384, 5715, 5716, 6033, 6034, 6346, 6347, 6460, 6461, 6651, 6652, 6663, 6664, 6747, 6748, 7207, 7208, 7209, 7309, 7310, 7697, 7698, 7723, 7724, 7899, 7900, 8289, 8290, 8690, 8691, 8742, 8743, 8769, 8774, 8955, 8960, 9417, 9422, 9553, 9557, 9784, 9785, 9797, 9798, 9800, 9808, 9809, 9887, 9888, 9978 ] }
{ "red_pajama_v2": { "ccnet_original_length": 9978, "ccnet_original_nlines": 182, "rps_doc_curly_bracket": 0.0016035300213843584, "rps_doc_ldnoobw_words": 0, "rps_doc_lorem_ipsum": 0, "rps_doc_stop_word_fraction": 0.20850901305675507, "rps_doc_ut1_blacklist": 0, "rps_doc_frac_all_caps_words": 0.039095439016819, "rps_doc_frac_lines_end_with_ellipsis": 0.005464479792863131, "rps_doc_frac_no_alph_words": 0.4039861857891083, "rps_doc_frac_unique_words": 0.3982371687889099, "rps_doc_mean_word_length": 5.688301086425781, "rps_doc_num_sentences": 193, "rps_doc_symbol_to_word_ratio": 0.00038328999653458595, "rps_doc_unigram_entropy": 5.622198104858398, "rps_doc_word_count": 1248, "rps_doc_frac_chars_dupe_10grams": 0.0200028195977211, "rps_doc_frac_chars_dupe_5grams": 0.08409634977579117, "rps_doc_frac_chars_dupe_6grams": 0.07170023769140244, "rps_doc_frac_chars_dupe_7grams": 0.04944358021020889, "rps_doc_frac_chars_dupe_8grams": 0.04944358021020889, "rps_doc_frac_chars_dupe_9grams": 0.036061421036720276, "rps_doc_frac_chars_top_2gram": 0.0063389199785888195, "rps_doc_frac_chars_top_3gram": 0.008874489925801754, "rps_doc_frac_chars_top_4gram": 0.011410060338675976, "rps_doc_books_importance": -1104.6068115234375, "rps_doc_books_importance_length_correction": -1104.6068115234375, "rps_doc_openwebtext_importance": -756.8065185546875, "rps_doc_openwebtext_importance_length_correction": -756.8065185546875, "rps_doc_wikipedia_importance": -564.1244506835938, "rps_doc_wikipedia_importance_length_correction": -564.1244506835938 }, "fasttext": { "dclm": 0.14489436149597168, "english": 0.8458560109138489, "fineweb_edu_approx": 1.476760745048523, "eai_general_math": 0.026899220421910286, "eai_open_web_math": 0.1973291039466858, "eai_web_code": 0.027371469885110855 } }
{ "free_decimal_correspondence": { "primary": { "code": "005.133", "labels": { "level_1": "General works, books and libraries, information sciences", "level_2": "", "level_3": "Computer programming" } }, "secondary": { "code": "004.02", "labels": { "level_1": "General works, books and libraries, information sciences", "level_2": "", "level_3": "Computers and Computer science" } } }, "bloom_cognitive_process": { "primary": { "code": "3", "label": "Apply" }, "secondary": { "code": "2", "label": "Understand" } }, "bloom_knowledge_domain": { "primary": { "code": "3", "label": "Procedural" }, "secondary": { "code": "2", "label": "Conceptual" } }, "document_type_v1": { "primary": { "code": "5", "label": "Social/Forum" }, "secondary": { "code": "-1", "label": "Abstain" } }, "extraction_artifacts": { "primary": { "code": "3", "label": "Irrelevant Content" }, "secondary": { "code": "-1", "label": "Abstain" } }, "missing_content": { "primary": { "code": "0", "label": "No missing content" }, "secondary": { "code": "-1", "label": "Abstain" } }, "document_type_v2": { "primary": { "code": "18", "label": "Q&A Forum" }, "secondary": { "code": "21", "label": "Customer Support" } }, "reasoning_depth": { "primary": { "code": "3", "label": "Intermediate Reasoning" }, "secondary": { "code": "2", "label": "Basic Reasoning" } }, "technical_correctness": { "primary": { "code": "4", "label": "Highly Correct" }, "secondary": { "code": "3", "label": "Mostly Correct" } }, "education_level": { "primary": { "code": "3", "label": "Undergraduate Level" }, "secondary": { "code": "2", "label": "High School Level" } } }
672f1e42c33a7f9846924a2431ea77df
-2,912,222,443,054,568,000
Mock Signature generateKeyPair const {accInfo} = await LoopringAPI.exchangeAPI.getAccount({ owner: LOOPRING_EXPORTED_ACCOUNT.address, }); const result = await signatureKeyPairMock(accInfo); console.log(result.sk); getEcDSASig: eth_signTypedData_v4 // test case is not allow brock by Mock provider const result = await sdk.getEcDSASig( web3, testTypedData, LOOPRING_EXPORTED_ACCOUNT.address, sdk.GetEcDSASigType.HasDataStruct, sdk.ChainId.GOERLI, LOOPRING_EXPORTED_ACCOUNT.accountId, "", sdk.ConnectorNames.Unknown ); console.log("getEcDSASig:eth_signTypedData_v4", result, "ecdsaSig+sdk.SigSuffix.Suffix02", result.ecdsaSig + sdk.SigSuffix.Suffix02 ); getEcDSASig: personalSign(WithoutDataStruct--Hardware wallet) const result = await sdk.getEcDSASig( web3, testTypedData, LOOPRING_EXPORTED_ACCOUNT.address, sdk.GetEcDSASigType.WithoutDataStruct, sdk.ChainId.GOERLI, LOOPRING_EXPORTED_ACCOUNT.accountId, "", sdk.ConnectorNames.Unknown ); console.log( "getEcDSASig:WithoutDataStruct(personalSign)", result, "ecdsaSig+sdk.SigSuffix.Suffix03", result.ecdsaSig + sdk.SigSuffix.Suffix03 ); getEcDSASig: personalSign(Contract) // test case is not allow brock by Mock provider const result = await sdk.getEcDSASig( web3, testTypedData, LOOPRING_EXPORTED_ACCOUNT.address, sdk.GetEcDSASigType.Contract, sdk.ChainId.GOERLI, LOOPRING_EXPORTED_ACCOUNT.accountId, "", sdk.ConnectorNames.Unknown ); console.log( "getEcDSASig:personalSign(Contract)", result ); Last updated
{ "url": "https://docs-protocol.loopring.io/resources/signature/sdk-signature/mock-signature", "source_domain": "docs-protocol.loopring.io", "snapshot_id": "CC-MAIN-2024-18", "warc_metadata": { "Content-Length": "455711", "Content-Type": "application/http; msgtype=response", "WARC-Block-Digest": "sha1:D5LVSKLK4PAB7NV75KDUWD2RLPIP6EHW", "WARC-Concurrent-To": "<urn:uuid:142320f7-3212-4f84-b342-06153847a311>", "WARC-Date": "2024-04-23T08:51:01Z", "WARC-IP-Address": "104.18.40.47", "WARC-Identified-Payload-Type": "text/html", "WARC-Payload-Digest": "sha1:EKXBOGRE2FFSZU7SB6IRDYDTFSSL5H6L", "WARC-Record-ID": "<urn:uuid:24005bd9-e686-4906-b2ce-984e91d8f9dc>", "WARC-Target-URI": "https://docs-protocol.loopring.io/resources/signature/sdk-signature/mock-signature", "WARC-Truncated": null, "WARC-Type": "response", "WARC-Warcinfo-ID": "<urn:uuid:d82e1180-8352-4821-a5ff-60fa369505a6>" }, "warc_info": "isPartOf: CC-MAIN-2024-18\r\npublisher: Common Crawl\r\ndescription: Wide crawl of the web for April 2024\r\noperator: Common Crawl Admin ([email protected])\r\nhostname: ip-10-67-67-180\r\nsoftware: Apache Nutch 1.19 (modified, https://github.com/commoncrawl/nutch/)\r\nrobots: checked via crawler-commons 1.5-SNAPSHOT (https://github.com/crawler-commons/crawler-commons)\r\nformat: WARC File Format 1.1\r\nconformsTo: https://iipc.github.io/warc-specifications/specifications/warc-format/warc-1.1/" }
{ "line_start_idx": [ 0, 15, 16, 32, 33, 94, 138, 142, 194, 218, 219, 253, 254, 303, 341, 349, 366, 403, 440, 462, 501, 507, 536, 539, 587, 597, 634, 677, 680, 681, 743, 744, 782, 790, 807, 844, 885, 907, 946, 952, 981, 984, 997, 1046, 1056, 1093, 1136, 1139, 1140, 1176, 1177, 1226, 1264, 1272, 1289, 1326, 1358, 1380, 1419, 1425, 1454, 1457, 1470, 1510, 1519, 1522, 1523 ], "line_end_idx": [ 15, 16, 32, 33, 94, 138, 142, 194, 218, 219, 253, 254, 303, 341, 349, 366, 403, 440, 462, 501, 507, 536, 539, 587, 597, 634, 677, 680, 681, 743, 744, 782, 790, 807, 844, 885, 907, 946, 952, 981, 984, 997, 1046, 1056, 1093, 1136, 1139, 1140, 1176, 1177, 1226, 1264, 1272, 1289, 1326, 1358, 1380, 1419, 1425, 1454, 1457, 1470, 1510, 1519, 1522, 1523, 1535 ] }
{ "red_pajama_v2": { "ccnet_original_length": 1535, "ccnet_original_nlines": 66, "rps_doc_curly_bracket": 0.002605859888717532, "rps_doc_ldnoobw_words": 0, "rps_doc_lorem_ipsum": 0, "rps_doc_stop_word_fraction": 0.029739780351519585, "rps_doc_ut1_blacklist": 0, "rps_doc_frac_all_caps_words": 0.03717472031712532, "rps_doc_frac_lines_end_with_ellipsis": 0, "rps_doc_frac_no_alph_words": 0.45724907517433167, "rps_doc_frac_unique_words": 0.5113636255264282, "rps_doc_mean_word_length": 13.443181991577148, "rps_doc_num_sentences": 46, "rps_doc_symbol_to_word_ratio": 0, "rps_doc_unigram_entropy": 3.6194894313812256, "rps_doc_word_count": 88, "rps_doc_frac_chars_dupe_10grams": 0.1927303522825241, "rps_doc_frac_chars_dupe_5grams": 0.2578190863132477, "rps_doc_frac_chars_dupe_6grams": 0.2578190863132477, "rps_doc_frac_chars_dupe_7grams": 0.2578190863132477, "rps_doc_frac_chars_dupe_8grams": 0.1927303522825241, "rps_doc_frac_chars_dupe_9grams": 0.1927303522825241, "rps_doc_frac_chars_top_2gram": 0.03719358146190643, "rps_doc_frac_chars_top_3gram": 0.05409974977374077, "rps_doc_frac_chars_top_4gram": 0.07607776671648026, "rps_doc_books_importance": -93.28372955322266, "rps_doc_books_importance_length_correction": -81.9677734375, "rps_doc_openwebtext_importance": -83.08937072753906, "rps_doc_openwebtext_importance_length_correction": -83.08937072753906, "rps_doc_wikipedia_importance": -71.21663665771484, "rps_doc_wikipedia_importance_length_correction": -57.986995697021484 }, "fasttext": { "dclm": 0.18637555837631226, "english": 0.3955950438976288, "fineweb_edu_approx": 1.6400970220565796, "eai_general_math": 0.27210307121276855, "eai_open_web_math": 0.018461940810084343, "eai_web_code": 0.3563689589500427 } }
{ "free_decimal_correspondence": { "primary": { "code": "005.1", "labels": { "level_1": "General works, books and libraries, information sciences", "level_2": "", "level_3": "Computer programming" } }, "secondary": { "code": "332.872", "labels": { "level_1": "Social sciences", "level_2": "Economics", "level_3": "Finance" } } }, "bloom_cognitive_process": { "primary": { "code": "3", "label": "Apply" }, "secondary": { "code": "2", "label": "Understand" } }, "bloom_knowledge_domain": { "primary": { "code": "3", "label": "Procedural" }, "secondary": { "code": "2", "label": "Conceptual" } }, "document_type_v1": { "primary": { "code": "4", "label": "Code/Software" }, "secondary": { "code": "3", "label": "Reference/Encyclopedic/Educational" } }, "extraction_artifacts": { "primary": { "code": "0", "label": "No Artifacts" }, "secondary": { "code": "3", "label": "Irrelevant Content" } }, "missing_content": { "primary": { "code": "0", "label": "No missing content" }, "secondary": { "code": "-1", "label": "Abstain" } }, "document_type_v2": { "primary": { "code": "8", "label": "Documentation" }, "secondary": { "code": "23", "label": "Tutorial" } }, "reasoning_depth": { "primary": { "code": "2", "label": "Basic Reasoning" }, "secondary": { "code": "3", "label": "Intermediate Reasoning" } }, "technical_correctness": { "primary": { "code": "4", "label": "Highly Correct" }, "secondary": { "code": "3", "label": "Mostly Correct" } }, "education_level": { "primary": { "code": "4", "label": "Graduate/Expert Level" }, "secondary": { "code": "3", "label": "Undergraduate Level" } } }
672f1e42c33a7f9846924a2431ea77df
-8,661,276,561,971,650,000
Answers Solutions by everydaycalculation.com Everydaycalculation.com » Answers » Subtract fractions Subtract 10/42 from 2/15 2/15 - 10/42 is -11/105. Steps for subtracting fractions 1. Find the least common denominator or LCM of the two denominators: LCM of 15 and 42 is 210 2. For the 1st fraction, since 15 × 14 = 210, 2/15 = 2 × 14/15 × 14 = 28/210 3. Likewise, for the 2nd fraction, since 42 × 5 = 210, 10/42 = 10 × 5/42 × 5 = 50/210 4. Subtract the two fractions: 28/210 - 50/210 = 28 - 50/210 = -22/210 5. After reducing the fraction, the answer is -11/105 Related: Use fraction calculator with our all-in-one calculator app: Download for Android, Download for iOS © everydaycalculation.com
{ "url": "https://answers.everydaycalculation.com/subtract-fractions/2-15-minus-10-42", "source_domain": "answers.everydaycalculation.com", "snapshot_id": "crawl=CC-MAIN-2019-30", "warc_metadata": { "Content-Length": "8322", "Content-Type": "application/http; msgtype=response", "WARC-Block-Digest": "sha1:JYHKGHQOXIGQZVHGD5KXEO5QMZHUKZUA", "WARC-Concurrent-To": "<urn:uuid:7cfbd422-2799-4fb9-a0a5-1a129928a822>", "WARC-Date": "2019-07-19T04:39:23Z", "WARC-IP-Address": "96.126.107.130", "WARC-Identified-Payload-Type": "text/html", "WARC-Payload-Digest": "sha1:EGXV3EAE33ZOYVGE5ISN5PCYM6VSO7DB", "WARC-Record-ID": "<urn:uuid:6e27aaa3-e49d-4222-a58b-bf8cb6e8141e>", "WARC-Target-URI": "https://answers.everydaycalculation.com/subtract-fractions/2-15-minus-10-42", "WARC-Truncated": null, "WARC-Type": "response", "WARC-Warcinfo-ID": "<urn:uuid:c2c47f16-748f-4f41-bbe2-e58e637f4cdf>" }, "warc_info": "isPartOf: CC-MAIN-2019-30\r\npublisher: Common Crawl\r\ndescription: Wide crawl of the web for July 2019\r\noperator: Common Crawl Admin ([email protected])\r\nhostname: ip-10-232-173-83.ec2.internal\r\nsoftware: Apache Nutch 1.15 (modified, https://github.com/commoncrawl/nutch/)\r\nrobots: checked via crawler-commons 1.1-SNAPSHOT (https://github.com/crawler-commons/crawler-commons)\r\nformat: WARC File Format 1.1\r\nconformsTo: http://iipc.github.io/warc-specifications/specifications/warc-format/warc-1.1/" }
{ "line_start_idx": [ 0, 8, 9, 46, 47, 102, 103, 128, 129, 154, 155, 187, 188, 259, 287, 335, 370, 427, 462, 495, 539, 595, 596, 605, 606, 705, 706 ], "line_end_idx": [ 8, 9, 46, 47, 102, 103, 128, 129, 154, 155, 187, 188, 259, 287, 335, 370, 427, 462, 495, 539, 595, 596, 605, 606, 705, 706, 731 ] }
{ "red_pajama_v2": { "ccnet_original_length": 731, "ccnet_original_nlines": 26, "rps_doc_curly_bracket": 0, "rps_doc_ldnoobw_words": 0, "rps_doc_lorem_ipsum": 0, "rps_doc_stop_word_fraction": 0.1783783733844757, "rps_doc_ut1_blacklist": 0, "rps_doc_frac_all_caps_words": 0.010810810141265392, "rps_doc_frac_lines_end_with_ellipsis": 0, "rps_doc_frac_no_alph_words": 0.6054053902626038, "rps_doc_frac_unique_words": 0.5535714030265808, "rps_doc_mean_word_length": 4.660714149475098, "rps_doc_num_sentences": 10, "rps_doc_symbol_to_word_ratio": 0, "rps_doc_unigram_entropy": 3.9414010047912598, "rps_doc_word_count": 112, "rps_doc_frac_chars_dupe_10grams": 0, "rps_doc_frac_chars_dupe_5grams": 0, "rps_doc_frac_chars_dupe_6grams": 0, "rps_doc_frac_chars_dupe_7grams": 0, "rps_doc_frac_chars_dupe_8grams": 0, "rps_doc_frac_chars_dupe_9grams": 0, "rps_doc_frac_chars_top_2gram": 0.026819920167326927, "rps_doc_frac_chars_top_3gram": 0, "rps_doc_frac_chars_top_4gram": 0, "rps_doc_books_importance": -69.91941833496094, "rps_doc_books_importance_length_correction": -69.92440795898438, "rps_doc_openwebtext_importance": -32.54460525512695, "rps_doc_openwebtext_importance_length_correction": -32.54959487915039, "rps_doc_wikipedia_importance": -17.034914016723633, "rps_doc_wikipedia_importance_length_correction": -17.03990364074707 }, "fasttext": { "dclm": 0.8755266666412354, "english": 0.864651620388031, "fineweb_edu_approx": 2.7803595066070557, "eai_general_math": 0.33393359184265137, "eai_open_web_math": 0.4043734669685364, "eai_web_code": -0.000004529999841906829 } }
{ "free_decimal_correspondence": { "primary": { "code": "513.24", "labels": { "level_1": "Science and Natural history", "level_2": "Mathematics", "level_3": "Geometry" } }, "secondary": { "code": "510", "labels": { "level_1": "Science and Natural history", "level_2": "Mathematics", "level_3": "" } } }, "bloom_cognitive_process": { "primary": { "code": "3", "label": "Apply" }, "secondary": { "code": "2", "label": "Understand" } }, "bloom_knowledge_domain": { "primary": { "code": "3", "label": "Procedural" }, "secondary": { "code": "1", "label": "Factual" } }, "document_type_v1": { "primary": { "code": "3", "label": "Reference/Encyclopedic/Educational" }, "secondary": { "code": "-1", "label": "Abstain" } }, "extraction_artifacts": { "primary": { "code": "0", "label": "No Artifacts" }, "secondary": { "code": "3", "label": "Irrelevant Content" } }, "missing_content": { "primary": { "code": "0", "label": "No missing content" }, "secondary": { "code": "-1", "label": "Abstain" } }, "document_type_v2": { "primary": { "code": "23", "label": "Tutorial" }, "secondary": { "code": "21", "label": "Customer Support" } }, "reasoning_depth": { "primary": { "code": "2", "label": "Basic Reasoning" }, "secondary": { "code": "3", "label": "Intermediate Reasoning" } }, "technical_correctness": { "primary": { "code": "4", "label": "Highly Correct" }, "secondary": { "code": "3", "label": "Mostly Correct" } }, "education_level": { "primary": { "code": "2", "label": "High School Level" }, "secondary": { "code": "1", "label": "General Audience" } } }
672f1e42c33a7f9846924a2431ea77df
8,792,490,026,881,009,000
Stravinsky Conducts Stravinsky - The Original Jackets Collection. Firebird not recognized as work Firebird suite (L’Oiseau de Feu) not recognized as work. Happens on discs 2 and 4. Couldn’t solve it with tags You should be able to easily fix this by filling the WORK and PART tags in your files using your favourite tagging program. Then inside Roon go to „Edit“ „Metadata“ tab selecting „use File“ for Track and Multipart Composition. Did that and it works fine. I’ll keep reporting these kind of issues because I think it’s better if it gets fixed in the metadata database(s). Also manually fixing it means you have to use the correct title to make sure it shows up in the list of different recordings of the same work. Finding the correct title is not a trivial thing. 1 Like
{ "url": "https://community.roonlabs.com/t/stravinsky-conducts-stravinsky-the-original-jackets-collection-firebird-not-recognized-as-work/113949", "source_domain": "community.roonlabs.com", "snapshot_id": "crawl=CC-MAIN-2022-40", "warc_metadata": { "Content-Length": "25625", "Content-Type": "application/http; msgtype=response", "WARC-Block-Digest": "sha1:WEZQB3KICS63JTBGMQ3E4KUTDUKY3XCP", "WARC-Concurrent-To": "<urn:uuid:c546459c-889c-4aab-8f33-6a3d4ca3ff83>", "WARC-Date": "2022-09-25T21:01:41Z", "WARC-IP-Address": "167.172.230.239", "WARC-Identified-Payload-Type": "text/html", "WARC-Payload-Digest": "sha1:E4PCQQLQPI4GWQ4UBDJ2RWYLNQKSZXML", "WARC-Record-ID": "<urn:uuid:1e67576f-5f54-4a53-aee4-5dfa3f2029f0>", "WARC-Target-URI": "https://community.roonlabs.com/t/stravinsky-conducts-stravinsky-the-original-jackets-collection-firebird-not-recognized-as-work/113949", "WARC-Truncated": null, "WARC-Type": "response", "WARC-Warcinfo-ID": "<urn:uuid:36d62797-20e2-4a2b-8aaa-ad7f949f5ac8>" }, "warc_info": "isPartOf: CC-MAIN-2022-40\r\npublisher: Common Crawl\r\ndescription: Wide crawl of the web for September/October 2022\r\noperator: Common Crawl Admin ([email protected])\r\nhostname: ip-10-67-67-14\r\nsoftware: Apache Nutch 1.19 (modified, https://github.com/commoncrawl/nutch/)\r\nrobots: checked via crawler-commons 1.4-SNAPSHOT (https://github.com/crawler-commons/crawler-commons)\r\nformat: WARC File Format 1.1\r\nconformsTo: https://iipc.github.io/warc-specifications/specifications/warc-format/warc-1.1/" }
{ "line_start_idx": [ 0, 98, 99, 182, 183, 211, 212, 439, 440, 776, 777 ], "line_end_idx": [ 98, 99, 182, 183, 211, 212, 439, 440, 776, 777, 783 ] }
{ "red_pajama_v2": { "ccnet_original_length": 783, "ccnet_original_nlines": 10, "rps_doc_curly_bracket": 0, "rps_doc_ldnoobw_words": 0, "rps_doc_lorem_ipsum": 0, "rps_doc_stop_word_fraction": 0.39024388790130615, "rps_doc_ut1_blacklist": 0, "rps_doc_frac_all_caps_words": 0.030487800016999245, "rps_doc_frac_lines_end_with_ellipsis": 0, "rps_doc_frac_no_alph_words": 0.15853658318519592, "rps_doc_frac_unique_words": 0.75, "rps_doc_mean_word_length": 4.617647171020508, "rps_doc_num_sentences": 10, "rps_doc_symbol_to_word_ratio": 0, "rps_doc_unigram_entropy": 4.466564655303955, "rps_doc_word_count": 136, "rps_doc_frac_chars_dupe_10grams": 0, "rps_doc_frac_chars_dupe_5grams": 0, "rps_doc_frac_chars_dupe_6grams": 0, "rps_doc_frac_chars_dupe_7grams": 0, "rps_doc_frac_chars_dupe_8grams": 0, "rps_doc_frac_chars_dupe_9grams": 0, "rps_doc_frac_chars_top_2gram": 0.041401270776987076, "rps_doc_frac_chars_top_3gram": 0.047770701348781586, "rps_doc_frac_chars_top_4gram": 0.06050955131649971, "rps_doc_books_importance": -78.67758178710938, "rps_doc_books_importance_length_correction": -78.67758178710938, "rps_doc_openwebtext_importance": -48.98242950439453, "rps_doc_openwebtext_importance_length_correction": -48.98240661621094, "rps_doc_wikipedia_importance": -43.5433464050293, "rps_doc_wikipedia_importance_length_correction": -43.5433464050293 }, "fasttext": { "dclm": 0.1345551609992981, "english": 0.8720661401748657, "fineweb_edu_approx": 1.5498096942901611, "eai_general_math": 0.8384155631065369, "eai_open_web_math": 0.35523778200149536, "eai_web_code": 0.38609880208969116 } }
{ "free_decimal_correspondence": { "primary": { "code": "005.4", "labels": { "level_1": "General works, books and libraries, information sciences", "level_2": "", "level_3": "Computer programming" } }, "secondary": { "code": "780.194", "labels": { "level_1": "Arts", "level_2": "Music", "level_3": "" } } }, "bloom_cognitive_process": { "primary": { "code": "3", "label": "Apply" }, "secondary": { "code": "2", "label": "Understand" } }, "bloom_knowledge_domain": { "primary": { "code": "3", "label": "Procedural" }, "secondary": { "code": "2", "label": "Conceptual" } }, "document_type_v1": { "primary": { "code": "5", "label": "Social/Forum" }, "secondary": { "code": "3", "label": "Reference/Encyclopedic/Educational" } }, "extraction_artifacts": { "primary": { "code": "0", "label": "No Artifacts" }, "secondary": { "code": "3", "label": "Irrelevant Content" } }, "missing_content": { "primary": { "code": "0", "label": "No missing content" }, "secondary": { "code": "-1", "label": "Abstain" } }, "document_type_v2": { "primary": { "code": "18", "label": "Q&A Forum" }, "secondary": { "code": "21", "label": "Customer Support" } }, "reasoning_depth": { "primary": { "code": "2", "label": "Basic Reasoning" }, "secondary": { "code": "3", "label": "Intermediate Reasoning" } }, "technical_correctness": { "primary": { "code": "3", "label": "Mostly Correct" }, "secondary": { "code": "4", "label": "Highly Correct" } }, "education_level": { "primary": { "code": "1", "label": "General Audience" }, "secondary": { "code": "2", "label": "High School Level" } } }
672f1e42c33a7f9846924a2431ea77df
-8,826,998,093,904,369,000
all 18 comments [–]ehird 18 points19 points  (3 children) sorry, this has been archived and can no longer be voted on A promise with result a can simply be represented with the Haskell type IO a; you can then use a function like spawn to start the execution of a promise, and get an action to demand the result. With GHC's lightweight threads and non-blocking IO, this should be as efficient as you might hope for. [–]pjdelport 1 point2 points  (0 children) sorry, this has been archived and can no longer be voted on The equivalent of the promises is not IO (which is implicit in JavaScript), but Either. In the spawn library, it's: type Result a = Either SomeException a and is obtained using: spawnTry :: IO a -> IO (IO (Result a)) which is the equivalent of a function returning a promise. The Either value yielded by spawnTry is processed with either (or equivalent) in exactly the same way as JavaScript promises are processed with then() (or Deferreds are processed with addCallbacks()). The spawn function you linked to is actually a wrapper of spawnTry that handles an unsuccessful Either by re-throwing the provided error value as a Haskell IO exception instead: this is the equivalent of a JavaScript wrapper that handles an unsuccessful promise or Deferred by re-throwing the provided error value as a JavaScript exception instead. [–]bas_van_dijk 1 point2 points  (0 children) sorry, this has been archived and can no longer be voted on Shameless plug: Also take a look at the threads package which offers similar functions for all the different forks. [–]geezusfreeek 13 points14 points  (4 children) sorry, this has been archived and can no longer be voted on Just use threads. Fork a thread that eventually writes a result to a shared MVar and take from the MVar at the point that you need the result. [–]jerf 33 points34 points * (2 children) sorry, this has been archived and can no longer be voted on This, a thousand times this. The answer to "what's the Haskell equivalent of [crappy synchronization mechanism]?" is "Don't use crappy synchronization mechanisms." The worst thing that has happened to programming in the past three years is that just as the programming community was about to collectively discover that compilers can manage this stuff for you and give you really nice synchronization mechanisms that make sane programs that are easy to reason about (Erlang, Haskell, Go, etc.), Node.js came along and convinced everyone that event-loop-based programming was some sort of fantastic breakthrough, rather than a deployed-for-decades well-understood technology that was generally considered a bad idea, and then further convinced a lot of developers that anything not Node must not be any good, even as they are stuck using bad ideas from 30 years ago. I have noticed the JS community rapidly recapitulating the developments of other software communities. They're up to the late 90s with promises being the new hotness now. Pity their language choice will stop them before they actually reach sane choices. (Also, as ehird points out, it's basically so simple it's transparent in Haskell. I'm discussing the more general case here. The Haskell answer is that the compiler worries about non-blocking, rather than the programmer, and does a much better job at it... especially if we're talking Javascript which still can't handle using multiple cores yet.) [–]naasking 2 points3 points  (1 child) sorry, this has been archived and can no longer be voted on especially if we're talking Javascript which still can't handle using multiple cores yet. That's why promises/futures are a good fit for Javascript: all the asynchrony you want, without having to deal with the multithreading. Add some E-style promise pipelining, and the programming model can be somewhat pleasant. You can be as dismissive as you like, but the fact is that JS can't make the assumptions you can make in Haskell, namely that all references are immutable and thus thread-safe by default. Automated program transformations just aren't feasible. Node.js is actually a good start for a mutation-heavy language like JS. They aren't repeating the mistakes of the past, they're taking a different route entirely and avoiding the past mutation-heavy designs, and going share-nothing message-passing by default. Next they have to make spawning processes cheap and easy, and if they could wrap this all up in a simple API, then they have something truly compelling. [–]jerf 4 points5 points  (0 children) sorry, this has been archived and can no longer be voted on They aren't repeating the mistakes of the past, they're taking a different route entirely and avoiding the past mutation-heavy designs, and going share-nothing message-passing by default. As far as I know, that's the future plans for the standardization of the language and isn't here yet, or at least not for Node. In the meantime Node has simply recapitulated the ideas of other languages ad naseum, hailing each and every one as a breakthrough uniquely possessed by Node. And once you integrate the other plans for JS a lot of the utility of the Node design goes "poof". [–]simonmar 7 points8 points  (0 children) sorry, this has been archived and can no longer be voted on if you'll excuse a bit (more) self-promotion, I called this simple mechanism "Async" on page 37 of my tutorial [–]smog_alado 4 points5 points  (0 children) sorry, this has been archived and can no longer be voted on Don't forget that JS promises also usually have an exception handling component. Hopefully the enlightened Haskell hackers here won't forget to clear that up too ;) [–]Vulpyne 3 points4 points  (3 children) sorry, this has been archived and can no longer be voted on forkIO and a TMVar? Simple untested example: slowfuncresult <- atomically newEmptyTMVar forkIO $ slowFunction >>= (atomically . putTMVar slowfuncresult) -- do fun stuff until result is needed. atomically (takeTMVar slowfuncresult) >>= functionthatusesresult Of course you would probably want error handling and such, but that is the very general idea. [–]Axman6 2 points3 points  (1 child) sorry, this has been archived and can no longer be voted on This is not a situation where using STM makes much sense, unless you intend to be able to take multiple results in a single transaction… which doesn't make much sense anyway. A simple MVar will suffice. [–]Vulpyne 0 points1 point  (0 children) sorry, this has been archived and can no longer be voted on Good point. I guess I'm just in the habit of using STM typically. As you said, a simple MVar would be more appropriate. [–]pjdelport 2 points3 points * (0 children) sorry, this has been archived and can no longer be voted on Note that if slowFunction is simply a slow function, and not an IO action, you can run it in a separate thread much more easily using Control.Parallel.Strategies: functionthatusesresult $|| rseq $ slowFunction (replacing rseq with a more type-specific evaluation strategy, if needed) Of course you would probably want error handling and such, Right, this is the main point of promises. In Haskell, the equivalent would be for slowFunction to result in Either; functionthatusesresult would then be replaced with either onError onResult (where onError and onResult are handlers as you would pass to then()). [–]pjdelport 3 points4 points * (4 children) sorry, this has been archived and can no longer be voted on In Haskell, this is just the Either data type, with Right and Left representing successful and unsuccessful results. Addendum: While there's some discussion about I/O and threads here, note that these things are orthogonal to promises / Deferreds / Either, and they should not be confused with each other. The latter are just one way to construct computations involving results of the former, and there are plenty of ways to use either without the other. [–]geezusfreeek 0 points1 point  (3 children) sorry, this has been archived and can no longer be voted on Huh? Did you perhaps intend to reply directly to another comment here? [–]pjdelport 0 points1 point  (2 children) sorry, this has been archived and can no longer be voted on No? [–]geezusfreeek 1 point2 points  (1 child) sorry, this has been archived and can no longer be voted on I just don't see what promises have to do with Either then. [–]pjdelport 1 point2 points * (0 children) sorry, this has been archived and can no longer be voted on They're exactly the same thing. then() is either; resolve and reject are Right and Left. All this talk about I/O and threads is entirely missing the point: Promises and Deferreds do not specifically involve I/O, or threading, or anything else. They are just a pure-JavaScript way to structure two-path computations in exactly the same way as Either in Haskell. If you have some plain JavaScript code that returns or results in a promise or Deferred of some error and result type, the Haskell equivalent is the same code that returns or results in Either ErrorType ResultType. If you have some JavaScript that binds a promise or Deferred to the result of an I/O operation, like loading a Tweet, the Haskell Either would be wrapped inside IO, like IO (Either Error Tweet) (or alternately inside whatever other abstraction the Haskell program is using for I/O). For other things, it's the same: whatever you can use promises and Deferreds with in JavaScript, you can use Either with in Haskell.
{ "url": "http://www.reddit.com/r/haskell/comments/sijsg/what_is_the_haskell_equivalent_to_javascript/", "source_domain": "www.reddit.com", "snapshot_id": "crawl=CC-MAIN-2014-41", "warc_metadata": { "Content-Length": "85020", "Content-Type": "application/http; msgtype=response", "WARC-Block-Digest": "sha1:CQ4MI3BAVSGF62NO4XO3W45552U7YS5Z", "WARC-Concurrent-To": "<urn:uuid:31138fe0-ab6f-44b4-b6d7-e76bf68eb55e>", "WARC-Date": "2014-09-16T22:32:26Z", "WARC-IP-Address": "198.41.208.137", "WARC-Identified-Payload-Type": null, "WARC-Payload-Digest": "sha1:KWEB7RP5IGAJNF5TPOTDANXON5DXHHFW", "WARC-Record-ID": "<urn:uuid:55902f5b-e28e-4c4e-8bca-8e6967b3f983>", "WARC-Target-URI": "http://www.reddit.com/r/haskell/comments/sijsg/what_is_the_haskell_equivalent_to_javascript/", "WARC-Truncated": "length", "WARC-Type": "response", "WARC-Warcinfo-ID": "<urn:uuid:f28b8e4c-e892-4ed6-a581-f525ea94d6e5>" }, "warc_info": "robots: classic\r\nhostname: ip-10-196-40-205.us-west-1.compute.internal\r\nsoftware: Nutch 1.6 (CC)/CC WarcExport 1.0\r\nisPartOf: CC-MAIN-2014-41\r\noperator: CommonCrawl Admin\r\ndescription: Wide crawl of the web with URLs provided by Blekko for September 2014\r\npublisher: CommonCrawl\r\nformat: WARC File Format 1.0\r\nconformsTo: http://bibnum.bnf.fr/WARC/WARC_ISO_28500_version1_latestdraft.pdf" }
{ "line_start_idx": [ 0, 16, 17, 59, 60, 120, 121, 418, 419, 462, 463, 523, 524, 612, 613, 641, 642, 681, 682, 705, 706, 745, 746, 1006, 1007, 1356, 1357, 1403, 1404, 1464, 1465, 1581, 1582, 1631, 1632, 1692, 1693, 1836, 1837, 1879, 1880, 1940, 1941, 2105, 2106, 2807, 2808, 3062, 3063, 3411, 3412, 3452, 3453, 3513, 3514, 3604, 3605, 4074, 4075, 4488, 4489, 4528, 4529, 4589, 4590, 4778, 4779, 5165, 5166, 5209, 5210, 5270, 5271, 5382, 5383, 5428, 5429, 5489, 5490, 5655, 5656, 5698, 5699, 5759, 5760, 5780, 5781, 5806, 5807, 5850, 5915, 5955, 6020, 6021, 6115, 6116, 6154, 6155, 6215, 6216, 6419, 6420, 6461, 6462, 6522, 6523, 6643, 6644, 6689, 6690, 6750, 6751, 6914, 6915, 6962, 6963, 7037, 7038, 7097, 7098, 7361, 7362, 7407, 7408, 7468, 7469, 7586, 7587, 7925, 7926, 7972, 7973, 8033, 8034, 8105, 8106, 8149, 8150, 8210, 8211, 8215, 8216, 8259, 8260, 8320, 8321, 8381, 8382, 8426, 8427, 8487, 8488, 8520, 8521, 8578, 8579, 8851, 8852, 9067, 9068, 9351, 9352 ], "line_end_idx": [ 16, 17, 59, 60, 120, 121, 418, 419, 462, 463, 523, 524, 612, 613, 641, 642, 681, 682, 705, 706, 745, 746, 1006, 1007, 1356, 1357, 1403, 1404, 1464, 1465, 1581, 1582, 1631, 1632, 1692, 1693, 1836, 1837, 1879, 1880, 1940, 1941, 2105, 2106, 2807, 2808, 3062, 3063, 3411, 3412, 3452, 3453, 3513, 3514, 3604, 3605, 4074, 4075, 4488, 4489, 4528, 4529, 4589, 4590, 4778, 4779, 5165, 5166, 5209, 5210, 5270, 5271, 5382, 5383, 5428, 5429, 5489, 5490, 5655, 5656, 5698, 5699, 5759, 5760, 5780, 5781, 5806, 5807, 5850, 5915, 5955, 6020, 6021, 6115, 6116, 6154, 6155, 6215, 6216, 6419, 6420, 6461, 6462, 6522, 6523, 6643, 6644, 6689, 6690, 6750, 6751, 6914, 6915, 6962, 6963, 7037, 7038, 7097, 7098, 7361, 7362, 7407, 7408, 7468, 7469, 7586, 7587, 7925, 7926, 7972, 7973, 8033, 8034, 8105, 8106, 8149, 8150, 8210, 8211, 8215, 8216, 8259, 8260, 8320, 8321, 8381, 8382, 8426, 8427, 8487, 8488, 8520, 8521, 8578, 8579, 8851, 8852, 9067, 9068, 9351, 9352, 9484 ] }
{ "red_pajama_v2": { "ccnet_original_length": 9484, "ccnet_original_nlines": 161, "rps_doc_curly_bracket": 0, "rps_doc_ldnoobw_words": 1, "rps_doc_lorem_ipsum": 0, "rps_doc_stop_word_fraction": 0.44520190358161926, "rps_doc_ut1_blacklist": 0, "rps_doc_frac_all_caps_words": 0.02045097015798092, "rps_doc_frac_lines_end_with_ellipsis": 0, "rps_doc_frac_no_alph_words": 0.17986366152763367, "rps_doc_frac_unique_words": 0.30694353580474854, "rps_doc_mean_word_length": 4.8526930809021, "rps_doc_num_sentences": 63, "rps_doc_symbol_to_word_ratio": 0.001048770034685731, "rps_doc_unigram_entropy": 5.392013072967529, "rps_doc_word_count": 1541, "rps_doc_frac_chars_dupe_10grams": 0.22947311401367188, "rps_doc_frac_chars_dupe_5grams": 0.2564857006072998, "rps_doc_frac_chars_dupe_6grams": 0.25167158246040344, "rps_doc_frac_chars_dupe_7grams": 0.2391013652086258, "rps_doc_frac_chars_dupe_8grams": 0.2391013652086258, "rps_doc_frac_chars_dupe_9grams": 0.22947311401367188, "rps_doc_frac_chars_top_2gram": 0.021663550287485123, "rps_doc_frac_chars_top_3gram": 0.028884729370474815, "rps_doc_frac_chars_top_4gram": 0.03851297125220299, "rps_doc_books_importance": -878.4126586914062, "rps_doc_books_importance_length_correction": -878.4126586914062, "rps_doc_openwebtext_importance": -474.1763000488281, "rps_doc_openwebtext_importance_length_correction": -474.1763000488281, "rps_doc_wikipedia_importance": -428.2503967285156, "rps_doc_wikipedia_importance_length_correction": -428.2503967285156 }, "fasttext": { "dclm": 0.3808326721191406, "english": 0.9452120065689087, "fineweb_edu_approx": 1.5775587558746338, "eai_general_math": 0.7394994497299194, "eai_open_web_math": 0.3691883683204651, "eai_web_code": 0.07320886850357056 } }
{ "free_decimal_correspondence": { "primary": { "code": "005.133", "labels": { "level_1": "General works, books and libraries, information sciences", "level_2": "", "level_3": "Computer programming" } }, "secondary": { "code": "005.1", "labels": { "level_1": "General works, books and libraries, information sciences", "level_2": "", "level_3": "Computer programming" } } }, "bloom_cognitive_process": { "primary": { "code": "4", "label": "Analyze" }, "secondary": { "code": "3", "label": "Apply" } }, "bloom_knowledge_domain": { "primary": { "code": "2", "label": "Conceptual" }, "secondary": { "code": "3", "label": "Procedural" } }, "document_type_v1": { "primary": { "code": "5", "label": "Social/Forum" }, "secondary": { "code": "3", "label": "Reference/Encyclopedic/Educational" } }, "extraction_artifacts": { "primary": { "code": "0", "label": "No Artifacts" }, "secondary": { "code": "-1", "label": "Abstain" } }, "missing_content": { "primary": { "code": "0", "label": "No missing content" }, "secondary": { "code": "-1", "label": "Abstain" } }, "document_type_v2": { "primary": { "code": "5", "label": "Comment Section" }, "secondary": { "code": "18", "label": "Q&A Forum" } }, "reasoning_depth": { "primary": { "code": "3", "label": "Intermediate Reasoning" }, "secondary": { "code": "2", "label": "Basic Reasoning" } }, "technical_correctness": { "primary": { "code": "3", "label": "Mostly Correct" }, "secondary": { "code": "4", "label": "Highly Correct" } }, "education_level": { "primary": { "code": "3", "label": "Undergraduate Level" }, "secondary": { "code": "4", "label": "Graduate/Expert Level" } } }
672f1e42c33a7f9846924a2431ea77df
8,837,708,513,538,146,000
It is currently November 13th, 2019, 6:06 am Czech View and contribute to Rainmeter translations. oZone Posts: 82 Joined: May 14th, 2018, 4:46 pm Re: Czech oZone » September 28th, 2018, 5:34 pm Brian wrote:@gathering2: Thank you for your updated translations, however, some of the new translations are a bit too long to fit nicely in the areas we have defined. For example, the string STR_STARTLOGGING was previously translated to Spustit logování and is now Spustit zapisování do logovacího souboru. This string appears on the context menu, so a shorter translation would be preferred. Using https://translate.google.com converts Start logging to Začněte protokolovat, but I have no idea if that is accurate or not and would not rely on such translation engines. There are several more instances where the new translations are too long. Also, some punctuation was removed (STR_BACKUPE needs an ellipsis), so make sure all the correct punctuation and correct capitalization is there. STR_NEWVERSIONREQUIRED (and others) needs some capitalization. -Brian oZone wrote:According to my friend the current translation is ok. And the string STR_LANGUAGEOBSOLETE should be "Jazyk je neaktuální". Also won't be better to use HTTPS instead of HTTP for STR_GETLATESTVERSION and STR_COPYRIGHTNOTICE. Does the translation need to be updated or the current one is fine, I can ask friend to help. Last edited by oZone on September 28th, 2018, 6:43 pm, edited 1 time in total. User avatar kyriakos876 Posts: 915 Joined: January 30th, 2017, 2:01 am Location: Greece Re: Czech kyriakos876 » September 28th, 2018, 6:35 pm oZone wrote:Does the translation need to be updated or the current one is fine, I can ask friend to help. You can see here if you scroll down, Czech is marked incomplete. I'd suggest you download the files from the first post and follow the instructions Brian gave above. And if you see anything more, translate that too, then copy and paste both files in Code: Select all [ /code] brackets as a reply to this thread. User avatar Brian Developer Posts: 1920 Joined: November 24th, 2011, 1:42 am Location: Utah Re: Czech Brian » October 9th, 2018, 3:04 am oZone wrote:According to my friend the current translation is ok. And the string STR_LANGUAGEOBSOLETE should be "Jazyk je neaktuální". oZone wrote:Does the translation need to be updated or the current one is fine, I can ask friend to help. Sorry for not getting back to you, I was actually hoping user "gathering2" would respond since he was the one who initially attempted to update the language files. Since we have not heard back from that user, I went ahead with your friend's suggestion. Thank you for the help! -Brian
{ "url": "https://forum.rainmeter.net/viewtopic.php?t=11263&p=153295", "source_domain": "forum.rainmeter.net", "snapshot_id": "crawl=CC-MAIN-2019-47", "warc_metadata": { "Content-Length": "18984", "Content-Type": "application/http; msgtype=response", "WARC-Block-Digest": "sha1:JW7ONUUJBLPTGEQCXWEKV2SWWKJXPNWR", "WARC-Concurrent-To": "<urn:uuid:3fce322f-3bf2-4881-810f-ae80a829d596>", "WARC-Date": "2019-11-13T06:06:06Z", "WARC-IP-Address": "104.28.2.61", "WARC-Identified-Payload-Type": "text/html", "WARC-Payload-Digest": "sha1:ZG5Z325N2QY7ROSVWIKELVMZFE7E7YPH", "WARC-Record-ID": "<urn:uuid:e0b4c8b8-0c6a-4c24-a166-66f3c4add4b0>", "WARC-Target-URI": "https://forum.rainmeter.net/viewtopic.php?t=11263&p=153295", "WARC-Truncated": null, "WARC-Type": "response", "WARC-Warcinfo-ID": "<urn:uuid:f6d95851-63b0-4bb8-b14f-d1bb4da868d0>" }, "warc_info": "isPartOf: CC-MAIN-2019-47\r\npublisher: Common Crawl\r\ndescription: Wide crawl of the web for November 2019\r\noperator: Common Crawl Admin ([email protected])\r\nhostname: ip-10-67-67-147.ec2.internal\r\nsoftware: Apache Nutch 1.16 (modified, https://github.com/commoncrawl/nutch/)\r\nrobots: checked via crawler-commons 1.1-SNAPSHOT (https://github.com/crawler-commons/crawler-commons)\r\nformat: WARC File Format 1.1\r\nconformsTo: http://iipc.github.io/warc-specifications/specifications/warc-format/warc-1.1/" }
{ "line_start_idx": [ 0, 45, 46, 52, 53, 100, 106, 116, 148, 149, 159, 160, 198, 199, 224, 366, 367, 770, 771, 1054, 1055, 1062, 1197, 1297, 1391, 1470, 1482, 1494, 1505, 1541, 1558, 1559, 1569, 1570, 1614, 1615, 1721, 1971, 1972, 1989, 1990, 2035, 2047, 2053, 2063, 2075, 2112, 2127, 2128, 2138, 2139, 2174, 2175, 2310, 2416, 2669, 2670, 2694, 2695 ], "line_end_idx": [ 45, 46, 52, 53, 100, 106, 116, 148, 149, 159, 160, 198, 199, 224, 366, 367, 770, 771, 1054, 1055, 1062, 1197, 1297, 1391, 1470, 1482, 1494, 1505, 1541, 1558, 1559, 1569, 1570, 1614, 1615, 1721, 1971, 1972, 1989, 1990, 2035, 2047, 2053, 2063, 2075, 2112, 2127, 2128, 2138, 2139, 2174, 2175, 2310, 2416, 2669, 2670, 2694, 2695, 2701 ] }
{ "red_pajama_v2": { "ccnet_original_length": 2701, "ccnet_original_nlines": 58, "rps_doc_curly_bracket": 0, "rps_doc_ldnoobw_words": 0, "rps_doc_lorem_ipsum": 0, "rps_doc_stop_word_fraction": 0.3615107834339142, "rps_doc_ut1_blacklist": 0, "rps_doc_frac_all_caps_words": 0.028776979073882103, "rps_doc_frac_lines_end_with_ellipsis": 0, "rps_doc_frac_no_alph_words": 0.23381294310092926, "rps_doc_frac_unique_words": 0.4954337775707245, "rps_doc_mean_word_length": 4.89497709274292, "rps_doc_num_sentences": 26, "rps_doc_symbol_to_word_ratio": 0, "rps_doc_unigram_entropy": 5.031179904937744, "rps_doc_word_count": 438, "rps_doc_frac_chars_dupe_10grams": 0.21408581733703613, "rps_doc_frac_chars_dupe_5grams": 0.21408581733703613, "rps_doc_frac_chars_dupe_6grams": 0.21408581733703613, "rps_doc_frac_chars_dupe_7grams": 0.21408581733703613, "rps_doc_frac_chars_dupe_8grams": 0.21408581733703613, "rps_doc_frac_chars_dupe_9grams": 0.21408581733703613, "rps_doc_frac_chars_top_2gram": 0.02332090027630329, "rps_doc_frac_chars_top_3gram": 0.02378731034696102, "rps_doc_frac_chars_top_4gram": 0.027985069900751114, "rps_doc_books_importance": -279.55322265625, "rps_doc_books_importance_length_correction": -279.55322265625, "rps_doc_openwebtext_importance": -158.48025512695312, "rps_doc_openwebtext_importance_length_correction": -158.48025512695312, "rps_doc_wikipedia_importance": -94.44991302490234, "rps_doc_wikipedia_importance_length_correction": -94.44991302490234 }, "fasttext": { "dclm": 0.020103100687265396, "english": 0.915337085723877, "fineweb_edu_approx": 1.3973419666290283, "eai_general_math": 0.26237988471984863, "eai_open_web_math": 0.4848114252090454, "eai_web_code": 0.00014555000234395266 } }
{ "free_decimal_correspondence": { "primary": { "code": "005.4", "labels": { "level_1": "General works, books and libraries, information sciences", "level_2": "", "level_3": "Computer programming" } }, "secondary": { "code": "418.02", "labels": { "level_1": "Philology; or, Language and languages", "level_2": "Linguistics", "level_3": "Language acquisition and Applied linguistics" } } }, "bloom_cognitive_process": { "primary": { "code": "3", "label": "Apply" }, "secondary": { "code": "2", "label": "Understand" } }, "bloom_knowledge_domain": { "primary": { "code": "3", "label": "Procedural" }, "secondary": { "code": "2", "label": "Conceptual" } }, "document_type_v1": { "primary": { "code": "5", "label": "Social/Forum" }, "secondary": { "code": "-1", "label": "Abstain" } }, "extraction_artifacts": { "primary": { "code": "3", "label": "Irrelevant Content" }, "secondary": { "code": "0", "label": "No Artifacts" } }, "missing_content": { "primary": { "code": "0", "label": "No missing content" }, "secondary": { "code": "-1", "label": "Abstain" } }, "document_type_v2": { "primary": { "code": "18", "label": "Q&A Forum" }, "secondary": { "code": "21", "label": "Customer Support" } }, "reasoning_depth": { "primary": { "code": "2", "label": "Basic Reasoning" }, "secondary": { "code": "3", "label": "Intermediate Reasoning" } }, "technical_correctness": { "primary": { "code": "3", "label": "Mostly Correct" }, "secondary": { "code": "4", "label": "Highly Correct" } }, "education_level": { "primary": { "code": "2", "label": "High School Level" }, "secondary": { "code": "3", "label": "Undergraduate Level" } } }
672f1e42c33a7f9846924a2431ea77df
1,248,518,960,576,374,000
Search Images Maps Play YouTube News Gmail Drive More » Sign in Screen reader users: click this link for accessible mode. Accessible mode has the same essential features but works better with your reader. Patents 1. Advanced Patent Search Publication numberUS6347361 B1 Publication typeGrant Application numberUS 09/024,587 Publication dateFeb 12, 2002 Filing dateFeb 17, 1998 Priority dateFeb 17, 1998 Fee statusLapsed Publication number024587, 09024587, US 6347361 B1, US 6347361B1, US-B1-6347361, US6347361 B1, US6347361B1 InventorsRavi Kumar Arimilli, John Steven Dodson, Jerry Don Lewis Original AssigneeInternational Business Machines Corporation Export CitationBiBTeX, EndNote, RefMan External Links: USPTO, USPTO Assignment, Espacenet Cache coherency protocols with posted operations US 6347361 B1 Abstract A method of avoiding deadlocks in cache coherency protocol for a multi-processor computer system, by loading a memory value into a plurality of cache blocks, assigning a first coherency state having a higher collision priority to only one of the cache blocks, and assigning one or more additional coherency states having lower collision priorities to all of the remaining cache blocks. Different system bus codes can be used to indicate the priority of conflicting requests (e.g., DClaim operations) to modify the memory value. The invention also allows folding or elimination of redundant DClaim operations, and can be applied in a global versus local manner within a multi-processor computer system having processing units grouped into at least two clusters. Images(6) Previous page Next page Claims(31) What is claimed is: 1. A method of avoiding deadlocks in a multi-processor computer system between two or more caches which are sharing a value corresponding to a system memory block, comprising the steps of: loading a value corresponding to a specific memory block of a system memory device into a plurality of cache blocks of respective processing units, said loading step including the step of sourcing the value from the system memory device to at least one of the plurality of cache blocks, such that all of the plurality of cache blocks are consistent with the specific memory block; assigning a first cache coherency state having a first collision priority to a single one of the plurality of cache blocks; assigning one or more additional cache coherency states having one or more additional collision priorities which are lower than the first collision priority, to all of the remaining plurality of cache blocks other than the single cache block; issuing a first request associated with one of the remaining cache blocks to claim the specific memory block for future modification, wherein the first request provides an indication of the one or more additional collision priorities; issuing a second request associated with the single cache block to claim the specific memory block for future modification, such that the memory block claimed by the second request conflicts with the memory block claimed by the first request, and wherein the second request provides an indication of the first collision priority; and withdrawing the first request in response to the second request, based on the collision priority indications provided by the first and second requests. 2. The method of claim 1 wherein: said step of loading the value into the plurality of cache blocks includes the step of copying the value Into the single cache block after the value has been loaded into the remaining cache blocks; and said step of assigning the first cache coherency state is performed in response to said copying step. 3. The method of claim 1 wherein said loading step includes the steps of: sourcing the value from the system memory device to a first one of the plurality of cache blocks; assigning an exclusive cache coherency state to the first cache block in response to said sourcing step; copying the value into the single cache block after the value has been sourced to the first cache block; and switching the assignment of the first cache block from the exclusive cache coherency state to the one or more additional cache coherency states in response to said copying step. 4. The method of claim 2 wherein: at least one of the remaining cache blocks had been assigned the first coherency state prior to said step of assigning the first coherency state to the single cache block; and said step of assigning the one or more additional cache coherency states includes the step of changing the assignment of the at least one of the remaining cache blocks from the first cache coherency state to the one or more additional cache coherency states as the value is loaded into the single cache block. 5. The method of claim 2 wherein said loading step includes the step of sourcing the value from the system memory device to at least one of the plurality of cache blocks, such that all of the plurality of cache blocks are consistent with the memory block. 6. The method of claim 3 wherein said copying step is performed by copying the value from the firs cache block. 7. The method of claim 1 further comprising the step of completing a store instruction associated with the second request, without waiting for any response to the second request. 8. The method of claim 7 further comprising the steps of: assigning a modified cache coherency state to the single cache block in response to the second request; and assigning an invalid cache coherency state to the remaining cache blocks in response to the second request. 9. A computer system comprising: a memory device having a plurality of memory blocks; a bus connected to said memory device; a plurality of processing units connected to said bus, each processing unit having a cache with at least one cache block; and cache coherency means for (i) assigning a first cache coherency state having a first collision priority to a single cache block in a plurality of said cache blocks having a shared value which is associated with a corresponding memory block of said memory device, and (ii) assigning one or more additional cache coherency states having one or more additional collision priorities which are lower than said first collision priority, to all remaining cache blocks in said plurality of cache blocks other than said single cache block which share the value, said cache coherency means ensuring, when a given cache of said remaining cache blocks issues a first request to claim said corresponding memory block for future modification wherein the first request provides an indication of the one or more additional collision priorities, and a cache of said single cache block Issues a second, conflicting request to similarly claim said corresponding memory block for future modification wherein the second request provides an indication of the first collision priority, that said given cache withdraws said first request in response to said second request based on the collision priority indications provided by the first and second requests. 10. The computer system of claim 9 wherein: the value is copied into said single cache block after the value has been loaded into said remaining cache blocks; and said cache coherency means assigns said first cache coherency state as the value is copied Into said single cache block. 11. The computer system of claim 9 wherein said cache coherency means provides at least three coherency states for other cache blocks which do not share the value, including: an invalid state; an exclusive state; and a modified state. 12. The computer system of claim 9 wherein said cache coherency means further provides for intervention of the value by sourcing the value from said single cache block. 13. The computer system of claim 9 wherein said first cache coherency state is a recently read state. 14. The computer system of claim 9 wherein said cache coherency means allows completion of a store instruction associated with said second request, without waiting for any response to said second request. 15. The computer system of claim 13 wherein said cache coherency means further provides for intervention of the value by sourcing the value from said single cache block. 16. The computer system of claim 15 wherein said cache coherency means provides at least three coherency states for other cache blocks which do not share the value, including: an invalid state; an exclusive state; and a modified state. 17. A method of maintaining cache coherency in a multiprocessor computer system, comprising the steps of: assigning a first cache coherency state to a first cache block which has shared control of a specific memory block of a system memory device; assigning a second cache coherency state to a second cache block which has shared control of the specific memory block of the system memory device; issuing a first system bus code to indicate that the first cache block is requesting modification of the specific memory block, wherein the first system bus code has a first collision priority to resolve conflicting claims for the specific memory block; issuing a second system bus code, different from the first system bus code, to indicate that the second cache block is requesting modification of the specific memory block, wherein the second system bus code has a second collision priority to resolve conflicting claims for the specific memory block, the second collision priority being lower than the first collision priority; and completing a store instruction associated with the first system bus code, without waiting for any response to said issuing of the first system bus code, in response to a determination that the first system bus code has the first collision priority. 18. The method of claim 17 further comprising the step of withdrawing a modification request associated with the second system bus code in response to detection of said issuing of the first system bus code. 19. The method of claim 17 wherein said step of issuing the first system bus code occurs prior to said stop of issuing the second system bus code. 20. The method of claim 17 further comprising the step of loading a value corresponding to the specific memory block of the system memory device Into a plurality of cache blocks of respective processing units, by copying the value into the first cache block after the value has been loaded into the second cache block, and wherein said step of assigning the first cache coherency state is performed in response to said copying step. 21. The method of claim 17 wherein: the second cache block had been assigned the first coherency state prior to said step of assigning the first coherency state to the first cache block; and said step of assigning the second cache coherency state includes the step of changing the assignment of the second cache block from the first cache coherency state to the second cache coherency state as the value is loaded into the first cache block. 22. The method of claim 20 wherein said loading step includes the step of sourcing the value from the system memory device to at least one of the plurality of cache blocks, such that all of the plurality of cache blocks are consistent with the specific memory block. 23. The method of claim 20 wherein said loading step includes the steps of: storing the value from an associated processing unit into the second cache block; and sourcing the value from the second cache block to the first cache block, such that the first and second cache blocks are not consistent with the specific memory block. 24. The method of claim 17 further comprising the step of assigning a modified cache coherency state to the first cache block in response to said step of issuing the first system bus code. 25. The method of claim 24 further comprising the step of assigning an invalid cache coherency state to the second cache block in response to said step of issuing the first system bus code. 26. A computer system comprising: a memory device; a bus connected to said memory device; a plurality of processing units connected to said bus, each processing unit having a cache, and each cache having a plurality of cache blocks for storing data values associated with respective memory blocks of said memory device; and cache coherency means for assigning a first cache coherency state to a first cache block which has shared control of a specific memory block of said memory device, assigning a second cache coherency state to a second cache block which has shared control of said specific memory block of said memory device, issuing a first system bus code to indicate that said first cache block is requesting modification of said specific memory block, wherein the first system bus code has a first collision priority to resolve conflicting claims for said specific memory block, and issuing a second system bus code different from said first system bus code, to indicate that said second cache block is requesting modification of said specific memory block, wherein said second system bus code has a second collision priority to resolve conflicting claims for said specific memory block, said second collision priority being lower than said first collision priority, said cache coherency means allows completion of a store instruction associated with said first system bus code, without waiting for any response to said first system bus code in response to a determination that said first system bus code has said first collision priority. 27. The computer system of claim 26 wherein said cache coherency means includes means for with drawing a modification request associated with said second system bus code in response to detection of said first system bus code. 28. The computer system of claim 26 wherein: a value corresponding to said specific memory block is copied into said first cache block after the value has been loaded into said second cache block; and said cache coherency means assigns said first cache coherency state as the value is copied into said first cache block. 29. The computer system of claim 26 wherein said cache coherency means provides at least three coherency states for other cache blocks which do not have shared control of said memory block of said memory device, including: an invalid state; an exclusive state; and a modified state. 30. The computer system of claim 26 wherein: said second cache block had been assigned said first coherency state prior to said step of assigning said first coherency state to the first cache block; and said cache coherency means assigns said second cache coherency state to said second cache block by changing said assignment of said second cache block from said first cache coherency state to said second cache coherency state as the value is loaded into said first cache block. 31. The computer system of claim 28 wherein said cache coherency means further provides for intervention of the value by sourcing the value from said first cache block. Description BACKGROUND OF THE INVENTION 1. Field of the Invention The present invention generally relates to computer systems, particularly to a method of maintaining cache coherency in a multi-processor computer system, while allowing the posting of certain cache operations such that a broadcast of an operation may be delayed but the operation may nevertheless execute immediately, and further relates to more efficient handling of such posted operations. 2. Description of Related Art The basic structure of a conventional multi-processor computer system 10 is shown in FIG. 1. Computer system 10 has several processing units, two of which 12 a and 12 b are depicted, which are connected to various peripheral devices, including input/output (I/O) devices 14 (such as a display monitor, keyboard, and permanent storage device), memory device 16 (such as random access memory or RAM) that is used by the processing units to carry out program instructions, and firmware 18 whose primary purpose is to seek out and load an operating system from one of the peripherals (usually the permanent memory device) whenever the computer is first turned on. Processing units 12 a and 12 b communicate with the peripheral devices by various means, including a generalized interconnect or bus 20. Computer system 10 may have many additional components which are not shown, like serial and parallel ports for connection to modems or printers. Those skilled in the art will further appreciate that there are other components that might be used in conjunction with those shown in the block diagram of FIG. 1; a display adapter might be used to control a video display monitor, a memory controller can be used to access memory 16, etc. The computer can also have more than two processing units. In a symmetric multi-processor (SMP) computer, all of the processing units are generally identical, that is, they all use a common set or subset of instructions and protocols to operate, and generally have the same architecture. A typical SMP architecture is shown in FIG. 1. A processing unit includes a processor core 22 having a plurality of registers and execution units, which carry out program instructions in order to operate the computer. An exemplary processing unit includes the PowerPC™ processor marketed by International Business Machines Corp. The processing unit can also have one or more caches, typically an instruction cache 24 and a data cache 26, which are implemented using high speed memory devices. Caches are commonly used to temporarily store values (instructions and/or data) that might be repeatedly accessed by a processor, in order to speed up processing by avoiding the longer step of loading the values from memory 16. These caches are referred to as “on-board” when they are integrally packaged with the processor core on a single integrated chip 28. Each cache is associated with a cache controller (not shown) that manages the transfer of data between the processor core and the cache memory. A processing unit can include additional caches, such as cache 30, which is referred to as a level 2 (L2) cache since it supports the on-board (level 1) caches 24 and 26. In other words, cache 30 acts as an intermediary between memory 16 and the on-board caches, and can store a much larger amount of information (instructions and data) than the on-board caches can, but at a longer access penalty. Cache 30 may be a chip having a storage capacity of 256 or 512 kilobytes, while the processor may be an IBM PowerPC™ 604-series processor having on-board caches-with 64 kilobytes of total storage. Cache 30 is connected to bus 20, and all loading of information from memory 16 into processor core 22 must come through cache 30. Although FIG. 1 depicts only a two-level cache hierarchy, multi-level cache hierarchies can be provided where there are many levels (L3, L4, etc.) of serially connected caches. In an SMP computer, it is important to provide a coherent memory system, that is, to cause write operations to each individual memory location to be serialized in some order for all processors. Assuming that a location in memory is modified by a sequence of write operations to take on the specific successive values of “1,” “2,” “3,” and “4,” in a cache coherent system all processors will observe the writes to the given location to take place in the order shown. However, it is possible for a processing element to miss a write to the memory location. A given processing element reading the memory location could see the sequence 1, 3, 4, missing the update to the value 2. A system that implements these properties is said to be “coherent”. Virtually all coherency protocols operate only to the granularity of the size of a cache block. That is to say, the coherency protocol controls the movement of and write permissions for data on a cache block basis and not separately for each individual memory location. There are a number of protocols and techniques for achieving cache coherence that are known to those skilled in the art. At the heart of all these mechanisms for maintaining coherency is the requirement that the protocols allow only one processor to have a “permission” that allows a write to a given memory location (cache block) at any given point in time. As a consequence of this requirement, whenever a processing element attempts to write to a memory location, it must first inform all other processing elements of its desire to write the location and receive permission from all other processing elements to carry out the write. The key issue is that all other processors in the system must be informed of the write by the initiating processor before the write occurs. Furthermore, if a block is present in the L1 cache of a given processing unit, it is also present in the L2 and L3 caches of that processing unit. This property is known as inclusion. To implement cache coherency in a system, the processors communicate over a common generalized interconnect (i.e., bus 20). The processors pass messages over the interconnect indicating their desire to read or write memory locations. When an operation is placed on the interconnect, all of the other processors “snoop” (monitor) this operation and decide if the state of their caches can allow the requested operation to proceed and if so, under what conditions. There are several bus transactions that require snooping and follow-up action to honor the bus transactions and maintain memory coherency. The snooping operation is triggered by the receipt of a qualified snoop request, generated by the assertion of certain bus signals. Instruction processing is interrupted only when a snoop hit occurs and the snoop state machine determines that an additional cache snoop is required to resolve the coherency of the offended sector. This communication is necessary because, in systems with caches, the most recent valid copy of a given block of memory may have moved from the system memory 16 to one or more of the caches in the system (as mentioned above). If a processor (say 12 a) attempts to access a memory location not present within its cache hierarchy, the correct version of the block, which contains the actual (current) value for the memory location, may either be in the system memory 16 or in one of more of the caches in another processing unit, such as processing unit 12 b. If the correct version is in one or more of the other caches in the system, it is necessary to obtain the correct value from the cache(s) in the system instead of system memory. For example, consider a processor, say 12 a, attempting to read a location in memory. It first polls its own L1 cache (24 or 26). If the block is not present in the L1 cache, the request is forwarded to the L2 cache (30). If the block is not present in the L2 cache, the request is forwarded on to lower cache levels, like the L3 cache. If the block is not present in the lower level caches, the request is then presented on the generalized interconnect (20) to be serviced. Once an operation has been placed on the generalized interconnect, all other processing units snoop the operation and determine if the block is present in their caches. If a given processing unit has the block of data requested by processing unit in its L1 cache, and that data is modified, by the principle of inclusion the L2 cache and any lower level caches also have copies of the block (however, their copies are stale, since the copy in the processor's cache is modified). Therefore, when the lowest level cache (e.g., L3) of the processing unit snoops the read instruction, it will determine that the block requested is present and modified in a higher level cache. When this occurs, the L3 cache places a message on the generalized interconnect informing the processing unit that it must “retry” its operation again at a later time, because the actual value of the memory location is in the L1 cache at the top of the memory hierarchy and must be retrieved to make it available to service the read request of the initiating processing unit. Once the request from an initiating processing unit has been retried, the L3 cache begins a process to retrieve the modified data from the L1 cache and make it available at the L3 cache, main memory or both, depending on the exact details of the implementation which are not specifically relevant to this invention. To retrieve the block from the higher level caches, the L3 cache sends messages through the inter-cache connections to the higher level caches, requesting that the block be retrieved. These messages propagate up the processing unit hierarchy until they reach the L1 cache and cause the block to be moved down the hierarchy to the lowest level (L3 or main memory) to be able to service the request from the initiating processing unit. The initiating processing unit eventually re-presents the read request on the generalized interconnect. At this point, however, the modified data has been retrieved from the L1 cache of a processing unit and the read request from the initiating processor will be satisfied. The scenario just described is commonly referred to as a “snoop push”. A read request is snooped on the generalized interconnect which causes the processing unit to “push” the block to the bottom of the hierarchy to satisfy the read request made by the initiating processing unit. The essential point is that, when a processor wishes to read or write a block, it must communicate that desire with the other processing units in the system in order to maintain cache coherence. To achieve this, the cache coherence protocol associates with each block in each level of the cache hierarchy, a status indicator indicating the current “state” of the block. The state information is used to allow certain optimizations in the coherency protocol that reduce message traffic on the generalized interconnect and the inter-cache connections. As one example of this mechanism, when a processing unit executes a read it receives a message indicating whether or not the read must be retired later. If the read operation is not retried, the message usually includes information allowing the processing unit to determine if any other processing unit also has a still active copy of the block (this is accomplished by having the other lowest level caches give a “shared” or “not shared” indication for any read they do not retry). Therefore, a processing unit can determine whether any other processor in the system has a copy of the block. If no other processing unit has an active copy of the block, the reading processing unit marks the state of the block as “exclusive”. If a block is marked exclusive it is permissible to allow the processing unit to later write the block without first communicating with other processing units in the system because no other processing unit has a copy of the block. Therefore, it is possible for a processor to read or write a location without first communicating this intention onto the interconnection, but only where the coherency protocol has insured that no other processor has an interest in the block. The foregoing cache coherency technique is implemented in one prior art protocol referred to as “MESI,” and illustrated in FIG. 2. In this protocol, a cache block can be in one of four states, “M” (Modified), “E” (Exclusive), “S” (Shared) or “I” (Invalid). Under the MESI protocol, each cache entry (e.g., a 32-byte sector) has two additional bits which indicate the state of the entry, out of the four possible states. Depending upon the initial state of the entry and the type of access sought by the requesting processor, the state may be changed, and a particular state is set for the entry in the requesting processor's cache. When a block is in the Modified state, the addressed block is valid only in the cache having the modified block, and the modified data has not been written back to system memory. When a block is Exclusive, it is present only in the noted block, and is consistent with system memory. If a block is Shared, it is valid in that cache and in at least one other cache, all of the shared blocks being consistent with system memory. Finally, when a block is Invalid, it means that any resident value is not valid with respect to any corresponding address indicated for the block, i.e., the value is not consistent with system memory. As seen in FIG. 2, if a block is in any of the Modified, Shared or Invalid states, it can move between the states depending upon the particular bus transaction. While a block in an Exclusive state can move to any other state, a block can only become Exclusive if it is first Invalid. A cache's block can become Invalid (e.g., from the Shared state) if the cache snoops an operation from a different processor indicating that the value held in the cache block is to be modified by the other processor, such as by snooping a read-with-intent-to-modify (RWITM) operation. Some processor architectures, including the PowerPC™ processor, allow the execution of one or more special operations, other than the RWITM operation, when a processor wants to claim a memory block for a future store instruction (modifying the block). The “DClaim” operation is one example. This operation is used in lieu of the RWITM bus transaction when a valid value for the subject block is already held in the same processor's cache, e.g., in a Shared state (if the value were currently held in a Modified or Exclusive state, there would be no need to broadcast either a RWITM or DClaim request since the processor would already have exclusive control of the block). The processor may be adapted to execute a DClaim operation initially, by examining its on-board (L1) cache to see if the valid value is resident there. If not, the processor can issue a RWITM request, and any lower level cache having the valid value will, upon receiving the RWITM request, convert it into a DClaim operation to be passed to the system bus. The DClaim operation accordingly is an address-only operation since the value does not need to be read (from system memory or any intervening cache). Because of this attribute, the DClaim operation is more efficient than a RWITM operation, which would force the read operation across the system bus. When another cache has the same addressed block in a valid (Shared) state and snoops a DClaim transaction for the block, that other cache switches its corresponding block to an Invalid state, releasing the block so that the requesting processor can proceed to modify the value. In other words, a DClaim transaction appears just like a RWITM operation from a snooper perspective. One problem with DClaim-type operations is that they occasionally (sometimes frequently) suffer significant performance degradation, since completion of the operation can be delayed by coherency responses from other devices in the memory hierarchy. For example, if several caches of different processing units are holding a value in Shared states and they snoop a DClaim operation, their respective processors may repeatedly issue retry messages in response to the DClaim snoop (if these processors are currently busy or otherwise unable to handle the snoop, for whatever reason). This outcome means that the processor of the cache issuing the DClaim request must effectively halt processing of the associated program instruction set (an instruction “thread”), since the processor cannot complete the desired store of the modified value until the DClaim request is re-issued, possibly repeatedly, and appropriate (non-retry) responses are received from all other caches. A significant delay might also occur due to the operation having to wait in line in the cache operation queue. The same problems can occur with a typical RWITM operation, but with a DClaim operation it might actually be unnecessary to wait to execute the DClaim store. Every other cache (including those responding with “retry”) must have the targeted block in either the Invalid state or the Shared state. If the cache block were Invalid, then the eventual response (“null” or “clean”) would not interfere with the operation. If the cache block were Shared, and presuming that none of the other processors were contemporaneously issuing DClaim requests for the same block, it would be unnecessary for the initiating processor to wait to execute the DClaim store instruction, since no other action would be required of the responding caches before the DClaim operation could properly complete. Unfortunately, the prior art does not provide any method of knowing or ensuring that no other Shared cache blocks are attempting to execute conflicting DClaim operations at approximately the same time. If two or more caches hold a value in the Shared state and they do issue DClaim requests simultaneously or nearly simultaneously (i.e., they attempt to cross-invalidate each other), then a deadlock can occur wherein each associated processor becomes ensnared in an endless cycle of retry responses. One method for handling such pipeline collisions is to modify the coherency protocol to preclude a cache from issuing a retry response to a DClaim request if that cache itself has an outstanding DClaim request for the same block and a response to the latter request has not been received yet. In other words, if a cache having a Shared block has issued a DClaim request and has not yet received appropriate responses allowing completion of the store instruction, and it snoops a second DClaim request from another cache for the same block, then it must not retry the second Dclaim until it has received null responses for its own request. These problems can be especially acute where the processing environment is such that a significant number of memory blocks end up getting shared among at least two processors. Other factors can compound the problems, such as when the processing system allows read-write operations to be atomized using load-and-reserve instructions (in the PowerPC™ instruction set, “lwarx” for 32-bit implementations and “ldarx” for 64-bit implementations) followed by associated conditional store instructions (“stwcx” for 32-bit implementations and “stdcx” for 64-bit implementations). The conditional store instructions can result in delayed DClaim operations, possibly nullifying the benefits of attempting to atomize the read-write operation. The foregoing problems are also exacerbated by the fact that it might be unnecessary to broadcast a DClaim request (and therefore unnecessary to handle subsequent responses and replies). An example is the eviction of the value from the cache of the processor which issued a particular DClaim operation, as described above, that does not require waiting for responses. An eviction can result when the processor executes other instructions from other threads, such that other unrelated values must be loaded in the cache and displace (cast out) one or more existing blocks. The eviction algorithm, such as a least recently used (LRU) algorithm, might pick a block for eviction that is the subject of a recent DClaim operation. The block may be cast out in a burst write operation. In such cases (and again presuming that none of the other processors were contemporaneously issuing DClaim requests for the same block), the DClaim operation becomes extraneous, since the write operation would be broadcast to any other caches having the Shared value, and so those other cache blocks would switch to Invalid anyway. Conventional cache coherency protocols cannot eliminate such redundant broadcasts. In light of the foregoing problems, it would be desirable to provide a method of handling DClaim-type operations which did not require an initiating processor to wait for unnecessary cache responses. It would be further advantageous if the method could avoid deadlocks without losing the benefits of the DClaim operation, or could eliminate unnecessary DClaim operations which had become moot as a result of intervening events. SUMMARY OF THE INVENTION It is therefore one object of the present invention to provide an improved cache to be used by a processor of a computer system. It is another object of the present invention to provide a protocol to maintain cache coherency in a multi-processor system, and to allow a processor in the system to proceed with certain operations without waiting for unnecessary coherency responses. It is yet another object of the present invention to provide a method of eliminating cache coherency operations which become unnecessary due to intervening processing. The foregoing objects are achieved in a method of avoiding deadlocks in a multi-processor computer system between two or more caches which are sharing a value corresponding to a system memory block, generally comprising the steps of, loading a value corresponding to a memory block of a system memory device into a plurality of cache blocks of respective processing units, assigning a first cache coherency state having a first collision priority to a single one of the plurality of cache blocks, and assigning one or more additional cache coherency states having one or more additional collision priorities which are lower than the first collision priority, to all of the remaining plurality of cache blocks other than the single cache block. A first system bus code can be issued to indicate that the first cache block is requesting modification of the memory block, while a second system bus code, different from the first system bus code, can be issued to indicate that the second cache block is requesting modification of the memory block. The invention also allows folding or elimination of certain redundant cache operations, by loading a first cache operation in the cache operations queue to request modification of a value for a cache block already held in a cache associated with the queue, loading a second cache operation in the cache operations queue to request writing of a new value for the cache block, and removing the first cache operation from the queue in response to said step of loading the second cache operation in the queue. The invention can also be applied in a global versus local manner within a multi-processor computer system having a plurality of processing units grouped into at least two clusters, each processing unit cluster having at least two cache levels wherein a given one of a plurality of caches in the first cache level is used by only a single processing unit, and a given one of a plurality of caches in the second cache level is used by two or more processing units in the same cluster, by assigning a first cache coherency state having a first collision priority to a first cache line of a cache in the first cache level associated with a first processing unit in the first processing unit cluster, and assigning one or more additional cache coherency states having one or more additional collision priorities which are lower than the first collision priority, to one or more additional cache lines of a cache in the first cache level associated with a second processing unit in the second processing unit cluster. The above as well as additional objectives, features, and advantages of the present invention will become apparent in the following detailed written description. BRIEF DESCRIPTION OF THE DRAWINGS The novel features believed characteristic of the invention are set forth in the appended claims. The invention itself, however, as well as a preferred mode of use, further objectives, and advantages thereof, will best be understood by reference to the following detailed description of an illustrative embodiment when read in conjunction with the accompanying drawings, wherein: FIG. 1 is a block diagram of a prior art multi-processor computer system; FIG. 2 is a state diagram depicting a prior art cache coherency protocol (MESI); FIG. 3 is a state diagram depicting one embodiment of a cache coherency protocol (R-MESI) that may be used in accordance with the present invention; FIG. 4 is a state diagram depicting another embodiment of a cache coherency protocol (MRSI) that may be used in accordance with the present invention; FIG. 5 is a state diagram depicting still another embodiment of a cache coherency protocol (T-MESI) that may be used in accordance with the present invention; FIG. 6 is a block diagram of one embodiment of a multi-processor computer system having a multi-level cache architecture that may be used to implement the present invention on both a global scale (system wide) and a local scale (CPU cluster); and FIG. 7 is a block diagram of one embodiment of a cache construction which may be used in accordance with the present invention to streamline a cache operations queue. DESCRIPTION OF AN ILLUSTRATIVE EMBODIMENT The present invention is directed to a method of handling cache operations in a memory-coherent, multi-processor system, such as the system of FIG. 1, but the present invention could be applied to computer systems that are not necessarily conventional, i.e., they could include new hardware components not shown in FIG. 1, or have a novel interconnection architecture for existing components (or both). Therefore, the present invention is not limited to the generalized system shown in that figure. The improved method of the present invention, which utilizes information relating to coherency states of cache blocks, is particularly directed to the handling of cache operations like the PowerPC™ DClaim bus transaction, viz., any type of operation that is used to claim a memory block for a store (write) instruction whenever a valid (current) value for the subject block is already present in a cache of the processing unit which initiated the instruction. While the following description refers specifically to the DClaim operation, the present invention is equally applicable to similar operations which might be implemented with different cache instruction sets, so any reference to the “DClaim” operation should not be construed in a limiting sense. Also, as will become apparent, the present invention may be used at any cache level(s) in a multi-level cache architecture (L1, L2, L3, etc.), although the architecture need not be multi-level. Furthermore, the present invention may be used in multi-processor computer systems other than symmetric multi-processor (SMP) systems; for example, the invention could be applied to a non-uniform memory access (NUMA) system wherein the system memory (RAM) is divided among two or more memory arrays (having separate memory controllers connected to the system bus), and allocated among the processing units. As discussed in the Description of Related Art, conventional cache coherency protocols are somewhat inefficient in their handling of a DClaim-type operation since they often result in a significant delay in the completion of the DClaim store instruction while waiting for proper coherency responses. The present invention reduces the effects of these adverse results by providing a cache coherency protocol which ensures, in certain instances, that only one cache will persist in any attempt to execute a DClaim-type operation whenever two or more caches are issuing conflicting DClaim operations at approximately the same time. With reference now to FIG. 3, there is depicted a state diagram of one embodiment of a cache coherency protocol which may be used to carry out the present invention. This protocol is similar to the prior art MESI protocol of FIG. 2, in that it includes the same four states (Modified, Exclusive, Shared and Invalid), but it also includes a new “R” state (Recent), for providing an indication of the most recently referenced block that would otherwise have a Shared indication. In other words, when two or more caches each hold a valid copy of the same memory block, all of the copies will be in the Shared state except for the single copy in the processing unit which most recently accessed the value, and that copy will be in the Recent state. The R-MESI protocol is described more fully in U.S. patent application Ser. No. 08/839,548 entitled “Cache Coherency Protocol With Recently Read State for Data and Instructions,” filed on Apr. 14, 1997, which is hereby incorporated. By designating a single cache out of all of the other caches at the same level of the memory hierarchy that are concurrently sharing a particular value, the R-MESI protocol elevates that block to a special status, which can then be used for several different purposes. For instance, as discussed in the above-incorporated disclosure, the R-MESI protocol is especially useful in administering “intervention” of a shared value. Intervention allows a cache in a first processing unit to source the current value directly to another cache in a different processing unit (via the system bus, or an alternative interconnect), bypassing the much more protracted step of sourcing the value from system memory. The present invention further takes advantage of this attribute of the basic R-MESI protocol, by granting a higher priority to any block which is in the R state when a DClaim collision occurs. In other words, the R state can be used to ensure that only one cache (at most) will persist in any attempt to secure exclusive control of a previously shared memory address. A cache block in the R state can effectively obtain priority in any DClaim collision by designing the coherency protocol such that any processing unit having a cache block in the S state will revoke its own pending DClaim request for that block (i.e., convert its request into a RWITM operation) if any conflicting DClaim request from an R state block is snooped before the S state block has received null responses to its own DClaim, but a processing unit will not so revoke its pending DClaim request if it has the relevant block in the R state. It may still be necessary, however, to retry a DClaim request from an R state block in certain cases, such as when the snoop queue is full and the operation cannot be snooped. However, as noted further below, the actual coherency response from the S state snooper is irrelevant with respect to the CPU that resulted in the R block DClaim operation; the main requirement is the withdrawal of any DClaim operation attempted by a cache having the block Shared, if a conflicting DClaim operation is requested by a cache having the block Recent. In this manner, if a DClaim cross-invalidation is attempted, and one of the caches requesting the DClaim has a block with the current value in the R state, no deadlock occurs. The present invention avoids collision deadlocks (involving R state blocks) while still retaining the benefits of a DClaim-type operation. Such an operation never gets downgraded to a RWITM operation. It is possible that a DClaim collision could occur where the requesting caches have the addressed block only in the S state (there would be an R state cache block somewhere in the system, but it would just not be issuing a contemporaneous DClaim request). In such a case, the prior art collision avoidance technique can be used. Those skilled in the art will further appreciate that the coherency response of any cache having a block in the S state is often irrelevant to a DClaim request from a corresponding block in the R state block, regardless of whether such Shared caches are issuing conflicting DClaim requests. If there is a conflicting DClaim request, it is resolved as explained above. If there is no pending DClaim request at any Shared cache, then it would issue a null response anyway, and just invalidate its block. The only time an S state block must issue a retry is if its snoop queue is full, so that it can get the operation in the snoop queue later. Accordingly, a CPU store instruction on an R state block incurs no coherency response delay, which might otherwise be particularly significant if any caches with corresponding Shared blocks were busy and had to issue repeated retry messages. Such a DClaim request may be posted (e.g., placed in a cache operations queue) for eventual broadcast to the remainder of the memory hierarchy, but the DClaim store instruction can be completed immediately, which contributes to overall faster operation of the system. This feature is particularly beneficial with multi-level cache architectures, and also results in a significant improvement in the execution of atomic read-write operations which are accomplished using load-and-reserve instructions followed by associated conditional store instructions. This reduction of coherency response delays may be affected by other cache operations, such as the PowerPC™ “sync” operation which is used to synchronize threads running on different processors, by ensuring that the effects of all operations initiated by one processor have been completely propagated within all other processing elements in the system. A sync broadcast must be retried until after any DClaim from an R block is complete. Such a requirement does not apply to all implementations of the present invention, but it is necessary for this specific PowerPC™ storage model. In order to fully implement the foregoing, two different bus codes are provided for the two types of DClaim requests used in this embodiment, namely, the “posted” DClaim which is issued only for R state blocks, and the “regular” DClaim which is issued for S state blocks. If a given cache does not currently contain a valid copy of a value targeted by a DClaim snoop, then its snooper treats these two bus codes in exactly the same manner (for the specific cache coherency protocol depicted in FIG. 3, this situation arises only if the cache block is Invalid, so its snooper would transmit a null message in response to either type of DClaim request). Other coherency protocols may be devised in accordance with the present invention, having more than two states that are “shared” (in the broad sense that a cache block contains a current, valid copy of a value that is also present in a cache block of another processing unit). For any such coherency protocol, more than two types of DClaim requests (i.e., bus codes) may be provided in order to establish more than two different collision priority levels, further enhancing the efficiencies of the present invention. It can be seen that a cache block in the R state is effectively treated by its associated processor in the same manner as a cache block in the E state would be treated. For either state, the processor (or a higher-level cache) can immediately locate a valid copy of the current value in the lower cache, and the cache block is uniquely designated; that is, for a given address there can only be one cache block that is in the Recent state, or only one cache block that is in the Exclusive state. It is therefore possible to implement the present invention with an alternative coherency protocol that completely omits the Exclusive state. FIG. 4 is a state diagram depicting such an embodiment of an “MRSI” cache coherency protocol wherein the Recent state further functions as a substitute for the Exclusive state. In this MRSI embodiment, if the current value for a memory block is not located in any cache of the computer system (the value is only stored in system RAM), and a processor issues a read command for the address, then the value will be loaded into the processor's cache(s) and the block(s) labelled with the Recent state, rather than the Exclusive state. The only potential disadvantage in omitting the Exclusive state is that a cache will have to broadcast any modification of an R block, even if no other valid copies of the value are present in other caches; in this embodiment, the R state is ambiguous and so it must be presumed that other shared copies are in use. This broadcast would not be required if the block were in the E state. Nevertheless, in many multi-processor environments the E state would rarely exist due to heavy sharing of data, so the benefit gained by simplifying the coherency states can outweigh the requirement of broadcasting modifications even when the broadcast is unnecessary. Other protocols can be used with the present invention to designate a single cache block out of a plurality of shared blocks as having a DClaim collision priority. FIG. 5 is a state diagram depicting another embodiment of such a cache coherency protocol. This embodiment is also similar to the prior art MESI protocol of FIG. 2, but it additionally includes a “T” (Tagged) state to indicate that the corresponding value has been modified but not yet written out to system memory. The cache line holding the modified value in the T state is (for the moment, at least) responsible for ensuring that the modified value is eventually written to system memory. The T state is therefore similar to the M state, but the T state is distinguished from the M state to intervene a cache read operation without requiring the involvement of system memory, that is, without system memory reading the modified value during intervention (freeing up bandwidth in the memory controller). When a cache block in the T state sources a value, it then switches to the S state (or to one of many optional states which provide historical cache state information). The cache which receives the value holds it as the new T state cache block. Thus, “ownership” of the T state migrates in a manner similar to the R state. The Tagged state is described more fully in U.S. patent application Ser. No. 09/024,393 entitled “Cache Coherency Protocol with Tagged State for Modified Values,” filed contemporaneously with the present application, and which is hereby incorporated. As with the R state, for a given memory address there can only be one cache block in the system (horizontally) that is in the T state. Therefore, the foregoing method may be easily adapted to the T state, by still requiring a cache block in the S state (or one of the historical states) to revoke its own pending DClaim request in a collision, but not revoking a DClaim request for a T state block. Two different bus codes are again used for the two variations of the DClaim request, and a DClaim request for a T state block can similarly be posted. The present invention may further be implemented with a cache coherency protocol which combines elements of different protocols. An “RT-MESI” protocol, described in the aforementioned U.S. patent application Ser. No. 09/024,393 filed contemporaneously with the present application, includes both the R state and the T state as previously described. The same system bus code may be used for posted DClaim operations associated with both R and T state blocks. In this manner, the benefits of the DClaim operation are still achieved for collisions which involve either the R or T state. Referring now to FIG. 6, one embodiment 40 of a multi-processor computer system having a multi-level cache architecture is depicted which may be used to implement the present invention on both a global scale (system wide) and a local scale (CPU cluster). Multi-processor computer system 40 includes two CPU clusters 42 a and 42 b. CPU cluster 42 a has four CPU's 44 a, 44 b, 44 c, and 44 d, each having a processor core with on-board (L1) instruction and data caches, and an L2 cache. The L2 caches of these four CPUs 44 a, 44 b, 44 c, and 44 d are connected to a shared L3 cache 46 a, which is connected to the system memory (RAM) 48 via the generalized interconnect, or bus, 50. CPU cluster 42 b similarly has four CPU's 44 e, 44 f, 44 g, and 44 h, each also having a processor core with on-board (L1) instruction and data caches, and an L2 cache. The L2 caches of these four CPUs 44 e, 44 f, 44 g, and 44 h are connected to another shared L3 cache 46 b, which is again connected to memory 48 via bus 50. In this hierarchical variation of the R-MESI protocol, up to three corresponding cache lines can be found in the Recent state: one cache line among the L2 caches of CPUs 44 a, 44 b, 44 c, and 44 d; one cache line among the L2 caches of CPUs 44 e, 44 f, 44 g, and 44 h; and one cache line between the two L3 caches 46 a and 46 b. Consider the following example, wherein all of the corresponding cache lines in CPUs 44 a-44 h begin in the Invalid state. Processor 44 a executes a read operation, and so its cache line (L2) goes from an Invalid state to a Recent state; the corresponding cache line in L3 cache 46 a also goes from Invalid to Recent. Thereafter, processor 44 b requests a read of the cache line; processor 44 a intervenes and its cache line (L2) switches to the Shared state, while the cache line (L2) of processor 44 b goes from the Invalid state to the Recent state. The cache line in L3 cache 46 a remains Recent. Later, processor 44 e requests a read of the cache line; processor 44 b intervenes, but its cache line (L2) remains in the Recent state, since it is in a different CPU cluster from processor 44 e. The cache line (L2) in processor 44 e nonetheless switches from the Invalid state to the Recent state. Also, since the intervened value has passed through both L3 caches, the cache line in L3 cache 46 a switches from Recent to Shared, and the cache line in L3 cache 46 b switches from Invalid to Recent. Thereafter, if processor 44 f requests a read of the cache line, it can be sourced by the cache line (L2) of processor 44 e. In such a case, the cache line (L2) of processor 44 e switches from the Recent state to the Shared state, and the cache line (L2) of processor 44 f switches from the Invalid state to the Recent state. These steps are illustrated in Table 1: TABLE 1 L244a L244b L244e L244f L346a L346b Initial States I I I I I I P44a Read R I I I R I P44b Read S R I I R I P44e Read S R R I S R P44f Read S R S R S R In the last row of Table 1, each CPU cluster has one cache line in the “R” state, and one of the L3 caches has a line in the “R” state as well. Since a particular block can be held in the R state by more than one cache in a given horizontal level, this implementation of the R-MESI protocol requires further refinement to be able to designate only one of possibly many R state L2 blocks for collision priority, and thereby support the present invention. This refinement may come in the form of a special R state, hereinafter referred to as the global R state or “Rg.” This state is applied to only one block in a given level of the cache hierarchy, namely, to the most recently read block among all caches in the global system, not just with reference to the local CPU cluster. For example, in the embodiment of FIG. 6, only one RG block can be assigned among all of the L2 caches in both processing units 42 a and 42 b (this state is not used in the L3 caches of this embodiment since it is unnecessary). Table 2 illustrates how the RG state would apply for the same sequence of operations described for Table 1: TABLE 2 L244a L244b L244e L244f L346a L346b Initial States I I I I I I P44a Read RG I I I R I P44b Read S RG I I R I P44e Read S R RG I S R P44f Read S R S RG S R In such a global/local cache hierarchy, only the RG state can be posted (not the plain R state), for reasons similar to those described above with respect to the RT-MESI protocol. This concept can be extended to cache architectures having even more than three cache levels. It is also applicable to a global/local hierarchy using T-MESI instead of R-MESI (or using RT-MESI). As noted in the Description of Related Art, conventional cache coherency protocols are also inefficient in their handling of DClaim-type operations since they force the complete execution of all such operations (broadcast of the operation and evaluation of responses) even if those operations have become unnecessary or redundant. In the case where a value is cast out of a cache which just issued a DClaim request, a broadcast of the request becomes extraneous since the write operation will be broadcast anyway. FIG. 7 is a block diagram of one embodiment 51 of a memory hierarchy which may be used in accordance with the present invention to mitigate such inefficient handling of a DClaim-type broadcast. Memory hierarchy 51 includes a memory device 52 (e.g., RAM) connected to a system bus 54, and two caches 56 a and 56 b which are also connected to system bus 54. In the preferred embodiment, each cache 56 a, 56 b is identical. The memory hierarchy may further include additional caches if more than two processors are provided in the multi-processor computer system. Cache 56 a has a cache directory 58 a, a cache entry array 60 a, an LRU unit 62 a, and appropriate logic 64 a to write to the cache lines in directory 58 a and array 60 a, to detect cache hits, and to read stored values. The cache is connected to the processor, or a processor side of the memory hierarchy, by processor bus 66 a. Cache 56 a further has one or more queues, including at least one cache operations queue 68 a. Cache operations queue 68 a contains a serialized list of cache operations, each of which results in a bus transaction, i.e., an operation that must be broadcast to system bus 54 (or to a bus on the system memory side of the memory hierarchy if the cache is at a higher lever in the cache architecture). A queue controller 70 a includes appropriate logic to associatively compare current entries in queue 68 a to a new entry which is to be loaded in the queue. If the results of the comparison indicate that a current entry is redundant or unnecessary in light of a new entry, then the current entry is dynamically folded or collapsed into the new entry, i.e., the current entry is eliminated or deleted from queue 68 a without broadcasting its associated bus operation. Consider the following example, wherein a DClaim operation is first issued by a processor (or a higher level cache) associated with cache 56 a, in order to claim exclusive ownership of a particular memory block which is currently held in the R (or RG) state. This posted DClaim operation is placed in queue 68 a. Shortly thereafter (while the posted DClaim operation is still in the queue), the same processor (or higher level cache) issues one or more other instructions such that LRU unit 62 a selects for eviction the same cache block that is the subject of the pending Posted DClaim operation. In this case, the value will be written to system memory and the associated write operation will be entered in queue 68 a. Queue controller 70 a will associatively compare the address of the write operation with the previous entries in queue 68 a to determine if an earlier posted DClaim operation was issued for the same address. In this example, the result of the comparison will be positive, so queue controller 70 a will remove the posted DClaim operation from queue 68 a prior to its being broadcast, and the more recent operations in queue 68 a (including the write instruction) will all move up one place. The cache controller protects the coherency of the cache line which is the subject of a Dclaim operation. Once the target line of a posted DClaim is selected as a victim and the posted Dclaim operation is folded out, the cast-out control logic continues to protect the line from other access attempts (that is, retry associated bus transactions) until the cast-out/write-back to memory is completed. The cache controller begins protecting the line as soon as the posted Dclaim operation is placed in queue. Other caches could be in Shared states, so the cast-out operation takes over protecting the line. The dynamic folding of a pending cache operation into a new queue entry can significantly reduce address operations on system bus. From the cache controller's perspective, the collapsed operation was fully executed, but it required no latency. The advantages increase with increasing depth of the cache operations queue. In the depicted embodiment the cache operation queue is eight deep. The logic required for queue controller 68 a to make the comparison and remove posted DClaim requests is relatively simple, particularly in relation to the benefits achieved. Although the invention has been described with reference to specific embodiments, this description is not meant to be construed in a limiting sense. Various modifications of the disclosed embodiments, as well as alternative embodiments of the invention, will become apparent to persons skilled in the art upon reference to the description of the invention. For example, this protocol can be used with features from coherency protocols other than, or in addition to, those discussed above, including the “HRT-MESI” protocol, described in U.S. patent application Ser. No. 09/024,319 entitled “Cache Coherency Protocol Having Hovering (H), Recent (R) and Tagged (T) States,” filed contemporaneously with the present application, which combines a “Hover” state with the aforementioned R and T states. It is therefore contemplated that such modifications can be made without departing from the spirit or scope of the present invention as defined in the appended claims. Patent Citations Cited PatentFiling datePublication dateApplicantTitle US5627992 *May 4, 1995May 6, 1997Advanced Micro DevicesOrganization of an integrated cache unit for flexible usage in supporting microprocessor operations US5671391 *Sep 10, 1996Sep 23, 1997Ncr CorporationCoherent copyback protocol for multi-level cache memory systems US5713004 *May 18, 1995Jan 27, 1998Data General CorporationCache control for use in a multiprocessor to prevent data from ping-ponging between caches US5749095 *Jul 1, 1996May 5, 1998Sun Microsystems, Inc.Multiprocessing system configured to perform efficient write operations US5765196 *Feb 27, 1996Jun 9, 1998Sun Microsystems, Inc.System and method for servicing copyback requests in a multiprocessor system with a shared memory US5787490 *Jun 10, 1996Jul 28, 1998Fujitsu LimitedMultiprocess execution system that designates cache use priority based on process priority US5832276 *Oct 7, 1996Nov 3, 1998International Business Machines CorporationResolving processor and system bus address collision in a high-level cache US5903908 *Oct 15, 1996May 11, 1999Intel CorporationMethod and apparatus for maintaining cache coherency using a single controller for multiple cache memories US5946709 *Apr 14, 1997Aug 31, 1999International Business Machines CorporationShared intervention protocol for SMP bus using caches, snooping, tags and prioritizing Referenced by Citing PatentFiling datePublication dateApplicantTitle US6810457 *Jul 5, 2001Oct 26, 2004Nec CorporationParallel processing system in which use efficiency of CPU is improved and parallel processing method for the same US6944721Aug 8, 2002Sep 13, 2005International Business Machines CorporationAsynchronous non-blocking snoop invalidation US7000078 *Oct 1, 1999Feb 14, 2006Stmicroelectronics Ltd.System and method for maintaining cache coherency in a shared memory system US7200717 *Oct 14, 2004Apr 3, 2007International Business Machines CorporationProcessor, data processing system and method for synchronizing access to data in shared memory US7228389Dec 20, 2005Jun 5, 2007Stmicroelectronics, Ltd.System and method for maintaining cache coherency in a shared memory system US7366847Feb 6, 2006Apr 29, 2008Azul Systems, Inc.Distributed cache coherence at scalable requestor filter pipes that accumulate invalidation acknowledgements from other requestor filter pipes using ordering messages from central snoop tag US7536513 *Mar 31, 2005May 19, 2009International Business Machines CorporationData processing system, cache system and method for issuing a request on an interconnect fabric without reference to a lower level cache based upon a tagged cache state US8347035 *Jan 1, 2013Intel CorporationPosting weakly ordered transactions US8392668 *Mar 5, 2013Hewlett-Packard Development Company, L.P.Distributed-state-information-based distributed computing systems and methods and protocols for managing distributed state information US20020004868 *Jul 5, 2001Jan 10, 2002Nec CorporationParallel processing system in which use efficiency of CPU is improved and parallel processing method for the same US20040030843 *Aug 8, 2002Feb 12, 2004International Business Machines CorporationAsynchronous non-blocking snoop invalidation US20060085604 *Oct 14, 2004Apr 20, 2006International Business Machines CorporationProcessor, data processing system and method for synchronizing access to data in shared memory US20060155935 *Dec 20, 2005Jul 13, 2006Jones Andrew MSystem and method for maintaining cache coherency in a shared memory system US20060221720 *Apr 4, 2005Oct 5, 2006Reuter James MDistributed-state-information-based distributed computing systems and methods and protocols for managing distributed state information US20060224833 *Mar 31, 2005Oct 5, 2006International Business Machines CorporationData processing system, cache system and method for issuing a request on an interconnect fabric without reference to a lower level cache based upon a tagged cache state US20070186054 *Feb 6, 2006Aug 9, 2007Kruckemyer David ADistributed Cache Coherence at Scalable Requestor Filter Pipes that Accumulate Invalidation Acknowledgements from other Requestor Filter Pipes Using Ordering Messages from Central Snoop Tag US20100161907 *Dec 18, 2008Jun 24, 2010Santhanakrishnan Geeyarpuram NPosting weakly ordered transactions Classifications U.S. Classification711/141, 711/145, 711/158, 711/120, 711/146, 711/E12.033 International ClassificationG06F12/08 Cooperative ClassificationG06F12/0831 European ClassificationG06F12/08B4P4 Legal Events DateCodeEventDescription Feb 17, 1998ASAssignment Owner name: INTERNATIONAL BUSINESS MACHINES CORPORATION, NEW Y Free format text: ASSIGNMENT OF ASSIGNORS INTEREST;ASSIGNORS:ARIMILLI, RAVI K.;DODSON, JOHN S.;LEWIS, JERRY D.;REEL/FRAME:008988/0856 Effective date: 19971121 Jul 2, 2002CCCertificate of correction Jul 7, 2005FPAYFee payment Year of fee payment: 4 Sep 21, 2009REMIMaintenance fee reminder mailed Feb 12, 2010LAPSLapse for failure to pay maintenance fees Apr 6, 2010FPExpired due to failure to pay maintenance fee Effective date: 20100212
{ "url": "http://www.google.com/patents/US6347361?dq=U.S.+patent+number+7,325,728&ei=Y93TTteOAe702wW6uqi1BQ", "source_domain": "www.google.com", "snapshot_id": "crawl=CC-MAIN-2016-22", "warc_metadata": { "Content-Length": "129018", "Content-Type": "application/http; msgtype=response", "WARC-Block-Digest": "sha1:4PPUAMLKCHNVVJT5ZO63DI4MBCV27ZRV", "WARC-Concurrent-To": "<urn:uuid:5abdc008-fed5-42da-80b7-4daf2bba8470>", "WARC-Date": "2016-05-30T13:09:11Z", "WARC-IP-Address": "216.58.195.132", "WARC-Identified-Payload-Type": null, "WARC-Payload-Digest": "sha1:N2BAMHAQ3SIOUMUB2DSMMGGMFMXF3YEZ", "WARC-Record-ID": "<urn:uuid:24fdb023-5798-46f5-b61d-476406e0d51d>", "WARC-Target-URI": "http://www.google.com/patents/US6347361?dq=U.S.+patent+number+7,325,728&ei=Y93TTteOAe702wW6uqi1BQ", "WARC-Truncated": null, "WARC-Type": "response", "WARC-Warcinfo-ID": "<urn:uuid:5484c8a0-fbd0-4403-9e38-bf94709155d5>" }, "warc_info": "robots: classic\r\nhostname: ip-10-185-217-139.ec2.internal\r\nsoftware: Nutch 1.6 (CC)/CC WarcExport 1.0\r\nisPartOf: CC-MAIN-2016-22\r\noperator: CommonCrawl Admin\r\ndescription: Wide crawl of the web for May 2016\r\npublisher: CommonCrawl\r\nformat: WARC File Format 1.0\r\nconformsTo: http://bibnum.bnf.fr/WARC/WARC_ISO_28500_version1_latestdraft.pdf" }
{ "line_start_idx": [ 0, 56, 64, 205, 206, 214, 215, 243, 274, 296, 328, 357, 381, 407, 424, 530, 596, 657, 696, 747, 796, 810, 819, 1580, 1590, 1604, 1614, 1625, 1645, 1834, 2215, 2339, 2582, 2817, 3151, 3303, 3337, 3539, 3641, 3715, 3813, 3918, 4027, 4205, 4239, 4415, 4725, 4981, 5093, 5272, 5330, 5438, 5546, 5579, 5632, 5671, 5797, 7033, 7077, 7196, 7317, 7492, 7510, 7534, 7552, 7721, 7823, 8028, 8198, 8374, 8392, 8416, 8434, 8540, 8682, 8830, 9084, 9466, 9715, 9922, 10069, 10502, 10538, 10693, 10944, 11211, 11287, 11373, 11541, 11730, 11920, 11954, 11971, 12010, 12244, 13469, 13695, 13740, 13896, 14016, 14239, 14257, 14281, 14299, 14344, 14502, 14780, 14949, 14961, 14989, 14990, 15016, 15017, 15410, 15411, 15441, 15442, 16733, 16734, 17961, 17962, 18865, 18866, 19881, 19882, 20842, 20843, 21775, 21776, 22511, 22512, 24036, 24037, 24787, 24788, 25343, 25344, 27095, 27096, 27728, 27729, 28925, 28926, 30634, 30635, 31717, 31718, 32703, 32704, 33642, 33643, 34375, 34376, 35570, 35571, 35999, 36000, 36025, 36026, 36155, 36156, 36408, 36409, 36577, 36578, 39142, 39143, 39305, 39306, 39340, 39341, 39721, 39722, 39796, 39797, 39878, 39879, 40028, 40029, 40180, 40181, 40340, 40341, 40588, 40589, 40756, 40757, 40799, 40800, 41299, 41300, 42658, 42659, 43288, 43289, 44267, 44268, 45338, 45339, 46428, 46429, 47135, 47136, 48575, 48576, 49159, 49160, 50329, 50330, 51145, 51146, 52157, 52158, 53702, 53703, 54253, 54254, 54838, 54839, 56175, 56176, 57644, 57645, 57653, 57689, 57716, 57738, 57760, 57782, 57804, 57805, 58919, 58920, 58928, 58964, 58991, 59014, 59037, 59060, 59083, 59084, 59459, 59460, 60168, 60169, 60866, 60867, 61733, 61734, 62945, 62946, 63551, 63552, 64116, 64117, 65082, 65083, 65100, 65154, 65309, 65423, 65573, 65700, 65854, 65995, 66146, 66305, 66470, 66484, 66539, 66702, 66822, 66955, 67127, 67259, 67499, 67746, 67821, 68019, 68186, 68312, 68489, 68618, 68804, 69054, 69299, 69404, 69420, 69496, 69534, 69572, 69609, 69622, 69647, 69672, 69735, 69869, 69894, 69933, 69960, 69983, 70031, 70089, 70148 ], "line_end_idx": [ 56, 64, 205, 206, 214, 215, 243, 274, 296, 328, 357, 381, 407, 424, 530, 596, 657, 696, 747, 796, 810, 819, 1580, 1590, 1604, 1614, 1625, 1645, 1834, 2215, 2339, 2582, 2817, 3151, 3303, 3337, 3539, 3641, 3715, 3813, 3918, 4027, 4205, 4239, 4415, 4725, 4981, 5093, 5272, 5330, 5438, 5546, 5579, 5632, 5671, 5797, 7033, 7077, 7196, 7317, 7492, 7510, 7534, 7552, 7721, 7823, 8028, 8198, 8374, 8392, 8416, 8434, 8540, 8682, 8830, 9084, 9466, 9715, 9922, 10069, 10502, 10538, 10693, 10944, 11211, 11287, 11373, 11541, 11730, 11920, 11954, 11971, 12010, 12244, 13469, 13695, 13740, 13896, 14016, 14239, 14257, 14281, 14299, 14344, 14502, 14780, 14949, 14961, 14989, 14990, 15016, 15017, 15410, 15411, 15441, 15442, 16733, 16734, 17961, 17962, 18865, 18866, 19881, 19882, 20842, 20843, 21775, 21776, 22511, 22512, 24036, 24037, 24787, 24788, 25343, 25344, 27095, 27096, 27728, 27729, 28925, 28926, 30634, 30635, 31717, 31718, 32703, 32704, 33642, 33643, 34375, 34376, 35570, 35571, 35999, 36000, 36025, 36026, 36155, 36156, 36408, 36409, 36577, 36578, 39142, 39143, 39305, 39306, 39340, 39341, 39721, 39722, 39796, 39797, 39878, 39879, 40028, 40029, 40180, 40181, 40340, 40341, 40588, 40589, 40756, 40757, 40799, 40800, 41299, 41300, 42658, 42659, 43288, 43289, 44267, 44268, 45338, 45339, 46428, 46429, 47135, 47136, 48575, 48576, 49159, 49160, 50329, 50330, 51145, 51146, 52157, 52158, 53702, 53703, 54253, 54254, 54838, 54839, 56175, 56176, 57644, 57645, 57653, 57689, 57716, 57738, 57760, 57782, 57804, 57805, 58919, 58920, 58928, 58964, 58991, 59014, 59037, 59060, 59083, 59084, 59459, 59460, 60168, 60169, 60866, 60867, 61733, 61734, 62945, 62946, 63551, 63552, 64116, 64117, 65082, 65083, 65100, 65154, 65309, 65423, 65573, 65700, 65854, 65995, 66146, 66305, 66470, 66484, 66539, 66702, 66822, 66955, 67127, 67259, 67499, 67746, 67821, 68019, 68186, 68312, 68489, 68618, 68804, 69054, 69299, 69404, 69420, 69496, 69534, 69572, 69609, 69622, 69647, 69672, 69735, 69869, 69894, 69933, 69960, 69983, 70031, 70089, 70148, 70172 ] }
{ "red_pajama_v2": { "ccnet_original_length": 70172, "ccnet_original_nlines": 303, "rps_doc_curly_bracket": 0, "rps_doc_ldnoobw_words": 0, "rps_doc_lorem_ipsum": 0, "rps_doc_stop_word_fraction": 0.4202006459236145, "rps_doc_ut1_blacklist": 0, "rps_doc_frac_all_caps_words": 0.030323760583996773, "rps_doc_frac_lines_end_with_ellipsis": 0, "rps_doc_frac_no_alph_words": 0.13619090616703033, "rps_doc_frac_unique_words": 0.12462058663368225, "rps_doc_mean_word_length": 4.955945014953613, "rps_doc_num_sentences": 397, "rps_doc_symbol_to_word_ratio": 0, "rps_doc_unigram_entropy": 5.60441780090332, "rps_doc_word_count": 11531, "rps_doc_frac_chars_dupe_10grams": 0.18832135200500488, "rps_doc_frac_chars_dupe_5grams": 0.33655309677124023, "rps_doc_frac_chars_dupe_6grams": 0.28888654708862305, "rps_doc_frac_chars_dupe_7grams": 0.2545015513896942, "rps_doc_frac_chars_dupe_8grams": 0.23240065574645996, "rps_doc_frac_chars_dupe_9grams": 0.20604756474494934, "rps_doc_frac_chars_top_2gram": 0.011461669579148293, "rps_doc_frac_chars_top_3gram": 0.010306750424206257, "rps_doc_frac_chars_top_4gram": 0.005984569899737835, "rps_doc_books_importance": -6171.205078125, "rps_doc_books_importance_length_correction": -6171.205078125, "rps_doc_openwebtext_importance": -3370.84814453125, "rps_doc_openwebtext_importance_length_correction": -3370.84814453125, "rps_doc_wikipedia_importance": -2875.38623046875, "rps_doc_wikipedia_importance_length_correction": -2875.38623046875 }, "fasttext": { "dclm": 0.28278011083602905, "english": 0.9229327440261841, "fineweb_edu_approx": 2.017214775085449, "eai_general_math": 0.43372052907943726, "eai_open_web_math": 0.19713592529296875, "eai_web_code": 0.23077529668807983 } }
{ "free_decimal_correspondence": { "primary": { "code": "004.62", "labels": { "level_1": "General works, books and libraries, information sciences", "level_2": "", "level_3": "Computers and Computer science" } }, "secondary": { "code": "005.4", "labels": { "level_1": "General works, books and libraries, information sciences", "level_2": "", "level_3": "Computer programming" } } }, "bloom_cognitive_process": { "primary": { "code": "3", "label": "Apply" }, "secondary": { "code": "-1", "label": "Abstain" } }, "bloom_knowledge_domain": { "primary": { "code": "3", "label": "Procedural" }, "secondary": { "code": "-1", "label": "Abstain" } }, "document_type_v1": { "primary": { "code": "11", "label": "Legal/Regulatory" }, "secondary": { "code": "-1", "label": "Abstain" } }, "extraction_artifacts": { "primary": { "code": "3", "label": "Irrelevant Content" }, "secondary": { "code": "-1", "label": "Abstain" } }, "missing_content": { "primary": { "code": "4", "label": "Missing Images or Figures" }, "secondary": { "code": "-1", "label": "Abstain" } }, "document_type_v2": { "primary": { "code": "8", "label": "Documentation" }, "secondary": { "code": "11", "label": "Legal Notices" } }, "reasoning_depth": { "primary": { "code": "4", "label": "Advanced Reasoning" }, "secondary": { "code": "3", "label": "Intermediate Reasoning" } }, "technical_correctness": { "primary": { "code": "4", "label": "Highly Correct" }, "secondary": { "code": "3", "label": "Mostly Correct" } }, "education_level": { "primary": { "code": "4", "label": "Graduate/Expert Level" }, "secondary": { "code": "3", "label": "Undergraduate Level" } } }
672f1e42c33a7f9846924a2431ea77df
4,114,602,105,341,392,000
• <Página inicial Processor Benchmark Limitations Benchmark and performance tests ("benchmarks") measure different aspects of processor and/or system performance. While no single numerical measurement can completely describe the performance of a complex device like a microprocessor or a personal computer, benchmarks can be useful tools for comparing different components and systems. The only totally accurate way to measure the performance of your system, however, is to test the software applications you use on your computer system. Benchmark results published by Intel are measured on specific systems or components using specific hardware and software configurations, and any differences between those configurations (including software) and your configuration may very well make those results inapplicable to your component or system. Benchmarks can be divided into two kinds, component and system. Component benchmarks measure the performance of specific parts of a computer system, such as a microprocessor or hard disk drive, while system benchmarks typically measure the performance of the entire computer system. In either case, the performance you see in day to day use will almost certainly vary from benchmark performance, for a number of reasons. First, individual components must usually be tested in a complete computer system, and it is not always possible to eliminate the considerable effects that differences in system design and configuration will have on benchmark results. For instance, system vendors sell systems with a wide variety of disk capabilities and speeds, system memory, system bus features and video and graphics capabilities, all of which influence how the system components (such as the microprocessor) and the computer system perform in actual use and can dramatically affect benchmark results. Also, differences in software, including operating systems and compilers, will affect component and system performance. Finally, benchmark tests are typically written to be exemplary of only a certain type of computer application, which may or may not be similar to your applications. Benchmarks are, at most, only one kind of information that you may use during the purchasing process. To get a true picture of the performance of a component or system you are considering purchasing, you must consult other sources of information (such as performance information on the exact system you are considering purchasing). If you have any questions about the performance of any Intel microprocessor, please view the detailed performance briefs and reports published by Intel. SPECint and SPECfp benchmark tests reflect the performance of the microprocessor, memory architecture and compiler of a computer system on compute-intensive, 32-bit applications. SPEC benchmark tests results for Intel microprocessors are determined using particular, well-configured systems. These results may or may not reflect the relative performance of Intel microprocessor in systems with different hardware or software designs or configurations (including compilers). Buyers should consult other sources of information, including system benchmarks, to evaluate the performance of systems they are considering purchasing. For more information about SPEC, including a description of the systems used to obtain these test result, and other information about microprocessor and system performance and benchmarks, visit Intel's World Wide Web site at www.intel.com. Performance tests and ratings are measured using specific computer systems and/or components and reflect the approximate performance of Intel products as measured by those tests. Any difference in system hardware or software design or configuration, as well as system use patterns including wireless connectivity, may affect actual test results and ratings.   Software and workloads used in performance tests may have been optimized for performance only on Intel® microprocessors. Performance tests, such as SYSmark* and MobileMark*, are measured using specific computer systems, components, software, operations, and functions. Any change to any of those factors may cause the results to vary. You should consult other information and performance tests to assist you in fully evaluating your contemplated purchases, including the performance of that product when combined with other products. Additional information: 1 Informações de produto e desempenho open 1. O software e as workloads utilizados nos testes de desempenho foram otimizados apenas para desempenho em microprocessadores Intel. Os testes de desempenho como o SYSmark* e o MobileMark são medidos usando-se sistemas de computador, componentes, software, operações e funções específicos. Qualquer modificação nesses fatores pode provocar variação nos resultados. Consulte outras informações e outros testes de desempenho para ajudá-lo a avaliar melhor as suas compras, incluindo o desempenho desse produto quando combinado com outros produtos. Para obter mais informações, visite http://www.intel.com/performance. Os resultados foram medidos pela Intel com software, benchmark ou outros dados de terceiros que são fornecidos apenas para fins de informação. Qualquer diferença no design ou configuração do hardware ou software do sistema pode afetar o desempenho real. A Intel não controla ou audita o design ou a implementação dos dados de terceiros mencionados neste documento. A Intel incentiva todos os seus clientes a visitarem os websites de terceiros citados ou outras fontes para confirmar se os dados mencionados são precisos e refletem o desempenho dos sistemas disponíveis para compra.
{ "url": "http://www.intel.com.br/content/www/br/pt/benchmarks/resources-benchmark-limitations.html", "source_domain": "www.intel.com.br", "snapshot_id": "crawl=CC-MAIN-2014-41", "warc_metadata": { "Content-Length": "138312", "Content-Type": "application/http; msgtype=response", "WARC-Block-Digest": "sha1:365QGPOA6JYGU4FUNXOUZGP7X3HNXWEN", "WARC-Concurrent-To": "<urn:uuid:be834214-5f1f-4ace-ab60-c1f1402a827b>", "WARC-Date": "2014-09-24T04:25:49Z", "WARC-IP-Address": "23.0.160.33", "WARC-Identified-Payload-Type": null, "WARC-Payload-Digest": "sha1:HCQKB5HXHXRPQKZ6DQHELHHHDBU5ZYEL", "WARC-Record-ID": "<urn:uuid:f050537e-71e0-47b0-a3d5-38d1b323d6ac>", "WARC-Target-URI": "http://www.intel.com.br/content/www/br/pt/benchmarks/resources-benchmark-limitations.html", "WARC-Truncated": null, "WARC-Type": "response", "WARC-Warcinfo-ID": "<urn:uuid:b219930c-324d-480c-8ffc-4f1b183acd78>" }, "warc_info": "robots: classic\r\nhostname: ip-10-234-18-248.ec2.internal\r\nsoftware: Nutch 1.6 (CC)/CC WarcExport 1.0\r\nisPartOf: CC-MAIN-2014-41\r\noperator: CommonCrawl Admin\r\ndescription: Wide crawl of the web with URLs provided by Blekko for September 2014\r\npublisher: CommonCrawl\r\nformat: WARC File Format 1.0\r\nconformsTo: http://bibnum.bnf.fr/WARC/WARC_ISO_28500_version1_latestdraft.pdf" }
{ "line_start_idx": [ 0, 20, 21, 53, 54, 847, 848, 2127, 2128, 2613, 2614, 3481, 3482, 3840, 3841, 3843, 3844, 4378, 4379, 4405, 4406, 4442, 4443, 4448, 4449 ], "line_end_idx": [ 20, 21, 53, 54, 847, 848, 2127, 2128, 2613, 2614, 3481, 3482, 3840, 3841, 3843, 3844, 4378, 4379, 4405, 4406, 4442, 4443, 4448, 4449, 5647 ] }
{ "red_pajama_v2": { "ccnet_original_length": 5647, "ccnet_original_nlines": 24, "rps_doc_curly_bracket": 0, "rps_doc_ldnoobw_words": 0, "rps_doc_lorem_ipsum": 0, "rps_doc_stop_word_fraction": 0.3379679024219513, "rps_doc_ut1_blacklist": 0, "rps_doc_frac_all_caps_words": 0.005347589962184429, "rps_doc_frac_lines_end_with_ellipsis": 0, "rps_doc_frac_no_alph_words": 0.12085560709238052, "rps_doc_frac_unique_words": 0.39162561297416687, "rps_doc_mean_word_length": 5.83004903793335, "rps_doc_num_sentences": 39, "rps_doc_symbol_to_word_ratio": 0, "rps_doc_unigram_entropy": 5.205270767211914, "rps_doc_word_count": 812, "rps_doc_frac_chars_dupe_10grams": 0, "rps_doc_frac_chars_dupe_5grams": 0.04393747076392174, "rps_doc_frac_chars_dupe_6grams": 0.016476549208164215, "rps_doc_frac_chars_dupe_7grams": 0, "rps_doc_frac_chars_dupe_8grams": 0, "rps_doc_frac_chars_dupe_9grams": 0, "rps_doc_frac_chars_top_2gram": 0.030207009986042976, "rps_doc_frac_chars_top_3gram": 0.03041825070977211, "rps_doc_frac_chars_top_4gram": 0.014575409702956676, "rps_doc_books_importance": -380.33001708984375, "rps_doc_books_importance_length_correction": -380.33001708984375, "rps_doc_openwebtext_importance": -200.98431396484375, "rps_doc_openwebtext_importance_length_correction": -200.98431396484375, "rps_doc_wikipedia_importance": -127.05985260009766, "rps_doc_wikipedia_importance_length_correction": -127.05985260009766 }, "fasttext": { "dclm": 0.16090691089630127, "english": 0.7714897990226746, "fineweb_edu_approx": 2.6757612228393555, "eai_general_math": 0.24870896339416504, "eai_open_web_math": 0.1998918056488037, "eai_web_code": 0.4812620282173157 } }
{ "free_decimal_correspondence": { "primary": { "code": "004.16", "labels": { "level_1": "General works, books and libraries, information sciences", "level_2": "", "level_3": "Computers and Computer science" } }, "secondary": { "code": "004.17", "labels": { "level_1": "General works, books and libraries, information sciences", "level_2": "", "level_3": "Computers and Computer science" } } }, "bloom_cognitive_process": { "primary": { "code": "2", "label": "Understand" }, "secondary": { "code": "3", "label": "Apply" } }, "bloom_knowledge_domain": { "primary": { "code": "2", "label": "Conceptual" }, "secondary": { "code": "3", "label": "Procedural" } }, "document_type_v1": { "primary": { "code": "3", "label": "Reference/Encyclopedic/Educational" }, "secondary": { "code": "-1", "label": "Abstain" } }, "extraction_artifacts": { "primary": { "code": "3", "label": "Irrelevant Content" }, "secondary": { "code": "0", "label": "No Artifacts" } }, "missing_content": { "primary": { "code": "0", "label": "No missing content" }, "secondary": { "code": "-1", "label": "Abstain" } }, "document_type_v2": { "primary": { "code": "8", "label": "Documentation" }, "secondary": { "code": "10", "label": "Knowledge Article" } }, "reasoning_depth": { "primary": { "code": "2", "label": "Basic Reasoning" }, "secondary": { "code": "3", "label": "Intermediate Reasoning" } }, "technical_correctness": { "primary": { "code": "4", "label": "Highly Correct" }, "secondary": { "code": "3", "label": "Mostly Correct" } }, "education_level": { "primary": { "code": "2", "label": "High School Level" }, "secondary": { "code": "1", "label": "General Audience" } } }
672f1e42c33a7f9846924a2431ea77df
-4,834,037,622,933,577,000
Wednesday, December 18, 2019 Android Enterprise Dedicated Devices and SCEP Hello Everyone!  Recently SCEP certificate authentication was released for Intune with Android Enterprise devices. This means both COPE and Kiosk devices or whatever they are calling them these days. I just finished setting this up for a customer and let me tell you there were some challenges. I don't have any screenshots of the issues but I just want to run down a list of gotchas that we ran into to help you do the same. Once we had all other platforms working (iOS, Android Legacy, Android Work Profile) we thought Android Fully Managed would be a simple reconfig. It was not.  1.) Deploy the sub cert out with the root, this should always be done in my opinion. 2.) Make sure the devices have a Compliance Policy assigned. Our kiosk devices originally were marked as non compliant because we did not have one assigned as they were already so locked down (this is just the way the customer had their environment configured). We were seeing the SCEP, Root, and Sub certs stuck as 'Pending'. This went on for a day or so until we got Microsoft Support on the line who suggested the Compliance policy as a general fix. He eluded that this is something he does as a baseline because of, well, just Intune being Intune.  3.) I have done iOS kiosk devices in the past that are without user affinity and I have used the DNS attribute in the same name historically. You can not do that with Android from what I have found. The WiFi settings on the device itself will not recognize a certificate unless it has the UPN in the SAN name. It will never even attempt a connection if you give it a DNS SAN cert. 4.) This could be just coincidence but we supplied an external identity in our WiFi profile as well. We just used a generic name of Android Kiosk and once it actually authenticated the identity changed to {{serialnumber}}@domain.com like it was supposed to 5.) We had some issues with time outs attempting to fetch the SCEP certs and WiFi policies. We were able to solve this by syncing from both the Intune app and the built in Android Device Policy app. My running theory on this (and im sure I am going to butcher it) is that the Intune certificate connector doesn't look at any Google API syncs from the Device Policy app. So when you sync from there you receive the SCEP profile, you hit IIS, hit the connector, and then it just sits waiting for the Intune sync to validate and eventually times out. Moral of the story is to sync from both the Intune App and the Device Policy App. This is all also assuming you have a healthy SCEP and PKI infra underneath everything which can be a task itself! These are all just some thoughts from someone who has spent far too long poking at SCEP.  Send help in the form of miniature paints and Chipotle. Best of luck! 8 comments: 1. Having the same issue. I can push root cert to A4W but stays as pending for Kiosk (Dedicated) and Fully Managed builds. Driving me nuts :-( ReplyDelete 2. could you possibly provide more information around the parameters used in the Certificate Generation? What was used for the certificate SUBJECT? just CN={{Device_Serial}}? Did you create a device account in Active Directory for each device? What type of account was required? generic user account? ReplyDelete Replies 1. Hello Stephen, The certs are not tied to anything in local AD. You supply the paramaters in the Intune cert profile (subject name and SAN name). As far as what I put in the fields I normally use (or whatever it actually is) {{AzureADDeviceID}}@domain.com with the domain.com part being static. I will usually put the same info in the SAN field. Hopefully that helps some, if not let me know Delete 2. Thanks for the response, Sorry What I meant with the AD is the authentication that radius performs with AD from the received ID during the EAP authentication, i.e. if it exists in AD then radius approves connection. I have got things working with a fixed ID for EAP-TLS, but finding that if the outer ID and cert UPN differ it fails to AUTH as I want to use the {{AAD_Device_Id}}@xyz.com in my CERT UPN and this gets generated correctly, however connection fails when the outer ID differs from the UPN, as the outer ID fields wont allow dynamic generation of the name like the certificate profiles does using {{AAD_Device_Id}} Delete 3. The outer identity, in say a wifi profile, shouldnt have to match the values on a cert. A lot of times Ive just used [email protected]. To me it sounds like your radius is trying to do a check for some value or object that exists with internal AD and that just wont work. I usually tell my auth system, lets say CISCO ISE as an example, to check the cert and if this cert has the correct chain from my internal PKI and has a SAN name of DNS that ends in Domain.com (basically *@domain.com is what its checking) then to allow access. I am no network guy by any stretch of the imagination so I may be off base here. Delete 3. Thanks for the description, I have a ticket opened with Microsoft to resolve the outer auth requirement that I'm currently seeing. ReplyDelete 4. hi stephen, did u get any solution here as i face the same issue ReplyDelete Replies 1. Yes I did, the outer identity needs to be identical to that of the identity in the Device certificate UPN, this is how Microsoft works around an Android limitation (as told by support) only problem with this solution is the WIFI profile outer identity is not dynamic like the UPN in the device certificate so is fixed. Delete
{ "url": "https://www.amobileattempt.com/2019/12/android-enterprise-dedicated-devices.html", "source_domain": "www.amobileattempt.com", "snapshot_id": "crawl=CC-MAIN-2022-27", "warc_metadata": { "Content-Length": "81792", "Content-Type": "application/http; msgtype=response", "WARC-Block-Digest": "sha1:LO7YO6IFLK7CDNWJ22FGZ4GVL7LCS2U6", "WARC-Concurrent-To": "<urn:uuid:67794ac8-dfef-4477-8341-bb41f96f74d9>", "WARC-Date": "2022-07-03T08:43:24Z", "WARC-IP-Address": "172.253.63.121", "WARC-Identified-Payload-Type": "text/html", "WARC-Payload-Digest": "sha1:LEOWPRUOMNDHFIONFC3HZRWXOALD2BLF", "WARC-Record-ID": "<urn:uuid:af9e443d-5f9e-4615-9e8f-ae25dfec1336>", "WARC-Target-URI": "https://www.amobileattempt.com/2019/12/android-enterprise-dedicated-devices.html", "WARC-Truncated": null, "WARC-Type": "response", "WARC-Warcinfo-ID": "<urn:uuid:4c3ce562-f2cd-4a5a-b27b-e08e80947063>" }, "warc_info": "isPartOf: CC-MAIN-2022-27\r\npublisher: Common Crawl\r\ndescription: Wide crawl of the web for June/July 2022\r\noperator: Common Crawl Admin ([email protected])\r\nhostname: ip-10-67-67-153\r\nsoftware: Apache Nutch 1.18 (modified, https://github.com/commoncrawl/nutch/)\r\nrobots: checked via crawler-commons 1.3-SNAPSHOT (https://github.com/crawler-commons/crawler-commons)\r\nformat: WARC File Format 1.1\r\nconformsTo: https://iipc.github.io/warc-specifications/specifications/warc-format/warc-1.1/" }
{ "line_start_idx": [ 0, 29, 30, 76, 77, 94, 95, 278, 279, 663, 664, 749, 750, 1303, 1304, 1685, 1686, 1943, 1944, 2574, 2575, 2689, 2690, 2780, 2781, 2837, 2838, 2852, 2853, 2854, 2866, 2867, 3012, 3013, 3029, 3206, 3207, 3337, 3338, 3354, 3366, 3388, 3389, 3525, 3526, 3732, 3733, 3785, 3786, 3799, 4433, 4434, 4447, 4591, 4592, 4995, 4996, 5083, 5084, 5097, 5233, 5234, 5250, 5320, 5321, 5337, 5349, 5675, 5676 ], "line_end_idx": [ 29, 30, 76, 77, 94, 95, 278, 279, 663, 664, 749, 750, 1303, 1304, 1685, 1686, 1943, 1944, 2574, 2575, 2689, 2690, 2780, 2781, 2837, 2838, 2852, 2853, 2854, 2866, 2867, 3012, 3013, 3029, 3206, 3207, 3337, 3338, 3354, 3366, 3388, 3389, 3525, 3526, 3732, 3733, 3785, 3786, 3799, 4433, 4434, 4447, 4591, 4592, 4995, 4996, 5083, 5084, 5097, 5233, 5234, 5250, 5320, 5321, 5337, 5349, 5675, 5676, 5688 ] }
{ "red_pajama_v2": { "ccnet_original_length": 5688, "ccnet_original_nlines": 68, "rps_doc_curly_bracket": 0.0035161699634045362, "rps_doc_ldnoobw_words": 0, "rps_doc_lorem_ipsum": 0, "rps_doc_stop_word_fraction": 0.47110334038734436, "rps_doc_ut1_blacklist": 0, "rps_doc_frac_all_caps_words": 0.05954466015100479, "rps_doc_frac_lines_end_with_ellipsis": 0, "rps_doc_frac_no_alph_words": 0.12609456479549408, "rps_doc_frac_unique_words": 0.39017051458358765, "rps_doc_mean_word_length": 4.405215740203857, "rps_doc_num_sentences": 70, "rps_doc_symbol_to_word_ratio": 0, "rps_doc_unigram_entropy": 5.362374305725098, "rps_doc_word_count": 997, "rps_doc_frac_chars_dupe_10grams": 0, "rps_doc_frac_chars_dupe_5grams": 0.011839710175991058, "rps_doc_frac_chars_dupe_6grams": 0.011839710175991058, "rps_doc_frac_chars_dupe_7grams": 0.011839710175991058, "rps_doc_frac_chars_dupe_8grams": 0, "rps_doc_frac_chars_dupe_9grams": 0, "rps_doc_frac_chars_top_2gram": 0.01138434000313282, "rps_doc_frac_chars_top_3gram": 0.01024589966982603, "rps_doc_frac_chars_top_4gram": 0.00500911008566618, "rps_doc_books_importance": -441.53631591796875, "rps_doc_books_importance_length_correction": -441.53631591796875, "rps_doc_openwebtext_importance": -246.55760192871094, "rps_doc_openwebtext_importance_length_correction": -246.55760192871094, "rps_doc_wikipedia_importance": -184.89549255371094, "rps_doc_wikipedia_importance_length_correction": -184.89549255371094 }, "fasttext": { "dclm": 0.22819137573242188, "english": 0.9609840512275696, "fineweb_edu_approx": 1.2743175029754639, "eai_general_math": 0.3016761541366577, "eai_open_web_math": 0.45393502712249756, "eai_web_code": 0.4059374928474426 } }
{ "free_decimal_correspondence": { "primary": { "code": "004.0285636", "labels": { "level_1": "General works, books and libraries, information sciences", "level_2": "", "level_3": "Computers and Computer science" } }, "secondary": { "code": "005.457", "labels": { "level_1": "General works, books and libraries, information sciences", "level_2": "", "level_3": "Computer programming" } } }, "bloom_cognitive_process": { "primary": { "code": "3", "label": "Apply" }, "secondary": { "code": "2", "label": "Understand" } }, "bloom_knowledge_domain": { "primary": { "code": "3", "label": "Procedural" }, "secondary": { "code": "2", "label": "Conceptual" } }, "document_type_v1": { "primary": { "code": "3", "label": "Reference/Encyclopedic/Educational" }, "secondary": { "code": "-1", "label": "Abstain" } }, "extraction_artifacts": { "primary": { "code": "0", "label": "No Artifacts" }, "secondary": { "code": "-1", "label": "Abstain" } }, "missing_content": { "primary": { "code": "0", "label": "No missing content" }, "secondary": { "code": "-1", "label": "Abstain" } }, "document_type_v2": { "primary": { "code": "16", "label": "Personal Blog" }, "secondary": { "code": "21", "label": "Customer Support" } }, "reasoning_depth": { "primary": { "code": "3", "label": "Intermediate Reasoning" }, "secondary": { "code": "2", "label": "Basic Reasoning" } }, "technical_correctness": { "primary": { "code": "3", "label": "Mostly Correct" }, "secondary": { "code": "4", "label": "Highly Correct" } }, "education_level": { "primary": { "code": "3", "label": "Undergraduate Level" }, "secondary": { "code": "4", "label": "Graduate/Expert Level" } } }
672f1e42c33a7f9846924a2431ea77df
2,782,441,625,472,569,000
["On the other hand, consider a situation when we cannot just discard a message, it could be a transaction in the banking domain for example.","We are going to work through a couple of those features, but at the end of this lab you should feel comfortable with some of the basics.","How many attempts should be made to reconnect after failure.","For Time dependency and Batch, its order id.","My question is how to access the messages in dead letter queue and perform retry on the same by consuming it in the consumer and send to a webservice or socket etc.","Verify that the test queue and DLQ are empty.","The JMS client route handles messages one by one and not in parallel.","Reuse the same transaction used to read the message from dlq to enqueue the message to the application queue.","This should be greater than the ping period.","Looks like the example is incomplete.","Now does anyone know if this can be added to my activemq.","Solution Architect at Pure Storage.","Here are the common uses of Markdown.","This is where the error handling in Camel comes into play now.","How often to check for slow consumers on a particular queue.","Thus, although the consumer can continue to retrieve ordered messages from another message group, the first message group remains unavailable until the message blocking the queue is processed successfully.","DLQ back to the original queue at intervals and in bulk.","He is specialized in web application development with experience in both frontend and backend.","You must set the web.","Such a message goes back to its queue ready to be redelivered.","If delivery failed because the target queue quota was reached, you can reattempt delivery in the hope that the quota problem was resolved.","Next define th following properties in application.","For this, Camel offers us the following configuration, where the stack trace of the last failed delivery will be logged to the console, right before the message gets moved to the DLQ.","JTA implementation from the server.","We shall park the incoming data in a property.","It also process all passwords using the new defined decoder.","Link copied to clipboard!","Next, I indicate that the listener will be executed within a transaction.","Join Sterling Supply Chain Academy, a digital learning platform to help you acquire knowledge and best practices.","Table of Contents will stay that way until you close it.","Do circuit breakers trip on total or real power?","Spring JMS documentation and have not much details in that.","Where do the consumers even live?","Out of these, the cookies that are categorized as necessary are stored on your browser as they are essential for the working of basic functionalities of the website.","This usually poses no security issues as those files should be well protected from unauthorized accessing.","CONSUMER_SLOW management notification which an application could receive and take action with.","Any code example would be great.","Below is a list of response headers you should expect when interacting with a Queue resource.","What will the messaging system do with a message it cannot deliver?","In the server configuration, Connectors and Acceptors sometimes needs to specify passwords.","So it cannot raise the exception to the broker nor can we put the message back on top of the queue.","When you use Apache Camel routes for your integration, when a failure occurs, a classic pattern is to retry the message.","In this case since we have a very short timeline, we had to use camel routes which seems to work for our use case.","Once the messages have been unsuccessfully delivered for a given number of attempts, they are removed from their queue and sent to the relevant dead letter address.","Scripting appears to be disabled or not supported for your browser.","If you are not logged in please log in and try again.","The MAIN processor throws several different types of exception.","All Queues have file stores.","Ozan: If you take Camel down while it processes msgs, then yes the msg would be lost.","Just do the next right thing.","More importantly, this helps to prevent infinite loops in queues that are constantly processing bad messages and degrading system performance.","If you are interested in sharing your experience with an IBM research and design team, please follow the button below to fill out a short recruitment survey.","Or that some configuration you changed is working as expcted?","Each message listener invocation will then operate within an active JMS transaction, with message reception rolled back in case of listener execution failure.","And finally, the client code.","Bean Utils and the configuration would be replaced.","When the messaging system moves the message, it may also record the original channel the message was supposed to be delivered on.","Do temp queues not work with bridging AMQ and Tibco?","Therefore, if one message is delayed, no other message is read from the JMS queue.","All other passwords in the configuration file, Connectors, Acceptors and Bridges, will also use masked passwords.","If true uses CORE queue naming convention for AMQP.","Create the transacted MSMQ queue for storing dead message if necessary.","But opting out of some of these cookies may affect your browsing experience.","Releasing the locks results in the removal of the log files.","What is a Dead Letter Queue?","The default action for an expired message is, to send it to the DLQ.","Unfortunately, those messages might be required by the target application.","JMS queue keeps the request message and when the service starts up it delivers the message to the service and get back the response.","This Account has been suspended.","With Spring Integration, it is the way to enable rolling back of messages using transactions.","It looks like nothing was found at this location.","AMQ dynamically handle the bridge for this scenario?","With this configuration, both passwords in ra.","The document is sent through a webservice provider deployed in the bus and serving as a route endpoint.","What is the maximum interval after which retry should not be performed.","Once there are this number of backup journal files the server will stop permanently after if fails back.","This project is read only now.","DTO class to encapsulate messages along with their destinations and timestamps.","The expiration of a message is always based on its original enqueue timestamp.","Message header field keys start with the letters JMS and ignore the key type setting.","This website uses cookies and other tracking technology to analyse traffic, personalise ads and learn how we can improve the experience for our visitors and customers.","JMS topics once the last subscription on the topic has been closed.","The percentage of live data on which we consider compacting the journal.","Just got to my own thread looking for something.","The JMX operations shutdown on the Camel context and the Camel route work that way.","No search has been performed.","Is it dangerous to use a gas range for heating?","In most messaging use cases, the messages are consumed, acknowledged and forgotten as soon as they are consumed.","Queues forward messages to one consumer at a time.","Allows for a Message that was already processed by a DLQ to be rolled back in case of a move or a retry of that message, otherwise the Message would be considered a duplicate if this strategy is doing Message Auditing.","Next, we create our message and send it.","Percentage of available memory which will trigger a warning log.","The max percentage of data we should use from disks.","As soon as the flow is deployed, it starts polling the queue for messages.","Neither for that matter will JUnit, which makes testing your transaction config problematic.","QUEUE on the Tibco broker.","For the remainder of this section, please verify that your broker is running and able to take connections.","Search in all products.","Helpful answers to award.","Hassen is totally right regarding the possibility to set up a MDB consuming DLQ entries.","Please check the URL or go back a page.","Also, I want to load any poison messages to DLQ.","XA transactions also have an impact on performance and might not always be needed.","This topic has been locked.","Help pages for instructions.","It supports industry standard protocols so users get the benefits of client choices across a broad range of languages and platforms.","Camel route, then you have to deal with that, the message is already removed from the queue.","Setup a broker instead of using the org.","The broker resends the message.","Two input parameter as string value.","You have removed this from featured content.","If the console is configured and running, you can use it to purge the DLQ by releasing the active locks in the kahadb log files.","Let us know what you found helpful.","But my poison messages are not getting into DLQ.","The compensation logic depends on the cause of the failure.","Optionally users can use or implement their own encoder and decoder for masking the passwords.","We now see one entry in the queue.","Can you think of a solution for this problem?","Assuming that a message is present in the fund.","As we can see from the output, after several failed attempts, the message was sent to the Parking Lot Queue.","The amount in bytes before all addresses are considered full.","Schedules the respective time event for execution.","How Messages are moved to a Dead Letter Queue?","Every system needs a specific error handling strategy.","Kkanakamma was able to get the JMX metrics reporting by implementing JMX instrumentation as below documentation detailed.","Fund Transfer to a different bank.","Each of the next sections assumes this is the case.","Open the IBM MQ Explorer.","You will want to configure your connection factory appropriately.","In Apache Camel, this redelivery policy is configured in the error handler.","Can you please review and advise if any error or mistakes in my code.","FIFO queues allow a lower number of inflight messages.","Getting The Access Token And Using it to Fetch Data.","Helpful answer to award.","Dequeued messages are not designed can be seen in activemq, you need your own logic to save them elsewhere.","Use of Mozilla Firefox is recommended.","It is important not to rely on the scheme of the URLs returned within these headers as they are an implementation detail.","The high level overview of all the articles on the site.","The syntax of each task remains as described below.","This could be useful for stock price updates where we really only care about the most recent updates.","Please enter a title.","What can we do to improve the content?","Of course, the reason we moved them to the Dead Letter Queue is so they can be reprocessed at another time.","This tutorial is explained in the below Youtube Video.","If specified, the server uses the named channel for discovery.","Timeout on waitin for clients to disconnect before server shutdown.","If you are a new customer, register now for access to product evaluations and purchasing capabilities.","This item is now featured!","Would you mind sharing what did you do to monitor DLQ?","This topic has been unlocked.","If I throw the exception from Catch Block, the message is going to DLQ after respective retries.","Messaging is typically used to decouple producers and consumers in two main dimensions: time, and space.","The best stories sent monthly to your email.","Basically it checks within the indicated packages to autodetect the application beans: producer and consumer.","Batching Errors: If the batching is configured, database Notifications are batched and executed and committed in batch.","AWS SDK for Java.","Determine whether you have given your consumer sufficient time to process messages.","JTA is the tool for the job.","This will cause every invocation of the listener to be executed within a local JMS transaction.","It is possible to delay messages redelivery.","Your email address will not be published.","Then, create the Main class.","Should stop backup on live restart.","Software Engineer working on security at Utility Warehouse.","The important setting is the concurrency element which limits the number of threads that are can simultaneously handle messages on that destination.","More other, in term of performance, we might want to still get the exchanges going through.","Pull consumers and pull subscriptions can time out.","Comments are closed on this article!","Dead Letter Queue behavior is closely related to the way messages are delivered.","The messages are looked through and forwarded to stuff responsible to process data changes in App.","API and is not meant to have such automation.","For a stream receiving messages via an input queue I want to be able to set message priority.","How we send messages to the broker will depend on what programming language and API you wish to use.","Neo in the movie The Matrix?","Find a Redbook, check out IBM Developer for technical insight, improve your skills, or go to IBM Support.","Consequently, by default, we will see an infinite number of such messages in the output.","But no message is logged.","We will now store the incoming message into a JMS queue.","CLI command as an example.","Could you provide the full example.","MQ Appliance firmware version.","For example, in the case of authentication failure, you can correct the certificate attached with the message and resend the message.","IBM KC did not find an exactly matching topic in that version.","DLQ by changing their expiration.","Start the Producer and Consumer applications.","If input is empty string; it will list out all the pending messages on all Orchestrator related dead queues.","XML document to a URL.","This code is for Internal Salesforce use only, and subject to change without notice.","However, sometimes we may need to catch all errors, for example for logging or persisting them to the database.","HCL will acquire select IBM collaboration, commerce, digital experience and security software products.","To work around this issue, you have to purge the DLQ.","The failed delivery can be caused by reasons such as network failures, a deleted queue, a full queue, authentication failure, or a failure to deliver on time.","Proof read your work.","Allowed Values are TIME_DEPENDENCY and BATCH.","Atomicity is easily achieved.","The string tokenizer class allows an application to break a string into tokens.","You can not post a blank message.","It has both encoding and decoding capabilities.","Make sure to check the Persistent Delivery.","The specified item was not found.","Red Hat and contributing to various Open Source projects.","This is a directory where push registrations for queues are stored.","This is to help you customize artemis on embedded systems.","Message is rejected by another queue exchange.","All other interactions remain the same as discussed earlier.","Class name and its parameters for the Decoder used to decode the masked password.","Xavier is a software developer working in a consulting firm based in Barcelona.","In these cases, you need to make use of an external JTA implementation, such as Atomikos or JTOM.","The systems requirements links off this site are no longer active on IBM.","My friend got the whole code, please can Share?","Every week, every fortnight, whatever.","Create Queue named javainuse and dlq.","But this behavior is not always desirable.","When this count reaches a configured limit, the message is placed in a designated Dead Letter Queue.","The only problem left is the unlimited option.","As you can see, a single queue is created.","Try to use images to augment the text, but you obviously must own the copyright.","Nothing to see here!","Check the configured inbox.","Wait for the message to expire.","Timestamp when record was last updated.","There are a number of different strategies here, and of course the best way to handle this comes down to your requirements.","Search in this product.","The encoder takes in the real password and outputs the masked version.","However, if the delivery count is not updated persistently before the message delivery happens, in the event of a server crash, messages might have been delivered but that will not have been reflected in the delivery count.","Other portions of the text, such as text chapters or the full pattern text, are protected by copyright.","It will do the polling on the destination and recover in case of failures of the broker, etc.","Save my name, email, and website in this browser for the next time I comment.","When reading from a queue we might not have any messages.","Maximum number of threads to use for the scheduled thread pool.","URL you can post a message to.","Timer Events for Time dependent plan items fails after max retries: Time Events that fail after the max number of retries will be moved to dead letter table in database.","DLQ only when respective exception occurs.","Was this content helpful?","Would you try this out and see how it works for you?","It is possible to use System properties to replace some of the configuration properties.","CPU and network resources.","Also, please advise some good examples to accomplish this task.","Increase visibility into IT operations to detect and resolve technical issues before they impact your business.","Displays list of all dead letters according to the input parameter.","We use cookies on our website to give you the most relevant experience by remembering your preferences and repeat visits.","Below is a list of response headers you should expect when interacting with a Topic resource.","Javascript is disabled or is unavailable in your browser.","Currently as per my understanding, any exceptions thrown from Message Listeners are moved to DLQ after specified retry parameters.","The user can replay it by moving the dead letter queue messages back to source queue events or by using JMX operations provided to move them to source queue.","Your browser is not supported.","Moving message from one queue to another using Apache.","Opinions expressed by DZone contributors are their own.","Display the help messages for the activemq script.","This post will show you how an error in the execution of the consumer during the asynchronous reception of messages with JMS, can lead to the loss of messages.","The answer depends on specific system requirements, the kind of the exception or type of the message.","This specifies how many times a replicated backup server can restart after moving its files on start.","However, you need a full JEE server for this.","Tomcat for example, does not come with JTA.","If RESTEasy is not installed within your environment, you must add the RESTEasy jar files within the lib directory as well.","Error occurred after processing the message: The consumer retrieves the message, stores it to the DB, and then the execution fails.","In this step, we assume that an exception has happened.","This can be still triggered from JConsole using JMX interface.","Later on we can analyze the messages in the DLQ to know the reason why the messages are failing.","Thus, to maintain the efficiency of your queue, make sure that your application correctly handles message processing.","The latest news, tips, and insights from the world of New Relic and digital intelligence.","In this example, the delay increases for each attempt.","No number of retries can help process the message successfully without changing the incorrect data.","You are no longer logged in.","May we contact you about your feedback?","Default time to live for posted messages.","Any Timer events on those orders before manually executing these requests are moved to DB dead letter queue.","It is mandatory to procure user consent prior to running these cookies on your website.","Based on this configuration, each Dead Letter Queue will now have a prefix of DLQ.","After which it will be sent to dead letter Queue.","Creating charts in GWT was never so easy.","Please let me know if I can acheive these through Spring JMS.","This website uses cookies to improve your experience while you navigate through the website.","DLQ should fix the issue.","Hi, is there any way in which I can see the messages from a queue at any moment?","But what if you just want to quickly test whether the broker is up?","When we restart the broker, we exepct our messages to still be on the broker and availabe for consumption.","Any exceptions thrown will now cause the broker to redeliver the message according to the redelivery policy specified in the connection URI, and once that is exceeded to send the offending message to the dead letter queue.","Welcome to the Talend Community!","This option controls whether passwords in server configuration need be masked.","Maybe I need not a change in Java code but a different point of view on the subject?","Sorry, your blog cannot share posts by email.","Any pointers on this will be appreciated.","If the exception recurs, the message will not be deleted from the queue.","Want to see me live?","You must enter at least one character.","XD when using a rabbit transport.","Hope this clarifies things a bit more.","Click on the element name to go to the specific chapter.","By default, all failed messages will be immediately requeued at the head of the target queue over and over again.","The question is how to implement it using Camel.","IBM Sterling CPQ transforms and automates configuration, pricing, and quoting of complex products and services.","Returns the number of notifications stored to the DB.","This section is a quick index for looking up configuration.","The transaction will start when the message is received.","DLX is a normal exchange.","The setting located here: In the right side of hawtio application there is your user picture with a small arrow.","This is great for most use cases but not the best choice for every scenario.","Then, the message dispatcher.","Active MQ server offers custom DLQ configuration.","Java world, as well as insights about Android, Scala, Groovy and other related technologies.","Maximum number of threads to use for the thread pool.","Service class that implements the service contract.","Limit before message is considered large.","Consuming messages from a queue or topic looks very similar.","This example configures the dead letter channel with a custom redelivery policy.","In this blog, I will illustrate a possible implementation of a persistent redelivery policy with backoff support.","This article has been made free for everyone, thanks to Medium Members.","The max number of concurrent reads allowed on paging.","Why do guitarists specialize on particular techniques?","An internal error occurred while following the object.","Date meta tag, same as dcterms.","Buying a house with my new partner as Tenants in common.","For software releases that are not yet generally available, the Fixed Release is the software release in which the problem is planned to be fixed.","Over a million developers have joined DZone.","If your processing is not idempotent or if your application does not support duplicate message detection, then you will have to use distributed transactions.","Ramblings on pretty much anything technical.","Your browser does not support the video tag.","This is a directory where push registrations for topics are stored.","Click on a version in the dropdown to find the same page in that version of the product if available, or select a different product.","If specified, the server uses JGroups for discovery.","You can use the Message Driven POJO infrastructure provided by the Spring framework.","The connections grow for as many retries as allowed and do not actually close until the broker is restarted, but that is another issue.","Name of a JGroups configuration file.","Vinicius: The cost is the redelivery of your failed msg are few times before moving to DLQ.","Note: In the following examples if related attributed or properties are absent, it means they are not specified in the configure file.","On a side note, consuming dead letter queues is basically the same as consuming any other queues.","HTML code to begin with!","You should not have received any mails yet.","Last but not least, once you get your own decoder, please add it to the classpath.","But what does this look like?","Whether a posted message should be persisted by default if the user does not specify a durable query parameter.","Period the discovery group waits after receiving the last broadcast from a particular server before removing that servers connector pair entry from its list.","Adding local transactions to the message reception avoids losing messages.","JMS that makes development easier because all connection, session, and JNDI contexts are all abstracted away.","Net, and more is available.","This is a URL for creating subscribers that will pull from a topic.","Country meta tag, same as geo.","Finally getting to the root of the problem: I downloaded the source and added some additional stack trace printouts.","Please read and accept our website Terms and Privacy Policy to post a comment.","It is also possible to configure a dead letter address so that after a specified number of unsuccessful deliveries, messages are removed from their queue and sent to the dead letter address.","Message length limit exceeded.","URI schemes are published to the outside world.","String, and send to a JMS queue.","So what happened here?","Number of retries before trying another node.","This might be helpful for preventing queues to grow infinitely.","The default and dead letter error handlers supports such policy.","Replace the SOAP sender with the JMS sender: this is because, we now want the integration flow to poll the JMS queue, not expose a SOAP endpoint.","There are a handful of nice examples here.","You must also have a broker.","Get MSMQ queue name from appsettings in configuration.","Copyright The Closure Library Authors.","Btw, its possible to shutdown the Camel Context or an individual Camel route gracefully so that it waits for outstanding requests to be completed before shutting down.","To use the AWS Documentation, Javascript must be enabled.","Name of a JGroups Channel.","Any JMS events on those orders before manually executing these requests are moved to JMS dead letter queue.","This also records what machine the message died on.","Why does catting a symlinked file and redirecting the output to the original file make the latter file empty?","Now we can save the failed message to the database or perhaps send an email notification.","There is also a connection leak involved.","This category only includes cookies that ensures basic functionalities and security features of the website.","This is not always optimal.","Red Hat build of Eclipse Vert.","However when finally giving up on the retries, the message would still be discarded and lost.","Following JMX operations are exposed to the user for viewing and executing the dead letter queue.","Listener, the poison messages are not getting queued there.","Will you be able to help us with this.","The message sort key or the name of a message header field on which to sort.","Here, expert and undiscovered voices alike dive into the heart of any topic and bring new ideas to the surface.","The URL for the connector.","If the transaction fails, each notification from the batch is executed separately and committed in the database.","Platform transaction manager with Policy as PROPAGATION_REQUIRED.","Try to stick to a schedule if you can.","Displays list of messages pending on respective dead queue provided as input.","This gives the client some time to recover from any transient failures and to prevent overloading its network or CPU resources.","However, this means it is possible for a message to be delivered again and again without success thus remaining in the queue indefinitely, clogging the system.","To get a mask for a password, just run the main class at org.","Red Hat services, please be sure to log out.","By default, when you create a Camel route, Camel automatically enables the default error handler.","Please make sure that your session did not expire while viewing this page.","This indicates that the flow did not complete processing.","Further as the Camel route involves two different JMS endpoints, you would need to configure Camel for XA transactions, involving an XA transaction monitor.","The entire Camel route above could therefore spawn a single transaction.","The other day I felt a compelling need to implement a JMS redelivery scenario.","Queued messages can fail delivery.","Cluas you are correct i had to use the same transaction manager across.","There is no configuration.","So the message is basically lost.","JCGs serve the Java, SOA, Agile and Telecom communities with daily news written by domain experts, articles, tutorials, reviews, announcements, code snippets and open source projects.","User can execute them manually using JMX console.","If no address is defined here then such messages will simply be discarded.","No problem here, the queue is empty since the message has been correctly received and stored to the database.","There are several solutions to achieve this.","It can be an empty string.","That number of sessions will be created at startup time.","We sent a message to a queue and got a response.","Configure various components of the Configure, Price, Quote system.","Necessary cookies are absolutely essential for the website to function properly.","Retry for custom exceptions.","Your Red Hat account gives you access to your profile, preferences, and services, depending on your status.","Click on the design Palette.","When will a consumer see messages?","With partitioned destinations, there is one DLQ for all partitions and we determine the original queue from the headers.","Sending and reception of a message: The consumer will process the received message, storing it to a database.","Host the service within this EXE console application.","You must also add an additional listener to your web.","That means Camel will receive the message from the broker and ack it straight away, before routing the message any further.","Want to read more in depth?","Debugging this locally using an embedded AMQ broker and a local Tibco instance I was able to reproduce the issue.","This can be viewed using JMS Clients.","Queue length limit exceeded.","JMS Client that is sending messages to a Weblogic Queue with priority.","Since this is a single bag for all the failed messages from all queues, you may want to configure this aspect differently.","For this test we are interested in knowing how many notifications are still waiting to be processed.","Sends a message to the message broker on localhost.","Print will print just the topic content.","Dummy handler implementation that can be used to disable the error handler.","Main class directly App.","Dead Letter Queue approach.","Click on the alert to see relevant links that include support tech notes, APAR defect info, and videos that you can use to continue your content journey and get the info that you need.","JMS clients can send messages to it.","At this point, it would be hard to know if a message in our system has ended up on the DLQ.","Do I need to specify and inbound bridge for the TMP queue?","What is wrong with this page?","The reason why this happens is because the JMS transaction is independent to other transactional resources like the DB.","He is interested in everything related to Java and the Spring framework.","False means that this only happens after a message has been cancelled.","This is a URL for creating consumers that will pull from a queue.","My app is a Spring based application.","What are the goals of the REST interface?","There was an error loading that profile information.","This is a generic type interface but normally for a password you just need String type.","Is this page helpful?","Speaker and open source contributor.","The interval where the disk is scanned for percentual usage.","Diagram created in miro.","General tab: o In Sort Key, select a message sort key name or the name of a message header field on which to sort.","Should I need to create the DLQ explicity from the code or does activemq creates after error message is retried after some time?","MDBs will have to be in masked form.","The default queue manager is the queue manager that applications connect to if they do not specify a queue manager name in an MQCONN call.","So what happens if the listener throws an exception?","To check if the priority was set or not we print JMSMEssage.","Make a queued call to submit the purchase order.","They are controlled by the following two optional Resource Adapter properties in ra.","Apache Camel route, but as it turns out, Camel fails to deliver facilities for exact JMS redelivery.","The minimal number of data files before we can start compacting.","How does the spring know that failure message should go to DLQ?","Sometimes you might have network problems when posting new messages to a queue or topic.","Prefix for replacing configuration settings using Bean Utils.","Rewording the question again.","You need to avoid processing of expired messages, which means: discard them.","There are other cases where the error is caused due to bad data or incorrect URLs.","Embedded or an External one.","You are not authorized to perform this operation.","Making statements based on opinion; back them up with references or personal experience.","Examples Java Code Geeks is not connected to Oracle Corporation and is not sponsored by Oracle Corporation.","What needs to be provisioned when you need to start using Enterprise Messaging in SAP Cloud Platform Integration?","Flow Control for the Cluster connection bridge.","You may do a POST and never receive a response.","Dead Letter Queue for each project.","The Apache Camel library is, among other things, great for consuming messages on a retry schedule, but the setup can be a bit daunting.","Did this page help you?","Together, we can continue to improve IBM Knowledge Center.","Queued messages can remain in the queue for a long time if the receiving application does not read them from the queue in a timely fashion.","These classes will both execute the three cases above described.","Spring Boot which favors convention over configuration.","It is also the queue manager that processes MQSC commands when you invoke the runmqsccommand without specifying a queue manager name.","Was this page helpful?","Java Code Geeks and hone your writing skills!","Also if I try to use consumer it retrieves all the messages but it removes it from the queue.","If input is an empty string, it will process messages pending on all dead queues.","Why would it do this and is there something that could fix this issue?","In a system that processes thousands of messages, having a large number of messages that the consumer repeatedly fails to acknowledge and delete might increase costs and place extra load on the hardware.","Description of this script is valid for Windows platform.","This means that a client can create a new Queue or Topic dynamically either by.","Java is a trademark or registered trademark of Oracle Corporation in the United States and other countries.","If it still fails, order id that is related to the notification is marked as batch error and notification is moved to dead letter table in database.","TODO: we should review the class names and whatnot in use here.","Integrate with other systems by using the services defined in the Service Definition Framework.","It stores messages that have expired or that have failed transfer or delivery.","Sorry, did not spot the comments until today.","DLQ allows us to handle faulty or bad messages, monitor failure patterns and recover from exceptions in a system.","For this purpose, we cannot take the message on the pipeline as we can never be sure where an exception can occur.","Allows you to check which messages are still pending in the specified queue.","Synchronous service bus using the JMS, but I was not able to turn it to Asynchronous.","Sorry just working through my own issue here.","DB before the execution failed.","The consumer will use this value as the window size if the value is not set on the client.","To avoid confusion I only included the relevant part but if for some reason you would like to see the entire configuration, here it is: activemq.","To subscribe to this RSS feed, copy and paste this URL into your RSS reader.","Create the default dead letter queue.","Would you take one minute to complete this survey?","URI must be of the form: net.","For more info about the coronavirus, see cdc.","AIO queue at any one time.","So technically my bridge is working now with a camel route.","It is the same format as that for JMS Bridges.","The messages are summarized by the reason that they were placed on the queue.","The second argument is as equally important as setting routing keys for simple messages.","This handler will move the message to a configurable dead letter queue if it cannot be routed.","In this chapter, we will model an automated retry mechanism.","Or use some Java code with JMS API to remove the messages etc.","Automated retry mechanism will only be helpful in the first case.","Do you want to.","Lastly if Spring JMS support is not cutting it you will find more advanced integration techniques supported in the Spring Integration project.","Display the activemq version information.","This is to handle cases where the route fails and I want to keep the problem message and log the cause.","This flow shall poll the queue after regular intervals as configured in the JMS channel in the next exercise.","Print book on Amazon.","The queue manager starts.","The listener is responsible for retrieval of notifications from the queue and stores them to the database.","If your company has an existing Red Hat account, your organization administrator can grant you access.","To secure against that, use transactions.","Message reaches a threshold read counter number, because it is not consumed.","If False delivery would be always asynchronous.","To toggle press enter.","You can just use a route for the dead letter queue, and do logging and sending to JMS queue.","After changes of partners data in App.","Forward messages to DLA when no queues subscribing.","However a user can if they so wish implement their own.","After processing a message, the consumer is responsible for deleting the message.","Thanks for contributing an answer to Stack Overflow!","This is the size of the pool.","CORS is not supported for this browser!","Any other notifications for the same order are moved to the dead letter table.","The server configuration file has a property that defines the default masking behaviors over the entire file scope.","Select the queue manager from the Navigator View.","Do most amateur players play aggressively?","If the addresses match, the message is dispatched; if not, the message is not dispatched.","However, I still see that the messages are getting stored in DLQ.","Before we send it, however, we set a JMS property to specify where the response should be sent.","Given that in such scenario you often cannot afford to loose msgs, the cost is low.","Can any one help me in this.","Now let us create a retry logic.","Move backwards or forwards from the current topic position in the documentation.","Is possible to retrieve a copy of message to send to Aggregator reporting that an error has occurred related to original message?","Sign in to start or join a conversation or to ask a question.","Insiders are already enjoying weekly updates and complimentary whitepapers!","DLQ is the default Dead Letter Queue.","The spam filter is quite aggressive.","This topic has been unpinned.","DLQ is like any topic or queue and you can subscribe to it like any topic or queue and consume messages accumulated.","There are cases where the error is caused by a temporary reason and the processing usually succeeds after a while, like endpoint temporarily unavailable.","Can that be done with a wildcard?","Thanks for letting us know this page needs work.","You are using a device that only enables you to add content.","Again, not a detail a producer should be coupled to.","If so, there is nothing special needed on the XD side, but you have to configure the queue to support priority.","In this way, we do not lose the message.","You must discover all other resources to interact with by looking for and traversing links.","MSMQ service to send messages.","Very truly, I tell you, before Abraham was, I am.","It is called right after the decoder has been created.","Create a transaction scope.","The System will block while the disk is full.","In this way, we will cause an error before storing the message to the DB.","So the message will be removed from the queue only if it has been processed completely and succesfully.","Logs the exception text whenever an exception occurs.","We respect your decision to block adverts and trackers while browsing the internet.","Because the listener failed before storing the message to the DB, we have lost the message!","BMC, the BMC logo, and other BMC marks are assets of BMC Software, Inc.","Create the purchase order.","Transactional, but this is not the case with Asynchronous Messaging.","You may already have usecases implemented like this using some legacy proprietary queuing system and would like a nice replacement for that legacy system but cannot change your application.","Failed to execute main task.","JBOSS FUSE MANAGEMENT CONSOLE creates a connection for sending a message and stops it after that.","Amazon SQS moves messages from the former to the latter if the consumer of the source queue fails to process a message a specified number of times.","ALL my messages in Hawtio.","There might be too much traffic or a configuration error.","If you are using Spring Initialzr, the Main class might be already present.","Please use Artemis CLI command to add a password.","When a JMS message reaches the maximum retry defined; it is sent to a dead queue configured with the respective route.","When this checkbox is checked, the retry interval is doubled after every retry.","What does Texas gain from keeping its electrical grid independent?","All trademarks and registered trademarks appearing on Java Code Geeks are the property of their respective owners.","Engage with our Red Hat Product Security team, access security updates, and ensure your environments are not exposed to any known security vulnerabilities.","If this is true, no duplicate detection protocol will be enforced for message posting.","The dead letter service is ready.","This usually requires duplicate message detection in the listener implementation, covering the case where database processing has committed but message processing failed to commit.","You must use this URL to POST your next message.","Since the situation persisted, the message has been sent to the special DLQ queue.","Dead Letter queue to handle exceptions occurring within a route.","If we try to consume from the broker at this point, we should see that we cannot establish a connection.","The duplicate detection circular cache size.","Red Hat build of Node.","Searching from a product topic page returns results specific to that product or version, by default.","When a consumer fetches a message from a queue, the messages remains on the queue, but is simply made invisible to keep other consumers from fetching and processing the same message.","To enable retry, we will store the failed message in a JMS queue and pick it up repeatedly to retry processing, until it is processed successfully.","Dead letter queue storing of these messages allows developers to look for common patterns and potential software problems.","IBM wants to learn more about how we can improve technical content for YOU.","The HTTP protocol is strong enough to be our message exchange protocol.","Do you have any tips and hints for beginner blog writers?","Instead some sort of a socket exception will be thrown.","It will be retained here for a limited time for the convenience of our customers but may be removed in whole in part at any time.","Combine this with Spring Integration Framework which allows services to communicate via message channels and in return makes the overall architecture loosely coupled.","If input is empty string; it will list out all the dead letters including batch errors and time events.","Try again later, or contact the app or website owner.","In such case, the message need only be reprocessed as is and the processing succeeds.","WITHOUT WARRANTIES OR CONDITIONS OF ANY KIND, either express or implied.","This is just a named Spring component which will be realized by Camel when a message is added to the queue.","Please type your message and try again.","The JMS Bridges are configured and deployed as separate beans so they need separate configuration to control the password masking.","The problem is, we cannot just redeliver for ever because it will starve other consumers and messages it that come later in the queue.","This is sufficient for most cases.","These cookies do not store any personal information.","This content has been marked as final.","The size of the internal buffer on the journal in KB.","It applies to all cluster configurations.","Topics forward messages to many consumers at once.","Set a data attribute on document.","Error while calling _satellite.","Let us now execute the flow to write a message into this queue.","What should happen when a slow consumer is detected.","Your PDF request was successfully submitted.","Luckily, Artemis comes with some tools for exploring these scenarios.","Thank you for your feedback!","So ultimately Adding outbound.","Configuration is pretty simple.","The request is badly formed.","We were unable to process your PDF request.","Propagates exceptions back to the caller and supports the redelivery policy, but it does not support a dead letter queue.","Although not the additional configuration we wanted to do.","Below is the format of the XML configuration file and the default values for each.","Message that is sent to a queue that does not exist.","Default is no ttl.","So i went to my broker.","One option is to use a Camel route embedded in the broker Another would be to set a message expiration time, allow the message to be automatically moved to the DLQ, and have a consumer process it from there.","FIFO queue must also be a FIFO queue.","This component uses AUTO_ACKNOWLEDGE mode by default.","Returns the dead letter queue for the given message and subscription.","The WCF stack on the receiver matches addresses that the service is listening on with the address on the message.","Silly me, I did not configure destination keys for sorting.","Occasionally, producers and consumers might fail to interpret aspects of the protocol that they use to communicate, causing message corruption or loss.","Distributed transactions are beyond the scope of this post.","The default behavior of this error handler is to propagate the error back to the caller.","It does not make sense to retry forever on a msg that potentially might always fail.","So, how do I get the dead letter channel to log the exception trace as well as putting the message onto the queue?","JMS Event fails after max retries: JMS messages are routed to Dead letter Queue in JMS.","Learn how to resolve issues associated with CPQ.","An unexpected error has occurred.","The relevant Enterprise Integration Pattern is the transactional client.","In English vocabulary Dead letter mail is an undeliverable mail that cannot be delivered to the addressee.","In case of time dependencies if time dependency is not executed within the predefined retry count it is saved into database and will not be picked by Orchestrator.","This describes the root of the XML configuration.","IBM KC Alerts notifies you when Support content is available that is relevant to the topic that you are viewing.","Other subsequent messages will be delivery regularly, only the cancelled message will be sent asynchronously back to the queue after the delay.","Thus, to keep your FIFO queue from getting blocked by a message, make sure that your application correctly handles message processing.","Camel for XA transactions.","Remove the slashes in front to use.","This only applies within transactions, since the broker will try to resend the message after listener processing failure led to a rollback.","This is the id of the embedded instance.","Automate the workflow for your mobile, web, games and other software.","Error handled in camel.","Streamlining data pipelines and scaling AI projects.","This content is presented in an iframe, which your browser does not support.","The following are highly recommended additional parameters.","Since we do not raise an exception in the retry flow, this time, the flow will execute successfully and you should receive an email in the configured inbox.","Continuous message processing also provides faster recovery for your queue.","This chapter discusses the protocol for posting messages to a queue or a topic.","Connect and share knowledge within a single location that is structured and easy to search.","Click to view the Table of Contents.","These cookies will be stored in your browser only with your consent.","Java related sites around.","Certainly, this list of failures is not exhaustive but contains the most common type of errors.","If you have any questions, please contact customer service.","As described in the previous sections, all password masking requires a decoder.","Create a duplicate flow which reads from the queue and retries message processing.","The following is code for the client configuration file.","This option changes the initial routing key of the message for further routing by DLX.","Thanks for posting, and keeping us updated.","Save to DB or send a notification.","This title links to the home page.","This should give you a broker running in the foreground.","What we have to take into account is that duplicate messages can occur, so our listener will have to detect it, or our processing will have to be idempotent to process it again without any problem.","Also here we will be checking the incoming message for salary field.","Please note: a moderator needs to approve your content before it can be published.","Exchanging messages from a microservice to another has never been easier.","This topic has been pinned.","Hope this is helpful for you.","Only, when one message leaves the queue, the next queue can enter the route.","Address wildcards can also be used to configure dead letter settings for a set of addresses.","Both options can be combined for maximum flexibility.","From the command line you can run To send a text message with content obtained from an url.","This is certainly not ideal if you cannot afford to loose messages.","Asking for help, clarification, or responding to other answers.","After maximum retries it will then be put in dead letter queue.","Redirection to the dead letter channel will not occur, however, until all attempts at redelivery have been exhausted.","Jews want to throw stones at Jesus for saying this?","Furthermore, using Individual Dead Letter Strategy would allow us to have a separate Dead Letter Queue for each queue.","User mailing list archive at Nabble.","How safe is it to mount a TV tight to the wall with steel studs?","Anonymous: When using transactions, you use the transactional error handler.","ID can somehow be used for this.","Is there an error in my configuration?","An error handler for transacted routes.","From the point of view of the broker, the message has been consumed and in fact the message is now entirely in the hands of Camel.","Execution of Rabbit message listener failed.","Delaying redelivery can often be useful in cases where clients regularly fail or rollback.","How are we going to handle each of those messages differently?","This section gives an overview of common issues and shows how to resolve them.","However, if the problem is a permanent issue, that could cause an infinite loop.","Define a service contract.","Spring bean definition of the activemq and webspheremq JMS components for simplicity.","This site uses Akismet to reduce spam.","URL will be uniquely generated for each message and used for duplicate detection.","Next, perform your HEAD or GET request on this URI.","Timeout before redistributing values after no consumers.","Standard queues allow a high number of inflight messages.","This article is free for everyone, thanks to Medium Members.","My only mandatory process is to proceed message in the order of arrival and do the treatment before handling the next message.","If so, the failed message will be rejected.","The service can now be accessed.","As a result, the message from the queue should now be picked by the retry integration flow and processed.","Is the Scala Cookbook worth the price?","This is a URL you POST messages to.","Using the REST interface Messages can be produced and consumed by sending and receiving simple HTTP messages that contain the content you want to push around.","The abstract base class for all number formats.","Retry message processing in case of an error, instead of setting the message status to Failed or asking the sender to resend the message.","These messages will not be delivered again from this queue.","Partners details is not OK in App.","WAR for this environment.","The amount of retries is configurable.","Need access to an account?","See the original article here.","There is currently no data in the queue.","Name of the connector reference to use.","The JMS component documentation also provides some information about transaction.","Would you like to go to the _VERSIONNAME_ home page?","The manual purge has no effect on newly created kahadb log files.","It defines whether a queue only uses last values or not.","View messages from a queue.","You can also change the prefix through the broker.","DLQ to the original queue.","Because Acceptors and Connectors are pluggable implementations, each transport will have different password masking needs.","An error occurred while trying to submit your post.","By setting it to one, only one thread consumes a message at a time.","How do we work out what is fair for us both?","In the previous chapter, we simply sent an intimation of error through mail and expected the sender to resend the message in case of an error.","The change is pretty simple and it does not imply modifying a single line of code from our application.","Thanks for sharing this!","We are using the Transaction approach.","What happens when a consumer cannot process a message?","If the listener execution fails, message reception will be rolled back.","The request could not be satisfied.","So I went ahead and added info for an inbound bridge and got an exception, so that answers the temp queue question.","The meaning of the specific settings are explained fully throughout the user manual, however here is a brief description with a link to the appropriate chapter if available.","URI pattern with the name of the queue or topic you are interested in interacting with.","Ray vision prevent Shadow Step?","It is worth noting that when deploying a WAR in a Java EE application server like Wildfly the URL for the resulting application will include the name of the WAR by default.","Or what happens when a transaction is rolled back?","We will notify you when it will be ready for download.","For the best experience, update your browser to the latest version, or switch to another browser.","Error occurred before message processing: The consumer retrieves the message but the execution fails before storing it to the DB.","True means that the delivery count is persisted before delivery.","The comment form collects your name, email and content to allow us keep track of the comments placed on the website.","You can always check the source code of this application by clicking the link at the beginning of this article.","Dealing with Asynchronous Messaging, we can achieve Eventual Consistency using Dead Letter Queues.","How to tell coworker to stop trying to protect me?","What version of Camel are you using?","Supports the same features as the default error handler and, in addition, supports a dead letter queue.","Table of Contents open.","Constantly being on the lookout for partners; we encourage you to join us.","Is there a way to detect when message is moved in DLQ?","The message has been duplicated.","Producer flow control size on the bridge.","Spring Integration i could use a retry pattern on the endpoint, but as the application is in a cluster, another JVM may have better luck getting to the database.","If this is not possible, we will have to go for distributed transactions, since they support transactions that involve different resources.","Resend the message if timed out.","Processes all messages pending on respective dead queue.","Otherwise the decoding may not be successful.","It stores a notification to the DB.","The number of redelivery attempt can be changed on the URL used by connection factory.","We have integrated the Web Console into the binary distribution.","CLI tools to simulate consumption from addresses on the broker.","Provide details and share your research!","Do I need to fix anything in the config?","This configuration of a Camel error handler will never loose any persistent messages!","And more we have redeliveries, more we want to increase this delay.","The HEAD or GET response contains a number of custom response headers that are URLs to additional REST resources that allow you to interact with the queue or topic in different ways.","Join the DZone community and get the full member experience.","To view this site, enable cookies in your browser.","API is available via JMX.","The algorithm used for decoding must match that for encoding.","Allowed values: TIME_DEPENDENCY and BATCH.","By default, the console is not configured to run.","You need to be a registered member to rate this.","How do different queue types handle message failure?","Use Individual Dead Letter Strategy.","This must be a relative or absolute file system path.","We will only need to change the configuration file.","Can you please let me know what could be causing this?","The canonical reference for building a production grade API with Spring.","Your reply was added successfully, but a moderator needs to approve your reply before it can be posted."]
{ "url": "https://star-exchanger.space/text/activemq%20dead%20letter%20queue%20example.txt", "source_domain": "star-exchanger.space", "snapshot_id": "crawl=CC-MAIN-2021-17", "warc_metadata": { "Content-Length": "59817", "Content-Type": "application/http; msgtype=response", "WARC-Block-Digest": "sha1:QIIFVJFIM6UOXKMJNABTWXBIFK4DNHZI", "WARC-Concurrent-To": "<urn:uuid:74119036-9ae6-46e3-a067-e2ca863c50c7>", "WARC-Date": "2021-04-22T23:24:35Z", "WARC-IP-Address": "172.67.159.161", "WARC-Identified-Payload-Type": "text/plain", "WARC-Payload-Digest": "sha1:6RGDRRWSFVXNDYX4PQOYELWEG22H3FZX", "WARC-Record-ID": "<urn:uuid:ad28e40e-a7bc-4243-9132-cfe80f77ffbc>", "WARC-Target-URI": "https://star-exchanger.space/text/activemq%20dead%20letter%20queue%20example.txt", "WARC-Truncated": null, "WARC-Type": "response", "WARC-Warcinfo-ID": "<urn:uuid:3e538941-3c0b-4b13-808e-3d16078213b9>" }, "warc_info": "isPartOf: CC-MAIN-2021-17\r\npublisher: Common Crawl\r\ndescription: Wide crawl of the web for April 2021\r\noperator: Common Crawl Admin ([email protected])\r\nhostname: ip-10-67-67-37.ec2.internal\r\nsoftware: Apache Nutch 1.18 (modified, https://github.com/commoncrawl/nutch/)\r\nrobots: checked via crawler-commons 1.2-SNAPSHOT (https://github.com/crawler-commons/crawler-commons)\r\nformat: WARC File Format 1.1\r\nconformsTo: https://iipc.github.io/warc-specifications/specifications/warc-format/warc-1.1/" }
{ "line_start_idx": [ 0 ], "line_end_idx": [ 58749 ] }
{ "red_pajama_v2": { "ccnet_original_length": 58749, "ccnet_original_nlines": 0, "rps_doc_curly_bracket": 0, "rps_doc_ldnoobw_words": 0, "rps_doc_lorem_ipsum": 0, "rps_doc_stop_word_fraction": 0.4347182512283325, "rps_doc_ut1_blacklist": 0, "rps_doc_frac_all_caps_words": 0.02470620907843113, "rps_doc_frac_lines_end_with_ellipsis": 0, "rps_doc_frac_no_alph_words": 0.1033589318394661, "rps_doc_frac_unique_words": 0.2717770040035248, "rps_doc_mean_word_length": 5.209171772003174, "rps_doc_num_sentences": 794, "rps_doc_symbol_to_word_ratio": 0, "rps_doc_unigram_entropy": 6.285645008087158, "rps_doc_word_count": 8897, "rps_doc_frac_chars_dupe_10grams": 0.008630730211734772, "rps_doc_frac_chars_dupe_5grams": 0.02951711043715477, "rps_doc_frac_chars_dupe_6grams": 0.02183575928211212, "rps_doc_frac_chars_dupe_7grams": 0.015060629695653915, "rps_doc_frac_chars_dupe_8grams": 0.012341950088739395, "rps_doc_frac_chars_dupe_9grams": 0.010184270329773426, "rps_doc_frac_chars_top_2gram": 0.006041510030627251, "rps_doc_frac_chars_top_3gram": 0.003085490083321929, "rps_doc_frac_chars_top_4gram": 0.0016182600520551205, "rps_doc_books_importance": -4366.77734375, "rps_doc_books_importance_length_correction": -4366.77734375, "rps_doc_openwebtext_importance": -2567.767822265625, "rps_doc_openwebtext_importance_length_correction": -2567.767822265625, "rps_doc_wikipedia_importance": -1947.0386962890625, "rps_doc_wikipedia_importance_length_correction": -1947.0386962890625 }, "fasttext": { "dclm": 0.05601483955979347, "english": 0.9336718320846558, "fineweb_edu_approx": 1.5369491577148438, "eai_general_math": 0.3974325656890869, "eai_open_web_math": 0.10677576065063477, "eai_web_code": 0.7484399676322937 } }
{ "free_decimal_correspondence": { "primary": { "code": "005.1", "labels": { "level_1": "General works, books and libraries, information sciences", "level_2": "", "level_3": "Computer programming" } }, "secondary": { "code": "004.678", "labels": { "level_1": "General works, books and libraries, information sciences", "level_2": "", "level_3": "Computers and Computer science" } } }, "bloom_cognitive_process": { "primary": { "code": "3", "label": "Apply" }, "secondary": { "code": "2", "label": "Understand" } }, "bloom_knowledge_domain": { "primary": { "code": "3", "label": "Procedural" }, "secondary": { "code": "2", "label": "Conceptual" } }, "document_type_v1": { "primary": { "code": "3", "label": "Reference/Encyclopedic/Educational" }, "secondary": { "code": "5", "label": "Social/Forum" } }, "extraction_artifacts": { "primary": { "code": "3", "label": "Irrelevant Content" }, "secondary": { "code": "-1", "label": "Abstain" } }, "missing_content": { "primary": { "code": "2", "label": "Click Here References" }, "secondary": { "code": "-1", "label": "Abstain" } }, "document_type_v2": { "primary": { "code": "18", "label": "Q&A Forum" }, "secondary": { "code": "8", "label": "Documentation" } }, "reasoning_depth": { "primary": { "code": "3", "label": "Intermediate Reasoning" }, "secondary": { "code": "2", "label": "Basic Reasoning" } }, "technical_correctness": { "primary": { "code": "3", "label": "Mostly Correct" }, "secondary": { "code": "4", "label": "Highly Correct" } }, "education_level": { "primary": { "code": "3", "label": "Undergraduate Level" }, "secondary": { "code": "4", "label": "Graduate/Expert Level" } } }
672f1e42c33a7f9846924a2431ea77df
-3,147,591,906,316,863,500
Tools for linux devops: lsbond.pl Slowly and surely, I am scratching the itches I’ve had for a while with regards to data extraction from a running system. One of the big issues I deal with all the time is to extract what the state and components (and their states) of a linux network bond. Its an annoying combination of /sys/class/net, /proc/net/bonding/, and ethtool/ip commands. So I decided to simplify it. bond0: mac 00:11:22:33:44:55 state up mode load balancing (xor) xmit_hash layer2+3 (2) polling 100 ms up_delay 200 ms down_delay 200 ms ipv4 10.20.30.40/16 ipv6 fe80::123:00ff:fe80:4455/64 eth2: mac 00:11:22:33:44:55, link 1, state up, speed 10000, driver ixgbe eth3: mac 00:11:22:33:44:56, link 0, state down, speed 65535, driver ixgbe eth4: mac 00:11:22:33:44:57, link 1, state up, speed 10000, driver ixgbe eth5: mac 00:11:22:33:44:58, link 0, state down, speed 65535, driver ixgbe Eventually, I’ll provide csv and json output modules. But this is the first of many. Look at/grab the source on github. Viewed 37705 times by 3155 viewers
{ "url": "http://scalability.org/2015/07/tools-for-linux-devops-lsbonds-pl/", "source_domain": "scalability.org", "snapshot_id": "crawl=CC-MAIN-2017-51", "warc_metadata": { "Content-Length": "97428", "Content-Type": "application/http; msgtype=response", "WARC-Block-Digest": "sha1:4FXZFEIDOQXETDWSE3IET3K4DWHZ3PCN", "WARC-Concurrent-To": "<urn:uuid:64c261d8-2596-4bca-917c-1e701b76d184>", "WARC-Date": "2017-12-16T09:13:30Z", "WARC-IP-Address": "96.66.33.105", "WARC-Identified-Payload-Type": "text/html", "WARC-Payload-Digest": "sha1:7JR5SYQOWEES4F4NDX4IBTHKXW3DSMB2", "WARC-Record-ID": "<urn:uuid:92143ce3-92f0-451f-8cc4-bd2ac06725e0>", "WARC-Target-URI": "http://scalability.org/2015/07/tools-for-linux-devops-lsbonds-pl/", "WARC-Truncated": "length", "WARC-Type": "response", "WARC-Warcinfo-ID": "<urn:uuid:0719cec8-7e74-4c48-8815-cec17240b2dd>" }, "warc_info": "robots: classic\r\nhostname: ip-10-140-161-138.ec2.internal\r\nsoftware: Nutch 1.6 (CC)\r\nisPartOf: CC-MAIN-2017-51\r\noperator: Common Crawl Admin\r\ndescription: Wide crawl of the web for December 2017\r\npublisher: Common Crawl\r\nformat: WARC File Format 1.0\r\nconformsTo: http://bibnum.bnf.fr/WARC/WARC_ISO_28500_version1_latestdraft.pdf" }
{ "line_start_idx": [ 0, 34, 35, 413, 414, 443, 455, 482, 506, 522, 539, 558, 581, 617, 694, 771, 848, 925, 926, 1046, 1047 ], "line_end_idx": [ 34, 35, 413, 414, 443, 455, 482, 506, 522, 539, 558, 581, 617, 694, 771, 848, 925, 926, 1046, 1047, 1081 ] }
{ "red_pajama_v2": { "ccnet_original_length": 1081, "ccnet_original_nlines": 20, "rps_doc_curly_bracket": 0, "rps_doc_ldnoobw_words": 0, "rps_doc_lorem_ipsum": 0, "rps_doc_stop_word_fraction": 0.16271185874938965, "rps_doc_ut1_blacklist": 0, "rps_doc_frac_all_caps_words": 0.01694915071129799, "rps_doc_frac_lines_end_with_ellipsis": 0, "rps_doc_frac_no_alph_words": 0.4677966237068176, "rps_doc_frac_unique_words": 0.6746988296508789, "rps_doc_mean_word_length": 4.807229042053223, "rps_doc_num_sentences": 12, "rps_doc_symbol_to_word_ratio": 0, "rps_doc_unigram_entropy": 4.527350425720215, "rps_doc_word_count": 166, "rps_doc_frac_chars_dupe_10grams": 0, "rps_doc_frac_chars_dupe_5grams": 0.17042607069015503, "rps_doc_frac_chars_dupe_6grams": 0.17042607069015503, "rps_doc_frac_chars_dupe_7grams": 0.17042607069015503, "rps_doc_frac_chars_dupe_8grams": 0.17042607069015503, "rps_doc_frac_chars_dupe_9grams": 0, "rps_doc_frac_chars_top_2gram": 0.055137839168310165, "rps_doc_frac_chars_top_3gram": 0.02506265975534916, "rps_doc_frac_chars_top_4gram": 0.0300751905888319, "rps_doc_books_importance": -129.50572204589844, "rps_doc_books_importance_length_correction": -129.50572204589844, "rps_doc_openwebtext_importance": -55.875911712646484, "rps_doc_openwebtext_importance_length_correction": -45.33260726928711, "rps_doc_wikipedia_importance": -60.658817291259766, "rps_doc_wikipedia_importance_length_correction": -60.658817291259766 }, "fasttext": { "dclm": 0.22158873081207275, "english": 0.8396313190460205, "fineweb_edu_approx": 1.539062738418579, "eai_general_math": 0.5648750066757202, "eai_open_web_math": 0.11292433738708496, "eai_web_code": 0.41114526987075806 } }
{ "free_decimal_correspondence": { "primary": { "code": "004.02856", "labels": { "level_1": "General works, books and libraries, information sciences", "level_2": "", "level_3": "Computers and Computer science" } }, "secondary": { "code": "005.456", "labels": { "level_1": "General works, books and libraries, information sciences", "level_2": "", "level_3": "Computer programming" } } }, "bloom_cognitive_process": { "primary": { "code": "3", "label": "Apply" }, "secondary": { "code": "2", "label": "Understand" } }, "bloom_knowledge_domain": { "primary": { "code": "3", "label": "Procedural" }, "secondary": { "code": "2", "label": "Conceptual" } }, "document_type_v1": { "primary": { "code": "3", "label": "Reference/Encyclopedic/Educational" }, "secondary": { "code": "4", "label": "Code/Software" } }, "extraction_artifacts": { "primary": { "code": "0", "label": "No Artifacts" }, "secondary": { "code": "-1", "label": "Abstain" } }, "missing_content": { "primary": { "code": "0", "label": "No missing content" }, "secondary": { "code": "-1", "label": "Abstain" } }, "document_type_v2": { "primary": { "code": "8", "label": "Documentation" }, "secondary": { "code": "16", "label": "Personal Blog" } }, "reasoning_depth": { "primary": { "code": "2", "label": "Basic Reasoning" }, "secondary": { "code": "3", "label": "Intermediate Reasoning" } }, "technical_correctness": { "primary": { "code": "3", "label": "Mostly Correct" }, "secondary": { "code": "4", "label": "Highly Correct" } }, "education_level": { "primary": { "code": "3", "label": "Undergraduate Level" }, "secondary": { "code": "4", "label": "Graduate/Expert Level" } } }
672f1e42c33a7f9846924a2431ea77df
-122,652,057,148,251,540
Ir al contenido principal Ralsina.Me — El sitio web de Roberto Alsina Safe Remote Blogging with PyDS New tu­to­ri­al: En­abling re­mote blog­ging se­cure­ly in PyD­S, us­ing au­then­ti­ca­tion and a HTTPS re­verse prox­y. Use­ful if you want to, for ex­am­ple, put PyDS on the In­ter­net to blog from any­where! Georg Bauer / 2006-04-03 04:58: Great article! I will link it from the pyds homepage. Oh, one note: medusa (the web server used in PyDS) does support https - only problem is, you need some encryption support for Python. If you have the relevant libraries, it should be possible to directly use https in medusa. Then one could set up an additional https server port in PyDS. I didn't include it in the source, because this would add a dependency on the encryption stuff and that would complicate the compile process. Though I think I might add it in a way that it is used conditionally - use SSL if the relevant libs are available, don't use it otherwise. Roberto Alsina / 2006-04-03 04:59: Glad you liked it :-) One thing I can't figure out: how can I make PyDS not trust ANY addresses? Right now, if your box has accounts for other users, they can do anything they want. Georg Bauer / 2006-04-03 05:00: There isn't a way to don't trust any addresses. But if your SSL reverse proxy is running at the same machine than your PyDS, you just can set remoteip to 127.0.0.1 and remoteport to something different from 4334. This would allow the SSL reverse proxy to still access the PyDS (and allow all local users to access PyDS directly, but they can already by accessing the standard interface) but would prevent outside access. If your PyDS and your SSL reverse proxy run on different hosts, you should use the systems firewalling to restrict access to the PyDS port to only the proxy machine. Contents © 2000-2022 Roberto Alsina
{ "url": "https://ralsina.me/tr/es/weblog/posts/P65.html", "source_domain": "ralsina.me", "snapshot_id": "CC-MAIN-2023-06", "warc_metadata": { "Content-Length": "8069", "Content-Type": "application/http; msgtype=response", "WARC-Block-Digest": "sha1:QDNNHUEITFTDBNTX2IASSLXPYDU2SFCD", "WARC-Concurrent-To": "<urn:uuid:a05f1142-63ab-43b2-8cf5-2202303e31d6>", "WARC-Date": "2023-02-07T14:22:12Z", "WARC-IP-Address": "172.67.145.72", "WARC-Identified-Payload-Type": "text/html", "WARC-Payload-Digest": "sha1:Z7I2BFG4Z7HNGL3ZYRNRRQ4ZUNMBOPUU", "WARC-Record-ID": "<urn:uuid:01030a9f-3e74-41ec-be5f-cf6b26fe89cb>", "WARC-Target-URI": "https://ralsina.me/tr/es/weblog/posts/P65.html", "WARC-Truncated": null, "WARC-Type": "response", "WARC-Warcinfo-ID": "<urn:uuid:070b792b-11ff-4cdd-b550-2d4895abf7f0>" }, "warc_info": "isPartOf: CC-MAIN-2023-06\r\npublisher: Common Crawl\r\ndescription: Wide crawl of the web for January/February 2023\r\noperator: Common Crawl Admin ([email protected])\r\nhostname: ip-10-67-67-138\r\nsoftware: Apache Nutch 1.19 (modified, https://github.com/commoncrawl/nutch/)\r\nrobots: checked via crawler-commons 1.4-SNAPSHOT (https://github.com/crawler-commons/crawler-commons)\r\nformat: WARC File Format 1.1\r\nconformsTo: https://iipc.github.io/warc-specifications/specifications/warc-format/warc-1.1/" }
{ "line_start_idx": [ 0, 26, 27, 71, 72, 103, 104, 225, 226, 316, 317, 349, 350, 973, 974, 1009, 1010, 1032, 1033, 1034, 1035, 1195, 1196, 1228, 1229, 1650, 1651, 1652, 1653, 1819, 1820, 1821 ], "line_end_idx": [ 26, 27, 71, 72, 103, 104, 225, 226, 316, 317, 349, 350, 973, 974, 1009, 1010, 1032, 1033, 1034, 1035, 1195, 1196, 1228, 1229, 1650, 1651, 1652, 1653, 1819, 1820, 1821, 1856 ] }
{ "red_pajama_v2": { "ccnet_original_length": 1856, "ccnet_original_nlines": 31, "rps_doc_curly_bracket": 0, "rps_doc_ldnoobw_words": 0, "rps_doc_lorem_ipsum": 0, "rps_doc_stop_word_fraction": 0.3718820810317993, "rps_doc_ut1_blacklist": 0, "rps_doc_frac_all_caps_words": 0.029478460550308228, "rps_doc_frac_lines_end_with_ellipsis": 0, "rps_doc_frac_no_alph_words": 0.2471655309200287, "rps_doc_frac_unique_words": 0.5523809790611267, "rps_doc_mean_word_length": 4.612698554992676, "rps_doc_num_sentences": 20, "rps_doc_symbol_to_word_ratio": 0, "rps_doc_unigram_entropy": 4.8505473136901855, "rps_doc_word_count": 315, "rps_doc_frac_chars_dupe_10grams": 0, "rps_doc_frac_chars_dupe_5grams": 0, "rps_doc_frac_chars_dupe_6grams": 0, "rps_doc_frac_chars_dupe_7grams": 0, "rps_doc_frac_chars_dupe_8grams": 0, "rps_doc_frac_chars_dupe_9grams": 0, "rps_doc_frac_chars_top_2gram": 0.026841020211577415, "rps_doc_frac_chars_top_3gram": 0.030970409512519836, "rps_doc_frac_chars_top_4gram": 0.02615278959274292, "rps_doc_books_importance": -183.9871826171875, "rps_doc_books_importance_length_correction": -183.86143493652344, "rps_doc_openwebtext_importance": -98.42544555664062, "rps_doc_openwebtext_importance_length_correction": -98.42544555664062, "rps_doc_wikipedia_importance": -55.14350128173828, "rps_doc_wikipedia_importance_length_correction": -55.14341354370117 }, "fasttext": { "dclm": 0.18308204412460327, "english": 0.8925389647483826, "fineweb_edu_approx": 2.227058172225952, "eai_general_math": 0.7933946251869202, "eai_open_web_math": 0.25805312395095825, "eai_web_code": 0.792348325252533 } }
{ "free_decimal_correspondence": { "primary": { "code": "005.1", "labels": { "level_1": "General works, books and libraries, information sciences", "level_2": "", "level_3": "Computer programming" } }, "secondary": { "code": "004.678", "labels": { "level_1": "General works, books and libraries, information sciences", "level_2": "", "level_3": "Computers and Computer science" } } }, "bloom_cognitive_process": { "primary": { "code": "3", "label": "Apply" }, "secondary": { "code": "2", "label": "Understand" } }, "bloom_knowledge_domain": { "primary": { "code": "3", "label": "Procedural" }, "secondary": { "code": "2", "label": "Conceptual" } }, "document_type_v1": { "primary": { "code": "3", "label": "Reference/Encyclopedic/Educational" }, "secondary": { "code": "4", "label": "Code/Software" } }, "extraction_artifacts": { "primary": { "code": "3", "label": "Irrelevant Content" }, "secondary": { "code": "-1", "label": "Abstain" } }, "missing_content": { "primary": { "code": "0", "label": "No missing content" }, "secondary": { "code": "-1", "label": "Abstain" } }, "document_type_v2": { "primary": { "code": "16", "label": "Personal Blog" }, "secondary": { "code": "23", "label": "Tutorial" } }, "reasoning_depth": { "primary": { "code": "2", "label": "Basic Reasoning" }, "secondary": { "code": "3", "label": "Intermediate Reasoning" } }, "technical_correctness": { "primary": { "code": "3", "label": "Mostly Correct" }, "secondary": { "code": "4", "label": "Highly Correct" } }, "education_level": { "primary": { "code": "2", "label": "High School Level" }, "secondary": { "code": "3", "label": "Undergraduate Level" } } }
672f1e42c33a7f9846924a2431ea77df
-5,489,045,440,432,603,000
Cooling down an iMac Discussion in 'iMac' started by Hugbees, Apr 2, 2009. 1. Hugbees macrumors regular Joined: Dec 13, 2007 #1 When I play games on my iMac, it generally gets hot to the touch in the back. I'm not sure if this is dangerous or not, but it stays pretty cool otherwise. Is there a safe way to avoid my iMac from getting hot, or just stay at a warm temperature? I would try increasing the speed of my fans but I hear that isn't the safest solution, and I'm hoping to find another   2. aki macrumors 6502a Joined: Mar 2, 2004 Location: Japan #2 faster fans is less heat its the only way its not dangerous worst thing is if your fan wears out i use smcfancontrol on imac it works v well when gaming for a long time in bootcamp about "hot to touch" thats actually good its the imac loosing heat....but if u want to watch things install istat pro or another temp monitor app some people are too paranoid about heat....it really depends on how paranoid you are and what you use your imac for eg bootcamp 3d gaming for 8 hours a day and how long you need your imac or hd to last there are many many threads arguing about this good luck!   3. eXan macrumors 601 eXan Joined: Jan 10, 2005 Location: Russia 4. Sambo110 macrumors 68000 Joined: Mar 12, 2007 Location: Australia #4 Mine gets extremely hot sometimes too, mainly while gaming. So it is normal I think.   5. Hugbees thread starter macrumors regular Joined: Dec 13, 2007 #5 To me this sounds like you're saying that iMacs are susceptible to easily breaking due to gaming. My only reason for the concern is when I am gaming, after a while it starts to make a humming noise that sounds like its working hard. If I minimize or quit it stops making the noise and returns to normal. If the warm touch means its losing heat, then thats okay, but I don't want the fans and processor or whatever to have to work so hard at running that it breaks down So is this humming normal? To be more specific, it hums a bit when running anything, which is pretty quiet and it just sounds like its running, but when playing games, Warhammer Online to be exact, it gets a bit more intense like it needs to work harder to run the game. If there is a way to quiet it and give me some relief, I'd like that :p   6. Benguitar Guest Benguitar Joined: Jan 30, 2009 #6 Turn a compressed air can for cleaning keyboards upside down and blow it at the back of the computer.. :p Don't worry about it, I think it is just a normal thing. Most if not all computers get hot when being heavily used.   7. Hugbees thread starter macrumors regular Joined: Dec 13, 2007 #7 I've had issues before with Warhammer months ago where I'd play and at random intervals it would freeze up and reboot my machine. After investigating the matter I found that it may have been caused by overheating, and thus I couldn't play Warhammer or most other MMO games. It was a problem I could never fix because of how randomly it crashes and the selection of games it crashed on. EX: Warhammer Everquest 2 - these 2 crashed a lot Perfect World - once a month and every other game, nothing... and this is only in Windows only. Any mac games like WoW never once crash or give me problems. So thats the only reason why I'm concerned   8. eXan macrumors 601 eXan Joined: Jan 10, 2005 Location: Russia #8 Crashing is not normal, assuming its caused by heat. Download the iStat Pro widget and report us your idle and 100% load temps/fan speeds.   9. Fonzijr1964 macrumors 68000 Fonzijr1964 Joined: Feb 26, 2008 Location: Maryland 10. aki macrumors 6502a Joined: Mar 2, 2004 Location: Japan #10 well i use smcfancontrol because i game in windows under bootcamp and apple doesnt give the imac great fan control in windows so the heats there are sort of abnormal the imac gets hot in osx gaming too of course but apples temp control system works normally in that case i dont think the imac is a weak machine or anything.....its just it is a very small case and not much room so heat is more an issue than in a roomy tower or something i really think if you plan to upgrade your imac say every 2-3 years then forget heat....however its just a fact that the hd lifespan is shorter under high temps so if you want your imac 4-5 years ooooooor possibly if you spend all your time in windows then its something you might want to just think about i think tho 90% of people should just forget the whole heat issue....unless they are crashing or something obviously thats not good about your humming i guess that is the fan sound??if it is then again thats good it means air is moving ignore it....if it was hot with no fan noise then you should worry :)   Share This Page
{ "url": "https://forums.macrumors.com/threads/cooling-down-an-imac.679829/", "source_domain": "forums.macrumors.com", "snapshot_id": "crawl=CC-MAIN-2017-51", "warc_metadata": { "Content-Length": "176709", "Content-Type": "application/http; msgtype=response", "WARC-Block-Digest": "sha1:GB6VHX4D2ROQIGK7ET34LS6PZ2WNZK2Q", "WARC-Concurrent-To": "<urn:uuid:51fe331a-bbc8-4a39-a940-2f6cceb362c1>", "WARC-Date": "2017-12-17T18:28:50Z", "WARC-IP-Address": "162.254.116.250", "WARC-Identified-Payload-Type": "text/html", "WARC-Payload-Digest": "sha1:24K4VZVVM4T7RYKAWZ5FAZYTR45FJPBV", "WARC-Record-ID": "<urn:uuid:b62b038e-c909-4c8e-b533-acfcfb0476e0>", "WARC-Target-URI": "https://forums.macrumors.com/threads/cooling-down-an-imac.679829/", "WARC-Truncated": null, "WARC-Type": "response", "WARC-Warcinfo-ID": "<urn:uuid:dce66172-c823-4088-af8d-a5ccbc516a22>" }, "warc_info": "robots: classic\r\nhostname: ip-10-141-32-201.ec2.internal\r\nsoftware: Nutch 1.6 (CC)\r\nisPartOf: CC-MAIN-2017-51\r\noperator: Common Crawl Admin\r\ndescription: Wide crawl of the web for December 2017\r\npublisher: Common Crawl\r\nformat: WARC File Format 1.0\r\nconformsTo: http://bibnum.bnf.fr/WARC/WARC_ISO_28500_version1_latestdraft.pdf" }
{ "line_start_idx": [ 0, 21, 22, 76, 77, 108, 109, 121, 138, 145, 514, 520, 545, 546, 558, 574, 588, 598, 605, 651, 652, 711, 712, 800, 801, 951, 952, 1158, 1159, 1210, 1211, 1226, 1232, 1256, 1257, 1266, 1267, 1279, 1296, 1310, 1321, 1351, 1352, 1364, 1381, 1395, 1409, 1416, 1505, 1511, 1557, 1558, 1570, 1587, 1594, 1696, 1697, 1907, 1908, 2077, 2078, 2109, 2110, 2430, 2436, 2457, 2458, 2472, 2473, 2485, 2502, 2509, 2619, 2620, 2740, 2746, 2792, 2793, 2805, 2822, 2829, 3107, 3108, 3224, 3225, 3233, 3234, 3248, 3288, 3321, 3322, 3454, 3455, 3502, 3508, 3532, 3533, 3542, 3543, 3555, 3572, 3586, 3597, 3604, 3747, 3753, 3786, 3787, 3803, 3804, 3816, 3833, 3847, 3860, 3886, 3887, 3899, 3915, 3929, 3939, 3947, 4117, 4118, 4227, 4228, 4399, 4400, 4710, 4711, 4847, 4848, 5026, 5032, 5033 ], "line_end_idx": [ 21, 22, 76, 77, 108, 109, 121, 138, 145, 514, 520, 545, 546, 558, 574, 588, 598, 605, 651, 652, 711, 712, 800, 801, 951, 952, 1158, 1159, 1210, 1211, 1226, 1232, 1256, 1257, 1266, 1267, 1279, 1296, 1310, 1321, 1351, 1352, 1364, 1381, 1395, 1409, 1416, 1505, 1511, 1557, 1558, 1570, 1587, 1594, 1696, 1697, 1907, 1908, 2077, 2078, 2109, 2110, 2430, 2436, 2457, 2458, 2472, 2473, 2485, 2502, 2509, 2619, 2620, 2740, 2746, 2792, 2793, 2805, 2822, 2829, 3107, 3108, 3224, 3225, 3233, 3234, 3248, 3288, 3321, 3322, 3454, 3455, 3502, 3508, 3532, 3533, 3542, 3543, 3555, 3572, 3586, 3597, 3604, 3747, 3753, 3786, 3787, 3803, 3804, 3816, 3833, 3847, 3860, 3886, 3887, 3899, 3915, 3929, 3939, 3947, 4117, 4118, 4227, 4228, 4399, 4400, 4710, 4711, 4847, 4848, 5026, 5032, 5033, 5048 ] }
{ "red_pajama_v2": { "ccnet_original_length": 5048, "ccnet_original_nlines": 133, "rps_doc_curly_bracket": 0, "rps_doc_ldnoobw_words": 0, "rps_doc_lorem_ipsum": 0, "rps_doc_stop_word_fraction": 0.4340931475162506, "rps_doc_ut1_blacklist": 0, "rps_doc_frac_all_caps_words": 0.01883053034543991, "rps_doc_frac_lines_end_with_ellipsis": 0, "rps_doc_frac_no_alph_words": 0.17145688831806183, "rps_doc_frac_unique_words": 0.4251151978969574, "rps_doc_mean_word_length": 4.179723262786865, "rps_doc_num_sentences": 41, "rps_doc_symbol_to_word_ratio": 0.014866200275719166, "rps_doc_unigram_entropy": 5.443251132965088, "rps_doc_word_count": 868, "rps_doc_frac_chars_dupe_10grams": 0.027012130245566368, "rps_doc_frac_chars_dupe_5grams": 0.08792722970247269, "rps_doc_frac_chars_dupe_6grams": 0.08792722970247269, "rps_doc_frac_chars_dupe_7grams": 0.07938258349895477, "rps_doc_frac_chars_dupe_8grams": 0.07938258349895477, "rps_doc_frac_chars_dupe_9grams": 0.07938258349895477, "rps_doc_frac_chars_top_2gram": 0.007717750035226345, "rps_doc_frac_chars_top_3gram": 0.01819184049963951, "rps_doc_frac_chars_top_4gram": 0.020672550424933434, "rps_doc_books_importance": -527.36181640625, "rps_doc_books_importance_length_correction": -527.36181640625, "rps_doc_openwebtext_importance": -296.9643249511719, "rps_doc_openwebtext_importance_length_correction": -296.9643249511719, "rps_doc_wikipedia_importance": -270.6412353515625, "rps_doc_wikipedia_importance_length_correction": -270.6412353515625 }, "fasttext": { "dclm": 0.19731491804122925, "english": 0.942471444606781, "fineweb_edu_approx": 1.2108782529830933, "eai_general_math": 0.07333022356033325, "eai_open_web_math": 0.21889853477478027, "eai_web_code": 0.002774599939584732 } }
{ "free_decimal_correspondence": { "primary": { "code": "004.16", "labels": { "level_1": "General works, books and libraries, information sciences", "level_2": "", "level_3": "Computers and Computer science" } }, "secondary": { "code": "004.17", "labels": { "level_1": "General works, books and libraries, information sciences", "level_2": "", "level_3": "Computers and Computer science" } } }, "bloom_cognitive_process": { "primary": { "code": "2", "label": "Understand" }, "secondary": { "code": "3", "label": "Apply" } }, "bloom_knowledge_domain": { "primary": { "code": "2", "label": "Conceptual" }, "secondary": { "code": "3", "label": "Procedural" } }, "document_type_v1": { "primary": { "code": "5", "label": "Social/Forum" }, "secondary": { "code": "3", "label": "Reference/Encyclopedic/Educational" } }, "extraction_artifacts": { "primary": { "code": "3", "label": "Irrelevant Content" }, "secondary": { "code": "0", "label": "No Artifacts" } }, "missing_content": { "primary": { "code": "0", "label": "No missing content" }, "secondary": { "code": "4", "label": "Missing Images or Figures" } }, "document_type_v2": { "primary": { "code": "18", "label": "Q&A Forum" }, "secondary": { "code": "21", "label": "Customer Support" } }, "reasoning_depth": { "primary": { "code": "2", "label": "Basic Reasoning" }, "secondary": { "code": "3", "label": "Intermediate Reasoning" } }, "technical_correctness": { "primary": { "code": "3", "label": "Mostly Correct" }, "secondary": { "code": "2", "label": "Partially Correct" } }, "education_level": { "primary": { "code": "1", "label": "General Audience" }, "secondary": { "code": "2", "label": "High School Level" } } }
672f1e42c33a7f9846924a2431ea77df
1,570,420,269,520,886,500
Tell me more × Stack Overflow is a question and answer site for professional and enthusiast programmers. It's 100% free, no registration required. I'm doing a Java Record/Replay tool and I need to launch Java applications from my main Java app. I need access to the EventDispatchThread in order to intercept the events and record them, so I'm launching the application through reflection with (code snippet simplified): Class<?> app = Class.forName(mainClass); Method m = app.getMethod("main", new Class[] { String[].class }); m.invoke(null, new Object[] { new String[] {} }); I previously dynamically load all the jars to the classpath and the application is launching almost perfectly. The problem occurs when the application needs to access any file and does it with relative paths. Because the application is launched through my application the path is not the same as launched from its normal path and the files are not found. What can I do to solve this problem? dynamically change the execution environment? any ideas? share|improve this question 2 Answers up vote 2 down vote accepted I would suggest loading your code as a "Java Agent" whilst starting the target application. (With your method you will also find that the system class loader is wrong.) share|improve this answer I'll check how to run the code as a "Java Agent" and what that exactly means. I didn't understood your parenthesis, though... – jpsstavares Mar 5 '10 at 17:39 In general, there's no way to do this. http://bugs.sun.com/bugdatabase/view_bug.do;jsessionid=30b24551130ee4ffffffffc17df8d7ce8a9c3?bug_id=4117557 You can try System.setProperty("user.dir", "C:\\Some\\Location"); but it probably won't work for all cases and will give you weird behavior. It's a pretty bad design for an app to rely on the directory from which it was launched. The best advice I can give you is to launch your app from the directory from which the misbehaving app expects to be launched (assuming you know what that directory is). Otherwise, hell, copy/symlink the data files into your directory so that the client app can find them... share|improve this answer Your Answer   discard By posting your answer, you agree to the privacy policy and terms of service. Not the answer you're looking for? Browse other questions tagged or ask your own question.
{ "url": "http://stackoverflow.com/questions/2388634/launch-a-java-application-from-another-java-application", "source_domain": "stackoverflow.com", "snapshot_id": "crawl=CC-MAIN-2013-20", "warc_metadata": { "Content-Length": "47582", "Content-Type": "application/http; msgtype=response", "WARC-Block-Digest": "sha1:4G7XBTCOHZUAFUKRVFTOCTTH27WCS3KS", "WARC-Concurrent-To": "<urn:uuid:c0e75362-cfe3-45aa-ab90-6e36d75ff879>", "WARC-Date": "2013-05-24T04:54:36Z", "WARC-IP-Address": "198.252.206.16", "WARC-Identified-Payload-Type": null, "WARC-Payload-Digest": "sha1:JYYFE4M4K2HIJSRPB2MER32Y6P7IWX4S", "WARC-Record-ID": "<urn:uuid:338116d2-3320-4038-931b-b39dc20961c5>", "WARC-Target-URI": "http://stackoverflow.com/questions/2388634/launch-a-java-application-from-another-java-application", "WARC-Truncated": null, "WARC-Type": "response", "WARC-Warcinfo-ID": "<urn:uuid:d6f6abfb-2d07-4a37-a010-8f74ef2ba11e>" }, "warc_info": "robots: classic\r\nhostname: ip-10-60-113-184.ec2.internal\r\nsoftware: Nutch 1.6 (CC)/CC WarcExport 1.0\r\nisPartOf: CC-MAIN-2013-20\r\noperator: CommonCrawl Admin\r\ndescription: Wide crawl of the web with URLs provided by Blekko for Spring 2013\r\npublisher: CommonCrawl\r\nformat: WARC File Format 1.0\r\nconformsTo: http://bibnum.bnf.fr/WARC/WARC_ISO_28500_version1_latestdraft.pdf" }
{ "line_start_idx": [ 0, 15, 147, 148, 421, 422, 463, 530, 580, 581, 692, 693, 937, 938, 1032, 1033, 1061, 1062, 1072, 1073, 1102, 1103, 1195, 1196, 1273, 1274, 1300, 1459, 1460, 1499, 1500, 1608, 1609, 1750, 1751, 1840, 1841, 2011, 2012, 2117, 2118, 2144, 2145, 2157, 2158, 2160, 2168, 2169, 2247, 2248 ], "line_end_idx": [ 15, 147, 148, 421, 422, 463, 530, 580, 581, 692, 693, 937, 938, 1032, 1033, 1061, 1062, 1072, 1073, 1102, 1103, 1195, 1196, 1273, 1274, 1300, 1459, 1460, 1499, 1500, 1608, 1609, 1750, 1751, 1840, 1841, 2011, 2012, 2117, 2118, 2144, 2145, 2157, 2158, 2160, 2168, 2169, 2247, 2248, 2338 ] }
{ "red_pajama_v2": { "ccnet_original_length": 2338, "ccnet_original_nlines": 49, "rps_doc_curly_bracket": 0.0025663000997155905, "rps_doc_ldnoobw_words": 0, "rps_doc_lorem_ipsum": 0, "rps_doc_stop_word_fraction": 0.3641975224018097, "rps_doc_ut1_blacklist": 0, "rps_doc_frac_all_caps_words": 0.022633740678429604, "rps_doc_frac_lines_end_with_ellipsis": 0.019999999552965164, "rps_doc_frac_no_alph_words": 0.20987653732299805, "rps_doc_frac_unique_words": 0.5490196347236633, "rps_doc_mean_word_length": 5.112044811248779, "rps_doc_num_sentences": 32, "rps_doc_symbol_to_word_ratio": 0.0041152299381792545, "rps_doc_unigram_entropy": 4.902354717254639, "rps_doc_word_count": 357, "rps_doc_frac_chars_dupe_10grams": 0, "rps_doc_frac_chars_dupe_5grams": 0.017534250393509865, "rps_doc_frac_chars_dupe_6grams": 0, "rps_doc_frac_chars_dupe_7grams": 0, "rps_doc_frac_chars_dupe_8grams": 0, "rps_doc_frac_chars_dupe_9grams": 0, "rps_doc_frac_chars_top_2gram": 0.030684929341077805, "rps_doc_frac_chars_top_3gram": 0.017534250393509865, "rps_doc_frac_chars_top_4gram": 0.012054789811372757, "rps_doc_books_importance": -162.31939697265625, "rps_doc_books_importance_length_correction": -162.31939697265625, "rps_doc_openwebtext_importance": -103.46517181396484, "rps_doc_openwebtext_importance_length_correction": -103.46517181396484, "rps_doc_wikipedia_importance": -40.28302001953125, "rps_doc_wikipedia_importance_length_correction": -40.28302001953125 }, "fasttext": { "dclm": 0.6000203490257263, "english": 0.9179166555404663, "fineweb_edu_approx": 1.5639292001724243, "eai_general_math": 0.00033330998849123716, "eai_open_web_math": 0.02718365006148815, "eai_web_code": 0.0016858599847182631 } }
{ "free_decimal_correspondence": { "primary": { "code": "005.133", "labels": { "level_1": "General works, books and libraries, information sciences", "level_2": "", "level_3": "Computer programming" } }, "secondary": { "code": "005.1", "labels": { "level_1": "General works, books and libraries, information sciences", "level_2": "", "level_3": "Computer programming" } } }, "bloom_cognitive_process": { "primary": { "code": "3", "label": "Apply" }, "secondary": { "code": "2", "label": "Understand" } }, "bloom_knowledge_domain": { "primary": { "code": "3", "label": "Procedural" }, "secondary": { "code": "2", "label": "Conceptual" } }, "document_type_v1": { "primary": { "code": "5", "label": "Social/Forum" }, "secondary": { "code": "-1", "label": "Abstain" } }, "extraction_artifacts": { "primary": { "code": "3", "label": "Irrelevant Content" }, "secondary": { "code": "-1", "label": "Abstain" } }, "missing_content": { "primary": { "code": "0", "label": "No missing content" }, "secondary": { "code": "-1", "label": "Abstain" } }, "document_type_v2": { "primary": { "code": "18", "label": "Q&A Forum" }, "secondary": { "code": "8", "label": "Documentation" } }, "reasoning_depth": { "primary": { "code": "3", "label": "Intermediate Reasoning" }, "secondary": { "code": "2", "label": "Basic Reasoning" } }, "technical_correctness": { "primary": { "code": "3", "label": "Mostly Correct" }, "secondary": { "code": "4", "label": "Highly Correct" } }, "education_level": { "primary": { "code": "3", "label": "Undergraduate Level" }, "secondary": { "code": "4", "label": "Graduate/Expert Level" } } }
672f1e42c33a7f9846924a2431ea77df
-744,657,546,709,665,900
When your home button stops working on your Android smartphone, your world is on standstill as there is very little you can do with your smartphone in this condition. In this article, we show you how to work around this problem yourself.   The home button is one of the most important, if not the most important button on a smartphone’s display. It lets users enjoy a better smartphone experience and navigate between apps almost seamlessly. However, there are times when this button starts malfunctioning. It could be due to a software glitch or some hardware issues, whatever the case is, a home button that malfunctions is annoying and can make your device unnecessarily slow. And this is why today, we are going to show you some quick tips to take care of your Android smartphone by yourself when your home button stops working. • Restart your smartphone: As with many issues that beguile Android smartphones, restarting your smartphone when your home button stops working can help you regain control of your device and make it return to its normal operation. You can restart your smartphone by holding down the power button and clicking on the reboot option when it comes up. Your Android smartphone should start working properly once it comes on. oppo Reno3 pro homescreen in hand what to do when home button stops working • Try resetting the phone in recovery mode: To reboot the Android device in recovery mode, follow these simple steps. 1. Switch off your device. 2. Hold down the power and volume down buttons simultaneously. 3. Select Recovery mode. 4. Select Factory Reset using the volume buttons to navigate the options.   • Try using a third-party app: You can also try using a third-party app when your home button stops working. One of such apps is Button Savior, which is available on Google Play Store. This app can help you navigate your Android device’s functions whenever your home button stops working, all you need to do is: 1. Download Button Savior on your phone through the link above. 2. Click open options by clicking on the toggle arrow at the edge of the screen. 3. Pick the icon that helps you complete the action you want on your device. You can use this for a myriad of functions.   • Try clearing your cache partition: Your cache partition is a group of logs, temporary files and other bits of information from apps you have downloaded on your device. when you don’t clean this collection from time to time, it may begin to slow the phone and make the home button stop working. To tackle this problem, follow these simple steps: 1. Turn off your phone and hold down the power and volume down buttons. 2. Pick Recovery mode. 3. Select Wipe Cache partition. 4. Confirm your selection.   • Try a factory reset: If any of the tips above do not work, then you may need to go down the lonely, dark path of a factory reset. Just before you do this, ensure your device is in Safe mode. This will tell you if the source of the problem is a software or app malfunction. If you can, back up your data before trying a factory reset so you do not lose any important information. To carry out this reset, follow these steps: 1. Navigate to Settings. 2. Select Backup and Reset. 3. Select Factory data reset. 4. Select Reset phone. 5. Insert your device’s password (if you have one). 6. Select Erase everything.   These tips are particularly helpful when your home button stops working due to a software problem. If it is a hardware issue, these steps may not suffice and you may have to consult a technician before things get worse.   You are reading Mobility Arena, the essential guide for mobile phone users around the world. Be sure to check out our homepage for all the latest news and resources. To be notified of the most important articles and the best smartphone deals, join our WhatsApp Group and follow us on Instagram, Facebook, Twitter, and YouTube. Don’t know where to start? Check out our reviews. f
{ "url": "https://ezefidelity.com/what-to-do-when-your-home-button-stops-working-on-your-android-smartphone-mobility-arena-mobile-phone-smartphone-cell-phone-reviews/", "source_domain": "ezefidelity.com", "snapshot_id": "crawl=CC-MAIN-2021-49", "warc_metadata": { "Content-Length": "178451", "Content-Type": "application/http; msgtype=response", "WARC-Block-Digest": "sha1:2WONTCWDTWARTRMCF4LXCEAKZNQFHUQE", "WARC-Concurrent-To": "<urn:uuid:5dae0fdb-1a3a-4a03-b0bd-486796bfa2b9>", "WARC-Date": "2021-12-01T15:58:08Z", "WARC-IP-Address": "198.54.119.222", "WARC-Identified-Payload-Type": "text/html", "WARC-Payload-Digest": "sha1:GR6JLHW3TJNIXBEAHOFCCH6MTXJJURFO", "WARC-Record-ID": "<urn:uuid:703efd92-4cd7-430d-a62c-190c82680219>", "WARC-Target-URI": "https://ezefidelity.com/what-to-do-when-your-home-button-stops-working-on-your-android-smartphone-mobility-arena-mobile-phone-smartphone-cell-phone-reviews/", "WARC-Truncated": null, "WARC-Type": "response", "WARC-Warcinfo-ID": "<urn:uuid:d5378326-9b19-478d-8bba-45ee81835aa7>" }, "warc_info": "isPartOf: CC-MAIN-2021-49\r\npublisher: Common Crawl\r\ndescription: Wide crawl of the web for November/December 2021\r\noperator: Common Crawl Admin ([email protected])\r\nhostname: ip-10-67-67-123\r\nsoftware: Apache Nutch 1.18 (modified, https://github.com/commoncrawl/nutch/)\r\nrobots: checked via crawler-commons 1.3-SNAPSHOT (https://github.com/crawler-commons/crawler-commons)\r\nformat: WARC File Format 1.1\r\nconformsTo: https://iipc.github.io/warc-specifications/specifications/warc-format/warc-1.1/" }
{ "line_start_idx": [ 0, 238, 239, 241, 242, 444, 445, 836, 837, 1259, 1260, 1336, 1337, 1457, 1486, 1551, 1578, 1654, 1655, 1657, 1658, 1972, 2038, 2121, 2244, 2245, 2247, 2248, 2597, 2671, 2696, 2730, 2759, 2760, 2762, 2763, 3191, 3218, 3248, 3280, 3305, 3359, 3389, 3390, 3392, 3393, 3613, 3614, 3616, 3617, 3994, 3995, 3996, 3997 ], "line_end_idx": [ 238, 239, 241, 242, 444, 445, 836, 837, 1259, 1260, 1336, 1337, 1457, 1486, 1551, 1578, 1654, 1655, 1657, 1658, 1972, 2038, 2121, 2244, 2245, 2247, 2248, 2597, 2671, 2696, 2730, 2759, 2760, 2762, 2763, 3191, 3218, 3248, 3280, 3305, 3359, 3389, 3390, 3392, 3393, 3613, 3614, 3616, 3617, 3994, 3995, 3996, 3997, 3998 ] }
{ "red_pajama_v2": { "ccnet_original_length": 3998, "ccnet_original_nlines": 53, "rps_doc_curly_bracket": 0, "rps_doc_ldnoobw_words": 0, "rps_doc_lorem_ipsum": 0, "rps_doc_stop_word_fraction": 0.4157303273677826, "rps_doc_ut1_blacklist": 0, "rps_doc_frac_all_caps_words": 0, "rps_doc_frac_lines_end_with_ellipsis": 0, "rps_doc_frac_no_alph_words": 0.1548064947128296, "rps_doc_frac_unique_words": 0.3843930661678314, "rps_doc_mean_word_length": 4.52890157699585, "rps_doc_num_sentences": 62, "rps_doc_symbol_to_word_ratio": 0, "rps_doc_unigram_entropy": 5.043166637420654, "rps_doc_word_count": 692, "rps_doc_frac_chars_dupe_10grams": 0, "rps_doc_frac_chars_dupe_5grams": 0.1155073419213295, "rps_doc_frac_chars_dupe_6grams": 0.07083599269390106, "rps_doc_frac_chars_dupe_7grams": 0.022973839193582535, "rps_doc_frac_chars_dupe_8grams": 0.022973839193582535, "rps_doc_frac_chars_dupe_9grams": 0, "rps_doc_frac_chars_top_2gram": 0.031908098608255386, "rps_doc_frac_chars_top_3gram": 0.03350351005792618, "rps_doc_frac_chars_top_4gram": 0.04913847893476486, "rps_doc_books_importance": -379.8621826171875, "rps_doc_books_importance_length_correction": -379.8621826171875, "rps_doc_openwebtext_importance": -203.4541473388672, "rps_doc_openwebtext_importance_length_correction": -203.4541473388672, "rps_doc_wikipedia_importance": -135.88137817382812, "rps_doc_wikipedia_importance_length_correction": -135.88137817382812 }, "fasttext": { "dclm": 0.03478759899735451, "english": 0.8822320699691772, "fineweb_edu_approx": 1.5579404830932617, "eai_general_math": 0.0007739100256003439, "eai_open_web_math": 0.09439439326524734, "eai_web_code": 0.0010570900049060583 } }
{ "free_decimal_correspondence": { "primary": { "code": "004.16", "labels": { "level_1": "General works, books and libraries, information sciences", "level_2": "", "level_3": "Computers and Computer science" } }, "secondary": { "code": "005.1", "labels": { "level_1": "General works, books and libraries, information sciences", "level_2": "", "level_3": "Computer programming" } } }, "bloom_cognitive_process": { "primary": { "code": "3", "label": "Apply" }, "secondary": { "code": "2", "label": "Understand" } }, "bloom_knowledge_domain": { "primary": { "code": "3", "label": "Procedural" }, "secondary": { "code": "2", "label": "Conceptual" } }, "document_type_v1": { "primary": { "code": "3", "label": "Reference/Encyclopedic/Educational" }, "secondary": { "code": "-1", "label": "Abstain" } }, "extraction_artifacts": { "primary": { "code": "0", "label": "No Artifacts" }, "secondary": { "code": "-1", "label": "Abstain" } }, "missing_content": { "primary": { "code": "0", "label": "No missing content" }, "secondary": { "code": "-1", "label": "Abstain" } }, "document_type_v2": { "primary": { "code": "23", "label": "Tutorial" }, "secondary": { "code": "21", "label": "Customer Support" } }, "reasoning_depth": { "primary": { "code": "2", "label": "Basic Reasoning" }, "secondary": { "code": "1", "label": "No Reasoning" } }, "technical_correctness": { "primary": { "code": "4", "label": "Highly Correct" }, "secondary": { "code": "3", "label": "Mostly Correct" } }, "education_level": { "primary": { "code": "1", "label": "General Audience" }, "secondary": { "code": "2", "label": "High School Level" } } }
672f1e42c33a7f9846924a2431ea77df
6,652,783,941,065,234,000
Algebra 2 Common Core Published by Prentice Hall ISBN 10: 0133186024 ISBN 13: 978-0-13318-602-4 Chapter 1 - Expressions, Equations, and Inequalities - 1-2 Properties of Real Numbers - Practice and Problem-Solving Exercises: 72 Answer B Work Step by Step $\pi=\approx3.14$, and $\sqrt 8=$ a little less than 3 because $\sqrt 9=3$. So, we know that $\sqrt 8$ is the smallest because it is less than 3, $\pi$ is in the middle, and $3.5$ is the greatest. Update this answer! You can help us out by revising, improving and updating this answer. Update this answer After you claim an answer you’ll have 24 hours to send in a draft. An editor will review the submission and either publish your submission or provide feedback.
{ "url": "https://www.gradesaver.com/textbooks/math/algebra/algebra-2-common-core/chapter-1-expressions-equations-and-inequalities-1-2-properties-of-real-numbers-practice-and-problem-solving-exercises-page-17/72", "source_domain": "www.gradesaver.com", "snapshot_id": "crawl=CC-MAIN-2018-26", "warc_metadata": { "Content-Length": "84662", "Content-Type": "application/http; msgtype=response", "WARC-Block-Digest": "sha1:NTNMFPJK5WFBJICAG34QGJOXXPC7XFCF", "WARC-Concurrent-To": "<urn:uuid:50129ebd-8065-4fab-9f30-aeaecbbfdf8e>", "WARC-Date": "2018-06-22T02:16:31Z", "WARC-IP-Address": "107.23.12.46", "WARC-Identified-Payload-Type": "text/html", "WARC-Payload-Digest": "sha1:BEZSPQVVLD33CUP5ICYMDOYPTC4UB3BZ", "WARC-Record-ID": "<urn:uuid:1418645c-891c-45d8-9614-88186f682bf6>", "WARC-Target-URI": "https://www.gradesaver.com/textbooks/math/algebra/algebra-2-common-core/chapter-1-expressions-equations-and-inequalities-1-2-properties-of-real-numbers-practice-and-problem-solving-exercises-page-17/72", "WARC-Truncated": null, "WARC-Type": "response", "WARC-Warcinfo-ID": "<urn:uuid:dace13bf-12a5-4510-884f-7529f5c9376b>" }, "warc_info": "robots: classic\r\nhostname: ip-10-136-156-31.ec2.internal\r\nsoftware: Nutch 1.6 (CC)\r\nisPartOf: CC-MAIN-2018-26\r\noperator: Common Crawl Admin\r\ndescription: Wide crawl of the web for June 2018\r\npublisher: Common Crawl\r\nformat: WARC File Format 1.0\r\nconformsTo: http://bibnum.bnf.fr/WARC/WARC_ISO_28500_version1_latestdraft.pdf" }
{ "line_start_idx": [ 0, 22, 23, 50, 70, 97, 98, 229, 230, 237, 238, 240, 241, 259, 260, 457, 477, 478, 547, 548, 567, 568 ], "line_end_idx": [ 22, 23, 50, 70, 97, 98, 229, 230, 237, 238, 240, 241, 259, 260, 457, 477, 478, 547, 548, 567, 568, 727 ] }
{ "red_pajama_v2": { "ccnet_original_length": 727, "ccnet_original_nlines": 21, "rps_doc_curly_bracket": 0, "rps_doc_ldnoobw_words": 0, "rps_doc_lorem_ipsum": 0, "rps_doc_stop_word_fraction": 0.28823527693748474, "rps_doc_ut1_blacklist": 0, "rps_doc_frac_all_caps_words": 0.01764705963432789, "rps_doc_frac_lines_end_with_ellipsis": 0, "rps_doc_frac_no_alph_words": 0.3705882430076599, "rps_doc_frac_unique_words": 0.7142857313156128, "rps_doc_mean_word_length": 4.613445281982422, "rps_doc_num_sentences": 8, "rps_doc_symbol_to_word_ratio": 0, "rps_doc_unigram_entropy": 4.293430805206299, "rps_doc_word_count": 119, "rps_doc_frac_chars_dupe_10grams": 0, "rps_doc_frac_chars_dupe_5grams": 0, "rps_doc_frac_chars_dupe_6grams": 0, "rps_doc_frac_chars_dupe_7grams": 0, "rps_doc_frac_chars_dupe_8grams": 0, "rps_doc_frac_chars_dupe_9grams": 0, "rps_doc_frac_chars_top_2gram": 0.054644808173179626, "rps_doc_frac_chars_top_3gram": 0.03278689086437225, "rps_doc_frac_chars_top_4gram": 0, "rps_doc_books_importance": -78.15816497802734, "rps_doc_books_importance_length_correction": -78.17105102539062, "rps_doc_openwebtext_importance": -45.739891052246094, "rps_doc_openwebtext_importance_length_correction": -45.752777099609375, "rps_doc_wikipedia_importance": -19.61572265625, "rps_doc_wikipedia_importance_length_correction": -19.628610610961914 }, "fasttext": { "dclm": 0.018165409564971924, "english": 0.8112767338752747, "fineweb_edu_approx": 1.7164806127548218, "eai_general_math": 0.6291782259941101, "eai_open_web_math": 0.38890838623046875, "eai_web_code": 0.0021310399752110243 } }
{ "free_decimal_correspondence": { "primary": { "code": "512", "labels": { "level_1": "Science and Natural history", "level_2": "Mathematics", "level_3": "Algebra" } }, "secondary": { "code": "510", "labels": { "level_1": "Science and Natural history", "level_2": "Mathematics", "level_3": "" } } }, "bloom_cognitive_process": { "primary": { "code": "2", "label": "Understand" }, "secondary": { "code": "3", "label": "Apply" } }, "bloom_knowledge_domain": { "primary": { "code": "2", "label": "Conceptual" }, "secondary": { "code": "3", "label": "Procedural" } }, "document_type_v1": { "primary": { "code": "3", "label": "Reference/Encyclopedic/Educational" }, "secondary": { "code": "-1", "label": "Abstain" } }, "extraction_artifacts": { "primary": { "code": "0", "label": "No Artifacts" }, "secondary": { "code": "3", "label": "Irrelevant Content" } }, "missing_content": { "primary": { "code": "0", "label": "No missing content" }, "secondary": { "code": "-1", "label": "Abstain" } }, "document_type_v2": { "primary": { "code": "23", "label": "Tutorial" }, "secondary": { "code": "21", "label": "Customer Support" } }, "reasoning_depth": { "primary": { "code": "2", "label": "Basic Reasoning" }, "secondary": { "code": "3", "label": "Intermediate Reasoning" } }, "technical_correctness": { "primary": { "code": "4", "label": "Highly Correct" }, "secondary": { "code": "3", "label": "Mostly Correct" } }, "education_level": { "primary": { "code": "2", "label": "High School Level" }, "secondary": { "code": "3", "label": "Undergraduate Level" } } }
672f1e42c33a7f9846924a2431ea77df
-1,375,032,755,728,828,700
Bug #10468 Lengthen url allowed for ArchivesSpace settings Added by Sarah Romkey over 5 years ago. Status:NewStart date:10/24/2016 Priority:MediumDue date: Assignee:-% Done: 0% Category:Access Target version:- Google Code Legacy ID: Pull Request: Sponsored:No Requires documentation: Description As reported on the user forum, the character length allowed for the ArchivesSpace settings is sometimes not long enough. Unless there is a compelling reason to limit it, I would suggest making it unlimited. Also available in: Atom PDF
{ "url": "https://projects.artefactual.com/issues/10468", "source_domain": "projects.artefactual.com", "snapshot_id": "crawl=CC-MAIN-2022-21", "warc_metadata": { "Content-Length": "8030", "Content-Type": "application/http; msgtype=response", "WARC-Block-Digest": "sha1:BZRAMBJEIQBKTAK3MVXUE5NEL3VLISE6", "WARC-Concurrent-To": "<urn:uuid:f71185df-bd3d-4c1e-8a46-a3e76bbf13e2>", "WARC-Date": "2022-05-28T17:39:53Z", "WARC-IP-Address": "167.114.216.99", "WARC-Identified-Payload-Type": "text/html", "WARC-Payload-Digest": "sha1:RN6LWXGUC4PZIZFHFD7NVW5UNZ5G4MXG", "WARC-Record-ID": "<urn:uuid:49063e2d-0f3b-41e0-8104-49d1d7855375>", "WARC-Target-URI": "https://projects.artefactual.com/issues/10468", "WARC-Truncated": null, "WARC-Type": "response", "WARC-Warcinfo-ID": "<urn:uuid:f052ccd2-6f73-4f5b-80f7-4fbf372a21a9>" }, "warc_info": "isPartOf: CC-MAIN-2022-21\r\npublisher: Common Crawl\r\ndescription: Wide crawl of the web for May 2022\r\noperator: Common Crawl Admin ([email protected])\r\nhostname: ip-10-67-67-177\r\nsoftware: Apache Nutch 1.18 (modified, https://github.com/commoncrawl/nutch/)\r\nrobots: checked via crawler-commons 1.3-SNAPSHOT (https://github.com/crawler-commons/crawler-commons)\r\nformat: WARC File Format 1.1\r\nconformsTo: https://iipc.github.io/warc-specifications/specifications/warc-format/warc-1.1/" }
{ "line_start_idx": [ 0, 11, 12, 60, 61, 101, 102, 134, 159, 177, 178, 181, 182, 198, 215, 252, 289, 290, 302, 303, 510, 511 ], "line_end_idx": [ 11, 12, 60, 61, 101, 102, 134, 159, 177, 178, 181, 182, 198, 215, 252, 289, 290, 302, 303, 510, 511, 538 ] }
{ "red_pajama_v2": { "ccnet_original_length": 538, "ccnet_original_nlines": 21, "rps_doc_curly_bracket": 0, "rps_doc_ldnoobw_words": 0, "rps_doc_lorem_ipsum": 0, "rps_doc_stop_word_fraction": 0.20192308723926544, "rps_doc_ut1_blacklist": 0, "rps_doc_frac_all_caps_words": 0.028846150264143944, "rps_doc_frac_lines_end_with_ellipsis": 0, "rps_doc_frac_no_alph_words": 0.26923078298568726, "rps_doc_frac_unique_words": 0.8933333158493042, "rps_doc_mean_word_length": 5.74666690826416, "rps_doc_num_sentences": 4, "rps_doc_symbol_to_word_ratio": 0.009615380316972733, "rps_doc_unigram_entropy": 4.16264009475708, "rps_doc_word_count": 75, "rps_doc_frac_chars_dupe_10grams": 0, "rps_doc_frac_chars_dupe_5grams": 0, "rps_doc_frac_chars_dupe_6grams": 0, "rps_doc_frac_chars_dupe_7grams": 0, "rps_doc_frac_chars_dupe_8grams": 0, "rps_doc_frac_chars_dupe_9grams": 0, "rps_doc_frac_chars_top_2gram": 0.046403709799051285, "rps_doc_frac_chars_top_3gram": 0, "rps_doc_frac_chars_top_4gram": 0, "rps_doc_books_importance": -49.216670989990234, "rps_doc_books_importance_length_correction": -62.36472702026367, "rps_doc_openwebtext_importance": -24.30844497680664, "rps_doc_openwebtext_importance_length_correction": -37.456504821777344, "rps_doc_wikipedia_importance": -24.08222198486328, "rps_doc_wikipedia_importance_length_correction": -37.23027801513672 }, "fasttext": { "dclm": 0.08564674854278564, "english": 0.8255191445350647, "fineweb_edu_approx": 1.2808654308319092, "eai_general_math": 0.006755889859050512, "eai_open_web_math": 0.4575389623641968, "eai_web_code": 0.0024645898956805468 } }
{ "free_decimal_correspondence": { "primary": { "code": "005.4", "labels": { "level_1": "General works, books and libraries, information sciences", "level_2": "", "level_3": "Computer programming" } }, "secondary": { "code": "004.67", "labels": { "level_1": "General works, books and libraries, information sciences", "level_2": "", "level_3": "Computers and Computer science" } } }, "bloom_cognitive_process": { "primary": { "code": "2", "label": "Understand" }, "secondary": { "code": "3", "label": "Apply" } }, "bloom_knowledge_domain": { "primary": { "code": "2", "label": "Conceptual" }, "secondary": { "code": "3", "label": "Procedural" } }, "document_type_v1": { "primary": { "code": "4", "label": "Code/Software" }, "secondary": { "code": "-1", "label": "Abstain" } }, "extraction_artifacts": { "primary": { "code": "0", "label": "No Artifacts" }, "secondary": { "code": "3", "label": "Irrelevant Content" } }, "missing_content": { "primary": { "code": "0", "label": "No missing content" }, "secondary": { "code": "-1", "label": "Abstain" } }, "document_type_v2": { "primary": { "code": "8", "label": "Documentation" }, "secondary": { "code": "21", "label": "Customer Support" } }, "reasoning_depth": { "primary": { "code": "2", "label": "Basic Reasoning" }, "secondary": { "code": "1", "label": "No Reasoning" } }, "technical_correctness": { "primary": { "code": "6", "label": "Not Applicable/Indeterminate" }, "secondary": { "code": "4", "label": "Highly Correct" } }, "education_level": { "primary": { "code": "1", "label": "General Audience" }, "secondary": { "code": "2", "label": "High School Level" } } }
672f1e42c33a7f9846924a2431ea77df
-7,454,604,773,460,016,000
2 From this article, I can understand that a punishment-based bidirectional payment channel (the type used by Lightning) would contain roughly this bitcoin script as one of the outputs: OP_IF “+7 days” OP_CHECKSEQUENCEVERIFY OP_DROP <Bob pubkey> OP_CHECKSIG OP_ELSE OP_SHA256 <sha256(Bob secret)> OP_EQUALVERIFY <Alice pubkey> OP_CHECKSIG OP_ENDIF I guess there are some block explorers that let you visualize the real script being used in a transaction, so can anyone point me to a real LN transaction (in mainnet) that follows the template above? Thanks 2 The actual script implemented on Lightning Network is a little bit different from what you quoted. This is the script below and the specification is here OP_IF # Penalty transaction <revocationpubkey> OP_ELSE `to_self_delay` OP_CSV OP_DROP <local_delayedpubkey> OP_ENDIF OP_CHECKSIG I've created that transaction on Testnet as the following, Txid: 4f8d776c85b1fc15c1125e7043a9aee70e33f0793b472823e3946a8de075bec4 Rawdata: 6321026644cb387614f66421d14da3596c21cffa239011416c9adf3f351ee8551a9fc767029000b27521029654f80732769d7c435a184a3559f12178315526c53bbf003349390811c7590a68ac Decoded by here { "result": { "asm": "OP_IF 026644cb387614f66421d14da3596c21cffa239011416c9adf3f351ee8551a9fc7 OP_ELSE 144 OP_CHECKSEQUENCEVERIFY OP_DROP 029654f80732769d7c435a184a3559f12178315526c53bbf003349390811c7590a OP_ENDIF OP_CHECKSIG", "type": "nonstandard", "p2sh": "35M7MGVGd3ajyiTMvriUiyUM3H2cT5sgb4" }, "error": null, "id": null } | improve this answer | | 0 blockchain.com has a "show scripts" link. Here's a mainnet LN transaction: https://www.blockchain.com/btc/tx/2facfe9147121b64761bef31c3808f43f92311349dbba01217bb666f0bd9b83c?show_adv=true From this channel: https://1ml.com/channel/586011110465142785 | improve this answer | | • 2 mmm but those scripts don't match the template I posted in my question – knocte Aug 10 '18 at 8:56 Your Answer By clicking “Post Your Answer”, you agree to our terms of service, privacy policy and cookie policy Not the answer you're looking for? Browse other questions tagged or ask your own question.
{ "url": "https://bitcoin.stackexchange.com/questions/74719/lightning-close-channel-transaction-example", "source_domain": "bitcoin.stackexchange.com", "snapshot_id": "crawl=CC-MAIN-2020-24", "warc_metadata": { "Content-Length": "148479", "Content-Type": "application/http; msgtype=response", "WARC-Block-Digest": "sha1:EHMBFW6WQ745VMCLLVOU5FHC3RXT37MM", "WARC-Concurrent-To": "<urn:uuid:6e7a7825-7ef4-4038-9c77-dc42c7de359a>", "WARC-Date": "2020-05-30T19:06:09Z", "WARC-IP-Address": "151.101.65.69", "WARC-Identified-Payload-Type": "text/html", "WARC-Payload-Digest": "sha1:5NEVET3HYRL35IDMNEA7Y5YYBST6KF7X", "WARC-Record-ID": "<urn:uuid:bb01e066-fe04-4b6a-9299-8668ceec0f1e>", "WARC-Target-URI": "https://bitcoin.stackexchange.com/questions/74719/lightning-close-channel-transaction-example", "WARC-Truncated": null, "WARC-Type": "response", "WARC-Warcinfo-ID": "<urn:uuid:2ed08153-6963-4690-821c-07f34a6dcd88>" }, "warc_info": "isPartOf: CC-MAIN-2020-24\r\npublisher: Common Crawl\r\ndescription: Wide crawl of the web for May/June 2020\r\noperator: Common Crawl Admin ([email protected])\r\nhostname: ip-10-67-67-236.ec2.internal\r\nsoftware: Apache Nutch 1.16 (modified, https://github.com/commoncrawl/nutch/)\r\nrobots: checked via crawler-commons 1.1-SNAPSHOT (https://github.com/crawler-commons/crawler-commons)\r\nformat: WARC File Format 1.1\r\nconformsTo: http://iipc.github.io/warc-specifications/specifications/warc-format/warc-1.1/" }
{ "line_start_idx": [ 0, 2, 3, 187, 188, 194, 235, 260, 268, 314, 341, 350, 351, 552, 553, 560, 561, 563, 564, 718, 719, 725, 751, 774, 782, 802, 813, 825, 851, 860, 872, 873, 932, 933, 1004, 1005, 1014, 1015, 1170, 1171, 1187, 1188, 1190, 1206, 1428, 1459, 1512, 1519, 1538, 1553, 1555, 1581, 1583, 1584, 1626, 1627, 1773, 1774, 1836, 1837, 1863, 1869, 1972, 1973, 1985, 1986, 2086, 2087 ], "line_end_idx": [ 2, 3, 187, 188, 194, 235, 260, 268, 314, 341, 350, 351, 552, 553, 560, 561, 563, 564, 718, 719, 725, 751, 774, 782, 802, 813, 825, 851, 860, 872, 873, 932, 933, 1004, 1005, 1014, 1015, 1170, 1171, 1187, 1188, 1190, 1206, 1428, 1459, 1512, 1519, 1538, 1553, 1555, 1581, 1583, 1584, 1626, 1627, 1773, 1774, 1836, 1837, 1863, 1869, 1972, 1973, 1985, 1986, 2086, 2087, 2177 ] }
{ "red_pajama_v2": { "ccnet_original_length": 2177, "ccnet_original_nlines": 67, "rps_doc_curly_bracket": 0.0018373900093138218, "rps_doc_ldnoobw_words": 0, "rps_doc_lorem_ipsum": 0, "rps_doc_stop_word_fraction": 0.2631579041481018, "rps_doc_ut1_blacklist": 0, "rps_doc_frac_all_caps_words": 0.07894737273454666, "rps_doc_frac_lines_end_with_ellipsis": 0, "rps_doc_frac_no_alph_words": 0.2923976480960846, "rps_doc_frac_unique_words": 0.6553191542625427, "rps_doc_mean_word_length": 7.310638427734375, "rps_doc_num_sentences": 10, "rps_doc_symbol_to_word_ratio": 0.002923979889601469, "rps_doc_unigram_entropy": 4.843400955200195, "rps_doc_word_count": 235, "rps_doc_frac_chars_dupe_10grams": 0, "rps_doc_frac_chars_dupe_5grams": 0, "rps_doc_frac_chars_dupe_6grams": 0, "rps_doc_frac_chars_dupe_7grams": 0, "rps_doc_frac_chars_dupe_8grams": 0, "rps_doc_frac_chars_dupe_9grams": 0, "rps_doc_frac_chars_top_2gram": 0.009313150309026241, "rps_doc_frac_chars_top_3gram": 0.019790450111031532, "rps_doc_frac_chars_top_4gram": 0, "rps_doc_books_importance": -137.3669891357422, "rps_doc_books_importance_length_correction": -137.3669891357422, "rps_doc_openwebtext_importance": -57.06635665893555, "rps_doc_openwebtext_importance_length_correction": -57.06635665893555, "rps_doc_wikipedia_importance": -38.73905563354492, "rps_doc_wikipedia_importance_length_correction": -38.73905563354492 }, "fasttext": { "dclm": 0.4063562750816345, "english": 0.7598992586135864, "fineweb_edu_approx": 1.515447735786438, "eai_general_math": 0.00004292000085115433, "eai_open_web_math": 0.07007443904876709, "eai_web_code": -0.000009299999874201603 } }
{ "free_decimal_correspondence": { "primary": { "code": "004.678", "labels": { "level_1": "General works, books and libraries, information sciences", "level_2": "", "level_3": "Computers and Computer science" } }, "secondary": { "code": "332.872", "labels": { "level_1": "Social sciences", "level_2": "Economics", "level_3": "Finance" } } }, "bloom_cognitive_process": { "primary": { "code": "2", "label": "Understand" }, "secondary": { "code": "3", "label": "Apply" } }, "bloom_knowledge_domain": { "primary": { "code": "2", "label": "Conceptual" }, "secondary": { "code": "3", "label": "Procedural" } }, "document_type_v1": { "primary": { "code": "5", "label": "Social/Forum" }, "secondary": { "code": "3", "label": "Reference/Encyclopedic/Educational" } }, "extraction_artifacts": { "primary": { "code": "3", "label": "Irrelevant Content" }, "secondary": { "code": "-1", "label": "Abstain" } }, "missing_content": { "primary": { "code": "2", "label": "Click Here References" }, "secondary": { "code": "-1", "label": "Abstain" } }, "document_type_v2": { "primary": { "code": "18", "label": "Q&A Forum" }, "secondary": { "code": "8", "label": "Documentation" } }, "reasoning_depth": { "primary": { "code": "2", "label": "Basic Reasoning" }, "secondary": { "code": "3", "label": "Intermediate Reasoning" } }, "technical_correctness": { "primary": { "code": "3", "label": "Mostly Correct" }, "secondary": { "code": "4", "label": "Highly Correct" } }, "education_level": { "primary": { "code": "3", "label": "Undergraduate Level" }, "secondary": { "code": "4", "label": "Graduate/Expert Level" } } }
672f1e42c33a7f9846924a2431ea77df
580,210,067,291,970,600
[Free] 2017(Oct) EnsurePass Examcollection Microsoft 70-743 Dumps with VCE and PDF 51-60 Ensurepass.com : Ensure you pass the IT Exams 2017 Oct Microsoft Official New Released 70-743 100% Free Download! 100% Pass Guaranteed! http://www.EnsurePass.com/70-743.html Upgrading Your Skills to MCSA: Windows Server 2016 Question No: 51 HOTSPOT You have a server named Server1 that runs Windows Server 2016. Server1 has the Web Application Proxy role service installed. You are publishing an application named App1 that will use Integrated Windows authentication as shown in the following graphic. Ensurepass 2017 PDF and VCE Use the drop-down menus to select the answer choice that completes each statement based on the information presented in the graphic. Ensurepass 2017 PDF and VCE Answer: Ensurepass 2017 PDF and VCE Explanation: Ensurepass 2017 PDF and VCE Publish an Integrated Windows authenticated-based Application for WebBrowser Clients Step 1: (configure the Backend server SPN – see first bulleted item below) Before you begin, make sure that you have done the following: Step 2: http//server2.contoso.com/publish/app1 Use the same URL as the backend server URL. Web Application Proxy can translate host names in URLs, but cannot translate path names. Therefore, you can enter different host names, but you must enter the same path name. For example, you can enter an external URL of https://apps.contoso.com/app1/ and a backend server URL of http://app-server/app1/. However, you cannot enter an external URL of https://apps.contoso.com/app1/ and a backend server URL of https://apps.contoso.com/internal-app1/. References: https://technet.microsoft.com/en-us/library/dn383640(v=ws.11).aspx Question No: 52 You have a Hyper-V host named Server1 that runs Windows Server 2016. Server1 hosts a virtual machine named VM1. You need to provide VM1 with direct access to a graphics processing unit (GPU) on Server1. What should you do first? 1. OnVM1, install the Quality Windows Audio Video Experience (qWave) feature. 2. Disable the display adapter device on Server1. 3. In the settings of VM1, add a RemoteFX 3D Video Adapter. 4. Dismount the display adapter on Server1. Answer: B Explanation: Before the physical device is allowed to be passed through to the VM, the device must be disabled on the host system. The physical device must be accessible/available exclusively to the VM only. References: Introduction to Windows Server 2016 Hyper-V DiscreteDevice Assignment, page 5 https://lenovopress.com/lp0088.pdf Question No: 53 Note: This question is part of a series of questions that present the same scenario. Each question in the series contains a unique solution that might meet the stated goals. Some question sets might have more than one correct solution, while others might not have a correct solution. After your answer a question in this section, you will NOT be able to return to it. As a result, these questions will not appear in the review screen. Your network contains an Active Directory forest named contoso.com. You need to identify which server is the schema master. Solution: From a command prompt, you run netdom query fsmo. Does this meet the goal? 1. Yes 2. No Answer: A Explanation: References: https://blogs.technet.microsoft.com/mempson/2007/11/08/how-to-find-out- who-has-your-fsmo-roles/ Question No: 54 HOTSPOT You have a Hyper-V host that runs Windows Server 2016. The Hyper-V host has a virtual machine named VM1. You have a VHD named VHD1.vhdx that has a generalized image of Windows Server 2016. You plan to create multiple virtual machines that will use the generalized image. You need to create differencing disks based on VHD1.vhdx. What command should you run? To answer, select the appropriate options in the answer area. Ensurepass 2017 PDF and VCE Answer: Ensurepass 2017 PDF and VCE Explanation: Ensurepass 2017 PDF and VCE References: https://technet.microsoft.com/en-us/itpro/powershell/windows/hyper-v/new-vhd Question No: 55 Note: This question is part of a series of questions that present the same scenario. Each question in the series contains a unique solution that might meet the stated goals. Some questions sets might have more than one correct solutions, while others might not have a correct solution. After you answer a question in this section, you will NOT be able to return to it. As a result, these questions will not appear in the review screen. Your network contains an Active Directory forest named contoso.com. The forest contains a member server named Server1 that runs Windows Server 2016. All domain controllers run Windows Server 2012 R2. Contoso.com has the following configuration: Ensurepass 2017 PDF and VCE You plan to deploy an Active Directory Federation Services (AD FS) farm on Server1 and to configure device registration. You need to configure Active Directory to support the planned deployment. Solution: You raise the forest functional level to Windows Server 2012 R2. Does this meet the goal? 1. Yes 2. No Answer: B Explanation: For a Windows Server 2012 R2 AD FS server, this solution would work. However, new installations of AD FS 2016 require the Active Directory 2016 schema (minimum version 85). References: https://technet.microsoft.com/en-us/windows-server-docs/identity/ad- fs/operations/configure-device-based-conditional-access-on-premises Question No: 56 You have an Active Directory domain named contoso.com. The computers in contoso.com are installed by using Windows Deployment Services. You have a server named Server1 that runs Windows Server 2016. Server1 is a member of contoso.com. Server1 has the Hyper-V role installed. Virtual machines on Server1 are connected to an external switch named Switch1. You create a virtual machine named VM1 on Server1 by running the following cmdlets. Ensurepass 2017 PDF and VCE You need to ensure that you can install the operating system on VM1 by using Windows Deployment Services. What should you do? 1. Add a legacy network adapter to VM1. 2. Modify the SwitchType parameter of Switch1. 3. Modify the DefaultFlowMinimumBandwidthWeigth parameter of Switch1. 4. Add a SCSI controller to VM1. Answer: A Explanation: A legacy network adapter is required for PXE boot. Not B: The switch is an External switch which is what is required. Question No: 57 Active Directory Recycle Bin is enabled. You discover that a support technician accidentally removed 100 users from an Active Directory group named Group1 an hour ago. You need to restore the membership of Group1. What should you do? 1. Perform tombstone reanimation. 2. Export and import data by using Dsamain. 3. Perform a non-authoritative restore. 4. Recover the items by using Active Directory Recycle Bin. Answer: B Explanation: A group has been modified. Nothing has been deleted. Therefore, answers A and D will not work. Answer C would work if it was an authoritative restore, but not a non-authoritative restore. The solution is to recover an earlier copy of the group from a backup or active directory snapshot by using DSadmain. Question No: 58 You create a Nano server image named Nano1.vhdx by using the New-NanoServerImage cmdlet. You attach Nano1.vhdx to a Generation 1 virtual machine named Nano1. When you start Nano1, you get the following error message: “Boot failure. Reboot and select proper Boot device or Insert Boot Media in selected Boot device” You need to successfully start Nano server. What should you do? 1. Attach Nano1.vhdx to a SCSIcontroller. 2. Recreate Nano1 as a Generation 2 virtual machine. 3. Increase the memory of Nano1 to 512 Mb. 4. Modify the BIOS settings of Nano1. Answer: B Explanation: A Generation 1 VM cannot boot from a vhdx disk; only a Generation 2 VM can boot from an vhdx disk. Question No: 59 HOTSPOT Your network contains an Active Directory domain named contoso.com. The domain contains a domain controller named Server1 and a member server named Server2. Server1 has the DNS Server role installed. Server2 has IP Address Management (IPAM) installed. The IPAM server retrieves zones from Server1 as shown in the following table. Ensurepass 2017 PDF and VCE The IPAM server has one access policy configured as shown in the exhibit. (Click the Exhibit button.) Ensurepass 2017 PDF and VCE For each of the following statements, select Yes if the statement is true. Otherwise, select No. Ensurepass 2017 PDF and VCE Answer: Ensurepass 2017 PDF and VCE Explanation: Ensurepass 2017 PDF and VCE Box 1: Yes As a member of the IPAM DNS Administrator Role of the ADatum zone, User1 can add DNS records to it. Box 2: Yes As a member of the DNS Record Administrator Role of the Fabrikam zone, User1 can add DNS records to it. Box 3: No DNS Record Administrators cannot delete zones, only administer DNS records. References: https://technet.microsoft.com/en-us/library/hh831353(v=ws.11).aspx Question No: 60 Your network contains an Active Directory domain named contoso.com. The domain contains two servers named Server1 and Server2 that run Windows Server 2016. The servers have the same hardware configuration. You need to asynchronously replicate volume F: from Server1 to Server2. What should you do? 1. Install the Failover Clustering feature and create a new cluster resource group. 2. Run Set-DfsrServiceConfiguration and specify the -RPCPort parameter. 3. Run New-SRPartnership and specify the -ReplicationMode parameter. 4. Install the Failover Clustering feature and use Cluster Shared Volumes (CSV). Answer: A Explanation: Deploy Stretch Cluster with Volume Replication by implementing the following steps: References: https://www.starwindsoftware.com/blog/how-to-configure-storage-replication- using-windows-server-2016-part-2 100% Ensurepass Free Download! Download Free Demo:70-743 Demo PDF 100% Ensurepass Free Guaranteed! Download 2017 EnsurePass 70-743 Dumps EnsurePass ExamCollection Testking Lowest Price Guarantee Yes No No Up-to-Dated Yes No No Real Questions Yes No No Explanation Yes No No PDF VCE Yes No No Free VCE Simulator Yes No No Instant Download Yes No No 2017 EnsurePass IT Certification PDF and VCE Leave a Reply Your email address will not be published. Required fields are marked * This site uses Akismet to reduce spam. Learn how your comment data is processed.
{ "url": "http://www.70-740.com/free-2017oct-ensurepass-examcollection-microsoft-70-743-dumps-with-vce-and-pdf-51-60/", "source_domain": "www.70-740.com", "snapshot_id": "crawl=CC-MAIN-2019-22", "warc_metadata": { "Content-Length": "197312", "Content-Type": "application/http; msgtype=response", "WARC-Block-Digest": "sha1:ETGCPNOHFNGSR3KPRBONH5EO4S3LS3CQ", "WARC-Concurrent-To": "<urn:uuid:16ae292e-e1aa-445f-8713-eadadf7f2e7d>", "WARC-Date": "2019-05-21T19:55:45Z", "WARC-IP-Address": "198.252.107.233", "WARC-Identified-Payload-Type": "text/html", "WARC-Payload-Digest": "sha1:2NDCA5YU4B7VQ3NMOS4YJXQVRMZBQVQN", "WARC-Record-ID": "<urn:uuid:4751629a-f2de-46fe-8127-ec1bbdc0259e>", "WARC-Target-URI": "http://www.70-740.com/free-2017oct-ensurepass-examcollection-microsoft-70-743-dumps-with-vce-and-pdf-51-60/", "WARC-Truncated": null, "WARC-Type": "response", "WARC-Warcinfo-ID": "<urn:uuid:e2a26a38-75ee-4f01-aae9-dc7e410dcc4e>" }, "warc_info": "isPartOf: CC-MAIN-2019-22\r\npublisher: Common Crawl\r\ndescription: Wide crawl of the web for May 2019\r\noperator: Common Crawl Admin ([email protected])\r\nhostname: ip-10-7-214-88.ec2.internal\r\nsoftware: Apache Nutch 1.15 (modified, https://github.com/commoncrawl/nutch/)\r\nrobots: checked via crawler-commons 1.1-SNAPSHOT (https://github.com/crawler-commons/crawler-commons)\r\nformat: WARC File Format 1.1\r\nconformsTo: http://iipc.github.io/warc-specifications/specifications/warc-format/warc-1.1/" }
{ "line_start_idx": [ 0, 89, 90, 136, 184, 226, 264, 265, 316, 317, 341, 342, 467, 468, 596, 597, 625, 626, 759, 760, 788, 789, 797, 798, 826, 827, 840, 841, 869, 870, 1030, 1031, 1093, 1094, 1185, 1186, 1595, 1596, 1637, 1638, 1717, 1718, 1734, 1735, 1847, 1848, 1939, 1940, 1966, 1967, 2047, 2048, 2100, 2101, 2163, 2164, 2210, 2211, 2234, 2235, 2430, 2431, 2521, 2522, 2557, 2558, 2574, 2575, 2859, 2860, 3011, 3012, 3136, 3137, 3222, 3223, 3232, 3233, 3241, 3242, 3265, 3266, 3375, 3376, 3400, 3401, 3506, 3507, 3591, 3592, 3732, 3733, 3824, 3825, 3853, 3854, 3862, 3863, 3891, 3892, 3905, 3906, 3934, 3935, 3947, 3948, 4025, 4026, 4042, 4043, 4329, 4330, 4480, 4481, 4681, 4682, 4727, 4728, 4756, 4757, 4878, 4879, 5053, 5054, 5063, 5064, 5072, 5073, 5096, 5097, 5270, 5271, 5420, 5421, 5437, 5438, 5493, 5494, 5575, 5576, 5794, 5795, 5879, 5880, 5908, 5909, 6015, 6016, 6036, 6037, 6079, 6080, 6129, 6130, 6202, 6203, 6238, 6239, 6262, 6263, 6314, 6315, 6382, 6383, 6399, 6400, 6614, 6615, 6635, 6636, 6672, 6673, 6719, 6720, 6762, 6763, 6825, 6826, 6849, 6850, 7038, 7039, 7157, 7158, 7174, 7175, 7264, 7265, 7334, 7335, 7492, 7493, 7537, 7538, 7558, 7559, 7603, 7604, 7659, 7660, 7705, 7706, 7746, 7747, 7770, 7771, 7870, 7871, 7895, 7896, 8053, 8054, 8227, 8228, 8256, 8257, 8359, 8360, 8388, 8389, 8486, 8487, 8515, 8516, 8524, 8525, 8553, 8554, 8567, 8568, 8596, 8597, 8608, 8609, 8709, 8710, 8721, 8722, 8826, 8827, 8837, 8838, 8993, 8994, 9010, 9011, 9217, 9218, 9310, 9311, 9397, 9398, 9472, 9473, 9544, 9545, 9628, 9629, 9652, 9653, 9737, 9738, 9859, 9860, 9891, 9926, 9959, 9997, 9998, 10033, 10066, 10088, 10113, 10135, 10153, 10182, 10209, 10210, 10255, 10256, 10270, 10271, 10342, 10343 ], "line_end_idx": [ 89, 90, 136, 184, 226, 264, 265, 316, 317, 341, 342, 467, 468, 596, 597, 625, 626, 759, 760, 788, 789, 797, 798, 826, 827, 840, 841, 869, 870, 1030, 1031, 1093, 1094, 1185, 1186, 1595, 1596, 1637, 1638, 1717, 1718, 1734, 1735, 1847, 1848, 1939, 1940, 1966, 1967, 2047, 2048, 2100, 2101, 2163, 2164, 2210, 2211, 2234, 2235, 2430, 2431, 2521, 2522, 2557, 2558, 2574, 2575, 2859, 2860, 3011, 3012, 3136, 3137, 3222, 3223, 3232, 3233, 3241, 3242, 3265, 3266, 3375, 3376, 3400, 3401, 3506, 3507, 3591, 3592, 3732, 3733, 3824, 3825, 3853, 3854, 3862, 3863, 3891, 3892, 3905, 3906, 3934, 3935, 3947, 3948, 4025, 4026, 4042, 4043, 4329, 4330, 4480, 4481, 4681, 4682, 4727, 4728, 4756, 4757, 4878, 4879, 5053, 5054, 5063, 5064, 5072, 5073, 5096, 5097, 5270, 5271, 5420, 5421, 5437, 5438, 5493, 5494, 5575, 5576, 5794, 5795, 5879, 5880, 5908, 5909, 6015, 6016, 6036, 6037, 6079, 6080, 6129, 6130, 6202, 6203, 6238, 6239, 6262, 6263, 6314, 6315, 6382, 6383, 6399, 6400, 6614, 6615, 6635, 6636, 6672, 6673, 6719, 6720, 6762, 6763, 6825, 6826, 6849, 6850, 7038, 7039, 7157, 7158, 7174, 7175, 7264, 7265, 7334, 7335, 7492, 7493, 7537, 7538, 7558, 7559, 7603, 7604, 7659, 7660, 7705, 7706, 7746, 7747, 7770, 7771, 7870, 7871, 7895, 7896, 8053, 8054, 8227, 8228, 8256, 8257, 8359, 8360, 8388, 8389, 8486, 8487, 8515, 8516, 8524, 8525, 8553, 8554, 8567, 8568, 8596, 8597, 8608, 8609, 8709, 8710, 8721, 8722, 8826, 8827, 8837, 8838, 8993, 8994, 9010, 9011, 9217, 9218, 9310, 9311, 9397, 9398, 9472, 9473, 9544, 9545, 9628, 9629, 9652, 9653, 9737, 9738, 9859, 9860, 9891, 9926, 9959, 9997, 9998, 10033, 10066, 10088, 10113, 10135, 10153, 10182, 10209, 10210, 10255, 10256, 10270, 10271, 10342, 10343, 10423 ] }
{ "red_pajama_v2": { "ccnet_original_length": 10423, "ccnet_original_nlines": 283, "rps_doc_curly_bracket": 0, "rps_doc_ldnoobw_words": 0, "rps_doc_lorem_ipsum": 0, "rps_doc_stop_word_fraction": 0.2572377920150757, "rps_doc_ut1_blacklist": 0, "rps_doc_frac_all_caps_words": 0.05363075062632561, "rps_doc_frac_lines_end_with_ellipsis": 0, "rps_doc_frac_no_alph_words": 0.23967726528644562, "rps_doc_frac_unique_words": 0.29241645336151123, "rps_doc_mean_word_length": 5.302699089050293, "rps_doc_num_sentences": 183, "rps_doc_symbol_to_word_ratio": 0, "rps_doc_unigram_entropy": 5.401104927062988, "rps_doc_word_count": 1556, "rps_doc_frac_chars_dupe_10grams": 0.1621621549129486, "rps_doc_frac_chars_dupe_5grams": 0.27039146423339844, "rps_doc_frac_chars_dupe_6grams": 0.23403224349021912, "rps_doc_frac_chars_dupe_7grams": 0.20361168682575226, "rps_doc_frac_chars_dupe_8grams": 0.19112834334373474, "rps_doc_frac_chars_dupe_9grams": 0.18058417737483978, "rps_doc_frac_chars_top_2gram": 0.01090777013450861, "rps_doc_frac_chars_top_3gram": 0.016361650079488754, "rps_doc_frac_chars_top_4gram": 0.03393527865409851, "rps_doc_books_importance": -757.0741577148438, "rps_doc_books_importance_length_correction": -757.0741577148438, "rps_doc_openwebtext_importance": -460.3758850097656, "rps_doc_openwebtext_importance_length_correction": -460.3758850097656, "rps_doc_wikipedia_importance": -407.8163146972656, "rps_doc_wikipedia_importance_length_correction": -407.8163146972656 }, "fasttext": { "dclm": 0.09065812826156616, "english": 0.8287144899368286, "fineweb_edu_approx": 1.6429171562194824, "eai_general_math": 0.2113785743713379, "eai_open_web_math": 0.12407994270324707, "eai_web_code": 0.002958540106192231 } }
{ "free_decimal_correspondence": { "primary": { "code": "005.445", "labels": { "level_1": "General works, books and libraries, information sciences", "level_2": "", "level_3": "Computer programming" } }, "secondary": { "code": "004.162", "labels": { "level_1": "General works, books and libraries, information sciences", "level_2": "", "level_3": "Computers and Computer science" } } }, "bloom_cognitive_process": { "primary": { "code": "3", "label": "Apply" }, "secondary": { "code": "2", "label": "Understand" } }, "bloom_knowledge_domain": { "primary": { "code": "3", "label": "Procedural" }, "secondary": { "code": "2", "label": "Conceptual" } }, "document_type_v1": { "primary": { "code": "3", "label": "Reference/Encyclopedic/Educational" }, "secondary": { "code": "-1", "label": "Abstain" } }, "extraction_artifacts": { "primary": { "code": "3", "label": "Irrelevant Content" }, "secondary": { "code": "-1", "label": "Abstain" } }, "missing_content": { "primary": { "code": "4", "label": "Missing Images or Figures" }, "secondary": { "code": "-1", "label": "Abstain" } }, "document_type_v2": { "primary": { "code": "23", "label": "Tutorial" }, "secondary": { "code": "8", "label": "Documentation" } }, "reasoning_depth": { "primary": { "code": "2", "label": "Basic Reasoning" }, "secondary": { "code": "3", "label": "Intermediate Reasoning" } }, "technical_correctness": { "primary": { "code": "4", "label": "Highly Correct" }, "secondary": { "code": "3", "label": "Mostly Correct" } }, "education_level": { "primary": { "code": "3", "label": "Undergraduate Level" }, "secondary": { "code": "4", "label": "Graduate/Expert Level" } } }
672f1e42c33a7f9846924a2431ea77df
5,729,768,876,649,411,000
tag:blogger.com,1999:blog-36893639291015764402020-03-23T09:11:25.207+01:00Math Geek Rock ChickAnnahttp://www.blogger.com/profile/[email protected]:blogger.com,1999:blog-3689363929101576440.post-56816546239170590152012-09-24T18:11:00.000+02:002012-09-24T18:11:24.858+02:00Geek goes to PyCon PL 2012 - day 1Last week, I went to my first programming conference ever - <a href="http://pl.pycon.org/2012/">PyCon PL 2012</a>!<br /><br />Here's what I wore on the first day: <br /><br /><div class="separator" style="clear: both; text-align: center;"><a href="http://3.bp.blogspot.com/-ZhozM2NLAAI/UGB9ZYbh83I/AAAAAAAAGk8/KQmCZS5cRsg/s1600/20120913_195215.jpg" imageanchor="1" style="margin-left: 1em; margin-right: 1em;"><img border="0" height="400" src="http://3.bp.blogspot.com/-ZhozM2NLAAI/UGB9ZYbh83I/AAAAAAAAGk8/KQmCZS5cRsg/s400/20120913_195215.jpg" width="300" /></a></div><br />I fooled a few people that I worked at Google, which wasn't my intention, but this misunderstanding was promptly explained.<br /><br />Day one was mainly the trip there and being too tired afterwards to party like it's PyCon 2012. Still, the lectures were rich in content and very interesting. <br /><br /><a href="http://lukasz.langa.pl/">Łukasz Langa</a> talked about <a href="http://ralph.allegrogroup.com/">Ralph</a>, an asset management system for your data&nbsp;centers. Written in-house to manage the biggest cloud in Poland, it is now open-source. Łukasz talked about how Ralph was born: out of need, when adding new admins wasn't helping to keep the huge and varied infrastructure in order. The beginnings were tough, as there was only one developer and no process like scrum was in place. This one developer wanted to do everything for everyone, which of course couldn't work well. The solutions for this situation were:<br /><ol><li>more devs </li><li>applying scrum by the book (I never knew that in the real scrum, a task from outside the sprint cancels the entire sprint!)</li><li>git workflow</li><li>one good changelog</li><li>one good backlog</li><li>thematical sprints.</li></ol>&nbsp;The second talk was by PyCon PL's youngest speaker ever, 18-year-old Marek Šuppa. He talked about programming Lego NXT robots with Python and how he taught it to kids. He created his own IDE, <a href="https://github.com/xlcteam/nxtIDE">nxtIDE</a>, complete with a simulator, for that purpose. The show was the best part:<br /><br /><div class="separator" style="clear: both; text-align: center;"><a href="http://2.bp.blogspot.com/--UE477qmero/UGCE9B_xxzI/AAAAAAAAGlM/HPZvR5MpfwY/s1600/ENIMAGE1347553439264.jpg" style="margin-left: 1em; margin-right: 1em;"><img border="0" height="250" src="http://2.bp.blogspot.com/--UE477qmero/UGCE9B_xxzI/AAAAAAAAGlM/HPZvR5MpfwY/s400/ENIMAGE1347553439264.jpg" width="400" /></a></div><br /><div class="separator" style="clear: both; text-align: center;"><a href="http://2.bp.blogspot.com/-TtvD84d-3CE/UGCE-cik_PI/AAAAAAAAGlU/mtgee4hOjYc/s1600/ENIMAGE1347553551426.jpg" style="margin-left: 1em; margin-right: 1em;"><img border="0" height="300" src="http://2.bp.blogspot.com/-TtvD84d-3CE/UGCE-cik_PI/AAAAAAAAGlU/mtgee4hOjYc/s400/ENIMAGE1347553551426.jpg" width="400" /></a></div><br /><div class="separator" style="clear: both; text-align: center;"><a href="http://1.bp.blogspot.com/-mGJu8L38dbQ/UGCE_jW3ssI/AAAAAAAAGlc/9tGy0l1k58Q/s1600/ENIMAGE1347553930728.jpg" style="margin-left: 1em; margin-right: 1em;"><img border="0" height="300" src="http://1.bp.blogspot.com/-mGJu8L38dbQ/UGCE_jW3ssI/AAAAAAAAGlc/9tGy0l1k58Q/s400/ENIMAGE1347553930728.jpg" width="400" /></a></div><br /><br />&nbsp;The third talk was by <a href="http://lvh.cc/">lvh</a>, about fractal architectures. Big data is big these days and everyobody is scaling up - but why not scale down first? Why not, for example, shard your data by user and for every user, set up a separate SQLite database? Once you've scaled down this much, scaling up is easy. Well, except when it's not. Some problems arise, like wide queries for statistics, global, non-user-specific data, handling transactions... Depending on your project, these issues might be bigger or smaller and a fractal architecture can be a good solution for you or not. It's not a silver bullet, but definitely an interesting concept. <br /><br />The last talk on that day was about forms in Python, their challenges and existing solutions. <a href="https://github.com/zefciu">Szymon Pyżalski</a> presented them, along with a framework of his own: Anthrax. (See his page for PyCon materials and more on Anthrax.) Szymon works for STX Next, the daughter company of my previous employer. Having dealt a lot with forms at that job, I knew exactly what Szymon was talking about. :) Since my current job is so different, however, (I basically have one big form!), I wasn't very interested in the details and didn't take many notes.<br /><br />After the talks, I had a few drinks with STX Next people, then went to sleep.<br /><br />To be continued...<br /><br />Annahttp://www.blogger.com/profile/[email protected]:blogger.com,1999:blog-3689363929101576440.post-9962558011350466772012-05-26T11:52:00.003+02:002012-05-26T11:52:58.698+02:00On my wish list: a ready-to-use, convertible houseI'm fascinated by little convertible houses, where all space and furniture is designed to be multi-functional. I'd love to live in one.<br /><br />Why don't I? Well, as with everything in life, there's some trade-offs. Not only that sort of home needs a lot of time to be designed, you also have to <b>build most of the furniture yourself</b>, as the commercial options are limited. (I think that my most functional piece of furniture is my sofa, that doubles as a guest bed and has storage underneath.) And let's face it, you need quite a bit of resources to make your own furniture, starting with lots of space (at least temporarily).<br /><br />Well, as it turns out, there is another option: <a href="http://www.nytimes.com/2012/05/17/garden/the-founder-of-treehugger-and-his-apartment-of-the-future.html">Graham Hill recently designed such an apartment</a> for sale.<br /><div class="separator" style="clear: both; text-align: center;"><br /></div>&nbsp;The sofa hides a Murphy bed:<br /><br /><div class="separator" style="clear: both; text-align: center;"><a href="http://2.bp.blogspot.com/-FgWEO6OjYNw/T8ClmexbXpI/AAAAAAAAFN0/oJoULLugF_Q/s1600/20120517-HOUSEOFFUTURE-slide-PSQW-slide.jpg" imageanchor="1" style="margin-left: 1em; margin-right: 1em;"><img border="0" height="203" src="http://2.bp.blogspot.com/-FgWEO6OjYNw/T8ClmexbXpI/AAAAAAAAFN0/oJoULLugF_Q/s320/20120517-HOUSEOFFUTURE-slide-PSQW-slide.jpg" width="320" /></a></div><div class="separator" style="clear: both; text-align: center;"><br /></div><div class="separator" style="clear: both; text-align: left;">The kitchen counter expands to a table where you can sit up to 12 people! (Stackable chairs are stored behind one of the walls.) This is what amazes me the most. One of the biggest trade-offs of minimalism is the one of hospitality. Many people would feel offended if they were invited to stay in uncomfortable conditions.</div><br /><div class="separator" style="clear: both; text-align: center;"><a href="http://3.bp.blogspot.com/-xNq5BgOi2BE/T8Clnjal_ZI/AAAAAAAAFOE/X57PQBDwlMU/s1600/20120517-HOUSEOFFUTURE-slide-RGZR-slide.jpg" imageanchor="1" style="margin-left: 1em; margin-right: 1em;"><img border="0" height="320" src="http://3.bp.blogspot.com/-xNq5BgOi2BE/T8Clnjal_ZI/AAAAAAAAFOE/X57PQBDwlMU/s320/20120517-HOUSEOFFUTURE-slide-RGZR-slide.jpg" width="251" /></a></div><div class="separator" style="clear: both; text-align: center;"><br /></div><div class="separator" style="clear: both; text-align: left;">Even the tableware was carefully chosen to be multi-functional!</div><div class="separator" style="clear: both; text-align: center;"><br /></div><div class="separator" style="clear: both; text-align: center;"><a href="http://2.bp.blogspot.com/-H4Jyiwk-yA8/T8Cll0PR9lI/AAAAAAAAFNw/vkzLkVTAT48/s1600/20120517-HOUSEOFFUTURE-slide-ME7X-slide.jpg" imageanchor="1" style="margin-left: 1em; margin-right: 1em;"><img border="0" height="320" src="http://2.bp.blogspot.com/-H4Jyiwk-yA8/T8Cll0PR9lI/AAAAAAAAFNw/vkzLkVTAT48/s320/20120517-HOUSEOFFUTURE-slide-ME7X-slide.jpg" width="251" /></a></div><div class="separator" style="clear: both; text-align: center;"><br /></div><div class="separator" style="clear: both; text-align: center;"><br /></div>For now, there is just one apartment of the kind, but with the rising popularity of minimalism, there will hopefully be more and more options widely available. Hopefully my next house will have lots of well-designed, multi-purpose convertible space for kids, guests, hobbies, everything.<br /><br />[<a href="http://www.nytimes.com/2012/05/17/garden/the-founder-of-treehugger-and-his-apartment-of-the-future.html">Picture credit</a>]Annahttp://www.blogger.com/profile/[email protected]:blogger.com,1999:blog-3689363929101576440.post-86356025889327052082012-05-21T20:30:00.002+02:002012-05-21T20:30:54.450+02:00Become awesome in 366 daysIt started with&nbsp;<a href="http://lifehacker.com/281626/jerry-seinfelds-productivity-secret">Jerry Seinfeld's productivity secret</a>. Well, it's not technically a secret, since quite a few fellas &nbsp;know that it is:<br /><blockquote class="tr_bq">Don't break the chain.</blockquote>&nbsp;I'm not gonna praise it here. On the contrary: it's one of the productivity tricks that work the least well for me. There's always a weird day where a lot happens and keeping chains of my good old habits is really hard for me, let alone some new habit I'm working on. And when things go back to normal, I easily and happily get back on track. (Bah, as you see on this blog, getting back on some track can happen at the weirdest of times for me.)<br /><br />But, there's me, and there's other people. Like <a href="http://japhr.blogspot.fr/">Chris Strom</a>. That guy used the secret above to become awesome in 366 days. That's how his chain goes:<br /><blockquote class="tr_bq">Every night, I ask a question to which I don't know the answer and I try my damnedest to answer it.</blockquote>Sticking to that chain for 366 days made him learn a lot and write 3 smart books (the details are on <a href="http://lifehacker.com/5907717/366-days-or-how-i-tricked-myself-into-being-awesome">LifeHacker</a>).<br /><br />There's also yet other people. That guy doesn't need 366 days to become awesome:<br /><br /><div class="separator" style="clear: both; text-align: center;"><a href="http://theroyce.files.wordpress.com/2012/03/awesomeness-motivational-poster-1.jpg" imageanchor="1" style="margin-left: 1em; margin-right: 1em;"><img border="0" height="256" src="http://theroyce.files.wordpress.com/2012/03/awesomeness-motivational-poster-1.jpg" width="320" /></a></div><div class="separator" style="clear: both; text-align: center;"><br /></div>However, let's leave Barney out of this - none of us can hope to be half as awesome as he is!<br /><br />Chris' example (and the comments below the article) made me think. We all know it's not easy to start a good new habit. We all have lots of ways to fill our time (otherwise boredom would drive me right into the good habits' arms). We all need to rest too. You can only add so many one-hour-long daily activities (24 - to be exact). But how about 5-15-minute ones? (The number is instantly way bigger!) I know some people are for instance taking care of small children and have a hard time getting even 10 extra minutes from their day (do you know what small children can do in 10 minutes?), but not me. I could squeeze one in the morning and a few ones after work. Is it worth the shot? Will it make me even more tired, or on the contrary, energized and happy about my productivity? How many ways to find out are there? And finally, as a wise commenter on Lifechaker asks:<br /><blockquote class="tr_bq"> If I hadn't spent that time doing those small things, what might I have gained, a high score on spider solitaire?</blockquote>[<a href="http://theroyce.wordpress.com/2012/03/28/a-momentary-absence-of-awesome/">Picture credit</a>]Annahttp://www.blogger.com/profile/[email protected]:blogger.com,1999:blog-3689363929101576440.post-17080372050772387742012-05-20T14:41:00.002+02:002012-05-20T14:41:52.163+02:00DilemmasWow, it's been almost a year... Screw the excuses, I just didn't feel like blogging. And I'm still not totally sure I do.<br /><br />I thought that being funemployed would give a lot of time for blogging and that the stress of writing my thesis would encourage my to seek comfort in this diary-like therapy. Somehow that didn't happen. The thesis went as usual: it started with a feeling like "I'm writing the best thesis ever!" and ended with "I'm so tired of it! When will it be over?". Of one thing I am absolutely sure (got the Twilight allusion?): school is not my happy place. Happily, school is over: I graduated last December. I now have two masters. It's really cool when it's over.<br /><br />What do you do when you graduate? You get a full-time job. (Well, depending on your industry, this my be easier said than done, but in mine, staying unemployed is more of a challenge than finding work.) That's what I did. Since it's only one query away, I'll just say it: it's at WebInterpret. (Now my physical location is only another query away - is that the way things should be? Dilemma... Can I openly write about where I am or is that an entry for please-rob-me.com?&nbsp;Dilemma...)<br /><br />My life is so much more grown-up now. Grown-up in the sense that my party small talk involves tales of recent purchases and maintenance actions performed in my home. I come home from work tired (being tired is a common denominator for all adults) and cook and clean. When I browse the web, it's for cooking recipes. Am I interesting enough? Will I provide entertaining content?&nbsp;Dilemma...<br /><br />Or maybe I'm just ranting too much? (Forgive me, I'm currently in bed with a cold.) Because on the other hand, I'm feeling like I'm gradually getting so much stronger. In October-December, I was just getting used to full-time work and it drained all my energy. I came home, made dinner, sat at the computer. In January, I added 3 hours of physical activity a week. In March, I was up to 6. (I still am.) Maybe it's time to add some more hobbies and interests to the mix. Maybe it's time to read, do and write more cool stuff. Let's hope it is!<br /><br /><br /><br /><br /><br />Annahttp://www.blogger.com/profile/[email protected]:blogger.com,1999:blog-3689363929101576440.post-14580886523291946702011-08-06T16:29:00.005+02:002011-08-06T16:45:40.378+02:00Blogging my thesis - second edtitionLadies and gentlemen, welcome to the second edition of "<a href="http://math-geek-rock-chick.blogspot.com/search/label/thesis">Blogging my thesis</a>".<br /><br />This time, I'm doing a cloud version of the <a href="http://nb.sagemath.org/">Sage Notebook</a>. I'm doing the front-end in ASP.NET MVC 3 with C# and Razor, the back-end in Python and an SQL Server database in-between.<br /><br />Since a screenshot is worth around 1024 words, here's a little preview:<br /><br /><a href="http://2.bp.blogspot.com/-ge3qJSsN6qE/Tj1RVS4pydI/AAAAAAAADxc/RfKEzEu1h6g/s1600/CloudNotebook.png"><img style="display:block; margin:0px auto 10px; text-align:center;cursor:pointer; cursor:hand;width: 400px; height: 288px;" src="http://2.bp.blogspot.com/-ge3qJSsN6qE/Tj1RVS4pydI/AAAAAAAADxc/RfKEzEu1h6g/s400/CloudNotebook.png" alt="" id="BLOGGER_PHOTO_ID_5637751734826289618" border="0" /></a><br />The good news is that it's cool and I'm learning a lot while writing it (like MVC or jQuery).<br /><br />The bad news is that it's due on September the 30th. I wished to have it finished by the beginning of next month, but I'm afraid it will take too much work.<br /><br />Well, however it turns out, it's cool.Annahttp://www.blogger.com/profile/[email protected]:blogger.com,1999:blog-3689363929101576440.post-38855656487703738882011-06-23T21:36:00.002+02:002011-06-23T21:40:00.709+02:00Hugging catDear readers,<br /><br />For your amusement, below is a picture of a cat hugging a little girl.<br /><br /><a href="http://1.bp.blogspot.com/-evNpO7jBwFU/TgOWZnlps4I/AAAAAAAADtc/hljkg6nEoms/s1600/hugging_cat.jpg"><img style="display:block; margin:0px auto 10px; text-align:center;cursor:pointer; cursor:hand;width: 300px; height: 400px;" src="http://1.bp.blogspot.com/-evNpO7jBwFU/TgOWZnlps4I/AAAAAAAADtc/hljkg6nEoms/s400/hugging_cat.jpg" alt="" id="BLOGGER_PHOTO_ID_5621502126756311938" border="0" /></a><br />[<a href="http://www.flickr.com/photos/ooh_food/2800359116/in/photostream">Source</a>]Annahttp://www.blogger.com/profile/[email protected]:blogger.com,1999:blog-3689363929101576440.post-67069015397569796662011-05-01T18:08:00.002+02:002011-05-01T18:21:22.816+02:00Geek goes funemployedHi guys, I'm funemployed!<br /><br />This is good news for two reasons:<br /><ol><li>I now have more time to do loads of great stuff,</li><li>I'm gonna blog more, since I'll be looking for a job and trying look my best out there in the interwebs.</li></ol>So what exactly are these loads of great stuff?<br /><ol><li>Meeting lots of friends (I've started already and it's going great.)</li><li>Writing my master thesis in CS (I've started already and it's going to be going great.)</li><li>Getting another Microsoft certificate, 70-562 this time.</li><li>Writing the best Android app ever, <a href="http://bigbangtheory.wikia.com/wiki/The_Bus_Pants_Utilization">the one from the Big Bang Theory</a> that solves differential equations (yeah, I'm 100% serious).</li><li>Running and biking in the sunshine.</li><li>Surely many more - funemployed doesn't mean idle!</li></ol>I am told not to show off my material status on the Internet, so I won't get into the details, but let me just say I am now the lucky owner of an Android device - all the app out there are really great and I feel inspired to contribute some of my own. Even cooler, I have a few friends who feel the same way - bring on the SDK!Annahttp://www.blogger.com/profile/[email protected]:blogger.com,1999:blog-3689363929101576440.post-63159940755897924982010-11-27T15:43:00.004+01:002010-11-27T16:02:28.460+01:00Creative gift wrappingThree years ago (boy, is my blog old!) I blogged about some <a href="http://math-geek-rock-chick.blogspot.com/2007/12/christmas-is-coming.html">creative gift wrappings</a>. Well, it's that time of the year soon and people are publishing many more great and innovative ideas.<br /><a href="http://www.countryliving.com/crafts/projects/creative-gift-wrap-ideas-1209"><br />Country Living</a> has fun with tape:<br /><br /><a href="http://4.bp.blogspot.com/_R51qCdBeLQU/TPEbCpk-CWI/AAAAAAAADpI/zYGMnPVUzDY/s1600/kraft-paper-wrap-1209-de.jpg"><img style="display: block; margin: 0px auto 10px; text-align: center; cursor: pointer; width: 400px; height: 313px;" src="http://4.bp.blogspot.com/_R51qCdBeLQU/TPEbCpk-CWI/AAAAAAAADpI/zYGMnPVUzDY/s400/kraft-paper-wrap-1209-de.jpg" alt="" id="BLOGGER_PHOTO_ID_5544242348604393826" border="0" /></a><a href="http://howaboutorange.blogspot.com/2009/10/make-gift-bow-from-magazine-page.html"><br />How About Orange</a> explains how to make a gift bow from a magazine page:<br /><br /><a href="http://2.bp.blogspot.com/_R51qCdBeLQU/TPEbCxn3WsI/AAAAAAAADpQ/0_2i8KURZP8/s1600/Magazine-bow-finished.jpg"><img style="display: block; margin: 0px auto 10px; text-align: center; cursor: pointer; width: 400px; height: 300px;" src="http://2.bp.blogspot.com/_R51qCdBeLQU/TPEbCxn3WsI/AAAAAAAADpQ/0_2i8KURZP8/s400/Magazine-bow-finished.jpg" alt="" id="BLOGGER_PHOTO_ID_5544242350764022466" border="0" /></a><br /><a href="http://mondocherry.blogspot.com/2009/11/creative-gift-wrapping-part-one.html">Mondo Cherry</a> has fun with aluminum foil:<br /><br /><a href="http://1.bp.blogspot.com/_R51qCdBeLQU/TPEbDdAsqvI/AAAAAAAADpY/R1qX3NkDRI8/s1600/P1190213.JPG"><img style="display: block; margin: 0px auto 10px; text-align: center; cursor: pointer; width: 400px; height: 400px;" src="http://1.bp.blogspot.com/_R51qCdBeLQU/TPEbDdAsqvI/AAAAAAAADpY/R1qX3NkDRI8/s400/P1190213.JPG" alt="" id="BLOGGER_PHOTO_ID_5544242362410904306" border="0" /></a><br /><a href="http://www.realsimple.com/holidays-entertaining/gifts/wrapping/creative-gift-wrapping-ideas-00000000014208/page12.html">Real Simple</a> features, among other ideas, a ribbon with holes:<br /><br /><a href="http://2.bp.blogspot.com/_R51qCdBeLQU/TPEbNP2yALI/AAAAAAAADpo/hX2yzfmhzUg/s1600/wrap-green-gift_300.jpg"><img style="display: block; margin: 0px auto 10px; text-align: center; cursor: pointer; width: 300px; height: 357px;" src="http://2.bp.blogspot.com/_R51qCdBeLQU/TPEbNP2yALI/AAAAAAAADpo/hX2yzfmhzUg/s400/wrap-green-gift_300.jpg" alt="" id="BLOGGER_PHOTO_ID_5544242530678341810" border="0" /></a><br /><a href="http://www.designspongeonline.com/2008/11/paper-holiday-gift-wrapping.html">Design Sponge</a> replaced the classical bow with a pom pom:<br /><br /><a href="http://4.bp.blogspot.com/_R51qCdBeLQU/TPEbF3VpV4I/AAAAAAAADpg/AGxyWZsKj3g/s1600/pompomgiftwrap.jpg"><img style="display: block; margin: 0px auto 10px; text-align: center; cursor: pointer; width: 400px; height: 284px;" src="http://4.bp.blogspot.com/_R51qCdBeLQU/TPEbF3VpV4I/AAAAAAAADpg/AGxyWZsKj3g/s400/pompomgiftwrap.jpg" alt="" id="BLOGGER_PHOTO_ID_5544242403837826946" border="0" /></a><br />Even more ideas are featured at <a href="http://www.squidoo.com/how-to-tie-different-types-of-decorative-bows-gift-wrap-packaging">Squidoo.com</a> - check it out!Annahttp://www.blogger.com/profile/[email protected]:blogger.com,1999:blog-3689363929101576440.post-26044553752013261012010-11-19T20:45:00.004+01:002010-11-19T21:06:05.130+01:00Lanvin for H&M: a 200$ disasterIt's no secret that I love clothes, the more colorful, the better. I like looking at clothing pictures on the web and when I first saw that Lanvin was doing a collection for H&amp;M, I had a serious "WANT!!!" moment.<br /><br />Lanvin is a fashion designer famous for making clothing with amazing ruffles:<br /><br /><br /><a href="http://2.bp.blogspot.com/_R51qCdBeLQU/TObVpdYYznI/AAAAAAAADoc/E4lpuArJGVo/s1600/lanvin-trends.jpg"><img style="display: block; margin: 0px auto 10px; text-align: center; cursor: pointer; width: 400px; height: 369px;" src="http://2.bp.blogspot.com/_R51qCdBeLQU/TObVpdYYznI/AAAAAAAADoc/E4lpuArJGVo/s400/lanvin-trends.jpg" alt="" id="BLOGGER_PHOTO_ID_5541351299764244082" border="0" /></a><br />What H&amp;M is famous for is clothing that anyone can afford. So, beautiful, designer and affordable - what's not to want? I glued my eyes to the screen.<br /><br />Not for long. Soon after pictures were published, prices got announced. These weren't the usual H&amp;M prices: while these stores are filled with 40$ dresses, the Lanvin ones cost 200$. Quite a lot.<br /><br />I didn't give up on the dream so soon: ok, it was expensive, but the dresses were so nice and trendy (though that means they will stat screaming "I'm so last season!" soon) and perhaps the quality was worth it.<br /><br />It turns out it is not. On the promotional shot below, you can see on the collar how the fabric is raveling.<br /><br /><a href="http://3.bp.blogspot.com/_R51qCdBeLQU/TObUYzBX8YI/AAAAAAAADoM/Fylv7EenhY8/s1600/20c788e0684b88de_Screen_shot_2010-11-02_at_11.11.13_AM.png"><img style="display: block; margin: 0px auto 10px; text-align: center; cursor: pointer; width: 329px; height: 400px;" src="http://3.bp.blogspot.com/_R51qCdBeLQU/TObUYzBX8YI/AAAAAAAADoM/Fylv7EenhY8/s400/20c788e0684b88de_Screen_shot_2010-11-02_at_11.11.13_AM.png" alt="" id="BLOGGER_PHOTO_ID_5541349914003894658" border="0" /></a><br />And my, it's a promotional picture! If that looks so much better than in reality, how bad must these dresses look like!<br /><br />It seems I'm not the only one to disapprove of the quality of those dresses. A few days ago, a polish gossip site published pictures of a star wearing one. They didn't mention the names of Lanvin and H&amp;M, but the lack of hem was heavily criticised. No wonder, it's so visible!<br /><br /><a href="http://4.bp.blogspot.com/_R51qCdBeLQU/TObUZYo6UEI/AAAAAAAADoU/EyjZBjhQM-0/s1600/1b44951f001341fb4ce3cd6b.jpg"><img style="display: block; margin: 0px auto 10px; text-align: center; cursor: pointer; width: 400px; height: 318px;" src="http://4.bp.blogspot.com/_R51qCdBeLQU/TObUZYo6UEI/AAAAAAAADoU/EyjZBjhQM-0/s400/1b44951f001341fb4ce3cd6b.jpg" alt="" id="BLOGGER_PHOTO_ID_5541349924101836866" border="0" /></a><br />Honestly, I expect way more for 200$ and way more from a world-famous designer. Doesn't he have a reputation to loose?<br /><br />[Picture <a href="http://designerdirection.wordpress.com/2009/10/29/the-ss10-monster-trend-report-part-2/">source</a>, <a href="http://www.pudelek.pl/artykul/28538/cichopek_w_prujacej_sie_sukience_zdjecia/">source</a>, <a href="http://www.fabsugar.com/Pictures-Lanvin-HM-Collection-11779973?page=0,0,0#6">source</a>]Annahttp://www.blogger.com/profile/[email protected]:blogger.com,1999:blog-3689363929101576440.post-26462707941979689102010-11-19T19:00:00.003+01:002010-11-19T20:14:17.218+01:00Geek diggs minimalismMinimalism is recently really growing on me.<br /><br />I guess it started with reading <a href="http://lifehacker.com/">Lifehacker</a>, which lead me to reading <a href="http://unclutterer.com/">Unclutter</a>, which lead me to wanting less clutter and here I am.<br /><br />That transformation started at a lucky point: about a year before my wedding, which also was about a year before moving out of my parents' place and starting a new home. It's not everyday that you get to start from scratch - it's a 0-3 times in a lifetime opportunity! So I did my best to take advantage of it and tossed away lots of things that I didn't want to have in my new home. I got down to 4 moving boxes and a few bags of stuff and my moving was almost unnoticeable - my fiance visited me as usual and took my stuff to our new home in his car.<br /><br />Today, my home is nowhere near like the nice minimalist pictures you can find on the web, nor would our stuff fit into 4 boxes. We keep buying and receiving stuff, however, I try to buy as little as possible and set high expectations for the stuff I decide to pay for, especially clothing: I have the habit of buying a lot if it, which might have been a good idea when I was young and just building my wardrobe, but definitely isn't anymore. I also prefer getting nothing than any gift, I enjoy tossing stuff away (but only if I can prove it's worthless) and I try to have as little e-mails in my Inbox as possible (never went under 7, actually) . So I guess I'm becoming a minimalist.<br /><br />At this point, all minimalist bloggers feel obliged to point out that they are minimalists, but not in a "have at most 100 prosessions" or "screw all material possessions" kind of way. So, minimalism, extreme minimalism - not the same. There, done.<br /><br />What I feel the need to point out is the difference between realistic and unrealistic minimalism.<br /><br />Take the picture below:<br /><br /><br /><a href="http://3.bp.blogspot.com/_R51qCdBeLQU/TObHm92L8SI/AAAAAAAADn8/oBmjxn10FG0/s1600/small-home-minimalist-interior.jpg"><img style="display: block; margin: 0px auto 10px; text-align: center; cursor: pointer; width: 400px; height: 300px;" src="http://3.bp.blogspot.com/_R51qCdBeLQU/TObHm92L8SI/AAAAAAAADn8/oBmjxn10FG0/s400/small-home-minimalist-interior.jpg" alt="" id="BLOGGER_PHOTO_ID_5541335863776768290" border="0" /></a><br />That room looks fantastic! There's a lot of light and the amount of stuff is just optimal for a room to relax in. Plus, I don't exactly know how, but you can tell form the picture that it's an actual room in someone's home - perhaps an extra room besides a bigger and less minimalist living room, but still, it's real and it's inspiring.<br /><br />Next picture:<br /><br /><a href="http://4.bp.blogspot.com/_R51qCdBeLQU/TObHnCmfOGI/AAAAAAAADoE/BOj5pzWbleM/s1600/Nature-and-minimalism-in-the-pool-mirror-Eden.jpg"><img style="display: block; margin: 0px auto 10px; text-align: center; cursor: pointer; width: 400px; height: 239px;" src="http://4.bp.blogspot.com/_R51qCdBeLQU/TObHnCmfOGI/AAAAAAAADoE/BOj5pzWbleM/s400/Nature-and-minimalism-in-the-pool-mirror-Eden.jpg" alt="" id="BLOGGER_PHOTO_ID_5541335865053100130" border="0" /></a><br />Nice bathroom, but even if it isn't rendered in 3d, I don't see myself ever owning a bathroom even close to that one. I can't imagine having that much room for a bathroom nor the walls being white and yet so clean on an everyday basis. That's just too unrealistic.<br /><br />There are a lot of great blogs on minimalism, written by people doing it for various reasons (travelers often become minimalists) and in various degrees - In the spirit of minimalism, I subscribed a dozen today. If you're interested in the subject, here's a great article to start: <a title="Permanent Link to The Rewards Of Adopting A Minimalist Lifestyle: 13 Bloggers Share Their Views" href="http://www.mindthebeginner.net/2010/11/rewards-of-adopting-a-minimalist-lifestyle/" rel="bookmark"><span style="font-weight: bold;"></span>The Rewards Of Adopting A Minimalist Lifestyle: 13 Bloggers Share Their Views</a>.<br /><br />[Picture <a href="http://www.housedesignblogs.com/2010/modular-contemporary-small-house-minimalist-interior-design-ideas/">credit</a>, <a href="http://didid.net/76/nature-and-minimalism-of-the-eden-bathroom-design/">credit</a>]Annahttp://www.blogger.com/profile/[email protected]:blogger.com,1999:blog-3689363929101576440.post-26912204214556261842010-11-13T21:58:00.005+01:002010-11-14T21:21:14.271+01:00Where the Geek is when she's not blogging (in Egypt or on a Eurotrip)Hi there. Yeah, I know it's been a long time, my apologies. Thing is, I've been alternately getting sick and travelling - I am now in a sick phase, hopefully the last one in a while. Since my sicknesses are not half as interesting as my travels, let me tell you about those.<br /><br />The first one was a total <a href="http://www.imdb.com/title/tt0356150/">Eurotrip</a>. The deal was that a friend of Marcin is currently working at Cern and he invited us to stay with him for what turned out to be an entire week. We took him with us on one way plus his brother both ways. That trip included:<br /><ul><li>me crashing a Linux event in a Microsoft t-shirt (guys were staying, but hell knows if it was the t-shirt or the gender)</li><li>a night and a morning in Prague</li><li>visiting Bayreuth (but unfortunately not <a href="http://en.wikipedia.org/wiki/Bayreuth_Festspielhaus">Bayreuth Festival Theatre</a> because it had shorter opening hours than the student's office at my school)</li><li>me meeting some friends I hadn't seen in at least 12 years (my, have they grown!)</li><li>hanging at the Cern, especially at lunch time (which felt like being in a BBT episode, except they were many many more geeks - epic!)</li><li>hanging at an English pub and playing some trivia games (there was a Star Trek category!)</li><li>living maybe 50 meters from my middle school (ah, memories!)</li><li>me visiting the place I grew up in, which is the most beautiful place in the world to me</li><li>all of us visiting Geneva and Annecy</li><li>the guys going to Milan to grab a pizza after climbing on the <a href="http://en.wikipedia.org/wiki/Matterhorn">Matterhorn</a></li><li>shopping</li><li>drinking french wine</li><li>losing a debit card and finding it soon after<br /></li><li>crossing the french/swiss border at least four times a day</li><li>getting a flat tyre on the german highway.</li></ul><br />All in all, that's more adventures than in Eurotrip. (Afterwards, I got scolded by my boss for taking all that overtime without asking him.) We're looking forward to organizing a similar trip next year.<br /><br />Then I spent 3 days at home. These included working full time plus going to school on the evening (these were the first classes of the semester, so I didn't want to miss them). Consequently, cooking became microwaving and "taking everything out of the bags and dropping it on the floor" was the new unpacking.<br /><br />Soon, we hit the road again, this time to a "real" summer vacation in the middle of October: we spent two weeks in Egypt. That trip included way less that the previous one:<br /><ul><li>bathing</li><li>sunbathing</li><li>a bit of sightseeing.</li></ul>That holiday did us both lots of good and we enjoyed every minute of it.<br /><br />Okay, now, for the pictures: I'm still trying to get the ones that weren't taken with my camera, so below are a few of my own, featuring absolutely no one.<br /><br />That's the nice place where I grew up:<br /><br /><a href="http://4.bp.blogspot.com/_R51qCdBeLQU/TOBCZmMHskI/AAAAAAAADn0/MVrw6Ii0iv8/s1600/sergy.jpg"><img style="display: block; margin: 0px auto 10px; text-align: center; cursor: pointer; width: 400px; height: 300px;" src="http://4.bp.blogspot.com/_R51qCdBeLQU/TOBCZmMHskI/AAAAAAAADn0/MVrw6Ii0iv8/s400/sergy.jpg" alt="" id="BLOGGER_PHOTO_ID_5539500549181256258" border="0" /></a><br />That's the previously mentioned middle school:<br /><br /><a href="http://3.bp.blogspot.com/_R51qCdBeLQU/TOBCZlgSZkI/AAAAAAAADns/67T1h-97bQk/s1600/middle-school.jpg"><img style="display: block; margin: 0px auto 10px; text-align: center; cursor: pointer; width: 400px; height: 300px;" src="http://3.bp.blogspot.com/_R51qCdBeLQU/TOBCZlgSZkI/AAAAAAAADns/67T1h-97bQk/s400/middle-school.jpg" alt="" id="BLOGGER_PHOTO_ID_5539500548997408322" border="0" /></a><br />And here is a little piece of Egypt (<a href="http://en.wikipedia.org/wiki/Deir_el-Bahari">Deir el-Bahari</a>):<br /><br /><a href="http://1.bp.blogspot.com/_R51qCdBeLQU/TOBCZQZlGzI/AAAAAAAADnk/9qdWvpYbWlY/s1600/egypt.jpg"><img style="display: block; margin: 0px auto 10px; text-align: center; cursor: pointer; width: 400px; height: 300px;" src="http://1.bp.blogspot.com/_R51qCdBeLQU/TOBCZQZlGzI/AAAAAAAADnk/9qdWvpYbWlY/s400/egypt.jpg" alt="" id="BLOGGER_PHOTO_ID_5539500543332129586" border="0" /></a><br /><br />Now, I'm back to the life I love! I recently came across a quote by Seth Godin:<br /><br />"Instead of wondering when your next vacation is, maybe you ought to set up a life you don’t need to escape from."<br /><br />I definitely don't feel the need to escape from my life, but a vacation now and again is a great way to spend more time with loved ones and friends, get a fresh perspective on everything, load your batteries, etc. etc.Annahttp://www.blogger.com/profile/[email protected]:blogger.com,1999:blog-3689363929101576440.post-70782719177865607832010-10-06T19:26:00.062+02:002010-11-19T19:00:16.125+01:00The world is wrong about the Monty Hall problemToday, at school, we considered the probability theory paradox called <a href="http://en.wikipedia.org/wiki/Monty_Hall_problem">the Monty Hall problem</a>:<br /><blockquote>Suppose you're on a game show, and you're given the choice of three doors: Behind one door is a car; behind the others, goats. You pick a door, say No. 1, and the host, who knows what's behind the doors, opens another door, say No. 3, which he knows has a goat. He then says to you, "Do you want to pick door No. 2?" Is it to your advantage to switch your choice?</blockquote><br />The world says that by seeing that goat, you learn that the probability of the car being behind door number 2 is 2/3 and Wikipedia has <a href="http://en.wikipedia.org/wiki/Monty_Hall_problem#Solutions">various explanations</a>.<br /><br /><br /><a href="http://3.bp.blogspot.com/_R51qCdBeLQU/TKyyFfkXqVI/AAAAAAAADnY/xgP7AefQJ4o/s1600/197px-Monty_open_door_chances.svg.png"><img style="display: block; margin: 0px auto 10px; text-align: center; cursor: pointer; width: 197px; height: 200px;" src="http://3.bp.blogspot.com/_R51qCdBeLQU/TKyyFfkXqVI/AAAAAAAADnY/xgP7AefQJ4o/s400/197px-Monty_open_door_chances.svg.png" alt="" id="BLOGGER_PHOTO_ID_5524986650319890770" border="0" /></a><br />I claim it's 1/2. My first argument is that there is full symmetry between doors 1 and 2. How could you lower the probability of door 1 being the winning one by picking it? Nonsense.<br /><br />For my second argument, let's treat it like the conditional probability that it is. All probabilities will be from your, the contestant 's, point of view.<br /><br />At the start, we've got 3 equally probable situations: I'll denote them as CGG, GCG and GGC hoping it's clear enough.<br /><br />Now the host opens a door. You see the goat: you <span style="font-weight: bold;">gain information</span>. <span style="font-weight: bold;">Probabilities change</span>. The probability of the door number 3 being the winning one falls down to 0.<br /><br />So far, no difference from the "official" solutions. However, those maintain that while the probability of the car being behind the first door <span style="font-weight: bold;">stays the same</span>, the probability of it being behind the second one <span style="font-weight: bold;">increases</span>. Why the incoherence?<br /><br />An important remark here: with the <span style="font-weight: bold;">new information</span>, we now have <span style="font-weight: bold;">new events</span>. We know that GGC didn't happen, so we have 3 events that assume ~GGC, namely:<br /><ol><li>if the first door wins, it's CGG | ~GGC and P(CGG | ~GGC) is 1/2</li><li>if the second door wins, it's GCG | ~GGC and P(GCG | ~GGC) is 1/2</li><li>if the third door wins, it's GGC | ~GGC and P(GGC | ~GGC) is 0,</li></ol>the final results being basic <a href="http://en.wikipedia.org/wiki/Conditional_probability">conditional probability</a> calculations.<br /><br /><br />According to Wikipedia, "even Nobel physicists systematically give the wrong answer, and that they <i>insist</i> on it, and they are ready to berate in print those who propose the right answer". That's comforting: I may be wrong, but at least I have excellent company.<br /><br />[UPDATE] After reading all the comments, I can see that I was wrong. Boy, is that problem counterintuitive! Thanks everyone for contributing! [/UPDATE]Annahttp://www.blogger.com/profile/[email protected]:blogger.com,1999:blog-3689363929101576440.post-2865116213122022502010-09-22T13:27:00.005+02:002010-09-22T13:34:59.397+02:005 reasons why I prefer cats over dogsBelow are 5 reasons why I prefer cats over dogs.<br /><br />Reason #1:<br /><br /><a href="http://1.bp.blogspot.com/_R51qCdBeLQU/TJnpXMtZs8I/AAAAAAAADm4/Gx5ZkW2i9K4/s1600/cat-in-the-dog-bed-4726-1236195894-3.jpg"><img style="display: block; margin: 0px auto 10px; text-align: center; cursor: pointer; width: 400px; height: 231px;" src="http://1.bp.blogspot.com/_R51qCdBeLQU/TJnpXMtZs8I/AAAAAAAADm4/Gx5ZkW2i9K4/s400/cat-in-the-dog-bed-4726-1236195894-3.jpg" alt="" id="BLOGGER_PHOTO_ID_5519699403077170114" border="0" /></a><br /><br />Reason #2:<br /><br /><a href="http://2.bp.blogspot.com/_R51qCdBeLQU/TJnpXVx5omI/AAAAAAAADnA/fBaRKTvLKaU/s1600/cats-kick-out-the-dog.jpg"><img style="display: block; margin: 0px auto 10px; text-align: center; cursor: pointer; width: 400px; height: 300px;" src="http://2.bp.blogspot.com/_R51qCdBeLQU/TJnpXVx5omI/AAAAAAAADnA/fBaRKTvLKaU/s400/cats-kick-out-the-dog.jpg" alt="" id="BLOGGER_PHOTO_ID_5519699405511959138" border="0" /></a><br /><br />Reason #3:<br /><br /><a href="http://1.bp.blogspot.com/_R51qCdBeLQU/TJnpW1eCt_I/AAAAAAAADmw/s9pxq19J8l8/s1600/3060598834_d3070b7108_o.jpg"><img style="display: block; margin: 0px auto 10px; text-align: center; cursor: pointer; width: 400px; height: 276px;" src="http://1.bp.blogspot.com/_R51qCdBeLQU/TJnpW1eCt_I/AAAAAAAADmw/s9pxq19J8l8/s400/3060598834_d3070b7108_o.jpg" alt="" id="BLOGGER_PHOTO_ID_5519699396838733810" border="0" /></a><br /><br />Reason #4:<br /><br /><a href="http://3.bp.blogspot.com/_R51qCdBeLQU/TJnpWkM5GyI/AAAAAAAADmo/m-G2WKIFDHE/s1600/30f5bdbe-e5d2-4ac1-af84-884d3110453e.jpg"><img style="display: block; margin: 0px auto 10px; text-align: center; cursor: pointer; width: 400px; height: 299px;" src="http://3.bp.blogspot.com/_R51qCdBeLQU/TJnpWkM5GyI/AAAAAAAADmo/m-G2WKIFDHE/s400/30f5bdbe-e5d2-4ac1-af84-884d3110453e.jpg" alt="" id="BLOGGER_PHOTO_ID_5519699392203397922" border="0" /></a><br /><br />Reason #5:<br /><br /><a href="http://4.bp.blogspot.com/_R51qCdBeLQU/TJnpXkDaW3I/AAAAAAAADnI/poiVkfeCSYI/s1600/funny-pictures-cat-hogs-the-mat.jpg"><img style="display: block; margin: 0px auto 10px; text-align: center; cursor: pointer; width: 400px; height: 300px;" src="http://4.bp.blogspot.com/_R51qCdBeLQU/TJnpXkDaW3I/AAAAAAAADnI/poiVkfeCSYI/s400/funny-pictures-cat-hogs-the-mat.jpg" alt="" id="BLOGGER_PHOTO_ID_5519699409343503218" border="0" /></a><br /><br />[Picture <a href="http://www.buzzfeed.com/peggy/cat-in-the-dog-bed">credit</a>, <a href="http://www.ahmbay.info/pictures-6821-cats_kick_out_the_dog.html">credit</a>, <a href="http://feverishthoughts.com/oddplanet/2008/11/26/dog-kicked-out-of-his-own-dog-house/">credit</a>, <a href="http://icanhascheezburger.com/2010/09/12/funny-pictures-seemz-faer-to-me/">credit</a>, <a href="http://icanhascheezburger.com/2008/12/22/funny-pictures-for-meh-nao/">credit</a>.]Annahttp://www.blogger.com/profile/[email protected]:blogger.com,1999:blog-3689363929101576440.post-70353346793616472542010-09-18T00:38:00.005+02:002010-09-18T01:06:16.819+02:00Figuring out the right and wrong women's figuresWhen it comes to women's figures, the norms keep on changing. It however used to be simple: when sweets were a luxury, fat meant rich and thus was "in". Then it was thin's turn: thin people were the ones who could resist the temptation of tasty food - a kind of saint. That one has been taken too far, up to <a href="http://blogs.news.com.au/fullychic/index.php/news/comments/we_dont_want_you_to_be_anorexic_we_just_want_you_to_look_like_you_are/">anorectic chick</a>, and too many deaths later, something seems to be slightly changing. But how exactly?<br /><br />If you're slightly too thin, there's always someone for you to ask whether you've got anorexia:<br /><br /><a href="http://4.bp.blogspot.com/_R51qCdBeLQU/TJPw8W111SI/AAAAAAAADmY/t0Ic7mhPSww/s1600/angelina_thin_500.jpg"><img style="display: block; margin: 0px auto 10px; text-align: center; cursor: pointer; width: 284px; height: 400px;" src="http://4.bp.blogspot.com/_R51qCdBeLQU/TJPw8W111SI/AAAAAAAADmY/t0Ic7mhPSww/s400/angelina_thin_500.jpg" alt="" id="BLOGGER_PHOTO_ID_5518018888173212962" border="0" /></a><br />Yet, a model's silhouette is still this:<br /><br /><a href="http://4.bp.blogspot.com/_R51qCdBeLQU/TJPw8AsO5eI/AAAAAAAADmQ/6V5IzfJD2Kc/s1600/872bc6cc002596f44c92b7e7.jpg"><img style="display: block; margin: 0px auto 10px; text-align: center; cursor: pointer; width: 400px; height: 297px;" src="http://4.bp.blogspot.com/_R51qCdBeLQU/TJPw8AsO5eI/AAAAAAAADmQ/6V5IzfJD2Kc/s400/872bc6cc002596f44c92b7e7.jpg" alt="" id="BLOGGER_PHOTO_ID_5518018882227332578" border="0" /></a><br />Can you see the difference?<br />(In reality, if an anorexic person is that skinny, it means that his or her illness has already gone very far and done severe damage. Read more about it on <a href="http://www.psychologytoday.com/blog/brave-girl-eating/201008/shes-not-skinny-is-she">Psychology Today</a>.)<br /><br />If too thin is sick, what about the other way? There just was a plus-size show at New York Fashion Week:<br /><br /><a href="http://2.bp.blogspot.com/_R51qCdBeLQU/TJPw7xOh6mI/AAAAAAAADmI/oKeUwv_hS2w/s1600/104156939_10.jpg"><img style="display: block; margin: 0px auto 10px; text-align: center; cursor: pointer; width: 267px; height: 400px;" src="http://2.bp.blogspot.com/_R51qCdBeLQU/TJPw7xOh6mI/AAAAAAAADmI/oKeUwv_hS2w/s400/104156939_10.jpg" alt="" id="BLOGGER_PHOTO_ID_5518018878076217954" border="0" /></a><br />Does that mean that extra weight is now socially acceptable? I'm not sure: Jessica Simpson has recently been heavily criticized for her weight gain, while she looks good and is far from being obese:<br /><br /><a href="http://3.bp.blogspot.com/_R51qCdBeLQU/TJPw9DY1f7I/AAAAAAAADmg/zvBrJskvFuU/s1600/wenn2066612a.jpg"><img style="display: block; margin: 0px auto 10px; text-align: center; cursor: pointer; width: 203px; height: 400px;" src="http://3.bp.blogspot.com/_R51qCdBeLQU/TJPw9DY1f7I/AAAAAAAADmg/zvBrJskvFuU/s400/wenn2066612a.jpg" alt="" id="BLOGGER_PHOTO_ID_5518018900131151794" border="0" /></a><br />So what's right and what's wrong when it comes to a woman's figure? Go figure!<br /><br />[Picture <a href="http://jezebel.com/5638831/this-week-in-tabloids-ashton-cheats-on-demi-again/gallery/?skyline=true&amp;s=i">source</a>, <a href="http://www.pudelek.pl/artykul/27242/ewa_minge_na_nowojorskim_tygodniu_mody/">source</a>, <a href="http://jezebel.com/5641047/images-from-the-only-plus+size-show-at-fashion-week/gallery/">source</a>, <a href="http://www.radaronline.com/photos/image/52325/2010/03/jessica-simpsons-weight-fluctuations">source</a>.]Annahttp://www.blogger.com/profile/[email protected]:blogger.com,1999:blog-3689363929101576440.post-80204312720869324162010-08-07T15:20:00.008+02:002010-08-07T21:57:49.879+02:00Silverlight 4 with C# and Python (it's three!)I recently felt like making a new Silverlight 4 app, but not just any app. I wanted it to have the following features:<br /><ol><li>use an existing Python library</li><li>be written in C#.</li></ol>There is a way better way of using Python code - it is to write the app in Iron Python from the start. Picking C# to achieve that sounds like a bad decision.<br /><br />However, I wanted to do something new and challenging, so I went for C#. Doing it on my own was hard, way harder than just following an "embedding Python in C#" tutorial. Finally, I found <a href="http://www.voidspace.org.uk/ironpython/silverlight/embedding_ironpython.shtml">that working example </a>and got something starting from there.<br /><br />What I got is a three on a golden background:<br /><br /><a onblur="try {parent.deselectBloggerImageGracefully();} catch(e) {}" href="http://4.bp.blogspot.com/_R51qCdBeLQU/TF1n4fUPo0I/AAAAAAAADlM/Dn0vXh7tins/s1600/3.png"><img style="display: block; margin: 0px auto 10px; text-align: center; cursor: pointer; width: 255px; height: 204px;" src="http://4.bp.blogspot.com/_R51qCdBeLQU/TF1n4fUPo0I/AAAAAAAADlM/Dn0vXh7tins/s400/3.png" alt="" id="BLOGGER_PHOTO_ID_5502668539893359426" border="0" /></a><br />Okay, maybe that's not the right angle to start with. What I did was used <a href="http://userpages.umbc.edu/%7Ercampbel/Computers/Python/numbthy.html">the numbthy Python library</a> and used it's gcd function to compute the greatest common divisor of 15 and 3, and it's three!<br /><br />Getting there was hard. Finding the right libraries to embed Python, importing them the right way were tough tasks for an inexperienced user - some error messages would be very cryptic. Then, the app would hang on the splash screen if something was wrong in the Python code, the debugger wouldn't show where the problem was, and the "problems" in the Python code were:<br /><ol><li>the print statement</li><li>a class inheriting from object.<br /></li></ol>Not obvious, and definitely not the kind of problems PyDev would warn you about. So I am afraid that importing bigger chunks of code could be hard - that is to be explored further.Annahttp://www.blogger.com/profile/[email protected]:blogger.com,1999:blog-3689363929101576440.post-53132741829754880482010-08-05T16:12:00.006+02:002010-08-05T16:40:25.391+02:00Yet another personal websiteThanks to my <a href="http://math-geek-rock-chick.blogspot.com/2010/07/geek-gets-surgery.html">surgery</a>, I recently had a bit of time to spend at home, so I built myself a new <a href="http://students.mimuw.edu.pl/%7Eaw214573/lilavati/">personal website</a>. I spent a bit of time investigating the cool new features of <a href="http://www.webappers.com/2009/08/10/70-must-have-css3-and-html5-tutorials-and-resources/">HTML 5, CSS 3</a> and jQuery and ended up using a cool thing called <a href="http://desandro.com/resources/jquery-masonry/">jQuery Masonry</a>, and here is the result:<br /><br /><br /><a onblur="try {parent.deselectBloggerImageGracefully();} catch(e) {}" href="http://3.bp.blogspot.com/_R51qCdBeLQU/TFrJnLHa8BI/AAAAAAAADk0/S8tbyIuU-MY/s1600/website.png"><img style="display: block; margin: 0px auto 10px; text-align: center; cursor: pointer; width: 400px; height: 273px;" src="http://3.bp.blogspot.com/_R51qCdBeLQU/TFrJnLHa8BI/AAAAAAAADk0/S8tbyIuU-MY/s400/website.png" alt="" id="BLOGGER_PHOTO_ID_5501931569622478866" border="0" /></a><br />This time, it's just a single page, because I can't really imagine what more I could share (besides the blog, of course). It's also only in English because... er... because.<br /><br />Technically, I used some JavaScript not to do any copy-paste between the eight divs but ended up publishing a static html file with the end result.<br /><br />Artistically, when I compare it to all the great designs featured on Smashing Magazine, I honestly think it sucks, but well, I'm more of a programmer than an artist. Plus, I put a lot of effort into editing it. Html is cheap and my mind has been to many many places, starting with Moroccan souks:<br /><br /><a onblur="try {parent.deselectBloggerImageGracefully();} catch(e) {}" href="http://4.bp.blogspot.com/_R51qCdBeLQU/TFrJm3XYsMI/AAAAAAAADks/bn2816uR6TQ/s1600/morocco-4.jpg"><img style="display: block; margin: 0px auto 10px; text-align: center; cursor: pointer; width: 400px; height: 266px;" src="http://4.bp.blogspot.com/_R51qCdBeLQU/TFrJm3XYsMI/AAAAAAAADks/bn2816uR6TQ/s400/morocco-4.jpg" alt="" id="BLOGGER_PHOTO_ID_5501931564320731330" border="0" /></a><br />museums:<br /><br /><a onblur="try {parent.deselectBloggerImageGracefully();} catch(e) {}" href="http://3.bp.blogspot.com/_R51qCdBeLQU/TFrMIFOqBDI/AAAAAAAADk8/laPpr9YyNMc/s1600/OrnamentsVarious0034_thumbhuge.jpg"><img style="display: block; margin: 0px auto 10px; text-align: center; cursor: pointer; width: 220px; height: 340px;" src="http://3.bp.blogspot.com/_R51qCdBeLQU/TFrMIFOqBDI/AAAAAAAADk8/laPpr9YyNMc/s400/OrnamentsVarious0034_thumbhuge.jpg" alt="" id="BLOGGER_PHOTO_ID_5501934334001153074" border="0" /></a><br />and texture factories:<br /><br /><a onblur="try {parent.deselectBloggerImageGracefully();} catch(e) {}" href="http://2.bp.blogspot.com/_R51qCdBeLQU/TFrMgX08GDI/AAAAAAAADlE/LNz_k0nSpUM/s1600/292E68c.png"><img style="display: block; margin: 0px auto 10px; text-align: center; cursor: pointer; width: 200px; height: 200px;" src="http://2.bp.blogspot.com/_R51qCdBeLQU/TFrMgX08GDI/AAAAAAAADlE/LNz_k0nSpUM/s400/292E68c.png" alt="" id="BLOGGER_PHOTO_ID_5501934751310420018" border="0" /></a><br />All in all, I like it a lot, and I hope you do as well.<br /><br />[Picture <a href="http://peonymoon.wordpress.com/2010/02/12/writing-morocco/">credit</a>, <a href="http://www.cgtextures.com/">credit</a>, <a href="http://www.colourlovers.com/">credit</a>.]Annahttp://www.blogger.com/profile/[email protected]:blogger.com,1999:blog-3689363929101576440.post-17497763694235675142010-07-26T20:59:00.001+02:002010-07-26T21:03:08.575+02:00Geek gets surgery<span id="zw-129f405cc98k61J44126383">A few days ago, I underwent my first surgery ever. The surgery itself wasn't as much of a big deal as the experience - I have wondered how it felt like since my childhood, when my little brother underwent one and I didn't. My curiosity finally got satisfied.<br /></span><p id="zw-129f4068b76Hyt0dp126383" style="margin-bottom: 12pt; margin-top: 0pt; font-style: italic;"><span id="zw-129f4068b76zgwO1z126383">Wednesday, July 14th</span></p><p id="zw-129f406cf4e1_buOn126383" style="margin-bottom: 12pt; margin-top: 0pt;"><span id="zw-129f406cf4es6wwck126383">I arrive at the hospital at noon. Waiting, paperwork, waiting, questions, waiting. Boring. They tell me not to eat to be ready the next day at 7h45 - I set my alarm clock to 7h30. I take a nap, then answer some e-mails, then go to bead early. Sleeping is something I'm actually really good at.</span></p><p id="zw-129f40ab117v8oWmj126383" style="margin-bottom: 12pt; margin-top: 0pt;"><span style="font-style: italic;" id="zw-129f40a8f18XJB5r126383">Thursday, July 15th</span><span id="zw-129f40ab118FTzlET126383"></span></p><p id="zw-129f40ab117v8oWmj126383" style="margin-bottom: 12pt; margin-top: 0pt;"><span id="zw-129f40ab118FTzlET126383">They wake me up at six and tell me to get ready now. I do. Then boredom comes back and I get back to sleep. (I guess there should be a part when I'm worried, but I'm really better at sleeping.) They wake me up just before eight. "Come on, it's time to go. (...) Okay, now the door on the right. No, on the right! But I can see you're smiling - that's the spirit!" When they lie me on the table, that's when I start to get nervous, but not for long: within a few minutes, I am asleep again.</span></p><p id="zw-129f40ab117v8oWmj126383" style="margin-bottom: 12pt; margin-top: 0pt;"><a onblur="try {parent.deselectBloggerImageGracefully();} catch(e) {}" href="http://1.bp.blogspot.com/_R51qCdBeLQU/TE3bve24nhI/AAAAAAAADkk/tARMKhBVrd0/s1600/szpital_msw8_9.jpg"><img style="display: block; margin: 0px auto 10px; text-align: center; cursor: pointer; width: 400px; height: 185px;" src="http://1.bp.blogspot.com/_R51qCdBeLQU/TE3bve24nhI/AAAAAAAADkk/tARMKhBVrd0/s400/szpital_msw8_9.jpg" alt="" id="BLOGGER_PHOTO_ID_5498292328872844818" border="0" /></a></p><p id="zw-129f41c039b1Xkcb126383" style="margin-bottom: 12pt; margin-top: 0pt;"><span id="zw-129f41c039ciisR3U126383">I wake up to a doctor calling my name. Phew, I made it! Something is hurting me in the head. It must be the hair elastic I'm lying on. The algorithm to solve that problem is simple: lift head, move elastic, lay head back down. Ouch - lifting my head hurts. Oh yeah, they had to cut through my neck so now I can't really move it. I mange to pull the elastic away. Now what? Let's get back to sleep.</span></p><p id="zw-129f421866bkLtR0u126383" style="margin-bottom: 12pt; margin-top: 0pt;"><span id="zw-129f421866bLvTW7-126383">I wake up quite a few times that day and it's not pleasant. I miss Marcin, take a look at the clock and count the hours down to his visit on the following day. I want to drink - they give me a 9 ml bottle. I want to go to the bathroom. I want to cough. I want to lie on my side. I want to see Marcin. None of these are possible, but the deprivations are really bearable, as within a few minutes I am asleep again. I pass those minutes by observing all the IO interfaces I am connected to.</span></p><p id="zw-129f424e16dj3ojf126383" style="margin-bottom: 12pt; margin-top: 0pt; font-style: italic;"><span id="zw-129f4253d7dbQTiyp126383">Friday, </span><span id="zw-129f4253df0yDQtP5126383">July 16th</span></p><p id="zw-129f42554defPCWta126383" style="margin-bottom: 12pt; margin-top: 0pt;">I<span id="zw-129f426302cINMynn126383"> am back in my room at the hospital and am encouraged to start moving. I get to the promised land of the bathroom. I enjoy my position: no work obligations, no school ones, no social ones. I don't have to do anything. I can sleep all I want - and hell, I do! Visitors brighten my day, but I can't stay awake for the whole four hours when visits are allowed. As soon as they leave, I am asleep again.</span></p> <p id="zw-12a1014956e-QHYTp126383" style="margin-bottom: 12pt; margin-top: 0pt; font-style: italic;"><span id="zw-12a1014956ea4JUuI126383">Saturday, July 17th</span></p> <p id="zw-12a101d8deeRyKbNN126383" style="margin-bottom: 12pt; margin-top: 0pt;"><span id="zw-12a1015078dnL9ZZN126383">Time to go! My stitches and all my IO interfaces get removed. I go to my Mom's for the week-end and spend most of the time - you guessed it - asleep.</span></p> <p id="zw-12a101da98fcRh2ez126383" style="margin-bottom: 12pt; margin-top: 0pt;"><span id="zw-12a101da98ftqP59b126383"></span><span style="font-style: italic;" id="zw-12a101da99003ay6n126383">Later on</span></p> <p id="zw-12a101db4aeSZnhH126383" style="margin-bottom: 12pt; margin-top: 0pt;"><span id="zw-12a101db4aecm13126383">I get two weeks of sick leave from work. So cool. So much time to sit at the computer, and, of course, so much time to sleep.<br /></span></p><p id="zw-12a101db4aeSZnhH126383" style="margin-bottom: 12pt; margin-top: 0pt;"><span id="zw-12a101db4aecm13126383">[Picture <a href="http://www.kvp.pl/contents/pl/gallery/8.xml">credit</a>]<br /></span></p>Annahttp://www.blogger.com/profile/[email protected]:blogger.com,1999:blog-3689363929101576440.post-8747993612132246382010-06-26T20:36:00.003+02:002010-06-26T20:46:37.209+02:00I got my masters degree!Two months after having my first defense and getting <a href="http://math-geek-rock-chick.blogspot.com/2010/04/im-bachelor-of-science-computer-science.html">my bachelor degree in computer science</a>, I am happy to announce I also got a master's degree in mathematics.<br /><br />I now need to find a thesis subject in computer science, and besides that, I'm free for the summer!Annahttp://www.blogger.com/profile/[email protected]:blogger.com,1999:blog-3689363929101576440.post-6469662400362202142010-06-05T00:04:00.006+02:002010-06-05T00:40:39.592+02:00Rijndael S-box Sage implementationIt's no secret I like <a href="http://math-geek-rock-chick.blogspot.com/search/label/python">Python</a> and <a href="http://math-geek-rock-chick.blogspot.com/2009/11/why-i-love-sage-in-5-simple-lines.html">Sage</a> a lot. Today, I had the pleasure of working with Sage again: as a school assignment, I was required to show the calculations that prove that the value of the <a href="http://en.wikipedia.org/wiki/Rijndael_S-box">Rijndael S-box</a> in row 2, column d, was indeed d8. While I am able to invert a polynome of this size by hand (let alone apply an affine transformation), I see no reason to do it, as a suitable tool is available.<br /><br />So, here we go, the <a href="http://www.sagenb.org/home/pub/2120/">Rijndael S-box in Sage</a>:<br /><pre class="python" style="font-family:monospace;">F.<span style="color: rgb(102, 204, 102);">&lt;</span>y<span style="color: rgb(102, 204, 102);">&gt;</span> = GF<span style="color:black;">(</span><span style="color: rgb(255, 69, 0);">2</span>, name=<span style="color: rgb(72, 61, 139);">'y'</span><span style="color:black;">)</span><span style="color:black;">[</span><span style="color:black;">]</span><br />K.<span style="color: rgb(102, 204, 102);">&lt;</span>x<span style="color: rgb(102, 204, 102);">&gt;</span> = GF<span style="color:black;">(</span><span style="color: rgb(255, 69, 0);">2</span><span style="color: rgb(102, 204, 102);">**</span><span style="color: rgb(255, 69, 0);">8</span>, name=<span style="color: rgb(72, 61, 139);">'x'</span>, modulus=y^8 + y^4 + y^3 + y + 1<span style="color:black;">)</span><br /><br />bin_digits = <span style="color: rgb(255, 119, 0); font-weight: bold;">lambda</span> h: <span style="color:black;">[</span><span style="color:black;">(</span><span style="color:black;">(</span><span style="color: rgb(0, 128, 0);">int</span><span style="color:black;">(</span><span style="color: rgb(0, 128, 0);">str</span><span style="color:black;">(</span>h<span style="color:black;">)</span>, 16<span style="color:black;">)</span><span style="color:black;">)</span>//<span style="color:black;">(</span>2^i<span style="color:black;">)</span><span style="color:black;">)</span> <span style="color: rgb(102, 204, 102);">%</span> 2 <br /> <span style="color: rgb(255, 119, 0); font-weight: bold;">for</span> i <span style="color: rgb(255, 119, 0); font-weight: bold;">in</span> <span style="color: rgb(0, 128, 0);">range</span><span style="color:black;">(</span>3, -1, -1<span style="color:black;">)</span><span style="color:black;">]</span><br />bin_poly = <span style="color: rgb(255, 119, 0); font-weight: bold;">lambda</span> n : <span style="color: rgb(0, 128, 0);">sum</span><span style="color:black;">(</span><span style="color:black;">[</span>b <span style="color: rgb(102, 204, 102);">*</span> x^<span style="color:black;">(</span>3-i<span style="color:black;">)</span><br /> <span style="color: rgb(255, 119, 0); font-weight: bold;">for</span> i,b <span style="color: rgb(255, 119, 0); font-weight: bold;">in</span> <span style="color: rgb(0, 128, 0);">enumerate</span><span style="color:black;">(</span>bin_digits<span style="color:black;">(</span>n<span style="color:black;">)</span><span style="color:black;">)</span><span style="color:black;">]</span><span style="color:black;">)</span><br /><br />b1 = <span style="color: rgb(255, 69, 0);">2</span><br />b2 = <span style="color: rgb(72, 61, 139);">'d'</span><br /><br />s = x^4 <span style="color: rgb(102, 204, 102);">*</span> bin_poly<span style="color:black;">(</span>b1<span style="color:black;">)</span> + bin_poly<span style="color:black;">(</span>b2<span style="color:black;">)</span><br />p = <span style="color:black;">(</span>1/s<span style="color:black;">)</span>.<span style="color:black;">polynomial</span><span style="color:black;">(</span><span style="color:black;">)</span>.<span style="color:black;">coeffs</span><span style="color:black;">(</span><span style="color:black;">)</span><br />p.<span style="color:black;">reverse</span><span style="color:black;">(</span><span style="color:black;">)</span><br /><br />b = <span style="color:black;">[</span>0<span style="color:black;">]</span><span style="color: rgb(102, 204, 102);">*</span><span style="color:black;">(</span>8 - <span style="color: rgb(0, 128, 0);">len</span><span style="color:black;">(</span>p<span style="color:black;">)</span><span style="color:black;">)</span> + p<br />b.<span style="color:black;">reverse</span><span style="color:black;">(</span><span style="color:black;">)</span><br />c = <span style="color:black;">[</span>1,1,0,0,0,1,1,0<span style="color:black;">]</span><br />d = <span style="color:black;">[</span><span style="color: rgb(0, 128, 0);">int</span><span style="color:black;">(</span><span style="color:black;">(</span>b<span style="color:black;">[</span>i<span style="color:black;">]</span> + b<span style="color:black;">[</span><span style="color:black;">(</span>i + 4<span style="color:black;">)</span> <span style="color: rgb(102, 204, 102);">%</span> 8<span style="color:black;">]</span> + b<span style="color:black;">[</span><span style="color:black;">(</span>i + 5<span style="color:black;">)</span> <span style="color: rgb(102, 204, 102);">%</span> 8<span style="color:black;">]</span> +<br /> b<span style="color:black;">[</span><span style="color:black;">(</span>i + 6<span style="color:black;">)</span> <span style="color: rgb(102, 204, 102);">%</span> 8<span style="color:black;">]</span> + b<span style="color:black;">[</span><span style="color:black;">(</span>i + 7<span style="color:black;">)</span> <span style="color: rgb(102, 204, 102);">%</span> 8<span style="color:black;">]</span> + c<span style="color:black;">[</span>i<span style="color:black;">]</span><span style="color:black;">)</span>.<span style="color:black;">n</span><span style="color:black;">(</span><span style="color:black;">)</span><span style="color:black;">)</span> <span style="color: rgb(102, 204, 102);">%</span> 2 <span style="color: rgb(255, 119, 0); font-weight: bold;"><br /> for</span> i <span style="color: rgb(255, 119, 0); font-weight: bold;">in</span> <span style="color: rgb(0, 128, 0);">range</span><span style="color:black;">(</span>8<span style="color:black;">)</span><span style="color:black;">]</span><br />d.<span style="color:black;">reverse</span><span style="color:black;">(</span><span style="color:black;">)</span><br /><br />s1 = <span style="color: rgb(0, 128, 0);">sum</span><span style="color:black;">(</span><span style="color:black;">[</span>d<span style="color:black;">[</span>i<span style="color:black;">]</span> <span style="color: rgb(102, 204, 102);">*</span> 2^<span style="color:black;">(</span>3-i<span style="color:black;">)</span> <span style="color: rgb(255, 119, 0); font-weight: bold;">for</span> i <span style="color: rgb(255, 119, 0); font-weight: bold;">in</span> <span style="color: rgb(0, 128, 0);">range</span><span style="color:black;">(</span>4<span style="color:black;">)</span><span style="color:black;">]</span><span style="color:black;">)</span><br />s2 = <span style="color: rgb(0, 128, 0);">sum</span><span style="color:black;">(</span><span style="color:black;">[</span>d<span style="color:black;">[</span>i+4<span style="color:black;">]</span> <span style="color: rgb(102, 204, 102);">*</span> 2^<span style="color:black;">(</span>3-i<span style="color:black;">)</span> <span style="color: rgb(255, 119, 0); font-weight: bold;">for</span> i <span style="color: rgb(255, 119, 0); font-weight: bold;">in</span> <span style="color: rgb(0, 128, 0);">range</span><span style="color:black;">(</span>4<span style="color:black;">)</span><span style="color:black;">]</span><span style="color:black;">)</span><br /><br /><span style="color: rgb(255, 119, 0); font-weight: bold;">print</span> <span style="color: rgb(0, 128, 0);">hex</span><span style="color:black;">(</span>s1<span style="color:black;">)</span>, <span style="color: rgb(0, 128, 0);">hex</span><span style="color:black;">(</span>s2<span style="color:black;">)</span></pre><br />I admit: the above is code, not calculations, but I did the calculations in class and worked hard on the code. My main problem was the bit order. Nowhere in the original publication did it say to reverse the bytes. Adding all the reverses was a moment of desperation, and imagine my surprise when it worked. I wonder how much credit that solution will get me.<br /><br />A great Friday night it was.Annahttp://www.blogger.com/profile/[email protected]:blogger.com,1999:blog-3689363929101576440.post-48993437254596308442010-06-03T12:34:00.004+02:002010-06-03T12:53:49.316+02:00Xen virtual machines are the creepiest thing everAmong the things that have always creeped people out are all kinds of ghosts and zombies, for one simple reason: since they are already dead, you can't kill them.<br /><br />Along that line, here's what creeps me out: <a href="http://en.wikipedia.org/wiki/Xen">Xen virtual machines</a>.<br /><br /><a onblur="try {parent.deselectBloggerImageGracefully();} catch(e) {}" href="http://2.bp.blogspot.com/_R51qCdBeLQU/TAeJccC_RmI/AAAAAAAADjk/IAtTPKpuoeE/s1600/800px-Xen_screenshot.png"><img style="display: block; margin: 0px auto 10px; text-align: center; cursor: pointer; width: 400px; height: 300px;" src="http://2.bp.blogspot.com/_R51qCdBeLQU/TAeJccC_RmI/AAAAAAAADjk/IAtTPKpuoeE/s400/800px-Xen_screenshot.png" alt="" id="BLOGGER_PHOTO_ID_5478498593377568354" border="0" /></a><br />See what Wiki says about them:<br /><br /><blockquote>Administrators can "live migrate" Xen virtual machines between physical hosts across a LAN without loss of availability. During this procedure, the LAN iteratively copies the memory of the virtual machine to the destination without stopping its execution. The process requires a stoppage of around 60–300 ms to perform final synchronization before the virtual machine begins executing at its final destination, providing an illusion of seamless migration.<br /><br /></blockquote>Imagine that. On a traditional machine, if it was doing something you did not want it to do, you would slam the keyboard. On a VM, you'd kill the monitor. Then you'd pull the host's plug. <span style="font-weight: bold;">Something </span>would work. But here,<span style="font-weight: bold;"> as you try to kill the guest, it floats through the air to another host</span>. There's no stopping it. There's nothing you can do but watch it do its evil.<br /><br />Next, the robots revolt against us and it's Battlestar Galactica all over again. Brrr.Annahttp://www.blogger.com/profile/[email protected]:blogger.com,1999:blog-3689363929101576440.post-41537164102957912442010-05-03T20:13:00.005+02:002010-05-03T20:51:31.231+02:00Music in me: longest album titles everHave you ever wondered what was the longest music album title ever?<br /><br />Until today, I thought the record belonged to <a href="http://en.wikipedia.org/wiki/Fiona_Apple">Fiona Apple</a>. It did for a long time and was even mentioned in the Guinness Book of Records for 2001. Ladies and gentlemen, it's <a href="http://en.wikipedia.org/wiki/When_the_Pawn..."><span style="font-style: italic;">When the Pawn</span></a>!<br /><br /><a onblur="try {parent.deselectBloggerImageGracefully();} catch(e) {}" href="http://4.bp.blogspot.com/_R51qCdBeLQU/S98UfeqpNEI/AAAAAAAADiw/I2qTHwV74KA/s1600/Fiona_apple_when_the_pawn.jpg"><img style="display: block; margin: 0px auto 10px; text-align: center; cursor: pointer; width: 300px; height: 300px;" src="http://4.bp.blogspot.com/_R51qCdBeLQU/S98UfeqpNEI/AAAAAAAADiw/I2qTHwV74KA/s400/Fiona_apple_when_the_pawn.jpg" alt="" id="BLOGGER_PHOTO_ID_5467111003691496514" border="0" /></a><blockquote><br />When the pawn hits the conflicts he thinks like a king<br />What he knows throws the blows when he goes to the fight<br />And he'll win the whole thing 'fore he enters the ring<br />There's no body to batter when your mind is your might<br />So when you go solo, you hold your own hand<br />And remember that depth is the greatest of heights<br />And if you know where you stand, then you know where to land<br />And if you fall it won't matter, cuz you'll know that you're right.</blockquote><br /><br />You can also listen to it <a href="http://www.youtube.com/watch?v=08p32RmGBZc">here</a>, told by the author herself - I recommend checking it out.<br /><br />Fiona Apple held the record for quite a few years, until 2007. Then came <a href="http://en.wikipedia.org/wiki/Soulwax">Soulwax</a> with an album of remixes, <a style="font-style: italic;" href="http://en.wikipedia.org/wiki/Most_of_the_Remixes">Most of the Remixes</a>, with a title of 552 characters:<br /><br /><a onblur="try {parent.deselectBloggerImageGracefully();} catch(e) {}" href="http://1.bp.blogspot.com/_R51qCdBeLQU/S98Uf8mAteI/AAAAAAAADi4/65B0Fu451vQ/s1600/Soulwax_Most_of_the_Remixes_cover.jpg"><img style="display: block; margin: 0px auto 10px; text-align: center; cursor: pointer; width: 362px; height: 354px;" src="http://1.bp.blogspot.com/_R51qCdBeLQU/S98Uf8mAteI/AAAAAAAADi4/65B0Fu451vQ/s400/Soulwax_Most_of_the_Remixes_cover.jpg" alt="" id="BLOGGER_PHOTO_ID_5467111011725129186" border="0" /></a>The following year, <a href="http://en.wikipedia.org/wiki/Chumbawamba" title="Chumbawamba">Chumbawamba</a> went even further, 865 characters in <a href="http://en.wikipedia.org/wiki/The_Boy_Bands_Have_Won"><span style="font-style: italic;">The Boy Bands Have Won</span></a>:<br /><br /><a onblur="try {parent.deselectBloggerImageGracefully();} catch(e) {}" href="http://4.bp.blogspot.com/_R51qCdBeLQU/S98UgFTycNI/AAAAAAAADjA/ccBbq_Xku-o/s1600/TheBoyBandsHaveWon.jpg"><img style="display: block; margin: 0px auto 10px; text-align: center; cursor: pointer; width: 400px; height: 400px;" src="http://4.bp.blogspot.com/_R51qCdBeLQU/S98UgFTycNI/AAAAAAAADjA/ccBbq_Xku-o/s400/TheBoyBandsHaveWon.jpg" alt="" id="BLOGGER_PHOTO_ID_5467111014064615634" border="0" /></a><br />I only quoted the first one, as the two others have pretty clear covers (or you can read them on the Wikipedia pages). Plus, Fiona's one is way more creative: it's <span style="font-weight: bold;">a real poem with lots of rhymes</span>, structure, and most importantly, a deep message. Soulwax's title is like <span style="font-weight: bold;">a long explanation of what to except from the album</span>, written in a very casual tone: I except more artistic value and sweat and tears in the creative process. Chumbawamba is not poetic either, but definitely <span style="font-weight: bold;">does carry a message</span>, and an interesting one, although the words "the boy bands have won" almost discouraged me from reading the rest. (Last time I listened to a boyband was when I was eleven and a friend made me a mix tape, then I totally moved on to the Spice Girls, so I don't really feel like the boy bands have won anything with me.)<br /><br />Giving your album a very long title is an easy way to attract attention to it (honestly, I have never heard of Soulwax before and here I am blogging about them), but it doesn't make it automatically remarkable. To me, Fiona's one is definitely the longest great album title ever.Annahttp://www.blogger.com/profile/[email protected]:blogger.com,1999:blog-3689363929101576440.post-20992913377017267772010-05-02T20:07:00.004+02:002010-05-02T20:34:38.514+02:00China gymnastics 2000 medal revokedRemember how back in 2008, I was <a href="http://math-geek-rock-chick.blogspot.com/2008/08/summer-olympics-did-china-cheat.html">wondering if the China olympic female gymnastics team had cheated</a> by sending underage girls? Remember how while investigating the matter, I found that a gymnast from the 2000 team had admitted to cheating the same way?<br /><br />Well, I learned recently that China's 2000 team medal got revoked! One of the gymnasts, Dong Fangxiao, was found to have falsified her age. (It's not the one I was writing about two years ago. Yang Yun is considered innocent due to lack of proof.) The team taking the spot is U.S.<br /><br /><a onblur="try {parent.deselectBloggerImageGracefully();} catch(e) {}" href="http://1.bp.blogspot.com/_R51qCdBeLQU/S93DgZS0c5I/AAAAAAAADio/7EjfwJxE1cw/s1600/DongOS004.jpg"><img style="display: block; margin: 0px auto 10px; text-align: center; cursor: pointer; width: 278px; height: 378px;" src="http://1.bp.blogspot.com/_R51qCdBeLQU/S93DgZS0c5I/AAAAAAAADio/7EjfwJxE1cw/s400/DongOS004.jpg" alt="" id="BLOGGER_PHOTO_ID_5466740484010701714" border="0" /></a><br />Well, I'm happy. I'm not happy for the Chinese girls themselves, as they were just put in the middle of such a situation. You can't really put all the blame on a fourteen-year-old for anything: it's also the parents, the coaches, the teachers who are co-responsible. Plus, the incident doesn't change the fact that both Dong and Yang put thousands of hours into perfecting their skills. But, so did other girls, and competition should be fair: putting younger girls against older ones is not. China nor any other country should be allowed to cheat that way.<br /><br />[<a href="http://www.nytimes.com/2010/02/27/sports/olympics/27gymnasts.html">Article</a>, <a href="http://www.gymbox.net/dong_fangxiao.htm">picture credit</a>]Annahttp://www.blogger.com/profile/[email protected]:blogger.com,1999:blog-3689363929101576440.post-86686408843588876122010-04-27T22:11:00.003+02:002010-04-27T23:00:15.143+02:00I'm a Bachelor of Science. Computer Science.As I was writing a little bit before <a href="http://math-geek-rock-chick.blogspot.com/2010/04/tomorrow.html">yesterday</a>, I recently had the opportunity to defend myself, and I'm very happy to announce that I succeeded: I now am a Bachelor of Science in Computer Science. (On the other hand, I don't really know what I should capitalize here, but it sure looks great that way.)<br /><br />Boy was I stressed. Well, I kinda mentioned it already. Plus, it's evident: it was my first defense, but most importantly, I hadn't revised everything I wished.<br /><br />My stress was quickly relieved, however. I came to school over two hours earlier for some last revisions on-line and was drinking a vanilloccino when one of my professor asked me whether I was free. Without thinking too much, I said yes and got taken to a room where defenses were taking place. I took the exam and was free two hours earlier than expected - lucky me!<br /><br />Next defense, coming soon! This time, in mathematics.Annahttp://www.blogger.com/profile/[email protected]:blogger.com,1999:blog-3689363929101576440.post-48893430169713785772010-04-20T10:04:00.004+02:002010-04-20T10:38:47.028+02:00TomorrowTomorrow, I'm defending my Bsc in computer science.<br /><br />Which means I'm revising my eyes out. And napping.<br /><br /><br /><a onblur="try {parent.deselectBloggerImageGracefully();} catch(e) {}" href="http://4.bp.blogspot.com/_R51qCdBeLQU/S81h8XKPBhI/AAAAAAAADiE/-zrHGTgF--s/s1600/funny-pictures-bunny-naps-on-homework.jpg"><img style="display: block; margin: 0px auto 10px; text-align: center; cursor: pointer; width: 400px; height: 375px;" src="http://4.bp.blogspot.com/_R51qCdBeLQU/S81h8XKPBhI/AAAAAAAADiE/-zrHGTgF--s/s400/funny-pictures-bunny-naps-on-homework.jpg" alt="" id="BLOGGER_PHOTO_ID_5462129612707268114" border="0" /></a><br />That's totally a good thing, because Lifehacker keeps saying <a href="http://lifehacker.com/tag/napping/">how cool naps are</a>.<br /><br />Two things I'm not doing are procrastinating (I'll do that later) and panicking. It's weird. I should panic. If I take <a href="http://www.mimuw.edu.pl/studia/dyplomy/egzaminy-koncowe/pytania-licencjacki.html">the list of requirements</a>, the speed I'm revising them, the amount of time left and do the math, I've got reasons to panic. Just not the time, I guess. I need 2-3 more days of 100% focus on studying, but that thing is tomorrow.<br /><br />Unless it isn't, because I need some signature here, something else here and the professor I need them from doesn't have any classes this semester, so he's kinda hard to catch. Honestly, I wouldn't be too disappointed if tomorrow's defense wasn't to happen.<br /><br />[Picture <a href="http://icanhascheezburger.com/2009/10/09/funny-pictures-studying/">credit</a>]Annahttp://www.blogger.com/profile/[email protected]:blogger.com,1999:blog-3689363929101576440.post-4132077460443890112010-03-06T12:29:00.003+01:002010-03-06T12:46:02.997+01:00Random linkage: Cat LaddersThere are blogs about absolutely anything these days. Today, I'll feature <a href="http://catladder.blogspot.com/">Cat Ladders</a>, a blog about - you guessed it - cat ladders! The site features over 800 pictures of cat ladders from 29 countries.<br /><br />Most posts feature simple solutions, like <a href="http://catladder.blogspot.com/2009/09/warsaw-poland.html">this one</a> from Poland:<br /><br /><a onblur="try {parent.deselectBloggerImageGracefully();} catch(e) {}" href="http://2.bp.blogspot.com/_R51qCdBeLQU/S5I-ulf363I/AAAAAAAADfo/5mT743LXrqA/s1600-h/Warsaw-poland-catladder3.JPG"><img style="display: block; margin: 0px auto 10px; text-align: center; cursor: pointer; width: 400px; height: 267px;" src="http://2.bp.blogspot.com/_R51qCdBeLQU/S5I-ulf363I/AAAAAAAADfo/5mT743LXrqA/s400/Warsaw-poland-catladder3.JPG" alt="" id="BLOGGER_PHOTO_ID_5445483869505055602" border="0" /></a><br />There's also <a href="http://catladder.blogspot.com/2009/11/big-cat-ladder-in-noahs-ark-zoo-farm-in.html">something for the big cats</a>:<br /><br /><a onblur="try {parent.deselectBloggerImageGracefully();} catch(e) {}" href="http://4.bp.blogspot.com/_R51qCdBeLQU/S5I-uDu3XmI/AAAAAAAADfY/eGov2Uqc_5Q/s1600-h/article-1215310-068AE64D000005DC-820_634x889.jpg"><img style="display: block; margin: 0px auto 10px; text-align: center; cursor: pointer; width: 285px; height: 400px;" src="http://4.bp.blogspot.com/_R51qCdBeLQU/S5I-uDu3XmI/AAAAAAAADfY/eGov2Uqc_5Q/s400/article-1215310-068AE64D000005DC-820_634x889.jpg" alt="" id="BLOGGER_PHOTO_ID_5445483860441128546" border="0" /></a><br />... and <a href="http://catladder.blogspot.com/2009/08/off-topic-but-wtf-goat-towers-there-is.html">something for not cats</a>:<br /><br /><a onblur="try {parent.deselectBloggerImageGracefully();} catch(e) {}" href="http://4.bp.blogspot.com/_R51qCdBeLQU/S5I-uSaOMJI/AAAAAAAADfg/iarqL608p0w/s1600-h/goat-tower.jpg"><img style="display: block; margin: 0px auto 10px; text-align: center; cursor: pointer; width: 267px; height: 400px;" src="http://4.bp.blogspot.com/_R51qCdBeLQU/S5I-uSaOMJI/AAAAAAAADfg/iarqL608p0w/s400/goat-tower.jpg" alt="" id="BLOGGER_PHOTO_ID_5445483864381075602" border="0" /></a><br />My personal favorite is <a href="http://catladder.blogspot.com/2008/02/ladder-with-style-lengnau-switzerland.html">this one</a>:<br /><br /><a onblur="try {parent.deselectBloggerImageGracefully();} catch(e) {}" href="http://3.bp.blogspot.com/_R51qCdBeLQU/S5I-1NAuBMI/AAAAAAAADf4/9Bhbasfmxhc/s1600-h/katzentreppe1.jpg"><img style="display: block; margin: 0px auto 10px; text-align: center; cursor: pointer; width: 400px; height: 271px;" src="http://3.bp.blogspot.com/_R51qCdBeLQU/S5I-1NAuBMI/AAAAAAAADf4/9Bhbasfmxhc/s400/katzentreppe1.jpg" alt="" id="BLOGGER_PHOTO_ID_5445483983191016642" border="0" /></a><br />Pure royalty! I wonder if it has a little red carpet on it.<br /><br />If you want to learn more about cat ladders and Cat Ladders, there's also <a href="http://catladder.blogspot.com/2009/11/hammarbyhojden-stockholm-sweden.html">an interview with the blog's author</a>. Enjoy!Annahttp://www.blogger.com/profile/[email protected]
{ "url": "http://math-geek-rock-chick.blogspot.com/feeds/posts/default", "source_domain": "math-geek-rock-chick.blogspot.com", "snapshot_id": "crawl=CC-MAIN-2020-24", "warc_metadata": { "Content-Length": "137905", "Content-Type": "application/http; msgtype=response", "WARC-Block-Digest": "sha1:IMSN4AQO7BQPJTHU4QUYY2Z4KEXO3IVK", "WARC-Concurrent-To": "<urn:uuid:346442e1-0b9f-4b4b-9d39-ce0e76de5fb8>", "WARC-Date": "2020-06-05T15:46:18Z", "WARC-IP-Address": "172.217.15.65", "WARC-Identified-Payload-Type": "application/atom+xml", "WARC-Payload-Digest": "sha1:ED4BJM34UQ2O2JDE7CQ2UVAQI6YTKNKG", "WARC-Record-ID": "<urn:uuid:6180b05d-aea6-4c39-a48b-8c073433ab50>", "WARC-Target-URI": "http://math-geek-rock-chick.blogspot.com/feeds/posts/default", "WARC-Truncated": null, "WARC-Type": "response", "WARC-Warcinfo-ID": "<urn:uuid:016eb9de-33b0-4dd3-bf77-9538f46dbdd7>" }, "warc_info": "isPartOf: CC-MAIN-2020-24\r\npublisher: Common Crawl\r\ndescription: Wide crawl of the web for May/June 2020\r\noperator: Common Crawl Admin ([email protected])\r\nhostname: ip-10-67-67-60.ec2.internal\r\nsoftware: Apache Nutch 1.16 (modified, https://github.com/commoncrawl/nutch/)\r\nrobots: checked via crawler-commons 1.1-SNAPSHOT (https://github.com/crawler-commons/crawler-commons)\r\nformat: WARC File Format 1.1\r\nconformsTo: http://iipc.github.io/warc-specifications/specifications/warc-format/warc-1.1/" }
{ "line_start_idx": [ 0 ], "line_end_idx": [ 81576 ] }
{ "red_pajama_v2": { "ccnet_original_length": 81576, "ccnet_original_nlines": 0, "rps_doc_curly_bracket": 0.0007845400250516832, "rps_doc_ldnoobw_words": 1, "rps_doc_lorem_ipsum": 0, "rps_doc_stop_word_fraction": 0.20225143432617188, "rps_doc_ut1_blacklist": 0, "rps_doc_frac_all_caps_words": 0.021615739911794662, "rps_doc_frac_lines_end_with_ellipsis": 0, "rps_doc_frac_no_alph_words": 0.4116450846195221, "rps_doc_frac_unique_words": 0.28049519658088684, "rps_doc_mean_word_length": 7.41582727432251, "rps_doc_num_sentences": 1209, "rps_doc_symbol_to_word_ratio": 0.0011351800058037043, "rps_doc_unigram_entropy": 6.473629951477051, "rps_doc_word_count": 8239, "rps_doc_frac_chars_dupe_10grams": 0.07586049288511276, "rps_doc_frac_chars_dupe_5grams": 0.13685984909534454, "rps_doc_frac_chars_dupe_6grams": 0.11861077696084976, "rps_doc_frac_chars_dupe_7grams": 0.09895416349172592, "rps_doc_frac_chars_dupe_8grams": 0.08509141206741333, "rps_doc_frac_chars_dupe_9grams": 0.07831551879644394, "rps_doc_frac_chars_top_2gram": 0.0021604299545288086, "rps_doc_frac_chars_top_3gram": 0.014681089669466019, "rps_doc_frac_chars_top_4gram": 0.01659601926803589, "rps_doc_books_importance": -8617.158203125, "rps_doc_books_importance_length_correction": -8617.158203125, "rps_doc_openwebtext_importance": -4881.875, "rps_doc_openwebtext_importance_length_correction": -4881.875, "rps_doc_wikipedia_importance": -2984.984375, "rps_doc_wikipedia_importance_length_correction": -2984.984375 }, "fasttext": { "dclm": 0.19983041286468506, "english": 0.7449789047241211, "fineweb_edu_approx": 1.1094372272491455, "eai_general_math": 0.015686089172959328, "eai_open_web_math": 0.1487194299697876, "eai_web_code": 0.017877940088510513 } }
{ "free_decimal_correspondence": { "primary": { "code": "004.0151", "labels": { "level_1": "General works, books and libraries, information sciences", "level_2": "", "level_3": "Computers and Computer science" } }, "secondary": { "code": "720.92", "labels": { "level_1": "Arts", "level_2": "Architecture", "level_3": "" } } }, "bloom_cognitive_process": { "primary": { "code": "2", "label": "Understand" }, "secondary": { "code": "3", "label": "Apply" } }, "bloom_knowledge_domain": { "primary": { "code": "2", "label": "Conceptual" }, "secondary": { "code": "3", "label": "Procedural" } }, "document_type_v1": { "primary": { "code": "9", "label": "Personal/Misc" }, "secondary": { "code": "-1", "label": "Abstain" } }, "extraction_artifacts": { "primary": { "code": "3", "label": "Irrelevant Content" }, "secondary": { "code": "0", "label": "No Artifacts" } }, "missing_content": { "primary": { "code": "4", "label": "Missing Images or Figures" }, "secondary": { "code": "0", "label": "No missing content" } }, "document_type_v2": { "primary": { "code": "16", "label": "Personal Blog" }, "secondary": { "code": "10", "label": "Knowledge Article" } }, "reasoning_depth": { "primary": { "code": "2", "label": "Basic Reasoning" }, "secondary": { "code": "3", "label": "Intermediate Reasoning" } }, "technical_correctness": { "primary": { "code": "6", "label": "Not Applicable/Indeterminate" }, "secondary": { "code": "3", "label": "Mostly Correct" } }, "education_level": { "primary": { "code": "1", "label": "General Audience" }, "secondary": { "code": "2", "label": "High School Level" } } }
672f1e42c33a7f9846924a2431ea77df
-5,813,686,470,711,737,000
Learn how you can become a Python programmer in just 12 weeks. We respect your privacy. Unsubscribe at anytime. Learn Python Fast in 2023 The key to learning Python Fast Are you ready to unlock the power of programming and take your skills to the next level and learn Python fast in 2023? Look no further than Python, the versatile and dynamic language that’s taking the tech world by storm. Python has a user-friendly syntax and can be the perfect choice for you if you are looking to break into the exciting and ever-evolving world of coding. Whether you’re interested in building your own software, creating web applications, or analyzing data, Python has the tools you need to succeed. So why wait? Start your journey today and discover the endless possibilities that await Python! How to learn python fast In this guide, we will discover how you can learn Python fast in 2023. 1. Set a clear goal: Determine why you want to learn Python and what you want to accomplish with it. This will help you focus your learning efforts and motivate you to keep going. 2. Learn the basics: Start with the fundamentals of Python programming such as variables, data types, functions, loops, and conditionals. 3. Practice coding regularly: The more you code, the faster you’ll learn. Start with simple exercises and gradually move on to more complex programs. 4. Find a mentor or join a community: Learning from someone who has experience in Python can help you learn faster and avoid common pitfalls. 5. Use Python libraries and frameworks: Python has many powerful libraries and frameworks that can simplify your coding and help you build more complex applications faster. 6. Work on projects: Try building real-world applications using Python. This will help you gain practical experience and learn how to apply your knowledge to solve real problems. 7. Be patient and persistent: Learning Python takes time and effort. Don’t expect to become an expert overnight. Stay patient, be persistent, and keep practicing consistently. Remember that the key to learning Python fast is to stay focused, practice regularly, and stay motivated. With the right approach, you can quickly become proficient in Python and start building your own applications. Set a clear goal for your Python journey I know it can be difficult to set a goal when you are just starting to learn to program in Python. But this is one of the things that will speed up your learning. If you want to learn Python fast, you need to know what to learn and more importantly, what not to learn. Python is known for its versatility, which is a core strength of the language, but it also makes it difficult when you are a learner. There are so many options and possibilities to dive into. Python goal importance A clear goal will help you both to learn faster and stay focused. 1. Motivation: Having a specific goal in mind can help you stay motivated and focused as you learn Python. It gives you a reason to continue learning and practicing, even when the material may be challenging or confusing. 2. Relevance: Knowing why you are learning Python and what you hope to accomplish with it can help you stay engaged with the material. It makes the learning process more relevant to your needs and interests. 3. Efficiency: By having a clear goal, you can focus your learning on the specific skills and concepts that are most relevant to your goal. This can help you learn more efficiently and avoid wasting time on topics that are not important to you. 4. Measurability: Having a goal can help you track your progress and measure your success as you learn Python. You can set milestones or benchmarks for yourself and celebrate your achievements along the way. 5. Practicality: Learning Python without a goal can leave you with a bunch of disconnected knowledge that is hard to apply in the real world. Having a goal can help you see how the things you are learning can be used to solve real problems and create practical solutions. Learn the basics of Python fast If you want to learn Python fast you need to focus on the Python Essentials, which are the basics that you need in order to master programming and avoid learning fancy constructions that will not enable you to do anything you can’t do without. Roadmap to learn Python fast 1. Learn variables, data types, operators, and control structures. This is the basics of Python programs and will enable you to solve simple problems and will give you a solid foundation to build on. 2. Practice data structures, like lists, dictionaries, sets, and tuples. Data is the core of a program, and mastering the Python built-in data structures will help you work with data. 3. Learn functions, which will enable you to organize your code into smaller blocks of code. They enable you to solve bigger problems in a structured way, where you can test if each function works as expected. 4. Learn how to work with files. Learn how to read from and write to files in Python. This is a fundamental skill that you’ll need for many programming tasks. 5. Learn how to work with libraries. Python has a vast collection of libraries that you can use to extend its functionality. Learn how to install and use popular libraries like NumPy, Pandas, and Matplotlib. This will enable you to create powerful applications with Python. Practice coding regularly to learn Python fast While it shouldn’t be a surprise, you need to code to learn to code. And code a lot. The main reasons to code a lot include the following. 1. Practice makes perfect. The more you code, the more comfortable and efficient you will become with the language. 2. A better understanding of the language. Regular coding will help you better understand the syntax, rules, and concepts of Python. 3. Problem-solving skills. Coding regularly challenges you to solve problems and think critically, which improves your problem-solving skills. 4. Access to resources. When you code regularly, you become more involved in the programming community and have access to a wide range of resources, tutorials, and support. 5. Better retention. Regular coding ensures that you retain the information you learn and build on it over time. 6. Keeping up with updates. Python is an evolving language, and coding regularly ensures that you stay up-to-date with the latest updates and changes. 7. Career opportunities. Learning Python through regular coding can open up various career opportunities in fields such as data science, machine learning, web development, and more. 8. Fun and rewarding. Coding in Python can be a fun and rewarding experience, especially when you see the results of your efforts come to life. Please remember the last one. Keep it fun and rewarding. I know sitting with a problem alone can be frustrating. This is why the next one is important to learn Python fast in 2023. Find a mentor or join a community You sit with your Python program that just doesn’t work as you expect it to work. This problem bothers you. You asked an A.I. service, but the answer is not fulfilling and the solution does not work. Join Python community In reality, what you need is an experienced programmer to ask. What you need, is to join a community. 1. Access to a support system. Being part of a community or having a mentor provides access to a support system of like-minded individuals who can provide feedback, answer questions, and offer guidance. 2. Faster learning. Being part of a community or having a mentor can help accelerate the learning process by providing access to resources, tips, and strategies that you may not have discovered on your own. 3. Accountability. A community or mentor can help keep you accountable and motivated to continue learning and practicing Python regularly. 4. Networking opportunities. Being part of a community can help you expand your network and connect with other professionals in the field, which can lead to new opportunities. 5. Exposure to new ideas. Being part of a community or having a mentor can expose you to new ideas, perspectives, and approaches that you may not have considered on your own. 6. Feedback on your work. A community or mentor can provide valuable feedback on your code and projects, helping you improve and grow as a Python programmer. 7. Avoiding common mistakes. By learning from experienced programmers, you can avoid common mistakes and pitfalls that many beginners make. 8. Personalized learning. A mentor can provide personalized guidance and support tailored to your specific learning needs and goals. 9. Fun and socializing. Being part of a community can also be a fun and social experience, as you connect with others who share your passion for Python programming. Now you have joined a community it is time for the next step to learn Python fast in 2023. Use Python libraries and frameworks The to really utilize the power of Python is by using libraries and frameworks. Power of using libraries and frameworks in Python 1. Reduced Development Time. When a programmer uses a library or framework, they can leverage pre-built code modules that solve common problems, which means they can complete their work faster. For example, instead of writing code from scratch to handle HTTP requests, a programmer can use the popular requests library, which is optimized and pre-built, saving a significant amount of time. 2. Higher Productivity. By using libraries and frameworks, programmers can focus on high-level tasks and logic, rather than spending time on low-level implementation details. This results in increased productivity, allowing programmers to get more done in less time. 3. Increased Efficiency. Libraries and frameworks are often optimized for performance and efficiency, meaning that they can execute tasks faster than if a programmer were to write the code from scratch. For example, if a programmer needs to process large datasets, they can use the pandas’ library, which is specifically designed for high-performance data analysis, and can process large datasets much faster than if they were to write the code themselves. 4. Access to Expertise. Libraries and frameworks are often created and maintained by expert developers, who have already solved common problems and can provide solutions that are optimized and tested. This means that by using these pre-built tools, programmers can leverage the expertise of others, which can help them achieve results faster. 5. Community Support. Libraries and frameworks often have large and active communities of developers who share knowledge and provide support. This means that if a programmer encounters a problem or needs help, they can often find answers quickly, saving time and helping them achieve results faster. While this explains why you want to start using libraries and frameworks there are some other benefits. 1. Reusable Code. Libraries and frameworks provide pre-built and reusable code modules that can be easily integrated into your own projects. This saves time and effort that would otherwise be spent building everything from scratch. 2. Abstraction. Libraries and frameworks often abstract away the low-level details of a task, allowing you to focus on the high-level logic of your project. This allows you to be more productive and make progress faster. 3. Robustness. Libraries and frameworks are often tested and debugged by a large number of users, which can help to identify and resolve issues faster than if you were working alone. 4. Standardization. Many libraries and frameworks follow standardized best practices and coding conventions, which can help to ensure that your code is consistent and maintainable over time. Work on projects and learn Python fast Getting experience is the key to becoming a good programmer. Python project ideas With your community make projects that resemble real-world projects done with Python. 1. Build a website using Django or Flask. Learn to develop web applications using Python web frameworks, which can help you build robust, scalable, and maintainable web applications. 2. Develop a chatbot using Natural Language Processing techniques. Learn to use NLP techniques to build intelligent chatbots that can understand and respond to natural language queries. 3. Create a data visualization dashboard using libraries like Matplotlib, Plotly, or Bokeh. Learn to use data visualization libraries to create interactive and informative data dashboards, which can help you present your data in a visually appealing manner. 4. Develop a machine learning model to predict customer churn or classify images. Learn to use machine learning algorithms to build predictive models, which can help you solve real-world business problems such as customer churn prediction or image classification. 5. Build a recommendation system using collaborative filtering techniques. Learn to use collaborative filtering techniques to build personalized recommendation systems, which can help you provide a better user experience to your customers. 6. Develop a web scraper to collect data from websites using libraries like BeautifulSoup or Scrapy. Learn to extract data from websites by writing web scrapers, which can help you automate data collection tasks. 7. Implement a blockchain-based application using Python. Learn to use Python to build decentralized applications using blockchain technology, which can help you build secure and transparent applications. 8. Create a game using Pygame or Unity game engine with Python scripting. Learn to use Python for game development, which can help you build engaging and interactive games. Be patient and persistent While we live in a world where everything is fast-paced getting new skills takes time. Learning Python can sometimes feel like a Marathon. But remember, everyone, can finish a Marathon. Just slow down and walk the distance, then you will eventually reach the goal. Python for Finance: Unlock Financial Freedom and Build Your Dream Life Discover the key to financial freedom and secure your dream life with Python for Finance! Say goodbye to financial anxiety and embrace a future filled with confidence and success. If you’re tired of struggling to pay bills and longing for a life of leisure, it’s time to take action. Imagine breaking free from that dead-end job and opening doors to endless opportunities. With Python for Finance, you can acquire the invaluable skill of financial analysis that will revolutionize your life. Make informed investment decisions, unlock the secrets of business financial performance, and maximize your money like never before. Gain the knowledge sought after by companies worldwide and become an indispensable asset in today’s competitive market. Don’t let your dreams slip away. Master Python for Finance and pave your way to a profitable and fulfilling career. Start building the future you deserve today! Python for Finance a 21 hours course that teaches investing with Python. Learn pandas, NumPy, Matplotlib for Financial Analysis & learn how to Automate Value Investing. “Excellent course for anyone trying to learn coding and investing.” – Lorenzo B. Leave a Comment
{ "url": "https://www.learnpythonwithrune.org/learn-python-fast-in-2023/", "source_domain": "www.learnpythonwithrune.org", "snapshot_id": "CC-MAIN-2023-23", "warc_metadata": { "Content-Length": "124472", "Content-Type": "application/http; msgtype=response", "WARC-Block-Digest": "sha1:4VQFKK6UOJQJKUIWMY6JAZM3RLEPY3FY", "WARC-Concurrent-To": "<urn:uuid:b15739ff-c5e6-4964-90e9-7615e3819742>", "WARC-Date": "2023-06-08T11:45:49Z", "WARC-IP-Address": "34.160.81.203", "WARC-Identified-Payload-Type": "text/html", "WARC-Payload-Digest": "sha1:WIARYAMJXIOCMMIOZPPEJLVKBYHIPI3L", "WARC-Record-ID": "<urn:uuid:2b63a3ac-173e-4111-802e-72dd4b560749>", "WARC-Target-URI": "https://www.learnpythonwithrune.org/learn-python-fast-in-2023/", "WARC-Truncated": null, "WARC-Type": "response", "WARC-Warcinfo-ID": "<urn:uuid:591d9095-3710-48a6-9b9a-b0f51a0ee3b2>" }, "warc_info": "isPartOf: CC-MAIN-2023-23\r\npublisher: Common Crawl\r\ndescription: Wide crawl of the web for May/June 2023\r\noperator: Common Crawl Admin ([email protected])\r\nhostname: ip-10-67-67-248\r\nsoftware: Apache Nutch 1.19 (modified, https://github.com/commoncrawl/nutch/)\r\nrobots: checked via crawler-commons 1.4-SNAPSHOT (https://github.com/crawler-commons/crawler-commons)\r\nformat: WARC File Format 1.1\r\nconformsTo: https://iipc.github.io/warc-specifications/specifications/warc-format/warc-1.1/" }
{ "line_start_idx": [ 0, 63, 64, 117, 118, 148, 149, 185, 186, 309, 310, 417, 418, 575, 576, 725, 726, 743, 744, 831, 832, 861, 862, 937, 938, 1122, 1264, 1418, 1564, 1741, 1924, 2104, 2105, 2326, 2327, 2372, 2373, 2540, 2541, 2651, 2652, 2848, 2849, 2876, 2877, 2947, 2948, 3174, 3386, 3635, 3847, 4123, 4124, 4160, 4161, 4409, 4410, 4443, 4647, 4835, 5049, 5212, 5424, 5425, 5495, 5496, 5547, 5548, 5637, 5638, 5696, 5697, 5817, 5954, 6101, 6278, 6395, 6550, 6736, 6884, 6885, 6946, 6947, 7007, 7008, 7080, 7081, 7119, 7120, 7206, 7207, 7237, 7238, 7334, 7335, 7361, 7362, 7468, 7469, 7676, 7887, 8030, 8210, 8389, 8551, 8695, 8832, 9001, 9002, 9097, 9098, 9138, 9139, 9223, 9224, 9278, 9673, 9944, 10405, 10752, 11056, 11057, 11165, 11166, 11402, 11627, 11814, 12009, 12010, 12053, 12054, 12119, 12120, 12145, 12146, 12236, 12237, 12424, 12614, 12876, 13144, 13388, 13605, 13814, 13991, 13992, 14022, 14023, 14114, 14115, 14171, 14172, 14302, 14303, 14378, 14379, 14473, 14474, 14672, 14673, 14885, 14886, 15143, 15144, 15309, 15310, 15387, 15388, 15488, 15489, 15574, 15575 ], "line_end_idx": [ 63, 64, 117, 118, 148, 149, 185, 186, 309, 310, 417, 418, 575, 576, 725, 726, 743, 744, 831, 832, 861, 862, 937, 938, 1122, 1264, 1418, 1564, 1741, 1924, 2104, 2105, 2326, 2327, 2372, 2373, 2540, 2541, 2651, 2652, 2848, 2849, 2876, 2877, 2947, 2948, 3174, 3386, 3635, 3847, 4123, 4124, 4160, 4161, 4409, 4410, 4443, 4647, 4835, 5049, 5212, 5424, 5425, 5495, 5496, 5547, 5548, 5637, 5638, 5696, 5697, 5817, 5954, 6101, 6278, 6395, 6550, 6736, 6884, 6885, 6946, 6947, 7007, 7008, 7080, 7081, 7119, 7120, 7206, 7207, 7237, 7238, 7334, 7335, 7361, 7362, 7468, 7469, 7676, 7887, 8030, 8210, 8389, 8551, 8695, 8832, 9001, 9002, 9097, 9098, 9138, 9139, 9223, 9224, 9278, 9673, 9944, 10405, 10752, 11056, 11057, 11165, 11166, 11402, 11627, 11814, 12009, 12010, 12053, 12054, 12119, 12120, 12145, 12146, 12236, 12237, 12424, 12614, 12876, 13144, 13388, 13605, 13814, 13991, 13992, 14022, 14023, 14114, 14115, 14171, 14172, 14302, 14303, 14378, 14379, 14473, 14474, 14672, 14673, 14885, 14886, 15143, 15144, 15309, 15310, 15387, 15388, 15488, 15489, 15574, 15575, 15594 ] }
{ "red_pajama_v2": { "ccnet_original_length": 15594, "ccnet_original_nlines": 171, "rps_doc_curly_bracket": 0, "rps_doc_ldnoobw_words": 0, "rps_doc_lorem_ipsum": 0, "rps_doc_stop_word_fraction": 0.40188220143318176, "rps_doc_ut1_blacklist": 0, "rps_doc_frac_all_caps_words": 0.0041826399974524975, "rps_doc_frac_lines_end_with_ellipsis": 0, "rps_doc_frac_no_alph_words": 0.14952945709228516, "rps_doc_frac_unique_words": 0.28136420249938965, "rps_doc_mean_word_length": 4.985789775848389, "rps_doc_num_sentences": 218, "rps_doc_symbol_to_word_ratio": 0, "rps_doc_unigram_entropy": 5.5289483070373535, "rps_doc_word_count": 2463, "rps_doc_frac_chars_dupe_10grams": 0.012052119709551334, "rps_doc_frac_chars_dupe_5grams": 0.08843647688627243, "rps_doc_frac_chars_dupe_6grams": 0.034690551459789276, "rps_doc_frac_chars_dupe_7grams": 0.01938110962510109, "rps_doc_frac_chars_dupe_8grams": 0.014983709901571274, "rps_doc_frac_chars_dupe_9grams": 0.014983709901571274, "rps_doc_frac_chars_top_2gram": 0.011400650255382061, "rps_doc_frac_chars_top_3gram": 0.011400650255382061, "rps_doc_frac_chars_top_4gram": 0.009690550155937672, "rps_doc_books_importance": -1214.063232421875, "rps_doc_books_importance_length_correction": -1214.063232421875, "rps_doc_openwebtext_importance": -686.3749389648438, "rps_doc_openwebtext_importance_length_correction": -686.3749389648438, "rps_doc_wikipedia_importance": -492.6685791015625, "rps_doc_wikipedia_importance_length_correction": -492.6685791015625 }, "fasttext": { "dclm": 0.9704821705818176, "english": 0.9338273406028748, "fineweb_edu_approx": 2.7845537662506104, "eai_general_math": 0.1444912552833557, "eai_open_web_math": 0.07215554267168045, "eai_web_code": 0.770592212677002 } }
{ "free_decimal_correspondence": { "primary": { "code": "005.133", "labels": { "level_1": "General works, books and libraries, information sciences", "level_2": "", "level_3": "Computer programming" } }, "secondary": { "code": "332.0285", "labels": { "level_1": "Social sciences", "level_2": "Economics", "level_3": "Finance" } } }, "bloom_cognitive_process": { "primary": { "code": "3", "label": "Apply" }, "secondary": { "code": "2", "label": "Understand" } }, "bloom_knowledge_domain": { "primary": { "code": "3", "label": "Procedural" }, "secondary": { "code": "2", "label": "Conceptual" } }, "document_type_v1": { "primary": { "code": "3", "label": "Reference/Encyclopedic/Educational" }, "secondary": { "code": "6", "label": "Promotional/Advertisement" } }, "extraction_artifacts": { "primary": { "code": "0", "label": "No Artifacts" }, "secondary": { "code": "3", "label": "Irrelevant Content" } }, "missing_content": { "primary": { "code": "0", "label": "No missing content" }, "secondary": { "code": "2", "label": "Click Here References" } }, "document_type_v2": { "primary": { "code": "23", "label": "Tutorial" }, "secondary": { "code": "17", "label": "Product Page" } }, "reasoning_depth": { "primary": { "code": "2", "label": "Basic Reasoning" }, "secondary": { "code": "3", "label": "Intermediate Reasoning" } }, "technical_correctness": { "primary": { "code": "3", "label": "Mostly Correct" }, "secondary": { "code": "6", "label": "Not Applicable/Indeterminate" } }, "education_level": { "primary": { "code": "1", "label": "General Audience" }, "secondary": { "code": "2", "label": "High School Level" } } }
672f1e42c33a7f9846924a2431ea77df
-1,370,907,280,601,872,600
Start an iteration on first row of a group Pandas 0 votes I have a dataset like this: Policy | Customer | Employee | CoveragDate | LapseDate 123 | 1234 | 1234 | 2011-06-01 | 2015-12-31 124 | 1234 | 1234 | 2016-01-01 | ? 125 | 1234 | 1234 | 2011-06-01 | 2012-01-01 124 | 5678 | 5555 | 2014-01-01 | ? I'm trying to iterate through each policy for each employee of each customer (a customer can have many employees, an employee can have multiple policies) and compare the covered date against the lapse date for a particular employee. If the covered date and lapse date are within 5 days, I'd like to add that policy to a results list. So, expected output would be: Policy | Customer | Employee 123 | 1234 | 1234 because policy 123's lapse date was within 5 days of policy 124's covered date. I'm running into a problem while trying to iterate through each grouping of Customer/Employee numbers. I'm able to identify how many rows of data are in each EmployeeID/Customer number (EBCN below) group, but I need to reference specific data within those rows to assign variables for comparison. So far, I've been able to write this code: import pandas import datetime wd = pandas.read_csv(DATASOURCE) l = 0 for row, i in wd.groupby(['EMPID', 'EBCN']).size().iteritems(): Covdt = pandas.to_datetime(wd.loc[l, 'CoverageEffDate']) for each in range(i): LapseDt = wd.loc[l, 'LapseDate'] if LapseDt != '?': LapseDt = pandas.to_datetime(LapseDt) + datetime.timedelta(days=5) if Covdt < LapseDt: print('got one!') l = l + 1 This code is not working because I'm trying to reference the coverage date/lapse dates on a particular row with the loc function, with my row number stored in the 'l' variable. I initially thought that Pandas would iterate through groups in the order they appear in my dataset, so that I could simply start with l=0 (i.e. the first row in the data), assign the coverage date and lapse date variables based on that, and then move on, but it appears that Pandas starts iterating through groups randomly. As a result, I do indeed get a comparison of lapse/coverage dates, but they're not associated with the groups that end up getting output by the code. The best solution I can figure is to determine what the row number is for the first row of each group and then iterate forward by the number of rows in that group. I've read through a question regarding finding the first row of a group, and am able to do so by using wd.groupby(['EMPID','EBCN']).first() but I haven't been able to figure out what row number the results are stored on in a way that I can reference with the loc function. Is there a way to store the row number for the first row of a group in a variable or something so I can iterate my coverage date and lapse date comparison forward from there? Regarding my general method, I've read through the question here, which is very close to what I need: pandas computation in each group however, I need to compare each policy in the group against each other policy in the group - the question above just compares the last row in each group against the others. Is there a way to do what I'm attempting in Pandas/Python? Sep 6, 2018 in Python by bug_seeker • 15,400 points 911 views 1 answer to this question. 0 votes For anyone needing this information in the future - I was able to implement Boud's suggestion to use the pandas.merge_asof() function to replace my code above. I had to do some data manipulation to get the desired result: 1. Splitting the dataframe into two separate frames - one with CoverageDate and one with LapseDate. 2. Replacing the '?' (null values) in my data with a numpy.nan datatype 3. Sorting the left and right dataframes by the Date columns Once the data was in the correct format, I implemented the merge: pandas.merge_asof(cov, term, on='Date', by='EMP|EBCN', tolerance=pandas.Timedelta('5 days')) Note 'cov' is my dataframe containing coverage dates, term is the dataframe with lapses. The 'EMP|EBCN' column is a concatenated column of the employee ID and Customer # fields, to allow easy use of the 'by' field. answered Sep 6, 2018 by Priyaj • 57,550 points Related Questions In Python +1 vote 7 answers Count the frequency of an item in a python list To count the number of appearances: from collections ...READ MORE answered Oct 18, 2018 in Python by tinitales 8,351 views 0 votes 1 answer How can I find out the index of an element from row and column in Python? You probably want to use np.ravel_multi_index: [code] import numpy ...READ MORE answered Apr 16, 2018 in Python by charlie_brown • 7,780 points 274 views +1 vote 1 answer How to print first character of each word in upper case of a string in Python You can use split() using space as ...READ MORE answered Jun 1, 2018 in Python by george • 200 points 4,811 views 0 votes 1 answer Question on PyQt: How to connect a signal to a slot to start a background operation in Python It shouldn't matter whether the connection is ...READ MORE answered Nov 27, 2018 in Python by Nymeria • 3,520 points 489 views +1 vote 2 answers how can i count the items in a list? Syntax :            list. count(value) Code: colors = ['red', 'green', ...READ MORE answered Jul 6, 2019 in Python by Neha • 330 points edited Jul 8, 2019 by Kalgi 1,191 views 0 votes 0 answers 0 votes 1 answer How to sort a list of strings? Try  items = ["live", "like", "code", "cool", "bug"] ...READ MORE answered Jul 27, 2018 in Python by Priyaj • 57,550 points 89 views
{ "url": "https://www.edureka.co/community/19074/start-an-iteration-on-first-row-of-a-group-pandas?show=19075", "source_domain": "www.edureka.co", "snapshot_id": "crawl=CC-MAIN-2020-34", "warc_metadata": { "Content-Length": "160221", "Content-Type": "application/http; msgtype=response", "WARC-Block-Digest": "sha1:PI2YQPJHN3K5FPJJW22P4DUSJ6AOSTQK", "WARC-Concurrent-To": "<urn:uuid:fe339a7b-adf3-40b2-b684-859fec8cdb41>", "WARC-Date": "2020-08-09T07:39:11Z", "WARC-IP-Address": "52.85.144.100", "WARC-Identified-Payload-Type": "text/html", "WARC-Payload-Digest": "sha1:P4QBYKULH54Y3AQMRLLTEBFNABOSYXY6", "WARC-Record-ID": "<urn:uuid:d1921020-4a25-4c3d-9c3d-13e6b960b87b>", "WARC-Target-URI": "https://www.edureka.co/community/19074/start-an-iteration-on-first-row-of-a-group-pandas?show=19075", "WARC-Truncated": null, "WARC-Type": "response", "WARC-Warcinfo-ID": "<urn:uuid:b9a76de8-68b9-4b87-a93e-379ed9386cdf>" }, "warc_info": "isPartOf: CC-MAIN-2020-34\r\npublisher: Common Crawl\r\ndescription: Wide crawl of the web for August 2020\r\noperator: Common Crawl Admin ([email protected])\r\nhostname: ip-10-67-67-82.ec2.internal\r\nsoftware: Apache Nutch 1.17 (modified, https://github.com/commoncrawl/nutch/)\r\nrobots: checked via crawler-commons 1.2-SNAPSHOT (https://github.com/crawler-commons/crawler-commons)\r\nformat: WARC File Format 1.1\r\nconformsTo: http://iipc.github.io/warc-specifications/specifications/warc-format/warc-1.1/" }
{ "line_start_idx": [ 0, 50, 51, 59, 60, 88, 89, 144, 200, 247, 303, 350, 351, 685, 686, 716, 717, 746, 771, 772, 852, 853, 1150, 1151, 1194, 1195, 1209, 1225, 1226, 1259, 1265, 1329, 1390, 1416, 1457, 1484, 1563, 1595, 1629, 1647, 1648, 2300, 2301, 2465, 2466, 2569, 2570, 2607, 2608, 2916, 2917, 3019, 3020, 3053, 3054, 3227, 3228, 3287, 3288, 3324, 3340, 3350, 3351, 3378, 3379, 3387, 3388, 3610, 3611, 3713, 3787, 3850, 3851, 3917, 3918, 3947, 3962, 3981, 4023, 4024, 4239, 4240, 4271, 4287, 4288, 4316, 4317, 4325, 4335, 4336, 4384, 4385, 4451, 4452, 4497, 4509, 4517, 4526, 4527, 4601, 4602, 4682, 4683, 4732, 4747, 4757, 4765, 4774, 4775, 4853, 4854, 4902, 4903, 4944, 4957, 4969, 4977, 4986, 4987, 5081, 5082, 5141, 5142, 5185, 5200, 5210, 5218, 5228, 5229, 5266, 5267, 5351, 5352, 5391, 5404, 5405, 5445, 5453, 5463, 5471, 5480, 5481, 5512, 5513, 5579, 5580, 5622, 5638 ], "line_end_idx": [ 50, 51, 59, 60, 88, 89, 144, 200, 247, 303, 350, 351, 685, 686, 716, 717, 746, 771, 772, 852, 853, 1150, 1151, 1194, 1195, 1209, 1225, 1226, 1259, 1265, 1329, 1390, 1416, 1457, 1484, 1563, 1595, 1629, 1647, 1648, 2300, 2301, 2465, 2466, 2569, 2570, 2607, 2608, 2916, 2917, 3019, 3020, 3053, 3054, 3227, 3228, 3287, 3288, 3324, 3340, 3350, 3351, 3378, 3379, 3387, 3388, 3610, 3611, 3713, 3787, 3850, 3851, 3917, 3918, 3947, 3962, 3981, 4023, 4024, 4239, 4240, 4271, 4287, 4288, 4316, 4317, 4325, 4335, 4336, 4384, 4385, 4451, 4452, 4497, 4509, 4517, 4526, 4527, 4601, 4602, 4682, 4683, 4732, 4747, 4757, 4765, 4774, 4775, 4853, 4854, 4902, 4903, 4944, 4957, 4969, 4977, 4986, 4987, 5081, 5082, 5141, 5142, 5185, 5200, 5210, 5218, 5228, 5229, 5266, 5267, 5351, 5352, 5391, 5404, 5405, 5445, 5453, 5463, 5471, 5480, 5481, 5512, 5513, 5579, 5580, 5622, 5638, 5646 ] }
{ "red_pajama_v2": { "ccnet_original_length": 5646, "ccnet_original_nlines": 147, "rps_doc_curly_bracket": 0, "rps_doc_ldnoobw_words": 0, "rps_doc_lorem_ipsum": 0, "rps_doc_stop_word_fraction": 0.3301282227039337, "rps_doc_ut1_blacklist": 0, "rps_doc_frac_all_caps_words": 0.03766025975346565, "rps_doc_frac_lines_end_with_ellipsis": 0, "rps_doc_frac_no_alph_words": 0.2892628312110901, "rps_doc_frac_unique_words": 0.37254902720451355, "rps_doc_mean_word_length": 4.525054454803467, "rps_doc_num_sentences": 55, "rps_doc_symbol_to_word_ratio": 0.005608969833701849, "rps_doc_unigram_entropy": 5.239841938018799, "rps_doc_word_count": 918, "rps_doc_frac_chars_dupe_10grams": 0, "rps_doc_frac_chars_dupe_5grams": 0.0779971107840538, "rps_doc_frac_chars_dupe_6grams": 0.009147809818387032, "rps_doc_frac_chars_dupe_7grams": 0, "rps_doc_frac_chars_dupe_8grams": 0, "rps_doc_frac_chars_dupe_9grams": 0, "rps_doc_frac_chars_top_2gram": 0.021184399724006653, "rps_doc_frac_chars_top_3gram": 0.016851229593157768, "rps_doc_frac_chars_top_4gram": 0.020221469923853874, "rps_doc_books_importance": -478.00469970703125, "rps_doc_books_importance_length_correction": -478.00469970703125, "rps_doc_openwebtext_importance": -314.7068786621094, "rps_doc_openwebtext_importance_length_correction": -314.7068786621094, "rps_doc_wikipedia_importance": -214.91506958007812, "rps_doc_wikipedia_importance_length_correction": -214.91506958007812 }, "fasttext": { "dclm": 0.7784488797187805, "english": 0.901790976524353, "fineweb_edu_approx": 1.920082688331604, "eai_general_math": 0.9093532562255859, "eai_open_web_math": 0.1839374303817749, "eai_web_code": 0.8269235491752625 } }
{ "free_decimal_correspondence": { "primary": { "code": "005.1", "labels": { "level_1": "General works, books and libraries, information sciences", "level_2": "", "level_3": "Computer programming" } }, "secondary": { "code": "519.5", "labels": { "level_1": "Science and Natural history", "level_2": "Mathematics", "level_3": "Probabilities; or, Mathematical statistics" } } }, "bloom_cognitive_process": { "primary": { "code": "3", "label": "Apply" }, "secondary": { "code": "4", "label": "Analyze" } }, "bloom_knowledge_domain": { "primary": { "code": "3", "label": "Procedural" }, "secondary": { "code": "2", "label": "Conceptual" } }, "document_type_v1": { "primary": { "code": "5", "label": "Social/Forum" }, "secondary": { "code": "4", "label": "Code/Software" } }, "extraction_artifacts": { "primary": { "code": "0", "label": "No Artifacts" }, "secondary": { "code": "3", "label": "Irrelevant Content" } }, "missing_content": { "primary": { "code": "0", "label": "No missing content" }, "secondary": { "code": "-1", "label": "Abstain" } }, "document_type_v2": { "primary": { "code": "18", "label": "Q&A Forum" }, "secondary": { "code": "23", "label": "Tutorial" } }, "reasoning_depth": { "primary": { "code": "3", "label": "Intermediate Reasoning" }, "secondary": { "code": "2", "label": "Basic Reasoning" } }, "technical_correctness": { "primary": { "code": "4", "label": "Highly Correct" }, "secondary": { "code": "3", "label": "Mostly Correct" } }, "education_level": { "primary": { "code": "3", "label": "Undergraduate Level" }, "secondary": { "code": "2", "label": "High School Level" } } }
672f1e42c33a7f9846924a2431ea77df
5,661,330,389,683,406,000
User Tools Site Tools Sidebar Passage markup basics coding extra list section whitespace Macros basics colour data structure date and time game state live maths number revision saving showing and hiding string styling url deprecated Types of data Special keywords Special passage tags harlowe:lambda This is an old revision of the document! Lambda data Suppose you want to do a complicated task with an Array, like, say, convert all of its strings to lowercase, or check if its datamap have “health” data equal to 0, or join all of its strings together into a single string. You want to be able to tell Harlowe to search for “each string where the string's 1st letter is A”. You want to write a “function” for how the search is to be conducted. Lambdas are user-created functions that let you tell certain macros, like (find:), (altered:) and (folded:), precisely how to search, alter, or combine the data provided to them. There are several types of lambdas. • “where” lambdas, used by the (find:) macro, are used to search for and filter data. The lambda _item where _item's 1st is "A" tells the macro to searches for items whose 1st is the string “A”. • “via” lambdas, used by the (altered:) macro, are used to transform and change data. The lambda _item via _item + "s" tells the macro to add the string “s” to the end of each item. • “when” lambdas are a variant of “where” used exclusively by (event:), and are used to specify a live event when a hook should be shown. The lambda when $fuel > 8 tells (event:) to show the attached hook when $fuel is increased (due to an interaction macro like (click-repeat:), a (live:) macro, or anything else). This really shouln't be called a “lambda”, but you can perhaps think of it in terms of it filtering moments in time that pass or fail the condition. • “making” lambdas, used by the (folded:) are used to build or “make” a single data value by adding something from each item to it. The lambda _item making _total via _total + (max: _item, 0) tells the macro to add each item to the total, but only if the item is greater than 0. (Incidentally, you can also use “where” inside a “making” lambda - you could rewrite that lambda as _item making _total via _total + _item where _item > 0.) • For certain macros, like (for:), you may want to use a “where” lambda that doesn't filter out any of the values - _item where true, for instance, will include every item. There is a special, more readable shorthand for this type of “where” lambda: writing just each _item is equivalent. Lambdas use temp variables as “placeholders” for the actual values. For instance, in (find: _num where _num > 2, 5,6,0), the temp variable _num is used to mean each individual value given to the macro, in turn. It will be 5, 6 and 0, respectively. Importantly, this will not alter any existing temp variable called _num - the inside of a lambda can be thought of as a hook, so just as the inner _x in (set: _x to 1) |a>[ (set:_x to 2) ] is different from the outer _x, the _num in the lambda will not affect any other _num. An important feature is that you can save lambdas into variables, and reuse them in your story easily. You could, for instance, (set: $statsReadout to (_stat making _readout via _readout + "|" + _stat's name + ":" + _stat's value)), and then use $printStats with the (folded:) macro in different places, such as (folded: $statsReadout, ...(dataentries: $playerStats)) for displaying the player's stats, (folded: $statsReadout, ...(dataentries: $monsterStats)) for a monster's stats, etc. Lambdas are named after the lambda calculus, and the “lambda” keyword used in many popular programming languages. They may seem complicated, but as long as you think of them as just a special way of writing a repeating instruction, and understand how their macros work, you may find that they are very convenient. harlowe/lambda.1555401955.txt.gz · Last modified: 2019/04/16 04:05 by l
{ "url": "http://twinery.org/wiki/harlowe:lambda?rev=1555401955", "source_domain": "twinery.org", "snapshot_id": "crawl=CC-MAIN-2020-10", "warc_metadata": { "Content-Length": "47723", "Content-Type": "application/http; msgtype=response", "WARC-Block-Digest": "sha1:ZT6Z6F7VS6U5DD4QHLERJJOVK2ASMYWR", "WARC-Concurrent-To": "<urn:uuid:c53ba308-b750-45e7-9ffa-ba839afaf71b>", "WARC-Date": "2020-02-22T16:56:51Z", "WARC-IP-Address": "23.92.16.172", "WARC-Identified-Payload-Type": "text/html", "WARC-Payload-Digest": "sha1:WEDWUMZJVAFUSE75LJCDH7ZZ3HUDRKGK", "WARC-Record-ID": "<urn:uuid:c00e5f76-22fe-4ffe-9324-071cf9c389e3>", "WARC-Target-URI": "http://twinery.org/wiki/harlowe:lambda?rev=1555401955", "WARC-Truncated": null, "WARC-Type": "response", "WARC-Warcinfo-ID": "<urn:uuid:67d1556e-0262-4f8b-9f4b-a4b0f99b6853>" }, "warc_info": "isPartOf: CC-MAIN-2020-10\r\npublisher: Common Crawl\r\ndescription: Wide crawl of the web for February 2020\r\noperator: Common Crawl Admin ([email protected])\r\nhostname: ip-10-67-67-15.ec2.internal\r\nsoftware: Apache Nutch 1.16 (modified, https://github.com/commoncrawl/nutch/)\r\nrobots: checked via crawler-commons 1.1-SNAPSHOT (https://github.com/crawler-commons/crawler-commons)\r\nformat: WARC File Format 1.1\r\nconformsTo: http://iipc.github.io/warc-specifications/specifications/warc-format/warc-1.1/" }
{ "line_start_idx": [ 0, 11, 12, 23, 24, 25, 33, 34, 49, 50, 57, 58, 65, 66, 72, 73, 78, 79, 87, 88, 99, 100, 107, 108, 115, 116, 123, 124, 139, 140, 154, 155, 166, 167, 172, 173, 179, 180, 187, 188, 197, 198, 205, 206, 225, 226, 233, 234, 242, 243, 247, 248, 259, 260, 274, 275, 292, 293, 314, 315, 330, 331, 372, 373, 374, 386, 387, 779, 780, 959, 960, 996, 997, 1194, 1378, 1845, 2283, 2574, 2575, 3099, 3100, 3588, 3589, 3903, 3904 ], "line_end_idx": [ 11, 12, 23, 24, 25, 33, 34, 49, 50, 57, 58, 65, 66, 72, 73, 78, 79, 87, 88, 99, 100, 107, 108, 115, 116, 123, 124, 139, 140, 154, 155, 166, 167, 172, 173, 179, 180, 187, 188, 197, 198, 205, 206, 225, 226, 233, 234, 242, 243, 247, 248, 259, 260, 274, 275, 292, 293, 314, 315, 330, 331, 372, 373, 374, 386, 387, 779, 780, 959, 960, 996, 997, 1194, 1378, 1845, 2283, 2574, 2575, 3099, 3100, 3588, 3589, 3903, 3904, 3975 ] }
{ "red_pajama_v2": { "ccnet_original_length": 3975, "ccnet_original_nlines": 84, "rps_doc_curly_bracket": 0, "rps_doc_ldnoobw_words": 0, "rps_doc_lorem_ipsum": 0, "rps_doc_stop_word_fraction": 0.38307350873947144, "rps_doc_ut1_blacklist": 0, "rps_doc_frac_all_caps_words": 0.003340760013088584, "rps_doc_frac_lines_end_with_ellipsis": 0, "rps_doc_frac_no_alph_words": 0.2572382986545563, "rps_doc_frac_unique_words": 0.412202388048172, "rps_doc_mean_word_length": 4.464285850524902, "rps_doc_num_sentences": 32, "rps_doc_symbol_to_word_ratio": 0.0022271699272096157, "rps_doc_unigram_entropy": 5.0921735763549805, "rps_doc_word_count": 672, "rps_doc_frac_chars_dupe_10grams": 0, "rps_doc_frac_chars_dupe_5grams": 0.027333330363035202, "rps_doc_frac_chars_dupe_6grams": 0, "rps_doc_frac_chars_dupe_7grams": 0, "rps_doc_frac_chars_dupe_8grams": 0, "rps_doc_frac_chars_dupe_9grams": 0, "rps_doc_frac_chars_top_2gram": 0.017999999225139618, "rps_doc_frac_chars_top_3gram": 0.012000000104308128, "rps_doc_frac_chars_top_4gram": 0.01600000075995922, "rps_doc_books_importance": -335.0445556640625, "rps_doc_books_importance_length_correction": -335.0445556640625, "rps_doc_openwebtext_importance": -205.7473602294922, "rps_doc_openwebtext_importance_length_correction": -205.7473602294922, "rps_doc_wikipedia_importance": -166.1678466796875, "rps_doc_wikipedia_importance_length_correction": -166.1678466796875 }, "fasttext": { "dclm": 0.4177267551422119, "english": 0.9008737206459045, "fineweb_edu_approx": 2.904757499694824, "eai_general_math": 0.9124740958213806, "eai_open_web_math": 0.44682157039642334, "eai_web_code": 0.8997927904129028 } }
{ "free_decimal_correspondence": { "primary": { "code": "005.1", "labels": { "level_1": "General works, books and libraries, information sciences", "level_2": "", "level_3": "Computer programming" } }, "secondary": { "code": "794.8", "labels": { "level_1": "Arts", "level_2": "Amusements and Recreation", "level_3": "" } } }, "bloom_cognitive_process": { "primary": { "code": "2", "label": "Understand" }, "secondary": { "code": "3", "label": "Apply" } }, "bloom_knowledge_domain": { "primary": { "code": "2", "label": "Conceptual" }, "secondary": { "code": "3", "label": "Procedural" } }, "document_type_v1": { "primary": { "code": "3", "label": "Reference/Encyclopedic/Educational" }, "secondary": { "code": "4", "label": "Code/Software" } }, "extraction_artifacts": { "primary": { "code": "3", "label": "Irrelevant Content" }, "secondary": { "code": "0", "label": "No Artifacts" } }, "missing_content": { "primary": { "code": "0", "label": "No missing content" }, "secondary": { "code": "4", "label": "Missing Images or Figures" } }, "document_type_v2": { "primary": { "code": "8", "label": "Documentation" }, "secondary": { "code": "23", "label": "Tutorial" } }, "reasoning_depth": { "primary": { "code": "3", "label": "Intermediate Reasoning" }, "secondary": { "code": "2", "label": "Basic Reasoning" } }, "technical_correctness": { "primary": { "code": "4", "label": "Highly Correct" }, "secondary": { "code": "3", "label": "Mostly Correct" } }, "education_level": { "primary": { "code": "3", "label": "Undergraduate Level" }, "secondary": { "code": "2", "label": "High School Level" } } }
672f1e42c33a7f9846924a2431ea77df
4,134,920,760,248,546,300
Sunday December 8, 2013 Posts by kaitlyn Total # Posts: 191 college How do i calculate the number of equivalents of h2c4h4o4 titrated? Math Kay is making beaded belts for her friends from 6.5 meters of cord.On belt uses 0.625 meters of cord.How many belts can she make? Chemistry the hydrated salt is overheated and the anhydrous salt thermally decompose, one product being a gas; will the reported value of x be too high,low on the unaffected? explain. physics By how much does the speed of a vehicle moving in a straight line change each second when it is accelerating at 4 m/s2? Physics A ball is thrown upward from the ground with an initial speed of 26.8 m/s; at the same instant, a ball is dropped from a building 15.9 m high. After how long will the balls be at the same height? algebra write the inequality statement. The range of shoe sizes in geometry class is size 5 to size 14. really confused. Math i need help with diviging decimals. 7th grade I am trying to unscramble ecrfenine in my science homework and cannot get an answer. My mom has read the whole chapter as well with me and we cannot find it. It has to do with scientific method. Math 10 divied by 16 = 8th grade when two or more substances combine to make a complex compound,the process is called a......WHAT?? Pages: <<Prev | 5 | 6 | 7 | 8 | 9 | 10 | 11 | 12 | 13 | 14 | 15 | 16 | 17 | 18 | 19 | Next>> Search Members
{ "url": "http://www.jiskha.com/members/profile/posts.cgi?name=kaitlyn&page=13", "source_domain": "www.jiskha.com", "snapshot_id": "crawl=CC-MAIN-2013-48", "warc_metadata": { "Content-Length": "8866", "Content-Type": "application/http; msgtype=response", "WARC-Block-Digest": "sha1:U4V3S4VIPIMERUXQPFMPDLSWCTOYLG7P", "WARC-Concurrent-To": "<urn:uuid:b31aed34-7ef7-4dbf-982e-4a9e2ffaf5ed>", "WARC-Date": "2013-12-08T08:54:27Z", "WARC-IP-Address": "69.16.226.94", "WARC-Identified-Payload-Type": null, "WARC-Payload-Digest": "sha1:NGUEIPLUMQ7JJYD5SPOTQPVBNOPMBD43", "WARC-Record-ID": "<urn:uuid:c147315d-c1f0-4b33-8cb2-b455d5c51ad9>", "WARC-Target-URI": "http://www.jiskha.com/members/profile/posts.cgi?name=kaitlyn&page=13", "WARC-Truncated": "length", "WARC-Type": "response", "WARC-Warcinfo-ID": "<urn:uuid:3a704d84-a9de-49af-87ea-c49deee647d8>" }, "warc_info": "robots: classic\r\nhostname: ip-10-33-133-15.ec2.internal\r\nsoftware: Nutch 1.6 (CC)/CC WarcExport 1.0\r\nisPartOf: CC-MAIN-2013-48\r\noperator: CommonCrawl Admin\r\ndescription: Wide crawl of the web with URLs provided by Blekko for Winter 2013\r\npublisher: CommonCrawl\r\nformat: WARC File Format 1.0\r\nconformsTo: http://bibnum.bnf.fr/WARC/WARC_ISO_28500_version1_latestdraft.pdf" }
{ "line_start_idx": [ 0, 7, 24, 25, 42, 43, 44, 63, 64, 72, 139, 140, 145, 275, 276, 286, 460, 461, 469, 589, 590, 598, 794, 795, 803, 916, 917, 922, 958, 959, 969, 1164, 1165, 1170, 1188, 1189, 1199, 1298, 1299, 1392, 1393, 1400 ], "line_end_idx": [ 7, 24, 25, 42, 43, 44, 63, 64, 72, 139, 140, 145, 275, 276, 286, 460, 461, 469, 589, 590, 598, 794, 795, 803, 916, 917, 922, 958, 959, 969, 1164, 1165, 1170, 1188, 1189, 1199, 1298, 1299, 1392, 1393, 1400, 1407 ] }
{ "red_pajama_v2": { "ccnet_original_length": 1407, "ccnet_original_nlines": 41, "rps_doc_curly_bracket": 0, "rps_doc_ldnoobw_words": 0, "rps_doc_lorem_ipsum": 0, "rps_doc_stop_word_fraction": 0.35962143540382385, "rps_doc_ut1_blacklist": 0, "rps_doc_frac_all_caps_words": 0.00946372002363205, "rps_doc_frac_lines_end_with_ellipsis": 0, "rps_doc_frac_no_alph_words": 0.26498422026634216, "rps_doc_frac_unique_words": 0.687747061252594, "rps_doc_mean_word_length": 4.193675994873047, "rps_doc_num_sentences": 23, "rps_doc_symbol_to_word_ratio": 0.00946372002363205, "rps_doc_unigram_entropy": 4.935576915740967, "rps_doc_word_count": 253, "rps_doc_frac_chars_dupe_10grams": 0, "rps_doc_frac_chars_dupe_5grams": 0, "rps_doc_frac_chars_dupe_6grams": 0, "rps_doc_frac_chars_dupe_7grams": 0, "rps_doc_frac_chars_dupe_8grams": 0, "rps_doc_frac_chars_dupe_9grams": 0, "rps_doc_frac_chars_top_2gram": 0.015080110169947147, "rps_doc_frac_chars_top_3gram": 0.013195100240409374, "rps_doc_frac_chars_top_4gram": 0, "rps_doc_books_importance": -121.64389038085938, "rps_doc_books_importance_length_correction": -121.4222640991211, "rps_doc_openwebtext_importance": -59.93241882324219, "rps_doc_openwebtext_importance_length_correction": -59.93241882324219, "rps_doc_wikipedia_importance": -48.46835708618164, "rps_doc_wikipedia_importance_length_correction": -44.2458610534668 }, "fasttext": { "dclm": 0.24355214834213257, "english": 0.9504624605178833, "fineweb_edu_approx": 2.245277166366577, "eai_general_math": 0.9049364924430847, "eai_open_web_math": 0.435158371925354, "eai_web_code": 0.004095079842954874 } }
{ "free_decimal_correspondence": { "primary": { "code": "510", "labels": { "level_1": "Science and Natural history", "level_2": "Mathematics", "level_3": "" } }, "secondary": { "code": "530", "labels": { "level_1": "Science and Natural history", "level_2": "Physics", "level_3": "" } } }, "bloom_cognitive_process": { "primary": { "code": "3", "label": "Apply" }, "secondary": { "code": "2", "label": "Understand" } }, "bloom_knowledge_domain": { "primary": { "code": "3", "label": "Procedural" }, "secondary": { "code": "2", "label": "Conceptual" } }, "document_type_v1": { "primary": { "code": "5", "label": "Social/Forum" }, "secondary": { "code": "3", "label": "Reference/Encyclopedic/Educational" } }, "extraction_artifacts": { "primary": { "code": "0", "label": "No Artifacts" }, "secondary": { "code": "3", "label": "Irrelevant Content" } }, "missing_content": { "primary": { "code": "0", "label": "No missing content" }, "secondary": { "code": "-1", "label": "Abstain" } }, "document_type_v2": { "primary": { "code": "18", "label": "Q&A Forum" }, "secondary": { "code": "6", "label": "Content Listing" } }, "reasoning_depth": { "primary": { "code": "2", "label": "Basic Reasoning" }, "secondary": { "code": "3", "label": "Intermediate Reasoning" } }, "technical_correctness": { "primary": { "code": "6", "label": "Not Applicable/Indeterminate" }, "secondary": { "code": "2", "label": "Partially Correct" } }, "education_level": { "primary": { "code": "2", "label": "High School Level" }, "secondary": { "code": "3", "label": "Undergraduate Level" } } }
672f1e42c33a7f9846924a2431ea77df
1,303,952,610,820,477,400
Take the 2-minute tour × Super User is a question and answer site for computer enthusiasts and power users. It's 100% free, no registration required. There is quite some posts about this, but I can't find my exact situation... We have a small office network, which gets its internet through a 3com officeconnect 8 3C16794. On this switch are another 3com switch going to 5 computers, which are not relevant for the question, a printer and then we have to connect 7 computers to the remaining 6 ports. So on one of those ports I have a level one FBR-1418X router for two pc's. The cable goes into the wan port of the router. These 7 pc's are in the same workgroup, but computers on the router cannot find the others and the other way round. The switch dispatches ip adresses in 10.10.10.X range and the router in the 198.168.123.X range with the same subnet of 255.255.255.0. I have come so far that now I can ping computers in the 10.10.10.x range from the two on the router. The other way round does not work. Non of the computers "find" computers on the other lan. (in windows look for computers in the workgroup) They find all computers on the same lan in both cases. The router has a setting for ip table, but I don't know if putting anything there will be of any help, and of the officeConnect switch we don't find the password to log into it's web interface... Rebooting does not resolve... Any hints greately appreciated Update: On further thought, it would seem logical that if I could add the ip adress range of the router to the routing table of the switch then all computers should be able to find eachother, and that might resolve the issue... The problem is the default 3com passwords I find on the net don't work for logging into the switch and obviously nobody here has a clue of the password. So if anyone knows the default password for this model, might help. Does this make sense? share|improve this question   "switch dispatches ip address.." my understanding is that switches do not "dispatch" (assign) ip addresses. Are you sure that you don't have two routers setup, thus cuasing connection issues? –  KronoS Oct 18 '10 at 16:24   Why don't you arrange for everybody to be on the same segment? –  harrymc Oct 18 '10 at 16:29   I did a check and between the switch and the modem there is indeed another device called Cisco Soho 71. Possibly that is giving the ip addresses. –  ufotds Oct 18 '10 at 16:33 add comment 2 Answers up vote 1 down vote accepted First thoughts: Putting anything into the IP table of the router will not help; your problem is in the other direction. Your computers are in separate subnets, and not enough IP address translation is going on. Actually, none should need to be; there's no call for a separate subnet here - quite the opposite, you want everything on the same subnet. For a really quick fix to try: Stick the cable to the 3com switch in one of the other LAN ports of the FBR-1418X, not the WAN, as if the 3com was another PC. If you have to use this kind of router as just a switch, that's how to do it. By using the WAN port you're telling it to act as a router and run a local network - it's performing DHCP and assigning local addresses itself, which is really not what you want. Summary: A router is overkill for getting four computers into two LAN ports; a switch is the correct solution. Daisy-chaining switches is much less error-prone than daisy-chaining routers. (Of course most modern routers are switches, but your problem is that yours is acting as a router, not just a switch. ) (Also: A lot of OfficeConnects have the default password 'admin', so try that - if you don't know that there's been another password set. Also try 'password' or 'manager'.) share|improve this answer   ok, I tried that before and it didn't work, but now rebooted and reset the router and now it all works. Thanks alot, both of you who knew the answer. –  ufotds Oct 18 '10 at 16:54 add comment Ok - getting confused by a few things (it's me, not you (headache!)! If I miss anything, please say and I will re-read thoroughly) The router is simply being used to add additional ports, but it is also acting as a full router - providing NAT, DHCP etc. If the router has 3 or more standard ports, the easiest thing you can do is disable DHCP and then plug the two computers plus the uplink to the existing switch straight in to regular sockets. This will allow all computers to communicate just fine and they will appear to be on the same network. When you start using the WAN socket, you are basically using NAT to bridge the two networks together - you can use firewall / incoming rules to help, but it can just be a nightmare... You can always disable NAT, but what I wrote above is by far the easiest solution. Only negative is, remember to change the router's IP to something on the existing range or you will have to reset or manually change a machine address to be in its range in order to connect.... But once it is set, you shouldn't need to connect. Lastly, I would like to say that obviously, if you are doing anything data/network intensive you may want to upgrade your infrastructure as this link will obviously be a weak point in the network. share|improve this answer add comment Your Answer   discard By posting your answer, you agree to the privacy policy and terms of service. Not the answer you're looking for? Browse other questions tagged or ask your own question.
{ "url": "http://superuser.com/questions/200711/windows-home-network-doesnt-work-with-switch-and-router", "source_domain": "superuser.com", "snapshot_id": "crawl=CC-MAIN-2014-10", "warc_metadata": { "Content-Length": "71650", "Content-Type": "application/http; msgtype=response", "WARC-Block-Digest": "sha1:LEPUU4JFZL3NKOL7RRRRWB7WSSZJUXYG", "WARC-Concurrent-To": "<urn:uuid:47f9c222-25a4-4b7f-a7c5-dd8dcce75dc4>", "WARC-Date": "2014-03-08T23:27:22Z", "WARC-IP-Address": "198.252.206.140", "WARC-Identified-Payload-Type": null, "WARC-Payload-Digest": "sha1:DW5GUM7ZIAYNYCUYMXDEOYGWCQFVQDEK", "WARC-Record-ID": "<urn:uuid:125d6051-3924-40af-8ece-4aa35150dada>", "WARC-Target-URI": "http://superuser.com/questions/200711/windows-home-network-doesnt-work-with-switch-and-router", "WARC-Truncated": null, "WARC-Type": "response", "WARC-Warcinfo-ID": "<urn:uuid:7c1713cf-f79b-490d-a50c-86ca05a25cf4>" }, "warc_info": "robots: classic\r\nhostname: ip-10-183-142-35.ec2.internal\r\nsoftware: Nutch 1.6 (CC)/CC WarcExport 1.0\r\nisPartOf: CC-MAIN-2014-10\r\noperator: CommonCrawl Admin\r\ndescription: Wide crawl of the web with URLs provided by Blekko for March 2014\r\npublisher: CommonCrawl\r\nformat: WARC File Format 1.0\r\nconformsTo: http://bibnum.bnf.fr/WARC/WARC_ISO_28500_version1_latestdraft.pdf" }
{ "line_start_idx": [ 0, 25, 150, 151, 228, 229, 626, 627, 743, 744, 879, 880, 1176, 1177, 1373, 1374, 1404, 1405, 1436, 1437, 1908, 1909, 1937, 1939, 2161, 2163, 2257, 2259, 2435, 2447, 2448, 2458, 2459, 2488, 2489, 2839, 2840, 3255, 3256, 3565, 3566, 3739, 3740, 3766, 3768, 3948, 3960, 3961, 4092, 4093, 4216, 4217, 4409, 4410, 4513, 4514, 4781, 4782, 5027, 5028, 5225, 5226, 5252, 5264, 5265, 5277, 5278, 5280, 5288, 5289, 5367, 5368 ], "line_end_idx": [ 25, 150, 151, 228, 229, 626, 627, 743, 744, 879, 880, 1176, 1177, 1373, 1374, 1404, 1405, 1436, 1437, 1908, 1909, 1937, 1939, 2161, 2163, 2257, 2259, 2435, 2447, 2448, 2458, 2459, 2488, 2489, 2839, 2840, 3255, 3256, 3565, 3566, 3739, 3740, 3766, 3768, 3948, 3960, 3961, 4092, 4093, 4216, 4217, 4409, 4410, 4513, 4514, 4781, 4782, 5027, 5028, 5225, 5226, 5252, 5264, 5265, 5277, 5278, 5280, 5288, 5289, 5367, 5368, 5458 ] }
{ "red_pajama_v2": { "ccnet_original_length": 5458, "ccnet_original_nlines": 71, "rps_doc_curly_bracket": 0, "rps_doc_ldnoobw_words": 0, "rps_doc_lorem_ipsum": 0, "rps_doc_stop_word_fraction": 0.4613466262817383, "rps_doc_ut1_blacklist": 0, "rps_doc_frac_all_caps_words": 0.03075644001364708, "rps_doc_frac_lines_end_with_ellipsis": 0.0416666716337204, "rps_doc_frac_no_alph_words": 0.18287613987922668, "rps_doc_frac_unique_words": 0.3857142925262451, "rps_doc_mean_word_length": 4.319387912750244, "rps_doc_num_sentences": 62, "rps_doc_symbol_to_word_ratio": 0.004987529944628477, "rps_doc_unigram_entropy": 5.271448612213135, "rps_doc_word_count": 980, "rps_doc_frac_chars_dupe_10grams": 0, "rps_doc_frac_chars_dupe_5grams": 0.01370187010616064, "rps_doc_frac_chars_dupe_6grams": 0.007559650111943483, "rps_doc_frac_chars_dupe_7grams": 0, "rps_doc_frac_chars_dupe_8grams": 0, "rps_doc_frac_chars_dupe_9grams": 0, "rps_doc_frac_chars_top_2gram": 0.02126152068376541, "rps_doc_frac_chars_top_3gram": 0.008504609577357769, "rps_doc_frac_chars_top_4gram": 0.008504609577357769, "rps_doc_books_importance": -483.6148986816406, "rps_doc_books_importance_length_correction": -483.6148986816406, "rps_doc_openwebtext_importance": -284.0046081542969, "rps_doc_openwebtext_importance_length_correction": -284.0046081542969, "rps_doc_wikipedia_importance": -197.49630737304688, "rps_doc_wikipedia_importance_length_correction": -197.49630737304688 }, "fasttext": { "dclm": 0.5612674355506897, "english": 0.95351642370224, "fineweb_edu_approx": 1.681566596031189, "eai_general_math": 0.21804893016815186, "eai_open_web_math": 0.31057775020599365, "eai_web_code": 0.044434551149606705 } }
{ "free_decimal_correspondence": { "primary": { "code": "004.6", "labels": { "level_1": "General works, books and libraries, information sciences", "level_2": "", "level_3": "Computers and Computer science" } }, "secondary": { "code": "-1", "labels": { "level_1": "", "level_2": "", "level_3": "" } } }, "bloom_cognitive_process": { "primary": { "code": "3", "label": "Apply" }, "secondary": { "code": "4", "label": "Analyze" } }, "bloom_knowledge_domain": { "primary": { "code": "3", "label": "Procedural" }, "secondary": { "code": "2", "label": "Conceptual" } }, "document_type_v1": { "primary": { "code": "5", "label": "Social/Forum" }, "secondary": { "code": "-1", "label": "Abstain" } }, "extraction_artifacts": { "primary": { "code": "3", "label": "Irrelevant Content" }, "secondary": { "code": "0", "label": "No Artifacts" } }, "missing_content": { "primary": { "code": "0", "label": "No missing content" }, "secondary": { "code": "-1", "label": "Abstain" } }, "document_type_v2": { "primary": { "code": "18", "label": "Q&A Forum" }, "secondary": { "code": "21", "label": "Customer Support" } }, "reasoning_depth": { "primary": { "code": "3", "label": "Intermediate Reasoning" }, "secondary": { "code": "2", "label": "Basic Reasoning" } }, "technical_correctness": { "primary": { "code": "4", "label": "Highly Correct" }, "secondary": { "code": "3", "label": "Mostly Correct" } }, "education_level": { "primary": { "code": "3", "label": "Undergraduate Level" }, "secondary": { "code": "2", "label": "High School Level" } } }
672f1e42c33a7f9846924a2431ea77df
-180,436,473,043,223,200
Take the 2-minute tour × Stack Overflow is a question and answer site for professional and enthusiast programmers. It's 100% free. I want to inspect the request that will change window.location or whatever it may be using to change the url. I don't know where in the code it is changing the url so I can't put a breakpoint in there. How can I use Firebug to allow Ajax requests but prevent the url from changing? Or how can I otherwise inspect the request that will change the url? Thanks! share|improve this question 2 Answers 2 up vote 2 down vote accepted You can click the "persist" button on the net tab. (and other tabs) Was a real "oh duh" moment for me when I noticed it. share|improve this answer You can wrap the ajax request in a function and return false. Then you can click it and it wont take you away. For example, I have a function that looks like this $('.flag-commentflag a').click(function() { var url = $(this).attr('href'); $.get (url, function (data) { console.log(data); }); return false; }); Where .flag-commentflag is the class of the div surrounding the link. share|improve this answer      it's on a form, this is a good idea though, thanks! –  Lance Pollard Aug 1 '10 at 1:27 Your Answer   discard By posting your answer, you agree to the privacy policy and terms of service. Not the answer you're looking for? Browse other questions tagged or ask your own question.
{ "url": "http://stackoverflow.com/questions/3380393/how-to-prevent-page-change-when-using-firebug/3380403", "source_domain": "stackoverflow.com", "snapshot_id": "crawl=CC-MAIN-2015-32", "warc_metadata": { "Content-Length": "74866", "Content-Type": "application/http; msgtype=response", "WARC-Block-Digest": "sha1:EMSJ5B4LXIZJV3AVHEGERNSCUEZSYYUJ", "WARC-Concurrent-To": "<urn:uuid:8615a395-f7d2-471c-9f28-531ba40888e2>", "WARC-Date": "2015-07-29T18:45:24Z", "WARC-IP-Address": "104.16.24.251", "WARC-Identified-Payload-Type": null, "WARC-Payload-Digest": "sha1:MSFWB7CJMA5GGVAOP76GFAJKPCHHBSFD", "WARC-Record-ID": "<urn:uuid:f043b653-6f1b-469a-abe5-c1cdf7aa71d1>", "WARC-Target-URI": "http://stackoverflow.com/questions/3380393/how-to-prevent-page-change-when-using-firebug/3380403", "WARC-Truncated": "length", "WARC-Type": "response", "WARC-Warcinfo-ID": "<urn:uuid:bb96f80d-d4e0-4911-9720-92eb231e9e31>" }, "warc_info": "robots: classic\r\nhostname: ip-10-236-191-2.ec2.internal\r\nsoftware: Nutch 1.6 (CC)/CC WarcExport 1.0\r\nisPartOf: CC-MAIN-2015-32\r\noperator: CommonCrawl Admin\r\ndescription: Wide crawl of the web for August 2015\r\npublisher: CommonCrawl\r\nformat: WARC File Format 1.0\r\nconformsTo: http://bibnum.bnf.fr/WARC/WARC_ISO_28500_version1_latestdraft.pdf" }
{ "line_start_idx": [ 0, 25, 131, 132, 334, 335, 484, 485, 493, 494, 522, 523, 535, 536, 565, 566, 687, 688, 714, 715, 878, 879, 923, 959, 993, 1016, 1024, 1039, 1043, 1044, 1114, 1115, 1141, 1146, 1233, 1234, 1246, 1247, 1249, 1257, 1258, 1336, 1337 ], "line_end_idx": [ 25, 131, 132, 334, 335, 484, 485, 493, 494, 522, 523, 535, 536, 565, 566, 687, 688, 714, 715, 878, 879, 923, 959, 993, 1016, 1024, 1039, 1043, 1044, 1114, 1115, 1141, 1146, 1233, 1234, 1246, 1247, 1249, 1257, 1258, 1336, 1337, 1427 ] }
{ "red_pajama_v2": { "ccnet_original_length": 1427, "ccnet_original_nlines": 42, "rps_doc_curly_bracket": 0.002803080016747117, "rps_doc_ldnoobw_words": 0, "rps_doc_lorem_ipsum": 0, "rps_doc_stop_word_fraction": 0.3684210479259491, "rps_doc_ut1_blacklist": 0, "rps_doc_frac_all_caps_words": 0.021671829745173454, "rps_doc_frac_lines_end_with_ellipsis": 0, "rps_doc_frac_no_alph_words": 0.23219813406467438, "rps_doc_frac_unique_words": 0.5666666626930237, "rps_doc_mean_word_length": 4.420833110809326, "rps_doc_num_sentences": 23, "rps_doc_symbol_to_word_ratio": 0, "rps_doc_unigram_entropy": 4.608949184417725, "rps_doc_word_count": 240, "rps_doc_frac_chars_dupe_10grams": 0, "rps_doc_frac_chars_dupe_5grams": 0.05843544006347656, "rps_doc_frac_chars_dupe_6grams": 0.05843544006347656, "rps_doc_frac_chars_dupe_7grams": 0, "rps_doc_frac_chars_dupe_8grams": 0, "rps_doc_frac_chars_dupe_9grams": 0, "rps_doc_frac_chars_top_2gram": 0.022620169445872307, "rps_doc_frac_chars_top_3gram": 0.03204524144530296, "rps_doc_frac_chars_top_4gram": 0.03958529978990555, "rps_doc_books_importance": -142.34027099609375, "rps_doc_books_importance_length_correction": -140.7727508544922, "rps_doc_openwebtext_importance": -72.79228210449219, "rps_doc_openwebtext_importance_length_correction": -72.79228210449219, "rps_doc_wikipedia_importance": -57.80366134643555, "rps_doc_wikipedia_importance_length_correction": -51.44292449951172 }, "fasttext": { "dclm": 0.22237032651901245, "english": 0.8443328738212585, "fineweb_edu_approx": 1.2449324131011963, "eai_general_math": 0.000006790000043110922, "eai_open_web_math": 0.042974408715963364, "eai_web_code": -0.000005360000159271294 } }
{ "free_decimal_correspondence": { "primary": { "code": "005.1", "labels": { "level_1": "General works, books and libraries, information sciences", "level_2": "", "level_3": "Computer programming" } }, "secondary": { "code": "004.678", "labels": { "level_1": "General works, books and libraries, information sciences", "level_2": "", "level_3": "Computers and Computer science" } } }, "bloom_cognitive_process": { "primary": { "code": "3", "label": "Apply" }, "secondary": { "code": "2", "label": "Understand" } }, "bloom_knowledge_domain": { "primary": { "code": "3", "label": "Procedural" }, "secondary": { "code": "2", "label": "Conceptual" } }, "document_type_v1": { "primary": { "code": "5", "label": "Social/Forum" }, "secondary": { "code": "4", "label": "Code/Software" } }, "extraction_artifacts": { "primary": { "code": "3", "label": "Irrelevant Content" }, "secondary": { "code": "1", "label": "Leftover HTML" } }, "missing_content": { "primary": { "code": "0", "label": "No missing content" }, "secondary": { "code": "-1", "label": "Abstain" } }, "document_type_v2": { "primary": { "code": "18", "label": "Q&A Forum" }, "secondary": { "code": "8", "label": "Documentation" } }, "reasoning_depth": { "primary": { "code": "2", "label": "Basic Reasoning" }, "secondary": { "code": "3", "label": "Intermediate Reasoning" } }, "technical_correctness": { "primary": { "code": "3", "label": "Mostly Correct" }, "secondary": { "code": "4", "label": "Highly Correct" } }, "education_level": { "primary": { "code": "3", "label": "Undergraduate Level" }, "secondary": { "code": "4", "label": "Graduate/Expert Level" } } }
672f1e42c33a7f9846924a2431ea77df
8,834,890,914,666,907,000
Javatpoint Logo Javatpoint Logo FCFS with Overhead In the above Examples, we are assuming that all the processes are the CPU bound processes only. We were also neglecting the context switching time. However if the time taken by the scheduler in context switching is considered then the average waiting time of the system will be increased which also affects the efficiency of the system. Context Switching is always an overhead. The Following Example describeshow the efficiency will be affected if the context switching time is considered in the system. Example In the following Example, we are considering five processes P1, P2, P3, P4, P5 and P6. Their arrival time and Burst time are given below. Process ID Arrival Time Burst Time 1 0 3 2 1 2 3 2 1 4 3 4 5 4 5 6 5 2 If the context switching time of the system is 1 unit then the Gantt chart of the system will be prepared as follows. Given δ=1 unit; os FCFS with Overhead The system will take extra 1 unit of time (overhead) after the execution of every process to schedule the next process. Next TopicSJF Scheduling Youtube For Videos Join Our Youtube Channel: Join Now Help Others, Please Share facebook twitter pinterest Learn Latest Tutorials Preparation Trending Technologies B.Tech / MCA
{ "url": "https://www.javatpoint.com/os-fcfs-with-overhead", "source_domain": "www.javatpoint.com", "snapshot_id": "crawl=CC-MAIN-2021-21", "warc_metadata": { "Content-Length": "54395", "Content-Type": "application/http; msgtype=response", "WARC-Block-Digest": "sha1:HSIFHRAAIHBVFKEKL4MNCH4D4IFUSW3P", "WARC-Concurrent-To": "<urn:uuid:e2c17d58-9a4c-4c33-990d-3032cf47ea59>", "WARC-Date": "2021-05-08T16:02:50Z", "WARC-IP-Address": "194.169.80.121", "WARC-Identified-Payload-Type": "text/html", "WARC-Payload-Digest": "sha1:KPEWXFGZX7BJTQSETF7IO67P7KJOW327", "WARC-Record-ID": "<urn:uuid:cd604d74-870a-430a-acba-a8d8cbf5f900>", "WARC-Target-URI": "https://www.javatpoint.com/os-fcfs-with-overhead", "WARC-Truncated": null, "WARC-Type": "response", "WARC-Warcinfo-ID": "<urn:uuid:3fcf36ee-025d-4d78-be9e-c2be7cd85db0>" }, "warc_info": "isPartOf: CC-MAIN-2021-21\r\npublisher: Common Crawl\r\ndescription: Wide crawl of the web for May 2021\r\noperator: Common Crawl Admin ([email protected])\r\nhostname: ip-10-67-67-20.ec2.internal\r\nsoftware: Apache Nutch 1.18 (modified, https://github.com/commoncrawl/nutch/)\r\nrobots: checked via crawler-commons 1.2-SNAPSHOT (https://github.com/crawler-commons/crawler-commons)\r\nformat: WARC File Format 1.1\r\nconformsTo: https://iipc.github.io/warc-specifications/specifications/warc-format/warc-1.1/" }
{ "line_start_idx": [ 0, 16, 32, 33, 52, 53, 201, 202, 391, 392, 559, 560, 568, 569, 707, 708, 743, 749, 755, 761, 767, 773, 779, 780, 898, 899, 915, 916, 938, 939, 1059, 1060, 1061, 1062, 1087, 1088, 1089, 1090, 1091, 1145, 1146, 1172, 1173, 1200, 1201, 1224, 1225, 1226, 1238, 1239, 1240, 1262, 1263, 1264 ], "line_end_idx": [ 16, 32, 33, 52, 53, 201, 202, 391, 392, 559, 560, 568, 569, 707, 708, 743, 749, 755, 761, 767, 773, 779, 780, 898, 899, 915, 916, 938, 939, 1059, 1060, 1061, 1062, 1087, 1088, 1089, 1090, 1091, 1145, 1146, 1172, 1173, 1200, 1201, 1224, 1225, 1226, 1238, 1239, 1240, 1262, 1263, 1264, 1276 ] }
{ "red_pajama_v2": { "ccnet_original_length": 1276, "ccnet_original_nlines": 53, "rps_doc_curly_bracket": 0, "rps_doc_ldnoobw_words": 0, "rps_doc_lorem_ipsum": 0, "rps_doc_stop_word_fraction": 0.3181818127632141, "rps_doc_ut1_blacklist": 0, "rps_doc_frac_all_caps_words": 0.04958678036928177, "rps_doc_frac_lines_end_with_ellipsis": 0, "rps_doc_frac_no_alph_words": 0.1859504133462906, "rps_doc_frac_unique_words": 0.5115207433700562, "rps_doc_mean_word_length": 4.6497697830200195, "rps_doc_num_sentences": 11, "rps_doc_symbol_to_word_ratio": 0, "rps_doc_unigram_entropy": 4.342654705047607, "rps_doc_word_count": 217, "rps_doc_frac_chars_dupe_10grams": 0, "rps_doc_frac_chars_dupe_5grams": 0.0832507386803627, "rps_doc_frac_chars_dupe_6grams": 0, "rps_doc_frac_chars_dupe_7grams": 0, "rps_doc_frac_chars_dupe_8grams": 0, "rps_doc_frac_chars_dupe_9grams": 0, "rps_doc_frac_chars_top_2gram": 0.053518328815698624, "rps_doc_frac_chars_top_3gram": 0.04360752925276756, "rps_doc_frac_chars_top_4gram": 0.06838454306125641, "rps_doc_books_importance": -104.12747955322266, "rps_doc_books_importance_length_correction": -104.12747955322266, "rps_doc_openwebtext_importance": -56.84035873413086, "rps_doc_openwebtext_importance_length_correction": -56.84035873413086, "rps_doc_wikipedia_importance": -54.54378890991211, "rps_doc_wikipedia_importance_length_correction": -54.54378890991211 }, "fasttext": { "dclm": 0.10104119777679443, "english": 0.8882598876953125, "fineweb_edu_approx": 1.3011330366134644, "eai_general_math": 0.24864763021469116, "eai_open_web_math": 0.3695605993270874, "eai_web_code": 0.038654450327157974 } }
{ "free_decimal_correspondence": { "primary": { "code": "004.222", "labels": { "level_1": "General works, books and libraries, information sciences", "level_2": "", "level_3": "Computers and Computer science" } }, "secondary": { "code": "004.22", "labels": { "level_1": "General works, books and libraries, information sciences", "level_2": "", "level_3": "Computers and Computer science" } } }, "bloom_cognitive_process": { "primary": { "code": "2", "label": "Understand" }, "secondary": { "code": "3", "label": "Apply" } }, "bloom_knowledge_domain": { "primary": { "code": "2", "label": "Conceptual" }, "secondary": { "code": "3", "label": "Procedural" } }, "document_type_v1": { "primary": { "code": "3", "label": "Reference/Encyclopedic/Educational" }, "secondary": { "code": "-1", "label": "Abstain" } }, "extraction_artifacts": { "primary": { "code": "3", "label": "Irrelevant Content" }, "secondary": { "code": "0", "label": "No Artifacts" } }, "missing_content": { "primary": { "code": "4", "label": "Missing Images or Figures" }, "secondary": { "code": "0", "label": "No missing content" } }, "document_type_v2": { "primary": { "code": "23", "label": "Tutorial" }, "secondary": { "code": "10", "label": "Knowledge Article" } }, "reasoning_depth": { "primary": { "code": "2", "label": "Basic Reasoning" }, "secondary": { "code": "3", "label": "Intermediate Reasoning" } }, "technical_correctness": { "primary": { "code": "3", "label": "Mostly Correct" }, "secondary": { "code": "4", "label": "Highly Correct" } }, "education_level": { "primary": { "code": "3", "label": "Undergraduate Level" }, "secondary": { "code": "2", "label": "High School Level" } } }
672f1e42c33a7f9846924a2431ea77df
-3,397,508,823,589,768,000
Optimize wireless reception from a distance Ever need to optimize the reception at a specific point on a wireless network? If so, the following tip from MacOSXHints.com reader ALT147 could help. I recently had to connect to a wireless network operating from a point more than 100 meters away from the base station and in a metal shed—not the best environment for wireless communication! Using the following set-up, I was able to have someone else carry my MacBook Pro around the shed while I stayed near the wireless router and fiddled with its antennas. To do that, I connected to my MacBook Pro via ssh from a computer at the router, and ran the following command in Terminal: while x=1; do /System/Library/PrivateFrameworks/Apple80211.framework/Versions/Current/Resources/airport -I | grep CtlRSSI; sleep 0.5; done What this does is periodically display the received signal strength indicator (RSSI)—basically the power of the radio signal from the router. It returns a value in dBm. That's on a logarithmic scale, so an increase of 10 units means a factor of 10 increase in power, an increase of 20 means a factor of 100 increase in power, and so on. Watching this reading, I was able to position the router and antenna for maximum signal strength at various places in the shed. The airport -I command alone displays other useful information about your airport connection, like the network name, channel, authorization mode, and the noise on the signal. Shop ▾ arrow up Amazon Shop buttons are programmatically attached to all reviews, regardless of products' final review scores. Our parent company, IDG, receives advertisement revenue for shopping activity generated by the links. Because the buttons are attached programmatically, they should not be interpreted as editorial endorsements. recommended for you AT&T 3G MicroCell Read more » Subscribe to the Best of Macworld Newsletter Comments
{ "url": "http://www.macworld.com/article/1150738/optimizewirelessreception.html", "source_domain": "www.macworld.com", "snapshot_id": "crawl=CC-MAIN-2016-07", "warc_metadata": { "Content-Length": "103229", "Content-Type": "application/http; msgtype=response", "WARC-Block-Digest": "sha1:C2JIPJBMLEVTTY4SKTVCNDXNM26OW4X4", "WARC-Concurrent-To": "<urn:uuid:4ae5ba4f-4f78-4677-acd7-71cabb7cde3f>", "WARC-Date": "2016-02-10T15:42:06Z", "WARC-IP-Address": "52.71.159.23", "WARC-Identified-Payload-Type": null, "WARC-Payload-Digest": "sha1:ESLRET4RDF5YRRARHEQY53WP3TELS4E7", "WARC-Record-ID": "<urn:uuid:b6a94f26-0d19-474b-8d18-5b1f345d5784>", "WARC-Target-URI": "http://www.macworld.com/article/1150738/optimizewirelessreception.html", "WARC-Truncated": null, "WARC-Type": "response", "WARC-Warcinfo-ID": "<urn:uuid:095f362a-1026-4f8d-9949-a743804aa165>" }, "warc_info": "robots: classic\r\nhostname: ip-10-236-182-209.ec2.internal\r\nsoftware: Nutch 1.6 (CC)/CC WarcExport 1.0\r\nisPartOf: CC-MAIN-2016-07\r\noperator: CommonCrawl Admin\r\ndescription: Wide crawl of the web for February 2016\r\npublisher: CommonCrawl\r\nformat: WARC File Format 1.0\r\nconformsTo: http://bibnum.bnf.fr/WARC/WARC_ISO_28500_version1_latestdraft.pdf" }
{ "line_start_idx": [ 0, 44, 45, 196, 197, 389, 390, 558, 559, 683, 684, 823, 824, 966, 967, 1290, 1291, 1466, 1467, 1474, 1805, 1825, 1826, 1844, 1845, 1857, 1858, 1903, 1904 ], "line_end_idx": [ 44, 45, 196, 197, 389, 390, 558, 559, 683, 684, 823, 824, 966, 967, 1290, 1291, 1466, 1467, 1474, 1805, 1825, 1826, 1844, 1845, 1857, 1858, 1903, 1904, 1912 ] }
{ "red_pajama_v2": { "ccnet_original_length": 1912, "ccnet_original_nlines": 28, "rps_doc_curly_bracket": 0, "rps_doc_ldnoobw_words": 0, "rps_doc_lorem_ipsum": 0, "rps_doc_stop_word_fraction": 0.3854447305202484, "rps_doc_ut1_blacklist": 0, "rps_doc_frac_all_caps_words": 0.03504043072462082, "rps_doc_frac_lines_end_with_ellipsis": 0, "rps_doc_frac_no_alph_words": 0.17250673472881317, "rps_doc_frac_unique_words": 0.594684362411499, "rps_doc_mean_word_length": 5.132890224456787, "rps_doc_num_sentences": 16, "rps_doc_symbol_to_word_ratio": 0, "rps_doc_unigram_entropy": 4.832254886627197, "rps_doc_word_count": 301, "rps_doc_frac_chars_dupe_10grams": 0, "rps_doc_frac_chars_dupe_5grams": 0, "rps_doc_frac_chars_dupe_6grams": 0, "rps_doc_frac_chars_dupe_7grams": 0, "rps_doc_frac_chars_dupe_8grams": 0, "rps_doc_frac_chars_dupe_9grams": 0, "rps_doc_frac_chars_top_2gram": 0.009708739817142487, "rps_doc_frac_chars_top_3gram": 0.02071196958422661, "rps_doc_frac_chars_top_4gram": 0.012944979593157768, "rps_doc_books_importance": -180.14846801757812, "rps_doc_books_importance_length_correction": -180.1484375, "rps_doc_openwebtext_importance": -90.86450958251953, "rps_doc_openwebtext_importance_length_correction": -90.86450958251953, "rps_doc_wikipedia_importance": -77.37823486328125, "rps_doc_wikipedia_importance_length_correction": -77.37823486328125 }, "fasttext": { "dclm": 0.25348472595214844, "english": 0.9277593493461609, "fineweb_edu_approx": 1.8142865896224976, "eai_general_math": 0.4350232481956482, "eai_open_web_math": 0.1984947919845581, "eai_web_code": 0.08952593803405762 } }
{ "free_decimal_correspondence": { "primary": { "code": "004.6", "labels": { "level_1": "General works, books and libraries, information sciences", "level_2": "", "level_3": "Computers and Computer science" } }, "secondary": { "code": "621.3846", "labels": { "level_1": "Industrial arts, Technology, and Engineering", "level_2": "Engineering", "level_3": "Mechanical engineering and Machinery" } } }, "bloom_cognitive_process": { "primary": { "code": "3", "label": "Apply" }, "secondary": { "code": "2", "label": "Understand" } }, "bloom_knowledge_domain": { "primary": { "code": "3", "label": "Procedural" }, "secondary": { "code": "2", "label": "Conceptual" } }, "document_type_v1": { "primary": { "code": "3", "label": "Reference/Encyclopedic/Educational" }, "secondary": { "code": "-1", "label": "Abstain" } }, "extraction_artifacts": { "primary": { "code": "3", "label": "Irrelevant Content" }, "secondary": { "code": "-1", "label": "Abstain" } }, "missing_content": { "primary": { "code": "0", "label": "No missing content" }, "secondary": { "code": "-1", "label": "Abstain" } }, "document_type_v2": { "primary": { "code": "23", "label": "Tutorial" }, "secondary": { "code": "8", "label": "Documentation" } }, "reasoning_depth": { "primary": { "code": "2", "label": "Basic Reasoning" }, "secondary": { "code": "3", "label": "Intermediate Reasoning" } }, "technical_correctness": { "primary": { "code": "3", "label": "Mostly Correct" }, "secondary": { "code": "4", "label": "Highly Correct" } }, "education_level": { "primary": { "code": "2", "label": "High School Level" }, "secondary": { "code": "3", "label": "Undergraduate Level" } } }
672f1e42c33a7f9846924a2431ea77df
-7,961,331,115,347,374,000
Reliability Update 2 for the .NET Framework 4 Introduction Reliability Update 2 for the Microsoft .NET Framework 4 is available to fix some stability, reliability, and performance issues in the .NET Framework 4. This update includes the update packages that are listed in the "Update replacement information" section. This update package also resolves some issues that were not documented in a previously released Microsoft Knowledge Base (KB) article. For more information about these issues, see the "Issues that this update resolves" section. Resolution Update information The following files are available for download from the Microsoft Download Center: Download Download the Reliability Update 2 for the Microsoft .NET Framework 4 package now. For more information about how to download Microsoft support files, click the following article number to view the article in the Microsoft Knowledge Base: 119591 How to obtain Microsoft support files from online services Microsoft scanned this file for viruses. Microsoft used the most current virus-detection software that was available on the date that the file was posted. The file is stored on security-enhanced servers that help prevent any unauthorized changes to the file. Prerequisites To apply this update, you must have the .NET Framework 4 or the Microsoft .NET Framework 4 Client Profile installed. To install this update, you must have Windows Installer 3.1 or a later version installed. To obtain the latest version of Windows Installer, visit the following Microsoft website: Command-line switches for this update For more information about the various command-line switches that are supported by this update, visit the following Microsoft website: 227091 Command-Line Switches for the Microsoft Windows Installer Tool For more information about Windows Installer, visit the following MSDN website: Restart requirement After you apply this update, you must restart the computer if any affected files are locked or are being used. Update replacement information This update replaces the following hotfixes: 2628838 FIX: Memory leak when you use the FileSystemWatcher class in a .NET Framework 4.0-based application 2600088 A hotfix is available for the ASP.NET browser definition files in the Microsoft .NET Framework 4.0 Issues that this update resolves This update resolves the following issues. Click here to view or hide detailed information Issue 1 Consider the following scenario: • You use Windows Internet Explorer 10 to access a Microsoft ASP.NET-based webpage. • The webpage starts a postback operation. In this scenario, the postback operation fails, and you receive the following error message: Script Error encountered", "'__doPostBack' is undefined Note The webpage can start a postback operation in various ways. For example, the webpage can use a LinkButton control to start a postback operation. Issue 2 Consider the following scenario: • You create an ASP.NET-based webpage that has the MaintainScrollPositionOnPostBack attribute set to True. • You use Mozilla Firefox 4.0 or a later version of Mozilla Firefox to open the webpage. • The webpage starts a postback operation. In this scenario, the scroll position of the webpage is not maintained after the postback operation is complete. Note The third-party products that this article discusses are manufactured by companies that are independent of Microsoft. Microsoft makes no warranty, implied or otherwise, about the performance or reliability of these products. Issue 3 A Watson error occurs in the System.Web.Configuration class in a .NET Framework 4-based ASP.NET website because of a race condition in which the InitStandaloneConfig function is called two times by two different threads. When this issue occurs, the website crashes. Additionally, you receive the following access violation error message after the Watson error occurs: Exception Code - System.AccessViolationException Issue 4 Consider the following scenario: • A user sets up an Internet Information Services (IIS) user-mode caching policy to cache the responses for requests that contain a certain file name extension in IIS Manager output caching. • A request for a resource with the previously-configured extension is received by IIS Manager output caching. • A custom module sets a cookie on the response. Then, the response is cached by IIS Manager output caching. • Another request from a different user is received by IIS Manager output caching, and the original cached response together with the cookie is sent out to the different user. In this scenario, the response with the same cookie is sent out to different requestors unexpectedly. Issue 5 Symptoms When you use the FileSystemWatcher class in a .NET Framework 4-based application to watch for changes in a specified directory, a memory leak occurs. Cause This issue occurs because of an error in the FileSystemWatcher class. When a FileSystemWatcher object is created, the FileSystemWatcher class uses the GCHandle class to create a pinned garbage collector handle. The handle is 8 kilobytes (KB) and uses the byte[] data format. However, the FileSystemWatcher object never frees the garbage collector handle. Issue 6 Consider the following scenario: • You install and then enable the PowerPivot for Excel add-in application on a computer that has the Microsoft .NET Framework 3.5 Service Pack 1 (SP1) or the Microsoft .NET Framework 2.0 Service Pack 2 (SP2) installed. • You try to start Microsoft Excel 2010. In this scenario, the splash screen intermittently stays open longer than usual when you receive the following message in Excel 2010: Loading Add-ins (<x> of <y>) PowerPivot for Excel Note<x> is a placeholder that represents where in the loading sequence that PowerPivot is displayed among the Excel add-in applications. <y> is a placeholder that represents the total number of add-in applications in Excel. Issue 7 The servicing of .NET Framework 4.0 assemblies may cause a race condition. This behavior causes processes that are running managed code against the .NET Framework 4.0 common language runtime (CLR) to enter an inconsistent state. Additionally, the processes may stop unexpectedly. After you install this update, entries are added to the CLR unification list to make the list transitively closed. This prevents any related race conditions. File information The English version of this hotfix has the file attributes (or later file attributes) that are listed in the following table. The dates and times for these files are listed in Coordinated Universal Time (UTC). When you view the file information, it is converted to local time. To find the difference between UTC and local time, use the Time Zone tab in the Date and Time item in Control Panel. Click here to view or hide detailed information For all supported x86-based versions of the .NET Framework 4 for GDR service branch File nameFile versionFile sizeDateTime clr.dll4.0.30319.2616,727,42422-Nov-201106:26 mscordacwks.dll4.0.30319.2611,143,56822-Nov-201106:26 mscordbi.dll4.0.30319.261957,20022-Nov-201106:26 mscorlib.dll4.0.30319.2615,201,16822-Nov-201106:26 nlssorting.dll4.0.30319.26157,61622-Nov-201106:26 ServiceModelReg.exe4.0.30319.261192,79222-Nov-201106:29 SOS.dll4.0.30319.261518,40022-Nov-201106:26 System.dll4.0.30319.2613,512,07222-Nov-201106:27 aspnet_wp.exe4.0.30319.26133,55222-Nov-201105:21 blackberry.browserNot applicable2,39122-Nov-201105:02 firefox.browserNot applicable3,08222-Nov-201105:02 gateway.browserNot applicable5,63422-Nov-201105:02 ie.browserNot applicable4,28322-Nov-201105:02 System.Web.Extensions.dll4.0.30319.2611,863,46422-Nov-201105:21 System.Web.dll4.0.30319.2615,229,32822-Nov-201105:21 webengine.dll4.0.30319.26115,12022-Nov-201105:21 webengine4.dll4.0.30319.261496,40022-Nov-201105:21 For all supported x64-based versions of the .NET Framework 4 File nameFile versionFile sizeDateTime clr.dll4.0.30319.2619,793,28022-Nov-201107:55 clr.dll4.0.30319.2616,727,42422-Nov-201106:26 mscordacwks.dll4.0.30319.2611,515,79222-Nov-201107:55 mscordacwks.dll4.0.30319.2611,143,56822-Nov-201106:26 mscordbi.dll4.0.30319.2611,455,37622-Nov-201107:56 mscordbi.dll4.0.30319.261957,20022-Nov-201106:26 mscorlib.dll4.0.30319.2614,970,76822-Nov-201107:56 mscorlib.dll4.0.30319.2615,201,16822-Nov-201106:26 nlssorting.dll4.0.30319.26168,88022-Nov-201107:56 nlssorting.dll4.0.30319.26157,61622-Nov-201106:26 ServiceModelReg.exe4.0.30319.261261,91222-Nov-201107:57 ServiceModelReg.exe4.0.30319.261192,79222-Nov-201106:29 SOS.dll4.0.30319.261598,78422-Nov-201107:56 SOS.dll4.0.30319.261518,40022-Nov-201106:26 System.dll4.0.30319.2613,512,07222-Nov-201106:27 aspnet_wp.exe4.0.30319.26141,74422-Nov-201107:28 aspnet_wp.exe4.0.30319.26133,55222-Nov-201105:21 blackberry.browserNot applicable2,39122-Nov-201107:22 blackberry.browserNot applicable2,39122-Nov-201105:02 firefox.browserNot applicable3,08222-Nov-201107:22 firefox.browserNot applicable3,08222-Nov-201105:02 gateway.browserNot applicable5,63422-Nov-201107:22 gateway.browserNot applicable5,63422-Nov-201105:02 ie.browserNot applicable4,28322-Nov-201107:22 ie.browserNot applicable4,28322-Nov-201105:02 System.Web.Extensions.dll4.0.30319.2611,863,46422-Nov-201105:21 System.Web.dll4.0.30319.2615,198,60822-Nov-201107:28 System.Web.dll4.0.30319.2615,229,32822-Nov-201105:21 webengine.dll4.0.30319.26116,65622-Nov-201107:28 webengine.dll4.0.30319.26115,12022-Nov-201105:21 webengine4.dll4.0.30319.261721,68022-Nov-201107:28 webengine4.dll4.0.30319.261496,40022-Nov-201105:21 For all supported IA-64based versions of the .NET Framework 4 for GDR service branch File nameFile versionFile sizeDateTime clr.dll4.0.30319.26119,368,70422-Nov-201109:30 clr.dll4.0.30319.2616,727,42422-Nov-201106:26 mscordacwks.dll4.0.30319.2613,759,88822-Nov-201109:30 mscordacwks.dll4.0.30319.2611,143,56822-Nov-201106:26 mscordbi.dll4.0.30319.2613,296,52822-Nov-201109:30 mscordbi.dll4.0.30319.261957,20022-Nov-201106:26 mscorlib.dll4.0.30319.2614,469,52022-Nov-201109:31 mscorlib.dll4.0.30319.2615,201,16822-Nov-201106:26 nlssorting.dll4.0.30319.261145,16822-Nov-201109:31 nlssorting.dll4.0.30319.26157,61622-Nov-201106:26 ServiceModelReg.exe4.0.30319.261545,56022-Nov-201109:31 ServiceModelReg.exe4.0.30319.261192,79222-Nov-201106:29 SOS.dll4.0.30319.2611,204,48022-Nov-201109:31 SOS.dll4.0.30319.261518,40022-Nov-201106:26 System.dll4.0.30319.2613,512,07222-Nov-201106:27 aspnet_wp.exe4.0.30319.26181,16822-Nov-201109:02 aspnet_wp.exe4.0.30319.26133,55222-Nov-201105:21 blackberry.browserNot applicable2,39122-Nov-201108:58 blackberry.browserNot applicable2,39122-Nov-201105:02 firefox.browserNot applicable3,08222-Nov-201108:58 firefox.browserNot applicable3,08222-Nov-201105:02 gateway.browserNot applicable5,63422-Nov-201108:58 gateway.browserNot applicable5,63422-Nov-201105:02 ie.browserNot applicable4,28322-Nov-201108:58 ie.browserNot applicable4,28322-Nov-201105:02 System.Web.Extensions.dll4.0.30319.2611,863,46422-Nov-201105:21 System.Web.dll4.0.30319.2614,817,68022-Nov-201109:02 System.Web.dll4.0.30319.2615,229,32822-Nov-201105:21 webengine.dll4.0.30319.26126,38422-Nov-201109:02 webengine.dll4.0.30319.26115,12022-Nov-201105:21 webengine4.dll4.0.30319.2611,341,71222-Nov-201109:02 webengine4.dll4.0.30319.261496,40022-Nov-201105:21 For all supported x86-based versions of the .NET Framework 4 for LDR service branch File nameFile versionFile sizeDateTime clr.dll4.0.30319.5306,727,42421-Nov-201116:36 mscordacwks.dll4.0.30319.5301,143,56821-Nov-201116:37 mscordbi.dll4.0.30319.530956,68821-Nov-201116:37 mscorlib.dll4.0.30319.5305,202,19221-Nov-201116:37 nlssorting.dll4.0.30319.53057,10421-Nov-201116:37 ServiceModelReg.exe4.0.30319.530192,79221-Nov-201116:40 Setup.exe10.0.30319.53079,11221-Nov-201115:50 SetupEngine.dll10.0.30319.530810,25621-Nov-201115:50 SetupUi.dll10.0.30319.530296,71221-Nov-201115:50 SOS.dll4.0.30319.530517,88821-Nov-201116:37 System.dll4.0.30319.5303,512,58421-Nov-201116:39 aspnet_wp.exe4.0.30319.53033,55221-Nov-201115:52 blackberry.browserNot applicable2,39121-Nov-201115:36 firefox.browserNot applicable3,08221-Nov-201115:36 gateway.browserNot applicable5,63421-Nov-201115:36 ie.browserNot applicable4,28321-Nov-201115:36 System.Web.Extensions.dll4.0.30319.5301,863,46421-Nov-201115:52 System.Web.dll4.0.30319.5305,230,35221-Nov-201115:53 webengine.dll4.0.30319.53015,12021-Nov-201115:53 webengine4.dll4.0.30319.530496,40021-Nov-201115:53 For all supported x64-based versions of the .NET Framework 4 for LDR service branch File nameFile versionFile sizeDateTime clr.dll4.0.30319.5309,792,76821-Nov-201118:02 clr.dll4.0.30319.5306,727,42421-Nov-201116:36 mscordacwks.dll4.0.30319.5301,515,79221-Nov-201118:02 mscordacwks.dll4.0.30319.5301,143,56821-Nov-201116:37 mscordbi.dll4.0.30319.5301,455,37621-Nov-201118:02 mscordbi.dll4.0.30319.530956,68821-Nov-201116:37 mscorlib.dll4.0.30319.5304,971,79221-Nov-201118:03 mscorlib.dll4.0.30319.5305,202,19221-Nov-201116:37 nlssorting.dll4.0.30319.53068,36821-Nov-201118:03 nlssorting.dll4.0.30319.53057,10421-Nov-201116:37 ServiceModelReg.exe4.0.30319.530261,91221-Nov-201118:04 ServiceModelReg.exe4.0.30319.530192,79221-Nov-201116:40 Setup.exe10.0.30319.53079,11221-Nov-201115:50 SetupEngine.dll10.0.30319.530810,25621-Nov-201115:50 SetupUi.dll10.0.30319.530296,71221-Nov-201115:50 SOS.dll4.0.30319.530598,78421-Nov-201118:03 SOS.dll4.0.30319.530517,88821-Nov-201116:37 System.dll4.0.30319.5303,512,58421-Nov-201116:39 aspnet_wp.exe4.0.30319.53041,74421-Nov-201117:30 aspnet_wp.exe4.0.30319.53033,55221-Nov-201115:52 blackberry.browserNot applicable2,39121-Nov-201117:24 blackberry.browserNot applicable2,39121-Nov-201115:36 firefox.browserNot applicable3,08221-Nov-201117:24 firefox.browserNot applicable3,08221-Nov-201115:36 gateway.browserNot applicable5,63421-Nov-201117:24 gateway.browserNot applicable5,63421-Nov-201115:36 ie.browserNot applicable4,28321-Nov-201117:24 ie.browserNot applicable4,28321-Nov-201115:36 System.Web.Extensions.dll4.0.30319.5301,863,46421-Nov-201115:52 System.Web.dll4.0.30319.5305,199,12021-Nov-201117:30 System.Web.dll4.0.30319.5305,230,35221-Nov-201115:53 webengine.dll4.0.30319.53016,65621-Nov-201117:30 webengine.dll4.0.30319.53015,12021-Nov-201115:53 webengine4.dll4.0.30319.530721,68021-Nov-201117:30 webengine4.dll4.0.30319.530496,40021-Nov-201115:53 For all supported IA-64based versions of the .NET Framework 4 for LDR service branch File nameFile versionFile sizeDateTime clr.dll4.0.30319.53019,368,70421-Nov-201119:28 clr.dll4.0.30319.5306,727,42421-Nov-201116:36 mscordacwks.dll4.0.30319.5303,759,37621-Nov-201119:29 mscordacwks.dll4.0.30319.5301,143,56821-Nov-201116:37 mscordbi.dll4.0.30319.5303,296,52821-Nov-201119:29 mscordbi.dll4.0.30319.530956,68821-Nov-201116:37 mscorlib.dll4.0.30319.5304,470,54421-Nov-201119:30 mscorlib.dll4.0.30319.5305,202,19221-Nov-201116:37 nlssorting.dll4.0.30319.530144,14421-Nov-201119:30 nlssorting.dll4.0.30319.53057,10421-Nov-201116:37 ServiceModelReg.exe4.0.30319.530545,56021-Nov-201119:31 ServiceModelReg.exe4.0.30319.530192,79221-Nov-201116:40 Setup.exe10.0.30319.53079,11221-Nov-201115:50 SetupEngine.dll10.0.30319.530810,25621-Nov-201115:50 SetupUi.dll10.0.30319.530296,71221-Nov-201115:50 SOS.dll4.0.30319.5301,203,96821-Nov-201119:30 SOS.dll4.0.30319.530517,88821-Nov-201116:37 System.dll4.0.30319.5303,512,58421-Nov-201116:39 aspnet_wp.exe4.0.30319.53081,16821-Nov-201118:58 aspnet_wp.exe4.0.30319.53033,55221-Nov-201115:52 blackberry.browserNot applicable2,39121-Nov-201118:53 blackberry.browserNot applicable2,39121-Nov-201115:36 firefox.browserNot applicable3,08221-Nov-201118:53 firefox.browserNot applicable3,08221-Nov-201115:36 gateway.browserNot applicable5,63421-Nov-201118:53 gateway.browserNot applicable5,63421-Nov-201115:36 ie.browserNot applicable4,28321-Nov-201118:53 ie.browserNot applicable4,28321-Nov-201115:36 System.Web.Extensions.dll4.0.30319.5301,863,46421-Nov-201115:52 System.Web.dll4.0.30319.5304,818,70421-Nov-201118:59 System.Web.dll4.0.30319.5305,230,35221-Nov-201115:53 webengine.dll4.0.30319.53026,38421-Nov-201118:59 webengine.dll4.0.30319.53015,12021-Nov-201115:53 webengine4.dll4.0.30319.5301,341,71221-Nov-201118:59 webengine4.dll4.0.30319.530496,40021-Nov-201115:53
{ "url": "https://support.microsoft.com/en-us/help/2600217/reliability-update-2-for-the-net-framework-4/", "source_domain": "support.microsoft.com", "snapshot_id": "crawl=CC-MAIN-2018-47", "warc_metadata": { "Content-Length": "273655", "Content-Type": "application/http; msgtype=response", "WARC-Block-Digest": "sha1:PPQI6RFYO4QGGDY4O4WLFRO7NT2SLMIK", "WARC-Concurrent-To": "<urn:uuid:f5317f9f-9929-497e-8659-9d840d0fc5a4>", "WARC-Date": "2018-11-18T17:33:45Z", "WARC-IP-Address": "23.40.16.194", "WARC-Identified-Payload-Type": "text/html", "WARC-Payload-Digest": "sha1:VI22M5N7MQU6HNHW3DKTJPUJEILHUV5Z", "WARC-Record-ID": "<urn:uuid:387ffbc2-dd9e-4247-b56d-f45540f04638>", "WARC-Target-URI": "https://support.microsoft.com/en-us/help/2600217/reliability-update-2-for-the-net-framework-4/", "WARC-Truncated": null, "WARC-Type": "response", "WARC-Warcinfo-ID": "<urn:uuid:8b58bee5-9ee4-4ffc-a9e9-d572016422cd>" }, "warc_info": "isPartOf: CC-MAIN-2018-47\r\npublisher: Common Crawl\r\ndescription: Wide crawl of the web for November 2018\r\noperator: Common Crawl Admin ([email protected])\r\nhostname: ip-10-166-161-213.ec2.internal\r\nsoftware: Apache Nutch 1.15 (modified, https://github.com/commoncrawl/nutch/)\r\nrobots: checked via crawler-commons 0.11-SNAPSHOT (https://github.com/crawler-commons/crawler-commons)\r\nformat: WARC File Format 1.1\r\nconformsTo: http://iipc.github.io/warc-specifications/specifications/warc-format/warc-1.1/" }
{ "line_start_idx": [ 0, 46, 47, 48, 61, 62, 63, 216, 217, 323, 324, 552, 553, 564, 565, 566, 585, 586, 669, 670, 761, 762, 918, 984, 1243, 1244, 1258, 1259, 1376, 1377, 1557, 1558, 1596, 1597, 1598, 1733, 1803, 1804, 1884, 1885, 1905, 1906, 2017, 2018, 2049, 2050, 2095, 2203, 2310, 2311, 2344, 2345, 2388, 2389, 2437, 2438, 2446, 2447, 2480, 2566, 2611, 2704, 2760, 2761, 2762, 2912, 2913, 2921, 2922, 2955, 3064, 3155, 3200, 3313, 3314, 3544, 3545, 3553, 3554, 3922, 3971, 3972, 3980, 3981, 4014, 4207, 4320, 4431, 4609, 4711, 4712, 4720, 4721, 4730, 4731, 4881, 4882, 4888, 4889, 5244, 5245, 5253, 5254, 5287, 5508, 5551, 5685, 5735, 5736, 5737, 5961, 5962, 5970, 5971, 6409, 6410, 6427, 6428, 6822, 6823, 6871, 6955, 6994, 7040, 7094, 7143, 7194, 7244, 7300, 7344, 7393, 7442, 7496, 7547, 7598, 7644, 7708, 7761, 7810, 7861, 7922, 7961, 8007, 8053, 8107, 8161, 8212, 8261, 8312, 8363, 8413, 8463, 8519, 8575, 8619, 8663, 8712, 8761, 8810, 8864, 8918, 8969, 9020, 9071, 9122, 9168, 9214, 9278, 9331, 9384, 9433, 9482, 9533, 9584, 9669, 9708, 9755, 9801, 9855, 9909, 9960, 10009, 10060, 10111, 10162, 10212, 10268, 10324, 10370, 10414, 10463, 10512, 10561, 10615, 10669, 10720, 10771, 10822, 10873, 10919, 10965, 11029, 11082, 11135, 11184, 11233, 11286, 11337, 11421, 11460, 11506, 11560, 11609, 11660, 11710, 11766, 11812, 11865, 11914, 11958, 12007, 12056, 12110, 12161, 12212, 12258, 12322, 12375, 12424, 12475, 12559, 12598, 12644, 12690, 12744, 12798, 12849, 12898, 12949, 13000, 13050, 13100, 13156, 13212, 13258, 13311, 13360, 13404, 13448, 13497, 13546, 13595, 13649, 13703, 13754, 13805, 13856, 13907, 13953, 13999, 14063, 14116, 14169, 14218, 14267, 14318, 14369, 14454, 14493, 14540, 14586, 14640, 14694, 14745, 14794, 14845, 14896, 14947, 14997, 15053, 15109, 15155, 15208, 15257, 15303, 15347, 15396, 15445, 15494, 15548, 15602, 15653, 15704, 15755, 15806, 15852, 15898, 15962, 16015, 16068, 16117, 16166, 16219 ], "line_end_idx": [ 46, 47, 48, 61, 62, 63, 216, 217, 323, 324, 552, 553, 564, 565, 566, 585, 586, 669, 670, 761, 762, 918, 984, 1243, 1244, 1258, 1259, 1376, 1377, 1557, 1558, 1596, 1597, 1598, 1733, 1803, 1804, 1884, 1885, 1905, 1906, 2017, 2018, 2049, 2050, 2095, 2203, 2310, 2311, 2344, 2345, 2388, 2389, 2437, 2438, 2446, 2447, 2480, 2566, 2611, 2704, 2760, 2761, 2762, 2912, 2913, 2921, 2922, 2955, 3064, 3155, 3200, 3313, 3314, 3544, 3545, 3553, 3554, 3922, 3971, 3972, 3980, 3981, 4014, 4207, 4320, 4431, 4609, 4711, 4712, 4720, 4721, 4730, 4731, 4881, 4882, 4888, 4889, 5244, 5245, 5253, 5254, 5287, 5508, 5551, 5685, 5735, 5736, 5737, 5961, 5962, 5970, 5971, 6409, 6410, 6427, 6428, 6822, 6823, 6871, 6955, 6994, 7040, 7094, 7143, 7194, 7244, 7300, 7344, 7393, 7442, 7496, 7547, 7598, 7644, 7708, 7761, 7810, 7861, 7922, 7961, 8007, 8053, 8107, 8161, 8212, 8261, 8312, 8363, 8413, 8463, 8519, 8575, 8619, 8663, 8712, 8761, 8810, 8864, 8918, 8969, 9020, 9071, 9122, 9168, 9214, 9278, 9331, 9384, 9433, 9482, 9533, 9584, 9669, 9708, 9755, 9801, 9855, 9909, 9960, 10009, 10060, 10111, 10162, 10212, 10268, 10324, 10370, 10414, 10463, 10512, 10561, 10615, 10669, 10720, 10771, 10822, 10873, 10919, 10965, 11029, 11082, 11135, 11184, 11233, 11286, 11337, 11421, 11460, 11506, 11560, 11609, 11660, 11710, 11766, 11812, 11865, 11914, 11958, 12007, 12056, 12110, 12161, 12212, 12258, 12322, 12375, 12424, 12475, 12559, 12598, 12644, 12690, 12744, 12798, 12849, 12898, 12949, 13000, 13050, 13100, 13156, 13212, 13258, 13311, 13360, 13404, 13448, 13497, 13546, 13595, 13649, 13703, 13754, 13805, 13856, 13907, 13953, 13999, 14063, 14116, 14169, 14218, 14267, 14318, 14369, 14454, 14493, 14540, 14586, 14640, 14694, 14745, 14794, 14845, 14896, 14947, 14997, 15053, 15109, 15155, 15208, 15257, 15303, 15347, 15396, 15445, 15494, 15548, 15602, 15653, 15704, 15755, 15806, 15852, 15898, 15962, 16015, 16068, 16117, 16166, 16219, 16269 ] }
{ "red_pajama_v2": { "ccnet_original_length": 16269, "ccnet_original_nlines": 302, "rps_doc_curly_bracket": 0, "rps_doc_ldnoobw_words": 0, "rps_doc_lorem_ipsum": 0, "rps_doc_stop_word_fraction": 0.10487861931324005, "rps_doc_ut1_blacklist": 0, "rps_doc_frac_all_caps_words": 0.01531935017555952, "rps_doc_frac_lines_end_with_ellipsis": 0, "rps_doc_frac_no_alph_words": 0.5981616973876953, "rps_doc_frac_unique_words": 0.33822450041770935, "rps_doc_mean_word_length": 9.760821342468262, "rps_doc_num_sentences": 670, "rps_doc_symbol_to_word_ratio": 0, "rps_doc_unigram_entropy": 5.458103656768799, "rps_doc_word_count": 1363, "rps_doc_frac_chars_dupe_10grams": 0.058929648250341415, "rps_doc_frac_chars_dupe_5grams": 0.12620264291763306, "rps_doc_frac_chars_dupe_6grams": 0.09072458744049072, "rps_doc_frac_chars_dupe_7grams": 0.07711966335773468, "rps_doc_frac_chars_dupe_8grams": 0.07185809314250946, "rps_doc_frac_chars_dupe_9grams": 0.06584486365318298, "rps_doc_frac_chars_top_2gram": 0.01803969033062458, "rps_doc_frac_chars_top_3gram": 0.01172580011188984, "rps_doc_frac_chars_top_4gram": 0.010823809541761875, "rps_doc_books_importance": -1385.7608642578125, "rps_doc_books_importance_length_correction": -1385.7608642578125, "rps_doc_openwebtext_importance": -636.7028198242188, "rps_doc_openwebtext_importance_length_correction": -636.7028198242188, "rps_doc_wikipedia_importance": -313.50543212890625, "rps_doc_wikipedia_importance_length_correction": -313.50543212890625 }, "fasttext": { "dclm": 0.023391129449009895, "english": 0.7042748332023621, "fineweb_edu_approx": 1.9362026453018188, "eai_general_math": 0.08571624755859375, "eai_open_web_math": 0.1472790241241455, "eai_web_code": 0.026602569967508316 } }
{ "free_decimal_correspondence": { "primary": { "code": "005.1", "labels": { "level_1": "General works, books and libraries, information sciences", "level_2": "", "level_3": "Computer programming" } }, "secondary": { "code": "004.02", "labels": { "level_1": "General works, books and libraries, information sciences", "level_2": "", "level_3": "Computers and Computer science" } } }, "bloom_cognitive_process": { "primary": { "code": "2", "label": "Understand" }, "secondary": { "code": "3", "label": "Apply" } }, "bloom_knowledge_domain": { "primary": { "code": "2", "label": "Conceptual" }, "secondary": { "code": "3", "label": "Procedural" } }, "document_type_v1": { "primary": { "code": "3", "label": "Reference/Encyclopedic/Educational" }, "secondary": { "code": "4", "label": "Code/Software" } }, "extraction_artifacts": { "primary": { "code": "0", "label": "No Artifacts" }, "secondary": { "code": "3", "label": "Irrelevant Content" } }, "missing_content": { "primary": { "code": "0", "label": "No missing content" }, "secondary": { "code": "2", "label": "Click Here References" } }, "document_type_v2": { "primary": { "code": "8", "label": "Documentation" }, "secondary": { "code": "21", "label": "Customer Support" } }, "reasoning_depth": { "primary": { "code": "2", "label": "Basic Reasoning" }, "secondary": { "code": "3", "label": "Intermediate Reasoning" } }, "technical_correctness": { "primary": { "code": "4", "label": "Highly Correct" }, "secondary": { "code": "3", "label": "Mostly Correct" } }, "education_level": { "primary": { "code": "4", "label": "Graduate/Expert Level" }, "secondary": { "code": "3", "label": "Undergraduate Level" } } }
672f1e42c33a7f9846924a2431ea77df
-8,358,577,637,259,424,000
close تبلیغات در اینترنت خرید هاست روش کسب درامد از اینترنت به علت بروز بودن سایت از صفحات دیگر نیز دیدن کنید به سایت مرجع کسب درامد اینترنتی خوش آمدید پی تی سی (ptc) چیست؟ پی تی سی (ptc) چیست؟  سلام بازدید کننده های عزیز اول از همه بگم که در امد از اینترنت که داخل سایت هست اولا که نیاز به هیچ دانشی نداره و هیچ محدودیت سنی نداره همین الان  هزاران نفر مشغول به این کارن دوما  غیر قانونی نیستش و سوما نیاز به هیچ سرمایه گذاری نیس!!! واسه شروع میتونین به راحتی و فقط با یک خط اینترنت شروع به کار کنید و یک کارت عابر بانک برای واریز در امدتون نیاز دارید ... چون خیلی از افراد تو این چند سال ازم سوالاتی کردن که اشنایی ندارن و میخوان بیان تو این کار  سعی کردم این متن رو به طور ساده بگم تا کسی که میخواد شروع به کار کنه و هیچ اطلاعی نداره با خوندن این متن اشنا شه به در امد از اینترنت.... حالا 5 دقیقه وقت بذارید و بخونید و شروع به کار کنید برای خواندن  به ادامه مطلب بروید                                                                                            کانال تلگرامی ما:     کلیک کنید←                                                                                                            
{ "url": "http://ptclearning.rozblog.com/tag/%D8%B1%D9%88%D8%B4-%DA%A9%D8%B3%D8%A8-%D8%AF%D8%B1%D8%A7%D9%85%D8%AF-%D8%A7%D8%B2-%D8%A7%DB%8C%D9%86%D8%AA%D8%B1%D9%86%D8%AA", "source_domain": "ptclearning.rozblog.com", "snapshot_id": "crawl=CC-MAIN-2017-43", "warc_metadata": { "Content-Length": "31533", "Content-Type": "application/http; msgtype=response", "WARC-Block-Digest": "sha1:NJLAEZTC3VNO3WM3JBCPZ2JMXHKAYOQ3", "WARC-Concurrent-To": "<urn:uuid:e3131bf6-c1a9-4bc0-a237-6290dadca401>", "WARC-Date": "2017-10-20T23:19:40Z", "WARC-IP-Address": "79.127.127.68", "WARC-Identified-Payload-Type": "text/html", "WARC-Payload-Digest": "sha1:4MSAWBXRTVWNTROORBISOKUNDR4CIYCT", "WARC-Record-ID": "<urn:uuid:8420f189-24ea-4ae8-8149-5535ebaba4ab>", "WARC-Target-URI": "http://ptclearning.rozblog.com/tag/%D8%B1%D9%88%D8%B4-%DA%A9%D8%B3%D8%A8-%D8%AF%D8%B1%D8%A7%D9%85%D8%AF-%D8%A7%D8%B2-%D8%A7%DB%8C%D9%86%D8%AA%D8%B1%D9%86%D8%AA", "WARC-Truncated": null, "WARC-Type": "response", "WARC-Warcinfo-ID": "<urn:uuid:534800e8-542a-4f4a-8947-e2b9b4ba1329>" }, "warc_info": "robots: classic\r\nhostname: ip-10-178-166-219.ec2.internal\r\nsoftware: Nutch 1.6 (CC)\r\nisPartOf: CC-MAIN-2017-43\r\noperator: Common Crawl Admin\r\ndescription: Wide crawl of the web for October 2017\r\npublisher: Common Crawl\r\nformat: WARC File Format 1.0\r\nconformsTo: http://bibnum.bnf.fr/WARC/WARC_ISO_28500_version1_latestdraft.pdf" }
{ "line_start_idx": [ 0, 6, 25, 35, 60, 61, 111, 112, 154, 155, 176, 177, 198, 199, 227, 228, 562, 563, 793, 794, 846, 847, 971, 972 ], "line_end_idx": [ 6, 25, 35, 60, 61, 111, 112, 154, 155, 176, 177, 198, 199, 227, 228, 562, 563, 793, 794, 846, 847, 971, 972, 1112 ] }
{ "red_pajama_v2": { "ccnet_original_length": 1112, "ccnet_original_nlines": 23, "rps_doc_curly_bracket": 0, "rps_doc_ldnoobw_words": 0, "rps_doc_lorem_ipsum": 0, "rps_doc_stop_word_fraction": 0, "rps_doc_ut1_blacklist": 0, "rps_doc_frac_all_caps_words": 0, "rps_doc_frac_lines_end_with_ellipsis": 0.0833333283662796, "rps_doc_frac_no_alph_words": 0.9851484894752502, "rps_doc_frac_unique_words": 0.6387434601783752, "rps_doc_mean_word_length": 3.607329845428467, "rps_doc_num_sentences": 4, "rps_doc_symbol_to_word_ratio": 0.009900989942252636, "rps_doc_unigram_entropy": 4.539609909057617, "rps_doc_word_count": 191, "rps_doc_frac_chars_dupe_10grams": 0, "rps_doc_frac_chars_dupe_5grams": 0.04063861072063446, "rps_doc_frac_chars_dupe_6grams": 0, "rps_doc_frac_chars_dupe_7grams": 0, "rps_doc_frac_chars_dupe_8grams": 0, "rps_doc_frac_chars_dupe_9grams": 0, "rps_doc_frac_chars_top_2gram": 0.0391872301697731, "rps_doc_frac_chars_top_3gram": 0.0391872301697731, "rps_doc_frac_chars_top_4gram": 0.026124820113182068, "rps_doc_books_importance": -99.58673095703125, "rps_doc_books_importance_length_correction": -99.58673095703125, "rps_doc_openwebtext_importance": -54.853397369384766, "rps_doc_openwebtext_importance_length_correction": -47.279903411865234, "rps_doc_wikipedia_importance": -42.34223556518555, "rps_doc_wikipedia_importance_length_correction": -42.34223556518555 }, "fasttext": { "dclm": 0.032295580953359604, "english": 0, "fineweb_edu_approx": 2.2189393043518066, "eai_general_math": -0.000010009999641624745, "eai_open_web_math": 0.8822470307350159, "eai_web_code": 1.0000072717666626 } }
{ "free_decimal_correspondence": { "primary": { "code": "004.67", "labels": { "level_1": "General works, books and libraries, information sciences", "level_2": "", "level_3": "Computers and Computer science" } }, "secondary": { "code": "303.4833", "labels": { "level_1": "Social sciences", "level_2": "", "level_3": "Social sciences — Dictionaries" } } }, "bloom_cognitive_process": { "primary": { "code": "5", "label": "Evaluate" }, "secondary": { "code": "2", "label": "Understand" } }, "bloom_knowledge_domain": { "primary": { "code": "2", "label": "Conceptual" }, "secondary": { "code": "3", "label": "Procedural" } }, "document_type_v1": { "primary": { "code": "9", "label": "Personal/Misc" }, "secondary": { "code": "-1", "label": "Abstain" } }, "extraction_artifacts": { "primary": { "code": "3", "label": "Irrelevant Content" }, "secondary": { "code": "0", "label": "No Artifacts" } }, "missing_content": { "primary": { "code": "2", "label": "Click Here References" }, "secondary": { "code": "1", "label": "Truncated Snippets" } }, "document_type_v2": { "primary": { "code": "16", "label": "Personal Blog" }, "secondary": { "code": "10", "label": "Knowledge Article" } }, "reasoning_depth": { "primary": { "code": "2", "label": "Basic Reasoning" }, "secondary": { "code": "1", "label": "No Reasoning" } }, "technical_correctness": { "primary": { "code": "6", "label": "Not Applicable/Indeterminate" }, "secondary": { "code": "2", "label": "Partially Correct" } }, "education_level": { "primary": { "code": "1", "label": "General Audience" }, "secondary": { "code": "2", "label": "High School Level" } } }
672f1e42c33a7f9846924a2431ea77df
-4,805,077,652,035,624,000
시간 제한 메모리 제한 제출 정답 맞은 사람 정답 비율 2 초 128 MB 138 68 48 53.333% 문제 Some time the programmers have very strange ways to hide their passwords. See for example how Billy “Hacker” Geits hide his password. Billy chooses a string S composed of small Latin letters with length L. Then he makes all L-1 one-letter left cyclic shifts of the string and takes as a password one prefix of the lexicographically first of the obtained strings (including S). For example let consider the string alabala. The cyclic one-letter left shifts (including the initial string) are: alabala labalaa abalaal balaala alaalab laalaba aalabal and lexicographically first of them is the string aalabal. The first letter of this string is in position 6 in the initial string (the positions in the string are counted from 0). Write a program that for given string S finds the start position of the smallest lexicographically one-letter left cyclic shift of this string. If the smallest lexicographically left shift appears more than once then the program have to output the smallest initial position. 입력 Your program has to be ready to solve more than one test case. The first line of the input file will contains only the number T of the test cases. Each of the following T lines will describe one test case – first the length L of the string (5≤L≤100000) and then, separated by one space, the string S itself. 출력 The output file have to contain exactly T lines with a single number each – the initial position found by your program. 예제 입력 1 2 6 baabaa 7 alabala 예제 출력 1 1 6
{ "url": "https://www.acmicpc.net/problem/3789", "source_domain": "www.acmicpc.net", "snapshot_id": "crawl=CC-MAIN-2020-05", "warc_metadata": { "Content-Length": "29148", "Content-Type": "application/http; msgtype=response", "WARC-Block-Digest": "sha1:4PM75KKFUAK2WU3P4ATKISZETZRJTKUC", "WARC-Concurrent-To": "<urn:uuid:2665d4bb-a377-4111-8da2-1a3783595150>", "WARC-Date": "2020-01-24T03:17:23Z", "WARC-IP-Address": "13.230.81.61", "WARC-Identified-Payload-Type": "text/html", "WARC-Payload-Digest": "sha1:BDZEXSR7VOXCRRYD7WHAPCAG6RHBI3UK", "WARC-Record-ID": "<urn:uuid:5db7b95a-fb26-4609-860f-0e6ed17f546d>", "WARC-Target-URI": "https://www.acmicpc.net/problem/3789", "WARC-Truncated": null, "WARC-Type": "response", "WARC-Warcinfo-ID": "<urn:uuid:9e3ccc6b-d9af-4936-9435-c338c9007e09>" }, "warc_info": "isPartOf: CC-MAIN-2020-05\r\npublisher: Common Crawl\r\ndescription: Wide crawl of the web for January 2020\r\noperator: Common Crawl Admin ([email protected])\r\nhostname: ip-10-67-67-30.ec2.internal\r\nsoftware: Apache Nutch 1.16 (modified, https://github.com/commoncrawl/nutch/)\r\nrobots: checked via crawler-commons 1.1-SNAPSHOT (https://github.com/crawler-commons/crawler-commons)\r\nformat: WARC File Format 1.1\r\nconformsTo: http://iipc.github.io/warc-specifications/specifications/warc-format/warc-1.1/" }
{ "line_start_idx": [ 0, 31, 60, 61, 64, 65, 557, 558, 566, 574, 582, 590, 598, 606, 614, 615, 795, 796, 1071, 1072, 1075, 1076, 1384, 1385, 1388, 1389, 1509, 1510, 1518, 1519, 1522, 1532, 1542, 1543, 1551, 1552, 1554 ], "line_end_idx": [ 31, 60, 61, 64, 65, 557, 558, 566, 574, 582, 590, 598, 606, 614, 615, 795, 796, 1071, 1072, 1075, 1076, 1384, 1385, 1388, 1389, 1509, 1510, 1518, 1519, 1522, 1532, 1542, 1543, 1551, 1552, 1554, 1555 ] }
{ "red_pajama_v2": { "ccnet_original_length": 1555, "ccnet_original_nlines": 36, "rps_doc_curly_bracket": 0, "rps_doc_ldnoobw_words": 0, "rps_doc_lorem_ipsum": 0, "rps_doc_stop_word_fraction": 0.349056601524353, "rps_doc_ut1_blacklist": 0, "rps_doc_frac_all_caps_words": 0.03773584961891174, "rps_doc_frac_lines_end_with_ellipsis": 0, "rps_doc_frac_no_alph_words": 0.223270446062088, "rps_doc_frac_unique_words": 0.5053763389587402, "rps_doc_mean_word_length": 4.620071887969971, "rps_doc_num_sentences": 15, "rps_doc_symbol_to_word_ratio": 0, "rps_doc_unigram_entropy": 4.5692458152771, "rps_doc_word_count": 279, "rps_doc_frac_chars_dupe_10grams": 0, "rps_doc_frac_chars_dupe_5grams": 0, "rps_doc_frac_chars_dupe_6grams": 0, "rps_doc_frac_chars_dupe_7grams": 0, "rps_doc_frac_chars_dupe_8grams": 0, "rps_doc_frac_chars_dupe_9grams": 0, "rps_doc_frac_chars_top_2gram": 0.031031809747219086, "rps_doc_frac_chars_top_3gram": 0.02948022074997425, "rps_doc_frac_chars_top_4gram": 0, "rps_doc_books_importance": -111.03361511230469, "rps_doc_books_importance_length_correction": -98.71923065185547, "rps_doc_openwebtext_importance": -73.68265533447266, "rps_doc_openwebtext_importance_length_correction": -73.68265533447266, "rps_doc_wikipedia_importance": -58.79271697998047, "rps_doc_wikipedia_importance_length_correction": -45.140872955322266 }, "fasttext": { "dclm": 0.6777585744857788, "english": 0.8664999604225159, "fineweb_edu_approx": 3.0819361209869385, "eai_general_math": 0.9910460710525513, "eai_open_web_math": 0.13504564762115479, "eai_web_code": 0.7454504370689392 } }
{ "free_decimal_correspondence": { "primary": { "code": "005.1", "labels": { "level_1": "General works, books and libraries, information sciences", "level_2": "", "level_3": "Computer programming" } }, "secondary": { "code": "004.0151", "labels": { "level_1": "General works, books and libraries, information sciences", "level_2": "", "level_3": "Computers and Computer science" } } }, "bloom_cognitive_process": { "primary": { "code": "3", "label": "Apply" }, "secondary": { "code": "4", "label": "Analyze" } }, "bloom_knowledge_domain": { "primary": { "code": "3", "label": "Procedural" }, "secondary": { "code": "2", "label": "Conceptual" } }, "document_type_v1": { "primary": { "code": "4", "label": "Code/Software" }, "secondary": { "code": "-1", "label": "Abstain" } }, "extraction_artifacts": { "primary": { "code": "0", "label": "No Artifacts" }, "secondary": { "code": "3", "label": "Irrelevant Content" } }, "missing_content": { "primary": { "code": "0", "label": "No missing content" }, "secondary": { "code": "-1", "label": "Abstain" } }, "document_type_v2": { "primary": { "code": "8", "label": "Documentation" }, "secondary": { "code": "10", "label": "Knowledge Article" } }, "reasoning_depth": { "primary": { "code": "2", "label": "Basic Reasoning" }, "secondary": { "code": "3", "label": "Intermediate Reasoning" } }, "technical_correctness": { "primary": { "code": "4", "label": "Highly Correct" }, "secondary": { "code": "3", "label": "Mostly Correct" } }, "education_level": { "primary": { "code": "3", "label": "Undergraduate Level" }, "secondary": { "code": "2", "label": "High School Level" } } }
672f1e42c33a7f9846924a2431ea77df
-1,307,707,833,413,998,600
Free Software to Block Websites Awesome Free Software to Block Websites One point that will like to make is that this will be the shortest part of this guide. The reason is that if you want to do something simple and easy online without manually doing it then go the software route which is just that simple. I understand more thing anything that most people are not computer savvy so showing them how to change this or that setting when it comes to blocking websites isn’t something they want to doing. Most people like me and you prefer just plugging in a little information in a software like Window Blocker and allow it to do its job. This is the sole reason I’m writing this chapter first is for those people who just want to skip directly into blocking websites. If your one of those people then this section of the guide is for you.   Benefits of Using a Website Blocking Software 1. The first major reason to use a software is because of the flexibility which you can implement. The features a software has can make blocking websites a breeze instead of dong it the manual way. 2. Another great reason to use a software is that most of them have the ability for you to block websites at certain times. Now you can block websites up to certain times, hours or days. But doing it the manual way if you block a website its blocked until you undo the changes unlike using a software where it can be but on a timer. 3. and many more… Qustodio Blocker..Download It For FREE free software to block a website by Qustodio Super Great!! See You Made It Through Chapter 1! Lets Dive Into Chapter 2: How to Block Websites on Your Computer Take Me To Chapter 2 or Take Me To Introduction
{ "url": "http://www.colemanmg.com/tag/software-to-block-websites/", "source_domain": "www.colemanmg.com", "snapshot_id": "crawl=CC-MAIN-2017-26", "warc_metadata": { "Content-Length": "35468", "Content-Type": "application/http; msgtype=response", "WARC-Block-Digest": "sha1:OKUGWOUUBZS2EKRQ537PU6DSRVRHUIE3", "WARC-Concurrent-To": "<urn:uuid:a6506a3c-11a8-4332-bf76-b0ca4c808b39>", "WARC-Date": "2017-06-23T17:19:13Z", "WARC-IP-Address": "69.89.31.203", "WARC-Identified-Payload-Type": "text/html", "WARC-Payload-Digest": "sha1:7TTQI3JQQKNVN5KWELIIJSCL4FYYFWNK", "WARC-Record-ID": "<urn:uuid:b9625a67-9131-49ed-91f6-45ece2f90123>", "WARC-Target-URI": "http://www.colemanmg.com/tag/software-to-block-websites/", "WARC-Truncated": "length", "WARC-Type": "response", "WARC-Warcinfo-ID": "<urn:uuid:16b158b3-b0a1-4c29-87ad-c3e87cf6eda5>" }, "warc_info": "robots: classic\r\nhostname: ip-10-184-2-143.ec2.internal\r\nsoftware: Nutch 1.6 (CC)\r\nisPartOf: CC-MAIN-2017-26\r\noperator: Common Crawl Admin\r\ndescription: Wide crawl of the web for June 2017\r\npublisher: Common Crawl\r\nformat: WARC File Format 1.0\r\nconformsTo: http://bibnum.bnf.fr/WARC/WARC_ISO_28500_version1_latestdraft.pdf" }
{ "line_start_idx": [ 0, 32, 33, 73, 74, 506, 507, 843, 844, 846, 847, 893, 894, 1094, 1429, 1449, 1450, 1489, 1490, 1535, 1536, 1650, 1651, 1672, 1673, 1676, 1677 ], "line_end_idx": [ 32, 33, 73, 74, 506, 507, 843, 844, 846, 847, 893, 894, 1094, 1429, 1449, 1450, 1489, 1490, 1535, 1536, 1650, 1651, 1672, 1673, 1676, 1677, 1700 ] }
{ "red_pajama_v2": { "ccnet_original_length": 1700, "ccnet_original_nlines": 26, "rps_doc_curly_bracket": 0, "rps_doc_ldnoobw_words": 0, "rps_doc_lorem_ipsum": 0, "rps_doc_stop_word_fraction": 0.5044510364532471, "rps_doc_ut1_blacklist": 0, "rps_doc_frac_all_caps_words": 0.008902080357074738, "rps_doc_frac_lines_end_with_ellipsis": 0.037037041038274765, "rps_doc_frac_no_alph_words": 0.0830860510468483, "rps_doc_frac_unique_words": 0.4455128312110901, "rps_doc_mean_word_length": 4.320512771606445, "rps_doc_num_sentences": 18, "rps_doc_symbol_to_word_ratio": 0.0029673599638044834, "rps_doc_unigram_entropy": 4.560383319854736, "rps_doc_word_count": 312, "rps_doc_frac_chars_dupe_10grams": 0, "rps_doc_frac_chars_dupe_5grams": 0.07270029932260513, "rps_doc_frac_chars_dupe_6grams": 0.03264094889163971, "rps_doc_frac_chars_dupe_7grams": 0, "rps_doc_frac_chars_dupe_8grams": 0, "rps_doc_frac_chars_dupe_9grams": 0, "rps_doc_frac_chars_top_2gram": 0.025964390486478806, "rps_doc_frac_chars_top_3gram": 0.04451039060950279, "rps_doc_frac_chars_top_4gram": 0.04228486865758896, "rps_doc_books_importance": -143.7778778076172, "rps_doc_books_importance_length_correction": -131.71856689453125, "rps_doc_openwebtext_importance": -74.72982788085938, "rps_doc_openwebtext_importance_length_correction": -74.72982788085938, "rps_doc_wikipedia_importance": -52.18912124633789, "rps_doc_wikipedia_importance_length_correction": -42.969966888427734 }, "fasttext": { "dclm": 0.07681422680616379, "english": 0.929432213306427, "fineweb_edu_approx": 1.4912124872207642, "eai_general_math": 0.26769471168518066, "eai_open_web_math": 0.4689621925354004, "eai_web_code": 0.01840233989059925 } }
{ "free_decimal_correspondence": { "primary": { "code": "005.4", "labels": { "level_1": "General works, books and libraries, information sciences", "level_2": "", "level_3": "Computer programming" } }, "secondary": { "code": "004.67", "labels": { "level_1": "General works, books and libraries, information sciences", "level_2": "", "level_3": "Computers and Computer science" } } }, "bloom_cognitive_process": { "primary": { "code": "2", "label": "Understand" }, "secondary": { "code": "3", "label": "Apply" } }, "bloom_knowledge_domain": { "primary": { "code": "2", "label": "Conceptual" }, "secondary": { "code": "3", "label": "Procedural" } }, "document_type_v1": { "primary": { "code": "3", "label": "Reference/Encyclopedic/Educational" }, "secondary": { "code": "6", "label": "Promotional/Advertisement" } }, "extraction_artifacts": { "primary": { "code": "0", "label": "No Artifacts" }, "secondary": { "code": "3", "label": "Irrelevant Content" } }, "missing_content": { "primary": { "code": "0", "label": "No missing content" }, "secondary": { "code": "-1", "label": "Abstain" } }, "document_type_v2": { "primary": { "code": "23", "label": "Tutorial" }, "secondary": { "code": "17", "label": "Product Page" } }, "reasoning_depth": { "primary": { "code": "2", "label": "Basic Reasoning" }, "secondary": { "code": "1", "label": "No Reasoning" } }, "technical_correctness": { "primary": { "code": "6", "label": "Not Applicable/Indeterminate" }, "secondary": { "code": "4", "label": "Highly Correct" } }, "education_level": { "primary": { "code": "1", "label": "General Audience" }, "secondary": { "code": "2", "label": "High School Level" } } }
672f1e42c33a7f9846924a2431ea77df
1,837,067,105,731,986,400
SmsManager does not send sms I tried to send an SMS using SMS Manager. I have sufficient permissions. And I went through some answers given in Stackoverflow. This is my code. It works on my phone but doesnt send the SMS. ‘SMS Sent is shown’ Help me. Thank in advance package com.example.esms; import android.app.Activity; import android.os.Bundle; import android.telephony.SmsManager; import android.view.View; import android.view.View.OnClickListener; import android.widget.Button; import android.widget.EditText; import android.widget.Toast; public class MainActivity extends Activity { @Override protected void onCreate(Bundle savedInstanceState) { super.onCreate(savedInstanceState); setContentView(R.layout.activity_main); final EditText textName = (EditText) findViewById(R.id.editText1); final EditText textPhone = (EditText) findViewById(R.id.editText2); Button buttonSend = (Button) findViewById(R.id.button1); buttonSend.setOnClickListener(new OnClickListener() { @Override public void onClick(View v) { // TODO Auto-generated method stub try { SmsManager smsManager = SmsManager.getDefault(); smsManager.sendTextMessage(textPhone.toString(), null, textName.toString(), null, null); Toast.makeText(getApplicationContext(), "SMS Sent!", Toast.LENGTH_LONG).show(); } catch (Exception e) { Toast.makeText(getApplicationContext(), "SMS faild, please try again later!", Toast.LENGTH_LONG).show(); e.printStackTrace(); } } }); } } Source: eclipse Leave a Reply
{ "url": "https://clearstacknow.com/smsmanager-does-not-send-sms/", "source_domain": "clearstacknow.com", "snapshot_id": "crawl=CC-MAIN-2017-47", "warc_metadata": { "Content-Length": "55905", "Content-Type": "application/http; msgtype=response", "WARC-Block-Digest": "sha1:YNRL23FBHR54BTGR4J5OARBYM5MOKYOK", "WARC-Concurrent-To": "<urn:uuid:cc877350-fce3-4ba6-96d0-a0dc51cabc45>", "WARC-Date": "2017-11-20T17:08:09Z", "WARC-IP-Address": "37.72.171.98", "WARC-Identified-Payload-Type": "text/html", "WARC-Payload-Digest": "sha1:NWYXOTPFXBHOVRF2IUSXSEBF4KPW3VMJ", "WARC-Record-ID": "<urn:uuid:2b377c4a-1aeb-4b80-bae2-233345368717>", "WARC-Target-URI": "https://clearstacknow.com/smsmanager-does-not-send-sms/", "WARC-Truncated": "length", "WARC-Type": "response", "WARC-Warcinfo-ID": "<urn:uuid:251ac50e-0b9e-49ea-b2f0-18daee039556>" }, "warc_info": "robots: classic\r\nhostname: ip-10-230-1-184.ec2.internal\r\nsoftware: Nutch 1.6 (CC)\r\nisPartOf: CC-MAIN-2017-47\r\noperator: Common Crawl Admin\r\ndescription: Wide crawl of the web for November 2017\r\npublisher: Common Crawl\r\nformat: WARC File Format 1.0\r\nconformsTo: http://bibnum.bnf.fr/WARC/WARC_ISO_28500_version1_latestdraft.pdf" }
{ "line_start_idx": [ 0, 29, 30, 268, 269, 295, 296, 325, 351, 352, 389, 415, 457, 487, 519, 548, 549, 594, 595, 605, 658, 698, 742, 743, 744, 815, 887, 948, 949, 1007, 1008, 1026, 1064, 1111, 1129, 1194, 1299, 1368, 1419, 1455, 1511, 1573, 1624, 1661, 1678, 1688, 1696, 1697, 1698, 1700, 1702, 1703, 1704, 1720, 1721 ], "line_end_idx": [ 29, 30, 268, 269, 295, 296, 325, 351, 352, 389, 415, 457, 487, 519, 548, 549, 594, 595, 605, 658, 698, 742, 743, 744, 815, 887, 948, 949, 1007, 1008, 1026, 1064, 1111, 1129, 1194, 1299, 1368, 1419, 1455, 1511, 1573, 1624, 1661, 1678, 1688, 1696, 1697, 1698, 1700, 1702, 1703, 1704, 1720, 1721, 1734 ] }
{ "red_pajama_v2": { "ccnet_original_length": 1734, "ccnet_original_nlines": 54, "rps_doc_curly_bracket": 0.006920420099049807, "rps_doc_ldnoobw_words": 0, "rps_doc_lorem_ipsum": 0, "rps_doc_stop_word_fraction": 0.1071428582072258, "rps_doc_ut1_blacklist": 0, "rps_doc_frac_all_caps_words": 0.051948048174381256, "rps_doc_frac_lines_end_with_ellipsis": 0, "rps_doc_frac_no_alph_words": 0.3928571343421936, "rps_doc_frac_unique_words": 0.7407407164573669, "rps_doc_mean_word_length": 8.585185050964355, "rps_doc_num_sentences": 49, "rps_doc_symbol_to_word_ratio": 0, "rps_doc_unigram_entropy": 4.44045877456665, "rps_doc_word_count": 135, "rps_doc_frac_chars_dupe_10grams": 0, "rps_doc_frac_chars_dupe_5grams": 0, "rps_doc_frac_chars_dupe_6grams": 0, "rps_doc_frac_chars_dupe_7grams": 0, "rps_doc_frac_chars_dupe_8grams": 0, "rps_doc_frac_chars_dupe_9grams": 0, "rps_doc_frac_chars_top_2gram": 0.02243313007056713, "rps_doc_frac_chars_top_3gram": 0, "rps_doc_frac_chars_top_4gram": 0, "rps_doc_books_importance": -116.12228393554688, "rps_doc_books_importance_length_correction": -105.9530258178711, "rps_doc_openwebtext_importance": -82.71806335449219, "rps_doc_openwebtext_importance_length_correction": -82.71806335449219, "rps_doc_wikipedia_importance": -81.04439544677734, "rps_doc_wikipedia_importance_length_correction": -74.69329833984375 }, "fasttext": { "dclm": 0.5915303826332092, "english": 0.3818325400352478, "fineweb_edu_approx": 1.8660714626312256, "eai_general_math": 0.2263047695159912, "eai_open_web_math": 0.019728420302271843, "eai_web_code": 0.34830254316329956 } }
{ "free_decimal_correspondence": { "primary": { "code": "005.1", "labels": { "level_1": "General works, books and libraries, information sciences", "level_2": "", "level_3": "Computer programming" } }, "secondary": { "code": "621.392", "labels": { "level_1": "Industrial arts, Technology, and Engineering", "level_2": "Engineering", "level_3": "Mechanical engineering and Machinery" } } }, "bloom_cognitive_process": { "primary": { "code": "3", "label": "Apply" }, "secondary": { "code": "2", "label": "Understand" } }, "bloom_knowledge_domain": { "primary": { "code": "3", "label": "Procedural" }, "secondary": { "code": "2", "label": "Conceptual" } }, "document_type_v1": { "primary": { "code": "4", "label": "Code/Software" }, "secondary": { "code": "5", "label": "Social/Forum" } }, "extraction_artifacts": { "primary": { "code": "0", "label": "No Artifacts" }, "secondary": { "code": "3", "label": "Irrelevant Content" } }, "missing_content": { "primary": { "code": "0", "label": "No missing content" }, "secondary": { "code": "-1", "label": "Abstain" } }, "document_type_v2": { "primary": { "code": "18", "label": "Q&A Forum" }, "secondary": { "code": "8", "label": "Documentation" } }, "reasoning_depth": { "primary": { "code": "2", "label": "Basic Reasoning" }, "secondary": { "code": "1", "label": "No Reasoning" } }, "technical_correctness": { "primary": { "code": "2", "label": "Partially Correct" }, "secondary": { "code": "1", "label": "Technically Flawed" } }, "education_level": { "primary": { "code": "3", "label": "Undergraduate Level" }, "secondary": { "code": "2", "label": "High School Level" } } }
672f1e42c33a7f9846924a2431ea77df
-4,095,672,218,317,803,000
GPU Only Works In Second Slot? Quick Fixes GPU-only-works-in-second-slot Having trouble with your graphics card only working in the second PCIe slot? Don’t worry, there are a few solutions. In this quick guide, learn how to troubleshoot this common issue and get your graphics card running optimally! If your graphics card only works in the second PCIe slot, try reseating it, updating BIOS, disabling Fast Startup, checking for a switch, or inspecting for damage. GPU Only Works in Second Slot – What’s Going On? If your graphics card is only working when installed in the second PCIe slot on your motherboard, don’t panic. There are a few possible issues that could cause this, and most are easy fixes. Loose Connection First, make sure the GPU is securely connected in the top slot. Over time, the connection can come loose, preventing proper operation. Remove and reseat the card to ensure full contact. Reconnect any power cables as well. BIOS Settings It’s possible there’s a BIOS setting enabling the second slot and disabling the first. Enter your BIOS menu on startup and look for options related to PCIe slots. Ensure the top slot is enabled, and the second slot is disabled or set to auto. Save changes and exit. Damaged PCIe Slot In rare cases, the top PCIe slot itself could be damaged, preventing normal use. This is unlikely, but to check, install another PCIe card in the top slot to see if it’s recognized. If not, the slot itself may need repair or replacement. PCIE-slot Driver Issues Outdated, corrupt or incompatible graphics drivers can sometimes cause GPU detection issues. Uninstall your current graphics drivers completely, then download and install the latest version from your card manufacturer’s website. Test by rebooting the system to see if the issue reoccurs. Potential Causes of GPU Only Working In Second Slot So your graphics card is only working when plugged into the second PCIe slot – frustrating! A few things could be causing this problem. Outdated BIOS Before you begin, verify your motherboard has the most recent BIOS update. An outdated BIOS may not properly detect or configure a newer graphics card, especially in the primary slot. Flash your BIOS to the newest version and try the GPU in the first slot again. Incompatible motherboard If your motherboard is older, it may not fully support the bandwidth or features of your new GPU, especially in the primary PCIe slot. The secondary slot may be an older standard with less bandwidth, so the GPU functions in a basic ‘compatibility mode.’ Unfortunately, the only fix here is upgrading to a newer motherboard that fully supports your graphics card. Faulty PCIe slot It’s possible the primary PCIe slot on your motherboard is malfunctioning or damaged, preventing the GPU from operating properly in that slot. You may need to have the motherboard serviced or replaced to fix the faulty PCIe slot. For the time being, you can continue using the secondary slot, but performance and features may be limited. Incorrect BIOS configuration Double check that PCIe and graphics settings in your BIOS are configured correctly. In some cases, changing the primary display output, enabling or disabling onboard graphics, or manually setting the PCIe slot configuration to Gen 3 or Gen 4 can help get your GPU working in the primary slot. With some troubleshooting, you should be able to narrow down the issue and find a solution to get your graphics card running in the primary PCIe slot. If all else fails, at least you have the secondary slot as a fallback to keep you gaming and creating in the meantime! Troubleshooting Tips to Get Your GPU Working in the First Slot Reseat the graphics card The first thing to try is simply reseating your graphics card in the PCIe slot. Shut down your PC, unplug the power cable, and open up your case. Locate the graphics card in the first PCIe x16 slot (the top slot) and remove it. Check for any dirt or debris in the slot and remove it. Reinstall the graphics card, making sure it’s properly aligned and firmly inserted. Secure the card in place, put the case back on, and boot up your PC. This simple fix solves many GPU issues, so give it a shot. Update your motherboard BIOS An outdated BIOS (basic input/output system) on your motherboard can sometimes cause compatibility issues with graphics cards. Updating to the latest BIOS version may resolve the problem. However, updating the BIOS does come with risks, so proceed with caution. Check your motherboard manual for instructions on how to update the BIOS. You may be able to do it right from the BIOS menu, or you may need to download an update file onto a USB drive. Make sure not to interrupt the update process once it has started. Disable Fast Startup Windows 10 has a feature called Fast Startup that can sometimes interfere with graphics cards. Disabling Fast Startup may allow your GPU to function properly in the first PCIe slot. Open the Control Panel and click “Power Options.” Select “Choose what the power buttons do.” Click “Change settings that are currently unavailable.” Uncheck the box next to “Turn on fast startup.” Click “Save changes.” Restart your computer and see if the issue is resolved. Check for a BIOS switch or jumper Some motherboards have a physical switch or jumper that needs to be set for a graphics card to work in the primary PCIe slot. Consult your motherboard manual to see if it has such a switch or jumper, and make sure it’s set properly. Flipping the switch or changing the jumper setting may do the trick. Checking for Physical Damage or Issues Before you start troubleshooting software issues, it’s a good idea to inspect your graphics card for any physical damage. Open up your computer case and visually check your graphics card for any obvious signs of damage. Look for any corrosion, rust or debris on the card itself or the PCIe slot it plugs into. Check power cables Make sure the 6-pin or 8-pin power connector on your graphics card is properly plugged in. Over time, these power cables can come loose or get disconnected. With your computer powered off, reconnect any power cables to your graphics card and make sure they’re firmly in place. Try a different PCIe slot If your motherboard has multiple PCIe x16 slots, try moving your graphics card to another slot. Power down your computer, unplug the power cable and open the case. Release the graphics card from its current slot, move it to another PCIe x16 slot and reseat it until it clicks into place. Put the case back on, plug in the power and turn on your computer. See if the graphics card is now working properly in the new slot. Still not working? If your graphics card is still not working after trying the steps above, it’s possible there may be an issue with the card itself. You’ll want to have the card tested in another computer to determine if it needs repair or replacement. But before you give up hope, try updating your motherboard BIOS and chipset drivers, as an outdated BIOS can sometimes cause issues detecting hardware like graphics cards. FAQs Why does my GPU only work in the second slot? • Your GPU may only work in the second slot due to various factors such as compatibility issues, incorrect BIOS settings, or hardware faults. How can I troubleshoot my GPU working exclusively in the second slot? • To troubleshoot your GPU exclusively working in the second slot, check for compatibility issues, ensure proper installation and connections, update drivers and BIOS, and test the GPU in different slots if possible.  Is it OK to put GPU in second slot? • Yes, it’s generally okay to put the GPU in the second slot. However, ensure compatibility with your motherboard and check BIOS settings for optimal performance. Does the slot placement of my GPU affect its performance? • Yes, it does matter which slot you put your GPU in. The slot choice can impact performance and compatibility. Always refer to your motherboard manual for the recommended slot for optimal GPU performance. Why are there 2 graphics card slots? • The presence of two graphics card slots allows users to install multiple GPUs for improved graphics performance, gaming, or specialized computing tasks. Do 2 GPUs improve FPS? • Yes, having two GPUs can improve FPS (frames per second) in certain applications and games that support multi-GPU setups, such as SLI or CrossFire. Which slot to use for GPU? • For optimal GPU performance, use the primary PCIe slot on your motherboard. Conclusion A GPU that only works in the second slot can be frustrating, but usually there’s a simple fix.  • First, double check your hardware connections and make sure the GPU is seated properly.  • If that doesn’t work, try updating your motherboard BIOS and chipset drivers. Failing that, adjusting your PCIe settings in BIOS may do the trick.  • And as a last resort, removing your CMOS battery for a few minutes could reset your BIOS and get things working.  With a little troubleshooting, you’ll likely have your GPU running in no time. Just take it slow, stay calm, and methodically try each solution. Before you know it, your graphics card will be working perfectly fine in that second slot. Share Via: I'm Dave, your friendly tech troubleshooter from Tech Rebooter. Having GPU woes? No sweat, I break down fixes into bite-sized chunks to get you back in the game! Leave a comment
{ "url": "https://techrebooter.com/gpu-only-works-in-second-slot/", "source_domain": "techrebooter.com", "snapshot_id": "CC-MAIN-2024-33", "warc_metadata": { "Content-Length": "178918", "Content-Type": "application/http; msgtype=response", "WARC-Block-Digest": "sha1:EO2AMFR4ZTEBXKD5W4GU2ESOGU6AGOW7", "WARC-Concurrent-To": "<urn:uuid:844025d6-5d88-42d8-851e-7d4b2af7a750>", "WARC-Date": "2024-08-04T20:02:42Z", "WARC-IP-Address": "154.41.250.112", "WARC-Identified-Payload-Type": "text/html", "WARC-Payload-Digest": "sha1:PE2LP3HXXECW2OAHRQJZCHETIIGNBXLJ", "WARC-Record-ID": "<urn:uuid:12f620b1-0156-4d5c-a7b4-16f6d3e5c7cd>", "WARC-Target-URI": "https://techrebooter.com/gpu-only-works-in-second-slot/", "WARC-Truncated": null, "WARC-Type": "response", "WARC-Warcinfo-ID": "<urn:uuid:c1e0e665-cd6e-45cd-bdc1-c6f45c5aa5f8>" }, "warc_info": "isPartOf: CC-MAIN-2024-33\r\npublisher: Common Crawl\r\ndescription: Wide crawl of the web for August 2024\r\noperator: Common Crawl Admin ([email protected])\r\nhostname: ip-10-67-67-134\r\nsoftware: Apache Nutch 1.20 (modified, https://github.com/commoncrawl/nutch/)\r\nrobots: checked via crawler-commons 1.5-SNAPSHOT (https://github.com/crawler-commons/crawler-commons)\r\nformat: WARC File Format 1.1\r\nconformsTo: https://iipc.github.io/warc-specifications/specifications/warc-format/warc-1.1/" }
{ "line_start_idx": [ 0, 43, 44, 74, 75, 303, 304, 468, 469, 518, 519, 710, 711, 728, 729, 951, 952, 966, 967, 1233, 1234, 1252, 1253, 1491, 1492, 1502, 1503, 1517, 1518, 1806, 1807, 1859, 1860, 1996, 1997, 2011, 2012, 2275, 2276, 2301, 2302, 2665, 2666, 2683, 2684, 3022, 3023, 3052, 3053, 3346, 3347, 3617, 3618, 3681, 3682, 3707, 3708, 4204, 4205, 4234, 4235, 4750, 4751, 4772, 4773, 5230, 5231, 5265, 5266, 5568, 5569, 5608, 5609, 5919, 5920, 5939, 5940, 6217, 6218, 6244, 6245, 6666, 6667, 6686, 6687, 7094, 7095, 7100, 7101, 7147, 7148, 7292, 7293, 7363, 7364, 7584, 7585, 7621, 7622, 7787, 7788, 7846, 7847, 8055, 8056, 8093, 8094, 8251, 8252, 8275, 8276, 8428, 8429, 8456, 8457, 8537, 8538, 8549, 8550, 8646, 8647, 8740, 8892, 9010, 9011, 9247, 9248, 9259, 9260, 9422, 9423 ], "line_end_idx": [ 43, 44, 74, 75, 303, 304, 468, 469, 518, 519, 710, 711, 728, 729, 951, 952, 966, 967, 1233, 1234, 1252, 1253, 1491, 1492, 1502, 1503, 1517, 1518, 1806, 1807, 1859, 1860, 1996, 1997, 2011, 2012, 2275, 2276, 2301, 2302, 2665, 2666, 2683, 2684, 3022, 3023, 3052, 3053, 3346, 3347, 3617, 3618, 3681, 3682, 3707, 3708, 4204, 4205, 4234, 4235, 4750, 4751, 4772, 4773, 5230, 5231, 5265, 5266, 5568, 5569, 5608, 5609, 5919, 5920, 5939, 5940, 6217, 6218, 6244, 6245, 6666, 6667, 6686, 6687, 7094, 7095, 7100, 7101, 7147, 7148, 7292, 7293, 7363, 7364, 7584, 7585, 7621, 7622, 7787, 7788, 7846, 7847, 8055, 8056, 8093, 8094, 8251, 8252, 8275, 8276, 8428, 8429, 8456, 8457, 8537, 8538, 8549, 8550, 8646, 8647, 8740, 8892, 9010, 9011, 9247, 9248, 9259, 9260, 9422, 9423, 9438 ] }
{ "red_pajama_v2": { "ccnet_original_length": 9438, "ccnet_original_nlines": 130, "rps_doc_curly_bracket": 0, "rps_doc_ldnoobw_words": 0, "rps_doc_lorem_ipsum": 0, "rps_doc_stop_word_fraction": 0.3934338092803955, "rps_doc_ut1_blacklist": 0, "rps_doc_frac_all_caps_words": 0.0376749187707901, "rps_doc_frac_lines_end_with_ellipsis": 0, "rps_doc_frac_no_alph_words": 0.13240042328834534, "rps_doc_frac_unique_words": 0.29106807708740234, "rps_doc_mean_word_length": 4.713304042816162, "rps_doc_num_sentences": 105, "rps_doc_symbol_to_word_ratio": 0, "rps_doc_unigram_entropy": 5.2689008712768555, "rps_doc_word_count": 1601, "rps_doc_frac_chars_dupe_10grams": 0, "rps_doc_frac_chars_dupe_5grams": 0.10495626926422119, "rps_doc_frac_chars_dupe_6grams": 0.06122449040412903, "rps_doc_frac_chars_dupe_7grams": 0.03047971986234188, "rps_doc_frac_chars_dupe_8grams": 0.012456930242478848, "rps_doc_frac_chars_dupe_9grams": 0, "rps_doc_frac_chars_top_2gram": 0.017890270799398422, "rps_doc_frac_chars_top_3gram": 0.03180493041872978, "rps_doc_frac_chars_top_4gram": 0.01192685030400753, "rps_doc_books_importance": -750.6705322265625, "rps_doc_books_importance_length_correction": -750.6705322265625, "rps_doc_openwebtext_importance": -393.64654541015625, "rps_doc_openwebtext_importance_length_correction": -393.64654541015625, "rps_doc_wikipedia_importance": -398.629638671875, "rps_doc_wikipedia_importance_length_correction": -398.629638671875 }, "fasttext": { "dclm": 0.06512290239334106, "english": 0.9017159938812256, "fineweb_edu_approx": 1.55525541305542, "eai_general_math": 0.05701357126235962, "eai_open_web_math": 0.0684124231338501, "eai_web_code": 0.31072312593460083 } }
{ "free_decimal_correspondence": { "primary": { "code": "004.16", "labels": { "level_1": "General works, books and libraries, information sciences", "level_2": "", "level_3": "Computers and Computer science" } }, "secondary": { "code": "-1", "labels": { "level_1": "", "level_2": "", "level_3": "" } } }, "bloom_cognitive_process": { "primary": { "code": "3", "label": "Apply" }, "secondary": { "code": "-1", "label": "Abstain" } }, "bloom_knowledge_domain": { "primary": { "code": "3", "label": "Procedural" }, "secondary": { "code": "-1", "label": "Abstain" } }, "document_type_v1": { "primary": { "code": "3", "label": "Reference/Encyclopedic/Educational" }, "secondary": { "code": "-1", "label": "Abstain" } }, "extraction_artifacts": { "primary": { "code": "0", "label": "No Artifacts" }, "secondary": { "code": "3", "label": "Irrelevant Content" } }, "missing_content": { "primary": { "code": "0", "label": "No missing content" }, "secondary": { "code": "-1", "label": "Abstain" } }, "document_type_v2": { "primary": { "code": "23", "label": "Tutorial" }, "secondary": { "code": "21", "label": "Customer Support" } }, "reasoning_depth": { "primary": { "code": "2", "label": "Basic Reasoning" }, "secondary": { "code": "3", "label": "Intermediate Reasoning" } }, "technical_correctness": { "primary": { "code": "4", "label": "Highly Correct" }, "secondary": { "code": "3", "label": "Mostly Correct" } }, "education_level": { "primary": { "code": "2", "label": "High School Level" }, "secondary": { "code": "1", "label": "General Audience" } } }
672f1e42c33a7f9846924a2431ea77df
-2,241,912,402,867,214,300
Today's Question:  What are you most afraid of as a programmer?        GIVE A SHOUT SEARCH KEYWORD -- java   Are frameworks making developers dumb? Last week I got to take interviews to hire senior java developers with around 5 years of experience. But after the interview process is over I felt like the frameworks makes developers life easier but at the same time making them dumb.Everyone puts almost all the new frameworks on their resume claiming they have "Strong, working experience on Spring, Hibernate, Web Services etc".Here is how the interviews went on.Me: You have used Spring in your latest project. What are the advant...    Framework,Importance,Disadvantage,Easy     2011-09-06 08:10:08   Command Line Arguments Our Hello program still isn’t very general. We can’t change the name we say hello to without editing and recompiling the source code. This may be fine for the programmers, but what if the secretaries want their computers to say Hello to them? (I know. This is a little far-fetched but bear with me. I’m making a point.)What we need is a way to change the name at runtime rather than at compile time. (Runtime is when we type java HelloRusty. Compile time is when we type jav...    Java,Command line arguments,First elemen     2011-09-30 11:31:54   Difference between Enumeration and Iterator in java interview question and answer This tutorial explains about what are the differences between Iterators and Enumeration and similarity of both interface which may be asked in a core java interview. Functionalities of both Iterator & Enumeration interfaces are similar that means both generates a series of all elements of the object which is to have its values iterated that can be traversed one at a time using next() method incase of Iterator and nextElement() method incase of Enumeration. The more powe...    Java,Iterator,Enumeration     2012-05-01 07:41:52   Why 0.1+0.2 != 0.3 In programming languages such as JavaScript, c/c++, Java and Matlab, you will find that you will get unexpected result when doing float point calculation. For example, when calculating 0.1 + 0.1, you will not get 0.3: > 0.1 + 0.2 == 0.3 false > 0.1 + 0.2 0.30000000000000004 Don't be surprised of this result, this is the end result of IEEE 754 standard, float point number cannot be accurately represented according to IEEE 754 because: No enough memory is allocated for representing the num...    float point,comparison,JavaScript     2014-11-19 05:32:46   TIOBE : No news today TIOBE released the programming index for September 2012. There is no big change for this month compared to last month, the top 3 are still C, Java and Objective-C. In top 20, only Transact-SQL exchanged the position with VB.NET. Java shows trend of drop, will it be worse because of the ignorance of Java's vulnerabilities by Oracle. This index shows that the programing lnaguage market is quite normal, no big events. Except Objective-C, in recent years there are no other new languages which have b...    TIOBE,Objective-C,Index     2012-09-05 07:36:07   C++ 11 Memory Management Enterprise development and networking specialist Stephen B. Morris illustrates how to handle a classic C/C++ problem by using the new features in C++ 11 in conjunction with more established techniques.Memory management has always been one of the most error-prone areas of C++. The same is true of C. One of the strengths of managed languages, such as Java and C#, is their support for automatic garbage collection. Garbage collection still isn't a feature of C++ 11, so we must still be caref...    C++ 11,Memory management,GC,Memory leak     2012-01-10 01:14:59   Language Complexity? Some languages are complex, others are simple … right?  C++ versus just about anything else is a good example here.  But, it begs the question: what makes a language complex? So, I’ve just been reading Bruce Eckel’s Artima article on Scala.  It’s actually a nice article, and I enjoyed it.  But, one thing bugged me — and, it’s nicely summarised in this quote: But you can see from the code above that learning Scala should be a lot easier than learning Java! There...    Programming language,complexity     2011-06-15 02:16:05   Java Interview Questions Currently there are many articles online which summarize the list of Java interview questions. Some cover lots of basic questions and some cover some specific questions in specific area such as multithreading. In this post, we will not cover the really basic questions, we will cover something different. For basic question, you can read Java Interview Questions。 Basic What is primitive data type? How many primitive data types in Java? What are they? -- A primitive type is predefined...    JAVA,INTERVIEW,CAREER,MULTITHREADING,QUESTION,JAVA INTERVIEW,JAVA CORE     2015-11-03 08:30:33   Moving from Java to C++: An Interview with Rogers Cadenhead In this interview, co-author of Sams Teach Yourself C++ in 24 Hours, 5th Edition Rogers Cadenhead discusses moving from Java to C++, what brought him to C++, and the best tactics for learning C++.Danny Kalev: For how long were you a Java programmer? Can you tell us a bit about the nature of the projects in which you took part at that time?Rogers Cadenhead: I've been a Java programmer since the language was launched by Sun Microsystems in 1995. I was doing website develop...    Java,C++,Transfer,Transform,New challeng     2011-09-03 11:01:26   try { return } finally {} Do you know what value will be printed when following program is ran? class Test { public int aaa() { int x = 1; try { return ++x; } catch (Exception e) { } finally { ++x; } return x; } public static void main(String[] args) { Test t = new Test(); int y = t.aaa(); System.out.println(y); } } And before answering the above question, do you have answers to following questions? If ther...    JAVA,JAVA INTERVIEW QUESTION     2016-09-26 08:06:28
{ "url": "http://www.pixelstech.net/search.php?key_word=java&page=11", "source_domain": "www.pixelstech.net", "snapshot_id": "crawl=CC-MAIN-2017-17", "warc_metadata": { "Content-Length": "26107", "Content-Type": "application/http; msgtype=response", "WARC-Block-Digest": "sha1:WBMC4KRXLGLELFHUCQ6LUV7DEZ7IHSCD", "WARC-Concurrent-To": "<urn:uuid:b68d27d7-c563-4c72-aefa-0ae48532ed82>", "WARC-Date": "2017-04-27T05:19:04Z", "WARC-IP-Address": "192.185.36.119", "WARC-Identified-Payload-Type": null, "WARC-Payload-Digest": "sha1:KT2CY7U6XLLWMXN3A7F24UNP7RCDKEEI", "WARC-Record-ID": "<urn:uuid:a71cf101-1d95-4686-b0fd-2191098f9069>", "WARC-Target-URI": "http://www.pixelstech.net/search.php?key_word=java&page=11", "WARC-Truncated": null, "WARC-Type": "response", "WARC-Warcinfo-ID": "<urn:uuid:0148b153-f734-4321-a0b4-8ec9b659d40b>" }, "warc_info": "robots: classic\r\nhostname: ip-10-145-167-34.ec2.internal\r\nsoftware: Nutch 1.6 (CC)/CC WarcExport 1.0\r\nisPartOf: CC-MAIN-2017-17\r\noperator: CommonCrawl Admin\r\ndescription: Wide crawl of the web for April 2017\r\npublisher: CommonCrawl\r\nformat: WARC File Format 1.0\r\nconformsTo: http://bibnum.bnf.fr/WARC/WARC_ISO_28500_version1_latestdraft.pdf" }
{ "line_start_idx": [ 0, 84, 85, 108, 109, 110, 111, 152, 153, 154, 643, 644, 710, 711, 712, 737, 738, 739, 1217, 1218, 1286, 1287, 1288, 1372, 1373, 1374, 1852, 1853, 1906, 1907, 1908, 1929, 1930, 1931, 2427, 2428, 2489, 2490, 2491, 2515, 2516, 2517, 3021, 3022, 3073, 3074, 3075, 3102, 3103, 3104, 3600, 3601, 3668, 3669, 3670, 3693, 3694, 3695, 4164, 4165, 4224, 4225, 4226, 4253, 4254, 4255, 4745, 4746, 4844, 4845, 4846, 4908, 4909, 4910, 5389, 5390, 5458, 5459, 5460, 5488, 5489, 5490, 5878, 5879 ], "line_end_idx": [ 84, 85, 108, 109, 110, 111, 152, 153, 154, 643, 644, 710, 711, 712, 737, 738, 739, 1217, 1218, 1286, 1287, 1288, 1372, 1373, 1374, 1852, 1853, 1906, 1907, 1908, 1929, 1930, 1931, 2427, 2428, 2489, 2490, 2491, 2515, 2516, 2517, 3021, 3022, 3073, 3074, 3075, 3102, 3103, 3104, 3600, 3601, 3668, 3669, 3670, 3693, 3694, 3695, 4164, 4165, 4224, 4225, 4226, 4253, 4254, 4255, 4745, 4746, 4844, 4845, 4846, 4908, 4909, 4910, 5389, 5390, 5458, 5459, 5460, 5488, 5489, 5490, 5878, 5879, 5934 ] }
{ "red_pajama_v2": { "ccnet_original_length": 5934, "ccnet_original_nlines": 83, "rps_doc_curly_bracket": 0.0026963299605995417, "rps_doc_ldnoobw_words": 0, "rps_doc_lorem_ipsum": 0, "rps_doc_stop_word_fraction": 0.3210650086402893, "rps_doc_ut1_blacklist": 0, "rps_doc_frac_all_caps_words": 0.044635869562625885, "rps_doc_frac_lines_end_with_ellipsis": 0.1190476194024086, "rps_doc_frac_no_alph_words": 0.2826938033103943, "rps_doc_frac_unique_words": 0.4933481216430664, "rps_doc_mean_word_length": 5.028824806213379, "rps_doc_num_sentences": 75, "rps_doc_symbol_to_word_ratio": 0.009397019632160664, "rps_doc_unigram_entropy": 5.62021017074585, "rps_doc_word_count": 902, "rps_doc_frac_chars_dupe_10grams": 0, "rps_doc_frac_chars_dupe_5grams": 0.007495590019971132, "rps_doc_frac_chars_dupe_6grams": 0, "rps_doc_frac_chars_dupe_7grams": 0, "rps_doc_frac_chars_dupe_8grams": 0, "rps_doc_frac_chars_dupe_9grams": 0, "rps_doc_frac_chars_top_2gram": 0.014329809695482254, "rps_doc_frac_chars_top_3gram": 0.004409169778227806, "rps_doc_frac_chars_top_4gram": 0.005291009787470102, "rps_doc_books_importance": -559.4967651367188, "rps_doc_books_importance_length_correction": -559.4967651367188, "rps_doc_openwebtext_importance": -347.0892639160156, "rps_doc_openwebtext_importance_length_correction": -347.0892639160156, "rps_doc_wikipedia_importance": -255.22764587402344, "rps_doc_wikipedia_importance_length_correction": -255.22764587402344 }, "fasttext": { "dclm": 0.054792050272226334, "english": 0.8630961775779724, "fineweb_edu_approx": 2.0793628692626953, "eai_general_math": 0.2179388403892517, "eai_open_web_math": 0.1498599648475647, "eai_web_code": 0.029983339831233025 } }
{ "free_decimal_correspondence": { "primary": { "code": "005.133", "labels": { "level_1": "General works, books and libraries, information sciences", "level_2": "", "level_3": "Computer programming" } }, "secondary": { "code": "005.1332", "labels": { "level_1": "General works, books and libraries, information sciences", "level_2": "", "level_3": "Computer programming" } } }, "bloom_cognitive_process": { "primary": { "code": "2", "label": "Understand" }, "secondary": { "code": "3", "label": "Apply" } }, "bloom_knowledge_domain": { "primary": { "code": "2", "label": "Conceptual" }, "secondary": { "code": "3", "label": "Procedural" } }, "document_type_v1": { "primary": { "code": "3", "label": "Reference/Encyclopedic/Educational" }, "secondary": { "code": "-1", "label": "Abstain" } }, "extraction_artifacts": { "primary": { "code": "0", "label": "No Artifacts" }, "secondary": { "code": "-1", "label": "Abstain" } }, "missing_content": { "primary": { "code": "0", "label": "No missing content" }, "secondary": { "code": "-1", "label": "Abstain" } }, "document_type_v2": { "primary": { "code": "10", "label": "Knowledge Article" }, "secondary": { "code": "23", "label": "Tutorial" } }, "reasoning_depth": { "primary": { "code": "3", "label": "Intermediate Reasoning" }, "secondary": { "code": "2", "label": "Basic Reasoning" } }, "technical_correctness": { "primary": { "code": "3", "label": "Mostly Correct" }, "secondary": { "code": "4", "label": "Highly Correct" } }, "education_level": { "primary": { "code": "3", "label": "Undergraduate Level" }, "secondary": { "code": "2", "label": "High School Level" } } }
672f1e42c33a7f9846924a2431ea77df
-4,992,574,329,612,325,000
Interfaces on many homebrew apps. Discussion in 'Wii - Hacking' started by nas939, Dec 28, 2008. 1. nas939 OP nas939 Newbie Newcomer 1 Dec 28, 2008 Hey guys, I've noticed that many homebrew apps, while offering awesome functionality, have some pretty barebones (ugly) interfaces. Now I know virtually nothing about coding so have no idea if this is because it's hard to implement, or if it's because teams and developers would rather spend time perfecting the code than prettying the interface. If it is because of the latter then i'd just like to say that i'm fairly experienced with photoshop and, while i haven't done much interface development before i can definately learn and if anyone would like my help i'd love to be able to help. Applications like Geexbox, mplayer, and even the backup loaders, etc. could all use some prettying up. So feel free to pm me or just reply to this topic if you like.   2. jspicher jspicher Advanced Member Newcomer 2 Nov 18, 2008 United States I think the reason the GUIs are so compact/simple and unattractive (to you) is because the developers do not want to add a bunch of graphics that will A.) use up your wii memory blocks (and god knows everyone is already having a problem with that), and B.) adding a bunch of unnecessary artwork to the GUIs will increase the loading time of channels/homebrew apps. I like pretty GUIs just as much as the next guy; but l need all the memory blocks on my wii that l can get, and l also don't want to wait an extra 3 or 4 seconds every time l click on a channel/homebrew app just because it's got pretty rollover effects, and neat graphical elements. It's not how the app looks that makes it great, it's what it accomplishes.   3. nas939 OP nas939 Newbie Newcomer 1 Dec 28, 2008 I agree that the functionality of the apps is most important but i disagree with your logic. Most homebrew apps are stored on SD cards and most people already have SD cards with capacities in excess of 2gb. I also doubt it would affect the loading times that much, there is a very nice game called smashing! which i think is a good example of a homebrew app with a pretty nice interface. It has virtually no loading time and takes up an almost negligible amount of space on my SD card. Channels and savegames are what take up space (or memory blocks), not homebrew apps.   4. jspicher jspicher Advanced Member Newcomer 2 Nov 18, 2008 United States >Channels and savegames are what take up space (or memory blocks), not homebrew apps. You're right, l was considering the fact that useful homebrew apps would be injected into wads, and used as channels otherwise the only space that's being used is on the SD. In regards to loading times... I've read that background sounds, and overhead images can cause loading issues but have not encounter it myself.   Loading...
{ "url": "https://gbatemp.net/threads/interfaces-on-many-homebrew-apps.125747/", "source_domain": "gbatemp.net", "snapshot_id": "crawl=CC-MAIN-2018-39", "warc_metadata": { "Content-Length": "77497", "Content-Type": "application/http; msgtype=response", "WARC-Block-Digest": "sha1:NDQ6JUOGFYGJ3HPIPFOGBT5ISV5LHEPS", "WARC-Concurrent-To": "<urn:uuid:967687c0-a371-4a8b-a061-e55e1efd8aae>", "WARC-Date": "2018-09-19T06:12:16Z", "WARC-IP-Address": "163.172.23.33", "WARC-Identified-Payload-Type": "text/html", "WARC-Payload-Digest": "sha1:2X4IKNDDO7BYMNMLOFKXNC3VSNPDEPW2", "WARC-Record-ID": "<urn:uuid:8cc2567a-3e61-48ff-94a7-18e8e2a01409>", "WARC-Target-URI": "https://gbatemp.net/threads/interfaces-on-many-homebrew-apps.125747/", "WARC-Truncated": null, "WARC-Type": "response", "WARC-Warcinfo-ID": "<urn:uuid:e0ce4497-59f5-4140-a744-244cd63d4299>" }, "warc_info": "isPartOf: CC-MAIN-2018-39\r\npublisher: Common Crawl\r\ndescription: Wide crawl of the web for September 2018\r\noperator: Common Crawl Admin ([email protected])\r\nhostname: ip-10-47-244-60.ec2.internal\r\nsoftware: Apache Nutch 1.15 (modified, https://github.com/commoncrawl/nutch/)\r\nrobots: checked via crawler-commons 0.11-SNAPSHOT (https://github.com/crawler-commons/crawler-commons)\r\nformat: WARC File Format 1.1\r\nconformsTo: http://iipc.github.io/warc-specifications/specifications/warc-format/warc-1.1/" }
{ "line_start_idx": [ 0, 34, 35, 98, 99, 111, 118, 119, 137, 138, 151, 157, 174, 188, 189, 315, 534, 535, 784, 785, 892, 893, 960, 966, 980, 981, 1010, 1011, 1024, 1030, 1047, 1065, 1434, 1435, 1722, 1723, 1802, 1808, 1820, 1827, 1828, 1846, 1847, 1860, 1866, 1883, 1980, 1981, 2099, 2100, 2285, 2286, 2388, 2389, 2478, 2484, 2498, 2499, 2528, 2529, 2542, 2548, 2565, 2583, 2673, 2674, 2852, 2853, 3001, 3007 ], "line_end_idx": [ 34, 35, 98, 99, 111, 118, 119, 137, 138, 151, 157, 174, 188, 189, 315, 534, 535, 784, 785, 892, 893, 960, 966, 980, 981, 1010, 1011, 1024, 1030, 1047, 1065, 1434, 1435, 1722, 1723, 1802, 1808, 1820, 1827, 1828, 1846, 1847, 1860, 1866, 1883, 1980, 1981, 2099, 2100, 2285, 2286, 2388, 2389, 2478, 2484, 2498, 2499, 2528, 2529, 2542, 2548, 2565, 2583, 2673, 2674, 2852, 2853, 3001, 3007, 3017 ] }
{ "red_pajama_v2": { "ccnet_original_length": 3017, "ccnet_original_nlines": 69, "rps_doc_curly_bracket": 0, "rps_doc_ldnoobw_words": 0, "rps_doc_lorem_ipsum": 0, "rps_doc_stop_word_fraction": 0.4583333432674408, "rps_doc_ut1_blacklist": 0, "rps_doc_frac_all_caps_words": 0.02500000037252903, "rps_doc_frac_lines_end_with_ellipsis": 0.014285709708929062, "rps_doc_frac_no_alph_words": 0.16833333671092987, "rps_doc_frac_unique_words": 0.4721115529537201, "rps_doc_mean_word_length": 4.428287029266357, "rps_doc_num_sentences": 28, "rps_doc_symbol_to_word_ratio": 0.0033333299215883017, "rps_doc_unigram_entropy": 5.125762939453125, "rps_doc_word_count": 502, "rps_doc_frac_chars_dupe_10grams": 0.11426001042127609, "rps_doc_frac_chars_dupe_5grams": 0.14844803512096405, "rps_doc_frac_chars_dupe_6grams": 0.14844803512096405, "rps_doc_frac_chars_dupe_7grams": 0.14844803512096405, "rps_doc_frac_chars_dupe_8grams": 0.14844803512096405, "rps_doc_frac_chars_dupe_9grams": 0.14844803512096405, "rps_doc_frac_chars_top_2gram": 0.03238866105675697, "rps_doc_frac_chars_top_3gram": 0.012145750224590302, "rps_doc_frac_chars_top_4gram": 0.01799369975924492, "rps_doc_books_importance": -269.38848876953125, "rps_doc_books_importance_length_correction": -269.38848876953125, "rps_doc_openwebtext_importance": -169.65554809570312, "rps_doc_openwebtext_importance_length_correction": -169.65554809570312, "rps_doc_wikipedia_importance": -121.13976287841797, "rps_doc_wikipedia_importance_length_correction": -121.13976287841797 }, "fasttext": { "dclm": 0.2138749361038208, "english": 0.9611759781837463, "fineweb_edu_approx": 1.3872655630111694, "eai_general_math": 0.14774101972579956, "eai_open_web_math": 0.1397547721862793, "eai_web_code": 0.024871349334716797 } }
{ "free_decimal_correspondence": { "primary": { "code": "004.16", "labels": { "level_1": "General works, books and libraries, information sciences", "level_2": "", "level_3": "Computers and Computer science" } }, "secondary": { "code": "005.1", "labels": { "level_1": "General works, books and libraries, information sciences", "level_2": "", "level_3": "Computer programming" } } }, "bloom_cognitive_process": { "primary": { "code": "5", "label": "Evaluate" }, "secondary": { "code": "2", "label": "Understand" } }, "bloom_knowledge_domain": { "primary": { "code": "2", "label": "Conceptual" }, "secondary": { "code": "3", "label": "Procedural" } }, "document_type_v1": { "primary": { "code": "5", "label": "Social/Forum" }, "secondary": { "code": "-1", "label": "Abstain" } }, "extraction_artifacts": { "primary": { "code": "0", "label": "No Artifacts" }, "secondary": { "code": "-1", "label": "Abstain" } }, "missing_content": { "primary": { "code": "0", "label": "No missing content" }, "secondary": { "code": "-1", "label": "Abstain" } }, "document_type_v2": { "primary": { "code": "18", "label": "Q&A Forum" }, "secondary": { "code": "5", "label": "Comment Section" } }, "reasoning_depth": { "primary": { "code": "2", "label": "Basic Reasoning" }, "secondary": { "code": "3", "label": "Intermediate Reasoning" } }, "technical_correctness": { "primary": { "code": "6", "label": "Not Applicable/Indeterminate" }, "secondary": { "code": "3", "label": "Mostly Correct" } }, "education_level": { "primary": { "code": "1", "label": "General Audience" }, "secondary": { "code": "2", "label": "High School Level" } } }
672f1e42c33a7f9846924a2431ea77df
-7,129,950,641,770,198,000
Could not find a JavaScript runtime 16.05.2013, рубрики: FreeBSD | Теги: , | Комментариев нет » После установки Rails вывалилась ошибка: 1 Could not find a JavaScript runtime. See https://github.com/sstephenson/execjs for a list of available runtimes. (ExecJS::RuntimeUnavailable) Приводим /usr/local/lib/ruby/gems/1.8/gems/execjs-1.4.0/lib/execjs/runtimes.rb к такому виду 1 2 3 4 5 6   Node = ExternalRuntime.new(       :name        => "Node.js (V8)",       :command     => ["nodejs", "/usr/local/bin/node"],       :runner_path => ExecJS.root + "/support/node_runner.js",       :encoding    => 'UTF-8'     ) Оставить комментарий • × 6 = тридцать шесть
{ "url": "http://free-pc-help.ru/freebsd/could-not-find-a-javascript-runtime/", "source_domain": "free-pc-help.ru", "snapshot_id": "crawl=CC-MAIN-2019-09", "warc_metadata": { "Content-Length": "27616", "Content-Type": "application/http; msgtype=response", "WARC-Block-Digest": "sha1:R7TFLI4EOI3KEWHF4UXNGMRLAXOFOYEY", "WARC-Concurrent-To": "<urn:uuid:65dfa38a-280b-4f15-8ea4-aa8cbf98daaa>", "WARC-Date": "2019-02-16T04:04:45Z", "WARC-IP-Address": "5.9.105.108", "WARC-Identified-Payload-Type": "application/xhtml+xml", "WARC-Payload-Digest": "sha1:PEBILBPADK7DVFY2LZMMGRIUIZE4EZXF", "WARC-Record-ID": "<urn:uuid:87632972-7e5d-4aba-b781-7f6ad36fa58b>", "WARC-Target-URI": "http://free-pc-help.ru/freebsd/could-not-find-a-javascript-runtime/", "WARC-Truncated": null, "WARC-Type": "response", "WARC-Warcinfo-ID": "<urn:uuid:2cb44f69-d033-4204-8818-7efbf08092ff>" }, "warc_info": "isPartOf: CC-MAIN-2019-09\r\npublisher: Common Crawl\r\ndescription: Wide crawl of the web for February 2019\r\noperator: Common Crawl Admin ([email protected])\r\nhostname: ip-10-148-30-93.ec2.internal\r\nsoftware: Apache Nutch 1.15 (modified, https://github.com/commoncrawl/nutch/)\r\nrobots: checked via crawler-commons 0.11-SNAPSHOT (https://github.com/crawler-commons/crawler-commons)\r\nformat: WARC File Format 1.1\r\nconformsTo: http://iipc.github.io/warc-specifications/specifications/warc-format/warc-1.1/" }
{ "line_start_idx": [ 0, 36, 37, 97, 98, 139, 140, 142, 284, 285, 378, 379, 381, 383, 385, 387, 389, 391, 421, 459, 516, 579, 609, 615, 616, 617, 638, 639, 640 ], "line_end_idx": [ 36, 37, 97, 98, 139, 140, 142, 284, 285, 378, 379, 381, 383, 385, 387, 389, 391, 421, 459, 516, 579, 609, 615, 616, 617, 638, 639, 640, 664 ] }
{ "red_pajama_v2": { "ccnet_original_length": 664, "ccnet_original_nlines": 28, "rps_doc_curly_bracket": 0, "rps_doc_ldnoobw_words": 0, "rps_doc_lorem_ipsum": 0, "rps_doc_stop_word_fraction": 0.06748466193675995, "rps_doc_ut1_blacklist": 0, "rps_doc_frac_all_caps_words": 0.012269940227270126, "rps_doc_frac_lines_end_with_ellipsis": 0, "rps_doc_frac_no_alph_words": 0.6319018602371216, "rps_doc_frac_unique_words": 0.8461538553237915, "rps_doc_mean_word_length": 6.92307710647583, "rps_doc_num_sentences": 14, "rps_doc_symbol_to_word_ratio": 0, "rps_doc_unigram_entropy": 3.95306134223938, "rps_doc_word_count": 65, "rps_doc_frac_chars_dupe_10grams": 0, "rps_doc_frac_chars_dupe_5grams": 0.13333332538604736, "rps_doc_frac_chars_dupe_6grams": 0.13333332538604736, "rps_doc_frac_chars_dupe_7grams": 0, "rps_doc_frac_chars_dupe_8grams": 0, "rps_doc_frac_chars_dupe_9grams": 0, "rps_doc_frac_chars_top_2gram": 0.035555560141801834, "rps_doc_frac_chars_top_3gram": 0.053333330899477005, "rps_doc_frac_chars_top_4gram": 0.057777781039476395, "rps_doc_books_importance": -84.70523071289062, "rps_doc_books_importance_length_correction": -92.38311767578125, "rps_doc_openwebtext_importance": -29.207576751708984, "rps_doc_openwebtext_importance_length_correction": -36.885467529296875, "rps_doc_wikipedia_importance": -9.088409423828125, "rps_doc_wikipedia_importance_length_correction": -16.766300201416016 }, "fasttext": { "dclm": 0.056717399507761, "english": 0.0465644896030426, "fineweb_edu_approx": 0.8893826603889465, "eai_general_math": -0.0000016699999605407356, "eai_open_web_math": 0.6028164029121399, "eai_web_code": 0.7970314025878906 } }
{ "free_decimal_correspondence": { "primary": { "code": "005.133", "labels": { "level_1": "General works, books and libraries, information sciences", "level_2": "", "level_3": "Computer programming" } }, "secondary": { "code": "004.028", "labels": { "level_1": "General works, books and libraries, information sciences", "level_2": "", "level_3": "Computers and Computer science" } } }, "bloom_cognitive_process": { "primary": { "code": "3", "label": "Apply" }, "secondary": { "code": "2", "label": "Understand" } }, "bloom_knowledge_domain": { "primary": { "code": "3", "label": "Procedural" }, "secondary": { "code": "2", "label": "Conceptual" } }, "document_type_v1": { "primary": { "code": "3", "label": "Reference/Encyclopedic/Educational" }, "secondary": { "code": "4", "label": "Code/Software" } }, "extraction_artifacts": { "primary": { "code": "0", "label": "No Artifacts" }, "secondary": { "code": "3", "label": "Irrelevant Content" } }, "missing_content": { "primary": { "code": "0", "label": "No missing content" }, "secondary": { "code": "-1", "label": "Abstain" } }, "document_type_v2": { "primary": { "code": "23", "label": "Tutorial" }, "secondary": { "code": "21", "label": "Customer Support" } }, "reasoning_depth": { "primary": { "code": "2", "label": "Basic Reasoning" }, "secondary": { "code": "3", "label": "Intermediate Reasoning" } }, "technical_correctness": { "primary": { "code": "3", "label": "Mostly Correct" }, "secondary": { "code": "4", "label": "Highly Correct" } }, "education_level": { "primary": { "code": "3", "label": "Undergraduate Level" }, "secondary": { "code": "2", "label": "High School Level" } } }
672f1e42c33a7f9846924a2431ea77df
4,890,541,816,314,657,000
Search Images Maps Play YouTube News Gmail Drive More » Sign in Screen reader users: click this link for accessible mode. Accessible mode has the same essential features but works better with your reader. Patents 1. Advanced Patent Search Publication numberUS5845117 A Publication typeGrant Application numberUS 08/719,919 Publication date1 Dec 1998 Filing date25 Sep 1996 Priority date19 Mar 1993 Fee statusLapsed Also published asDE69422743D1, DE69422743T2, EP0618532A2, EP0618532A3, EP0618532B1 Publication number08719919, 719919, US 5845117 A, US 5845117A, US-A-5845117, US5845117 A, US5845117A InventorsKazuhiko Fujita Original AssigneeFujitsu Limited Export CitationBiBTeX, EndNote, RefMan External Links: USPTO, USPTO Assignment, Espacenet Deadlock detecting device US 5845117 A Abstract Start, commit and abort of transactions in a computer system are managed by a task manager. When a certain transaction locks a certain resource, this information is registered in a lock manager. Accordingly, when a transaction requests for gaining a resource, the lock manager can determine that the resource is already locked, if any, by another transaction. In such a case, the transaction should wait for the termination of the other transaction, so that this information is registered in a wait-for-graph table. A deadlock detector determines whether the deadlock is caused according to the registered information in the wait-for-graph table. Images(10) Previous page Next page Claims(15) We claim: 1. A deadlock detecting device of a multitasking system, said device detecting deadlock between plurality of tasks executed in the multitasking system, comprising; task managing means for managing parallel execution of plurality of the tasks; lock managing means for detecting that one task locks a common resource , and for detecting that another task is waiting for the common resource locked by the former task and outputting waiting relation information indicating that the latter task is waiting for the former task; a wait management table for storing a plural pieces of waiting relation information; registering means for registering said waiting relation information in said wait management table, every time said lock managing means outputs said waiting relation information; and deadlock detecting means for detecting the deadlock between plurality of the tasks, of plural pieces of waiting relation involving the tasks which are respectively indicated by any one of the waiting relation information stored in the wait management table forms a loop relation, said deadlock detecting means functioning independently and asynchronously from said lock managing means and said task managing means; whereby execution of said first task and said second task is not suspended during deadlock detection. 2. The deadlock detecting device as claimed in claim 1, wherein said registering means has a table in which said waiting relations are registered. 3. The deadlock detecting device as claimed in claim 2, wherein said deadlock detecting means registers the waiting relations in the table of said registering means and detects the presence or absence of deadlock by searching said table. 4. The deadlock detecting device as claimed in claim 2, wherein said deadlock detecting means comprises a request queue receiving unit for receiving a "waiting relation registering request" indicative of an instruction to detect the deadlock loop. 5. The deadlock detecting device as claimed in claim 2, wherein the deadlock detecting device is realized on a distribution system having a first and second system, wherein each system comprises said task managing means, said lock managing means, said wait management table and said deadlock detecting means. 6. The deadlock detecting device as claimed in claim 5, wherein said second waiting relation from said first system is communicated and registered in said wait management table of said second system, and the presence or absence of the deadlock loop is determined by looking up said wait management table in said second system. 7. The deadlock detecting device as claimed in claim 5, wherein a task x generated in a system i is defined as T(i, x), a task y generated in a system j is defined as T(j, y) and waiting of T(i, x) for T(j, y) is represented by T(i, x)→T(j, y), which is registered in said table as information of the third waiting relation to detect a loop of T(i, x)→T(j, y)→T(i, x) when both T(i, x)→T(j, y) and T(j, y)→T(i, x) hold, thereby detecting the deadlock loop. 8. The deadlock detecting device as claimed in claim 1, wherein the dead lock detecting device is realized on a distribution system having a plurality of systems and comprises a waiting time monitoring unit for use in reissuing a resource gaining request to the task of which waiting relation is continued for a predetermined time. 9. A deadlock detecting device as claimed in claim 1, wherein said task managing means manages the tasks through a two-phase lock system in which a lock is continuously made when the task starts to lock a data while the lock is continuously released when the task starts to release the lock. 10. The deadlock detecting device as claimed in claim 2, wherein said lock managing means registers the first waiting relation in said wait management table and issues a deadlock detecting instruction to the deadlock detecting means only when the first waiting relation is caused to be a resource gaining request, and wherein said lock managing means does not suspend the execution operation on the transaction for the resource gaining request accompanying an absence of a waiting relation. 11. A deadlock detecting device of a multitasking system, said device detecting deadlock between plurality of tasks executed in the multitasking system, comprising: a task manager which manages parallel execution of plurality of the tasks; a lock manager which makes one task a common resource, and which detects that another task is waiting for the common resource locked by the former task and consequently output waiting relation information indicating that the latter task is waiting for the former task; a wait management table for storing the waiting relation information; and deadlock detector which detects the deadlock between plurality of the tasks if plural pieces of the waiting relation involving the tasks which are respectively indicated by any one of the waiting relation information stored in the wait management table forms a loop relation, said deadlock detector functioning independently and asynchronously from said lock manager and said task manager; whereby execution of said first task and said second task is not suspended during the deadlock detection. 12. The deadlock detecting device as claimed in claim 11, wherein said task manager starts, commits and aborts the first and second tasks. 13. The deadlock detecting device as claimed in claim 11, wherein said lock manager includes a lock table for locking a corresponding relation between said first task and said first resource and between said second task and said second resource. 14. A deadlock detecting device of a distribution system having a plurality of systems, said device detecting deadlock between plurality of tasks each of which is executed in one of the systems and is usable common resources, each system comprising: a task manager which manages parallel execution of plurality of the task executed in own system; a lock manager which makes one task lock a common resource and which detects that another task is waiting for the common resource locked by the former task and consequently output waiting relation information indicating that the latter task is waiting for the former task; means for sending only waiting relation information indicating that one task is waiting for any task executed in other system to this other system, said sending means further sending waiting relation information indicating that one task is waiting for another task involved in the sent waiting relation information; a wait management table for storing the waiting relation information outputted by the lock manager of own system and the waiting relation information sent from the sending means of other system; a deadlock detector which detects the deadlock between plurality of the tasks, if plural pieces of the waiting relation involving the tasks which are respectively indicated by any waiting relation information stored in the wait management table forms a loop relation, said deadlock detector functioning independently and asynchronously from said lock manager and said task manager; whereby execution of said first task and said second task is not suspended during the deadlock detection. 15. A deadlock detecting device according to claim 14, further comprising: a watchdog timer watch which monitors, for every waiting relation information, elapsed time since the waiting relation information stored in the wait management table and, when the elapsed time exceeds a predetermined time period, makes the deadlock detector erase the waiting relation information from the wait management table to make the waiting transaction involved in the waiting relation information run again. Description This application is a continuation, of application Ser. No. 08/204,506, filed Mar. 2, 1994, now abandoned. BACKGROUND OF THE INVENTION 1. Field of the Invention This invention relates to a device for detecting deadlock in a multitasking system. 2. Description of the Related Art In recent years, multitasking systems have been developed in which a plurality of tasks or transactions are executed at the same time as a mode of operation of an information processing system using computers. A task is one or more sequence of instructions treated as a unit of work to be accomplished by a central processing unit (CPU). A transaction is a set of operations to carry out one complete data operation. The multitasking is a state in which two or more programs (tasks, transactions) are executed simultaneously in parallel on a single computer system or on a plurality of computer systems connected in an interleaved manner allowing exchange of information therebetween. In the operating systems that support this multitasking, two or more tasks may commonly use one computer resource. In such a case, each task may exclusively use portions of the resources that are necessary for the execution of the other task(s). As a result, the tasks are simultaneously waiting resources assigned to the other(s), with each one unable to carry out further processing. Such a situation is called deadlock. A term "lock" is commonly used as a technical term representing "use exclusively," so that a word "lock" is used hereinafter also as a verb in place of "use exclusively." An example of a deadlock situation is shown in FIG. 10. FIG. 10 illustrates an example in a distribution system consisting of two computer systems i and j. One computer system i executes a task x and the other computer j executes a task y. In addition, it is assumed that two resources A and B can be accessed by the individual computer systems i and j. A resource includes software such as programs, files and data allocated to the tasks. The resource used herein is the contents (pages or records) of a database located somewhere other than the computer systems i and j. In FIG. 10, the task x locks the resource A while the task y locks the resource B. At the same time, the task y asks for the resource A and awaits to lock the resource A. Likewise, the task x asks for the resource B and awaits to lock the resource B. In such a case, the task y cannot lock the resource A until the task x releases the lock on the resource A. Likewise, the task x cannot lock the resource B until the task y release the lock on the resource B. As a result, the tasks x and y are each awaiting the resources locked by the other. When both tasks x and y are suspended, it becomes impossible to release the resources A and B that are locked by them. Accordingly, this situation lasts endlessly and each task is thus unable to carry out further processing. Such deadlock is a problem that could be caused in the multitasking systems regardless of whether the computer system is a multi-processing system or a single processing system and whether the computer system performs stand-alone processing or serves as a distribution system. A way of recovering from deadlock, should be provided. To this end, the deadlocks should be detected. To detect the deadlocks, the following specifications are required to be satisfied by the considerations of improving the practicability. First, it is necessary to avoid detection of a phantom deadlock, i.e. an incorrect recognition, of a situation that is not the deadlock. (First requirement) Second, all deadlocks should be detected. In other words, the deadlocks, if actually caused, should be detected as a deadlock and it is not permitted to detect the deadlock or not as a case may be. (Second requirement) Third, any effects on the system due to detection of the deadlocks should be reduced as much as possible. More specifically, stopping the task for detection of the deadlock should be avoided, if possible. (Third requirement) If the multitasking system is achieved on a distribution system, a following specification is required along with the above mentioned requirements. While systems are required to intercommunicate with each other to detect the deadlocks, an overhead time that the systems spend for the communication should be reduced as much as possible. (Fourth requirement) Conventional deadlock detecting devices are based on detection of the deadlocks with satisfaction of the above mentioned first through third requirements by means of satisfying the following conditions. The conditions are: (a) free from asynchronous abort of one of the two contending transactions; (b) free from delay and loss of communication messages between the systems when the multitasking system is achieved on the distribution system; (c) free from modification of a transaction waiting relation during detection of the deadlocks; and (d) free from an asynchronous down of the system when the multitasking system is achieved on the distribution system. Described is a relation between the first through third requirements and conditions (a) through (d). For condition (a), the asynchronous abort of the transactions makes it impossible to avoid detection of the phantom deadlock in the above mentioned first requirement. For example, it is assumed that task (transaction) x locks resource A while task (transaction) y locks resource B and that task y is asynchronously aborted when task y asks for resource A and task x asks for the resource B. In such a case, resource B locked by task y is released as a result of the asynchronous aborting of task y and thus task x is allowed to lock on resource B. Accordingly, the deadlock is avoided. It is, however, impossible to detect immediately this release of resource B due to the asynchronous aborting of the task. This makes the situation be considered as the deadlock though no deadlock is caused actually. For condition (b), asynchronous down of the system sometimes allows detection of the deadlock and sometimes not. Accordingly, it is impossible to detect all deadlocks as required in the above mentioned second requirement. This is because even the occurrence of the deadlock becomes uncertain when transmission of a communication message is delayed or the message is lost due to abnormal communication, since in the distribution system each computer system is allowed to access a common resource through the intercommunication of the systems. Likewise, for condition (d), delay or loss of a communication message between systems sometimes allows detection of deadlock and sometimes not. Accordingly, it is impossible to detect all deadlocks as required in the above mentioned second requirement. This is because management information on the tasks in the system is lost and it becomes impossible to determine the deadlock when one system is downed during operation of the other system. For condition (c), the first or the second requirement cannot be satisfied when the transaction waiting relation is changed during detection of the deadlocks because information regarding which task locks which resource is lost. However, to satisfy condition (c) means to forbid causing of any fresh transactions (tasks) or causing of waiting relation during detection of the deadlocks. In other words, acceptance of any request in the system should be suspended temporarily to detect the deadlocks. Accordingly, if a task issues a request for a resource (z) that has no relation with the deadlock, the request should be suspended. As a result, pursuit of the condition (c) inhibits a smooth and effective operation of the system and the third requirement will not be satisfied. It is impossible in practice to avoid detection of the phantom deadlocks concerning with the condition (a). In other words, it is impossible to predict and avoid abnormal states that could be caused in individual systems and thus the asynchronous abort of the task cannot be avoided. SUMMARY OF THE INVENTION Accordingly, an object of the present invention is to reduce effects on a system due to detection of deadlocks by means of continuing detection of the deadlock even when a transaction waiting relation is changed during detection of the deadlock. In a distribution system, an object is to detect all deadlocks without detecting a phantom deadlock, reduce an overhead time in communication as much as possible, and reduce effects on the system due to detection of the deadlocks. Detection continues even when delay or loss of communication message between systems are caused, when a transaction waiting relation is changed during detection of the deadlock and when an asynchronous down of the system is caused. To achieve these objects, the present invention is provided with a deadlock detecting device for detecting, in a multitasking system where a plurality of tasks use a common resource, deadlocks in which tasks are each awaiting resources locked by the other(s) and unable to carry out further processing. The deadlock detecting device comprises a task manager for managing execution of the tasks to ensure parallel execution of two or more tasks; a lock manager for managing which resource is locked by the task; a waiting relation registering unit for registering, when one task asks for a resource locked by the other task, a "waiting relation" indicating that one task is awaiting for the other task; and a deadlock detector for detecting the deadlocks based on this "waiting relation" registered in the waiting relation registering unit. The deadlock detector is operated in asynchronism with the lock manager. BRIEF DESCRIPTION OF THE DRAWING The present invention will become more fully understood from the detailed description given hereinbelow and the accompanying drawings which are given by way of illustration only, and thus are not limitative of the present invention, and wherein: FIG. 1 is a block diagram illustrating a first embodiment of the present invention; FIG. 2 is a block diagram illustrating a second embodiment of the present invention; FIG. 3 is a block diagram illustrating a third embodiment of the present invention; FIG. 4 is a view for use in describing a relation between a local wait-for-graph and a global wait-for-graph; FIG. 5 is a flow chart illustrating operation of a transaction manager in FIG. 3; FIG. 6 is a flow chart illustrating operation of a resource manager in FIG. 3; FIG. 7 is a flow chart illustrating operation of a lock manager in FIG. 3; FIG. 8 is a flow chart illustrating operation of a deadlock detector in FIG. 3; FIG. 9 is a flow chart illustrating operation of a watchdog timer in FIG. 3; and FIG. 10 is a view for use in describing a deadlock situation. DESCRIPTION OF THE PREFERRED EMBODIMENT First Embodiment A first embodiment of the present invention is shown in FIG. 1. A deadlock detecting device according to the first embodiment detects deadlocks caused in a multitasking system in which a plurality of tasks 100 use a common resource 101. A typical multitasking system comprises a task manager (TM) and a lock manager (LM) 102. The task manager (TM) 102 manages execution status of the tasks (x, y) 100 to execute a plurality of tasks (x, y) 100 in parallel. The lock manager (LM) 103 detects and manages information regarding which task 100 is locking which resource 101 or waiting to lock which task 100. In this embodiment, such a multitasking system is provided with a wait managing table (LT) 105 and a deadlock detector (DD) 104. The wait managing table (LT) 105 registers a "waiting relation" concerning the task. The deadlock detector (DD) 104 detects the deadlocks based on the "waiting relations" registered in this managing table (LT) 105. A "task" usually means one or more sequence of instructions treated as a unit of work to be accomplished by a central processing unit (CPU). In this invention, the word "task" can also be referred to as a "transaction." The "transaction" means a set of operations for a complete data operation and is a concept involved in the "task," which is executed by programs. The present invention is directed to detect the lock situation in which each program commonly uses the resource when a plurality of programs are executed in parallel at the same time. Thus, a difference in words has no significant meaning in the present invention even if any one of the terms "task," "transaction," and an execution unit to be accomplished by the programs is used. The "task" is used hereinafter as an equivalence to the "transaction," which makes no matter in executing the present invention. In the present invention, the resource 101 commonly used by the tasks 100 maybe files (a collection of data), records stored in the files, and so on. The lock used in this invention means that a task or a transaction exclusively use the entire file or exclusively uses a record in the file. In the present embodiment, the deadlocks are detected in the following manner. A lock relation between the task (transaction) 100 and the resource 101, i.e., a "waiting relation" is registered in the above mentioned wait managing table (LT) 105. The deadlock detector (DD) 104 looks up the wait managing table (LT) 105 to detect the deadlock. This detection is made independently of the task management made by the above mentioned lock manager (LM) 103. Preferably, the deadlock detector (DD) 104 is required to register the "waiting relation" in the wait managing table (LT) 105 when the above mentioned lock manager (LM) 103 detects that "a certain task is in the "waiting relation" for a certain resource." The presence or absence of the deadlock is determined by means of looking up the registered contents therein. A following method may be advantageously used as a method of registering the waiting relation of the transaction in the wait managing table (LT) 105 and for detecting the deadlock. That is, the waiting relation of the transaction is expressed by a graph. This graph is referred herein to as a wait-for-graph (WFG). This graph is registered in the above mentioned wait managing table (LT) 105. In this graph, the transaction x caused in the system i is defined as T(i, x) while the transaction y caused in the system j is defined as T(j, y). In addition, waiting of T(i, x) for T(j, y), i.e., waiting of T(i, x) for release of the resource 101 locked by T(j, y) is represented herein by: T(i, x)→T(j, y). In this event, T(i, x) is unable to proceed with further operation until T(j, y) is terminated. When T(i, x)→T(j, y) and T(j, y)→T(i, x) are held at the same time, a loop of T(i, x)→T(j, y)→T(i, x) is formed. This case corresponds to the deadlock situation, so that detection of this loop allows detection of the deadlock. The above description is for the detection of the deadlock between two systems. The deadlock in the own system is represented by: T(i, x)→T(i, y)→T(i, x). A feature of this embodiment lies in a point that the deadlock detector (DD) 104 is provided in the systems (i, j) independently of a block such as the task manager (TM) 102 or the lock manager (LM) 103 required for executing the tasks to be operated in asynchronism with the task manager (TM) 102 and the lock manager (LM) 103. In a conventional deadlock detecting method, execution of the individual tasks 101 is temporarily stopped to detect the deadlock, during which the presence or absence of the deadlock is determined according to lock information in the lock manager (LM) 103. It is, however, impossible to ensure a smooth operation of the tasks 100. On the contrary, in the present embodiment the wait managing table (LT) 105 is provided independently of the task manager (TM) 102 and the lock manager (LM) 103, and the lock information, i.e. a "lock relation" or "waiting relation" produced by the above mentioned lock manager (LM) 103 is registered in the wait managing table (LT) 105. In addition, the deadlock detector (DD) 104 which operates independently of the execution of the tasks 100 is provided. The deadlock detector (DD) 104 determines the presence or absence of the deadlock according to the registered contents in the above mentioned wait managing table (LT) 105 in consideration of the waiting relation of the task when the lock manager (LM) 103 receives information indicating that the task 100 comes to a situation for waiting the resource 101. As mentioned above, in the present embodiment, the deadlock detector (DD) 104 is provided independently of the system such as the task manager (TM) 102 and the lock manager (LM) 103 (required for executing the task 100). The deadlock detector (DD) 104 is operated independently, so that it is not necessary to suspend the execution of the task 100 for the deadlock detection. Detection of the deadlocks by the deadlock detector (DD) 104 is preferably made when the waiting relation of the task 100 is detected by the above mentioned lock manager (LM) 103. More specifically, the deadlock detector (DD) 104 registers the waiting relation in the managing table (LT) 105 when the lock manager (LM) 103 detects the waiting relation of the task 100. This registration is used as a starting trigger for the deadlock detection. The deadlock detector (DD) 104 detects the presence or absence of the deadlock by means of looking up the wait managing table (LT) 105 in response to reception of the registration notification. To register the above mentioned "waiting relation" in the above mentioned wait managing table (LT) 105, it is enough to register the "waiting relation of each task 100 in the wait managing table (LT) 105 provided in a system when two or more tasks (x, y) 100 are present in the same system. On the other hand, if the task 100 in one system is in the "waiting" state for the task in the other system, this "waiting relation" is notified from one system to the wait managing table (LT) 105 of the other system. The wait managing table (LT) 105 registers this "waiting relation" in response to this notification. At the same time as this registration, the deadlock detector (DD) 104 of the other system looks up the wait managing table (LT) 105 to determine the presence or absence of the deadlock. It is also possible to notify the wait managing table (LT) 105 of one system from the other system of this "waiting relation" to determine the presence or absence of the deadlock by means of looking up this wait managing table (LT) 105. The above mentioned description is for a case where the "waiting relation" of the task 100 in the own system and the "waiting relation" of the task 100 in the waited system are both registered in the wait managing table (LT) 105 of the own system. On the contrary, only the "waiting relation" of the own system may be registered in the own managing table (LT) 105. In such a case, the wait managing table (LT) 105 of the waited system is accessed through communication to detect the deadlock. The "waiting relation" of the task 100 in the own system is compared with the "waiting relation" of the task 100 in the waited system. If the above mentioned loop is formed, this is detected as the deadlock. According to this embodiment, communication of the information among two or more systems for detecting the deadlock is not made to detect the deadlock in the own system. The communication of the information is made only when the deadlock is caused among two or more systems. In most cases, the deadlock is caused between two systems, so that the deadlock can be detected through one communication. In this embodiment, the deadlock detector (DD) 104 is independent of and separated from the task manager (TM) 102 and lock manager (LM) 103, and thus the detection of the deadlocks will never affect the execution of the tasks 100. When the deadlock is detected, it is necessary for restoring from the deadlock to forcibly abort either task 100. The systems are used to determine which task is forcedly aborted. For example, the task 100 starting time of which is later than the others is considered to have less amount of work and this task 100 is aborted. Alternatively, the amount of work may be calculated actually to abort the task 100 having the less amount of work. Second Embodiment A second embodiment of the present invention is shown in FIG. 2. A deadlock detecting device according to the second embodiment detects the deadlocks caused in the multitasking system in which a plurality of tasks 200 use a common resource 201. This second embodiment is characterized by comprising a watchdog timer (WT) 206 along with the components described in the first embodiment. The watchdog timer (WT) 206 is a block for issuing a resource 201 gaining request again for the task (transaction) 200 in the "waiting relation" for a given period of time. The watchdog timer (WT) 206 is provided for following reasons. It is impossible to restore from the deadlock when the deadlock, even if caused, is not detected. The reason why the deadlock that is actually caused cannot be detected may be that information of the "waiting relation" is not registered in the managing table (LT) 205 due to loss of communication. With this respect, the watchdog timer (WT) 206 issues the resource gaining request again when the "waiting relation" is continued for a certain period of time for the task 200. As a result, there is a chance of transmitting again the information of the "waiting relation" to the wait managing table (LT) 205 in the waited system, and thus the deadlock can be detected positively. Third Embodiment FIG. 3 shows a third embodiment of the present invention. In this embodiment, the word "transaction" is used rather than the "task" that is used above. This third embodiment is a specific example of a case where the present invention is carried out in a distribution system. <General Structure of the System> FIG. 3 shows structure of a distribution system. In this distribution system, two computer systems (system i and system j) are provided in a distributed manner and are connected to each other through a network (NW) 30. In addition, a database (DB) 20 is connected to the computer systems i, j through the network (NW) 30. The computer systems are allowed access to the database (DB) 20. Such a system is applicable to, for example, an account system of a bank. As apparent from FIG. 3, each computer system (system i, j) comprises a transaction manager (TM) 10, a resource manager (RM) 11, the lock manager (LM) 12, the deadlock detector (DD) 15, a wait-for-graph table T3, and the watchdog timer (WT) 13. The system j is equal in structure to the system i. Accordingly, FIG. 3 shows details for only the system i and illustration of the detailed structure of the system j is omitted. The database (DB) 20 stores a plurality of files or records as the resource. In FIG. 3, the resources A and B are provided as the resource. Each component block is described in detail. <Transaction Manager (TM)> The transaction manager (TM) 10 manages execution of two or more transactions. The transaction manager (TM) 10 can be referred to as the task manager (TM). The transaction manager (TM) 10 is a block that receives communication indicative of start, commit, abort of the transaction from an application program to manage the transactions in the system. More specifically, when the transaction x is started in the system i, the data in the form of T(i, x) is registered while this data in the form of T(i, x) is deleted when the transaction x is terminated or aborted. The transaction manager (TM) 10 passes, in response to acceptance of a request for the resource from the transaction, the request to the resource manager (RM) 11 and receives a response (OK/NO) thereof. The transaction manager (TM) 10 also accepts a deadlock notification transmitted from the deadlock detector (DD) 15 to abort the transaction. In addition, the transaction manager (TM) 10 accepts a retry notification transmitted from the deadlock detector (DD) 15 to reissue a resource gaining request to the resource manager (RM) 11. The transaction manager (TM) 10 receives communication indicative of the commit or the abort of the transaction in the other computer system (system j). In response to this, the transaction manager (TM) 10 requests the deadlock detector (DD) 15 to register or delete the wait-for-graph. In FIG. 3, "start" means to start execution of the transaction while "abort" means to abort the transaction. "Commit" means to commit the transaction. The resource gaining request and a resource releasing request in the transaction manager (TM) 10 is executed in a two-phase lock (2PL) system. This system is effective by the consideration of avoiding detection of the phantom deadlocks. The phantom deadlock is typically caused when the registration and the deletion of the graph are competed with each other. For example, when a graph of T(i, x)→T(j, y) is already registered, it is assumed that a request for deleting T(i, x)→T(j, y) and a request for registering T(j, y)→T(i, x) are generated at the same time to the deadlock detector (DD) 15. In such a case, no deadlock is caused when the request for deletion is accepted first. On the contrary, the phantom deadlock is caused when the request for registration is accepted first. The request for deleting T(i, x)→T(j, y) is generated when the waiting relation expressed by this graph is given up (i.e., when T(j, y) releases the lock on the resource). Release of the lock is made either when the transaction itself releases the lock on the own resource or when the transaction is aborted asynchronously. Two-phase locking protocol is a locking protocol consisting of two phases; when data once locked by the transaction goes on being locked and that once unlocked goes on being unlocked. According to this protocol, the same result can be obtained as a plurality of tasks or transactions that are serially executed. Also according to this system, the same result is provided as if the tasks or transactions are executed sequentially. If the transaction is assumed not to be aborted asynchronously, this system ensures that no additional request for lock gaining will be generated once the lock is released. In other words, the graph T(i, x)→T(j, y) once generated is not deleted until T(j, y) is terminated. Accordingly, when the request for deletion of T(i, x)→T(j, y) is generated, T(j, y) has already been terminated and thus no additional request for gaining the resource is made by T(j, y). Thus, it is possible to ensure that a request for registering T(j, y)→T(i, x) is not generated. For the above mentioned reasons, it is possible to restrict the cause of the phantom deadlock to the asynchronous abort of the transaction by means of applying this system. <Resource Manager (RM)> The resource manager (RM) 11 is connected to the transaction manager (TM) 10 in a two-way manner. The resource manager (RM) 11 comprises a resource managing table T1. The resource manager (RM) 11 maps and manages on the resource managing table T1 a corresponding relation between the transaction and the resource requested by the transaction according to the contents of the resource gaining request and the resource releasing request supplied from the transaction manager (TM) 10. The resource manager (RM) 11 issues a lock request to the lock manager (LM) 12 in response to the resource gaining request supplied from the transaction manager (TM) 10. The resource manager (RM) 11 issues a lock releasing request to the lock manager (LM) 12 in response to the resource releasing request supplied from the transaction manager (TM) 10. <Lock Manager (LM)> The lock manager (LM) 12 is connected to the resource manager (RM) 11 in a two-way manner. The lock manager (LM) 12 comprises a lock managing table T2. The lock manager (LM) 12 is a control unit for controlling the lock conditions using the lock managing table T2. In a case where the transactions x, y and the resources A, B are provided and when the transaction x locks the resource A while the transaction y locks the resource B, the lock manager (LM) 12 registers this relation in the lock managing table T2. More specifically, as shown in FIG. 3, a situation where x locks A is defined as, for example, (x:A) while a situation where y locks B is defined as (y:B). The lock manager (LM) 12 registers this information in the lock managing table T2. The lock managing table is used for managing lock information for the transactions in the other computer system (j or i) as well as for managing the lock information for the transactions in the own computer system (i or j). The lock information for the transactions in any other systems can be obtained through communication between the computer systems. Alternatively, necessity for the communication between the computer systems can be eliminated by means of creating a single lock managing table T2 on a common memory that is commonly used by the computer system (i, j) and managing generally the lock information for all transactions. In the above mentioned situation, it is further assumed that the resource manager (RM) 11 issues a lock request for the resource A by the transaction y and a lock request for the resource B by the transaction x. Then, the lock manager (LM) 12 looks up the information in the lock managing table T2 to confirm that it is impossible to make such lock. In this case, the transaction y waits for the release of the lock on the resource A by the transaction x while the transaction x waits for the release of the lock on the resource B by the transaction y. These situations are defined as (x→B) and (y→A). When these definitions are generated, the lock manager (LM) 12 determines that "waiting" is caused. In this embodiment, such "waiting relation" is registered in and managed on a wait-for-graph table T3 independent of the lock manager (LM) 12. When the "waiting relation" is caused, the lock manager (LM) 12 request the deadlock detector (DD) 15 to register the graph according to the above mentioned definitions. Upon requesting this, the deadlock detector (DD) 15 is also supplied with information regarding the transaction (x, y) in the above definition, and the resources (A, B) in the above definitions are locked currently, by transaction, in the computer system. The lock manager (LM) 12 immediately returns a response (OK) to the resource manager (RM) 11 when the lock request from the resource manager (RM) 11 is expected to be locked, that is, the request is not expected to be "waiting" for another transaction. A registration requesting queue is executed for registering the wait-for-graph indicative of the waiting relation in the wait-for-graph table T3 only when the "waiting" situation is caused, according to the determination by the lock manager (LM) 12. The execution processing of the transaction which requires gain of the resource is not stopped or suspended regardless of the detection or not of the deadlocks when the request is not expected to be "waiting" for another transaction. <Deadlock Detector (DD)> The deadlock detector (DD) 15 is a portion to determine the presence or absence of the deadlock according to the contents registered in the wait-for-graph table (T3). The deadlock detector (DD) 15 comprises a request queue receiving unit (QR) 14. The request queue receiving unit (QR) 14 receives a wait-for-graph registration request queue and a wait-for-graph deletion request queue from the lock manager (LM) 12 in the own computer system i. In addition, the request queue receiving unit (QR) 14 receives the wait-for-graph registration request queue and the wait-for-graph deletion request queues from other computer systems i. Further, when the transaction in other system i is aborted or committed, the request queue receiving unit (QR) 14 receives a wait-for-graph deletion request queue from the transaction manager (TM) 10. The deadlock detector (DD) 15 registers or deletes the wait-for-graph first in or from the wait-for-graph table T3 according to these request queues. The format of this wait-for-graph is as follows. For example, the transaction x caused in the system i=T(i, x) and the transaction y caused in the system j=T(j, y), then waiting of T(i, x) for T(j, y) is represented by T(i, x)→T(j, y). To register the wait-for-graph, the deadlock detector (DD) 15 previously creates the wait-for-graph according to the information notified from the lock manager (LM) 12. When the wait-for-graph is registered, the deadlock detector (DD) 15 starts detection of the deadlock. When the deadlock is detected, the deadlock detector (DD) 15 notifies the transaction manager (TM) 10 of the deadlock. When a request for deleting the graph is received, the graph in question is deleted and the deadlock detector (DD) notifies the enable transaction of retry. <Wait-for-Graph Table (T3)> The wait-for-graph table T3 is provided with a wait-for-graph that is registered therein. As mentioned above, it is assumed that the transaction x caused in the system i=T(i, x) and the transaction y caused in the system j=T(j, y), then waiting of T(i, x) for T(j, y) is represented by T(i, x)→T(j, y). In such a case, the transaction x is not allowed to use the resource B, to be locked, until the transaction y updates the locking resource B to terminate T(j, y). This situation is referred to as the "waiting relation" in which "T(i, x) waits for T(j, y)." The deadlock detector (DD) 15 registers this graph of T(i, x)→T(j, y) in the wait-for-graph table T3 as the "waiting relation." On the other hand, this graph of T(i, x)→T(j, y) may be held at the same time when T(j, y)→T(i, x) is held. In such a case, the transaction y is not allowed to use the resource A to be locked until the transaction x updates the resource A to terminate T(i, x). A loop of T(i, x)→T(j, y)→T(j, y)→T(i, x) is formed with these two waiting relations. Accordingly, detection of this loop corresponds to the situation where the deadlock is caused. The waiting relation is caused between the transactions x of the systems i and the transaction y of the system j, respectively. The graph of T(i, x)→T(j, y) is registered in the wait-for-graph table T3 of the system i while the graph of T(j, y)→T(i, x) is registered in the wait-for-graph table T3 of the system j. Accordingly, either one of them should be transmitted to the other to compare them with each other. In this embodiment, when the "waiting relation" is registered, this "waiting relation" is transmitted to the waited system. That is, when T(i, x)→T(j, y) is registered in the wait-for-graph table T3 of the system i, the deadlock detector (DD) 15 of the system i transmits and registers the same contents to and in the wait-for-graph table T3 of system j. As a result, it becomes possible to detect the deadlock in the waited system (i.e., the system j) by means of looking up the wait-for-graph table T3. When the "waiting relation" of the transaction is given up, the information regarding the "waiting relation" on the transaction x should be recovered (deleted) from the waited system (i.e., the system j), otherwise the deadlock is detected continuously. Accordingly, it is necessary to delete or recover the graph indicating the "waiting relation" from the wait-for-graph table T3 of the waited system (i.e., the system j) when the "waiting relation" is eliminated. The deadlock detector (DD) 15 also has a function of deleting and recovering the graph. Detection of no graph of the wait-for-graph indicates that there is a possibility of the deadlock in the other system if the top end of the wait-for-graph is the transaction in the other system. Thus, the other system in question is required to register the graph. To accept a request of registering the graph issued by other systems, the deadlock detector (DD) 15 registers a necessary graph to detect the loop. This waiting relation may be caused in the own system. For example, when a transaction x1=T(i, xl) caused in the system i waits for a transaction y1=T(i, yl) caused only in the own system (i.e., the system i), T(i, xl)→T(i, yl) is registered in the wait-for-graph table T3 of the own system (i.e., the system i). In this event, if T(i, yl)→T(i, xl) is registered in the wait-for-graph table T3 of the own system (i.e., the system i), a loop of T(i, xl)→T(i, yl)→T(i, yl)→T(i, xl) is formed, according to which the deadlock can be detected. In the distribution system, the registered contents such as the wait-for-graph related to the transaction caused in a given system is referred to as a local wait-for-graph of that system. In addition, a graph representing the waiting relation in the entire distribution system, i.e., a set of all local wait-for-graphs in that distribution system is referred to as a global wait-for-graph. FIG. 4 is a view illustrating an exemplified relation between the local wait-for-graph and the global wait-for-graph. In each computer system, only the local wait-for-graph is managed on the wait-for-graph table T3. In this embodiment, as mentioned above, the deadlock detector (DD) 15 does not transmit to other computer systems the registered contents such as the graph indicating the waiting relation between the transactions in the own computer system. On the other hand, the deadlock detector (DD) 15 transmits the graph indicating the waiting relation between the transaction in the other computer system and the transaction in the own computer system only to the associated system. Considering that statistically 90% or more of the deadlocks are caused between two tasks, the deadlock involved in the transaction in the other system could be detected with one communication. In addition, the deadlock caused only in the own computer system can be detected without communication. Accordingly, it becomes possible to reduce the overhead time required for the communication to detect the deadlocks. <Watchdog Timer> The watchdog timer (WT) 13 is a timer for use in monitoring the wait-for-graph table T3. This timer 13 monitors the "waiting relation" registered in the wait-for-graph table T3. If the "waiting relation" is continued when a predetermined time has elapsed since the "waiting relation" had been registered, the watchdog timer (WT) 13 issues a retry notification to make the waiting transaction in the "waiting relation" reissue the resource gaining request. The retry notification is supplied to the request queue receiving unit (QR) 14. When the retry notification is supplied to the request queue receiving unit (QR) 14, the deadlock detector (DD) 15 sends the retry notification to the transaction manager (TM) 10. In response to this retry signal, the transaction manager (TM) 10 reissues the resource gaining request to the transaction in the waiting relation. The deadlock that is actually caused may not be detected as a result of delay or loss of a communication message between the systems or the asynchronous down of the computer system. The watchdog timer (WT) 13 is for avoiding such trouble. More specifically, the computer i, j systems perform communication with each other in the distribution system, the graph indicative of the deadlock situation may be lost due to loss of communication message between the systems. This makes the detection of the deadlock impossible. To avoid this, the watchdog timer (WT) 13 is provided as a mechanism for monitoring the transactions in the waiting relation. This timer 13 urges, as mentioned above, the transaction in the waiting relation of the wait-for-graph registered in the table T3 for a predetermined time or longer to reissue the resource gaining request through the deadlock detector (DD) 15. In response to this, the transaction manager (TM) 10 reissues the resource gaining request. If the "waiting" has given up already at that time, then the resource gaining request is accepted. On the other hand, if the waiting has not eliminated, the graph is again transmitted to the other computer system. This makes up for the loss of the graph and thus the deadlock can be detected. <Exemplified Operation in each Component> Operation in the above mentioned components is described in conjunction with the flow charts. Operation of the Transaction Manager (TM)! As shown in the flow chart of FIG. 5, the transaction manager (TM) 11 waits for a request such as the start, the abort, the commit, the resource gaining request, the deadlock notification and the retry notification (Step 101). The abort and the commit used herein includes those notified from other computer systems. When any one of the request is received (Step 102), the transaction manager (TM) 10 allocates the processing according to the type of the request. When the request received at the step 102 is start, this transaction (referred to as T(i, x) for convenience) is registered in the transaction manager (TM) 10 itself (Step 103) and the transaction manager 10 waits for a subsequent request. When the request received at the step 102 is abort or commit, the transaction to be aborted or committed (referred to as T(i, x) for convenience) is deleted (Step 104). Next, the transaction manager (TM) 10 issues the resource releasing request to the resource manager (RM) 11 (Step 105). When a response to the resource releasing request is received from the resource manager (RM) 11(Step 106), the transaction manager (TM) 10 requests the deadlock detector (DD) 15 delete the graph (Step 107). Subsequently, the transaction manager (TM) 10 determines whether the request in question is from the own computer system (Step 108). When the request is transmitted from the other computer system, no operation is made for the time. On the contrary, if the request is from the own computer system, commit or abort is notified to the other computer system (Step 109). The computer system receiving the notification carries out processing at the steps 104 through 107. When the request received at the step 102 is the resource gaining request, the transaction manager (TM) 10 supplies the resource gaining request to the resource manager (RM) 11 (Step 110). When a response to the request is received from the resource manager (RM) 11 (Step 111), the transaction manager (TM) 10 sends a response to the transaction (Step 112). When the request received at the step 102 is the deadlock notification, the transaction manager (TM) 10 first selects the transaction to be aborted from the transactions in the deadlock situation (Step 120). More specifically, all transactions in the deadlock relation (referred herein to as T(i, x), T(j, y) for convenience) are specified in the deadlock notification. The transaction manager (TM) 10 thus selects the transaction to be aborted based on the names of the transactions included in the deadlock notification. Accordingly, the transaction manager (TM) 10 is allowed to specify the transaction in the other computer system as the transaction to be aborted. Subsequently, the transaction manager (TM) 10 notifies the selected transaction that the transaction is aborted (Step 121). If the selected transaction is in the other computer system, the transaction manager (TM) 10 notifies the selected transaction that the transaction is aborted through the transaction manager (TM) 10 of the other computer system. When the request received at the step S102 is the retry notification, the transaction manager (TM) 10 first supplies the resource gaining request to the resource manager (RM) 11 (Step 130). When a response to this request is received from the resource manager (RM) 11 (Step 131), the transaction manager (TM) 10 sends a response back to the transaction (Step 131). Operation of the Resource Manager (RM)! As apparent from the flow chart shown in FIG. 6, the resource manager (RM) 11 first waits for the resource gaining request and the resource releasing request (Step 201). When either one of the requests is made and received (step 202), the resource manager (RM) 11 attempts to lock the resource shown on the resource gaining table T1 and registers the relation in the resource gaining table T1 (Step 203). In other words, the resource manager (RM) 11 registers which transaction is trying to lock which resource. Subsequently, the resource manager (RM) 11 determines whether the request is the resource gaining request or the resource releasing request (Step 204). When the request is the resource gaining request, the resource manager (RM) 11 supplies the lock gaining request to the lock manager (LM) 12 (Step 205). On the contrary, when the request is the resource releasing request, the resource manager (RM) 11 supplies the lock releasing request to the lock manager (LM) 12 (Step 206). After receiving a response (OK/NO) to the lock gaining request or the lock releasing request from the lock manager (LM) 12 (Step 207), the resource manager (RM) 11 sends a response (OK/NO) back to the transaction manager (TM) 10 (Step 208). Operation of the Lock Manager (LM)! As apparent from the flow chart shown in FIG. 7, in response to the request at the step 205 or step 206 from the resource manager (RM) 11 (Step 301), the lock manager (LM) 12 receives the request (Step 302). Subsequently, the lock manager (LM) 12 determines whether the request is the lock gaining request or the lock releasing request (Step 303). When the request is the lock gaining request, the lock manager (LM) 12 determines whether it is possible to gain the lock (Step 304). If the lock on the resource is available, the lock manager (LM) 12 registers the lock situation in the lock managing table T2 (Step 305). On the contrary, if it is impossible to gain the lock on the resource indicating that there is the "waiting relation," the lock manager (LM) 12 requests the deadlock detector (DD) 15 to register the relation between the requesting transaction and the waiting transaction as a WFG graph in the wait-for-graph table T3 (Step 306). Subsequently, the lock manager (LM) 12 notifies the resource manager (RM) 11 that it fails to lock the resource (that is, "NO") (Step 309). If the request is determined as the resource releasing request at the step 303, then the lock manager (LM) 12 deletes the registration of the lock from the lock managing table T2 (Step 307). Then the lock manager (LM) 12 sends a response (that is, "OK") indicating completion of the registration or the deletion to the resource manager (RM) 11 (Step 308). Operation of the Deadlock Detector (DD)! As apparent from the flow chart illustrated in FIG. 8, the deadlock detector (DD) 15 receives the graph registering request, the graph deleting request and the retry notification at the request queue receiving unit (QR) 14. Accordingly, in response to the request (step 401), the deadlock detector (DD) 15 picks up the request from the request queue receiving unit (QR) 14 (Step 402) to determine the type of the request (Step 403). When the request is directed to the graph registration, the deadlock detector (DD) 15 first registers the wait-for-graph in the wait-for-graph table T3 (Step 404). If it is found, as a result of searching through the wait-for-graph table T3, that the same graph has already been registered therein, the deadlock detector (DD) 15 does not register the graph at that time. Next, the deadlock detector (DD) 15 follows the registered graph up to the top end thereof (Step 405). According to the result of the follow-up, the deadlock detector (DD) 15 determines whether a loop is formed (Step 406). If the loop is formed, the deadlock detector (DD) 15 notifies the transaction manager (TM) 10 of the deadlock (Step 407). If no loop is formed, the deadlock detector (DD) 15 determines whether the top end of the graph is a transaction in the own computer system (step 408). If the end is a transaction in the own computer system, then the control returns to the step 401. On the other hand, if the top end of the graph is a transaction in other computer system, the deadlock detector (DD) 15 sends the graph in question to the deadlock detector (DD) 15 of the other computer system to make it register the graph in the wait-for-graph table T3 of the other computer system (Step 409). Next, when the request at the step 403 is directed to delete the graph, the deadlock detector (DD) 15 searches through the wait-for-graph table T3 to find the graph in question (Step 410). When the graph in question is found, the deadlock detector (DD) 15 deletes this graph (Step 411). Subsequently, the deadlock detector (DD) 15 supplies the retry notification to the transaction manager (TM) 10 to operate the transaction for which the waiting relation is eliminated (Step 412). When the request at the step 403 is the retry request, the deadlock detector (DD) 15 deletes the graph of the transaction to be retried (Step 420). Then, the deadlock detector (DD) 15 supplies the retry notification to the transaction manager (TM) 10 (Step 421). Operation of the Watchdog Timer (WT)! As apparent from the flow chart illustrated in FIG. 9, the watchdog timer (WT) 13 successively searches the transactions (such as T(i, x)) registered in the wait-for-graph table T3 (Step 501). Subsequently, the watchdog timer (WT) 13 determines whether the transaction x is in the "waiting relation" (Step 502). If the searched transaction is not in the "waiting relation," the step 501 is again executed. On the other hand, if the searched transaction is in the "waiting relation," the watchdog timer (WT) 13 starts to count the time. When a predetermined time is elapsed (Step 503), the watchdog timer (WT) 13 supplies the retry notification to the deadlock detector (DD) 15 (Step 504). When the "waiting relation" is eliminated at the step 503 before the predetermined time is elapsed, the step 501 is again executed (Step 502). <Specific Examples of the Deadlock Detection> Next, examples of the deadlock detection carried out by the above mentioned components are described for a three different cases. Example 1 Detection of the Deadlock Between the Transactions Both of Which are in the Own Computer System! An example 1 is an example of deadlock detection between transactions which are both in the own computer system. More specifically, the following operation is carried out. This example reveals that the communication is not established with the other computer system. (1) First, it is assumed that transaction T(l, 1) notifies the transaction manager (TM) 10 of initiation of the transaction. (2) The transaction manager (TM) 10 registers transaction T(1, 1). (3) On the other hand, it is assumed that transaction T(1, 2) notifies the transaction manager (TM) 10 of initiation of the transaction. (4) The transaction manager (TM) 10 registers transaction T(1, 2). (5) It is assumed that transaction T(1, 1) asks for the resource A to the transaction manager (TM) 10. (6) The transaction manager (TM) 10 then requests for gaining the resource A to the resource manager (RM) 11. (7) The resource manager (RM) 11 requests for gaining the lock on the resource A to the lock manager (LM) 12. (8) The lock manager (LM) 12 sends OK back to the resource manager (RM) 11 if the resource A is not locked. (9) The resource manager (RM) 11 sends a response OK to the transaction manager (TM) 10. (10) On the other hand, it is assumed that transaction T(1, 2) asks for the resource B to the transaction manager (TM) 10. (11) The transaction manager (TM) 10 then requests for gaining the resource B to the resource manager (RM) 11. (12) The resource manager (RM) 11 requests for gaining the lock on the resource B to the lock manager (LM) 12. (13) The lock manager (LM) 12 sends OK back to the resource manager (RM) 11 if the resource B is not locked. (14) The resource manager (RM) 11 sends a response OK to the transaction manager (TM) 10. (15) At that time, it is assumed that transaction T(1, 1) asks for the resource B to the transaction manager (TM) 10. (16) The transaction manager (TM) 10 then requests for gaining the resource B to the resource manager (RM) 11. (17) The resource manager (RM) 11 requests for gaining the lock on the resource B to the lock manager (LM) 12. (18) However, the resource B has already been locked by transaction T(1, 2), so that transaction T(1, 1) waits for transaction T(l, 2). Accordingly, the lock manager (LM) 12 requests the deadlock detector (DD) 15 for registering the graph T(1, 1)→T(1, 2). The deadlock detector (DD) 15 receive the request and registers the graph in the wait-for-graph table T3. (19) On the other hand, it is assumed that transaction T(1, 2) asks for the resource A to the transaction manager (TM) 10. (20) The transaction manager (TM) 10 then requests for gaining the resource A to the resource manager (RM) 11. (21) The resource manager (RM) 11 requests for gaining the lock on the resource A to the lock manager (LM) 12. (22) However, the resource A has already been locked by transaction T(1, 1), so that transaction T(1, 2) waits for transaction T(1, 1). Accordingly, the lock manager (LM) 12 requests the deadlock detector (DD) 15 for registering the graph T(1, 2)→T(1, 1). The deadlock detector (DD) 15 receives the request and registers the graph in the wait-for-graph table T3. (23) The deadlock detector (DD) 15 detects the loop and notifies the transaction manager (TM) 10 of the presence of the deadlock. Example 2 Detection of the Deadlock Between Two Transactions each of which is in either of Two Computer Systems Respectively! An example 2 involves in a case where the deadlock is caused between two computer systems (system 1 and system 2 ). This example reveals that the communication for the deadlock detection is made only at once. (1) First, it is assumed that transaction T(1, 1) of the system 1 notifies the transaction manager (TM) 10 of the system 1 of initiation of the transaction. (2) The transaction manager (TM) 10 of the system 1 registers transaction T(1, 1). (3) It is assumed that transaction T(1, 1) asks for the resource A to the transaction manager (TM) 10 of the system 1. (4) The transaction manager (TM) 10 of the system 1 then requests for gaining the resource A to the resource manager (RM) 11. (5) The resource manager (RM) 11 of the system 1 requests for gaining the lock on the resource A to the lock manager (LM) 12. (6) The lock manager (LM) 12 of the system 1 sends OK back to the resource manager (RM) 11 if the resource A is not locked. (7) The resource manager (RM) 11 of the system 1 sends a response OK to the transaction manager (TM) 10. (1)' On the other hand, it is assumed that transaction T(2, 1) of the system 2 notifies the transaction manager (TM) 10 of the system 2 of initiation of the transaction. (2)' The transaction manager (TM) 10 of the system 2 registers transaction T(2, 1). (3)' It is assumed that transaction T(2, 1) asks for the resource B to the transaction manager (TM) 10 of the system 2. (4)' The transaction manager (TM) 10 of the system 2 then requests for gaining the resource B to the resource manager (RM) 11. (5)' The resource manager (RM) 11 of the system 2 requests for gaining the lock on the resource B to the lock manager (LM) 12. (6)' The lock manager (LM) 12 of the system 2 sends OK back to the resource manager (RM) 11 if the resource B is not locked. (7)' The resource manager (RM) 11 of the system 2 sends a response OK to the transaction manager (TM) 10. (8) At that time, it is assumed that transaction T(1, 1) asks for the resource B to the transaction manager (TM) 10 of the system 1. (9) The transaction manager (TM) 10 of the system 1 then requests for gaining the resource B to the resource manager (RM) 11. (10) The resource manager (RM) 11 of the system 1 requests for gaining the lock on the resource B to the lock manager (LM) 12. (11) However, the resource B has already been locked by transaction T(2, 1) of the system 2, so that transaction T(1, 1) waits for transaction T(2, 1). Accordingly, the lock manager (LM) 12 of the system 1 requests the deadlock detector (DD) 15 for registering the graph T(1, 1)→T(2, 1). (12) In response to this request, the deadlock detector (DD) 15 of the system 1 sends the graph T(1, 1)→T(2, 1) to the system 2. At the same time, the deadlock detector (DD) 15 of the system 1 registers the graph in the wait-for-graph table T3 of the system 1. (13) The deadlock detector (DD) 15 of the system 2 receives the graph T(1, 1)→T(2, 1) and registers it in the wait-for-graph table T3 of the system 2. (14) Subsequently, it is assumed that transaction T (2, 1) asks for the resource A to the transaction manager (TM) 10 of the system 2. (15) The transaction manager (TM) 10 of the system 2 then requests for gaining the resource A to the resource manager (RM) 11. (16) The resource manager (RM) 11 of the system 2 requests for gaining the lock on the resource A to the lock manager (LM) 12. (17) However, the resource A has already been locked by transaction T(1, 1) of the system 1, so that transaction T(2, 1) waits for transaction T(1, 1). Accordingly, the lock manager (LM) 12 of the system 2 requests the deadlock detector (DD) 15 for registering the graph T(2, 1)→T(1, 1). (18) The deadlock detector (DD) 15 of the system 2 detects the loop and notifies the transaction manager (TM) 10 of the system 2 of the presence of the deadlock. Example 3 Message is Lost During Detection of the Deadlock Between Two Transactions Each of Which is in Either of Two Computer Systems Respectively! An example 3 involves in a case where a massage is lost due to a communication error caused between two computer systems (system 1 and system 2). (1) First, it is assumed that transaction T(1, 1) of the system 1 notifies the transaction manager (TM) 10 of the system 1 of initiation of the transaction. (2) The transaction manager (TM) 10 of the system 1 registers transaction T (1, 1). (3) It is assumed that transaction T(1, 1) asks for the resource A to the transaction manager (TM) 10 of the system 1. (4) The transaction manager (TM) 10 of the system 1 then requests for gaining the resource A to the resource manager (RM) 11. (5) The resource manager (RM) 11 of the system 1 requests for gaining the lock on the resource A to the lock manager (LM) 12. (6) The lock manager (LM) 12 of the system 1 sends OK back to the resource manager (RM) 11 if the resource A is not locked. (7) The resource manager (RM) 11 of the system 1 sends a response OK to the transaction manager (TM) 10. (1)' On the other hand, it is assumed that transaction T(2, 1) of the system 2 notifies the transaction manager (TM) 10 of the system 2 of initiation of the transaction. (2)' The transaction manager (TM) 10 of the system 2 registers transaction T (2, 1). (3)' It is assumed that transaction T(2, 1) asks for the resource B to the transaction manager (TM) 10 of the system 2. (4)' The transaction manager (TM) 10 of the system 2 then requests for gaining the resource B to the resource manager (RM) 11. (5)' The resource manager (RM) 11 of the system 2 requests for gaining the lock on the resource B to the lock manager (LM) 12. (6)' The lock manager (LM) 12 of the system 2 sends OK back to the resource manager (RM) 11 if the resource B is not locked. (7)' The resource manager (RM) 11 of the system 2 sends a response OK to the transaction manager (TM) 10. (8) At that time, it is assumed that transaction T(1, 1) asks for the resource B to the transaction manager (TM) 10 of the system 1. (9) The transaction manager (TM) 10 of the system 1 then requests for gaining the resource B to the resource manager (RM) 11. (10) The resource manager (RM) 11 of the system 1 requests for gaining the lock on the resource B to the lock manager (LM) 12. (11) However, the resource B has already been locked by transaction T(2, 1) of the system 2, so that transaction T(1, 1) waits for transaction T(2, 1). Accordingly, the lock manager (LM) 12 of the system 1 requests the deadlock detector (DD) 15 for registering the graph T(1, 1)→T(2, 1). (12) In response to this request, the deadlock detector (DD) 15 of the system 1 registers the graph in the wait-for-graph table T3 of the system 1. At the same time, the deadlock detector (DD) 15 of the system 1 sends the graph T(l, 1)→T(2, 1) to the system 2. (13) However, the transmitted content does not arrive to the system 2 because it is lost as a result of an communication error. (14) Subsequently, it is assumed that transaction T(2, 1) asks for the resource A to the transaction manager (TM) 10 of the system 2. (15) The transaction manager (TM) 10 of the system 2 then requests for gaining the resource A to the resource manager (RM) 11. (16) The resource manager (RM) 11 of the system 2 requests for gaining the lock on the resource A to the lock manager (LM) 12. (17) However, the resource A has already been locked by transaction T(1, 1) of the system 1, so that transaction T(2, 1) waits for transaction T(1, 1). Accordingly, the lock manager (LM) 12 of the system 2 requests the deadlock detector (DD) 15 for registering the graph T(2, 1)→T(1, 1). At that time, the deadlock situation is caused actually. However, the deadlock cannot be detected due to the loss of the message. Accordingly, the deadlock is continued. (18) After a predetermined time, the watchdog timer (WT) 13 of the system 1 is operated to make a retry notification to the deadlock timer (DD) 15 of the system 1. (19) The deadlock timer (DD) 15 of the system 1 then notifies the transaction manager (TM) 10 of the retry of transaction T(1,1). (20) According to the retry notification, the transaction manager (TM) 10 of the system 1 requests the resource manager (RM) 11 for gaining the resource B. (21) The resource manager (RM) 11 of the system 1 again requests for gaining the lock on the resource B to the lock manager (LM) 12. (22) However, the resource B has already been locked by transaction T(2, 1) of the system 2, so that transaction T(1, 1) waits for transaction T(2, 1). Accordingly, the lock manager (LM) 12 of the system 1 again requests the deadlock detector (DD) 15 for registering the graph T(1, 1)→T(2, 1). (23) In response to this request, the deadlock detector (DD) 15 of the system 1 registers the graph in the wait-for-graph table T3 of the system 1. At the same time, the deadlock detector (DD) 15 of the system 1 again sends the graph T(1, 1)→T(2, 1) to the system 2. (24) The deadlock detector (DD) 15 of the system 2 receives the graph T(1, 1)→T(2, 1) and registers it in the wait-for-graph table T3 of the system 2. As a result, the loss of the graph can be made up. (25) The deadlock detector (DD) 15 of the system 2 detects the loop and notifies the transaction manager (TM) 10 of the system 2 of the presence of the deadlock. As mentioned above, according to this embodiment, the lock manager (LM) 12 for managing the lock situation on the resource by the task (transaction) is separated from the deadlock detector (DD) 15 and they are operated in an asynchronous manner with each other. Accordingly, when the task (transaction) is generated additionally and requests the resource, it is allowed to be operated without passing through the deadlock detector (DD) if the lock can be obtained without waiting. This contributes to a smooth and effective operation of the system, increasing the processing speed. In addition, even when the lock cannot be obtained, the effect thereof is less significant because the registration of the lock situation (graph) and the detection of the deadlock are made in asynchronism with the request for the lock. In particular, the effect of the deadlock detection on the system designed for reducing the deadlock becomes extremely insignificant. When the present invention is applied to the distribution system, the system establishes communication for the deadlock only when it comes under the waiting relation with the other system. Accordingly, the communication is not established for the deadlock detection in the own system. Even when the other system is involved in the deadlock with the own system, the deadlock could be detected one communication because 90% or more of the deadlocks is caused between two systems. Accordingly, it becomes possible to reduce the overhead time required for the communication to detect the deadlocks, allowing an effective operation of the system. In addition, if a waiting time monitoring unit (WT) 13 is provided with the present invention, the watchdog timer (WT) 13 give another chance to detect the deadlock even if the message is delayed or lost during communication of the message in the distribution system. Thus, all deadlocks can be detected positively. The invention being thus described, it will be obvious that the same may be varied in many ways. Such variations are not to be regarded as a departure from the spirit and scope of the invention, and all such modifications as would be obvious to one skilled in the art are intended to be included within the scope of the following claims. Patent Citations Cited PatentFiling datePublication dateApplicantTitle US4189771 *11 Oct 197719 Feb 1980International Business Machines CorporationMethod and means for the detection of deadlock among waiting tasks in a multiprocessing, multiprogramming CPU environment US4403285 *8 Dec 19806 Sep 1983Fujitsu LimitedSystem for automatically releasing a dead lock state in a data processing system US4494193 *30 Sep 198215 Jan 1985At&T Bell LaboratoriesDeadlock detection and resolution scheme US4791554 *7 Apr 198613 Dec 1988Hitachi, Ltd.Method and apparatus for preventing deadlock in a data base management system US5016167 *21 Dec 198714 May 1991Amdahl CorporationResource contention deadlock detection and prevention US5161227 *13 Nov 19893 Nov 1992International Business Machines CorporationMultilevel locking system and method US5274809 *3 Sep 199228 Dec 1993Hitachi, Ltd.Task execution control method for a multiprocessor system with enhanced post/wait procedure US5285528 *22 Feb 19918 Feb 1994International Business Machines CorporationData structures and algorithms for managing lock states of addressable element ranges US5377351 *7 Jun 199127 Dec 1994Oki Electric Industry Co., Ltd.Device for controlling multiple transactions contending concurrently for the same resource in a distributed database system US5440743 *13 Jun 19948 Aug 1995Fujitsu LimitedDeadlock detecting system US5442763 *6 Dec 199415 Aug 1995International Business Machines CorporationSystem and method for preventing deadlock in multiprocessor multiple resource instructions JPH0277868A * Title not available JPH0277960A * Title not available JPS6352263A * Title not available JPS62208139A * Title not available Non-Patent Citations Reference 1 *Computer, vol. 11, No. 11, Nov. 1989, Long Beach US, pp. 37 38, M. Singhal, Deadlock detection in distributed systems. . 2Computer, vol. 11, No. 11, Nov. 1989, Long Beach US, pp. 37-38, M. Singhal, "Deadlock detection in distributed systems.". 3 *IBM Technical Disclosure Bulletin, vol. 16, No. 10, Mar. 1974, New York US, pp. 3471 3481, A. Chandra et al, Communication protocol for deadlock detection in computer networks. . 4IBM Technical Disclosure Bulletin, vol. 16, No. 10, Mar. 1974, New York US, pp. 3471-3481, A. Chandra et al, "Communication protocol for deadlock detection in computer networks.". Referenced by Citing PatentFiling datePublication dateApplicantTitle US6009269 *10 Mar 199728 Dec 1999Digital Equipment CorporationDetecting concurrency errors in multi-threaded programs US6292488 *22 May 199818 Sep 2001Compaq Computer CorporationMethod and apparatus for resolving deadlocks in a distributed computer system US6400960 *13 Jul 20014 Jun 2002Lucent Technologies Inc.Power control of a communication traffic channel with discontinued transmission in the presence of a primary traffic channel US6434591 *17 Jun 199913 Aug 2002Kabushiki Kaisha ToshibaThread control system and method in a computer system US6546429 *21 Sep 19988 Apr 2003International Business Machines CorporationNon-uniform memory access (NUMA) data processing system that holds and reissues requests at a target processing node in response to a retry US6571270 *15 Mar 199927 May 2003International Business Machines CorporationTimeout detection facility US666220419 Jun 20029 Dec 2003Kabushiki Kaisha ToshibaThread control system and method in a computer system US668124112 Aug 199920 Jan 2004International Business Machines CorporationResource contention monitoring employing time-ordered entries in a blocking queue and waiting queue US6681242 *10 Jan 200020 Jan 2004Sun Microsystems, Inc.Method and apparatus for detecting dependency cycles between resources in a computer system US672177512 Aug 199913 Apr 2004International Business Machines CorporationResource contention analysis employing time-ordered entries in a blocking queue and waiting queue US6807540 *31 May 200119 Oct 2004International Business Machines CorporationSystem and method for deadlock management in database systems with demultiplexed connections US689860016 May 200224 May 2005International Business Machines CorporationMethod, system, and program for managing database operations US6983461 *7 Feb 20023 Jan 2006International Business Machines CorporationMethod and system for deadlock detection and avoidance US7284023 *28 Dec 200116 Oct 2007Hitachi, Ltd.Database management method and apparatus for addressing to transaction completion from/in a stored program written in an interpreted language and an implementing program therefor US72899921 May 200330 Oct 2007International Business Machines CorporationMethod, system, and program for lock and transaction management US7437727 *21 Mar 200214 Oct 2008Network Appliance, Inc.Method and apparatus for runtime resource deadlock avoidance in a raid system US7487152 *31 May 20003 Feb 2009International Business Machines CorporationMethod for efficiently locking resources of a global data repository US74900881 Sep 200410 Feb 2009International Business Machines CorporationApparatus, system, and method for preserving connection/position data integrity during file server serialization reinitialization US7496574 *1 May 200324 Feb 2009International Business Machines CorporationManaging locks and transactions US7627578 *1 Sep 20041 Dec 2009International Business Machines CorporationApparatus, system, and method for file system serialization reinitialization US76854628 Jan 200823 Mar 2010Netapp, Inc.Technique for coherent suspension of I/O operations in a RAID subsystem US77117211 Sep 20044 May 2010International Business Machines CorporationApparatus, system, and method for suspending a request during file server serialization reinitialization US784458517 Aug 200730 Nov 2010International Business Machines CorporationMethod, system, and program for managing locks and transactions US7844946 *26 Sep 200630 Nov 2010Intel CorporationMethods and apparatus to form a transactional objective instruction construct from lock-based critical sections US787011113 Jul 200711 Jan 2011International Business Machines CorporationMethod, system, and program for lock and transaction management US79260586 Feb 200812 Apr 2011Mba Sciences, Inc.Resource tracking method and apparatus US7962615 *7 Jan 201014 Jun 2011International Business Machines CorporationMulti-system deadlock reduction US80514236 Feb 20081 Nov 2011Mba Sciences, Inc.System and method for tracking resources during parallel processing US81084512 Feb 200931 Jan 2012International Business Machines CorporationSystem and program products for efficiently locking resources of a global data repository US816101826 Jun 200817 Apr 2012International Business Machines CorporationManaging locks and transactions US8196140 *11 Jan 20085 Jun 2012Microsoft CorporationService function redirection for avoiding function evaluation blockages US82006436 Dec 201012 Jun 2012International Business Machines CorporationLock and transaction management US84951318 Oct 200223 Jul 2013International Business Machines CorporationMethod, system, and program for managing locks enabling access to a shared resource US85337281 Nov 201110 Sep 2013Mba Sciences, Inc.Resource tracking method and apparatus US8626904 *9 Mar 20117 Jan 2014Symantec CorporationDetecting and reporting livelocks in a computer US876890512 Mar 20121 Jul 2014International Business Machines CorporationManaging locks and transactions US897773018 Nov 201010 Mar 2015International Business Machines CorporationMethod and system for reducing message passing for contention detection in distributed SIP server environments US9052967 *30 Jul 20109 Jun 2015Vmware, Inc.Detecting resource deadlocks in multi-threaded programs by controlling scheduling in replay US910450215 Dec 201211 Aug 2015International Business Machines CorporationManaging resource pools for deadlock avoidance US20040068563 *8 Oct 20028 Apr 2004International Business Machines CorporationMethod, system, and program for managing locks enabling access to a shared resource US20040220913 *1 May 20034 Nov 2004International Business Machines CorporationMethod, system, and program for lock and transaction management US20040220933 *1 May 20034 Nov 2004International Business Machines CorporationMethod, system, and program for managing locks and transactions US20050283227 *24 Jan 200522 Dec 2005Ev3 Peripheral, Inc.Stent with dual support structure US20120030657 *2 Feb 2012Qi GaoMethod and system for using a virtualization system to identify deadlock conditions in multi-threaded programs by controlling scheduling in replay Classifications U.S. Classification718/107, 718/104 International ClassificationG06F9/46 Cooperative ClassificationG06F9/524, G06F9/466 European ClassificationG06F9/52D, G06F9/46T Legal Events DateCodeEventDescription 13 Jul 1999CCCertificate of correction 9 May 2002FPAYFee payment Year of fee payment: 4 5 May 2006FPAYFee payment Year of fee payment: 8 5 Jul 2010REMIMaintenance fee reminder mailed 1 Dec 2010LAPSLapse for failure to pay maintenance fees 18 Jan 2011FPExpired due to failure to pay maintenance fee Effective date: 20101201
{ "url": "http://www.google.co.uk/patents/US5845117", "source_domain": "www.google.co.uk", "snapshot_id": "crawl=CC-MAIN-2015-35", "warc_metadata": { "Content-Length": "144872", "Content-Type": "application/http; msgtype=response", "WARC-Block-Digest": "sha1:IOXVLV46NP6C3LTIZC4EA4K2ITYLNNMR", "WARC-Concurrent-To": "<urn:uuid:bc07749f-aaee-486b-ad86-bda468268335>", "WARC-Date": "2015-08-31T08:52:03Z", "WARC-IP-Address": "216.58.217.131", "WARC-Identified-Payload-Type": null, "WARC-Payload-Digest": "sha1:H3SBYSYFYL2UNQ5NWWJUA6MMSLYM376I", "WARC-Record-ID": "<urn:uuid:5de54672-69b5-4adb-a249-2d01c3e6fe40>", "WARC-Target-URI": "http://www.google.co.uk/patents/US5845117", "WARC-Truncated": null, "WARC-Type": "response", "WARC-Warcinfo-ID": "<urn:uuid:a48e9ef8-055c-4282-915b-d32b25f9e38d>" }, "warc_info": "robots: classic\r\nhostname: ip-10-171-96-226.ec2.internal\r\nsoftware: Nutch 1.6 (CC)/CC WarcExport 1.0\r\nisPartOf: CC-MAIN-2015-35\r\noperator: CommonCrawl Admin\r\ndescription: Wide crawl of the web for August 2015\r\npublisher: CommonCrawl\r\nformat: WARC File Format 1.0\r\nconformsTo: http://bibnum.bnf.fr/WARC/WARC_ISO_28500_version1_latestdraft.pdf" }
{ "line_start_idx": [ 0, 56, 64, 205, 206, 214, 215, 243, 273, 295, 327, 354, 377, 402, 419, 502, 603, 628, 661, 700, 751, 777, 790, 799, 1446, 1457, 1471, 1481, 1492, 1502, 1666, 1745, 2024, 2109, 2291, 2706, 2808, 2955, 3193, 3441, 3750, 4077, 4534, 4866, 5158, 5649, 5814, 5889, 6158, 6232, 6622, 6728, 6867, 7113, 7363, 7460, 7733, 8049, 8244, 8626, 8732, 8807, 9224, 9236, 9237, 9344, 9345, 9373, 9374, 9400, 9401, 9485, 9486, 9520, 9521, 10206, 10207, 10801, 10802, 11375, 11376, 12145, 12146, 12423, 12424, 12526, 12527, 12665, 12666, 12823, 12824, 13043, 13044, 13269, 13270, 13628, 13629, 13852, 13853, 14291, 14292, 14393, 14394, 15196, 15197, 15739, 15740, 16183, 16184, 16413, 16414, 16964, 16965, 17249, 17250, 17275, 17276, 17985, 17986, 18899, 18900, 18933, 18934, 19180, 19181, 19265, 19266, 19351, 19352, 19436, 19437, 19547, 19548, 19630, 19631, 19710, 19711, 19786, 19787, 19867, 19868, 19949, 19950, 20012, 20013, 20053, 20054, 20071, 20072, 20309, 20310, 20678, 20679, 21023, 21024, 22192, 22193, 23013, 23014, 23407, 23408, 23702, 23703, 23720, 23721, 23817, 23818, 23823, 23824, 23840, 23841, 23845, 23846, 23862, 23863, 23900, 23901, 23925, 23926, 24051, 24052, 24182, 24183, 24208, 24209, 24538, 24539, 24870, 24871, 25685, 25686, 26062, 26063, 26702, 26703, 26994, 26995, 27737, 27738, 28439, 28440, 28838, 28839, 29070, 29071, 29512, 29513, 29531, 29532, 30154, 30155, 30833, 30834, 30851, 30852, 30910, 30911, 31128, 31129, 31163, 31164, 31625, 31626, 32050, 32051, 32191, 32192, 32237, 32238, 32265, 32266, 32422, 32423, 32833, 32834, 33658, 33659, 33810, 33811, 34048, 34049, 34201, 34202, 34218, 34219, 34284, 34285, 34301, 34302, 34332, 34333, 34349, 34350, 34628, 34629, 34645, 34646, 34929, 34930, 35360, 35361, 35560, 35561, 35577, 35578, 35683, 35684, 35700, 35701, 35889, 35890, 35906, 35907, 35925, 35926, 36099, 36100, 36124, 36125, 36607, 36608, 36960, 36961, 36981, 36982, 37247, 37248, 38374, 38375, 39646, 39647, 40384, 40385, 40410, 40411, 40578, 40579, 41457, 41458, 41511, 41512, 41616, 41617, 41634, 41635, 41804, 41805, 42027, 42028, 42185, 42186, 42214, 42215, 42501, 42502, 42519, 42520, 42831, 42832, 42848, 42849, 42907, 42908, 42941, 42942, 42958, 42959, 42993, 42994, 43010, 43011, 43183, 43184, 43216, 43217, 43356, 43357, 43594, 43595, 43611, 43612, 43912, 43913, 43929, 43930, 44131, 44132, 44836, 44837, 45250, 45251, 45461, 45462, 45480, 45481, 45584, 45585, 45603, 45604, 45699, 45700, 45736, 45737, 45797, 45798, 46306, 46307, 47292, 47293, 47310, 47311, 48175, 48176, 48822, 48823, 49452, 49453, 49495, 49496, 49590, 49591, 49634, 49635, 50099, 50100, 50340, 50341, 51303, 51304, 51662, 51663, 52685, 52686, 53051, 53052, 53092, 53093, 53605, 53606, 54085, 54086, 54327, 54328, 54364, 54365, 55810, 55811, 55852, 55853, 56286, 56287, 57565, 57566, 58311, 58312, 58350, 58351, 59183, 59184, 59230, 59231, 59361, 59362, 59469, 59470, 59737, 59738, 59863, 59864, 59931, 59932, 60069, 60070, 60137, 60138, 60241, 60242, 60352, 60353, 60463, 60464, 60572, 60573, 60662, 60663, 60786, 60787, 60898, 60899, 61010, 61011, 61120, 61121, 61211, 61212, 61330, 61331, 61442, 61443, 61554, 61555, 61794, 61795, 61812, 61813, 61919, 61920, 62043, 62044, 62155, 62156, 62267, 62268, 62507, 62508, 62525, 62526, 62633, 62634, 62764, 62765, 62891, 62892, 63101, 63102, 63259, 63260, 63343, 63344, 63463, 63464, 63590, 63591, 63717, 63718, 63842, 63843, 63948, 63949, 64119, 64120, 64204, 64205, 64325, 64326, 64453, 64454, 64581, 64582, 64707, 64708, 64814, 64815, 64948, 64949, 65075, 65076, 65203, 65204, 65475, 65476, 65493, 65494, 65590, 65591, 65607, 65608, 65757, 65758, 65828, 65829, 65845, 65846, 65911, 65912, 66047, 66048, 66175, 66176, 66303, 66304, 66575, 66576, 66593, 66594, 66756, 66757, 66906, 66907, 67053, 67054, 67211, 67212, 67296, 67297, 67416, 67417, 67543, 67544, 67670, 67671, 67795, 67796, 67901, 67902, 68072, 68073, 68158, 68159, 68279, 68280, 68407, 68408, 68535, 68536, 68661, 68662, 68768, 68769, 68902, 68903, 69029, 69030, 69157, 69158, 69429, 69430, 69447, 69448, 69676, 69677, 69693, 69694, 69711, 69712, 69840, 69841, 69975, 69976, 70103, 70104, 70231, 70232, 70503, 70504, 70521, 70522, 70692, 70693, 70857, 70858, 70988, 70989, 71145, 71146, 71279, 71280, 71557, 71558, 71575, 71576, 71810, 71811, 71827, 71828, 71845, 71846, 71916, 71917, 71933, 71934, 72050, 72051, 72213, 72214, 73032, 73033, 73167, 73168, 73810, 73811, 74127, 74128, 74466, 74467, 74484, 74538, 74736, 74863, 74959, 75082, 75187, 75299, 75436, 75597, 75784, 75857, 76023, 76057, 76091, 76125, 76160, 76181, 76191, 76315, 76438, 76620, 76801, 76815, 76870, 76988, 77126, 77307, 77418, 77633, 77736, 77844, 78018, 78165, 78337, 78506, 78641, 78770, 78995, 79132, 79266, 79410, 79613, 79720, 79871, 79985, 80162, 80300, 80462, 80600, 80687, 80794, 80909, 81072, 81178, 81303, 81408, 81565, 81652, 81751, 81856, 82041, 82177, 82298, 82460, 82602, 82744, 82835, 83013, 83029, 83065, 83102, 83149, 83193, 83206, 83231, 83270, 83296, 83319, 83345, 83368, 83414, 83470, 83529 ], "line_end_idx": [ 56, 64, 205, 206, 214, 215, 243, 273, 295, 327, 354, 377, 402, 419, 502, 603, 628, 661, 700, 751, 777, 790, 799, 1446, 1457, 1471, 1481, 1492, 1502, 1666, 1745, 2024, 2109, 2291, 2706, 2808, 2955, 3193, 3441, 3750, 4077, 4534, 4866, 5158, 5649, 5814, 5889, 6158, 6232, 6622, 6728, 6867, 7113, 7363, 7460, 7733, 8049, 8244, 8626, 8732, 8807, 9224, 9236, 9237, 9344, 9345, 9373, 9374, 9400, 9401, 9485, 9486, 9520, 9521, 10206, 10207, 10801, 10802, 11375, 11376, 12145, 12146, 12423, 12424, 12526, 12527, 12665, 12666, 12823, 12824, 13043, 13044, 13269, 13270, 13628, 13629, 13852, 13853, 14291, 14292, 14393, 14394, 15196, 15197, 15739, 15740, 16183, 16184, 16413, 16414, 16964, 16965, 17249, 17250, 17275, 17276, 17985, 17986, 18899, 18900, 18933, 18934, 19180, 19181, 19265, 19266, 19351, 19352, 19436, 19437, 19547, 19548, 19630, 19631, 19710, 19711, 19786, 19787, 19867, 19868, 19949, 19950, 20012, 20013, 20053, 20054, 20071, 20072, 20309, 20310, 20678, 20679, 21023, 21024, 22192, 22193, 23013, 23014, 23407, 23408, 23702, 23703, 23720, 23721, 23817, 23818, 23823, 23824, 23840, 23841, 23845, 23846, 23862, 23863, 23900, 23901, 23925, 23926, 24051, 24052, 24182, 24183, 24208, 24209, 24538, 24539, 24870, 24871, 25685, 25686, 26062, 26063, 26702, 26703, 26994, 26995, 27737, 27738, 28439, 28440, 28838, 28839, 29070, 29071, 29512, 29513, 29531, 29532, 30154, 30155, 30833, 30834, 30851, 30852, 30910, 30911, 31128, 31129, 31163, 31164, 31625, 31626, 32050, 32051, 32191, 32192, 32237, 32238, 32265, 32266, 32422, 32423, 32833, 32834, 33658, 33659, 33810, 33811, 34048, 34049, 34201, 34202, 34218, 34219, 34284, 34285, 34301, 34302, 34332, 34333, 34349, 34350, 34628, 34629, 34645, 34646, 34929, 34930, 35360, 35361, 35560, 35561, 35577, 35578, 35683, 35684, 35700, 35701, 35889, 35890, 35906, 35907, 35925, 35926, 36099, 36100, 36124, 36125, 36607, 36608, 36960, 36961, 36981, 36982, 37247, 37248, 38374, 38375, 39646, 39647, 40384, 40385, 40410, 40411, 40578, 40579, 41457, 41458, 41511, 41512, 41616, 41617, 41634, 41635, 41804, 41805, 42027, 42028, 42185, 42186, 42214, 42215, 42501, 42502, 42519, 42520, 42831, 42832, 42848, 42849, 42907, 42908, 42941, 42942, 42958, 42959, 42993, 42994, 43010, 43011, 43183, 43184, 43216, 43217, 43356, 43357, 43594, 43595, 43611, 43612, 43912, 43913, 43929, 43930, 44131, 44132, 44836, 44837, 45250, 45251, 45461, 45462, 45480, 45481, 45584, 45585, 45603, 45604, 45699, 45700, 45736, 45737, 45797, 45798, 46306, 46307, 47292, 47293, 47310, 47311, 48175, 48176, 48822, 48823, 49452, 49453, 49495, 49496, 49590, 49591, 49634, 49635, 50099, 50100, 50340, 50341, 51303, 51304, 51662, 51663, 52685, 52686, 53051, 53052, 53092, 53093, 53605, 53606, 54085, 54086, 54327, 54328, 54364, 54365, 55810, 55811, 55852, 55853, 56286, 56287, 57565, 57566, 58311, 58312, 58350, 58351, 59183, 59184, 59230, 59231, 59361, 59362, 59469, 59470, 59737, 59738, 59863, 59864, 59931, 59932, 60069, 60070, 60137, 60138, 60241, 60242, 60352, 60353, 60463, 60464, 60572, 60573, 60662, 60663, 60786, 60787, 60898, 60899, 61010, 61011, 61120, 61121, 61211, 61212, 61330, 61331, 61442, 61443, 61554, 61555, 61794, 61795, 61812, 61813, 61919, 61920, 62043, 62044, 62155, 62156, 62267, 62268, 62507, 62508, 62525, 62526, 62633, 62634, 62764, 62765, 62891, 62892, 63101, 63102, 63259, 63260, 63343, 63344, 63463, 63464, 63590, 63591, 63717, 63718, 63842, 63843, 63948, 63949, 64119, 64120, 64204, 64205, 64325, 64326, 64453, 64454, 64581, 64582, 64707, 64708, 64814, 64815, 64948, 64949, 65075, 65076, 65203, 65204, 65475, 65476, 65493, 65494, 65590, 65591, 65607, 65608, 65757, 65758, 65828, 65829, 65845, 65846, 65911, 65912, 66047, 66048, 66175, 66176, 66303, 66304, 66575, 66576, 66593, 66594, 66756, 66757, 66906, 66907, 67053, 67054, 67211, 67212, 67296, 67297, 67416, 67417, 67543, 67544, 67670, 67671, 67795, 67796, 67901, 67902, 68072, 68073, 68158, 68159, 68279, 68280, 68407, 68408, 68535, 68536, 68661, 68662, 68768, 68769, 68902, 68903, 69029, 69030, 69157, 69158, 69429, 69430, 69447, 69448, 69676, 69677, 69693, 69694, 69711, 69712, 69840, 69841, 69975, 69976, 70103, 70104, 70231, 70232, 70503, 70504, 70521, 70522, 70692, 70693, 70857, 70858, 70988, 70989, 71145, 71146, 71279, 71280, 71557, 71558, 71575, 71576, 71810, 71811, 71827, 71828, 71845, 71846, 71916, 71917, 71933, 71934, 72050, 72051, 72213, 72214, 73032, 73033, 73167, 73168, 73810, 73811, 74127, 74128, 74466, 74467, 74484, 74538, 74736, 74863, 74959, 75082, 75187, 75299, 75436, 75597, 75784, 75857, 76023, 76057, 76091, 76125, 76160, 76181, 76191, 76315, 76438, 76620, 76801, 76815, 76870, 76988, 77126, 77307, 77418, 77633, 77736, 77844, 78018, 78165, 78337, 78506, 78641, 78770, 78995, 79132, 79266, 79410, 79613, 79720, 79871, 79985, 80162, 80300, 80462, 80600, 80687, 80794, 80909, 81072, 81178, 81303, 81408, 81565, 81652, 81751, 81856, 82041, 82177, 82298, 82460, 82602, 82744, 82835, 83013, 83029, 83065, 83102, 83149, 83193, 83206, 83231, 83270, 83296, 83319, 83345, 83368, 83414, 83470, 83529, 83553 ] }
{ "red_pajama_v2": { "ccnet_original_length": 83553, "ccnet_original_nlines": 728, "rps_doc_curly_bracket": 0, "rps_doc_ldnoobw_words": 0, "rps_doc_lorem_ipsum": 0, "rps_doc_stop_word_fraction": 0.3384759724140167, "rps_doc_ut1_blacklist": 0, "rps_doc_frac_all_caps_words": 0.05150466039776802, "rps_doc_frac_lines_end_with_ellipsis": 0, "rps_doc_frac_no_alph_words": 0.2359946072101593, "rps_doc_frac_unique_words": 0.09480346739292145, "rps_doc_mean_word_length": 4.891432285308838, "rps_doc_num_sentences": 587, "rps_doc_symbol_to_word_ratio": 0, "rps_doc_unigram_entropy": 5.30915641784668, "rps_doc_word_count": 13586, "rps_doc_frac_chars_dupe_10grams": 0.24663305282592773, "rps_doc_frac_chars_dupe_5grams": 0.5103453397750854, "rps_doc_frac_chars_dupe_6grams": 0.4247686266899109, "rps_doc_frac_chars_dupe_7grams": 0.3567677438259125, "rps_doc_frac_chars_dupe_8grams": 0.30139192938804626, "rps_doc_frac_chars_dupe_9grams": 0.26950567960739136, "rps_doc_frac_chars_top_2gram": 0.020164020359516144, "rps_doc_frac_chars_top_3gram": 0.0148972999304533, "rps_doc_frac_chars_top_4gram": 0.02976449951529503, "rps_doc_books_importance": -8774.6708984375, "rps_doc_books_importance_length_correction": -8774.6708984375, "rps_doc_openwebtext_importance": -4901.4296875, "rps_doc_openwebtext_importance_length_correction": -4901.4296875, "rps_doc_wikipedia_importance": -3572.5185546875, "rps_doc_wikipedia_importance_length_correction": -3572.5185546875 }, "fasttext": { "dclm": 0.2687625288963318, "english": 0.8990500569343567, "fineweb_edu_approx": 2.5642168521881104, "eai_general_math": 0.5038243532180786, "eai_open_web_math": 0.06542176008224487, "eai_web_code": 0.5108840465545654 } }
{ "free_decimal_correspondence": { "primary": { "code": "005.1", "labels": { "level_1": "General works, books and libraries, information sciences", "level_2": "", "level_3": "Computer programming" } }, "secondary": { "code": "004.02", "labels": { "level_1": "General works, books and libraries, information sciences", "level_2": "", "level_3": "Computers and Computer science" } } }, "bloom_cognitive_process": { "primary": { "code": "2", "label": "Understand" }, "secondary": { "code": "3", "label": "Apply" } }, "bloom_knowledge_domain": { "primary": { "code": "2", "label": "Conceptual" }, "secondary": { "code": "3", "label": "Procedural" } }, "document_type_v1": { "primary": { "code": "11", "label": "Legal/Regulatory" }, "secondary": { "code": "-1", "label": "Abstain" } }, "extraction_artifacts": { "primary": { "code": "3", "label": "Irrelevant Content" }, "secondary": { "code": "-1", "label": "Abstain" } }, "missing_content": { "primary": { "code": "4", "label": "Missing Images or Figures" }, "secondary": { "code": "-1", "label": "Abstain" } }, "document_type_v2": { "primary": { "code": "8", "label": "Documentation" }, "secondary": { "code": "11", "label": "Legal Notices" } }, "reasoning_depth": { "primary": { "code": "4", "label": "Advanced Reasoning" }, "secondary": { "code": "3", "label": "Intermediate Reasoning" } }, "technical_correctness": { "primary": { "code": "4", "label": "Highly Correct" }, "secondary": { "code": "3", "label": "Mostly Correct" } }, "education_level": { "primary": { "code": "4", "label": "Graduate/Expert Level" }, "secondary": { "code": "3", "label": "Undergraduate Level" } } }
672f1e42c33a7f9846924a2431ea77df
2,551,926,446,445,132,300
/[gentoo-x86]/eclass/libtool.eclass Gentoo Contents of /eclass/libtool.eclass Parent Directory Parent Directory | Revision Log Revision Log Revision 1.97 - (show annotations) (download) Tue Dec 13 21:28:15 2011 UTC (2 years, 6 months ago) by vapier Branch: MAIN Changes since 1.96: +6 -1 lines avoid multiple inclusions when possible to speed caching up 1 # Copyright 1999-2011 Gentoo Foundation 2 # Distributed under the terms of the GNU General Public License v2 3 # $Header: /var/cvsroot/gentoo-x86/eclass/libtool.eclass,v 1.96 2011/11/18 17:32:14 vapier Exp $ 4 5 # @ECLASS: libtool.eclass 6 # @MAINTAINER: 7 # [email protected] 8 # @BLURB: quickly update bundled libtool code 9 # @DESCRIPTION: 10 # This eclass patches ltmain.sh distributed with libtoolized packages with the 11 # relink and portage patch among others 12 # 13 # Note, this eclass does not require libtool as it only applies patches to 14 # generated libtool files. We do not run the libtoolize program because that 15 # requires a regeneration of the main autotool files in order to work properly. 16 17 if [[ ${___ECLASS_ONCE_LIBTOOL} != "recur -_+^+_- spank" ]] ; then 18 ___ECLASS_ONCE_LIBTOOL="recur -_+^+_- spank" 19 20 # If an overlay has eclass overrides, but doesn't actually override the 21 # libtool.eclass, we'll have ECLASSDIR pointing to the active overlay's 22 # eclass/ dir, but libtool.eclass is still in the main Gentoo tree. So 23 # add a check to locate the ELT-patches/ regardless of what's going on. 24 ECLASSDIR_LOCAL=${BASH_SOURCE[0]%/*} 25 elt_patch_dir() { 26 local d="${ECLASSDIR}/ELT-patches" 27 if [[ ! -d ${d} ]] ; then 28 d="${ECLASSDIR_LOCAL}/ELT-patches" 29 fi 30 echo "${d}" 31 } 32 33 DESCRIPTION="Based on the ${ECLASS} eclass" 34 35 inherit multilib toolchain-funcs 36 37 # 38 # See if we can apply $2 on $1, and if so, do it 39 # 40 ELT_try_and_apply_patch() { 41 local ret=0 42 local file=$1 43 local patch=$2 44 local src=$3 45 local disp="${src} patch" 46 local log="${T}/elibtool.log" 47 48 if [[ -z ${__ELT_NOTED_TMP} ]] ; then 49 __ELT_NOTED_TMP=true 50 printf 'temp patch: %s\n' "${patch}" > "${log}" 51 fi 52 printf '\nTrying %s\n' "${disp}" >> "${log}" 53 54 # We only support patchlevel of 0 - why worry if its static patches? 55 if patch -p0 --dry-run "${file}" "${patch}" >> "${log}" 2>&1 ; then 56 einfo " Applying ${disp} ..." 57 patch -p0 -g0 --no-backup-if-mismatch "${file}" "${patch}" >> "${log}" 2>&1 58 ret=$? 59 export ELT_APPLIED_PATCHES="${ELT_APPLIED_PATCHES} ${src}" 60 else 61 ret=1 62 fi 63 64 return "${ret}" 65 } 66 67 # 68 # Get string version of ltmain.sh or ltconfig (passed as $1) 69 # 70 ELT_libtool_version() { 71 ( 72 unset VERSION 73 eval $(grep -e '^[[:space:]]*VERSION=' "$1") 74 echo "${VERSION:-0}" 75 ) 76 } 77 78 # 79 # Run through the patches in $2 and see if any 80 # apply to $1 ... 81 # 82 ELT_walk_patches() { 83 local patch tmp 84 local ret=1 85 local file=$1 86 local patch_set=$2 87 local patch_dir="$(elt_patch_dir)/${patch_set}" 88 local rem_int_dep=$3 89 90 [[ -z ${patch_set} ]] && return 1 91 [[ ! -d ${patch_dir} ]] && return 1 92 93 # Allow patches to use @GENTOO_LIBDIR@ replacements 94 local sed_args=( -e "s:@GENTOO_LIBDIR@:$(get_libdir):g" ) 95 if [[ -n ${rem_int_dep} ]] ; then 96 # replace @REM_INT_DEP@ with what was passed 97 # to --remove-internal-dep 98 sed_args+=( -e "s|@REM_INT_DEP@|${rem_int_dep}|g" ) 99 fi 100 101 pushd "$(elt_patch_dir)" >/dev/null || die 102 103 # Go through the patches in reverse order (newer version to older) 104 for patch in $(find "${patch_set}" -maxdepth 1 -type f | LC_ALL=C sort -r) ; do 105 tmp="${T}/libtool-elt.patch" 106 sed "${sed_args[@]}" "${patch}" > "${tmp}" || die 107 if ELT_try_and_apply_patch "${file}" "${tmp}" "${patch}" ; then 108 # Break to unwind w/popd rather than return directly 109 ret=0 110 break 111 fi 112 done 113 114 popd >/dev/null 115 return ${ret} 116 } 117 118 # @FUNCTION: elibtoolize 119 # @USAGE: [dirs] [--portage] [--reverse-deps] [--patch-only] [--remove-internal-dep=xxx] [--shallow] [--no-uclibc] 120 # @DESCRIPTION: 121 # Apply a smorgasbord of patches to bundled libtool files. This function 122 # should always be safe to run. If no directories are specified, then 123 # ${S} will be searched for appropriate files. 124 # 125 # If the --shallow option is used, then only ${S}/ltmain.sh will be patched. 126 # 127 # The other options should be avoided in general unless you know what's going on. 128 elibtoolize() { 129 local x 130 local do_portage="no" 131 local do_reversedeps="no" 132 local do_only_patches="no" 133 local do_uclibc="yes" 134 local deptoremove= 135 local do_shallow="no" 136 local force="false" 137 local elt_patches="install-sh ltmain portage relink max_cmd_len sed test tmp cross as-needed" 138 139 for x in "$@" ; do 140 case ${x} in 141 --portage) 142 # Only apply portage patch, and don't 143 # 'libtoolize --copy --force' if all patches fail. 144 do_portage="yes" 145 ;; 146 --reverse-deps) 147 # Apply the reverse-deps patch 148 # http://bugzilla.gnome.org/show_bug.cgi?id=75635 149 do_reversedeps="yes" 150 elt_patches+=" fix-relink" 151 ;; 152 --patch-only) 153 # Do not run libtoolize if none of the patches apply .. 154 do_only_patches="yes" 155 ;; 156 --remove-internal-dep=*) 157 # We will replace @REM_INT_DEP@ with what is needed 158 # in ELT_walk_patches() ... 159 deptoremove=${x#--remove-internal-dep=} 160 161 # Add the patch for this ... 162 [[ -n ${deptoremove} ]] && elt_patches+=" rem-int-dep" 163 ;; 164 --shallow) 165 # Only patch the ltmain.sh in ${S} 166 do_shallow="yes" 167 ;; 168 --no-uclibc) 169 do_uclibc="no" 170 ;; 171 --force) 172 force="true" 173 ;; 174 *) 175 eerror "Invalid elibtoolize option: ${x}" 176 die "elibtoolize called with ${x} ??" 177 esac 178 done 179 180 [[ ${do_uclibc} == "yes" ]] && elt_patches+=" uclibc-conf uclibc-ltconf" 181 182 case ${CHOST} in 183 *-aix*) elt_patches+=" hardcode aixrtl aix-noundef" ;; #213277 184 *-darwin*) elt_patches+=" darwin-ltconf darwin-ltmain darwin-conf" ;; 185 *-freebsd*) elt_patches+=" fbsd-conf fbsd-ltconf" ;; 186 *-hpux*) elt_patches+=" hpux-conf deplibs hc-flag-ld hardcode hardcode-relink relink-prog no-lc" ;; 187 *-irix*) elt_patches+=" irix-ltmain" ;; 188 *-mint*) elt_patches+=" mint-conf" ;; 189 esac 190 191 if $(tc-getLD) --version 2>&1 | grep -qs 'GNU gold'; then 192 elt_patches+=" gold-conf" 193 fi 194 195 # Reuse "$@" for dirs to patch 196 set -- 197 if [[ ${do_shallow} == "yes" ]] ; then 198 [[ -f ${S}/ltmain.sh ]] && set -- "${S}" 199 else 200 set -- $(find "${S}" -name ltmain.sh -printf '%h ') 201 fi 202 203 local d p 204 for d in "$@" ; do 205 export ELT_APPLIED_PATCHES= 206 207 if [[ -f ${d}/.elibtoolized ]] ; then 208 ${force} || continue 209 fi 210 211 einfo "Running elibtoolize in: ${d#${WORKDIR}/}/" 212 if [[ -f ${d}/.elibtoolized ]] ; then 213 ewarn " We've already been run in this tree; you should" 214 ewarn " avoid this if possible (perhaps by filing a bug)" 215 fi 216 217 for p in ${elt_patches} ; do 218 local ret=0 219 220 case ${p} in 221 portage) 222 # Stupid test to see if its already applied ... 223 if ! grep -qs 'We do not want portage' "${d}/ltmain.sh" ; then 224 ELT_walk_patches "${d}/ltmain.sh" "${p}" 225 ret=$? 226 fi 227 ;; 228 rem-int-dep) 229 ELT_walk_patches "${d}/ltmain.sh" "${p}" "${deptoremove}" 230 ret=$? 231 ;; 232 fix-relink) 233 # Do not apply if we do not have the relink patch applied ... 234 if grep -qs 'inst_prefix_dir' "${d}/ltmain.sh" ; then 235 ELT_walk_patches "${d}/ltmain.sh" "${p}" 236 ret=$? 237 fi 238 ;; 239 max_cmd_len) 240 # Do not apply if $max_cmd_len is not used ... 241 if grep -qs 'max_cmd_len' "${d}/ltmain.sh" ; then 242 ELT_walk_patches "${d}/ltmain.sh" "${p}" 243 ret=$? 244 fi 245 ;; 246 as-needed) 247 ELT_walk_patches "${d}/ltmain.sh" "${p}" 248 ret=$? 249 ;; 250 uclibc-conf) 251 if grep -qs 'Transform linux' "${d}/configure" ; then 252 ELT_walk_patches "${d}/configure" "${p}" 253 ret=$? 254 # ltmain.sh and co might be in a subdirectory ... 255 elif [[ ! -e ${d}/configure ]] && \ 256 grep -qs 'Transform linux' "${d}/../configure" ; then 257 ELT_walk_patches "${d}/../configure" "${p}" 258 ret=$? 259 fi 260 ;; 261 uclibc-ltconf) 262 # Newer libtoolize clears ltconfig, as not used anymore 263 if [[ -s ${d}/ltconfig ]] ; then 264 ELT_walk_patches "${d}/ltconfig" "${p}" 265 ret=$? 266 fi 267 ;; 268 fbsd-conf) 269 if grep -qs 'version_type=freebsd-' "${d}/configure" ; then 270 ELT_walk_patches "${d}/configure" "${p}" 271 ret=$? 272 # ltmain.sh and co might be in a subdirectory ... 273 elif [[ ! -e ${d}/configure ]] && \ 274 grep -qs 'version_type=freebsd-' "${d}/../configure" ; then 275 ELT_walk_patches "${d}/../configure" "${p}" 276 ret=$? 277 fi 278 ;; 279 fbsd-ltconf) 280 if [[ -s ${d}/ltconfig ]] ; then 281 ELT_walk_patches "${d}/ltconfig" "${p}" 282 ret=$? 283 fi 284 ;; 285 darwin-conf) 286 if grep -qs '&& echo \.so ||' "${d}/configure" ; then 287 ELT_walk_patches "${d}/configure" "${p}" 288 ret=$? 289 # ltmain.sh and co might be in a subdirectory ... 290 elif [[ ! -e ${d}/configure ]] && \ 291 grep -qs '&& echo \.so ||' "${d}/../configure" ; then 292 ELT_walk_patches "${d}/../configure" "${p}" 293 ret=$? 294 fi 295 ;; 296 darwin-ltconf) 297 # Newer libtoolize clears ltconfig, as not used anymore 298 if [[ -s ${d}/ltconfig ]] ; then 299 ELT_walk_patches "${d}/ltconfig" "${p}" 300 ret=$? 301 fi 302 ;; 303 darwin-ltmain) 304 # special case to avoid false positives (failing to apply 305 # ltmain.sh path message), newer libtools have this patch 306 # built in, so not much to patch around then 307 if [[ -e ${d}/ltmain.sh ]] && \ 308 ! grep -qs 'verstring="-compatibility_version' "${d}/ltmain.sh" ; then 309 ELT_walk_patches "${d}/ltmain.sh" "${p}" 310 ret=$? 311 fi 312 ;; 313 aixrtl|hpux-conf) 314 ret=1 315 local subret=0 316 # apply multiple patches as often as they match 317 while [[ $subret -eq 0 ]]; do 318 subret=1 319 if [[ -e ${d}/configure ]]; then 320 ELT_walk_patches "${d}/configure" "${p}" 321 subret=$? 322 # ltmain.sh and co might be in a subdirectory ... 323 elif [[ ! -e ${d}/configure && -e ${d}/../configure ]] ; then 324 ELT_walk_patches "${d}/../configure" "${p}" 325 subret=$? 326 fi 327 if [[ $subret -eq 0 ]]; then 328 # have at least one patch succeeded. 329 ret=0 330 fi 331 done 332 ;; 333 mint-conf|gold-conf) 334 ret=1 335 local subret=1 336 if [[ -e ${d}/configure ]]; then 337 ELT_walk_patches "${d}/configure" "${p}" 338 subret=$? 339 # ltmain.sh and co might be in a subdirectory ... 340 elif [[ -e ${d}/../configure ]] ; then 341 ELT_walk_patches "${d}/../configure" "${p}" 342 subret=$? 343 elif [[ -e ${d}/../../configure ]] ; then 344 ELT_walk_patches "${d}/../../configure" "${p}" 345 subret=$? 346 fi 347 if [[ $subret -eq 0 ]]; then 348 # have at least one patch succeeded. 349 ret=0 350 fi 351 ;; 352 install-sh) 353 ELT_walk_patches "${d}/install-sh" "${p}" 354 ret=$? 355 ;; 356 cross) 357 if tc-is-cross-compiler ; then 358 ELT_walk_patches "${d}/ltmain.sh" "${p}" 359 ret=$? 360 fi 361 ;; 362 *) 363 ELT_walk_patches "${d}/ltmain.sh" "${p}" 364 ret=$? 365 ;; 366 esac 367 368 if [[ ${ret} -ne 0 ]] ; then 369 case ${p} in 370 relink) 371 local version=$(ELT_libtool_version "${d}/ltmain.sh") 372 # Critical patch, but could be applied ... 373 # FIXME: Still need a patch for ltmain.sh > 1.4.0 374 if ! grep -qs 'inst_prefix_dir' "${d}/ltmain.sh" && \ 375 [[ $(VER_to_int "${version}") -ge $(VER_to_int "1.4.0") ]] ; then 376 ewarn " Could not apply relink.patch!" 377 fi 378 ;; 379 portage) 380 # Critical patch - for this one we abort, as it can really 381 # cause breakage without it applied! 382 if [[ ${do_portage} == "yes" ]] ; then 383 # Stupid test to see if its already applied ... 384 if ! grep -qs 'We do not want portage' "${d}/ltmain.sh" ; then 385 echo 386 eerror "Portage patch requested, but failed to apply!" 387 eerror "Please file a bug report to add a proper patch." 388 die "Portage patch requested, but failed to apply!" 389 fi 390 else 391 if grep -qs 'We do not want portage' "${d}/ltmain.sh" ; then 392 # ewarn " Portage patch seems to be already applied." 393 # ewarn " Please verify that it is not needed." 394 : 395 else 396 local version=$(ELT_libtool_version "${d}"/ltmain.sh) 397 echo 398 eerror "Portage patch failed to apply (ltmain.sh version ${version})!" 399 eerror "Please file a bug report to add a proper patch." 400 die "Portage patch failed to apply!" 401 fi 402 # We do not want to run libtoolize ... 403 ELT_APPLIED_PATCHES="portage" 404 fi 405 ;; 406 uclibc-*) 407 [[ ${CHOST} == *-uclibc ]] && ewarn " uClibc patch set '${p}' failed to apply!" 408 ;; 409 fbsd-*) 410 if [[ ${CHOST} == *-freebsd* ]] ; then 411 if [[ -z $(grep 'Handle Gentoo/FreeBSD as it was Linux' \ 412 "${d}/configure" "${d}/../configure" 2>/dev/null) ]]; then 413 eerror " FreeBSD patch set '${p}' failed to apply!" 414 die "FreeBSD patch set '${p}' failed to apply!" 415 fi 416 fi 417 ;; 418 darwin-*) 419 [[ ${CHOST} == *"-darwin"* ]] && ewarn " Darwin patch set '${p}' failed to apply!" 420 ;; 421 esac 422 fi 423 done 424 425 if [[ -z ${ELT_APPLIED_PATCHES} ]] ; then 426 if [[ ${do_portage} == "no" && \ 427 ${do_reversedeps} == "no" && \ 428 ${do_only_patches} == "no" && \ 429 ${deptoremove} == "" ]] 430 then 431 ewarn "Cannot apply any patches, please file a bug about this" 432 die 433 fi 434 fi 435 436 rm -f "${d}/libtool" 437 438 > "${d}/.elibtoolized" 439 done 440 } 441 442 uclibctoolize() { die "Use elibtoolize"; } 443 darwintoolize() { die "Use elibtoolize"; } 444 445 # char *VER_major(string) 446 # 447 # Return the Major (X of X.Y.Z) version 448 # 449 VER_major() { 450 [[ -z $1 ]] && return 1 451 452 local VER=$@ 453 echo "${VER%%[^[:digit:]]*}" 454 } 455 456 # char *VER_minor(string) 457 # 458 # Return the Minor (Y of X.Y.Z) version 459 # 460 VER_minor() { 461 [[ -z $1 ]] && return 1 462 463 local VER=$@ 464 VER=${VER#*.} 465 echo "${VER%%[^[:digit:]]*}" 466 } 467 468 # char *VER_micro(string) 469 # 470 # Return the Micro (Z of X.Y.Z) version. 471 # 472 VER_micro() { 473 [[ -z $1 ]] && return 1 474 475 local VER=$@ 476 VER=${VER#*.*.} 477 echo "${VER%%[^[:digit:]]*}" 478 } 479 480 # int VER_to_int(string) 481 # 482 # Convert a string type version (2.4.0) to an int (132096) 483 # for easy compairing or versions ... 484 # 485 VER_to_int() { 486 [[ -z $1 ]] && return 1 487 488 local VER_MAJOR=$(VER_major "$1") 489 local VER_MINOR=$(VER_minor "$1") 490 local VER_MICRO=$(VER_micro "$1") 491 local VER_int=$(( VER_MAJOR * 65536 + VER_MINOR * 256 + VER_MICRO )) 492 493 # We make version 1.0.0 the minimum version we will handle as 494 # a sanity check ... if its less, we fail ... 495 if [[ ${VER_int} -ge 65536 ]] ; then 496 echo "${VER_int}" 497 return 0 498 fi 499 500 echo 1 501 return 1 502 } 503 504 fi   ViewVC Help Powered by ViewVC 1.1.20  
{ "url": "http://sources.gentoo.org/cgi-bin/viewvc.cgi/gentoo-x86/eclass/libtool.eclass?revision=1.97&view=markup&pathrev=HEAD", "source_domain": "sources.gentoo.org", "snapshot_id": "crawl=CC-MAIN-2014-23", "warc_metadata": { "Content-Length": "90184", "Content-Type": "application/http; msgtype=response", "WARC-Block-Digest": "sha1:WX7TIE5U6HF22CKFQWNFKGHF3SOAPQVI", "WARC-Concurrent-To": "<urn:uuid:ef562c47-2774-4e7c-aeb6-336ded779cb6>", "WARC-Date": "2014-07-13T11:39:46Z", "WARC-IP-Address": "209.177.148.228", "WARC-Identified-Payload-Type": null, "WARC-Payload-Digest": "sha1:4ZZ6RBLGFHBCZ2ZLNLOK24AE7DKVA6YR", "WARC-Record-ID": "<urn:uuid:c696d020-e7af-4b16-a425-24843bd6326f>", "WARC-Target-URI": "http://sources.gentoo.org/cgi-bin/viewvc.cgi/gentoo-x86/eclass/libtool.eclass?revision=1.97&view=markup&pathrev=HEAD", "WARC-Truncated": null, "WARC-Type": "response", "WARC-Warcinfo-ID": "<urn:uuid:4ff0e510-3f6d-47fd-9213-df58337c4478>" }, "warc_info": "robots: classic\r\nhostname: ip-10-180-212-248.ec2.internal\r\nsoftware: Nutch 1.6 (CC)/CC WarcExport 1.0\r\nisPartOf: CC-MAIN-2014-23\r\noperator: CommonCrawl Admin\r\ndescription: Wide crawl of the web with URLs provided by Blekko for July 2014\r\npublisher: CommonCrawl\r\nformat: WARC File Format 1.0\r\nconformsTo: http://bibnum.bnf.fr/WARC/WARC_ISO_28500_version1_latestdraft.pdf" }
{ "line_start_idx": [ 0, 36, 43, 44, 79, 80, 142, 143, 144, 190, 253, 266, 298, 358, 359, 401, 470, 569, 571, 599, 616, 643, 691, 709, 791, 834, 839, 917, 997, 1080, 1083, 1153, 1201, 1204, 1279, 1354, 1428, 1503, 1543, 1564, 1602, 1631, 1669, 1675, 1690, 1695, 1698, 1745, 1748, 1784, 1787, 1792, 1844, 1849, 1880, 1895, 1912, 1930, 1946, 1975, 2008, 2011, 2052, 2076, 2127, 2133, 2181, 2184, 2256, 2327, 2360, 2439, 2449, 2511, 2519, 2528, 2534, 2537, 2556, 2561, 2564, 2569, 2633, 2638, 2665, 2670, 2687, 2735, 2759, 2764, 2769, 2772, 2777, 2827, 2848, 2853, 2877, 2896, 2911, 2928, 2950, 3001, 3025, 3028, 3065, 3104, 3107, 3162, 3223, 3260, 3308, 3338, 3393, 3399, 3403, 3450, 3454, 3525, 3609, 3642, 3696, 3764, 3821, 3831, 3841, 3848, 3857, 3861, 3881, 3899, 3905, 3909, 3938, 4057, 4077, 4154, 4228, 4279, 4285, 4366, 4372, 4458, 4478, 4490, 4516, 4546, 4577, 4603, 4626, 4652, 4676, 4774, 4778, 4801, 4818, 4833, 4875, 4930, 4951, 4958, 4978, 5013, 5067, 5092, 5123, 5130, 5148, 5208, 5234, 5241, 5270, 5326, 5358, 5402, 5406, 5439, 5498, 5505, 5520, 5559, 5580, 5587, 5604, 5623, 5630, 5643, 5660, 5667, 5674, 5720, 5762, 5771, 5780, 5784, 5861, 5865, 5886, 5953, 6027, 6084, 6188, 6232, 6274, 6283, 6287, 6349, 6379, 6386, 6390, 6425, 6436, 6479, 6524, 6533, 6589, 6596, 6600, 6614, 6637, 6669, 6673, 6715, 6740, 6747, 6751, 6805, 6847, 6908, 6970, 6977, 6981, 7014, 7030, 7034, 7051, 7064, 7116, 7183, 7228, 7239, 7246, 7253, 7270, 7332, 7343, 7350, 7366, 7432, 7490, 7535, 7546, 7553, 7560, 7577, 7628, 7682, 7727, 7738, 7745, 7752, 7767, 7812, 7823, 7830, 7847, 7905, 7950, 7961, 8015, 8055, 8113, 8161, 8172, 8179, 8186, 8205, 8265, 8302, 8346, 8357, 8364, 8371, 8386, 8450, 8495, 8506, 8560, 8600, 8664, 8712, 8723, 8730, 8737, 8754, 8791, 8835, 8846, 8853, 8860, 8877, 8935, 8980, 8991, 9045, 9085, 9143, 9191, 9202, 9209, 9216, 9235, 9295, 9332, 9376, 9387, 9394, 9401, 9420, 9482, 9544, 9593, 9629, 9704, 9749, 9760, 9767, 9774, 9796, 9806, 9825, 9877, 9911, 9924, 9961, 10006, 10020, 10074, 10140, 10188, 10202, 10209, 10242, 10283, 10293, 10300, 10309, 10316, 10341, 10351, 10370, 10407, 10452, 10466, 10520, 10563, 10611, 10625, 10671, 10722, 10736, 10743, 10776, 10817, 10827, 10834, 10841, 10857, 10903, 10914, 10921, 10932, 10967, 11012, 11023, 11030, 11037, 11044, 11089, 11100, 11107, 11116, 11120, 11153, 11170, 11182, 11240, 11287, 11341, 11399, 11469, 11512, 11519, 11526, 11539, 11602, 11643, 11686, 11738, 11805, 11814, 11873, 11934, 11990, 11997, 12006, 12071, 12129, 12181, 12187, 12196, 12254, 12263, 12338, 12399, 12440, 12447, 12490, 12524, 12531, 12538, 12552, 12636, 12643, 12655, 12698, 12760, 12823, 12879, 12931, 12938, 12945, 12952, 12966, 13053, 13060, 13069, 13076, 13085, 13089, 13135, 13172, 13207, 13243, 13271, 13280, 13347, 13355, 13362, 13369, 13373, 13398, 13402, 13429, 13438, 13444, 13448, 13495, 13542, 13546, 13576, 13582, 13626, 13632, 13650, 13678, 13682, 13699, 13732, 13738, 13742, 13772, 13778, 13822, 13828, 13846, 13874, 13878, 13895, 13913, 13946, 13952, 13956, 13986, 13992, 14037, 14043, 14061, 14089, 14093, 14110, 14130, 14163, 14169, 14173, 14202, 14208, 14271, 14313, 14319, 14338, 14366, 14370, 14408, 14446, 14484, 14557, 14561, 14627, 14677, 14718, 14740, 14753, 14760, 14764, 14775, 14788, 14794, 14798, 14805, 14806, 14820 ], "line_end_idx": [ 36, 43, 44, 79, 80, 142, 143, 144, 190, 253, 266, 298, 358, 359, 401, 470, 569, 571, 599, 616, 643, 691, 709, 791, 834, 839, 917, 997, 1080, 1083, 1153, 1201, 1204, 1279, 1354, 1428, 1503, 1543, 1564, 1602, 1631, 1669, 1675, 1690, 1695, 1698, 1745, 1748, 1784, 1787, 1792, 1844, 1849, 1880, 1895, 1912, 1930, 1946, 1975, 2008, 2011, 2052, 2076, 2127, 2133, 2181, 2184, 2256, 2327, 2360, 2439, 2449, 2511, 2519, 2528, 2534, 2537, 2556, 2561, 2564, 2569, 2633, 2638, 2665, 2670, 2687, 2735, 2759, 2764, 2769, 2772, 2777, 2827, 2848, 2853, 2877, 2896, 2911, 2928, 2950, 3001, 3025, 3028, 3065, 3104, 3107, 3162, 3223, 3260, 3308, 3338, 3393, 3399, 3403, 3450, 3454, 3525, 3609, 3642, 3696, 3764, 3821, 3831, 3841, 3848, 3857, 3861, 3881, 3899, 3905, 3909, 3938, 4057, 4077, 4154, 4228, 4279, 4285, 4366, 4372, 4458, 4478, 4490, 4516, 4546, 4577, 4603, 4626, 4652, 4676, 4774, 4778, 4801, 4818, 4833, 4875, 4930, 4951, 4958, 4978, 5013, 5067, 5092, 5123, 5130, 5148, 5208, 5234, 5241, 5270, 5326, 5358, 5402, 5406, 5439, 5498, 5505, 5520, 5559, 5580, 5587, 5604, 5623, 5630, 5643, 5660, 5667, 5674, 5720, 5762, 5771, 5780, 5784, 5861, 5865, 5886, 5953, 6027, 6084, 6188, 6232, 6274, 6283, 6287, 6349, 6379, 6386, 6390, 6425, 6436, 6479, 6524, 6533, 6589, 6596, 6600, 6614, 6637, 6669, 6673, 6715, 6740, 6747, 6751, 6805, 6847, 6908, 6970, 6977, 6981, 7014, 7030, 7034, 7051, 7064, 7116, 7183, 7228, 7239, 7246, 7253, 7270, 7332, 7343, 7350, 7366, 7432, 7490, 7535, 7546, 7553, 7560, 7577, 7628, 7682, 7727, 7738, 7745, 7752, 7767, 7812, 7823, 7830, 7847, 7905, 7950, 7961, 8015, 8055, 8113, 8161, 8172, 8179, 8186, 8205, 8265, 8302, 8346, 8357, 8364, 8371, 8386, 8450, 8495, 8506, 8560, 8600, 8664, 8712, 8723, 8730, 8737, 8754, 8791, 8835, 8846, 8853, 8860, 8877, 8935, 8980, 8991, 9045, 9085, 9143, 9191, 9202, 9209, 9216, 9235, 9295, 9332, 9376, 9387, 9394, 9401, 9420, 9482, 9544, 9593, 9629, 9704, 9749, 9760, 9767, 9774, 9796, 9806, 9825, 9877, 9911, 9924, 9961, 10006, 10020, 10074, 10140, 10188, 10202, 10209, 10242, 10283, 10293, 10300, 10309, 10316, 10341, 10351, 10370, 10407, 10452, 10466, 10520, 10563, 10611, 10625, 10671, 10722, 10736, 10743, 10776, 10817, 10827, 10834, 10841, 10857, 10903, 10914, 10921, 10932, 10967, 11012, 11023, 11030, 11037, 11044, 11089, 11100, 11107, 11116, 11120, 11153, 11170, 11182, 11240, 11287, 11341, 11399, 11469, 11512, 11519, 11526, 11539, 11602, 11643, 11686, 11738, 11805, 11814, 11873, 11934, 11990, 11997, 12006, 12071, 12129, 12181, 12187, 12196, 12254, 12263, 12338, 12399, 12440, 12447, 12490, 12524, 12531, 12538, 12552, 12636, 12643, 12655, 12698, 12760, 12823, 12879, 12931, 12938, 12945, 12952, 12966, 13053, 13060, 13069, 13076, 13085, 13089, 13135, 13172, 13207, 13243, 13271, 13280, 13347, 13355, 13362, 13369, 13373, 13398, 13402, 13429, 13438, 13444, 13448, 13495, 13542, 13546, 13576, 13582, 13626, 13632, 13650, 13678, 13682, 13699, 13732, 13738, 13742, 13772, 13778, 13822, 13828, 13846, 13874, 13878, 13895, 13913, 13946, 13952, 13956, 13986, 13992, 14037, 14043, 14061, 14089, 14093, 14110, 14130, 14163, 14169, 14173, 14202, 14208, 14271, 14313, 14319, 14338, 14366, 14370, 14408, 14446, 14484, 14557, 14561, 14627, 14677, 14718, 14740, 14753, 14760, 14764, 14775, 14788, 14794, 14798, 14805, 14806, 14820, 14846 ] }
{ "red_pajama_v2": { "ccnet_original_length": 14846, "ccnet_original_nlines": 520, "rps_doc_curly_bracket": 0.023440660908818245, "rps_doc_ldnoobw_words": 0, "rps_doc_lorem_ipsum": 0, "rps_doc_stop_word_fraction": 0.16028332710266113, "rps_doc_ut1_blacklist": 0, "rps_doc_frac_all_caps_words": 0.02151102013885975, "rps_doc_frac_lines_end_with_ellipsis": 0.030710170045495033, "rps_doc_frac_no_alph_words": 0.538300096988678, "rps_doc_frac_unique_words": 0.4780701696872711, "rps_doc_mean_word_length": 4.755847930908203, "rps_doc_num_sentences": 169, "rps_doc_symbol_to_word_ratio": 0.03147954121232033, "rps_doc_unigram_entropy": 6.201272010803223, "rps_doc_word_count": 2052, "rps_doc_frac_chars_dupe_10grams": 0.020801309496164322, "rps_doc_frac_chars_dupe_5grams": 0.11312633752822876, "rps_doc_frac_chars_dupe_6grams": 0.09099292755126953, "rps_doc_frac_chars_dupe_7grams": 0.06844963878393173, "rps_doc_frac_chars_dupe_8grams": 0.054923661053180695, "rps_doc_frac_chars_dupe_9grams": 0.020801309496164322, "rps_doc_frac_chars_top_2gram": 0.008607439696788788, "rps_doc_frac_chars_top_3gram": 0.028179120272397995, "rps_doc_frac_chars_top_4gram": 0.009222259745001793, "rps_doc_books_importance": -1757.36767578125, "rps_doc_books_importance_length_correction": -1757.36767578125, "rps_doc_openwebtext_importance": -1038.630615234375, "rps_doc_openwebtext_importance_length_correction": -1038.630615234375, "rps_doc_wikipedia_importance": -750.2073974609375, "rps_doc_wikipedia_importance_length_correction": -750.2073974609375 }, "fasttext": { "dclm": 0.05234944820404053, "english": 0.4317791759967804, "fineweb_edu_approx": 1.4523190259933472, "eai_general_math": 0.6181023120880127, "eai_open_web_math": 0.43162232637405396, "eai_web_code": 0.36895281076431274 } }
{ "free_decimal_correspondence": { "primary": { "code": "005.455", "labels": { "level_1": "General works, books and libraries, information sciences", "level_2": "", "level_3": "Computer programming" } }, "secondary": { "code": "004.02", "labels": { "level_1": "General works, books and libraries, information sciences", "level_2": "", "level_3": "Computers and Computer science" } } }, "bloom_cognitive_process": { "primary": { "code": "3", "label": "Apply" }, "secondary": { "code": "2", "label": "Understand" } }, "bloom_knowledge_domain": { "primary": { "code": "3", "label": "Procedural" }, "secondary": { "code": "2", "label": "Conceptual" } }, "document_type_v1": { "primary": { "code": "4", "label": "Code/Software" }, "secondary": { "code": "3", "label": "Reference/Encyclopedic/Educational" } }, "extraction_artifacts": { "primary": { "code": "0", "label": "No Artifacts" }, "secondary": { "code": "3", "label": "Irrelevant Content" } }, "missing_content": { "primary": { "code": "0", "label": "No missing content" }, "secondary": { "code": "-1", "label": "Abstain" } }, "document_type_v2": { "primary": { "code": "8", "label": "Documentation" }, "secondary": { "code": "3", "label": "Academic Writing" } }, "reasoning_depth": { "primary": { "code": "4", "label": "Advanced Reasoning" }, "secondary": { "code": "3", "label": "Intermediate Reasoning" } }, "technical_correctness": { "primary": { "code": "4", "label": "Highly Correct" }, "secondary": { "code": "3", "label": "Mostly Correct" } }, "education_level": { "primary": { "code": "4", "label": "Graduate/Expert Level" }, "secondary": { "code": "3", "label": "Undergraduate Level" } } }
672f1e42c33a7f9846924a2431ea77df
414,505,862,837,096,300
Automation Testing tool that can prettybadelyngeΛογισμικό & κατασκευή λογ/κού 18 Νοε 2013 (πριν από 3 χρόνια και 8 μήνες) 69 εμφανίσεις Automation Testing tool that can be used in software engineering course Ran Shi 04/29/2011 Why use automated performance testing tools? There is no way to manually test the way a Web application performs under the stress of hundreds of users until it is deployed on a public Web server. However, there are some tools that can help. Automated Performance Testing Tools Commercial - LoadRunner , Rational Robot, QALoad , SilkPerformer Open Source - Apache JMeter , OpenSTA Apache JMeter Apache JMeter is open source software, a 100% pure Java desktop application designed to load test functional behavior and measure performance. It was originally designed for testing Web Applications but has expanded to other test functions. What can it do? It can be used to simulate a heavy load on a server, network or object to test its strength or to analyze overall performance under different load types. Users can use it to make a graphical analysis of performance or to test their server/script/object behavior under heavy concurrent load. Apache JMeter Features It can test many different server types; Complete portability and 100% Java purity; The full multithreading framework allows concurrent sampling by many threads and simultaneous different functions sampling by separating thread groups; Its careful GUI design allows faster operation and more precise timings; Caching and offline analysis/replaying of test results. Adding Users Thread Group Users Thread Group (cont.) Thread Group tells JMeter the number of users that want to simulate, how often the users send requests and how many requests they send. Number of Threads: the number of users. Ramp - Up Period: how long to reach the maximum test stress. Loop Count: how many times to repeat the test. Adding Default HTTP Request Properties Adding HTTP Requests Apache JMeter Graph Results Apache JMeter Graph Results (cont.) Samples: the number of requests. Sample= Number of Threads * Loop Count. Average: the average response time Median: 50% users response time Throughput: the number of Requests/Minute the server handled. Apache JMeter Graph Results (cont.) The Graph Results listener generates a simple graph that plots all sample times. Users can increase the number of threads to discover their server’s maximum throughput. Spline Visualizer Spline Visualizer (cont.) The Spline Visualizer provides a view of all sample times from the start of the test till the end, regardless of how many samples have been taken. Summary Advantages - Open source and free - Detailed tutorial from Apache - Unlimited Load generation Disadvantages - In specific unicast scenarios, such as repeatedly downloading very large files, the ability of agents to generate load falls off abruptly due to memory issues. Further Tutorial or Information Apache Jakarta Project provides detailed tutorial for using Apache JMeter http://jakarta.apache.org/jmeter/usermanual/in dex.html
{ "url": "https://www.techylib.com/el/view/prettybadelynge/automation_testing_tool_that_can", "source_domain": "www.techylib.com", "snapshot_id": "crawl=CC-MAIN-2017-30", "warc_metadata": { "Content-Length": "40567", "Content-Type": "application/http; msgtype=response", "WARC-Block-Digest": "sha1:3GBOF6UB6XQS7N7KVF5WSZJNCDZHS3ZU", "WARC-Concurrent-To": "<urn:uuid:19210b57-e757-47e5-aa23-11caf8877985>", "WARC-Date": "2017-07-23T05:09:53Z", "WARC-IP-Address": "54.243.162.245", "WARC-Identified-Payload-Type": "application/xhtml+xml", "WARC-Payload-Digest": "sha1:Y74A3T5XLC7IIVIYDVAMAJ43BMDIHOLY", "WARC-Record-ID": "<urn:uuid:394ad935-b479-4eba-80e8-d76546d13a4b>", "WARC-Target-URI": "https://www.techylib.com/el/view/prettybadelynge/automation_testing_tool_that_can", "WARC-Truncated": "length", "WARC-Type": "response", "WARC-Warcinfo-ID": "<urn:uuid:b528187e-ad5f-4955-9275-37d8bc418b74>" }, "warc_info": "robots: classic\r\nhostname: ip-10-146-194-113.ec2.internal\r\nsoftware: Nutch 1.6 (CC)\r\nisPartOf: CC-MAIN-2017-30\r\noperator: Common Crawl Admin\r\ndescription: Wide crawl of the web for July 2017\r\npublisher: Common Crawl\r\nformat: WARC File Format 1.0\r\nconformsTo: http://bibnum.bnf.fr/WARC/WARC_ISO_28500_version1_latestdraft.pdf" }
{ "line_start_idx": [ 0, 33, 34, 79, 80, 124, 125, 139, 140, 173, 205, 212, 213, 214, 222, 223, 234, 235, 265, 280, 281, 282, 283, 326, 371, 415, 458, 479, 480, 481, 511, 517, 518, 519, 530, 531, 533, 534, 545, 563, 570, 572, 586, 587, 588, 589, 601, 602, 604, 605, 612, 619, 621, 629, 630, 631, 632, 633, 640, 647, 648, 649, 656, 663, 664, 691, 735, 780, 824, 869, 891, 892, 908, 909, 910, 955, 1005, 1052, 1064, 1065, 1066, 1112, 1144, 1186, 1203, 1204, 1211, 1218, 1219, 1228, 1229, 1230, 1271, 1272, 1273, 1316, 1317, 1318, 1359, 1399, 1444, 1470, 1471, 1472, 1523, 1545, 1546, 1547, 1594, 1603, 1604, 1605, 1631, 1632, 1633, 1660, 1661, 1662, 1681, 1688, 1689, 1709, 1757, 1799, 1800, 1801, 1820, 1841, 1842, 1843, 1848, 1850, 1861, 1883, 1904, 1905, 1906, 1918, 1947, 1953, 1954, 1982, 1993, 1994, 2015, 2016, 2023, 2030, 2031, 2045, 2046, 2053, 2060, 2061, 2075, 2083, 2084, 2085, 2094, 2095, 2119, 2159, 2160, 2161, 2170, 2171, 2197, 2198, 2199, 2207, 2208, 2232, 2233, 2234, 2246, 2247, 2277, 2297, 2298, 2305, 2312, 2313, 2327, 2335, 2336, 2337, 2383, 2418, 2419, 2420, 2464, 2508, 2509, 2516, 2517, 2528, 2529, 2536, 2537, 2548, 2549, 2557, 2558, 2559, 2563, 2570, 2571, 2582, 2583, 2606, 2655, 2696, 2708, 2709, 2717, 2718, 2719, 2730, 2731, 2733, 2754, 2755, 2757, 2787, 2788, 2790, 2816, 2817, 2818, 2832, 2833, 2835, 2836, 2848, 2856, 2857, 2887, 2939, 2989, 2997, 2998, 3030, 3031, 3032, 3073, 3099, 3106, 3107, 3108, 3155 ], "line_end_idx": [ 33, 34, 79, 80, 124, 125, 139, 140, 173, 205, 212, 213, 214, 222, 223, 234, 235, 265, 280, 281, 282, 283, 326, 371, 415, 458, 479, 480, 481, 511, 517, 518, 519, 530, 531, 533, 534, 545, 563, 570, 572, 586, 587, 588, 589, 601, 602, 604, 605, 612, 619, 621, 629, 630, 631, 632, 633, 640, 647, 648, 649, 656, 663, 664, 691, 735, 780, 824, 869, 891, 892, 908, 909, 910, 955, 1005, 1052, 1064, 1065, 1066, 1112, 1144, 1186, 1203, 1204, 1211, 1218, 1219, 1228, 1229, 1230, 1271, 1272, 1273, 1316, 1317, 1318, 1359, 1399, 1444, 1470, 1471, 1472, 1523, 1545, 1546, 1547, 1594, 1603, 1604, 1605, 1631, 1632, 1633, 1660, 1661, 1662, 1681, 1688, 1689, 1709, 1757, 1799, 1800, 1801, 1820, 1841, 1842, 1843, 1848, 1850, 1861, 1883, 1904, 1905, 1906, 1918, 1947, 1953, 1954, 1982, 1993, 1994, 2015, 2016, 2023, 2030, 2031, 2045, 2046, 2053, 2060, 2061, 2075, 2083, 2084, 2085, 2094, 2095, 2119, 2159, 2160, 2161, 2170, 2171, 2197, 2198, 2199, 2207, 2208, 2232, 2233, 2234, 2246, 2247, 2277, 2297, 2298, 2305, 2312, 2313, 2327, 2335, 2336, 2337, 2383, 2418, 2419, 2420, 2464, 2508, 2509, 2516, 2517, 2528, 2529, 2536, 2537, 2548, 2549, 2557, 2558, 2559, 2563, 2570, 2571, 2582, 2583, 2606, 2655, 2696, 2708, 2709, 2717, 2718, 2719, 2730, 2731, 2733, 2754, 2755, 2757, 2787, 2788, 2790, 2816, 2817, 2818, 2832, 2833, 2835, 2836, 2848, 2856, 2857, 2887, 2939, 2989, 2997, 2998, 3030, 3031, 3032, 3073, 3099, 3106, 3107, 3108, 3155, 3163 ] }
{ "red_pajama_v2": { "ccnet_original_length": 3163, "ccnet_original_nlines": 249, "rps_doc_curly_bracket": 0, "rps_doc_ldnoobw_words": 0, "rps_doc_lorem_ipsum": 0, "rps_doc_stop_word_fraction": 0.27617329359054565, "rps_doc_ut1_blacklist": 0, "rps_doc_frac_all_caps_words": 0.005415160208940506, "rps_doc_frac_lines_end_with_ellipsis": 0, "rps_doc_frac_no_alph_words": 0.18231047689914703, "rps_doc_frac_unique_words": 0.49673202633857727, "rps_doc_mean_word_length": 5.461873531341553, "rps_doc_num_sentences": 28, "rps_doc_symbol_to_word_ratio": 0, "rps_doc_unigram_entropy": 5.05040979385376, "rps_doc_word_count": 459, "rps_doc_frac_chars_dupe_10grams": 0, "rps_doc_frac_chars_dupe_5grams": 0.044674910604953766, "rps_doc_frac_chars_dupe_6grams": 0, "rps_doc_frac_chars_dupe_7grams": 0, "rps_doc_frac_chars_dupe_8grams": 0, "rps_doc_frac_chars_dupe_9grams": 0, "rps_doc_frac_chars_top_2gram": 0.0382927805185318, "rps_doc_frac_chars_top_3gram": 0.021938569843769073, "rps_doc_frac_chars_top_4gram": 0.028719589114189148, "rps_doc_books_importance": -212.51055908203125, "rps_doc_books_importance_length_correction": -212.51055908203125, "rps_doc_openwebtext_importance": -114.38481140136719, "rps_doc_openwebtext_importance_length_correction": -114.38481140136719, "rps_doc_wikipedia_importance": -116.37873077392578, "rps_doc_wikipedia_importance_length_correction": -116.37873077392578 }, "fasttext": { "dclm": 0.10070902109146118, "english": 0.8402018547058105, "fineweb_edu_approx": 2.867739200592041, "eai_general_math": 0.2841971516609192, "eai_open_web_math": 0.24110519886016846, "eai_web_code": 0.7519816756248474 } }
{ "free_decimal_correspondence": { "primary": { "code": "005.452", "labels": { "level_1": "General works, books and libraries, information sciences", "level_2": "", "level_3": "Computer programming" } }, "secondary": { "code": "005.1", "labels": { "level_1": "General works, books and libraries, information sciences", "level_2": "", "level_3": "Computer programming" } } }, "bloom_cognitive_process": { "primary": { "code": "2", "label": "Understand" }, "secondary": { "code": "3", "label": "Apply" } }, "bloom_knowledge_domain": { "primary": { "code": "2", "label": "Conceptual" }, "secondary": { "code": "3", "label": "Procedural" } }, "document_type_v1": { "primary": { "code": "3", "label": "Reference/Encyclopedic/Educational" }, "secondary": { "code": "-1", "label": "Abstain" } }, "extraction_artifacts": { "primary": { "code": "3", "label": "Irrelevant Content" }, "secondary": { "code": "0", "label": "No Artifacts" } }, "missing_content": { "primary": { "code": "4", "label": "Missing Images or Figures" }, "secondary": { "code": "0", "label": "No missing content" } }, "document_type_v2": { "primary": { "code": "23", "label": "Tutorial" }, "secondary": { "code": "8", "label": "Documentation" } }, "reasoning_depth": { "primary": { "code": "2", "label": "Basic Reasoning" }, "secondary": { "code": "3", "label": "Intermediate Reasoning" } }, "technical_correctness": { "primary": { "code": "4", "label": "Highly Correct" }, "secondary": { "code": "3", "label": "Mostly Correct" } }, "education_level": { "primary": { "code": "3", "label": "Undergraduate Level" }, "secondary": { "code": "2", "label": "High School Level" } } }
672f1e42c33a7f9846924a2431ea77df
4,391,347,559,819,911,000
6 I've made this Community Wiki because some may think it's open to debate and others might think it's a question of using words to mean what they really mean (in other words, it's a matter of opinion whether it's a matter of opinion). There's a general question on SO about declarative programming, which has some great answers. But I'm a bit thrown by this blog post from a Microsoft evangelist. One advantage of declarative programming is that you can indicate what you would like to do, but not how to do it. So far, so good - that exactly agrees with the accepted answer on the SO question, in fact. But then check out the part about "Service implementation", You can just look at a few dozen lines of xaml code and be able to determine how the WCF Service is configured and how the corresponding workflow is defined. Having looked at some examples, let me briefly answer that by saying "No, I can't". But rather than flippantly dismissing this stuff, let's look at the docs. It's taken a while but finally reality has caught up with satire... but that's not the point - of course they aren't seriously suggesting doing this to expose something trivial like addition. Nor am I complaining about the ridiculous verbosity, and the strange idea that anyone would ever write something like that by hand - it looks more like the output of a compiler than a human-readable language. The puzzle for me is that this is claimed to be "declarative". And yet the core of it is an assignment statement. There's more here: Declarative services are defined declaratively in XAML and provide another layer of abstraction. Basically, you create a model of the service by defining what you want the service to do rather than how to do it. The entire service can be defined declaratively, including the implementation of the operations. So if we say declarative or declaratively three times, that makes it declarative. Gotcha. And if we say the magic phrase "what you want to do rather than how to do it" then we can neglect to notice that in the very next sentence, we are going to specify "implementation of the operations" after all, and so we are going to say precisely how to do it. The example in that page is: <wma:Sequence> <wma:WriteLine Text ='[String.Concat(String.Concat(String.Concat(String.Concat("Add(", CType(op1, Object)), ","), CType(op2, Object)), ") called")]' /> <wma:Assign x:TypeArguments="xs:Int32" To="[result1]" Value="[op1 + op2]" /> </wma:Sequence> Which is to say, the whole thing (including a ton of junk I've cut out from the WF example) is precisely equivalent to: void Add(int op1, int op2, out int result1) { Console.WriteLine("Add(" + op1 + ", " + op2 + ") called"); result1 = op1 + op2; } So - a block of statements, to be executed in the order they appear, and having side effects. There are of course workflow activity elements for looping (and you can write your own activities if WF doesn't already have your favourite imperative statement). Apparently, "rewriting your code in an unreadable format" is the same thing as "adding a layer of abstraction". To reiterate, it's not the insane unreadable verbosity I'm complaining about - it's the fact that it is clearly Turing-complete imperative programming happening in the service implementation, so what's the point? Before you know it, we'll be stepping through our workflows in the debugger, trying to figure out which assignment statement mutated which value, or why the loop keeps going round forever. (The irony is that in the C# version, it's a little more declarative, because we haven't specified how the pieces of string should be concatenated, thus allowing the compiler to generate fewer calls to the Concat method.) So does writing something in XML make it declarative (as well as less readable)? 3 Yet another acknowledgment that using XML in capacities other than exchange format sucks big time. Service "definitions" in WCF have been declarative from day one. However, separating interface definitions from service definitions (by this I mean ServiceContractAttribute et al) is, IMO, good thing. However, using XML as a programming language really sucks. I personally feel quite explainable attacks of pure terror when looking at these XML docs. 0 There's a part of Windows Workflow Foundation that isn't much discussed (outside of Microsoft), that may clarify this for you. If you create a Workflow project, and look in the toolbox, you'll see a large number of "boxes and lines" that you can drag onto the design surface. You may get the impression that you are meant to create your workflows from this set of tools. That is not the case. You are expected to author custom activities specific to your problem domain. These are meant to be connected by the various out of the box activities. These are likely to be fairly high-level activities - things like "evaluate insurance policy" or "record patient stay". A declarative workflow (or service) will be declaring how to put together the problem-specific activities you have available to you. • "declaring how to put together" - but only in the sense that a method body in C# "declares" how to implement the method. And note the appearance of the word "how", which is what declarative programming is supposed to avoid. What is the line between declarative and imperative, given that the workflow activities carried out by a service are typically imperative steps (however chunky) and so have to operate in an imperative sequence, and glued by imperative out-of-the-box WF activities? – Daniel Earwicker Jul 27 '09 at 12:22 Your Answer By clicking “Post Your Answer”, you agree to our terms of service, privacy policy and cookie policy Not the answer you're looking for? Browse other questions tagged or ask your own question.
{ "url": "https://stackoverflow.com/questions/1187713/wf-wcf-and-declarative-services-or-what-does-microsoft-mean-by-declarative?noredirect=1", "source_domain": "stackoverflow.com", "snapshot_id": "crawl=CC-MAIN-2019-26", "warc_metadata": { "Content-Length": "135368", "Content-Type": "application/http; msgtype=response", "WARC-Block-Digest": "sha1:ONVTFX2RFQ6XKF6VYPZ7D2LQVPV6IEMO", "WARC-Concurrent-To": "<urn:uuid:7cd98f84-fa02-4add-b688-fccc4ccfcdfd>", "WARC-Date": "2019-06-19T08:19:00Z", "WARC-IP-Address": "151.101.1.69", "WARC-Identified-Payload-Type": "text/html", "WARC-Payload-Digest": "sha1:DMMMZ3F2U5MEJ3C63QHAFZL5I56QTJDE", "WARC-Record-ID": "<urn:uuid:a109b06b-a6c2-48ec-b303-a49435ef3bef>", "WARC-Target-URI": "https://stackoverflow.com/questions/1187713/wf-wcf-and-declarative-services-or-what-does-microsoft-mean-by-declarative?noredirect=1", "WARC-Truncated": null, "WARC-Type": "response", "WARC-Warcinfo-ID": "<urn:uuid:f3184d25-4b73-4783-a501-639aae74db06>" }, "warc_info": "isPartOf: CC-MAIN-2019-26\r\npublisher: Common Crawl\r\ndescription: Wide crawl of the web for June 2019\r\noperator: Common Crawl Admin ([email protected])\r\nhostname: ip-10-13-190-31.ec2.internal\r\nsoftware: Apache Nutch 1.15 (modified, https://github.com/commoncrawl/nutch/)\r\nrobots: checked via crawler-commons 1.1-SNAPSHOT (https://github.com/crawler-commons/crawler-commons)\r\nformat: WARC File Format 1.1\r\nconformsTo: http://iipc.github.io/warc-specifications/specifications/warc-format/warc-1.1/" }
{ "line_start_idx": [ 0, 2, 3, 237, 238, 332, 333, 401, 402, 517, 518, 610, 611, 671, 672, 830, 831, 989, 990, 1391, 1392, 1506, 1507, 1526, 1527, 1836, 1837, 2188, 2189, 2218, 2219, 2234, 2390, 2471, 2487, 2488, 2608, 2609, 2653, 2655, 2718, 2743, 2745, 2746, 3115, 3116, 3518, 3519, 3741, 3742, 3823, 3824, 3826, 3827, 3926, 3927, 4187, 4188, 4279, 4280, 4282, 4283, 4410, 4411, 4677, 4678, 4950, 4951, 5084, 5085, 5617, 5618, 5630, 5631, 5731, 5732 ], "line_end_idx": [ 2, 3, 237, 238, 332, 333, 401, 402, 517, 518, 610, 611, 671, 672, 830, 831, 989, 990, 1391, 1392, 1506, 1507, 1526, 1527, 1836, 1837, 2188, 2189, 2218, 2219, 2234, 2390, 2471, 2487, 2488, 2608, 2609, 2653, 2655, 2718, 2743, 2745, 2746, 3115, 3116, 3518, 3519, 3741, 3742, 3823, 3824, 3826, 3827, 3926, 3927, 4187, 4188, 4279, 4280, 4282, 4283, 4410, 4411, 4677, 4678, 4950, 4951, 5084, 5085, 5617, 5618, 5630, 5631, 5731, 5732, 5822 ] }
{ "red_pajama_v2": { "ccnet_original_length": 5822, "ccnet_original_nlines": 75, "rps_doc_curly_bracket": 0.0003435199905652553, "rps_doc_ldnoobw_words": 2, "rps_doc_lorem_ipsum": 0, "rps_doc_stop_word_fraction": 0.4406224489212036, "rps_doc_ut1_blacklist": 0, "rps_doc_frac_all_caps_words": 0.019656019285321236, "rps_doc_frac_lines_end_with_ellipsis": 0, "rps_doc_frac_no_alph_words": 0.1924651861190796, "rps_doc_frac_unique_words": 0.43663471937179565, "rps_doc_mean_word_length": 4.83280086517334, "rps_doc_num_sentences": 49, "rps_doc_symbol_to_word_ratio": 0.002457000082358718, "rps_doc_unigram_entropy": 5.404675006866455, "rps_doc_word_count": 939, "rps_doc_frac_chars_dupe_10grams": 0, "rps_doc_frac_chars_dupe_5grams": 0.032833848148584366, "rps_doc_frac_chars_dupe_6grams": 0.0101366201415658, "rps_doc_frac_chars_dupe_7grams": 0.0101366201415658, "rps_doc_frac_chars_dupe_8grams": 0.0101366201415658, "rps_doc_frac_chars_dupe_9grams": 0, "rps_doc_frac_chars_top_2gram": 0.0061701200902462006, "rps_doc_frac_chars_top_3gram": 0.0061701200902462006, "rps_doc_frac_chars_top_4gram": 0.007933010347187519, "rps_doc_books_importance": -464.96612548828125, "rps_doc_books_importance_length_correction": -464.96612548828125, "rps_doc_openwebtext_importance": -293.0729064941406, "rps_doc_openwebtext_importance_length_correction": -293.0729064941406, "rps_doc_wikipedia_importance": -261.23480224609375, "rps_doc_wikipedia_importance_length_correction": -261.23480224609375 }, "fasttext": { "dclm": 0.3063504099845886, "english": 0.948196291923523, "fineweb_edu_approx": 2.222031593322754, "eai_general_math": 0.3452810049057007, "eai_open_web_math": 0.2681114673614502, "eai_web_code": 0.17075115442276 } }
{ "free_decimal_correspondence": { "primary": { "code": "005.1", "labels": { "level_1": "General works, books and libraries, information sciences", "level_2": "", "level_3": "Computer programming" } }, "secondary": { "code": "005.457", "labels": { "level_1": "General works, books and libraries, information sciences", "level_2": "", "level_3": "Computer programming" } } }, "bloom_cognitive_process": { "primary": { "code": "5", "label": "Evaluate" }, "secondary": { "code": "4", "label": "Analyze" } }, "bloom_knowledge_domain": { "primary": { "code": "2", "label": "Conceptual" }, "secondary": { "code": "3", "label": "Procedural" } }, "document_type_v1": { "primary": { "code": "5", "label": "Social/Forum" }, "secondary": { "code": "-1", "label": "Abstain" } }, "extraction_artifacts": { "primary": { "code": "3", "label": "Irrelevant Content" }, "secondary": { "code": "0", "label": "No Artifacts" } }, "missing_content": { "primary": { "code": "0", "label": "No missing content" }, "secondary": { "code": "-1", "label": "Abstain" } }, "document_type_v2": { "primary": { "code": "18", "label": "Q&A Forum" }, "secondary": { "code": "8", "label": "Documentation" } }, "reasoning_depth": { "primary": { "code": "3", "label": "Intermediate Reasoning" }, "secondary": { "code": "2", "label": "Basic Reasoning" } }, "technical_correctness": { "primary": { "code": "3", "label": "Mostly Correct" }, "secondary": { "code": "4", "label": "Highly Correct" } }, "education_level": { "primary": { "code": "3", "label": "Undergraduate Level" }, "secondary": { "code": "4", "label": "Graduate/Expert Level" } } }
672f1e42c33a7f9846924a2431ea77df
-2,039,790,532,800,758,500
Как задать значение по умолчанию в Access? Установка значения по умолчанию Свойство Значение по умолчанию определяет значение, ко­торое Access вставит в таблицу автоматически. 1. Щелкните в поле ЭлектроннаяПочта. Перейдите в область свойств поля. 2. Установите курсор в первую позицию строки свойства Значение по умолчанию и введите @dom. ru. Теперь в каждой новой записи в поле ЭлектроннаяПочтабудет появляться текст @dom.ru. Чтобы получить возмож­ность внести изменения в заданное по умолчанию значе­ние или добавить к адресу имя пользователя, просто щелк­ните в этом поле таблицы или выделите его и нажмите клавишу F2. Обязательное поле При вводе данных можно случайно пропустить (не заполнить) некоторые поля. Чтобы этого не произошло, назначьте свойство Обязательное поле. 1. Щелкните в поле КодСотрудника и перейдите к строке свойства Обязательное поле. Щелкните на кнопке со стрел­кой и выберите вариант Да. 2. Повторите эту операцию для всех остальных полей. Другие свойства Поговорим немного об индексированном поле. Когда вы задали ключевое значение поля КодСотрудника, то програм­ма Access указала, что это поле является индексирован­ным и совпадения в нем не допускаются (в строке свой­ства Индексированное поле указан вариант Да (Совпадения не допускаются). Индексированными могут быть текстовые, числовые, денежные поля, а также поля даты/времени. Индекс ускоряет операции выполнения запросов, поиска и сортировки по данному полю. Однако введение индекса немного замедляет ввод, редактирование и обновление данных, так как индексы автоматически обновляются для всей таблицы. Свойство Условие на значение предназначено для задания ограничений на вводимые данные. Тип вводимых данных мы уже установили, но в любое время вы можете доба­вить к ним некоторые параметры данных. Числовые поля можно определить так, чтобы программа допускала ввод только указанных значений. Знаки больше (>) и меньше ( 2. Просмотрите отдельные поля и обратите внимание на внесенные изменения. Поле Имя электронной почты теперь по умолчанию содержит текст @dom.ru. При попытке ввода номера телефона на экране отображается маска ввода, позволяющая ввести в поле точно семь цифр номера. Давайте внесем в таблицу новую информацию. 1. Щелкните в поле КодСотрудникановой записи и введите 1258. Нажмите клавишу Enter. 2. Введите в таблицу следующие данные: Код сотрудника Табельный номер 15F-4695 15F-4368 Имя Игорь Михаил Отчество Романович Юрьевич Фамилия Васильев Иванов Должность Дизайнер Аналитик Имя электронной почты igor.vasiliev@dom. ru [email protected] Рабочий телефон 138-19-13 138-19-16 Код отдела Дата найма 23.09.96 23.08.96 При вводе данных обратите внимание на особенности, кото­рые обусловлены изменениями, внесенными в таблицу. Лабораторная работа №3 3 Работа с записями В данном разделе опишем несколько простых спосо­бов работы с данными и поиска необходимой информации. Найдите сотрудников с фамилией Иванов. Поиск записи Access позволяет легко выполнить поиск определенной записи (или записей). Рассмотрим на примере. 1. Выделить любую фамилию, для чего подведите курсор к левой границе поля и, когда он примет вид значка «плюс», щелкните мышью. 2. Щелкните на кнопке Найти на панели инструментов . Появится окно диалога, как показано на рисунке 15. 3. Щелкните на строке заголовка окна диалога и перета­щите его в сторону, чтобы вы могли видеть необходи­мые записи. 4. Введите текст: Иванов в поле Образец. В списке Совпаде­ниевыберите вариант С любой частью поля. Щелкните на кнопке Найти далее. Access выделит первую найденную запись. 5. Для поиска следующего элемента снова щелкните на кнопке Найти далее. 6. Для выхода из режима поиска щелкните на кнопке От­мена. Перемещение полей По « умолчанию» поля воспроизводятся на экране в режиме таблицы в том порядке, в котором они вводились при со­здании таблицы. Чтобы изменить порядок следования полей, достаточно выделить поле и перетащить его на но­вое место. 1. Выделите поле с подписью Дата приема на работу, щелк­нув на заголовке этого столбца. 2. Перетащите поле влево. При перемещении между столб­цами появляется разделительная линия, которая ука­зывает новое расположение столбца. 3. Когда разделительная линия будет находиться слева от поля Имя, отпустите кнопку мыши. Теперь выделенный столбец занимает третью позицию слева, как показано на рисунке 16. Сортировка записей Для поиска определенной записи (или записей) очень ча­сто применяется сортировка. 1. Выделите поле Фамилия, щелкнув мышью на заголовке соответствующего столбца. 2. Щелкните на кнопке Сортировка по возрастанию на панели инструментов . Рядом находится кнопка Сортировка по убыванию. 3. Щелкните на правой границе заголовка столбца Фами­лияи перетащите его границу в сторону, чтобы смогли увидеть фамилии на экране. Теперь фамилии рас­положены в алфавитном порядке. 4. Закрыть таблицу Сотрудники, сохранив её. Модернизация таблицы Мы создали базу данных, в которую включили про­стую таблицу и попробовали ввести в нее данные. Однако возможности Access намного превосходят такие скромные запросы пользователей. Теперь мы познакомимся с расши­ренными возможностями Access. Сначала мы создадим новую таблицу на основе уже существующей. 1. В окне базы данных в Области переходов вы­делите значок таблицы Сотрудники. По правой кнопке мыши из контекстного меню выберите команду Копировать. 2. Щелкните на кнопке Вставить. На экране появится окно диалога Вставка таблицы, как показано на рисунке 17. 3. В поле Имя таблицы введите текст Заказчики, установите переключатель Только структура и щелкните на кнопке ОК. В Области переходов появится новая таблица Заказчики. 4. Открыть таблицу Заказчики,щёлкнув дважды мышью в Области переходов.Таблица Заказчикиоткроется вРежиме таблицы. С помощью кнопки Вид,расположеннойна Панели быстрого доступа, перейдите в режим Конструктор. 5. Замените имя поля КодСотрудника на КодЗаказчика. В стро­ку свойства Подпись для этого поля введите текст Код заказчика. 6. Щелкните на маркере выделения строки для поля Та­бельный номер и аналогичным образом выделите поля Имя и Отчество. Поочерёдно нажмите клавишу Delete. 7. Измените название поля Фамилия на Фирма. Соответству­ющим образом измените подпись для этого поля. 8. Выделите поочерёдно строки для полей Должность, Код отдела и Дата найма. Удалите эти поля из таблицы. 9. Добавьте в таблицу новые поля Адрес и Город. Выдели­те их и перетащите вверх, чтобы поместить перед по­лем ЭлектроннаяПочта. 10. Замените имя поля РабочийТелефонна Телефон. Изме­ните подпись для этого поля. 11.Добавьте новое поле Код-Города с соответствующей под­писью. Размер поля – 30. Теперь структура таблицы выглядит так, как показано на рисунке 18. 12. Щелкните на кнопке Вид, чтобы перейти в Режим таб­лицы. Сохраните изменения в структуре таблицы. 13. Закройте таблицу Заказчики. Создание форм Вы уже знаете, как можно использовать формы для ввода и просмотра информации, содержащейся в таблицах. Сейчас мы созда-дим форму для ввода информации в таблицу Заказчики. 1. В Главном меню панели быстрого доступа найти вкладку Создание, щёлкнуть на кнопкеДругие формы. Выберите вариант МастерРисунок 19 форм. На экране появится следующее окно диалога, как на рисунке 19. 2. Чтобы выбрать все доступные поля таблицы, щелкните на кнопке », затем — на кнопке Далее. Появится второе окно мастера. 3. Установите переключатель В один столбецрисунок 20. 4. Выбрать стиль формы. Доступные стили можно просмотреть в списке справа, а слева показан внешний вид формы согласно выбранному стилю. Выберите вариант Обычнаяи щелкните на кнопке Далее. 5. Сохраните предложенное имя формы. Установите пе­реключатель Изменить макет формы и завершите работу с мастером, щелкнув на кнопке Готово. Форма откроет­ся в режиме конструктора. Результат смотрите на рисунке 22. Рисунок 21 Рисунок 22 Окно формы в режиме конструктора разделено на три области: заголовок, область данных и примечание. В заголовке формы обычно содержится название фирмы или дата. В области данных расположены элементы управления. 6. Если область данных не видна на экране целиком, не­много растяните макет формы. 7. Один раз щелкните на подписи к полю Электронная поч­та. Access выделит как само поле, так и подпись к нему. 8. Еще раз щелкните на подписи к полю. Оно будет выделено другим цветом, и вы сможете изменить текст подписи. Измените текст на Эл. почта. 9. Щелкните на кнопке Сохранить. 10. Щелкните на кнопке Вид. Откройте форму в режиме формы. 11. Теперь введите данные согласно таблице, приведенной ниже. Напоминаем, что для перехода к новой записи и от поля к полю используется клавиша Tab. Как задать значение по умолчанию в Access? Основные свойства полей MS Access зависят от выбранного типа данных. Если открыть таблицу в режиме конструктора, то весь набор свойств выбранного поля будет представлен в нижней части окна на двух вкладках: Общие (General) и Подстановка (Lookup). Приведем свойства полей, наиболее важные на первом этапе изучения баз данных. 1. Размер поля (Field Size) позволяет для текстового и числового поля уточнить тип данных или размер, задает максимальный размер данных, сохраняемых в поле. Для поля с типом данных Текстовый (Text) задается размер от 1 до 255 знаков. Для поля с типом данных Числовой (Number) можно задать: • Байт (Byte) для целых чисел от 0 до 255, длина поля 1 байт; • Целое (Integer) для целых чисел от –32 768 до +32 767, занимает 2 байта; • Длинное целое (Long Integer) для целых чисел от –2 147 483 648 до +2 147 483 647, занимает 4 байта; • Одинарное с плавающей точкой (Single) для чисел от –3,4 1038 до +3,4 1038 с точностью до 7 знаков, занимает 4 байта; • Двойное с плавающей точкой (Double) для чисел от –1,797 10308 до +1,797 10308 с числом отображаемых десятичных знаков до 15, занимает 8 байтов; • Действительное (Decimal) для целых чисел от –1038 до +1038 (при работе с проектами, которые хранятся в файлах типа adp) и от –1028 до 1028 (mdb и accdb) с числом отображаемых десятичных знаков до 28, занимает 12 байтов; • Код репликации (Replication ID). Глобальный уникальный идентификатор (Globally unique identifier, GUID), занимает 16 байтов. Эти длинные генерируемые случайным образом значения обеспечивают малую вероятность их совпадения. Поля такого типа используются Access для создания системных уникальных идентификаторов реплик, наборов реплик, таблиц, записей и других объектов при репликации баз данных. Могут быть использованы в приложениях пользователя для идентификации строк таблицы, например для идентификации товаров. Читайте также  Как можно узнать email человека? Для поля с типом данных Счетчик (AutoNumber) можно задать: • Длинное целое (Long Integer) — 4 байта; • Код репликации (Replication ID) — 16 байтов. Рекомендуется задавать минимально допустимый размер поля, который понадобится для сохраняемых значений, т. к. сохранение таких полей требует меньше памяти, и обработка данных меньшего размера выполняется быстрее. 1. Формат поля (Format) является форматом отображения выбранного типа данных при выводе их на экран или печать в режиме таблицы, в форме или отчете. В Access определены встроенные стандартные форматы отображения для полей с такими типами данных как Числовой (Number), Дата/время (Date/Time), Логический (Yes/No) и Денежный (Currency). Ряд этих форматов совпадает с настройкой региональных форматов, определяемых в окне Язык и региональные стандарты в Панели правления Windows. Пользователь может создать собственный формат для всех типов данных, кроме Поле объекта OLE (OLE Object), с помощью символов форматирования. 2. Число десятичных знаков (Decimal Places) задает для числового и денежного типов данных количество знаков после запятой. Можно задать число от 0 до 15. По умолчанию (значение Авто (Auto)) это число определяется установкой в свойстве Формат поля (Format). Следует иметь в виду, что установка этого свойства не действует, если свойство Формат поля (Format) не установлено или выбрано значение Основной (General Namber). Свойство Число десятичных знаков (Decimal Places) влияет только на количество десятичных знаков, отображаемых на экране, и не влияет на число сохраняемых десятичных знаков. Для изменения числа сохраняемых знаков нужно изменить свойство Размер поля (Field Size). 3. Подпись (Caption) поля задает текст, который выводится в таблицах, формах, отчетах. 4. Описание (Description) — краткий пользовательский комментарий к полю. 5. Значение по умолчанию (Default Value) определяет текст или выражение, значение которого автоматически вводится в поле при создании новой записи. Например, если задана функция =Now(), то в поле введется текущая дата и время. При добавлении записи в таблицу можно оставить значение, введенное по умолчанию, или ввести другое. Свойство Значение по умолчанию (Default Value) используется только при создании новой записи. Максимальная длина значения свойства составляет 255 знаков. Свойство не определено для полей с типом данных Счетчик (AutoNumber) или Поле объекта OLE (OLE Object). 6. Условие на значение (Validation Rule) позволяет осуществлять контроль ввода, задает ограничения на вводимые значения, при нарушении условий запрещает ввод и выводит текст, заданный свойством Сообщение об ошибке (Validation Text). 7. Сообщение об ошибке (Validation Text) задает текст сообщения, выводимый на экран при нарушении ограничений, заданных свойством Условие на значение (Validation Rule). В следующем уроке рассмотри процесс создания таблиц в MS Access. Задание значений по умолчанию для полей и элементов управления В этой статье объясняется, как установить значение по умолчанию для поля таблицы или для управления в форме базы данных Access. Застроимые значения по умолчанию будут отображаться в поле или в наборе управления при создании записи в базе данных. В этой статье Значения по умолчанию Значение по умолчанию добавляется в поле таблицы или форму, если вы хотите, чтобы Access автоматически вводл значение в новую запись. Например, можно сделать так, чтобы Access всегда добавлял текущую дату в новые заказы. Как правило, к полям таблицы добавляется значение по умолчанию. Чтобы добавить значение, нужно открыть таблицу в Режиме конструктора, а затем ввести значение в свойство «Значение по умолчанию» для поля. Если для поля таблицы за установлено значение по умолчанию, Access применяет это значение ко всем его базам управления. Если элемент управления не связывается с полем таблицы или не связывается с данными в других таблицах, можно установить значение по умолчанию для элементов управления формы. Для полей таблицы можно установить значения по умолчанию: «Текст», «Поле МЕМО», «Число», «Дата/время», «Валюта», «Да/Нет» и «Гиперссылка». Если значение не затеряется, поле остается пустым (пустым), пока вы не введите его. После определения значения по умолчанию Access применяет его ко всем новым добавляемой записям. При этом значение в записи можно изменить с значения по умолчанию на другое, если это не запрещено правилом проверки. Задание значения по умолчанию для поля таблицы Если для поля таблицы за установлено значение по умолчанию, все элементы управления, которые вы связываете с этим полем, будут отображать его значение. Настройка значения по умолчанию В области навигации щелкните правой кнопкой мыши таблицу, которую вы хотите изменить, и выберите «Конструктор». Выберите поле, которое требуется изменить. На вкладке «Общие» введите значение в поле свойства «Значение по умолчанию». Введите значение, которое вы можете ввести, в зависимости от типа данных, установленного для поля. Например, можно ввести =Date(), чтобы вставить текущую дату в поле «Дата/время». Примеры значений по умолчанию см. далее в этой статье в примерах значений поумолчанию. Настройка значения по умолчанию для управления Как правило, это значение устанавливается только в том случае, если его не привязывать к полю таблицы или при связывании с данными в другой таблице. Настройка значения по умолчанию В области навигации щелкните правой кнопкой мыши форму, которую вы хотите изменить, и выберите команду Конструктор. Щелкните правой кнопкой мыши нужный объект управления и выберите «Свойства» или нажмите F4. На вкладке «Все» в окне свойств найдите свойство «Значение по умолчанию» и введите значение по умолчанию. Чтобы сохранить изменения, нажмите сочетание клавиш CTRL+S. Настройка строки по умолчанию для списка или списка со списком По умолчанию в списках и в поле со списком отображаются списки двух типов: списки значений и списки подылки. Список значений — это жестко задаенный список элементов, которые находятся в свойстве «Источник строк» списка или списка. С другой стороны, список подылки извлекает данные из поля подниманий (поля, которое использует запрос для извлечения данных из другой таблицы) и загружает их в поле со списком. Для списков значений и списков под уточняемого списка можно указать, какой элемент списка отображается по умолчанию, но для каждого типа списков нужно выполнять разные процедуры. Ниже объясняется, как установить значения по умолчанию для списка значений и списка подстроек. Настройка списка значений по умолчанию В области навигации щелкните правой кнопкой мыши форму, которую вы хотите изменить, и выберите команду Конструктор. Щелкните правой кнопкой мыши список или текстовое поле, а затем выберите «Свойства» или нажмите F4. Убедитесь, что свойство «Источник строк» содержит список значений. Элементы в списке значений окружены двойными кавычками и разделены за первой 12-й парой. Например, список может выглядеть так: «Хорошо»;» Ярла»;» Poor «. Если свойство «Источник строк» содержит запрос (строку текста, которая начинается с select),перейдите к следующему набору действий. В поле свойства «Значение по умолчанию» введите следующее: В этом control_name имя элемента управления «список» или «поле со списком», а n — номер элемента списка, который вы хотите сделать элементом по умолчанию. Если вы не знаете имя этого управления, взгляните на значение в свойстве Name (Имя) в верхней части листа свойств. Предположим, у вас есть поле со списком «Владелец» и вы хотите использовать третье значение из списка в качестве значения по умолчанию. В поле свойства «Значение по умолчанию» введите следующее: Вместо 3 введите 2, так как itemData имеет нулевое значение, то есть начинает подсчет с нуля, а не с нуля. Сохраните изменения и переключиться обратно в представление формы. Выбранное значение по умолчанию должно отображаться в списке или в поле со списком при добавлении записи в базу данных. Настройка значения по умолчанию для списка подстройки Откройте таблицу, которая содержит исходные данные для поля подытов в представлении таблицы. Обратите внимание на значение ключа, связанное с элементом, который вы хотите сделать значением по умолчанию. Примечание: По умолчанию Access скрывает поля первичного и внешнего ключа. Если ключевое поле не видно, щелкните правой кнопкой мыши любую ячейку в строке заглавных колонок таблицы, выберите «Отобрать столбцы» ив диалоговом окне «Отображите столбцы» выберите поле ключа и нажмите кнопку «Закрыть». В области навигации щелкните правой кнопкой мыши таблицу, содержаную поле подытогов, и выберите «Конструктор». Выберите поле подобрания и на вкладке «Общие» в поле свойства «Значение по умолчанию» введите значение ключа, которое вы указали в шагах 1 и 2. Это значение, соответствующее элементу списка, который вы хотите сделать элементом по умолчанию. Предположим, у вас 10 поставщиков, и вы хотите, чтобы по умолчанию появлялись имена ваших самых часто используемых поставщиков. Для этого найдите значение ключа, которое однозначно определяет поставщика, и введите его в поле свойства «Значение по умолчанию» поля внешнего ключа. Сохраните изменения, перейдите в представление таблицы и введите новую запись. При добавлении записи в поле подстройки отображается значение по умолчанию. Примеры значений по умолчанию В таблице ниже перечислены и описаны некоторые значения по умолчанию. Эти примеры можно адаптировать для использования с собственными данными. Значение поля по умолчанию «Пушкино, Московская область» Нью-Йорк,Н.Y. (обратите внимание, что значение должно быть заключено в кавычках, если оно содержит знаки препинания) На локальном языке компьютера отображается «Да» Настройка свойств полей Хотя в Access существует только 9 типов полей данных, каждый из них допускает разнообразные настройки, осуществляемые с помощью свойств. Впервые в этой книге свойства рассматриваются не с функциональной, а с формальной точки зрения, поэтому уделим этому вопросу больше внимания. Утверждение о том, что бывают текстовые поля, числовые или поля гиперссылок, аналогично утверждению, что тот или иной пищевой продукт представляет собой овощ, фрукт или мясо. Если известно, что в меню будет фрукт, значит, кое-что о будущем завтраке становится понятным, но все же здесь существует масса вариантов. Фрукт может быть виноградом, арбузом, мандарином или апельсином. Кроме того, он будет, например, красным, что снижает количество вариантов. Итак, он красный, сладкий, сочный и маленького, круглого размера — круг поиска в конечном итоге привел к вишне. Читайте также  Какое назначение у тнвд? Цвет, вкус, поверхность и размер фрукта являются его свойствами, т.е. данными, описывающими определенный фрукт и позволяющими отличить его от других фруктов. Кроме того, у каждого свойства есть определенное значение: цвет — красный, вкус — сладкий и т.д. Возвращаясь к Access, мы говорили, что у каждого поля также имеются определенные свойства. Текстовое поле, например, обладает такими свойствами, как Размер, Маска ввода, Подпись и Базовое значение. С помощью значений свойств можно гораздо больше рассказать о конкретном поле. Безусловно, свойства используются не только для того, чтобы отличать одно поле от другого. От размера поля, к примеру, зависит объем данных, которые можно хранить в поле. Таким образом, свойства способны ограничить объем хранимой в базе информации. В следующем разделе этой главы описаны некоторые наиболее важные свойства различных типов данных. Свойства поля с типом данных «Текстовый» Для изучения свойств поля мы создадим новую таблицу Насаждения, в которой будут храниться сведения о семенах, посеянных в саду, в частности, о времени их посадки. Таблица будет построена исключительно в режиме конструктора. Начнем мы с создания одного поля. 1. Запустите Access и загрузите базу данных Растения. 2. Выберите ссылку Таблицы в окне Растения: база данных. 3. Дважды щелкните на ссылке Создание таблицы в режиме конструктора. 4. Введите Грядка в качестве названия поля в первой строке конструктора и Текстовый — как тип данных этого поля. Только что созданная таблица показана на рис. 11.2 в режиме конструктора. Рис. 11.2. Таблица в режиме конструктора Рассмотрим структуру таблицы в режиме конструктора более подробно. Об именах полей и типах данных мы уже неоднократно говорили в этой книге. Как видите, справа от поля типа данных находится свободное пространство для описания поля; введите текст «Грядка, на которой производится посадка» для поля Грядка. Кроме имени поля, типа данных и описания в нижней области окна содержится панель Свойства поля, состоящая их двух вкладок. В этой главе описывается вкладка Общие; свойства, описываемые на вкладке Подстановка, задаются мастером подстановок, поэтому вручную обычно не изменяются (вкратце они описываются в главе 6). На вкладке Общие каждому типу данных соответствует определенный набор свойств. В данный момент отображаются свойства, относящиеся к тестовому типу поля. Некоторые из свойств изменяются крайне редко, например свойства Сжатие Unicode и Режим IME. По мере выбора различных свойств в правой области панели Свойства поля отображаются относящиеся к ним различные советы и описания. Безусловно, для получения расширенной помощи всегда имеется возможность нажать клавишу . Далее приведен список основных свойств текстового поля. • Размер поля. С помощью этого свойства указывается максимальное количество символов, хранимых в поле. Access никогда не допускает превышения лимита, указанного в данном поле. (Как уже говорилось ранее, максимальное количество символов для текстового поля составляет 255.) • Формат поля. Позволяет определить способ отображения данных в поле. Более подробное описание этого поля приводится далее в главе. • Маска ввода. Указание последовательности символов, с которой должны совпадать вводимые в поле данные. Применение поля также описывается далее в этой главе. • Подпись. Свойство, которое применяется вместо имени поля при отображении последнего. Например, столбцы в таблице получат указанные заголовки подписей вместо имен полей. Это позволяет использовать более наглядные имена, доступные для понимания, одновременно применяя имена без пробелов для самих полей. • Значение по умолчанию. Задает значение для поля новой записи. Если значение по умолчанию не заменяется другим значением, оно сохраняется вместе с записью. • Условие на значение. Указывает шаблон, ограничивающий данные, которые вводятся в поле. Более подробное описание свойства приводится далее в главе. • Сообщение об ошибке. Ввод сообщения, отображаемого Access при попытке пользователя нарушить условие на значение. • Обязательное поле. Название говорит само за себя: если указать значение Да, запись в таблице можно будет сохранить только при условии, что будет заполнено поле. В продолжение работы с таблицей Насаждения укажем некоторые свойства поля Грядка, выполнив следующие действия. 1. Введите значение 30 для свойства Размер поля. 2. Для свойства Подпись укажите значение Садовая грядка. 3. Для свойства Обязательное поле укажите значение Да. 4. Выберите поле Прядка в качестве первичного ключа для новой таблицы, щелкнув на этом поле, а затем на кнопке Ключевое поле, расположенной на панели инструментов. 5. Сохраните таблицу Насаждения, выполнив для этого команду Файл>Сохранить. (Сохраненная таблица, по-прежнему в режиме конструктора, показана на рис. 11.3.) Теперь переключите режим отображения таблицы Насаждения, щелкнув на кнопке Вид на панели инструментов. С помощью мыши увеличьте размер столбца и введите некоторые данные (рис. 11.4). Очевидно, что определенные ранее свойства повлияли на пользовательский интерфейс таблицы. Влияние на структуру таблицы оказали и другие свойства, но пока что оно невидимо. Например, в поле Грядка невозможно ввести более 30 символов, так свойство Размер поля жестко ограничивает максимально возможное их количество. Свойство Размер поля относится не только к таблице, поскольку свойства Access в случае необходимости могут наследоваться. Вы поймете, что это значит, выполнив такие действия. 1. Закройте таблицу Насаждения. 2. Выберите эту же таблицу в основном окне базы данных. 3. Щелкните на кнопке Новый объект: автоформа на панели инструментов. Рис. 11.3. Таблица в режиме конструктора с указанными свойствами Рис. 11.4. Введите данные в поле с описанием и подписью На рис. 11.5 показан полученный результат. Свойства полей Подпись и Описание были повторно использованы для поля формы — текст свойства Описание отображается в элементе управления. Зачастую одного взгляда на подобный элемент управления достаточно, чтобы определить, какой тип данных требуется ввести. Наследование поддерживается запросами, отчетами и страницами, а также формами. При желании сохраните форму под именем Насаждения и закройте ее. Для изменения значения такого свойства, как Обязательное поле (со значения Да на значение Нет и наоборот), достаточно дважды щелкнуть на свойстве. Это можно сделать и без мыши, путем ввода первой буквы в названии нужного свойства, a Access автоматически дополнит его. Свойства поля с типом данных «Поле MEMO» Продолжая работу со свойствами поля, добавим к таблице Насаждения поле с типом данных Поле MEMO. 1. Откройте таблицу Насаждения в режиме конструктора. 2. Введите новое поле с именем Заметки. Укажите для него тип данных Поле MEMO. Как видите, значения в окне Свойства поля изменились по сравнению с использованием типа данных Текстовый. 3. Введите описание нового поля — Заметки о насаждениях. 4. Укажите для свойства Подпись значение Заметки насаждений. Рис. 11.5. Форма содержит поле с подписью и описанием Если сравнить списки свойств для типов данных Поле MEMO и Текстовый, окажется, что они практически совпадают. В этом нет ничего удивительного, так как Поле MEMO, по сути, является, «очень вместительным» текстовым полем. Свойства поля с типом данных «Числовой» Тип данных Числовой добавляет некоторые новые свойства и модификации к уже существующим свойствам. Для того чтобы добавить числовое поле к таблице Насаждения, выполните такие действия. 1. Добавьте к таблице новое поле с именем КоличествоПосадок. Установите тип данных Числовой. 2. Введите в поле Описание текст Количество посаженных растений. 3. Щелкните на свойстве Размер поля — справа появится значок со стрелкой. Щелкните на ней для открытия раскрывающегося списка с различными значениями размера поля (рис. 11.6). Выберите опцию Байт. 4. Укажите для свойства Число десятичных знаков значение 0. 5. Укажите для свойства Подпись значение Количество посадок. 6. Укажите для свойства Значение по умолчанию значение 1. Выбирая размер поля для числового типа данных, необходимо учесть ряд обстоятельств и ответить на следующие вопросы: Рис. 11.6. Укажите размер числового поля • Какое может потребоваться максимальное значение? • Будет число целым или дробным? • Существенны ли ошибки при округлении? Для хранения целых чисел лучше всего подходят поля Байт, Целое и Длинное целое. Выберите наименьший тип для хранения данных, поскольку такое поле быстрее всего обрабатывается. Дробные значения хранятся в полях Одинарное с плавающей точкой, Двойное с плавающей точной или Действительное. И опять же, свойство следует выбирать с учетом объема данных, но не следует забывать об округлении. Поля Одинарное с плавающей точкой и Двойное с плавающей точкой могут вернуть значение, несколько отличающееся от того, что вводилось ранее, особенно если значение было многоразрядным. Тип данных Действительное хранит введенное значение без изменений, однако на обработку такого поля уходит больше времени. Для поля КоличествоПосадок было указано свойство Условие на значение. Его можно озвучить так: «Введенное число не должно быть меньше или равным 5». Если условие соблюдается, Access позволит ввести число, в противном случае в поле Сообщение об ошибке появится предупреждение. Проверим на практике, так ли это. 1. Сохраните таблицу Насаждения. На экране появится несколько настораживающее сообщение, которое вы видите на рис. 11.7. Дело в том, что Access желает знать, следует ли использовать новое свойство Условие на значение для существующих строк в таблице. Щелкните на кнопке Нет. 2. Переключитесь в режим таблицы. Обратите внимание на то, что поле Количество посадок уже содержит значение 1 в строке новой записи, как и указано в свойстве Значение по умолчанию. Если бы запись была определена ранее, то значение 1 не было бы введено, поскольку свойство Значение по умолчанию добавлено уже после ввода записи; это свойство применяется только к новым записям. Читайте также  Можно ли не прописывать ребенка до 14 лет? 3. Введите в новую запись какие-либо данные, но обязательно задайте количество посаженных растений, равным 52; при попытке покинуть поле появится сообщение об ошибке, приведенное на рис. 11.8. До тех пор, пока указанное значение не будет изменено, вы не сможете перейти в другое поле или сохранить запись. Рис. 11.7. Предупреждение о возможном нарушении целостности данных 4. Щелкните на кнопке ОК и измените значение поля Количество посадок с 52 на 48. Теперь с помощью кнопки можно перейти из этого поля к любому другому и сохранить запись. Существует возможность создавать самые разнообразные условия на значение, нередко с достаточно сложным синтаксисом. К счастью, зубрить синтаксис нет необходимости — в табл. 11.2 приведены примеры основных условий на значение. Код репликации — это кодированные значения, предназначенные главным образом для использования в базах данных, одновременно хранимых на нескольких компьютерах. Это свойство едва ли вам когда-нибудь понадобится. Рис. 11.8. Сигнал о попытке нарушить свойство Условие на значение Таблица 11.2. Основные условия на значение Как задать значение по умолчанию в Access? Упражнение 6. Параметры поля Для того, чтобы обеспечить возможность хранения в базе данных разнообразной информации, Access предлагает большой набор типов данных, перечисленных в табл. 16.1. Поле объекта OLE (OLE Object) Чтобы сделать работу со списком контактов более продуктивной, следует изменить типы данных некоторых полей. В окне базы данных выделите таблицу Список и щелкните на кнопке Конструктор (Design), расположенной в верхней части окна. Выбранная таблица откроется в режиме конструктора. Описание контакта может быть достаточно длинным, а максимальная длина текстовых полей ограничена 256 символами. Поэтому измените тип поля Описание на Поле MEMO. Щелкните на поле Дата. Рис. 16.14. Выбор формата Для удобства прочтения даты контакта измените 4юрмат ее отображения, выбрав в раскрывающемся списке Формат поля (Format) пункт Длинный формат даты (Long Date), как показано на рис. 16.14. Кроме типа данных и формата отображения вкладка Общие (General) окна конструктора позволяет настроить некоторые дополнительные параметры поля, влияющие на способ ввода данных. Параметр Значение по умолчанию (Default Value) задает содержимое поля, которое записывается в новую запись в том случае, если пользователь не ввел в это поле никаких данных. Если вы регулярно обновляете список контактов, то дата контакта чаще всего будет совпадать с текущей датой. Чтобы упростить ввод информации, можно задать текущую дату в качестве значения по умолчанию для поля Дата. Введите в поле Значение по умолчанию (Default Value) функцию Date(), которая возвращает текущую дату. В некоторых случаях требуется не только задать тип данных поля, но и ограничить его возможные значения некоторым интервалом или более сложным правилом отбора. Такое правило вводится в поле Условие на значение (Validation Rule). Предположим, вы точно знаете, что все контакты, которые будут заноситься в список, состоятся после 1 января 1999 г. Чтобы избежать ошибок, можно заставить Access сообщать вам о вводе неверной даты. Для этого выполните следующие шаги. В поле Условие на значение щелкните на кнопке построителя выражений. Откроется окно диалога. Для задания условий или значений определенных параметров Access предлагает большой набор математических операций и функций. Построитель выражений помогает ориентироваться во всем многообразии функций Access. Верхнее прокручивающееся поле построителя содержит конструируемое выражение. Его можно изменять как путем непосредственного редактирования текста, так и с помощью других кнопок и списков построителя. Необходимо, чтобы дата записи была позднее, то есть больше, даты 1 января 1999 г. Поэтому щелкните на кнопке >, чтобы добавить нужное условие. Теперь нужно указать дату, с которой выполняется сравнение. Дяя этого следует воспользоваться специальной функцией, преобразующей строковую константу в дату в формате Access. Дважды щелкните на папке Функции (Functions) в левом списке построителя выражений. Щелкните на папке Встроенные функции (Built-in Function), чтобы вывести список стандартных функций Access. В среднем списке выберите категорию Дата/время. Двойным щелчком на имени функции DateValue добавьте ее в поле выражения. Вместо местозаполнителя аргумента «stringexpr» нужно ввести конкретную дату в строковом формате. В поле выражения щелчком выделите текст «stringexpr», а затем напечатайте «1.1.99». Щелкните на кнопке ОК. Рис. 16.15. Параметры поля Дата В поле Сообщение об ошибке (Validation Text) введите текст Неверная дата, который будет появляться при вводе даты, более ранней, чем 1 января 1999 г. Окончательный набор параметров поля Дата показан на рис. 16.15. Щелкните на кнопке закрытия окна конструктора. В окне запроса на обновление структуры таблицы щелкните на кнопке Да. По причине добавления условий на значения поля Дата Access запросит о необходимости проверки содержимого этого поля в уже имеющихся записях таблицы. Щелкните на кнопке Нет (No), чтобы не выполнять проверку. Давайте изучим влияние настроенных параметров на поведение базы данных. Выделите таблицу Список в окне базы данных и щелкните на кнопке Открыть (Open). Теперь все даты отображаются в длинном формате, а в ячейке Дата в строке новой записи автоматически появляется текущая дата. Щелкните в ячейке Дата второй строки, измените ее содержимое на 19.11.95 и нажмите клавишу Tab, чтобы перейти к следующему полю. Так как модифицированная дата предшествует 1 января 1999 г., что противоречит настроенному условию, Access не позволит убрать курсор из текущей ячейки и откроет окно сообщения с текстом, введенным вами в поле Сообщение об ошибке конструктора таблицы на шаге 14. Такое сообщение будет появляться до тех пор, пока вы не скорректируете дату. Базы данных Access. Свойства полей Access. Типы данных Access. Создание таблиц в базе данных Access. Всем привет. В этой статье, мы поговорим на такие темы как : Базы данных access, Свойства полей Access, Типы данных Access и Создание таблиц в Access. Access — это реляционная СУБД корпорации Microsoft. Имеет огромный список функций, включая связанные запросы, сортировку по разным полям, связь с внешними таблицами и базами данных. Его преимущество в том, что Access имеет встроенный язык VBA и в самой СУБД можно создавать приложения, которые будут работать с базами данных. Примечание СУБД — Система управления базами данных Создание таблиц в Access. Существуют следующие способы создание таблиц в Access : • Режим таблицы — Реализовывается путем ввода данных. • С помощью мастера — Создания таблиц, на основе предложенного образца. • Режим конструктора — Позволяет не только задать имена полей таблиц, но и выбрать типы данных и задать свойства полей. Каждому способу есть соответствующий ярлык, когда вы создаете базу данных в Access, то кроме этих ярлыков ничего нет. Примечание Если эти ярлыки не видны, необходимо открыть окно Параметры (Options) (с помощью команды Сервис, Параметры (Tools, Option),) затем раскрыть вкладку Вид (View) и установить флажок новые ярлыки объектов (New Object Shortcuts)… Типы данных Access. Ниже даю вам таблицу, в которой написаны Типы данных Access, и их описание. Название поля Описание Text (Тип данных текстовый). Символьные, текстовые данные, объем которых недолжен, превышать 255 символов, по умолчанию 50. Memo (Тип данных текстовый). Текстовый тип данных, ограничения до 64000 символов, поля этого типа не индексируются. Integer (Тип данных числовой). Числовые данные, размер которых определяется, числом, указанным в свойствах полей. Data, Time. Предназначен для хранения даты и времени, или их вместе. Денежный (Тип данных числовой). Разновидность типа данных для хранения, денежных эквивалентов, размером 15 разрядов до запятой, и четыре разряда после. Счетчик (Тип данных числовой). Является разновидностью числового типа, может иметь только целое значение, автоматически увеличивается на заданный шаг, по умолчанию +1, значения не могут повторяться. Логический. Предназначен для хранения логических значений, для команд и операций : ложистинна, данет, truefalse, 1. Ole. Предназначен для хранения объектов (файлов), которые созданы в других приложениях. Поле не индексируется. Гиперссылка В полях храниться ссылки, представляющие собой путь к файлу на каком-либо носители, или ресурсе сети. Мастер подстановок. Позволяет подставлять значение полей, в виде простого или комбинированного списка. Свойства полей Access. Ниже даю вам таблицу, в которой написаны Свойства полей Access, и их описание. Свойства поля Описание Размер поля. Определяет максимальную длину текстового или числового поля, так как если размер подобран неоптимальное, расходуется, лишня память. Формат поля. Устанавливает формат отображения данных в форме, запросе, отчете. Число десятичных знаков. Количество знаков после запятой в десятичном числе. Маска ввода. Задает маску (шаблон), при вводе данных в таблицу или форму. Значение по умолчанию. Содержит значение, установленное по умолчанию, для всех новых записей таблицы. Подпись. Задает подпись поля, которое выводиться в формах, отчетах, таблицах (не путать с именем поля). Условие на значение. Позволяет задать то условие, которое проверяется при вводе данных в поле. Сообщение об ошибке. Задается текст, сообщение выводится в диалоговом окне, если вводимые данные не соответствуют, заданному условию на значение. Обязательное поле. Определяет, может ли поле быть пустым или нет. Пустые строки. Определяет возможность ввода в поля пустых строк с пробелами. Индексированное поле. Задает индексы, для ускоренного поиска информации в таблице. Примечание Если в база данных включает несколько таблиц, то эти таблицы, как правило, должны быть связаны между собой. Связь организуется от родительской таблице к дочерней… На этом данную статью я заканчиваю, надеюсь, вы полностью разобрались с Базами данных Access. Понравилась статья? Поделиться с друзьями: Добавить комментарий ;-) :| :x :twisted: :smile: :shock: :sad: :roll: :razz: :oops: :o :mrgreen: :lol: :idea: :grin: :evil: :cry: :cool: :arrow: :???: :?: :!:
{ "url": "https://carnv.ru/zapchasti/kak-zadat-znachenie-po-umolchaniju-v-access.html", "source_domain": "carnv.ru", "snapshot_id": "crawl=CC-MAIN-2022-05", "warc_metadata": { "Content-Length": "138605", "Content-Type": "application/http; msgtype=response", "WARC-Block-Digest": "sha1:M2SF2ISD5EZSBEEDY7IADKA4C7Q524MK", "WARC-Concurrent-To": "<urn:uuid:dda0f299-02a7-4ee0-8072-d479f2321699>", "WARC-Date": "2022-01-23T17:37:44Z", "WARC-IP-Address": "45.130.41.13", "WARC-Identified-Payload-Type": "text/html", "WARC-Payload-Digest": "sha1:UYYAEWOAKLR4U7BZCH4SXH6D6U6C42X6", "WARC-Record-ID": "<urn:uuid:368e8ab2-a51c-402b-a635-4c343d496232>", "WARC-Target-URI": "https://carnv.ru/zapchasti/kak-zadat-znachenie-po-umolchaniju-v-access.html", "WARC-Truncated": null, "WARC-Type": "response", "WARC-Warcinfo-ID": "<urn:uuid:c83cd500-0c2e-442c-acc4-34052fdc7043>" }, "warc_info": "isPartOf: CC-MAIN-2022-05\r\npublisher: Common Crawl\r\ndescription: Wide crawl of the web for January 2022\r\noperator: Common Crawl Admin ([email protected])\r\nhostname: ip-10-67-67-24\r\nsoftware: Apache Nutch 1.18 (modified, https://github.com/commoncrawl/nutch/)\r\nrobots: checked via crawler-commons 1.3-SNAPSHOT (https://github.com/crawler-commons/crawler-commons)\r\nformat: WARC File Format 1.1\r\nconformsTo: https://iipc.github.io/warc-specifications/specifications/warc-format/warc-1.1/" }
{ "line_start_idx": [ 0, 43, 44, 76, 77, 178, 179, 250, 251, 347, 348, 627, 628, 646, 647, 785, 786, 923, 924, 976, 977, 993, 994, 1600, 1601, 1920, 1921, 1995, 1996, 2230, 2231, 2315, 2316, 2355, 2356, 2371, 2405, 2422, 2449, 2473, 2501, 2567, 2603, 2614, 2643, 2644, 2751, 2752, 2775, 2776, 2796, 2797, 2938, 2939, 2952, 2953, 3050, 3051, 3179, 3180, 3284, 3285, 3402, 3403, 3574, 3575, 3647, 3648, 3707, 3708, 3726, 3727, 4041, 4042, 4181, 4182, 4356, 4357, 4376, 4377, 4459, 4460, 4539, 4540, 4660, 4661, 4843, 4844, 4888, 4889, 4910, 4911, 5213, 5214, 5365, 5366, 5475, 5476, 5644, 5645, 5852, 5853, 5976, 5977, 6130, 6131, 6233, 6234, 6339, 6340, 6468, 6469, 6551, 6552, 6700, 6701, 6802, 6803, 6835, 6836, 6850, 6851, 7022, 7023, 7195, 7196, 7224, 7225, 7347, 7348, 7402, 7403, 7591, 7592, 7807, 7808, 7830, 7831, 8041, 8042, 8125, 8126, 8237, 8238, 8377, 8378, 8411, 8412, 8471, 8472, 8621, 8622, 8665, 8666, 8991, 8992, 9284, 9350, 9429, 9501, 9541, 9611, 9670, 9741, 9816, 9832, 9911, 9990, 10070, 10591, 10639, 10640, 10699, 10700, 10744, 10793, 10794, 11007, 11008, 11628, 12312, 12401, 12476, 13063, 13298, 13469, 13470, 13535, 13536, 13599, 13600, 13846, 13847, 13861, 13862, 13884, 13885, 14106, 14107, 14604, 14605, 15042, 15043, 15090, 15091, 15243, 15244, 15276, 15277, 15389, 15390, 15433, 15434, 15511, 15512, 15779, 15780, 15827, 15828, 15977, 15978, 16010, 16011, 16127, 16128, 16220, 16221, 16327, 16328, 16388, 16389, 16452, 16453, 16861, 16862, 17136, 17137, 17176, 17177, 17293, 17294, 17394, 17395, 17616, 17617, 17749, 17750, 17809, 17810, 18080, 18081, 18276, 18277, 18384, 18385, 18572, 18573, 18627, 18628, 18721, 18722, 18832, 18833, 19131, 19132, 19243, 19244, 19485, 19486, 19765, 19766, 19921, 19922, 19952, 19953, 20096, 20097, 20124, 20125, 20155, 20156, 20273, 20274, 20322, 20323, 20347, 20348, 20627, 20628, 21194, 21195, 21235, 21236, 21767, 21768, 22017, 22018, 22116, 22117, 22158, 22159, 22417, 22418, 22472, 22473, 22530, 22531, 22600, 22601, 22788, 22789, 22830, 22831, 23450, 23451, 23916, 23917, 23973, 23974, 24249, 24383, 24543, 24849, 25008, 25159, 25276, 25441, 25442, 25553, 25554, 25603, 25604, 25661, 25662, 25717, 25718, 25882, 25883, 26040, 26041, 26314, 26315, 26540, 26541, 26716, 26717, 26749, 26750, 26806, 26807, 26877, 26878, 26943, 26944, 27000, 27001, 27446, 27447, 27715, 27716, 27757, 27758, 27855, 27856, 27910, 27911, 28096, 28097, 28154, 28155, 28216, 28217, 28271, 28272, 28492, 28493, 28533, 28534, 28719, 28720, 28813, 28814, 28879, 28880, 29077, 29078, 29138, 29139, 29200, 29201, 29259, 29260, 29376, 29377, 29418, 29419, 29472, 29507, 29549, 29550, 29726, 29727, 30244, 30245, 30554, 30555, 30830, 30831, 31209, 31210, 31268, 31269, 31575, 31576, 31643, 31644, 31814, 31815, 32041, 32042, 32252, 32253, 32319, 32320, 32363, 32364, 32407, 32408, 32437, 32438, 32600, 32601, 32631, 32632, 32740, 32741, 32914, 32915, 33076, 33077, 33100, 33101, 33127, 33128, 33316, 33317, 33882, 33883, 34447, 34448, 34949, 34950, 35093, 35094, 35269, 35270, 35353, 35354, 35461, 35462, 35510, 35511, 35681, 35682, 35766, 35767, 35790, 35791, 35823, 35824, 36038, 36039, 36363, 36364, 36436, 36437, 36642, 36643, 37111, 37112, 37217, 37218, 37373, 37374, 37704, 37705, 37720, 37721, 37765, 37766, 37796, 37797, 37857, 37858, 37916, 37992, 38116, 38117, 38239, 38240, 38255, 38256, 38485, 38486, 38510, 38511, 38591, 38592, 38619, 38747, 38867, 38985, 39058, 39214, 39417, 39537, 39652, 39770, 39877, 39878, 39905, 39906, 39989, 39990, 40017, 40166, 40249, 40330, 40408, 40514, 40622, 40721, 40871, 40941, 41022, 41109, 41110, 41125, 41126, 41293, 41294, 41392, 41393, 41436, 41457, 41458 ], "line_end_idx": [ 43, 44, 76, 77, 178, 179, 250, 251, 347, 348, 627, 628, 646, 647, 785, 786, 923, 924, 976, 977, 993, 994, 1600, 1601, 1920, 1921, 1995, 1996, 2230, 2231, 2315, 2316, 2355, 2356, 2371, 2405, 2422, 2449, 2473, 2501, 2567, 2603, 2614, 2643, 2644, 2751, 2752, 2775, 2776, 2796, 2797, 2938, 2939, 2952, 2953, 3050, 3051, 3179, 3180, 3284, 3285, 3402, 3403, 3574, 3575, 3647, 3648, 3707, 3708, 3726, 3727, 4041, 4042, 4181, 4182, 4356, 4357, 4376, 4377, 4459, 4460, 4539, 4540, 4660, 4661, 4843, 4844, 4888, 4889, 4910, 4911, 5213, 5214, 5365, 5366, 5475, 5476, 5644, 5645, 5852, 5853, 5976, 5977, 6130, 6131, 6233, 6234, 6339, 6340, 6468, 6469, 6551, 6552, 6700, 6701, 6802, 6803, 6835, 6836, 6850, 6851, 7022, 7023, 7195, 7196, 7224, 7225, 7347, 7348, 7402, 7403, 7591, 7592, 7807, 7808, 7830, 7831, 8041, 8042, 8125, 8126, 8237, 8238, 8377, 8378, 8411, 8412, 8471, 8472, 8621, 8622, 8665, 8666, 8991, 8992, 9284, 9350, 9429, 9501, 9541, 9611, 9670, 9741, 9816, 9832, 9911, 9990, 10070, 10591, 10639, 10640, 10699, 10700, 10744, 10793, 10794, 11007, 11008, 11628, 12312, 12401, 12476, 13063, 13298, 13469, 13470, 13535, 13536, 13599, 13600, 13846, 13847, 13861, 13862, 13884, 13885, 14106, 14107, 14604, 14605, 15042, 15043, 15090, 15091, 15243, 15244, 15276, 15277, 15389, 15390, 15433, 15434, 15511, 15512, 15779, 15780, 15827, 15828, 15977, 15978, 16010, 16011, 16127, 16128, 16220, 16221, 16327, 16328, 16388, 16389, 16452, 16453, 16861, 16862, 17136, 17137, 17176, 17177, 17293, 17294, 17394, 17395, 17616, 17617, 17749, 17750, 17809, 17810, 18080, 18081, 18276, 18277, 18384, 18385, 18572, 18573, 18627, 18628, 18721, 18722, 18832, 18833, 19131, 19132, 19243, 19244, 19485, 19486, 19765, 19766, 19921, 19922, 19952, 19953, 20096, 20097, 20124, 20125, 20155, 20156, 20273, 20274, 20322, 20323, 20347, 20348, 20627, 20628, 21194, 21195, 21235, 21236, 21767, 21768, 22017, 22018, 22116, 22117, 22158, 22159, 22417, 22418, 22472, 22473, 22530, 22531, 22600, 22601, 22788, 22789, 22830, 22831, 23450, 23451, 23916, 23917, 23973, 23974, 24249, 24383, 24543, 24849, 25008, 25159, 25276, 25441, 25442, 25553, 25554, 25603, 25604, 25661, 25662, 25717, 25718, 25882, 25883, 26040, 26041, 26314, 26315, 26540, 26541, 26716, 26717, 26749, 26750, 26806, 26807, 26877, 26878, 26943, 26944, 27000, 27001, 27446, 27447, 27715, 27716, 27757, 27758, 27855, 27856, 27910, 27911, 28096, 28097, 28154, 28155, 28216, 28217, 28271, 28272, 28492, 28493, 28533, 28534, 28719, 28720, 28813, 28814, 28879, 28880, 29077, 29078, 29138, 29139, 29200, 29201, 29259, 29260, 29376, 29377, 29418, 29419, 29472, 29507, 29549, 29550, 29726, 29727, 30244, 30245, 30554, 30555, 30830, 30831, 31209, 31210, 31268, 31269, 31575, 31576, 31643, 31644, 31814, 31815, 32041, 32042, 32252, 32253, 32319, 32320, 32363, 32364, 32407, 32408, 32437, 32438, 32600, 32601, 32631, 32632, 32740, 32741, 32914, 32915, 33076, 33077, 33100, 33101, 33127, 33128, 33316, 33317, 33882, 33883, 34447, 34448, 34949, 34950, 35093, 35094, 35269, 35270, 35353, 35354, 35461, 35462, 35510, 35511, 35681, 35682, 35766, 35767, 35790, 35791, 35823, 35824, 36038, 36039, 36363, 36364, 36436, 36437, 36642, 36643, 37111, 37112, 37217, 37218, 37373, 37374, 37704, 37705, 37720, 37721, 37765, 37766, 37796, 37797, 37857, 37858, 37916, 37992, 38116, 38117, 38239, 38240, 38255, 38256, 38485, 38486, 38510, 38511, 38591, 38592, 38619, 38747, 38867, 38985, 39058, 39214, 39417, 39537, 39652, 39770, 39877, 39878, 39905, 39906, 39989, 39990, 40017, 40166, 40249, 40330, 40408, 40514, 40622, 40721, 40871, 40941, 41022, 41109, 41110, 41125, 41126, 41293, 41294, 41392, 41393, 41436, 41457, 41458, 41595 ] }
{ "red_pajama_v2": { "ccnet_original_length": 41595, "ccnet_original_nlines": 545, "rps_doc_curly_bracket": 0, "rps_doc_ldnoobw_words": 0, "rps_doc_lorem_ipsum": 0, "rps_doc_stop_word_fraction": 0.0006834300002083182, "rps_doc_ut1_blacklist": 0, "rps_doc_frac_all_caps_words": 0.010934939607977867, "rps_doc_frac_lines_end_with_ellipsis": 0.003662999952211976, "rps_doc_frac_no_alph_words": 0.9689721465110779, "rps_doc_frac_unique_words": 0.304578959941864, "rps_doc_mean_word_length": 5.8928141593933105, "rps_doc_num_sentences": 604, "rps_doc_symbol_to_word_ratio": 0.00027337000938132405, "rps_doc_unigram_entropy": 6.4440388679504395, "rps_doc_word_count": 5831, "rps_doc_frac_chars_dupe_10grams": 0.010622509755194187, "rps_doc_frac_chars_dupe_5grams": 0.08454351872205734, "rps_doc_frac_chars_dupe_6grams": 0.03797909989953041, "rps_doc_frac_chars_dupe_7grams": 0.032420478761196136, "rps_doc_frac_chars_dupe_8grams": 0.0293646901845932, "rps_doc_frac_chars_dupe_9grams": 0.016617679968476295, "rps_doc_frac_chars_top_2gram": 0.01888768933713436, "rps_doc_frac_chars_top_3gram": 0.013270859606564045, "rps_doc_frac_chars_top_4gram": 0.005500419996678829, "rps_doc_books_importance": -3340.479736328125, "rps_doc_books_importance_length_correction": -3340.479736328125, "rps_doc_openwebtext_importance": -2136.39453125, "rps_doc_openwebtext_importance_length_correction": -2136.39453125, "rps_doc_wikipedia_importance": -1578.710205078125, "rps_doc_wikipedia_importance_length_correction": -1578.710205078125 }, "fasttext": { "dclm": 0.9037362337112427, "english": 0, "fineweb_edu_approx": 2.21148943901062, "eai_general_math": -0.000009420000424142927, "eai_open_web_math": 0.34802865982055664, "eai_web_code": 0.307369589805603 } }
{ "free_decimal_correspondence": { "primary": { "code": "005.74", "labels": { "level_1": "General works, books and libraries, information sciences", "level_2": "", "level_3": "Computer programming" } }, "secondary": { "code": "005.1", "labels": { "level_1": "General works, books and libraries, information sciences", "level_2": "", "level_3": "Computer programming" } } }, "bloom_cognitive_process": { "primary": { "code": "3", "label": "Apply" }, "secondary": { "code": "2", "label": "Understand" } }, "bloom_knowledge_domain": { "primary": { "code": "3", "label": "Procedural" }, "secondary": { "code": "2", "label": "Conceptual" } }, "document_type_v1": { "primary": { "code": "3", "label": "Reference/Encyclopedic/Educational" }, "secondary": { "code": "-1", "label": "Abstain" } }, "extraction_artifacts": { "primary": { "code": "3", "label": "Irrelevant Content" }, "secondary": { "code": "0", "label": "No Artifacts" } }, "missing_content": { "primary": { "code": "4", "label": "Missing Images or Figures" }, "secondary": { "code": "0", "label": "No missing content" } }, "document_type_v2": { "primary": { "code": "23", "label": "Tutorial" }, "secondary": { "code": "8", "label": "Documentation" } }, "reasoning_depth": { "primary": { "code": "2", "label": "Basic Reasoning" }, "secondary": { "code": "3", "label": "Intermediate Reasoning" } }, "technical_correctness": { "primary": { "code": "4", "label": "Highly Correct" }, "secondary": { "code": "3", "label": "Mostly Correct" } }, "education_level": { "primary": { "code": "2", "label": "High School Level" }, "secondary": { "code": "1", "label": "General Audience" } } }
672f1e42c33a7f9846924a2431ea77df
4,938,797,926,569,649,000
GMAT Quant Practice | GMAT Rates Q6 Average Speed | Time Speed Distance | GMAT Sample Questions This GMAT quant practice question is from rates - speed, distance, and time. The concept tested is to compute the average speed of travel. An easy, GMAT 550 level, quantitative reasoning sample question. Question 6: Steve traveled the first 2 hours of his journey at 40 mph and the remaining 3 hours of his journey at 80 mph. What is his average speed for the entire journey? 1. 60 mph 2. 56.67 mph 3. 53.33 mph 4. 64 mph 5. 66.67 mph Get to 705+ in the GMAT Online GMAT Course @ INR 6000 Video Explanation GMAT Live Online Classes Starts Sun, Mar 3, 2024 Explanatory Answer | GMAT Average Speed Step 1: Compute Total Distance Travelled Average speed of travel = \\frac{\text{Total distance travelled}}{\text{Total time taken}}) Total distance traveled by Steve = Distance covered in the first 2 hours + distance covered in the next 3 hours. Distance covered in the first 2 hours = speed × time = 40 × 2 = 80 miles. Distance covered in the next 3 hours = speed × time = 80 × 3 = 240 miles. Therefore, total distance covered = 80 + 240 = 320 miles. Step 2: Compute Average Speed Total time taken = 2 + 3 = 5 hours. Total distance travelled = 320 miles. Hence, average speed = \\frac{\text{Total distance travelled}}{\text{Total time taken}} = \frac{320}{5}) = 64 miles per hour. Choice D is the correct answer. GMAT Online Course Try it free! Register in 2 easy steps and Start learning in 5 minutes! ★ Sign up Already have an Account? ★ Login GMAT Live Online Classes Next Batch Mar 3, 2024 ★ GMAT Live Info GMAT Rates Videos On YouTube GMAT Sample Questions | Topicwise GMAT Questions Work @ Wizako How to reach Wizako? Mobile: (91) 95000 48484 WhatsApp: WhatsApp Now Email: [email protected] Leave A Message
{ "url": "https://practice-questions.wizako.com/gmat/quant/rates-work-speed/computing-average-speed-6.shtml", "source_domain": "practice-questions.wizako.com", "snapshot_id": "CC-MAIN-2024-10", "warc_metadata": { "Content-Length": "62459", "Content-Type": "application/http; msgtype=response", "WARC-Block-Digest": "sha1:LUX6ZYRF6N2VUZ2N7RD7LY3BPTZFOGOH", "WARC-Concurrent-To": "<urn:uuid:f6035b69-cf61-48b1-bc5b-ac2b500ef2a2>", "WARC-Date": "2024-02-26T08:59:01Z", "WARC-IP-Address": "13.127.123.33", "WARC-Identified-Payload-Type": "text/html", "WARC-Payload-Digest": "sha1:M6CSCONQ37F7U4BVDIBKVBYNUD4B7DT2", "WARC-Record-ID": "<urn:uuid:3da7aa09-63ee-4b48-90ff-363985ffa1ef>", "WARC-Target-URI": "https://practice-questions.wizako.com/gmat/quant/rates-work-speed/computing-average-speed-6.shtml", "WARC-Truncated": null, "WARC-Type": "response", "WARC-Warcinfo-ID": "<urn:uuid:ad317abf-0313-4016-8049-0c8f04fabc04>" }, "warc_info": "isPartOf: CC-MAIN-2024-10\r\npublisher: Common Crawl\r\ndescription: Wide crawl of the web for February/March 2024\r\noperator: Common Crawl Admin ([email protected])\r\nhostname: ip-10-67-67-208\r\nsoftware: Apache Nutch 1.19 (modified, https://github.com/commoncrawl/nutch/)\r\nrobots: checked via crawler-commons 1.5-SNAPSHOT (https://github.com/crawler-commons/crawler-commons)\r\nformat: WARC File Format 1.1\r\nconformsTo: https://iipc.github.io/warc-specifications/specifications/warc-format/warc-1.1/" }
{ "line_start_idx": [ 0, 36, 37, 97, 98, 302, 303, 475, 476, 488, 503, 518, 530, 545, 546, 570, 571, 572, 591, 602, 603, 604, 622, 623, 624, 649, 650, 651, 675, 676, 677, 717, 718, 759, 760, 852, 965, 1039, 1113, 1171, 1172, 1202, 1203, 1239, 1277, 1403, 1404, 1436, 1437, 1438, 1439, 1458, 1471, 1472, 1501, 1530, 1531, 1541, 1542, 1567, 1568, 1576, 1577, 1602, 1603, 1626, 1627, 1644, 1645, 1674, 1675, 1676, 1725, 1726, 1727, 1741, 1742, 1763, 1764, 1789, 1812, 1836 ], "line_end_idx": [ 36, 37, 97, 98, 302, 303, 475, 476, 488, 503, 518, 530, 545, 546, 570, 571, 572, 591, 602, 603, 604, 622, 623, 624, 649, 650, 651, 675, 676, 677, 717, 718, 759, 760, 852, 965, 1039, 1113, 1171, 1172, 1202, 1203, 1239, 1277, 1403, 1404, 1436, 1437, 1438, 1439, 1458, 1471, 1472, 1501, 1530, 1531, 1541, 1542, 1567, 1568, 1576, 1577, 1602, 1603, 1626, 1627, 1644, 1645, 1674, 1675, 1676, 1725, 1726, 1727, 1741, 1742, 1763, 1764, 1789, 1812, 1836, 1851 ] }
{ "red_pajama_v2": { "ccnet_original_length": 1851, "ccnet_original_nlines": 81, "rps_doc_curly_bracket": 0.010804969817399979, "rps_doc_ldnoobw_words": 0, "rps_doc_lorem_ipsum": 0, "rps_doc_stop_word_fraction": 0.13216957449913025, "rps_doc_ut1_blacklist": 0, "rps_doc_frac_all_caps_words": 0.04738155007362366, "rps_doc_frac_lines_end_with_ellipsis": 0, "rps_doc_frac_no_alph_words": 0.36658352613449097, "rps_doc_frac_unique_words": 0.42953020334243774, "rps_doc_mean_word_length": 4.620805263519287, "rps_doc_num_sentences": 27, "rps_doc_symbol_to_word_ratio": 0, "rps_doc_unigram_entropy": 4.49190092086792, "rps_doc_word_count": 298, "rps_doc_frac_chars_dupe_10grams": 0, "rps_doc_frac_chars_dupe_5grams": 0.1859114021062851, "rps_doc_frac_chars_dupe_6grams": 0.08859840035438538, "rps_doc_frac_chars_dupe_7grams": 0.08859840035438538, "rps_doc_frac_chars_dupe_8grams": 0, "rps_doc_frac_chars_dupe_9grams": 0, "rps_doc_frac_chars_top_2gram": 0.061002179980278015, "rps_doc_frac_chars_top_3gram": 0.04938272014260292, "rps_doc_frac_chars_top_4gram": 0.05809731036424637, "rps_doc_books_importance": -134.92535400390625, "rps_doc_books_importance_length_correction": -134.69473266601562, "rps_doc_openwebtext_importance": -79.74188995361328, "rps_doc_openwebtext_importance_length_correction": -79.74188995361328, "rps_doc_wikipedia_importance": -52.33375549316406, "rps_doc_wikipedia_importance_length_correction": -52.333560943603516 }, "fasttext": { "dclm": 0.8597005009651184, "english": 0.8603203892707825, "fineweb_edu_approx": 3.1631879806518555, "eai_general_math": 0.9647905826568604, "eai_open_web_math": 0.7219842672348022, "eai_web_code": 0.0011904200073331594 } }
{ "free_decimal_correspondence": { "primary": { "code": "510", "labels": { "level_1": "Science and Natural history", "level_2": "Mathematics", "level_3": "" } }, "secondary": { "code": "378.166", "labels": { "level_1": "Social sciences", "level_2": "Education", "level_3": "Education, Higher and Universities and colleges" } } }, "bloom_cognitive_process": { "primary": { "code": "3", "label": "Apply" }, "secondary": { "code": "2", "label": "Understand" } }, "bloom_knowledge_domain": { "primary": { "code": "3", "label": "Procedural" }, "secondary": { "code": "2", "label": "Conceptual" } }, "document_type_v1": { "primary": { "code": "3", "label": "Reference/Encyclopedic/Educational" }, "secondary": { "code": "-1", "label": "Abstain" } }, "extraction_artifacts": { "primary": { "code": "3", "label": "Irrelevant Content" }, "secondary": { "code": "0", "label": "No Artifacts" } }, "missing_content": { "primary": { "code": "0", "label": "No missing content" }, "secondary": { "code": "-1", "label": "Abstain" } }, "document_type_v2": { "primary": { "code": "23", "label": "Tutorial" }, "secondary": { "code": "10", "label": "Knowledge Article" } }, "reasoning_depth": { "primary": { "code": "2", "label": "Basic Reasoning" }, "secondary": { "code": "3", "label": "Intermediate Reasoning" } }, "technical_correctness": { "primary": { "code": "4", "label": "Highly Correct" }, "secondary": { "code": "3", "label": "Mostly Correct" } }, "education_level": { "primary": { "code": "2", "label": "High School Level" }, "secondary": { "code": "3", "label": "Undergraduate Level" } } }
672f1e42c33a7f9846924a2431ea77df
-7,291,017,537,084,043,000
Составить программу вычисления числового ряда для известного числа ряда N. Y=((3^3)-1) ((3^-2)+2) ((3^1)-4) ((3^-0)-8) — QBasic(Бейсик) DEFDBL M-Y CLS INPUT "Vvedite kol-vo chlenov ryada: ", n m = 1: y = 1 FOR i = 0 TO n - 1 y = y * (3 ^ ((3 - i) * m) + (-m * 2 ^ i)) m = -m NEXT PRINT "Y = "; INT(y)
{ "url": "https://pgusapriem.ru/sostavit-programmu-vychisleniya-chislovogo-ryada-dlya-izvestnogo-chisla-ryada-n-y33-1-3-2-2-31-4-3-0-8-qbasicbejsik/", "source_domain": "pgusapriem.ru", "snapshot_id": "CC-MAIN-2024-22", "warc_metadata": { "Content-Length": "62887", "Content-Type": "application/http; msgtype=response", "WARC-Block-Digest": "sha1:ZYK5BT2CA6JGDIYJXKMFV32QSCDTTL7Z", "WARC-Concurrent-To": "<urn:uuid:affdf283-bb90-4129-a33d-682d472979a8>", "WARC-Date": "2024-05-29T02:34:23Z", "WARC-IP-Address": "91.219.194.4", "WARC-Identified-Payload-Type": "text/html", "WARC-Payload-Digest": "sha1:KIZ3LYFKJQIMDOKBPPMJTUAORP6TUGSM", "WARC-Record-ID": "<urn:uuid:60249f40-1edf-4ea8-b9b4-952f8ee7b0bf>", "WARC-Target-URI": "https://pgusapriem.ru/sostavit-programmu-vychisleniya-chislovogo-ryada-dlya-izvestnogo-chisla-ryada-n-y33-1-3-2-2-31-4-3-0-8-qbasicbejsik/", "WARC-Truncated": null, "WARC-Type": "response", "WARC-Warcinfo-ID": "<urn:uuid:f80b1dbc-552e-4199-b332-786e66d9a8d9>" }, "warc_info": "isPartOf: CC-MAIN-2024-22\r\npublisher: Common Crawl\r\ndescription: Wide crawl of the web for May 2024\r\noperator: Common Crawl Admin ([email protected])\r\nhostname: ip-10-67-67-76\r\nsoftware: Apache Nutch 1.19 (modified, https://github.com/commoncrawl/nutch/)\r\nrobots: checked via crawler-commons 1.5-SNAPSHOT (https://github.com/crawler-commons/crawler-commons)\r\nformat: WARC File Format 1.1\r\nconformsTo: https://iipc.github.io/warc-specifications/specifications/warc-format/warc-1.1/" }
{ "line_start_idx": [ 0, 136, 137, 148, 154, 196, 209, 228, 274, 284, 289 ], "line_end_idx": [ 136, 137, 148, 154, 196, 209, 228, 274, 284, 289, 309 ] }
{ "red_pajama_v2": { "ccnet_original_length": 309, "ccnet_original_nlines": 10, "rps_doc_curly_bracket": 0, "rps_doc_ldnoobw_words": 0, "rps_doc_lorem_ipsum": 0, "rps_doc_stop_word_fraction": 0.12389381229877472, "rps_doc_ut1_blacklist": 0, "rps_doc_frac_all_caps_words": 0.1150442510843277, "rps_doc_frac_lines_end_with_ellipsis": 0, "rps_doc_frac_no_alph_words": 0.7079645991325378, "rps_doc_frac_unique_words": 0.699999988079071, "rps_doc_mean_word_length": 3.4000000953674316, "rps_doc_num_sentences": 2, "rps_doc_symbol_to_word_ratio": 0, "rps_doc_unigram_entropy": 3.3869736194610596, "rps_doc_word_count": 50, "rps_doc_frac_chars_dupe_10grams": 0, "rps_doc_frac_chars_dupe_5grams": 0, "rps_doc_frac_chars_dupe_6grams": 0, "rps_doc_frac_chars_dupe_7grams": 0, "rps_doc_frac_chars_dupe_8grams": 0, "rps_doc_frac_chars_dupe_9grams": 0, "rps_doc_frac_chars_top_2gram": 0.023529410362243652, "rps_doc_frac_chars_top_3gram": 0.03529411926865578, "rps_doc_frac_chars_top_4gram": 0, "rps_doc_books_importance": -46.33544921875, "rps_doc_books_importance_length_correction": -46.33544921875, "rps_doc_openwebtext_importance": -39.14719772338867, "rps_doc_openwebtext_importance_length_correction": -39.14719772338867, "rps_doc_wikipedia_importance": -25.791824340820312, "rps_doc_wikipedia_importance_length_correction": -25.791824340820312 }, "fasttext": { "dclm": 0.9976088404655457, "english": 0.02539725974202156, "fineweb_edu_approx": 3.13950777053833, "eai_general_math": 0.9416186809539795, "eai_open_web_math": 0.3901137709617615, "eai_web_code": 0.8613191843032837 } }
{ "free_decimal_correspondence": { "primary": { "code": "005.1", "labels": { "level_1": "General works, books and libraries, information sciences", "level_2": "", "level_3": "Computer programming" } }, "secondary": { "code": "512", "labels": { "level_1": "Science and Natural history", "level_2": "Mathematics", "level_3": "Algebra" } } }, "bloom_cognitive_process": { "primary": { "code": "3", "label": "Apply" }, "secondary": { "code": "2", "label": "Understand" } }, "bloom_knowledge_domain": { "primary": { "code": "3", "label": "Procedural" }, "secondary": { "code": "2", "label": "Conceptual" } }, "document_type_v1": { "primary": { "code": "4", "label": "Code/Software" }, "secondary": { "code": "3", "label": "Reference/Encyclopedic/Educational" } }, "extraction_artifacts": { "primary": { "code": "0", "label": "No Artifacts" }, "secondary": { "code": "-1", "label": "Abstain" } }, "missing_content": { "primary": { "code": "0", "label": "No missing content" }, "secondary": { "code": "-1", "label": "Abstain" } }, "document_type_v2": { "primary": { "code": "8", "label": "Documentation" }, "secondary": { "code": "23", "label": "Tutorial" } }, "reasoning_depth": { "primary": { "code": "2", "label": "Basic Reasoning" }, "secondary": { "code": "3", "label": "Intermediate Reasoning" } }, "technical_correctness": { "primary": { "code": "4", "label": "Highly Correct" }, "secondary": { "code": "3", "label": "Mostly Correct" } }, "education_level": { "primary": { "code": "2", "label": "High School Level" }, "secondary": { "code": "3", "label": "Undergraduate Level" } } }
672f1e42c33a7f9846924a2431ea77df
-5,728,184,810,317,813,000
Back Explore Courses Blog Tutorials Interview Questions 0 votes 1 view in Python by (45.3k points) Why do I get the following error when doing this in python: >>> import locale >>> print str( locale.getlocale() ) (None, None) >>> locale.setlocale(locale.LC_ALL, 'de_DE') Traceback (most recent call last):   File "<stdin>", line 1, in <module>   File "/usr/lib/python2.7/locale.py", line 531, in setlocale     return _setlocale(category, locale) locale.Error: unsupported locale setting This works with other locales like fr or nl as well. I'm using Ubuntu 11.04. Update: Doing the following did not yield anything: dpkg-reconfigure locales perl: warning: Setting locale failed. perl: warning: Please check that your locale settings:     LANGUAGE = (unset),     LC_ALL = (unset),     LC_CTYPE = "UTF-8",     LANG = (unset)     are supported and installed on your system. perl: warning: Falling back to the standard locale ("C"). locale: Cannot set LC_CTYPE to default locale: No such file or directory locale: Cannot set LC_ALL to default locale: No such file or directory 1 Answer 0 votes by (16.8k points) Run following commands export LC_ALL="en_US.UTF-8" export LC_CTYPE="en_US.UTF-8" sudo dpkg-reconfigure locales It will solve this. Make sure to match the .UTF-8 part to the actual syntax found in the output of locale -a e.g. .utf8 on some systems. Browse Categories ...
{ "url": "https://intellipaat.com/community/15112/python-locale-error-unsupported-locale-setting", "source_domain": "intellipaat.com", "snapshot_id": "crawl=CC-MAIN-2022-27", "warc_metadata": { "Content-Length": "85972", "Content-Type": "application/http; msgtype=response", "WARC-Block-Digest": "sha1:KXSQ4NXLOF4QBGHWRTO4SWTQ4KR5YJWI", "WARC-Concurrent-To": "<urn:uuid:36977cff-0082-4ce1-9960-ab7ccf8728b9>", "WARC-Date": "2022-06-26T08:15:41Z", "WARC-IP-Address": "104.18.27.176", "WARC-Identified-Payload-Type": "text/html", "WARC-Payload-Digest": "sha1:AQFLBNHE7HZIHDPKCVB7UO4JLOPFIENP", "WARC-Record-ID": "<urn:uuid:49b3fd6e-e102-4144-9adf-bc9d294aefb1>", "WARC-Target-URI": "https://intellipaat.com/community/15112/python-locale-error-unsupported-locale-setting", "WARC-Truncated": null, "WARC-Type": "response", "WARC-Warcinfo-ID": "<urn:uuid:bce481c7-bccd-45ff-8ac0-6c1210f062a7>" }, "warc_info": "isPartOf: CC-MAIN-2022-27\r\npublisher: Common Crawl\r\ndescription: Wide crawl of the web for June/July 2022\r\noperator: Common Crawl Admin ([email protected])\r\nhostname: ip-10-67-67-34\r\nsoftware: Apache Nutch 1.18 (modified, https://github.com/commoncrawl/nutch/)\r\nrobots: checked via crawler-commons 1.3-SNAPSHOT (https://github.com/crawler-commons/crawler-commons)\r\nformat: WARC File Format 1.1\r\nconformsTo: https://iipc.github.io/warc-specifications/specifications/warc-format/warc-1.1/" }
{ "line_start_idx": [ 0, 5, 6, 57, 65, 72, 100, 101, 161, 162, 180, 181, 217, 218, 231, 232, 277, 278, 313, 314, 352, 353, 415, 416, 456, 457, 498, 499, 576, 577, 629, 630, 655, 656, 694, 695, 750, 751, 775, 776, 798, 799, 823, 824, 843, 844, 892, 893, 951, 952, 1025, 1026, 1097, 1098, 1107, 1108, 1116, 1134, 1135, 1158, 1159, 1187, 1188, 1218, 1219, 1249, 1250, 1270, 1271, 1388, 1389, 1407, 1408 ], "line_end_idx": [ 5, 6, 57, 65, 72, 100, 101, 161, 162, 180, 181, 217, 218, 231, 232, 277, 278, 313, 314, 352, 353, 415, 416, 456, 457, 498, 499, 576, 577, 629, 630, 655, 656, 694, 695, 750, 751, 775, 776, 798, 799, 823, 824, 843, 844, 892, 893, 951, 952, 1025, 1026, 1097, 1098, 1107, 1108, 1116, 1134, 1135, 1158, 1159, 1187, 1188, 1218, 1219, 1249, 1250, 1270, 1271, 1388, 1389, 1407, 1408, 1411 ] }
{ "red_pajama_v2": { "ccnet_original_length": 1411, "ccnet_original_nlines": 72, "rps_doc_curly_bracket": 0, "rps_doc_ldnoobw_words": 0, "rps_doc_lorem_ipsum": 0, "rps_doc_stop_word_fraction": 0.18412697315216064, "rps_doc_ut1_blacklist": 0, "rps_doc_frac_all_caps_words": 0.05079365149140358, "rps_doc_frac_lines_end_with_ellipsis": 0.013698630034923553, "rps_doc_frac_no_alph_words": 0.3523809611797333, "rps_doc_frac_unique_words": 0.6580311059951782, "rps_doc_mean_word_length": 5.269430160522461, "rps_doc_num_sentences": 22, "rps_doc_symbol_to_word_ratio": 0.0031745999585837126, "rps_doc_unigram_entropy": 4.646482944488525, "rps_doc_word_count": 193, "rps_doc_frac_chars_dupe_10grams": 0, "rps_doc_frac_chars_dupe_5grams": 0.07079645991325378, "rps_doc_frac_chars_dupe_6grams": 0.07079645991325378, "rps_doc_frac_chars_dupe_7grams": 0.07079645991325378, "rps_doc_frac_chars_dupe_8grams": 0.07079645991325378, "rps_doc_frac_chars_dupe_9grams": 0, "rps_doc_frac_chars_top_2gram": 0.032448381185531616, "rps_doc_frac_chars_top_3gram": 0.02949853055179119, "rps_doc_frac_chars_top_4gram": 0.03343166038393974, "rps_doc_books_importance": -100.85420989990234, "rps_doc_books_importance_length_correction": -100.49102783203125, "rps_doc_openwebtext_importance": -48.62539291381836, "rps_doc_openwebtext_importance_length_correction": -48.62539291381836, "rps_doc_wikipedia_importance": -20.946802139282227, "rps_doc_wikipedia_importance_length_correction": -16.276798248291016 }, "fasttext": { "dclm": 0.05290580168366432, "english": 0.7284966111183167, "fineweb_edu_approx": 2.345017671585083, "eai_general_math": 0.012764629907906055, "eai_open_web_math": 0.2549116015434265, "eai_web_code": 0.1461188793182373 } }
{ "free_decimal_correspondence": { "primary": { "code": "005.133", "labels": { "level_1": "General works, books and libraries, information sciences", "level_2": "", "level_3": "Computer programming" } }, "secondary": { "code": "004.02", "labels": { "level_1": "General works, books and libraries, information sciences", "level_2": "", "level_3": "Computers and Computer science" } } }, "bloom_cognitive_process": { "primary": { "code": "3", "label": "Apply" }, "secondary": { "code": "2", "label": "Understand" } }, "bloom_knowledge_domain": { "primary": { "code": "3", "label": "Procedural" }, "secondary": { "code": "2", "label": "Conceptual" } }, "document_type_v1": { "primary": { "code": "5", "label": "Social/Forum" }, "secondary": { "code": "3", "label": "Reference/Encyclopedic/Educational" } }, "extraction_artifacts": { "primary": { "code": "3", "label": "Irrelevant Content" }, "secondary": { "code": "0", "label": "No Artifacts" } }, "missing_content": { "primary": { "code": "0", "label": "No missing content" }, "secondary": { "code": "-1", "label": "Abstain" } }, "document_type_v2": { "primary": { "code": "18", "label": "Q&A Forum" }, "secondary": { "code": "21", "label": "Customer Support" } }, "reasoning_depth": { "primary": { "code": "2", "label": "Basic Reasoning" }, "secondary": { "code": "3", "label": "Intermediate Reasoning" } }, "technical_correctness": { "primary": { "code": "4", "label": "Highly Correct" }, "secondary": { "code": "3", "label": "Mostly Correct" } }, "education_level": { "primary": { "code": "3", "label": "Undergraduate Level" }, "secondary": { "code": "2", "label": "High School Level" } } }
672f1e42c33a7f9846924a2431ea77df
5,508,041,673,466,497,000
predict {stats}R Documentation Model Predictions Description predict is a generic function for predictions from the results of various model fitting functions. The function invokes particular methods which depend on the class of the first argument. Usage predict (object, ...) Arguments object a model object for which prediction is desired. ... additional arguments affecting the predictions produced. Details Most prediction methods which are similar to those for linear models have an argument newdata specifying the first place to look for explanatory variables to be used for prediction. Some considerable attempts are made to match up the columns in newdata to those used for fitting, for example that they are of comparable types and that any factors have the same level set in the same order (or can be transformed to be so). Time series prediction methods in package stats have an argument n.ahead specifying how many time steps ahead to predict. Many methods have a logical argument se.fit saying if standard errors are to returned. Value The form of the value returned by predict depends on the class of its argument. See the documentation of the particular methods for details of what is produced by that method. References Chambers, J. M. and Hastie, T. J. (1992) Statistical Models in S. Wadsworth & Brooks/Cole. See Also predict.glm, predict.lm, predict.loess, predict.nls, predict.poly, predict.princomp, predict.smooth.spline. SafePrediction for prediction from (univariable) polynomial and spline fits. For time-series prediction, predict.ar, predict.Arima, predict.arima0, predict.HoltWinters, predict.StructTS. Examples require(utils) ## All the "predict" methods found ## NB most of the methods in the standard packages are hidden. ## Output will depend on what namespaces are (or have been) loaded. ## IGNORE_RDIFF_BEGIN for(fn in methods("predict")) try({ f <- eval(substitute(getAnywhere(fn)$objs[[1]], list(fn = fn))) cat(fn, ":\n\t", deparse(args(f)), "\n") }, silent = TRUE) ## IGNORE_RDIFF_END [Package stats version 4.3.3 Index]
{ "url": "https://search.r-project.org/R/refmans/stats/html/predict.html", "source_domain": "search.r-project.org", "snapshot_id": "CC-MAIN-2024-10", "warc_metadata": { "Content-Length": "4905", "Content-Type": "application/http; msgtype=response", "WARC-Block-Digest": "sha1:G5L6O6OBANIS24BRPXHJ7HUCEWTDH6VH", "WARC-Concurrent-To": "<urn:uuid:71224cef-acaa-441f-ba41-1cf4c163f8cf>", "WARC-Date": "2024-02-28T06:15:06Z", "WARC-IP-Address": "137.208.57.46", "WARC-Identified-Payload-Type": "text/html", "WARC-Payload-Digest": "sha1:BR5KSBYIEVU5ICHAJ42J5ZDTKSTQNFFJ", "WARC-Record-ID": "<urn:uuid:8eb41cfa-ef4a-4238-90e1-5d22d5e9e13c>", "WARC-Target-URI": "https://search.r-project.org/R/refmans/stats/html/predict.html", "WARC-Truncated": null, "WARC-Type": "response", "WARC-Warcinfo-ID": "<urn:uuid:9077bc91-ef04-4c21-92b4-f56ec5c1c6ce>" }, "warc_info": "isPartOf: CC-MAIN-2024-10\r\npublisher: Common Crawl\r\ndescription: Wide crawl of the web for February/March 2024\r\noperator: Common Crawl Admin ([email protected])\r\nhostname: ip-10-67-67-154\r\nsoftware: Apache Nutch 1.19 (modified, https://github.com/commoncrawl/nutch/)\r\nrobots: checked via crawler-commons 1.5-SNAPSHOT (https://github.com/crawler-commons/crawler-commons)\r\nformat: WARC File Format 1.1\r\nconformsTo: https://iipc.github.io/warc-specifications/specifications/warc-format/warc-1.1/" }
{ "line_start_idx": [ 0, 31, 32, 50, 51, 63, 64, 252, 253, 259, 260, 282, 283, 293, 294, 301, 302, 350, 351, 355, 356, 413, 414, 422, 423, 846, 847, 969, 970, 1057, 1058, 1064, 1065, 1241, 1242, 1253, 1254, 1345, 1346, 1355, 1356, 1464, 1465, 1542, 1543, 1653, 1654, 1663, 1664, 1665, 1680, 1681, 1716, 1779, 1847, 1869, 1899, 1908, 1979, 2027, 2052, 2072, 2073, 2074 ], "line_end_idx": [ 31, 32, 50, 51, 63, 64, 252, 253, 259, 260, 282, 283, 293, 294, 301, 302, 350, 351, 355, 356, 413, 414, 422, 423, 846, 847, 969, 970, 1057, 1058, 1064, 1065, 1241, 1242, 1253, 1254, 1345, 1346, 1355, 1356, 1464, 1465, 1542, 1543, 1653, 1654, 1663, 1664, 1665, 1680, 1681, 1716, 1779, 1847, 1869, 1899, 1908, 1979, 2027, 2052, 2072, 2073, 2074, 2109 ] }
{ "red_pajama_v2": { "ccnet_original_length": 2109, "ccnet_original_nlines": 63, "rps_doc_curly_bracket": 0.0018966300413012505, "rps_doc_ldnoobw_words": 0, "rps_doc_lorem_ipsum": 0, "rps_doc_stop_word_fraction": 0.29481130838394165, "rps_doc_ut1_blacklist": 0, "rps_doc_frac_all_caps_words": 0.023584909737110138, "rps_doc_frac_lines_end_with_ellipsis": 0.015625, "rps_doc_frac_no_alph_words": 0.26179245114326477, "rps_doc_frac_unique_words": 0.5649122595787048, "rps_doc_mean_word_length": 5.687719345092773, "rps_doc_num_sentences": 40, "rps_doc_symbol_to_word_ratio": 0.028301889076828957, "rps_doc_unigram_entropy": 4.764373302459717, "rps_doc_word_count": 285, "rps_doc_frac_chars_dupe_10grams": 0, "rps_doc_frac_chars_dupe_5grams": 0, "rps_doc_frac_chars_dupe_6grams": 0, "rps_doc_frac_chars_dupe_7grams": 0, "rps_doc_frac_chars_dupe_8grams": 0, "rps_doc_frac_chars_dupe_9grams": 0, "rps_doc_frac_chars_top_2gram": 0.012338059954345226, "rps_doc_frac_chars_top_3gram": 0.012338059954345226, "rps_doc_frac_chars_top_4gram": 0.014805680140852928, "rps_doc_books_importance": -170.73536682128906, "rps_doc_books_importance_length_correction": -170.73536682128906, "rps_doc_openwebtext_importance": -102.26155853271484, "rps_doc_openwebtext_importance_length_correction": -102.26155853271484, "rps_doc_wikipedia_importance": -58.15476989746094, "rps_doc_wikipedia_importance_length_correction": -58.15476989746094 }, "fasttext": { "dclm": 0.02753991074860096, "english": 0.7935491800308228, "fineweb_edu_approx": 2.4126760959625244, "eai_general_math": 0.995101809501648, "eai_open_web_math": 0.4296053647994995, "eai_web_code": 0.9993476271629333 } }
{ "free_decimal_correspondence": { "primary": { "code": "005.133", "labels": { "level_1": "General works, books and libraries, information sciences", "level_2": "", "level_3": "Computer programming" } }, "secondary": { "code": "519.5", "labels": { "level_1": "Science and Natural history", "level_2": "Mathematics", "level_3": "Probabilities; or, Mathematical statistics" } } }, "bloom_cognitive_process": { "primary": { "code": "2", "label": "Understand" }, "secondary": { "code": "3", "label": "Apply" } }, "bloom_knowledge_domain": { "primary": { "code": "2", "label": "Conceptual" }, "secondary": { "code": "3", "label": "Procedural" } }, "document_type_v1": { "primary": { "code": "3", "label": "Reference/Encyclopedic/Educational" }, "secondary": { "code": "4", "label": "Code/Software" } }, "extraction_artifacts": { "primary": { "code": "0", "label": "No Artifacts" }, "secondary": { "code": "3", "label": "Irrelevant Content" } }, "missing_content": { "primary": { "code": "0", "label": "No missing content" }, "secondary": { "code": "-1", "label": "Abstain" } }, "document_type_v2": { "primary": { "code": "8", "label": "Documentation" }, "secondary": { "code": "10", "label": "Knowledge Article" } }, "reasoning_depth": { "primary": { "code": "2", "label": "Basic Reasoning" }, "secondary": { "code": "3", "label": "Intermediate Reasoning" } }, "technical_correctness": { "primary": { "code": "4", "label": "Highly Correct" }, "secondary": { "code": "5", "label": "Exceptionally Correct" } }, "education_level": { "primary": { "code": "3", "label": "Undergraduate Level" }, "secondary": { "code": "4", "label": "Graduate/Expert Level" } } }
672f1e42c33a7f9846924a2431ea77df
2,324,780,714,426,825,700
IQueryProvider 메서드 IQueryProvider 유형에서 다음 멤버를 표시합니다.   이름 설명 공용 메서드 Build(IList<INavigableItem>, INodeInformation, IFilterProvider) Builds the tree nodes with a filter that specifies nodes, source, and filter. 공용 메서드 Build(IList<INavigableItem>, INodeInformation, INavigableItem, IFilterProvider) Builds the tree nodes with a filter that specifies nodes, source, parent, and filter. 공용 메서드 GetColumns Retrieves a list of columns. 공용 메서드 GetFilterableColumns Retrieves a list of columns that can be filtered. 공용 메서드 GetQuery Retrieves the query that would be used to build the tree nodes. 공용 메서드 RebuildItem Requeries the properties of an individual item. 공용 메서드 UpdateItems Updates existing items and loads the requested fields for each item. 맨 위로 이동 커뮤니티 추가 항목 추가 표시:
{ "url": "https://technet.microsoft.com/ko-kr/library/microsoft.sqlserver.management.ui.vsintegration.objectexplorer.iqueryprovider_methods(v=sql.110).aspx", "source_domain": "technet.microsoft.com", "snapshot_id": "crawl=CC-MAIN-2017-51", "warc_metadata": { "Content-Length": "58685", "Content-Type": "application/http; msgtype=response", "WARC-Block-Digest": "sha1:MDZIJU6MSMT6AWAPHT73ARAXACZQK5I3", "WARC-Concurrent-To": "<urn:uuid:68266774-1bb9-4673-b777-7d402ceb246b>", "WARC-Date": "2017-12-16T14:07:58Z", "WARC-IP-Address": "157.56.148.23", "WARC-Identified-Payload-Type": "application/xhtml+xml", "WARC-Payload-Digest": "sha1:N7YFNKCOKQ62J6AA4UOIUMNDITS4NJ2W", "WARC-Record-ID": "<urn:uuid:41761266-20bf-4547-bc1a-5dab6d5dee28>", "WARC-Target-URI": "https://technet.microsoft.com/ko-kr/library/microsoft.sqlserver.management.ui.vsintegration.objectexplorer.iqueryprovider_methods(v=sql.110).aspx", "WARC-Truncated": null, "WARC-Type": "response", "WARC-Warcinfo-ID": "<urn:uuid:ec20ed58-dcce-4aaf-a147-8115cff27e0c>" }, "warc_info": "robots: classic\r\nhostname: ip-10-159-16-27.ec2.internal\r\nsoftware: Nutch 1.6 (CC)\r\nisPartOf: CC-MAIN-2017-51\r\noperator: Common Crawl Admin\r\ndescription: Wide crawl of the web for December 2017\r\npublisher: Common Crawl\r\nformat: WARC File Format 1.0\r\nconformsTo: http://bibnum.bnf.fr/WARC/WARC_ISO_28500_version1_latestdraft.pdf" }
{ "line_start_idx": [ 0, 19, 20, 54, 55, 63, 212, 385, 432, 510, 590, 657, 745, 753, 754, 765, 766, 769 ], "line_end_idx": [ 19, 20, 54, 55, 63, 212, 385, 432, 510, 590, 657, 745, 753, 754, 765, 766, 769, 772 ] }
{ "red_pajama_v2": { "ccnet_original_length": 772, "ccnet_original_nlines": 17, "rps_doc_curly_bracket": 0, "rps_doc_ldnoobw_words": 0, "rps_doc_lorem_ipsum": 0, "rps_doc_stop_word_fraction": 0.21678322553634644, "rps_doc_ut1_blacklist": 0, "rps_doc_frac_all_caps_words": 0, "rps_doc_frac_lines_end_with_ellipsis": 0, "rps_doc_frac_no_alph_words": 0.3776223659515381, "rps_doc_frac_unique_words": 0.5438596606254578, "rps_doc_mean_word_length": 6.359649181365967, "rps_doc_num_sentences": 9, "rps_doc_symbol_to_word_ratio": 0, "rps_doc_unigram_entropy": 3.8862392902374268, "rps_doc_word_count": 114, "rps_doc_frac_chars_dupe_10grams": 0.1875862032175064, "rps_doc_frac_chars_dupe_5grams": 0.2510344684123993, "rps_doc_frac_chars_dupe_6grams": 0.1875862032175064, "rps_doc_frac_chars_dupe_7grams": 0.1875862032175064, "rps_doc_frac_chars_dupe_8grams": 0.1875862032175064, "rps_doc_frac_chars_dupe_9grams": 0.1875862032175064, "rps_doc_frac_chars_top_2gram": 0.11586207151412964, "rps_doc_frac_chars_top_3gram": 0.04965516924858093, "rps_doc_frac_chars_top_4gram": 0.14344827830791473, "rps_doc_books_importance": -58.119258880615234, "rps_doc_books_importance_length_correction": -58.119258880615234, "rps_doc_openwebtext_importance": -38.60159683227539, "rps_doc_openwebtext_importance_length_correction": -38.601593017578125, "rps_doc_wikipedia_importance": -39.219337463378906, "rps_doc_wikipedia_importance_length_correction": -39.219337463378906 }, "fasttext": { "dclm": 0.9267113208770752, "english": 0.22671949863433838, "fineweb_edu_approx": 2.256488561630249, "eai_general_math": 0.28314435482025146, "eai_open_web_math": 0.23854303359985352, "eai_web_code": 0.7763820886611938 } }
{ "free_decimal_correspondence": { "primary": { "code": "005.1", "labels": { "level_1": "General works, books and libraries, information sciences", "level_2": "", "level_3": "Computer programming" } }, "secondary": { "code": "005.776", "labels": { "level_1": "General works, books and libraries, information sciences", "level_2": "", "level_3": "Computer programming" } } }, "bloom_cognitive_process": { "primary": { "code": "2", "label": "Understand" }, "secondary": { "code": "3", "label": "Apply" } }, "bloom_knowledge_domain": { "primary": { "code": "2", "label": "Conceptual" }, "secondary": { "code": "3", "label": "Procedural" } }, "document_type_v1": { "primary": { "code": "3", "label": "Reference/Encyclopedic/Educational" }, "secondary": { "code": "4", "label": "Code/Software" } }, "extraction_artifacts": { "primary": { "code": "0", "label": "No Artifacts" }, "secondary": { "code": "3", "label": "Irrelevant Content" } }, "missing_content": { "primary": { "code": "0", "label": "No missing content" }, "secondary": { "code": "-1", "label": "Abstain" } }, "document_type_v2": { "primary": { "code": "8", "label": "Documentation" }, "secondary": { "code": "10", "label": "Knowledge Article" } }, "reasoning_depth": { "primary": { "code": "2", "label": "Basic Reasoning" }, "secondary": { "code": "1", "label": "No Reasoning" } }, "technical_correctness": { "primary": { "code": "4", "label": "Highly Correct" }, "secondary": { "code": "3", "label": "Mostly Correct" } }, "education_level": { "primary": { "code": "3", "label": "Undergraduate Level" }, "secondary": { "code": "4", "label": "Graduate/Expert Level" } } }
672f1e42c33a7f9846924a2431ea77df
8,415,280,610,819,425,000
Skip to content Instantly share code, notes, and snippets. @VithuJey Last active Oct 22, 2021 Embed What would you like to do? Just a gist to explain Redux Toolkit import { createSlice, PayloadAction } from '@reduxjs/toolkit' export interface OrderState { item: string; qty: number; total: number; } const initialState: OrderState = { item: "", qty: 0, total: 0, } export const orderSlice = createSlice({ name: 'order', initialState, reducers: { addItem: (state, action: PayloadAction<string>) => { state.item = action.payload }, remItem: (state) => { state.item = "" }, incrementQty: (state) => { state.qty += 1 }, decrementQty: (state) => { state.qty -= 1 }, incrementTotal: (state, action: PayloadAction<number>) => { state.total += action.payload }, decrementTotal: (state, action: PayloadAction<number>) => { state.total -= action.payload }, }, }) export const { addItem, remItem, incrementQty, decrementQty, incrementTotal } = orderSlice.actions export default orderSlice.reducer Sign up for free to join this conversation on GitHub. Already have an account? Sign in to comment
{ "url": "https://gist.github.com/VithuJey/fe5d5811375ff354b9cc18c20ec0e030", "source_domain": "gist.github.com", "snapshot_id": "crawl=CC-MAIN-2022-05", "warc_metadata": { "Content-Length": "104425", "Content-Type": "application/http; msgtype=response", "WARC-Block-Digest": "sha1:2WUYEF4LARQ4GKYD6IUSWUVFGI3ODENZ", "WARC-Concurrent-To": "<urn:uuid:76d36f87-cc3f-4ac1-acd2-f411e05cb3de>", "WARC-Date": "2022-01-26T05:34:48Z", "WARC-IP-Address": "140.82.112.4", "WARC-Identified-Payload-Type": "text/html", "WARC-Payload-Digest": "sha1:63HIEG3F5CKSNWWD5R3C6XW6CWFLGRZP", "WARC-Record-ID": "<urn:uuid:be17fa1f-6bd4-4fa4-a9e7-fcadff8a6325>", "WARC-Target-URI": "https://gist.github.com/VithuJey/fe5d5811375ff354b9cc18c20ec0e030", "WARC-Truncated": null, "WARC-Type": "response", "WARC-Warcinfo-ID": "<urn:uuid:cc33e4ef-128e-405c-ab12-55ad095dd242>" }, "warc_info": "isPartOf: CC-MAIN-2022-05\r\npublisher: Common Crawl\r\ndescription: Wide crawl of the web for January 2022\r\noperator: Common Crawl Admin ([email protected])\r\nhostname: ip-10-67-67-141\r\nsoftware: Apache Nutch 1.18 (modified, https://github.com/commoncrawl/nutch/)\r\nrobots: checked via crawler-commons 1.3-SNAPSHOT (https://github.com/crawler-commons/crawler-commons)\r\nformat: WARC File Format 1.1\r\nconformsTo: https://iipc.github.io/warc-specifications/specifications/warc-format/warc-1.1/" }
{ "line_start_idx": [ 0, 16, 17, 60, 61, 71, 96, 102, 129, 166, 228, 258, 272, 285, 300, 302, 337, 347, 355, 365, 367, 407, 422, 436, 448, 501, 529, 532, 554, 570, 573, 600, 615, 618, 645, 660, 663, 723, 753, 756, 816, 846, 849, 852, 855, 954, 988 ], "line_end_idx": [ 16, 17, 60, 61, 71, 96, 102, 129, 166, 228, 258, 272, 285, 300, 302, 337, 347, 355, 365, 367, 407, 422, 436, 448, 501, 529, 532, 554, 570, 573, 600, 615, 618, 645, 660, 663, 723, 753, 756, 816, 846, 849, 852, 855, 954, 988, 1085 ] }
{ "red_pajama_v2": { "ccnet_original_length": 1085, "ccnet_original_nlines": 46, "rps_doc_curly_bracket": 0.02211982011795044, "rps_doc_ldnoobw_words": 0, "rps_doc_lorem_ipsum": 0, "rps_doc_stop_word_fraction": 0.08196721225976944, "rps_doc_ut1_blacklist": 0, "rps_doc_frac_all_caps_words": 0, "rps_doc_frac_lines_end_with_ellipsis": 0, "rps_doc_frac_no_alph_words": 0.4836065471172333, "rps_doc_frac_unique_words": 0.6837607026100159, "rps_doc_mean_word_length": 6.735042572021484, "rps_doc_num_sentences": 16, "rps_doc_symbol_to_word_ratio": 0, "rps_doc_unigram_entropy": 4.244481563568115, "rps_doc_word_count": 117, "rps_doc_frac_chars_dupe_10grams": 0, "rps_doc_frac_chars_dupe_5grams": 0.13451777398586273, "rps_doc_frac_chars_dupe_6grams": 0, "rps_doc_frac_chars_dupe_7grams": 0, "rps_doc_frac_chars_dupe_8grams": 0, "rps_doc_frac_chars_dupe_9grams": 0, "rps_doc_frac_chars_top_2gram": 0.041878171265125275, "rps_doc_frac_chars_top_3gram": 0.03553298860788345, "rps_doc_frac_chars_top_4gram": 0.10152284055948257, "rps_doc_books_importance": -91.37860107421875, "rps_doc_books_importance_length_correction": -91.37860107421875, "rps_doc_openwebtext_importance": -65.47919464111328, "rps_doc_openwebtext_importance_length_correction": -55.26974868774414, "rps_doc_wikipedia_importance": -37.38613510131836, "rps_doc_wikipedia_importance_length_correction": -37.38613510131836 }, "fasttext": { "dclm": 0.9213962554931641, "english": 0.7150333523750305, "fineweb_edu_approx": 1.3008410930633545, "eai_general_math": 0.003995299804955721, "eai_open_web_math": 0.0018762400140985847, "eai_web_code": 0.00008487999730277807 } }
{ "free_decimal_correspondence": { "primary": { "code": "005.1", "labels": { "level_1": "General works, books and libraries, information sciences", "level_2": "", "level_3": "Computer programming" } }, "secondary": { "code": "005.452", "labels": { "level_1": "General works, books and libraries, information sciences", "level_2": "", "level_3": "Computer programming" } } }, "bloom_cognitive_process": { "primary": { "code": "3", "label": "Apply" }, "secondary": { "code": "2", "label": "Understand" } }, "bloom_knowledge_domain": { "primary": { "code": "3", "label": "Procedural" }, "secondary": { "code": "2", "label": "Conceptual" } }, "document_type_v1": { "primary": { "code": "4", "label": "Code/Software" }, "secondary": { "code": "3", "label": "Reference/Encyclopedic/Educational" } }, "extraction_artifacts": { "primary": { "code": "3", "label": "Irrelevant Content" }, "secondary": { "code": "1", "label": "Leftover HTML" } }, "missing_content": { "primary": { "code": "0", "label": "No missing content" }, "secondary": { "code": "-1", "label": "Abstain" } }, "document_type_v2": { "primary": { "code": "8", "label": "Documentation" }, "secondary": { "code": "23", "label": "Tutorial" } }, "reasoning_depth": { "primary": { "code": "2", "label": "Basic Reasoning" }, "secondary": { "code": "3", "label": "Intermediate Reasoning" } }, "technical_correctness": { "primary": { "code": "4", "label": "Highly Correct" }, "secondary": { "code": "3", "label": "Mostly Correct" } }, "education_level": { "primary": { "code": "3", "label": "Undergraduate Level" }, "secondary": { "code": "4", "label": "Graduate/Expert Level" } } }
672f1e42c33a7f9846924a2431ea77df
-8,174,651,799,671,516,000
IETF URI Working Group Internet-Draft draft-ietf-uri-url-mailserver-01.txt Expires August 8, 1995 Mailserver URL Specification Status of This Memo This document is an Internet-Draft. Internet-Drafts are working documents of the Internet Engineering Task Force (IETF), its areas, and its working groups. Note that other groups may also distribute working documents as Internet-Drafts. Internet-Drafts are draft documents valid for a maximum of six months and may be updated, replaced, or obsoleted by other documents at any time. It is inappropriate to use Internet- Drafts as reference material or to cite them other than as ``work in progress.'' To learn the current status of any Internet-Draft, please check the ``1id-abstracts.txt'' listing contained in the Internet- Drafts Shadow Directories on ftp.is.co.za (Africa), nic.nordu.net (Europe), munnari.oz.au (Pacific Rim), ds.internic.net (US East Coast), or ftp.isi.edu (US West Coast). Abstract A new URL scheme, "mailserver", is defined. It allows mail client software to create RFC822 mail messages from a URL. Description In the URL specification, RFC1738, the "mailto" scheme is defined and is described as: Unlike many URLs, the mailto scheme does not represent a data object to be accessed directly; there is no sense in which it designates an object. However, there are many resources on the Internet that can only be accessed by mail that cannot be described by the mailto scheme. To access such an object, the mail message must have a specified subject and/or content. For instance, many mail response servers will return a file if you send a mail message with the proper request. The "mailserver" URL has the form: mailserver:// Client software would prepare a mail message with the given text as the subject header field and the text as the body of the message. and may have zero length. Thus, the "mailto" scheme will be used to give the mailing address of a person or of a mailserver that requires no subject or message body; the "mailserver" scheme is used to give a template that will cause the specified resource to be returned. The body text may span more than one line. Any "/" character in the body should be interpreted by the mail client as a CRLF sequence when translating a URL to a mail message. Examples A URL for a mail response system that requires the name of the file in the subject might be: A mail response system that requires a "send" request in the body might have a URL that looks like: A similar URL could have two lines with different "send" requests: The "mailserver" scheme would also help people get another type of Internet resource, namely mailing lists. For example: Encoding RFC1738 requires that many characters in URLs be encoded. This affects the mailserver scheme for some common characters that might appear in subjects or message contents. Two such characters are space (" ", ASCII hex 20) and forward slash ("/", ASCII hex 2F). Note the examples above that use "%20" for space in the message body. Note further that an unencoded forward slash in the body area is to be translated by the mail client to CRLF. People creating mailserver URLs must be careful to encode any reserved characters that are used in the URLs so that properly-written URL interpreters can read them. Also, client software that reads URLs must be careful to decode strings before creating the mail message so that the mail messages appear in a form that the recipient will understand. These strings should be decoded before showing the user the mesage. For security reasons, the characters 0A hexadecimal (US-ASCII character LF), and 0D (US-ASCII character CR) must not be decoded by client software. To indicate new lines in the body text, a URL should use the forward slash ("/") character, which client software will translate to CRLF. Additional BNF for RFC1738 mailserverurl = "mailserver:" encoded822addr "/" subject "/" body subject = *[uchar] body = [body_line] *["/" body_line] body_line = *[uchar] Security The mailserver scheme is intended to send a message from one user to another, and thus can introduce many security concerns. Mail messages can be logged at the originating site, the recipient site, and intermediary sites along the delivery path. If the messages are not encoded, they can also be read at any of those sites. A mailserver URL gives a template for a message that can be sent by mail client software. The contents of that template may be opaque or difficult to read by the user at the time of specifying the URL. Thus, a mail client should never send a message based on a mailserver URL without first showing the user the full message that will be sent (including all headers, including the subject specified in the URL), fully decoded, and asking the user for approval to send the message. Client software must not decode the characters 0A hexadecimal (US-ASCII character LF), and 0D (US-ASCII character CR). In the subject field, such decoding would permit header spoofing; there is no need for these characters in the body field because of the use of the "/" character. Examples of problems with sending unapproved mail include: - mail that breaks laws upon delivery, such as making illegal threats - mail that identifies the sender as someone interested in breaking laws - mail that identifies the sender to an unwanted third party - mail that causes a financial charge to be incurred on the sender - mail that causes an action on the recipient machine that causes damage that might be attributed to the sender Author contact information: Paul E. Hoffman Proper Publishing 127 Segre Place Santa Cruz, CA 95060 USA Tel: 408-426-6222 [email protected]
{ "url": "https://www.ietf.org/archive/id/draft-ietf-uri-url-mailserver-01.txt", "source_domain": "www.ietf.org", "snapshot_id": "CC-MAIN-2024-22", "warc_metadata": { "Content-Length": "7057", "Content-Type": "application/http; msgtype=response", "WARC-Block-Digest": "sha1:YBYHB6ZEG7VYPQR3Z4YCBPRZZRHWJTVQ", "WARC-Concurrent-To": "<urn:uuid:e8b6bbdb-f433-4f48-828b-ee9ce8c52f61>", "WARC-Date": "2024-05-25T12:08:17Z", "WARC-IP-Address": "104.16.45.99", "WARC-Identified-Payload-Type": "text/plain", "WARC-Payload-Digest": "sha1:OELX7U5TSI24QACG6247H3XTAC5C3JD6", "WARC-Record-ID": "<urn:uuid:e6caa931-2f85-4c5f-b184-5e742703aec3>", "WARC-Target-URI": "https://www.ietf.org/archive/id/draft-ietf-uri-url-mailserver-01.txt", "WARC-Truncated": null, "WARC-Type": "response", "WARC-Warcinfo-ID": "<urn:uuid:f78d6445-a68f-4f90-9fff-9b1d5c8d6919>" }, "warc_info": "isPartOf: CC-MAIN-2024-22\r\npublisher: Common Crawl\r\ndescription: Wide crawl of the web for May 2024\r\noperator: Common Crawl Admin ([email protected])\r\nhostname: ip-10-67-67-29\r\nsoftware: Apache Nutch 1.19 (modified, https://github.com/commoncrawl/nutch/)\r\nrobots: checked via crawler-commons 1.5-SNAPSHOT (https://github.com/crawler-commons/crawler-commons)\r\nformat: WARC File Format 1.1\r\nconformsTo: https://iipc.github.io/warc-specifications/specifications/warc-format/warc-1.1/" }
{ "line_start_idx": [ 0 ], "line_end_idx": [ 5664 ] }
{ "red_pajama_v2": { "ccnet_original_length": 5664, "ccnet_original_nlines": 0, "rps_doc_curly_bracket": 0, "rps_doc_ldnoobw_words": 0, "rps_doc_lorem_ipsum": 0, "rps_doc_stop_word_fraction": 0.3726872205734253, "rps_doc_ut1_blacklist": 0, "rps_doc_frac_all_caps_words": 0.04845815151929855, "rps_doc_frac_lines_end_with_ellipsis": 0, "rps_doc_frac_no_alph_words": 0.170925110578537, "rps_doc_frac_unique_words": 0.3657205104827881, "rps_doc_mean_word_length": 4.911571979522705, "rps_doc_num_sentences": 52, "rps_doc_symbol_to_word_ratio": 0, "rps_doc_unigram_entropy": 5.1475749015808105, "rps_doc_word_count": 916, "rps_doc_frac_chars_dupe_10grams": 0.029784400016069412, "rps_doc_frac_chars_dupe_5grams": 0.07246054708957672, "rps_doc_frac_chars_dupe_6grams": 0.04356523975729942, "rps_doc_frac_chars_dupe_7grams": 0.029784400016069412, "rps_doc_frac_chars_dupe_8grams": 0.029784400016069412, "rps_doc_frac_chars_dupe_9grams": 0.029784400016069412, "rps_doc_frac_chars_top_2gram": 0.013336299918591976, "rps_doc_frac_chars_top_3gram": 0.010002220049500465, "rps_doc_frac_chars_top_4gram": 0.007112689781934023, "rps_doc_books_importance": -606.757568359375, "rps_doc_books_importance_length_correction": -606.757568359375, "rps_doc_openwebtext_importance": -344.4145202636719, "rps_doc_openwebtext_importance_length_correction": -344.4145202636719, "rps_doc_wikipedia_importance": -254.3912811279297, "rps_doc_wikipedia_importance_length_correction": -254.3912811279297 }, "fasttext": { "dclm": 0.06333804130554199, "english": 0.8755079507827759, "fineweb_edu_approx": 2.338723659515381, "eai_general_math": 0.45748740434646606, "eai_open_web_math": 0.2486221194267273, "eai_web_code": 0.8399232029914856 } }
{ "free_decimal_correspondence": { "primary": { "code": "004.67", "labels": { "level_1": "General works, books and libraries, information sciences", "level_2": "", "level_3": "Computers and Computer science" } }, "secondary": { "code": "005.72", "labels": { "level_1": "General works, books and libraries, information sciences", "level_2": "", "level_3": "Computer programming" } } }, "bloom_cognitive_process": { "primary": { "code": "2", "label": "Understand" }, "secondary": { "code": "3", "label": "Apply" } }, "bloom_knowledge_domain": { "primary": { "code": "2", "label": "Conceptual" }, "secondary": { "code": "3", "label": "Procedural" } }, "document_type_v1": { "primary": { "code": "2", "label": "Academic/Research" }, "secondary": { "code": "3", "label": "Reference/Encyclopedic/Educational" } }, "extraction_artifacts": { "primary": { "code": "0", "label": "No Artifacts" }, "secondary": { "code": "3", "label": "Irrelevant Content" } }, "missing_content": { "primary": { "code": "0", "label": "No missing content" }, "secondary": { "code": "-1", "label": "Abstain" } }, "document_type_v2": { "primary": { "code": "8", "label": "Documentation" }, "secondary": { "code": "10", "label": "Knowledge Article" } }, "reasoning_depth": { "primary": { "code": "3", "label": "Intermediate Reasoning" }, "secondary": { "code": "2", "label": "Basic Reasoning" } }, "technical_correctness": { "primary": { "code": "4", "label": "Highly Correct" }, "secondary": { "code": "3", "label": "Mostly Correct" } }, "education_level": { "primary": { "code": "3", "label": "Undergraduate Level" }, "secondary": { "code": "4", "label": "Graduate/Expert Level" } } }
672f1e42c33a7f9846924a2431ea77df
9,194,583,906,539,862,000
Sign in to follow this   niner Win32 API: Difference between handle to an Instance and handle to a Window. Recommended Posts hi, first of all, this will help: hInstance -> handle to an instance hWindow -> handle to a window the title of the thread is pretty explanatory :) I'm a little bit confused about these concepts, the way I'm thinking is that the hInstance is the loaded program in memory, and the hWindow is the (what would be) visual part of the hInstance, so when we do the CreateWindow function we are "connecting", "linking" the hInstance to the hWindow. :p i dont really know if that's what it is, this is what makes sense to me. Any help would be appreciated Thanks :) Share this post Link to post Share on other sites Quote: Original post by niner hi, first of all, this will help: hInstance -> handle to an instance hWindow -> handle to a window the title of the thread is pretty explanatory :) I'm a little bit confused about these concepts, the way I'm thinking is that the hInstance is the loaded program in memory, and the hWindow is the (what would be) visual part of the hInstance, so when we do the CreateWindow function we are "connecting", "linking" the hInstance to the hWindow. :p i dont really know if that's what it is, this is what makes sense to me. Any help would be appreciated Thanks :) An application loaded into memory has a HINSTANCE. Every EXE or DLL has an HINSTANCE. A window created by an application has a HWND. An EXE or DLL may create zero windows, in which case it won't have a HWND, it may create one window, or it may create hundreds. There's one HWND for each window, and for each control on a window (A control is a window itself). The reason you need a HINSTANCE when you create a window (or rather when you register the window class) is that Windows needs to know what EXE or DLL your window procedure is in, so it can notify your program when something happens to the window. This may be relevant. Share this post Link to post Share on other sites Quote: Original post by niner I'm a little bit confused about these concepts, the way I'm thinking is that the hInstance is the loaded program in memory, and the hWindow is the (what would be) visual part of the hInstance, so when we do the CreateWindow function we are "connecting", "linking" the hInstance to the hWindow. There isn't a "visual part of the hInstance" until you create one. If you never called CreateWindow... well, give it a try. I guarantee you that a boring time will be had by all. Share this post Link to post Share on other sites the simplest explanation is that an HINSTANCE is just a number, an int that represents your program, so whenever you refer to your program, you use the HINSTANCE the same with an HWND, it is just a number that represents your window, so when you have to do something that involves a window, you use the HWND Share this post Link to post Share on other sites This sounds suspisiously like that old newbie standy "how do I find the main window of a process". The reply of course being that there is no such thing as a "main" window. This discussion may be relevant. To expand a bit on what you actually asked... An HINSTANCE is just a reference to an exe or dll. You need it because the exe or dll can say things like "load my menu, here's the id of the one I want". The OS needs to know who the heck you are before it can figure out where to start looking for "your" menu id. The HINSTANCE gives it that information. Note in Win32 HINSTANCE values are local to the process - a.exe and b.exe might have the same HINSTANCE value, and foo.dll might have different HINSTANCE values in each process even though it's loaded in both. This was *not* true in Win16 where HINSTANCE's were system-wide. This leaks through in a few places, the most notable being the "hPrevInstance" parameter to WinMain which is completely useless today. Similiarly HWND's are references to windows. There is no direct connection between HWND's and HINSTANCE's. The only reason you need to pass an HINSTANCE when you call CreateWindow is so CreateWindow can do things like find your menu resource - again, it needs to know who "you" are. Unlike HINSTANCE, HWND's are system-wide (well, session-wide anyway). Share this post Link to post Share on other sites Create an account or sign in to comment You need to be a member in order to leave a comment Create an account Sign up for a new account in our community. It's easy! Register a new account Sign in Already have an account? Sign in here. Sign In Now Sign in to follow this  
{ "url": "https://www.gamedev.net/forums/topic/484785-win32-api-difference-between-handle-to-an-instance-and-handle-to-a-window/", "source_domain": "www.gamedev.net", "snapshot_id": "crawl=CC-MAIN-2017-43", "warc_metadata": { "Content-Length": "143641", "Content-Type": "application/http; msgtype=response", "WARC-Block-Digest": "sha1:YRRS573BTR53MHWOPNQW4WNSBPD76ML3", "WARC-Concurrent-To": "<urn:uuid:145a7353-aefd-44be-abfc-452c3b403c5d>", "WARC-Date": "2017-10-20T16:22:36Z", "WARC-IP-Address": "130.211.24.203", "WARC-Identified-Payload-Type": "text/html", "WARC-Payload-Digest": "sha1:LYSPM45JVIRJORKJYNPXAXNUOP6CZTTQ", "WARC-Record-ID": "<urn:uuid:31cb7b2f-d427-470d-9c08-16af87fd5b7c>", "WARC-Target-URI": "https://www.gamedev.net/forums/topic/484785-win32-api-difference-between-handle-to-an-instance-and-handle-to-a-window/", "WARC-Truncated": null, "WARC-Type": "response", "WARC-Warcinfo-ID": "<urn:uuid:95a74c58-bc39-482a-8da1-d55bec31ed92>" }, "warc_info": "robots: classic\r\nhostname: ip-10-145-0-250.ec2.internal\r\nsoftware: Nutch 1.6 (CC)\r\nisPartOf: CC-MAIN-2017-43\r\noperator: Common Crawl Admin\r\ndescription: Wide crawl of the web for October 2017\r\npublisher: Common Crawl\r\nformat: WARC File Format 1.0\r\nconformsTo: http://bibnum.bnf.fr/WARC/WARC_ISO_28500_version1_latestdraft.pdf" }
{ "line_start_idx": [ 0, 25, 31, 32, 108, 109, 127, 128, 686, 687, 703, 704, 705, 718, 739, 746, 769, 773, 774, 839, 869, 870, 919, 920, 1214, 1215, 1291, 1292, 1322, 1323, 1333, 1419, 1693, 1694, 1941, 1942, 1964, 1965, 1981, 1982, 1983, 1996, 2017, 2024, 2047, 2341, 2342, 2521, 2522, 2538, 2539, 2540, 2553, 2574, 2736, 2737, 2883, 2884, 2900, 2901, 2902, 2915, 2936, 3142, 3143, 3495, 3496, 3906, 3907, 4260, 4261, 4277, 4278, 4279, 4292, 4313, 4314, 4354, 4355, 4407, 4408, 4426, 4427, 4482, 4483, 4506, 4507, 4515, 4516, 4555, 4556, 4568, 4569 ], "line_end_idx": [ 25, 31, 32, 108, 109, 127, 128, 686, 687, 703, 704, 705, 718, 739, 746, 769, 773, 774, 839, 869, 870, 919, 920, 1214, 1215, 1291, 1292, 1322, 1323, 1333, 1419, 1693, 1694, 1941, 1942, 1964, 1965, 1981, 1982, 1983, 1996, 2017, 2024, 2047, 2341, 2342, 2521, 2522, 2538, 2539, 2540, 2553, 2574, 2736, 2737, 2883, 2884, 2900, 2901, 2902, 2915, 2936, 3142, 3143, 3495, 3496, 3906, 3907, 4260, 4261, 4277, 4278, 4279, 4292, 4313, 4314, 4354, 4355, 4407, 4408, 4426, 4427, 4482, 4483, 4506, 4507, 4515, 4516, 4555, 4556, 4568, 4569, 4593 ] }
{ "red_pajama_v2": { "ccnet_original_length": 4593, "ccnet_original_nlines": 92, "rps_doc_curly_bracket": 0, "rps_doc_ldnoobw_words": 0, "rps_doc_lorem_ipsum": 0, "rps_doc_stop_word_fraction": 0.4969939887523651, "rps_doc_ut1_blacklist": 0, "rps_doc_frac_all_caps_words": 0.04108215868473053, "rps_doc_frac_lines_end_with_ellipsis": 0, "rps_doc_frac_no_alph_words": 0.14729459583759308, "rps_doc_frac_unique_words": 0.2953995168209076, "rps_doc_mean_word_length": 4.2953996658325195, "rps_doc_num_sentences": 40, "rps_doc_symbol_to_word_ratio": 0.0020040099043399096, "rps_doc_unigram_entropy": 4.870521068572998, "rps_doc_word_count": 826, "rps_doc_frac_chars_dupe_10grams": 0.37542277574539185, "rps_doc_frac_chars_dupe_5grams": 0.40248027443885803, "rps_doc_frac_chars_dupe_6grams": 0.37542277574539185, "rps_doc_frac_chars_dupe_7grams": 0.37542277574539185, "rps_doc_frac_chars_dupe_8grams": 0.37542277574539185, "rps_doc_frac_chars_dupe_9grams": 0.37542277574539185, "rps_doc_frac_chars_top_2gram": 0.04058625176548958, "rps_doc_frac_chars_top_3gram": 0.018320180475711823, "rps_doc_frac_chars_top_4gram": 0.023957159370183945, "rps_doc_books_importance": -455.0471496582031, "rps_doc_books_importance_length_correction": -455.0471496582031, "rps_doc_openwebtext_importance": -202.25840759277344, "rps_doc_openwebtext_importance_length_correction": -202.25840759277344, "rps_doc_wikipedia_importance": -116.99826049804688, "rps_doc_wikipedia_importance_length_correction": -116.99826049804688 }, "fasttext": { "dclm": 0.07456480711698532, "english": 0.9412298202514648, "fineweb_edu_approx": 1.76992666721344, "eai_general_math": 0.1800125241279602, "eai_open_web_math": 0.28082388639450073, "eai_web_code": 0.006594719830900431 } }
{ "free_decimal_correspondence": { "primary": { "code": "005.436", "labels": { "level_1": "General works, books and libraries, information sciences", "level_2": "", "level_3": "Computer programming" } }, "secondary": { "code": "005.1", "labels": { "level_1": "General works, books and libraries, information sciences", "level_2": "", "level_3": "Computer programming" } } }, "bloom_cognitive_process": { "primary": { "code": "2", "label": "Understand" }, "secondary": { "code": "3", "label": "Apply" } }, "bloom_knowledge_domain": { "primary": { "code": "2", "label": "Conceptual" }, "secondary": { "code": "3", "label": "Procedural" } }, "document_type_v1": { "primary": { "code": "5", "label": "Social/Forum" }, "secondary": { "code": "3", "label": "Reference/Encyclopedic/Educational" } }, "extraction_artifacts": { "primary": { "code": "3", "label": "Irrelevant Content" }, "secondary": { "code": "-1", "label": "Abstain" } }, "missing_content": { "primary": { "code": "0", "label": "No missing content" }, "secondary": { "code": "-1", "label": "Abstain" } }, "document_type_v2": { "primary": { "code": "18", "label": "Q&A Forum" }, "secondary": { "code": "8", "label": "Documentation" } }, "reasoning_depth": { "primary": { "code": "3", "label": "Intermediate Reasoning" }, "secondary": { "code": "2", "label": "Basic Reasoning" } }, "technical_correctness": { "primary": { "code": "4", "label": "Highly Correct" }, "secondary": { "code": "3", "label": "Mostly Correct" } }, "education_level": { "primary": { "code": "3", "label": "Undergraduate Level" }, "secondary": { "code": "2", "label": "High School Level" } } }
672f1e42c33a7f9846924a2431ea77df
-3,072,795,194,687,016,400
Apr 9th, 2022 🚨 The Required GitHub Status Check That Wasn't Hey there 👋 I would like to quickly plug a product I am working on to make teams move faster, happier. If you are working on a software product or are interested in features like preview environments, infrastructure secret management, access control and approval workflows for teams, and other topics in cloud resource management, make sure to check it out! A very important GitHub feature for teams is to configure branch protections. These typically include requiring pull requests and a minimum number of reviews, preventing force pushes to main branches, signing commits, and most importantly, passing status checks. Status checks can be added by external services to commits to signal issues. In addition to external services, GitHub Actions also reports workflow runs as status checks. This feature can be used to set up required CI steps (building the codebase, running tests, linting, etc.) that must pass before allowing to merge pull requests. At GraphCMS, we recently observed that GitHub would allow us to merge pull requests even though workflows failed. As this could lead to dangerous situations where broken code is merged based on false assumptions, I started investigating the matter. After reading up on a couple of forum posts and performing my own experiments to reproduce the issue, I came to the conclusion that combining the job matrix feature with required status checks leads to issues. We’ll first walk through a short primer on this specific feature, then see what is failing, and follow up by thinking about possible solutions. GitHub Actions Job Matrix or Sharding Jobs The job matrix feature is a great solution when you have multiple targets to run the same job against, or if you want to parallelize workflows like slow-running tests. You can define a matrix strategy, which will then schedule a dynamic number of jobs based on your requirements. Every job receives its own name. jobs: build: strategy: matrix: node: [10, 12, 14] steps: # Configures the node version used on GitHub-hosted runners - uses: actions/setup-node@v2 with: # The Node.js version to configure node-version: ${{ matrix.node }} I wrote a comprehensive post on the job matrix strategy last fall. Failing jobs will skip subsequent steps Whenever a job fails in CI, all subsequent jobs are skipped. This behaviour makes sense, as you want to fail fast in most cases. However, skipped jobs are not recognized by the required status check enforcement, allowing to merge in code even though the required step was never run. As an example, take the following GitHub Actions workflow name: Test on: pull_request: jobs: test: runs-on: ubuntu-20.04 steps: - run: exit 1 ci: runs-on: ubuntu-20.04 needs: test steps: - run: exit 0 We expect the test workflow to fail, after which the ci step will be skipped. If we check the GitHub status, we’ll see the issue with this process. skipped We can merge in our pull request even though the ci step never ran. The same happens when we use the job matrix on the first workflow. That’s really bad. So what can we do to solve this? Requiring job matrix shards A rather naive fix is that you can mark partial jobs from the job matrix as required. However, this also means that every time you add a new matrix item, you’ll have to update your branch configuration. It would be much better to have a required second job that always runs but only passes when all shards or previous jobs actually passed. Conditional success output After a lot of experimenting, I came across a pattern that works both for all possible configurations: Running two jobs after the build/test/etc. step. After running the tests, a success job will only run if all jobs before it succeeded. If any job before it failed, it will be skipped. In order to make it work with required status checks (to avoid allowing merges of skipped checks), we introduce another job, which always runs and only passes in case the success job ran. For this, we use GitHub Actions job outputs. These can be declared on jobs to reuse values in downstream jobs. First, let’s check out the job running after our tests. In this case, I used the job matrix strategy to parallelize tests. after-shards: needs: shards # run after shards runs-on: ubuntu-20.04 if: success() # only run when all shards have passed # store success output flag for ci job outputs: success: ${{ steps.setoutput.outputs.success }} steps: - id: setoutput run: echo "::set-output name=success::true" In this job, we merely set an output value to true in case we run this job. This only happens when all previous jobs it depends on have passed. ci: runs-on: ubuntu-20.04 if: always() # always run, so we never skip the check needs: [shards, after-shards] steps: # pass step only when output of previous after-shards job is set # in case at least one of the shard fails, after-shards is skipped # and the output will not be set, which will then cause the ci job to fail - run: | passed="${{ needs.after-shards.outputs.success }}" if [[ $passed == "true" ]]; then echo "Shards passed" exit 0 else echo "Shards failed" exit 1 fi The second step reads the previous job’s output and passes only if it’s set. The key piece of the second job is that it always runs, irrespective of any failed predecessors. Not adding always() will lead to the original issue, where the required job is skipped. Last but not least, make sure to require the ci job in your branch protection settings. failed passed Below is the complete test action, including an easy way to simulate failing tests to check different scenarios. name: Test on: pull_request: jobs: shards: strategy: matrix: shards: [1, 2, 3, 4] runs-on: ubuntu-20.04 steps: - run: echo doing stuff ${{ matrix.shards }} # simulate failure in one or more shards - name: fail one run: exit 1 if: ${{ matrix.shards == 1}} - run: exit 0 after-shards: needs: shards # run after shards runs-on: ubuntu-20.04 if: success() # only run when all shards have passed # store success output flag for ci job outputs: success: ${{ steps.setoutput.outputs.success }} steps: - id: setoutput run: echo "::set-output name=success::true" ci: runs-on: ubuntu-20.04 if: always() # always run, so we never skip the check needs: [shards, after-shards] steps: # pass step only when output of previous after-shards job is set # in case at least one of the shard fails, after-shards is skipped # and the output will not be set, which will then cause the ci job to fail - run: | passed="${{ needs.after-shards.outputs.success }}" if [[ $passed == "true" ]]; then echo "Shards passed" exit 0 else echo "Shards failed" exit 1 fi This was quite an adventure. Noticing that you can merge broken code when you rely on CI is a DevOps nightmare come reality. I do understand why GitHub Actions work the way they do, but the integration with required status checks isn’t optimal. Thanks for reading this post 🙌 I would like to quickly plug a product I am working on to make teams move faster, happier. If you are working on a software product or are interested in features like preview environments, infrastructure secret management, access control and approval workflows for teams, and other topics in cloud resource management, make sure to check it out! Bruno Scheufler Software Engineering, Management On other platforms
{ "url": "https://brunoscheufler.com/blog/2022-04-09-the-required-github-status-check-that-wasnt", "source_domain": "brunoscheufler.com", "snapshot_id": "crawl=CC-MAIN-2022-21", "warc_metadata": { "Content-Length": "80238", "Content-Type": "application/http; msgtype=response", "WARC-Block-Digest": "sha1:P67EFH6QFYPJ3SCUVIPLSLAVXV5HCYAD", "WARC-Concurrent-To": "<urn:uuid:fc768702-2594-4027-b1b8-1b56f5c84bd4>", "WARC-Date": "2022-05-28T19:12:11Z", "WARC-IP-Address": "76.76.21.21", "WARC-Identified-Payload-Type": "text/html", "WARC-Payload-Digest": "sha1:I5HSQ7HUZYZ3QMG6LLTHS76HGU7WIKL7", "WARC-Record-ID": "<urn:uuid:9f74ef9e-8abd-49a7-a1be-6004a34c6c1f>", "WARC-Target-URI": "https://brunoscheufler.com/blog/2022-04-09-the-required-github-status-check-that-wasnt", "WARC-Truncated": null, "WARC-Type": "response", "WARC-Warcinfo-ID": "<urn:uuid:8f3e24b1-98e3-4086-807c-8f343019a475>" }, "warc_info": "isPartOf: CC-MAIN-2022-21\r\npublisher: Common Crawl\r\ndescription: Wide crawl of the web for May 2022\r\noperator: Common Crawl Admin ([email protected])\r\nhostname: ip-10-67-67-98\r\nsoftware: Apache Nutch 1.18 (modified, https://github.com/commoncrawl/nutch/)\r\nrobots: checked via crawler-commons 1.3-SNAPSHOT (https://github.com/crawler-commons/crawler-commons)\r\nformat: WARC File Format 1.1\r\nconformsTo: https://iipc.github.io/warc-specifications/specifications/warc-format/warc-1.1/" }
{ "line_start_idx": [ 0, 14, 15, 62, 63, 421, 422, 685, 686, 1019, 1020, 1269, 1270, 1624, 1625, 1668, 1669, 1982, 1983, 1989, 1998, 2012, 2026, 2053, 2066, 2134, 2172, 2188, 2235, 2280, 2281, 2348, 2349, 2389, 2390, 2673, 2674, 2732, 2733, 2744, 2748, 2764, 2770, 2778, 2804, 2815, 2835, 2841, 2867, 2883, 2894, 2914, 2915, 3063, 3064, 3072, 3073, 3260, 3261, 3289, 3290, 3630, 3631, 3658, 3659, 4134, 4135, 4246, 4247, 4370, 4371, 4385, 4420, 4444, 4499, 4540, 4551, 4603, 4612, 4632, 4682, 4683, 4827, 4828, 4832, 4856, 4912, 4944, 4953, 5022, 5093, 5172, 5185, 5244, 5285, 5316, 5333, 5346, 5377, 5394, 5405, 5406, 5756, 5757, 5764, 5765, 5772, 5773, 5886, 5887, 5898, 5902, 5918, 5924, 5934, 5948, 5962, 5991, 6017, 6028, 6079, 6126, 6149, 6169, 6206, 6226, 6242, 6279, 6305, 6362, 6405, 6418, 6472, 6483, 6505, 6557, 6563, 6589, 6647, 6681, 6692, 6763, 6836, 6917, 6932, 6993, 7036, 7069, 7088, 7103, 7136, 7155, 7168, 7169, 7414, 7415, 7792, 7793, 7809, 7810, 7843, 7844 ], "line_end_idx": [ 14, 15, 62, 63, 421, 422, 685, 686, 1019, 1020, 1269, 1270, 1624, 1625, 1668, 1669, 1982, 1983, 1989, 1998, 2012, 2026, 2053, 2066, 2134, 2172, 2188, 2235, 2280, 2281, 2348, 2349, 2389, 2390, 2673, 2674, 2732, 2733, 2744, 2748, 2764, 2770, 2778, 2804, 2815, 2835, 2841, 2867, 2883, 2894, 2914, 2915, 3063, 3064, 3072, 3073, 3260, 3261, 3289, 3290, 3630, 3631, 3658, 3659, 4134, 4135, 4246, 4247, 4370, 4371, 4385, 4420, 4444, 4499, 4540, 4551, 4603, 4612, 4632, 4682, 4683, 4827, 4828, 4832, 4856, 4912, 4944, 4953, 5022, 5093, 5172, 5185, 5244, 5285, 5316, 5333, 5346, 5377, 5394, 5405, 5406, 5756, 5757, 5764, 5765, 5772, 5773, 5886, 5887, 5898, 5902, 5918, 5924, 5934, 5948, 5962, 5991, 6017, 6028, 6079, 6126, 6149, 6169, 6206, 6226, 6242, 6279, 6305, 6362, 6405, 6418, 6472, 6483, 6505, 6557, 6563, 6589, 6647, 6681, 6692, 6763, 6836, 6917, 6932, 6993, 7036, 7069, 7088, 7103, 7136, 7155, 7168, 7169, 7414, 7415, 7792, 7793, 7809, 7810, 7843, 7844, 7862 ] }
{ "red_pajama_v2": { "ccnet_original_length": 7862, "ccnet_original_nlines": 161, "rps_doc_curly_bracket": 0.003561429912224412, "rps_doc_ldnoobw_words": 0, "rps_doc_lorem_ipsum": 0, "rps_doc_stop_word_fraction": 0.3243412673473358, "rps_doc_ut1_blacklist": 0, "rps_doc_frac_all_caps_words": 0.00941029004752636, "rps_doc_frac_lines_end_with_ellipsis": 0, "rps_doc_frac_no_alph_words": 0.2302383929491043, "rps_doc_frac_unique_words": 0.3215785026550293, "rps_doc_mean_word_length": 4.811922550201416, "rps_doc_num_sentences": 73, "rps_doc_symbol_to_word_ratio": 0.010664990171790123, "rps_doc_unigram_entropy": 5.3844099044799805, "rps_doc_word_count": 1191, "rps_doc_frac_chars_dupe_10grams": 0.2917466461658478, "rps_doc_frac_chars_dupe_5grams": 0.30047112703323364, "rps_doc_frac_chars_dupe_6grams": 0.2917466461658478, "rps_doc_frac_chars_dupe_7grams": 0.2917466461658478, "rps_doc_frac_chars_dupe_8grams": 0.2917466461658478, "rps_doc_frac_chars_dupe_9grams": 0.2917466461658478, "rps_doc_frac_chars_top_2gram": 0.01256325002759695, "rps_doc_frac_chars_top_3gram": 0.01256325002759695, "rps_doc_frac_chars_top_4gram": 0.01256325002759695, "rps_doc_books_importance": -726.2105712890625, "rps_doc_books_importance_length_correction": -726.2105712890625, "rps_doc_openwebtext_importance": -420.6436462402344, "rps_doc_openwebtext_importance_length_correction": -420.6436462402344, "rps_doc_wikipedia_importance": -326.9654846191406, "rps_doc_wikipedia_importance_length_correction": -326.9654846191406 }, "fasttext": { "dclm": 0.1799188256263733, "english": 0.9136526584625244, "fineweb_edu_approx": 1.5164611339569092, "eai_general_math": 0.5529574751853943, "eai_open_web_math": 0.09537292271852493, "eai_web_code": 0.9564937353134155 } }
{ "free_decimal_correspondence": { "primary": { "code": "005.452", "labels": { "level_1": "General works, books and libraries, information sciences", "level_2": "", "level_3": "Computer programming" } }, "secondary": { "code": "005.1", "labels": { "level_1": "General works, books and libraries, information sciences", "level_2": "", "level_3": "Computer programming" } } }, "bloom_cognitive_process": { "primary": { "code": "4", "label": "Analyze" }, "secondary": { "code": "3", "label": "Apply" } }, "bloom_knowledge_domain": { "primary": { "code": "3", "label": "Procedural" }, "secondary": { "code": "2", "label": "Conceptual" } }, "document_type_v1": { "primary": { "code": "3", "label": "Reference/Encyclopedic/Educational" }, "secondary": { "code": "-1", "label": "Abstain" } }, "extraction_artifacts": { "primary": { "code": "0", "label": "No Artifacts" }, "secondary": { "code": "-1", "label": "Abstain" } }, "missing_content": { "primary": { "code": "0", "label": "No missing content" }, "secondary": { "code": "-1", "label": "Abstain" } }, "document_type_v2": { "primary": { "code": "16", "label": "Personal Blog" }, "secondary": { "code": "23", "label": "Tutorial" } }, "reasoning_depth": { "primary": { "code": "3", "label": "Intermediate Reasoning" }, "secondary": { "code": "2", "label": "Basic Reasoning" } }, "technical_correctness": { "primary": { "code": "4", "label": "Highly Correct" }, "secondary": { "code": "3", "label": "Mostly Correct" } }, "education_level": { "primary": { "code": "3", "label": "Undergraduate Level" }, "secondary": { "code": "2", "label": "High School Level" } } }
672f1e42c33a7f9846924a2431ea77df
-1,300,280,027,167,248,000
Coming up with and remembering passwords can be a challenge. PasswordCard offers a free, simple solution that leaves you a hard copy of your passwords. The credit card -sized card fits easily in your wallet or purse. When you arrive at http://www.passwordcard.org/en, you’ll see that it’s a pretty simple set-up. password-card-page A card number is generated for you. Make sure to save that card number somewhere. You can use it to regenerate the card if you ever lose it. password-card-page-card-number You can choose to have an area with only digits (great if you need PIN numbers) or to include symbols. password-card-page-card-digits Then you can print the card, laminate it and keep it in your wallet. Here’s how it works. You pick a symbol, color, and length for your password. Then choose to go left or right. For example, if I needed a 10-letter password, I could choose pink for  a color and musical note for a symbol. Then I would go 10 to the right for a password of WWyVGGKkCD. password-card-page-card-circle I could write down Pink-Music-Note-10-Right somewhere. Even if someone came across it, without my specific card, they wouldn’t be able to figure out the password. You can also randomly generate a new card at any time.
{ "url": "https://cynmackley.com/2021/01/16/a-handy-way-to-remember-your-password/", "source_domain": "cynmackley.com", "snapshot_id": "crawl=CC-MAIN-2021-10", "warc_metadata": { "Content-Length": "76212", "Content-Type": "application/http; msgtype=response", "WARC-Block-Digest": "sha1:7OTFGGDMJYEDF7GXBIUBSFJHRYXARLUJ", "WARC-Concurrent-To": "<urn:uuid:694c5b14-5783-40fe-842b-294177314b0c>", "WARC-Date": "2021-03-02T02:23:02Z", "WARC-IP-Address": "192.0.78.220", "WARC-Identified-Payload-Type": "text/html", "WARC-Payload-Digest": "sha1:SPRGT4VUXVB7NHVPG4OXGY4XJ7VVDZT4", "WARC-Record-ID": "<urn:uuid:f1444884-9b39-497e-8981-72f362c233b5>", "WARC-Target-URI": "https://cynmackley.com/2021/01/16/a-handy-way-to-remember-your-password/", "WARC-Truncated": null, "WARC-Type": "response", "WARC-Warcinfo-ID": "<urn:uuid:b95b9ea7-dc3d-4510-9357-9577ccf0f30d>" }, "warc_info": "isPartOf: CC-MAIN-2021-10\r\npublisher: Common Crawl\r\ndescription: Wide crawl of the web for February/March 2021\r\noperator: Common Crawl Admin ([email protected])\r\nhostname: ip-10-67-67-43.ec2.internal\r\nsoftware: Apache Nutch 1.18 (modified, https://github.com/commoncrawl/nutch/)\r\nrobots: checked via crawler-commons 1.2-SNAPSHOT (https://github.com/crawler-commons/crawler-commons)\r\nformat: WARC File Format 1.1\r\nconformsTo: http://iipc.github.io/warc-specifications/specifications/warc-format/warc-1.1/" }
{ "line_start_idx": [ 0, 152, 153, 218, 219, 315, 316, 335, 476, 477, 508, 509, 612, 613, 644, 645, 809, 997, 998, 1029, 1030 ], "line_end_idx": [ 152, 153, 218, 219, 315, 316, 335, 476, 477, 508, 509, 612, 613, 644, 645, 809, 997, 998, 1029, 1030, 1247 ] }
{ "red_pajama_v2": { "ccnet_original_length": 1247, "ccnet_original_nlines": 20, "rps_doc_curly_bracket": 0, "rps_doc_ldnoobw_words": 0, "rps_doc_lorem_ipsum": 0, "rps_doc_stop_word_fraction": 0.38013699650764465, "rps_doc_ut1_blacklist": 0, "rps_doc_frac_all_caps_words": 0.020547950640320778, "rps_doc_frac_lines_end_with_ellipsis": 0, "rps_doc_frac_no_alph_words": 0.2054794430732727, "rps_doc_frac_unique_words": 0.5748792290687561, "rps_doc_mean_word_length": 4.714975833892822, "rps_doc_num_sentences": 19, "rps_doc_symbol_to_word_ratio": 0, "rps_doc_unigram_entropy": 4.468183994293213, "rps_doc_word_count": 207, "rps_doc_frac_chars_dupe_10grams": 0, "rps_doc_frac_chars_dupe_5grams": 0, "rps_doc_frac_chars_dupe_6grams": 0, "rps_doc_frac_chars_dupe_7grams": 0, "rps_doc_frac_chars_dupe_8grams": 0, "rps_doc_frac_chars_dupe_9grams": 0, "rps_doc_frac_chars_top_2gram": 0.024590160697698593, "rps_doc_frac_chars_top_3gram": 0.024590160697698593, "rps_doc_frac_chars_top_4gram": 0, "rps_doc_books_importance": -136.53610229492188, "rps_doc_books_importance_length_correction": -136.53610229492188, "rps_doc_openwebtext_importance": -107.42752838134766, "rps_doc_openwebtext_importance_length_correction": -107.42752838134766, "rps_doc_wikipedia_importance": -81.82405090332031, "rps_doc_wikipedia_importance_length_correction": -81.82405090332031 }, "fasttext": { "dclm": 0.6638792753219604, "english": 0.8984573483467102, "fineweb_edu_approx": 1.6226073503494263, "eai_general_math": 0.5534031987190247, "eai_open_web_math": 0.2561175227165222, "eai_web_code": 0.0862017273902893 } }
{ "free_decimal_correspondence": { "primary": { "code": "005.822", "labels": { "level_1": "General works, books and libraries, information sciences", "level_2": "", "level_3": "Computer programming" } }, "secondary": { "code": "658.8", "labels": { "level_1": "Industrial arts, Technology, and Engineering", "level_2": "Business", "level_3": "Management" } } }, "bloom_cognitive_process": { "primary": { "code": "3", "label": "Apply" }, "secondary": { "code": "2", "label": "Understand" } }, "bloom_knowledge_domain": { "primary": { "code": "3", "label": "Procedural" }, "secondary": { "code": "2", "label": "Conceptual" } }, "document_type_v1": { "primary": { "code": "3", "label": "Reference/Encyclopedic/Educational" }, "secondary": { "code": "-1", "label": "Abstain" } }, "extraction_artifacts": { "primary": { "code": "0", "label": "No Artifacts" }, "secondary": { "code": "-1", "label": "Abstain" } }, "missing_content": { "primary": { "code": "0", "label": "No missing content" }, "secondary": { "code": "-1", "label": "Abstain" } }, "document_type_v2": { "primary": { "code": "23", "label": "Tutorial" }, "secondary": { "code": "17", "label": "Product Page" } }, "reasoning_depth": { "primary": { "code": "2", "label": "Basic Reasoning" }, "secondary": { "code": "1", "label": "No Reasoning" } }, "technical_correctness": { "primary": { "code": "6", "label": "Not Applicable/Indeterminate" }, "secondary": { "code": "4", "label": "Highly Correct" } }, "education_level": { "primary": { "code": "1", "label": "General Audience" }, "secondary": { "code": "2", "label": "High School Level" } } }
672f1e42c33a7f9846924a2431ea77df
-4,018,726,334,287,262,700
Do you think Apple should create a "Back Door" to people's Iphone,iPod etc... for the police? Yes 42.9% 3 votes No 57.1% 4 votes Be the first to add this poll to starred list! ▼Scroll down for more polls Comments (4) Whisker_Queer Absolutely reply Report on April 29 Rebel_Assasin "Back door"? reply RpgGamer A way they could get into your phone or ipod too see your stuff.. I heard of something like this in computer class where apple won't crack a phone for the FBI. (this would probably explain the situation better) http://techcrunch.com/2016/02/17/tim-cook-apple-wont-create-backdoor-to-unlock-san-bernardino-attackers-iphone/ reply Report on April 15 Report on April 15 DipperPinesShades It ruins people's privacy. There a risks both ways, though reply Report on April 15
{ "url": "https://www.qfeast.com/poll/AoX5b8/Do-you-think-Apple-should-create-a-Back-Door-to-people-s-Iphone-iPod-etc-for-the-police", "source_domain": "www.qfeast.com", "snapshot_id": "crawl=CC-MAIN-2016-50", "warc_metadata": { "Content-Length": "155103", "Content-Type": "application/http; msgtype=response", "WARC-Block-Digest": "sha1:IBBBSDVPCIUITOF5WLR5QAOTDIAUPKB3", "WARC-Concurrent-To": "<urn:uuid:feaa94e8-b307-47bd-8a88-e826a16680a4>", "WARC-Date": "2016-12-05T02:43:42Z", "WARC-IP-Address": "92.55.147.192", "WARC-Identified-Payload-Type": null, "WARC-Payload-Digest": "sha1:KQDYPXQLW65K5PC3IM3YRL2ZNBOWOHXG", "WARC-Record-ID": "<urn:uuid:4942bf33-364b-45df-8ca5-d888f6eb44ba>", "WARC-Target-URI": "https://www.qfeast.com/poll/AoX5b8/Do-you-think-Apple-should-create-a-Back-Door-to-people-s-Iphone-iPod-etc-for-the-police", "WARC-Truncated": null, "WARC-Type": "response", "WARC-Warcinfo-ID": "<urn:uuid:45679e61-e748-4b29-9405-8013dd84ee05>" }, "warc_info": "robots: classic\r\nhostname: ip-10-31-129-80.ec2.internal\r\nsoftware: Nutch 1.6 (CC)/CC WarcExport 1.0\r\nisPartOf: CC-MAIN-2016-50\r\noperator: CommonCrawl Admin\r\ndescription: Wide crawl of the web for November 2016\r\npublisher: CommonCrawl\r\nformat: WARC File Format 1.0\r\nconformsTo: http://bibnum.bnf.fr/WARC/WARC_ISO_28500_version1_latestdraft.pdf" }
{ "line_start_idx": [ 0, 94, 95, 99, 105, 113, 116, 122, 130, 177, 205, 206, 219, 220, 234, 245, 251, 258, 270, 284, 297, 303, 312, 378, 427, 472, 473, 524, 636, 642, 649, 661, 668, 680, 698, 725, 757, 763, 770 ], "line_end_idx": [ 94, 95, 99, 105, 113, 116, 122, 130, 177, 205, 206, 219, 220, 234, 245, 251, 258, 270, 284, 297, 303, 312, 378, 427, 472, 473, 524, 636, 642, 649, 661, 668, 680, 698, 725, 757, 763, 770, 781 ] }
{ "red_pajama_v2": { "ccnet_original_length": 781, "ccnet_original_nlines": 38, "rps_doc_curly_bracket": 0, "rps_doc_ldnoobw_words": 0, "rps_doc_lorem_ipsum": 0, "rps_doc_stop_word_fraction": 0.284153014421463, "rps_doc_ut1_blacklist": 0, "rps_doc_frac_all_caps_words": 0.01639343984425068, "rps_doc_frac_lines_end_with_ellipsis": 0, "rps_doc_frac_no_alph_words": 0.3060109317302704, "rps_doc_frac_unique_words": 0.7118644118309021, "rps_doc_mean_word_length": 5.177966117858887, "rps_doc_num_sentences": 11, "rps_doc_symbol_to_word_ratio": 0.005464479792863131, "rps_doc_unigram_entropy": 4.283017158508301, "rps_doc_word_count": 118, "rps_doc_frac_chars_dupe_10grams": 0, "rps_doc_frac_chars_dupe_5grams": 0.06546644866466522, "rps_doc_frac_chars_dupe_6grams": 0, "rps_doc_frac_chars_dupe_7grams": 0, "rps_doc_frac_chars_dupe_8grams": 0, "rps_doc_frac_chars_dupe_9grams": 0, "rps_doc_frac_chars_top_2gram": 0.05237315967679024, "rps_doc_frac_chars_top_3gram": 0.08510638028383255, "rps_doc_frac_chars_top_4gram": 0.08837971091270447, "rps_doc_books_importance": -72.49921417236328, "rps_doc_books_importance_length_correction": -72.49921417236328, "rps_doc_openwebtext_importance": -45.34700012207031, "rps_doc_openwebtext_importance_length_correction": -45.34698486328125, "rps_doc_wikipedia_importance": -37.814064025878906, "rps_doc_wikipedia_importance_length_correction": -37.814064025878906 }, "fasttext": { "dclm": 0.018230920657515526, "english": 0.9073490500450134, "fineweb_edu_approx": 1.3883334398269653, "eai_general_math": 0.0007346299826167524, "eai_open_web_math": 0.331534206867218, "eai_web_code": -0.000008820000402920414 } }
{ "free_decimal_correspondence": { "primary": { "code": "004.67", "labels": { "level_1": "General works, books and libraries, information sciences", "level_2": "", "level_3": "Computers and Computer science" } }, "secondary": { "code": "363.25", "labels": { "level_1": "Social sciences", "level_2": "Social service and Societies", "level_3": "Political activists" } } }, "bloom_cognitive_process": { "primary": { "code": "5", "label": "Evaluate" }, "secondary": { "code": "2", "label": "Understand" } }, "bloom_knowledge_domain": { "primary": { "code": "2", "label": "Conceptual" }, "secondary": { "code": "3", "label": "Procedural" } }, "document_type_v1": { "primary": { "code": "5", "label": "Social/Forum" }, "secondary": { "code": "-1", "label": "Abstain" } }, "extraction_artifacts": { "primary": { "code": "0", "label": "No Artifacts" }, "secondary": { "code": "-1", "label": "Abstain" } }, "missing_content": { "primary": { "code": "0", "label": "No missing content" }, "secondary": { "code": "-1", "label": "Abstain" } }, "document_type_v2": { "primary": { "code": "5", "label": "Comment Section" }, "secondary": { "code": "18", "label": "Q&A Forum" } }, "reasoning_depth": { "primary": { "code": "2", "label": "Basic Reasoning" }, "secondary": { "code": "3", "label": "Intermediate Reasoning" } }, "technical_correctness": { "primary": { "code": "6", "label": "Not Applicable/Indeterminate" }, "secondary": { "code": "2", "label": "Partially Correct" } }, "education_level": { "primary": { "code": "1", "label": "General Audience" }, "secondary": { "code": "2", "label": "High School Level" } } }
672f1e42c33a7f9846924a2431ea77df
5,839,484,673,775,747,000
Thoughts on MaxScale new license MaxScale has been open source until now, just like all MariaDB projects. But the 2.0 version is released under a new license called BSL, which basically makes the covered work non-free until the Change Date (in this case 2019-01-01), when the license will be converted to GPL. Looks like open source friendly, after all. The license will be GPL, just be patient. And the code is available. Right? No. Cmpletely wrong. For plenty of reasons. Some reasons It is a lock-in. No matter how many times Monty repeats that there is no lock-in, we have a brain. If you don’t allow anyone to fix bugs except for yourself, it is a lock-in. If you force your users to buy your support, they won’t buy your competitors support. MariaDB business moves to a non-free product. Yes, 1.4 is free an this won’t change. And yes, when 3.0 will be out, 2.0 will be free. But why should they maintain a free version, if money comes from non-free versions? Monty says that open source religion doesn’t put bread on the table. I suppose that maintaining free branches also doesn’t put bread on the table. I wasn’t able to find any official EOL date for any MaxScale version – if there is one, please comment below. MariaDB moves innovation to the non-free world. New features are non-free. When they will be old, they will be free. Monty also stated that this is the correct way to make money for a lot of projects. And he seems to advice this model to start-ups that use his venture capital, OpenOcean. Suddenly, BSL seems to be the only way for projects to survive. Is he protecting others projects interests, or using them for his own marketing? MariaDB accused Oracle several times. When Oracle implemented a couple features and only distributed them in a non-GPL edition (threadpool, PAM authentication), MariaDB told that they had the same features as open source. Which was great. Except that… now MySQL Router is open source, MaxScale 2.0 is not. Now Monty has several justifications for this. But I fail to understand why open core is evil and BSL is good. I mentioned Monty too many times. Is this an attack against Monty? Definitely not, but all articles I could find express Monty’s opinion, not MariaDB Corporation or anyone else’s opinion. I cannot answer the silence. What is the MariaDB Foundation? MariaDB Corporation has the legal right to make MaxScale non-free. They own it. They sometimes call it MariaDB MaxScale. They can: they also own MariaDB trademark. So, what’s the role of MariaDB Foundation? They claim they safeguard MariaDB. They don’t mention the ecosystem, the community, or other tools. They don’t mention, of course, MaxScale. Which is quite strange: they claimed that their model is Apache Foundation, which supports an entire ecosystem in many ways, and owns the trademarks. Also, the board of directors has 6 members. 3 are from MariaDB Foundation. In this situation, they cannot have an independent opinion on MariaDB Corporation actions. A curious aspect is that they declare they follow Ubuntu Code of Conduct. Please read its last paragraph and drawn your own conclusions. My position on MariaDB and MaxScale I am still grateful to MariaDB Corporation for creating and maintaining MariaDB (and to some of their engineers for creating MySQL). From a technical point of view, they have many interesting features that are not in MySQL. Some of them come from the community, for example the CONNECT engine and their implementation of encryption. And the reason is that MariaDB is very open to the community. Which brings us to a less technical point of view: MariaDB openness. Their JIRA account allows us to see the bugs (including their current status…). You can also see who is working on what, when next versions will be released, and what they will have. The team is active on the mailing lists and IRC. The documentation is a wiki and the license is free. I have been a MariaDB supporter for years. I wrote Mastering MariaDB and I am one of their Community Ambassadors chosen by Colin Charles (who recently left MariaDB). Will my position about MariaDB project change? I don’t know, it’s too early to answer. For sure, I won’t deny that its openness is amazing and should be a model for everyone. (And I hope this won’t change) And my position about MaxScale has changed? Of course it did. I wouldn’t use it for personal projects. Of course I could provide support but, given the license change, it seems to me unlikely. There are free alternatives: ProxySQL, MySQL Router, HAProxy. PoxySQL is by far the most interesting, if you ask me. My position has changed forever? The answer depends on another question: will MariaDB fix its big mistake? I have no logic reasons to be optimistic, but I still hope it will. In the past they have apparently been open to criticism. After a complain in this blog, they made MaxScale binaries freely available, and I wrote a thank you post. What I couldn’t know is that they were preparing to close MaxScale next versions.   Advertisements 11 thoughts on “Thoughts on MaxScale new license 1. Pingback: 최근 mariadb maxscale의 bsl 관련 글들 | osskorea 2. “I am still grateful to MariaDB Corporation for creating and maintaining MariaDB”: was it the MariaDB Corporation (MariaDB.com) that created MariaDB Server ? I know they created MaxScale (well, SkySQL did and SkySQL was renamed MariaDB.com), but I think MariaDB Server was created a long time before that. • I do not know the full history of the MariaDB Server, but I think Monty Program was involved, and then SkySQL, and then MariaDB.com. I was mostly struck by the affirmation “MariaDB Corporation creating MariaDB [Server]”. The maintaining part is correct, but the creating part is as right (or false) as Oracle creating MySQL. 3. I’m also not entirely clear why people think MariaDB development is entirely open. Can you find any JIRA or mailing list discussion about the license change before it happened? License changes don’t just suddenly happen in open source development. I’ve already documented my thoughts on the MariaDB bugs system here: http://mysqlblog.fivefarmers.com/2016/02/24/questions-about-mariadbs-bug-system/ Whether you give MariaDB the benefit of the doubt and assume that all we cannot see is appropriate or not will depend on your feelings towards MariaDB. • I don’t see that kind of discussion from any company I know. And if I was a manager I wouldn’t certainly make them public. I don’t really have any problems with MariaDB in this respect. But the BSL is a bad, radical change, in my opinion. • Hi JFG, That wasn’t the case when I asked who owned the new jira.mariadb.org system back in February. Maybe you have information that it’s changed since, but Sergei Golubchik responded at the time that the system was jointly owned. I still see non-MDEV projects in that JIRA system, so my belief is that nothing has changed. It’s just another example of confusion between the boundaries of the foundation and the corporation. • Yes kudos to Justin for a lot of reasons, including ensuring that a GPL branch of MaxScale remains public. But I don’t think he will fix bugs or develop new features. GPLScale will become an alternative if/when someone start to maintain and improve it, in my opinion. Leave a comment Fill in your details below or click an icon to log in: WordPress.com Logo You are commenting using your WordPress.com account. Log Out / Change ) Twitter picture You are commenting using your Twitter account. Log Out / Change ) Facebook photo You are commenting using your Facebook account. Log Out / Change ) Google+ photo You are commenting using your Google+ account. Log Out / Change ) Connecting to %s
{ "url": "https://falseisnotnull.wordpress.com/2016/08/24/thoughts-on-maxscale-new-license/", "source_domain": "falseisnotnull.wordpress.com", "snapshot_id": "crawl=CC-MAIN-2017-43", "warc_metadata": { "Content-Length": "73331", "Content-Type": "application/http; msgtype=response", "WARC-Block-Digest": "sha1:TZGL6B66MSMYIMUK4AWQFJYD6T6YP6DO", "WARC-Concurrent-To": "<urn:uuid:434f79ee-a214-423b-8d06-ffb26ef11d01>", "WARC-Date": "2017-10-21T04:42:04Z", "WARC-IP-Address": "192.0.78.12", "WARC-Identified-Payload-Type": "text/html", "WARC-Payload-Digest": "sha1:YPFR4WKMLKQHGBP4G7G2EI4D6ZKMA7GZ", "WARC-Record-ID": "<urn:uuid:0cae05ee-15c5-4daf-9e7f-c46cf9299397>", "WARC-Target-URI": "https://falseisnotnull.wordpress.com/2016/08/24/thoughts-on-maxscale-new-license/", "WARC-Truncated": "length", "WARC-Type": "response", "WARC-Warcinfo-ID": "<urn:uuid:0a6d189d-a3d3-4159-9cbf-6387a0c9e45a>" }, "warc_info": "robots: classic\r\nhostname: ip-10-157-174-116.ec2.internal\r\nsoftware: Nutch 1.6 (CC)\r\nisPartOf: CC-MAIN-2017-43\r\noperator: Common Crawl Admin\r\ndescription: Wide crawl of the web for October 2017\r\npublisher: Common Crawl\r\nformat: WARC File Format 1.0\r\nconformsTo: http://bibnum.bnf.fr/WARC/WARC_ISO_28500_version1_latestdraft.pdf" }
{ "line_start_idx": [ 0, 33, 34, 311, 312, 432, 433, 477, 478, 491, 492, 753, 754, 1119, 1120, 1230, 1231, 1665, 1666, 2083, 2084, 2301, 2302, 2334, 2335, 2499, 2500, 2543, 2544, 2835, 2836, 3002, 3003, 3140, 3141, 3177, 3178, 3311, 3312, 3574, 3575, 3929, 3930, 4302, 4303, 4613, 4614, 5035, 5036, 5038, 5039, 5054, 5055, 5104, 5105, 5162, 5163, 5474, 5475, 5616, 5617, 5817, 5818, 6071, 6072, 6145, 6146, 6231, 6232, 6388, 6389, 6634, 6635, 6651, 6652, 7078, 7079, 7353, 7354, 7370, 7371, 7426, 7427, 7446, 7447, 7519, 7520, 7536, 7537, 7603, 7604, 7619, 7620, 7687, 7688, 7702, 7703, 7769, 7770 ], "line_end_idx": [ 33, 34, 311, 312, 432, 433, 477, 478, 491, 492, 753, 754, 1119, 1120, 1230, 1231, 1665, 1666, 2083, 2084, 2301, 2302, 2334, 2335, 2499, 2500, 2543, 2544, 2835, 2836, 3002, 3003, 3140, 3141, 3177, 3178, 3311, 3312, 3574, 3575, 3929, 3930, 4302, 4303, 4613, 4614, 5035, 5036, 5038, 5039, 5054, 5055, 5104, 5105, 5162, 5163, 5474, 5475, 5616, 5617, 5817, 5818, 6071, 6072, 6145, 6146, 6231, 6232, 6388, 6389, 6634, 6635, 6651, 6652, 7078, 7079, 7353, 7354, 7370, 7371, 7426, 7427, 7446, 7447, 7519, 7520, 7536, 7537, 7603, 7604, 7619, 7620, 7687, 7688, 7702, 7703, 7769, 7770, 7786 ] }
{ "red_pajama_v2": { "ccnet_original_length": 7786, "ccnet_original_nlines": 98, "rps_doc_curly_bracket": 0, "rps_doc_ldnoobw_words": 0, "rps_doc_lorem_ipsum": 0, "rps_doc_stop_word_fraction": 0.3991389870643616, "rps_doc_ut1_blacklist": 0, "rps_doc_frac_all_caps_words": 0.032595328986644745, "rps_doc_frac_lines_end_with_ellipsis": 0, "rps_doc_frac_no_alph_words": 0.18265682458877563, "rps_doc_frac_unique_words": 0.3632148504257202, "rps_doc_mean_word_length": 4.748840808868408, "rps_doc_num_sentences": 120, "rps_doc_symbol_to_word_ratio": 0.0012300099479034543, "rps_doc_unigram_entropy": 5.497661590576172, "rps_doc_word_count": 1294, "rps_doc_frac_chars_dupe_10grams": 0.019528070464730263, "rps_doc_frac_chars_dupe_5grams": 0.045890968292951584, "rps_doc_frac_chars_dupe_6grams": 0.03775427117943764, "rps_doc_frac_chars_dupe_7grams": 0.02799024060368538, "rps_doc_frac_chars_dupe_8grams": 0.019528070464730263, "rps_doc_frac_chars_dupe_9grams": 0.019528070464730263, "rps_doc_frac_chars_top_2gram": 0.006834830157458782, "rps_doc_frac_chars_top_3gram": 0.010414970107376575, "rps_doc_frac_chars_top_4gram": 0.013669650070369244, "rps_doc_books_importance": -659.2125854492188, "rps_doc_books_importance_length_correction": -659.2125854492188, "rps_doc_openwebtext_importance": -382.0086364746094, "rps_doc_openwebtext_importance_length_correction": -382.0086364746094, "rps_doc_wikipedia_importance": -326.7724609375, "rps_doc_wikipedia_importance_length_correction": -326.7724609375 }, "fasttext": { "dclm": 0.05795770883560181, "english": 0.9404057264328003, "fineweb_edu_approx": 1.394026756286621, "eai_general_math": 0.044004980474710464, "eai_open_web_math": 0.18217939138412476, "eai_web_code": 0.003693999955430627 } }
{ "free_decimal_correspondence": { "primary": { "code": "005.4", "labels": { "level_1": "General works, books and libraries, information sciences", "level_2": "", "level_3": "Computer programming" } }, "secondary": { "code": "344.730285", "labels": { "level_1": "Social sciences", "level_2": "Law", "level_3": "Martial law" } } }, "bloom_cognitive_process": { "primary": { "code": "5", "label": "Evaluate" }, "secondary": { "code": "4", "label": "Analyze" } }, "bloom_knowledge_domain": { "primary": { "code": "2", "label": "Conceptual" }, "secondary": { "code": "3", "label": "Procedural" } }, "document_type_v1": { "primary": { "code": "9", "label": "Personal/Misc" }, "secondary": { "code": "-1", "label": "Abstain" } }, "extraction_artifacts": { "primary": { "code": "0", "label": "No Artifacts" }, "secondary": { "code": "-1", "label": "Abstain" } }, "missing_content": { "primary": { "code": "0", "label": "No missing content" }, "secondary": { "code": "-1", "label": "Abstain" } }, "document_type_v2": { "primary": { "code": "16", "label": "Personal Blog" }, "secondary": { "code": "10", "label": "Knowledge Article" } }, "reasoning_depth": { "primary": { "code": "3", "label": "Intermediate Reasoning" }, "secondary": { "code": "2", "label": "Basic Reasoning" } }, "technical_correctness": { "primary": { "code": "6", "label": "Not Applicable/Indeterminate" }, "secondary": { "code": "3", "label": "Mostly Correct" } }, "education_level": { "primary": { "code": "2", "label": "High School Level" }, "secondary": { "code": "1", "label": "General Audience" } } }
672f1e42c33a7f9846924a2431ea77df
-8,341,533,009,628,621,000
blob: b6fdacf3778e85040d02d7ea26d0f0cb14804857 [file] [log] [blame] // Copyright (c) 2012 The Chromium Authors. All rights reserved. // Use of this source code is governed by a BSD-style license that can be // found in the LICENSE file. #include "chrome/browser/extensions/extension_tab_util.h" #include <stddef.h> #include <algorithm> #include <utility> #include "base/no_destructor.h" #include "base/stl_util.h" #include "base/strings/string_number_conversions.h" #include "base/strings/stringprintf.h" #include "base/strings/utf_string_conversions.h" #include "chrome/browser/browser_process.h" #include "chrome/browser/extensions/api/tabs/tabs_constants.h" #include "chrome/browser/extensions/browser_extension_window_controller.h" #include "chrome/browser/extensions/chrome_extension_function.h" #include "chrome/browser/extensions/chrome_extension_function_details.h" #include "chrome/browser/extensions/tab_helper.h" #include "chrome/browser/profiles/profile.h" #include "chrome/browser/resource_coordinator/tab_lifecycle_unit_external.h" #include "chrome/browser/sessions/session_tab_helper.h" #include "chrome/browser/ui/browser.h" #include "chrome/browser/ui/browser_finder.h" #include "chrome/browser/ui/browser_navigator_params.h" #include "chrome/browser/ui/browser_window.h" #include "chrome/browser/ui/recently_audible_helper.h" #include "chrome/browser/ui/scoped_tabbed_browser_displayer.h" #include "chrome/browser/ui/singleton_tabs.h" #include "chrome/browser/ui/tab_contents/tab_contents_iterator.h" #include "chrome/browser/ui/tabs/tab_strip_model.h" #include "chrome/browser/ui/tabs/tab_utils.h" #include "chrome/common/extensions/api/tabs.h" #include "chrome/common/url_constants.h" #include "components/url_formatter/url_fixer.h" #include "content/public/browser/favicon_status.h" #include "content/public/browser/navigation_entry.h" #include "content/public/browser/web_contents.h" #include "extensions/common/constants.h" #include "extensions/common/error_utils.h" #include "extensions/common/extension.h" #include "extensions/common/feature_switch.h" #include "extensions/common/manifest_constants.h" #include "extensions/common/manifest_handlers/incognito_info.h" #include "extensions/common/manifest_handlers/options_page_info.h" #include "extensions/common/permissions/api_permission.h" #include "extensions/common/permissions/permissions_data.h" #include "url/gurl.h" #if defined(OS_CHROMEOS) #include "ash/public/cpp/window_pin_type.h" #endif using content::NavigationEntry; using content::WebContents; namespace extensions { namespace { // |error_message| can optionally be passed in and will be set with an // appropriate message if the window cannot be found by id. Browser* GetBrowserInProfileWithId(Profile* profile, const int window_id, bool match_incognito_profile, std::string* error_message) { Profile* incognito_profile = match_incognito_profile && profile->HasOffTheRecordProfile() ? profile->GetOffTheRecordProfile() : nullptr; for (auto* browser : *BrowserList::GetInstance()) { if ((browser->profile() == profile || browser->profile() == incognito_profile) && ExtensionTabUtil::GetWindowId(browser) == window_id && browser->window()) { return browser; } } if (error_message) *error_message = ErrorUtils::FormatErrorMessage( tabs_constants::kWindowNotFoundError, base::NumberToString(window_id)); return nullptr; } Browser* CreateBrowser(Profile* profile, int window_id, bool user_gesture, std::string* error) { Browser::CreateParams params(Browser::TYPE_TABBED, profile, user_gesture); Browser* browser = new Browser(params); browser->window()->Show(); return browser; } // Use this function for reporting a tab id to an extension. It will // take care of setting the id to TAB_ID_NONE if necessary (for // example with devtools). int GetTabIdForExtensions(const WebContents* web_contents) { Browser* browser = chrome::FindBrowserWithWebContents(web_contents); if (browser && !ExtensionTabUtil::BrowserSupportsTabs(browser)) return -1; return SessionTabHelper::IdForTab(web_contents).id(); } std::unique_ptr<ExtensionTabUtil::Delegate>& GetExtensionTabUtilDelegateWrapper() { static base::NoDestructor<std::unique_ptr<ExtensionTabUtil::Delegate>> delegate_wrapper; return *delegate_wrapper; } ExtensionTabUtil::Delegate* GetExtensionTabUtilDelegate() { return GetExtensionTabUtilDelegateWrapper().get(); } } // namespace ExtensionTabUtil::OpenTabParams::OpenTabParams() : create_browser_if_needed(false) { } ExtensionTabUtil::OpenTabParams::~OpenTabParams() { } // Opens a new tab for a given extension. Returns nullptr and sets |error| if an // error occurs. base::DictionaryValue* ExtensionTabUtil::OpenTab( UIThreadExtensionFunction* function, const OpenTabParams& params, bool user_gesture, std::string* error) { ChromeExtensionFunctionDetails chrome_details(function); Profile* profile = chrome_details.GetProfile(); // windowId defaults to "current" window. int window_id = extension_misc::kCurrentWindowId; if (params.window_id.get()) window_id = *params.window_id; Browser* browser = GetBrowserFromWindowID(chrome_details, window_id, error); if (!browser) { if (!params.create_browser_if_needed) { return nullptr; } browser = CreateBrowser(profile, window_id, user_gesture, error); if (!browser) return nullptr; } // Ensure the selected browser is tabbed. if (!browser->is_type_tabbed() && browser->IsAttemptingToCloseBrowser()) browser = chrome::FindTabbedBrowser( profile, function->include_incognito_information()); if (!browser || !browser->window()) { if (error) *error = tabs_constants::kNoCurrentWindowError; return nullptr; } // TODO(jstritar): Add a constant, chrome.tabs.TAB_ID_ACTIVE, that // represents the active tab. WebContents* opener = nullptr; Browser* opener_browser = nullptr; if (params.opener_tab_id.get()) { int opener_id = *params.opener_tab_id; if (!ExtensionTabUtil::GetTabById( opener_id, profile, function->include_incognito_information(), &opener_browser, nullptr, &opener, nullptr)) { if (error) { *error = ErrorUtils::FormatErrorMessage( tabs_constants::kTabNotFoundError, base::NumberToString(opener_id)); } return nullptr; } } // TODO(rafaelw): handle setting remaining tab properties: // -title // -favIconUrl GURL url; if (params.url.get()) { std::string url_string = *params.url; url = ExtensionTabUtil::ResolvePossiblyRelativeURL(url_string, function->extension()); if (!url.is_valid()) { *error = ErrorUtils::FormatErrorMessage(tabs_constants::kInvalidUrlError, url_string); return nullptr; } } else { url = GURL(chrome::kChromeUINewTabURL); } // Don't let extensions crash the browser or renderers. if (ExtensionTabUtil::IsKillURL(url)) { *error = tabs_constants::kNoCrashBrowserError; return nullptr; } // Default to foreground for the new tab. The presence of 'active' property // will override this default. bool active = true; if (params.active.get()) active = *params.active; // Default to not pinning the tab. Setting the 'pinned' property to true // will override this default. bool pinned = false; if (params.pinned.get()) pinned = *params.pinned; // We can't load extension URLs into incognito windows unless the extension // uses split mode. Special case to fall back to a tabbed window. if (url.SchemeIs(kExtensionScheme) && (!function->extension() || !IncognitoInfo::IsSplitMode(function->extension())) && browser->profile()->IsOffTheRecord()) { Profile* profile = browser->profile()->GetOriginalProfile(); browser = chrome::FindTabbedBrowser(profile, false); if (!browser) { Browser::CreateParams params = Browser::CreateParams(Browser::TYPE_TABBED, profile, user_gesture); browser = new Browser(params); browser->window()->Show(); } } if (opener_browser && browser != opener_browser) { if (error) { *error = "Tab opener must be in the same window as the updated tab."; } return nullptr; } // If index is specified, honor the value, but keep it bound to // -1 <= index <= tab_strip->count() where -1 invokes the default behavior. int index = -1; if (params.index.get()) index = *params.index; TabStripModel* tab_strip = browser->tab_strip_model(); index = std::min(std::max(index, -1), tab_strip->count()); int add_types = active ? TabStripModel::ADD_ACTIVE : TabStripModel::ADD_NONE; add_types |= TabStripModel::ADD_FORCE_INDEX; if (pinned) add_types |= TabStripModel::ADD_PINNED; NavigateParams navigate_params(browser, url, ui::PAGE_TRANSITION_LINK); navigate_params.disposition = active ? WindowOpenDisposition::NEW_FOREGROUND_TAB : WindowOpenDisposition::NEW_BACKGROUND_TAB; navigate_params.tabstrip_index = index; navigate_params.tabstrip_add_types = add_types; Navigate(&navigate_params); // The tab may have been created in a different window, so make sure we look // at the right tab strip. tab_strip = navigate_params.browser->tab_strip_model(); int new_index = tab_strip->GetIndexOfWebContents( navigate_params.navigated_or_inserted_contents); if (opener) { // Only set the opener if the opener tab is in the same tab strip as the // new tab. if (tab_strip->GetIndexOfWebContents(opener) != TabStripModel::kNoTab) tab_strip->SetOpenerOfWebContentsAt(new_index, opener); } if (active) navigate_params.navigated_or_inserted_contents->SetInitialFocus(); // Return data about the newly created tab. return ExtensionTabUtil::CreateTabObject( navigate_params.navigated_or_inserted_contents, kScrubTab, function->extension(), tab_strip, new_index) ->ToValue() .release(); } Browser* ExtensionTabUtil::GetBrowserFromWindowID( const ChromeExtensionFunctionDetails& details, int window_id, std::string* error) { if (window_id == extension_misc::kCurrentWindowId) { Browser* result = details.GetCurrentBrowser(); if (!result || !result->window()) { if (error) *error = tabs_constants::kNoCurrentWindowError; return nullptr; } return result; } else { return GetBrowserInProfileWithId( details.GetProfile(), window_id, details.function()->include_incognito_information(), error); } } int ExtensionTabUtil::GetWindowId(const Browser* browser) { return browser->session_id().id(); } int ExtensionTabUtil::GetWindowIdOfTabStripModel( const TabStripModel* tab_strip_model) { for (auto* browser : *BrowserList::GetInstance()) { if (browser->tab_strip_model() == tab_strip_model) return GetWindowId(browser); } return -1; } int ExtensionTabUtil::GetTabId(const WebContents* web_contents) { return SessionTabHelper::IdForTab(web_contents).id(); } std::string ExtensionTabUtil::GetTabStatusText(bool is_loading) { return is_loading ? tabs_constants::kStatusValueLoading : tabs_constants::kStatusValueComplete; } int ExtensionTabUtil::GetWindowIdOfTab(const WebContents* web_contents) { return SessionTabHelper::IdForWindowContainingTab(web_contents).id(); } // static std::string ExtensionTabUtil::GetBrowserWindowTypeText(const Browser& browser) { if (browser.is_devtools()) return tabs_constants::kWindowTypeValueDevTools; if (browser.is_type_popup()) return tabs_constants::kWindowTypeValuePopup; // TODO(devlin): Browser::is_app() returns true whenever Browser::app_name_ // is non-empty (and includes instances such as devtools). Platform apps // should no longer be returned here; are there any other cases (that aren't // captured by is_devtools() or is_type_popup() for an app-type browser? if (browser.is_app()) return tabs_constants::kWindowTypeValueApp; return tabs_constants::kWindowTypeValueNormal; } // static std::unique_ptr<api::tabs::Tab> ExtensionTabUtil::CreateTabObject( WebContents* contents, ScrubTabBehavior scrub_tab_behavior, const Extension* extension, TabStripModel* tab_strip, int tab_index) { if (!tab_strip) ExtensionTabUtil::GetTabStripModel(contents, &tab_strip, &tab_index); bool is_loading = contents->IsLoading(); auto tab_object = std::make_unique<api::tabs::Tab>(); tab_object->id = std::make_unique<int>(GetTabIdForExtensions(contents)); tab_object->index = tab_index; tab_object->window_id = GetWindowIdOfTab(contents); tab_object->status = std::make_unique<std::string>(GetTabStatusText(is_loading)); tab_object->active = tab_strip && tab_index == tab_strip->active_index(); tab_object->selected = tab_strip && tab_index == tab_strip->active_index(); tab_object->highlighted = tab_strip && tab_strip->IsTabSelected(tab_index); tab_object->pinned = tab_strip && tab_strip->IsTabPinned(tab_index); auto* audible_helper = RecentlyAudibleHelper::FromWebContents(contents); bool audible = false; if (audible_helper) { // WebContents in a tab strip have RecentlyAudible helpers. They endow the // tab with a notion of audibility that has a timeout for quiet periods. Use // that if available. audible = audible_helper->WasRecentlyAudible(); } else { // Otherwise use the instantaneous notion of audibility. audible = contents->IsCurrentlyAudible(); } tab_object->audible = std::make_unique<bool>(audible); auto* tab_lifeycle_unit_external = resource_coordinator::TabLifecycleUnitExternal::FromWebContents(contents); tab_object->discarded = tab_lifeycle_unit_external && tab_lifeycle_unit_external->IsDiscarded(); tab_object->auto_discardable = !tab_lifeycle_unit_external || tab_lifeycle_unit_external->IsAutoDiscardable(); tab_object->muted_info = CreateMutedInfo(contents); tab_object->incognito = contents->GetBrowserContext()->IsOffTheRecord(); gfx::Size contents_size = contents->GetContainerBounds().size(); tab_object->width = std::make_unique<int>(contents_size.width()); tab_object->height = std::make_unique<int>(contents_size.height()); tab_object->url = std::make_unique<std::string>(contents->GetURL().spec()); tab_object->title = std::make_unique<std::string>(base::UTF16ToUTF8(contents->GetTitle())); NavigationEntry* entry = contents->GetController().GetVisibleEntry(); if (entry && entry->GetFavicon().valid) { tab_object->fav_icon_url = std::make_unique<std::string>(entry->GetFavicon().url.spec()); } if (tab_strip) { WebContents* opener = tab_strip->GetOpenerOfWebContentsAt(tab_index); if (opener) { tab_object->opener_tab_id = std::make_unique<int>(GetTabIdForExtensions(opener)); } } if (scrub_tab_behavior == kScrubTab) ScrubTabForExtension(extension, contents, tab_object.get()); return tab_object; } std::unique_ptr<base::ListValue> ExtensionTabUtil::CreateTabList( const Browser* browser, const Extension* extension) { std::unique_ptr<base::ListValue> tab_list(new base::ListValue()); TabStripModel* tab_strip = browser->tab_strip_model(); for (int i = 0; i < tab_strip->count(); ++i) { tab_list->Append(CreateTabObject(tab_strip->GetWebContentsAt(i), kScrubTab, extension, tab_strip, i) ->ToValue()); } return tab_list; } // static std::unique_ptr<base::DictionaryValue> ExtensionTabUtil::CreateWindowValueForExtension( const Browser& browser, const Extension* extension, PopulateTabBehavior populate_tab_behavior) { auto result = std::make_unique<base::DictionaryValue>(); result->SetInteger(tabs_constants::kIdKey, browser.session_id().id()); result->SetString(tabs_constants::kWindowTypeKey, GetBrowserWindowTypeText(browser)); ui::BaseWindow* window = browser.window(); result->SetBoolean(tabs_constants::kFocusedKey, window->IsActive()); const Profile* profile = browser.profile(); result->SetBoolean(tabs_constants::kIncognitoKey, profile->IsOffTheRecord()); result->SetBoolean(tabs_constants::kAlwaysOnTopKey, window->IsAlwaysOnTop()); std::string window_state; if (window->IsMinimized()) { window_state = tabs_constants::kShowStateValueMinimized; } else if (window->IsFullscreen()) { window_state = tabs_constants::kShowStateValueFullscreen; #if defined(OS_CHROMEOS) if (ash::IsWindowTrustedPinned(window)) window_state = tabs_constants::kShowStateValueLockedFullscreen; #endif } else if (window->IsMaximized()) { window_state = tabs_constants::kShowStateValueMaximized; } else { window_state = tabs_constants::kShowStateValueNormal; } result->SetString(tabs_constants::kShowStateKey, window_state); gfx::Rect bounds; if (window->IsMinimized()) bounds = window->GetRestoredBounds(); else bounds = window->GetBounds(); result->SetInteger(tabs_constants::kLeftKey, bounds.x()); result->SetInteger(tabs_constants::kTopKey, bounds.y()); result->SetInteger(tabs_constants::kWidthKey, bounds.width()); result->SetInteger(tabs_constants::kHeightKey, bounds.height()); if (populate_tab_behavior == kPopulateTabs) result->Set(tabs_constants::kTabsKey, CreateTabList(&browser, extension)); return result; } // static std::unique_ptr<api::tabs::MutedInfo> ExtensionTabUtil::CreateMutedInfo( content::WebContents* contents) { DCHECK(contents); auto info = std::make_unique<api::tabs::MutedInfo>(); info->muted = contents->IsAudioMuted(); switch (chrome::GetTabAudioMutedReason(contents)) { case TabMutedReason::NONE: break; case TabMutedReason::CONTENT_SETTING: case TabMutedReason::CONTENT_SETTING_CHROME: case TabMutedReason::CONTEXT_MENU: info->reason = api::tabs::MUTED_INFO_REASON_USER; break; case TabMutedReason::MEDIA_CAPTURE: info->reason = api::tabs::MUTED_INFO_REASON_CAPTURE; break; case TabMutedReason::EXTENSION: info->reason = api::tabs::MUTED_INFO_REASON_EXTENSION; info->extension_id = std::make_unique<std::string>( LastMuteMetadata::FromWebContents(contents)->extension_id); DCHECK(!info->extension_id->empty()); break; } return info; } // static void ExtensionTabUtil::SetPlatformDelegate(std::unique_ptr<Delegate> delegate) { GetExtensionTabUtilDelegateWrapper() = std::move(delegate); } // static void ExtensionTabUtil::ScrubTabForExtension(const Extension* extension, content::WebContents* contents, api::tabs::Tab* tab) { bool has_permission = false; if (extension) { bool api_permission = false; std::string url; if (contents) { api_permission = extension->permissions_data()->HasAPIPermissionForTab( GetTabId(contents), APIPermission::kTab); url = contents->GetURL().spec(); } else { api_permission = extension->permissions_data()->HasAPIPermission(APIPermission::kTab); url = *tab->url; } bool host_permission = extension->permissions_data() ->active_permissions() .HasExplicitAccessToOrigin(GURL(url)); has_permission = api_permission || host_permission; } if (!has_permission) { tab->url.reset(); tab->title.reset(); tab->fav_icon_url.reset(); } if (GetExtensionTabUtilDelegate()) { GetExtensionTabUtilDelegate()->ScrubTabForExtension(extension, contents, tab); } } bool ExtensionTabUtil::GetTabStripModel(const WebContents* web_contents, TabStripModel** tab_strip_model, int* tab_index) { DCHECK(web_contents); DCHECK(tab_strip_model); DCHECK(tab_index); for (auto* browser : *BrowserList::GetInstance()) { TabStripModel* tab_strip = browser->tab_strip_model(); int index = tab_strip->GetIndexOfWebContents(web_contents); if (index != -1) { *tab_strip_model = tab_strip; *tab_index = index; return true; } } return false; } bool ExtensionTabUtil::GetDefaultTab(Browser* browser, WebContents** contents, int* tab_id) { DCHECK(browser); DCHECK(contents); *contents = browser->tab_strip_model()->GetActiveWebContents(); if (*contents) { if (tab_id) *tab_id = GetTabId(*contents); return true; } return false; } bool ExtensionTabUtil::GetTabById(int tab_id, content::BrowserContext* browser_context, bool include_incognito, Browser** browser, TabStripModel** tab_strip, WebContents** contents, int* tab_index) { if (tab_id == api::tabs::TAB_ID_NONE) return false; Profile* profile = Profile::FromBrowserContext(browser_context); Profile* incognito_profile = include_incognito && profile->HasOffTheRecordProfile() ? profile->GetOffTheRecordProfile() : nullptr; for (auto* target_browser : *BrowserList::GetInstance()) { if (target_browser->profile() == profile || target_browser->profile() == incognito_profile) { TabStripModel* target_tab_strip = target_browser->tab_strip_model(); for (int i = 0; i < target_tab_strip->count(); ++i) { WebContents* target_contents = target_tab_strip->GetWebContentsAt(i); if (SessionTabHelper::IdForTab(target_contents).id() == tab_id) { if (browser) *browser = target_browser; if (tab_strip) *tab_strip = target_tab_strip; if (contents) *contents = target_contents; if (tab_index) *tab_index = i; return true; } } } } return false; } GURL ExtensionTabUtil::ResolvePossiblyRelativeURL(const std::string& url_string, const Extension* extension) { GURL url = GURL(url_string); if (!url.is_valid()) url = extension->GetResourceURL(url_string); return url; } bool ExtensionTabUtil::IsKillURL(const GURL& url) { static const char* const kill_hosts[] = { chrome::kChromeUICrashHost, chrome::kChromeUIDelayedHangUIHost, chrome::kChromeUIHangUIHost, chrome::kChromeUIKillHost, chrome::kChromeUIQuitHost, chrome::kChromeUIRestartHost, content::kChromeUIBrowserCrashHost, content::kChromeUIMemoryExhaustHost, }; // Check a fixed-up URL, to normalize the scheme and parse hosts correctly. GURL fixed_url = url_formatter::FixupURL(url.possibly_invalid_spec(), std::string()); if (!fixed_url.SchemeIs(content::kChromeUIScheme)) return false; base::StringPiece fixed_host = fixed_url.host_piece(); for (size_t i = 0; i < base::size(kill_hosts); ++i) { if (fixed_host == kill_hosts[i]) return true; } return false; } void ExtensionTabUtil::CreateTab(std::unique_ptr<WebContents> web_contents, const std::string& extension_id, WindowOpenDisposition disposition, const gfx::Rect& initial_rect, bool user_gesture) { Profile* profile = Profile::FromBrowserContext(web_contents->GetBrowserContext()); Browser* browser = chrome::FindTabbedBrowser(profile, false); const bool browser_created = !browser; if (!browser) { Browser::CreateParams params = Browser::CreateParams(profile, user_gesture); browser = new Browser(params); } NavigateParams params(browser, std::move(web_contents)); // The extension_app_id parameter ends up as app_name in the Browser // which causes the Browser to return true for is_app(). This affects // among other things, whether the location bar gets displayed. // TODO(mpcomplete): This seems wrong. What if the extension content is hosted // in a tab? if (disposition == WindowOpenDisposition::NEW_POPUP) params.extension_app_id = extension_id; params.disposition = disposition; params.window_bounds = initial_rect; params.window_action = NavigateParams::SHOW_WINDOW; params.user_gesture = user_gesture; Navigate(&params); // Close the browser if Navigate created a new one. if (browser_created && (browser != params.browser)) browser->window()->Close(); } // static void ExtensionTabUtil::ForEachTab( const base::Callback<void(WebContents*)>& callback) { for (auto* web_contents : AllTabContentses()) callback.Run(web_contents); } // static WindowController* ExtensionTabUtil::GetWindowControllerOfTab( const WebContents* web_contents) { Browser* browser = chrome::FindBrowserWithWebContents(web_contents); if (browser != nullptr) return browser->extension_window_controller(); return nullptr; } bool ExtensionTabUtil::OpenOptionsPageFromAPI( const Extension* extension, content::BrowserContext* browser_context) { if (!OptionsPageInfo::HasOptionsPage(extension)) return false; Profile* profile = Profile::FromBrowserContext(browser_context); // This version of OpenOptionsPage() is only called when the extension // initiated the command via chrome.runtime.openOptionsPage. For a spanning // mode extension, this API could only be called from a regular profile, since // that's the only place it's running. DCHECK(!profile->IsOffTheRecord() || IncognitoInfo::IsSplitMode(extension)); Browser* browser = chrome::FindBrowserWithProfile(profile); if (!browser) browser = new Browser(Browser::CreateParams(profile, true)); return extensions::ExtensionTabUtil::OpenOptionsPage(extension, browser); } bool ExtensionTabUtil::OpenOptionsPage(const Extension* extension, Browser* browser) { if (!OptionsPageInfo::HasOptionsPage(extension)) return false; // Force the options page to open in non-OTR window if the extension is not // running in split mode, because it won't be able to save settings from OTR. // This version of OpenOptionsPage() can be called from an OTR window via e.g. // the action menu, since that's not initiated by the extension. std::unique_ptr<chrome::ScopedTabbedBrowserDisplayer> displayer; if (browser->profile()->IsOffTheRecord() && !IncognitoInfo::IsSplitMode(extension)) { displayer.reset(new chrome::ScopedTabbedBrowserDisplayer( browser->profile()->GetOriginalProfile())); browser = displayer->browser(); } GURL url_to_navigate; bool open_in_tab = OptionsPageInfo::ShouldOpenInTab(extension); if (open_in_tab) { // Options page tab is simply e.g. chrome-extension://.../options.html. url_to_navigate = OptionsPageInfo::GetOptionsPage(extension); } else { // Options page tab is Extension settings pointed at that Extension's ID, // e.g. chrome://extensions?options=... url_to_navigate = GURL(chrome::kChromeUIExtensionsURL); GURL::Replacements replacements; std::string query = base::StringPrintf("options=%s", extension->id().c_str()); replacements.SetQueryStr(query); url_to_navigate = url_to_navigate.ReplaceComponents(replacements); } NavigateParams params( GetSingletonTabNavigateParams(browser, url_to_navigate)); // We need to respect path differences because we don't want opening the // options page to close a page that might be open to extension content. // However, if the options page opens inside the chrome://extensions page, we // can override an existing page. // Note: ref behavior is to ignore. params.path_behavior = open_in_tab ? NavigateParams::RESPECT : NavigateParams::IGNORE_AND_NAVIGATE; params.url = url_to_navigate; ShowSingletonTabOverwritingNTP(browser, std::move(params)); return true; } // static bool ExtensionTabUtil::BrowserSupportsTabs(Browser* browser) { return browser && !browser->is_devtools(); } } // namespace extensions
{ "url": "https://chromium.googlesource.com/chromium/src/+/b253c36b668b630578a389267e4579659083881d/chrome/browser/extensions/extension_tab_util.cc", "source_domain": "chromium.googlesource.com", "snapshot_id": "crawl=CC-MAIN-2020-16", "warc_metadata": { "Content-Length": "314783", "Content-Type": "application/http; msgtype=response", "WARC-Block-Digest": "sha1:KJPJOODMT22NBIOKDX2WXQROUZ3NSSDZ", "WARC-Concurrent-To": "<urn:uuid:b5030744-541d-441b-a8da-c198e5971ab8>", "WARC-Date": "2020-04-09T15:28:48Z", "WARC-IP-Address": "173.194.175.82", "WARC-Identified-Payload-Type": "text/html", "WARC-Payload-Digest": "sha1:3BE3ZEAC3GQHZNDZ5TX5AAZJGPONC7DA", "WARC-Record-ID": "<urn:uuid:29381423-4ac3-4b54-80ce-4bca71f0b7bd>", "WARC-Target-URI": "https://chromium.googlesource.com/chromium/src/+/b253c36b668b630578a389267e4579659083881d/chrome/browser/extensions/extension_tab_util.cc", "WARC-Truncated": null, "WARC-Type": "response", "WARC-Warcinfo-ID": "<urn:uuid:5bd84ef4-fc4a-403a-8a70-b644fb101575>" }, "warc_info": "isPartOf: CC-MAIN-2020-16\r\npublisher: Common Crawl\r\ndescription: Wide crawl of the web for March/April 2020\r\noperator: Common Crawl Admin ([email protected])\r\nhostname: ip-10-67-67-177.ec2.internal\r\nsoftware: Apache Nutch 1.16 (modified, https://github.com/commoncrawl/nutch/)\r\nrobots: checked via crawler-commons 1.1-SNAPSHOT (https://github.com/crawler-commons/crawler-commons)\r\nformat: WARC File Format 1.1\r\nconformsTo: http://iipc.github.io/warc-specifications/specifications/warc-format/warc-1.1/" }
{ "line_start_idx": [ 0, 68, 133, 207, 237, 295, 315, 336, 355, 387, 414, 466, 505, 554, 598, 661, 736, 801, 874, 924, 969, 1046, 1102, 1141, 1187, 1243, 1289, 1344, 1407, 1453, 1519, 1571, 1617, 1664, 1705, 1753, 1804, 1857, 1906, 1947, 1990, 2031, 2077, 2127, 2191, 2258, 2316, 2376, 2398, 2423, 2467, 2474, 2506, 2534, 2557, 2569, 2640, 2700, 2753, 2774, 2804, 2834, 2863, 2924, 2960, 2971, 3023, 3061, 3105, 3160, 3181, 3197, 3199, 3201, 3220, 3269, 3341, 3357, 3359, 3400, 3415, 3434, 3456, 3531, 3571, 3598, 3614, 3616, 3685, 3749, 3776, 3837, 3906, 3970, 3981, 4035, 4037, 4082, 4121, 4192, 4210, 4236, 4238, 4298, 4349, 4351, 4366, 4415, 4451, 4453, 4505, 4507, 4588, 4605, 4655, 4692, 4721, 4740, 4762, 4819, 4867, 4909, 4959, 4987, 5018, 5095, 5111, 5151, 5167, 5169, 5235, 5249, 5265, 5267, 5309, 5382, 5419, 5472, 5510, 5521, 5569, 5585, 5587, 5654, 5684, 5715, 5750, 5784, 5823, 5858, 5921, 5968, 5981, 6022, 6091, 6093, 6109, 6111, 6113, 6172, 6182, 6197, 6207, 6231, 6269, 6332, 6356, 6379, 6453, 6466, 6482, 6484, 6493, 6533, 6535, 6591, 6631, 6678, 6694, 6696, 6772, 6803, 6823, 6848, 6873, 6946, 6977, 6998, 7023, 7048, 7124, 7190, 7228, 7255, 7310, 7350, 7411, 7464, 7480, 7511, 7579, 7610, 7637, 7639, 7641, 7692, 7705, 7775, 7777, 7793, 7795, 7859, 7935, 7951, 7975, 7998, 8053, 8112, 8190, 8235, 8247, 8287, 8359, 8396, 8440, 8485, 8525, 8573, 8601, 8678, 8705, 8761, 8811, 8860, 8874, 8947, 8959, 9030, 9086, 9088, 9100, 9167, 9211, 9253, 9312, 9357, 9369, 9381, 9383, 9434, 9481, 9496, 9518, 9571, 9618, 9654, 9665, 9713, 9729, 9731, 9746, 9755, 9789, 9822, 9883, 9885, 9887, 9947, 9982, 9984, 10034, 10074, 10126, 10177, 10206, 10208, 10219, 10221, 10287, 10341, 10343, 10409, 10465, 10505, 10507, 10581, 10651, 10653, 10663, 10744, 10771, 10820, 10849, 10895, 10971, 11044, 11121, 11194, 11216, 11260, 11307, 11309, 11319, 11386, 11409, 11446, 11474, 11500, 11517, 11533, 11603, 11644, 11698, 11771, 11802, 11854, 11875, 11936, 12010, 12086, 12162, 12231, 12304, 12326, 12348, 12423, 12500, 12522, 12570, 12579, 12636, 12678, 12680, 12735, 12770, 12845, 12869, 12942, 12973, 13004, 13053, 13105, 13178, 13243, 13309, 13377, 13453, 13473, 13545, 13615, 13657, 13684, 13747, 13749, 13766, 13836, 13850, 13878, 13932, 13934, 13936, 13973, 14034, 14053, 14055, 14121, 14145, 14175, 14241, 14296, 14343, 14419, 14444, 14458, 14460, 14477, 14479, 14489, 14528, 14577, 14601, 14629, 14674, 14731, 14802, 14852, 14888, 14931, 15000, 15044, 15122, 15200, 15226, 15255, 15312, 15349, 15407, 15432, 15472, 15536, 15543, 15579, 15636, 15645, 15699, 15701, 15765, 15783, 15810, 15848, 15853, 15883, 15941, 15998, 16061, 16126, 16170, 16245, 16260, 16262, 16272, 16345, 16379, 16397, 16451, 16491, 16543, 16570, 16577, 16615, 16660, 16695, 16745, 16752, 16788, 16841, 16848, 16880, 16935, 16987, 17047, 17085, 17092, 17094, 17107, 17109, 17119, 17200, 17260, 17262, 17272, 17344, 17376, 17399, 17428, 17445, 17474, 17491, 17507, 17579, 17621, 17654, 17663, 17680, 17750, 17767, 17769, 17822, 17845, 17884, 17936, 17938, 17961, 17979, 17999, 18026, 18028, 18065, 18138, 18144, 18146, 18148, 18221, 18254, 18272, 18294, 18319, 18338, 18390, 18445, 18505, 18524, 18554, 18574, 18587, 18589, 18591, 18605, 18607, 18662, 18686, 18701, 18718, 18736, 18800, 18817, 18829, 18860, 18873, 18875, 18889, 18891, 18937, 18979, 19003, 19022, 19049, 19073, 19091, 19129, 19143, 19208, 19237, 19292, 19328, 19339, 19398, 19442, 19492, 19561, 19615, 19685, 19751, 19764, 19791, 19806, 19837, 19851, 19880, 19895, 19911, 19924, 19926, 19928, 19930, 19932, 19946, 19948, 20029, 20059, 20088, 20109, 20154, 20166, 20168, 20220, 20262, 20326, 20382, 20439, 20512, 20515, 20591, 20608, 20677, 20728, 20742, 20797, 20851, 20884, 20897, 20899, 20913, 20915, 20991, 21024, 21059, 21090, 21111, 21130, 21194, 21256, 21295, 21311, 21388, 21419, 21421, 21478, 21547, 21617, 21681, 21760, 21773, 21826, 21866, 21900, 21937, 21989, 22025, 22044, 22096, 22148, 22176, 22178, 22188, 22223, 22277, 22323, 22351, 22353, 22363, 22425, 22460, 22529, 22553, 22600, 22616, 22618, 22665, 22693, 22737, 22786, 22800, 22865, 22936, 23012, 23091, 23130, 23207, 23267, 23281, 23342, 23416, 23418, 23485, 23505, 23554, 23568, 23644, 23722, 23801, 23866, 23931, 23975, 24017, 24075, 24119, 24151, 24153, 24175, 24239, 24258, 24330, 24392, 24401, 24475, 24515, 24571, 24604, 24624, 24683, 24716, 24783, 24785, 24808, 24866, 24939, 25012, 25090, 25124, 25160, 25221, 25260, 25290, 25350, 25363, 25365, 25375, 25438, 25481, 25483 ], "line_end_idx": [ 68, 133, 207, 237, 295, 315, 336, 355, 387, 414, 466, 505, 554, 598, 661, 736, 801, 874, 924, 969, 1046, 1102, 1141, 1187, 1243, 1289, 1344, 1407, 1453, 1519, 1571, 1617, 1664, 1705, 1753, 1804, 1857, 1906, 1947, 1990, 2031, 2077, 2127, 2191, 2258, 2316, 2376, 2398, 2423, 2467, 2474, 2506, 2534, 2557, 2569, 2640, 2700, 2753, 2774, 2804, 2834, 2863, 2924, 2960, 2971, 3023, 3061, 3105, 3160, 3181, 3197, 3199, 3201, 3220, 3269, 3341, 3357, 3359, 3400, 3415, 3434, 3456, 3531, 3571, 3598, 3614, 3616, 3685, 3749, 3776, 3837, 3906, 3970, 3981, 4035, 4037, 4082, 4121, 4192, 4210, 4236, 4238, 4298, 4349, 4351, 4366, 4415, 4451, 4453, 4505, 4507, 4588, 4605, 4655, 4692, 4721, 4740, 4762, 4819, 4867, 4909, 4959, 4987, 5018, 5095, 5111, 5151, 5167, 5169, 5235, 5249, 5265, 5267, 5309, 5382, 5419, 5472, 5510, 5521, 5569, 5585, 5587, 5654, 5684, 5715, 5750, 5784, 5823, 5858, 5921, 5968, 5981, 6022, 6091, 6093, 6109, 6111, 6113, 6172, 6182, 6197, 6207, 6231, 6269, 6332, 6356, 6379, 6453, 6466, 6482, 6484, 6493, 6533, 6535, 6591, 6631, 6678, 6694, 6696, 6772, 6803, 6823, 6848, 6873, 6946, 6977, 6998, 7023, 7048, 7124, 7190, 7228, 7255, 7310, 7350, 7411, 7464, 7480, 7511, 7579, 7610, 7637, 7639, 7641, 7692, 7705, 7775, 7777, 7793, 7795, 7859, 7935, 7951, 7975, 7998, 8053, 8112, 8190, 8235, 8247, 8287, 8359, 8396, 8440, 8485, 8525, 8573, 8601, 8678, 8705, 8761, 8811, 8860, 8874, 8947, 8959, 9030, 9086, 9088, 9100, 9167, 9211, 9253, 9312, 9357, 9369, 9381, 9383, 9434, 9481, 9496, 9518, 9571, 9618, 9654, 9665, 9713, 9729, 9731, 9746, 9755, 9789, 9822, 9883, 9885, 9887, 9947, 9982, 9984, 10034, 10074, 10126, 10177, 10206, 10208, 10219, 10221, 10287, 10341, 10343, 10409, 10465, 10505, 10507, 10581, 10651, 10653, 10663, 10744, 10771, 10820, 10849, 10895, 10971, 11044, 11121, 11194, 11216, 11260, 11307, 11309, 11319, 11386, 11409, 11446, 11474, 11500, 11517, 11533, 11603, 11644, 11698, 11771, 11802, 11854, 11875, 11936, 12010, 12086, 12162, 12231, 12304, 12326, 12348, 12423, 12500, 12522, 12570, 12579, 12636, 12678, 12680, 12735, 12770, 12845, 12869, 12942, 12973, 13004, 13053, 13105, 13178, 13243, 13309, 13377, 13453, 13473, 13545, 13615, 13657, 13684, 13747, 13749, 13766, 13836, 13850, 13878, 13932, 13934, 13936, 13973, 14034, 14053, 14055, 14121, 14145, 14175, 14241, 14296, 14343, 14419, 14444, 14458, 14460, 14477, 14479, 14489, 14528, 14577, 14601, 14629, 14674, 14731, 14802, 14852, 14888, 14931, 15000, 15044, 15122, 15200, 15226, 15255, 15312, 15349, 15407, 15432, 15472, 15536, 15543, 15579, 15636, 15645, 15699, 15701, 15765, 15783, 15810, 15848, 15853, 15883, 15941, 15998, 16061, 16126, 16170, 16245, 16260, 16262, 16272, 16345, 16379, 16397, 16451, 16491, 16543, 16570, 16577, 16615, 16660, 16695, 16745, 16752, 16788, 16841, 16848, 16880, 16935, 16987, 17047, 17085, 17092, 17094, 17107, 17109, 17119, 17200, 17260, 17262, 17272, 17344, 17376, 17399, 17428, 17445, 17474, 17491, 17507, 17579, 17621, 17654, 17663, 17680, 17750, 17767, 17769, 17822, 17845, 17884, 17936, 17938, 17961, 17979, 17999, 18026, 18028, 18065, 18138, 18144, 18146, 18148, 18221, 18254, 18272, 18294, 18319, 18338, 18390, 18445, 18505, 18524, 18554, 18574, 18587, 18589, 18591, 18605, 18607, 18662, 18686, 18701, 18718, 18736, 18800, 18817, 18829, 18860, 18873, 18875, 18889, 18891, 18937, 18979, 19003, 19022, 19049, 19073, 19091, 19129, 19143, 19208, 19237, 19292, 19328, 19339, 19398, 19442, 19492, 19561, 19615, 19685, 19751, 19764, 19791, 19806, 19837, 19851, 19880, 19895, 19911, 19924, 19926, 19928, 19930, 19932, 19946, 19948, 20029, 20059, 20088, 20109, 20154, 20166, 20168, 20220, 20262, 20326, 20382, 20439, 20512, 20515, 20591, 20608, 20677, 20728, 20742, 20797, 20851, 20884, 20897, 20899, 20913, 20915, 20991, 21024, 21059, 21090, 21111, 21130, 21194, 21256, 21295, 21311, 21388, 21419, 21421, 21478, 21547, 21617, 21681, 21760, 21773, 21826, 21866, 21900, 21937, 21989, 22025, 22044, 22096, 22148, 22176, 22178, 22188, 22223, 22277, 22323, 22351, 22353, 22363, 22425, 22460, 22529, 22553, 22600, 22616, 22618, 22665, 22693, 22737, 22786, 22800, 22865, 22936, 23012, 23091, 23130, 23207, 23267, 23281, 23342, 23416, 23418, 23485, 23505, 23554, 23568, 23644, 23722, 23801, 23866, 23931, 23975, 24017, 24075, 24119, 24151, 24153, 24175, 24239, 24258, 24330, 24392, 24401, 24475, 24515, 24571, 24604, 24624, 24683, 24716, 24783, 24785, 24808, 24866, 24939, 25012, 25090, 25124, 25160, 25221, 25260, 25290, 25350, 25363, 25365, 25375, 25438, 25481, 25483, 25508 ] }
{ "red_pajama_v2": { "ccnet_original_length": 25508, "ccnet_original_nlines": 680, "rps_doc_curly_bracket": 0.006586169824004173, "rps_doc_ldnoobw_words": 0, "rps_doc_lorem_ipsum": 0, "rps_doc_stop_word_fraction": 0.09090909361839294, "rps_doc_ut1_blacklist": 0, "rps_doc_frac_all_caps_words": 0.012010769918560982, "rps_doc_frac_lines_end_with_ellipsis": 0.0014684300404042006, "rps_doc_frac_no_alph_words": 0.472354531288147, "rps_doc_frac_unique_words": 0.4159495234489441, "rps_doc_mean_word_length": 11.2036714553833, "rps_doc_num_sentences": 221, "rps_doc_symbol_to_word_ratio": 0.010561189614236355, "rps_doc_unigram_entropy": 5.762380599975586, "rps_doc_word_count": 1743, "rps_doc_frac_chars_dupe_10grams": 0, "rps_doc_frac_chars_dupe_5grams": 0.043066371232271194, "rps_doc_frac_chars_dupe_6grams": 0.019356820732355118, "rps_doc_frac_chars_dupe_7grams": 0.00901270005851984, "rps_doc_frac_chars_dupe_8grams": 0, "rps_doc_frac_chars_dupe_9grams": 0, "rps_doc_frac_chars_top_2gram": 0.010036869905889034, "rps_doc_frac_chars_top_3gram": 0.005888979882001877, "rps_doc_frac_chars_top_4gram": 0.004301520064473152, "rps_doc_books_importance": -2281.036865234375, "rps_doc_books_importance_length_correction": -2281.036865234375, "rps_doc_openwebtext_importance": -1248.2027587890625, "rps_doc_openwebtext_importance_length_correction": -1248.2027587890625, "rps_doc_wikipedia_importance": -1047.991455078125, "rps_doc_wikipedia_importance_length_correction": -1047.991455078125 }, "fasttext": { "dclm": 0.959517240524292, "english": 0.20836053788661957, "fineweb_edu_approx": 2.9430432319641113, "eai_general_math": 0.970403790473938, "eai_open_web_math": 0.10218536853790283, "eai_web_code": 0.8153948187828064 } }
{ "free_decimal_correspondence": { "primary": { "code": "005.4", "labels": { "level_1": "General works, books and libraries, information sciences", "level_2": "", "level_3": "Computer programming" } }, "secondary": { "code": "004.67", "labels": { "level_1": "General works, books and libraries, information sciences", "level_2": "", "level_3": "Computers and Computer science" } } }, "bloom_cognitive_process": { "primary": { "code": "3", "label": "Apply" }, "secondary": { "code": "4", "label": "Analyze" } }, "bloom_knowledge_domain": { "primary": { "code": "3", "label": "Procedural" }, "secondary": { "code": "2", "label": "Conceptual" } }, "document_type_v1": { "primary": { "code": "4", "label": "Code/Software" }, "secondary": { "code": "-1", "label": "Abstain" } }, "extraction_artifacts": { "primary": { "code": "0", "label": "No Artifacts" }, "secondary": { "code": "0", "label": "No Artifacts" } }, "missing_content": { "primary": { "code": "0", "label": "No missing content" }, "secondary": { "code": "0", "label": "No missing content" } }, "document_type_v2": { "primary": { "code": "8", "label": "Documentation" }, "secondary": { "code": "3", "label": "Academic Writing" } }, "reasoning_depth": { "primary": { "code": "4", "label": "Advanced Reasoning" }, "secondary": { "code": "3", "label": "Intermediate Reasoning" } }, "technical_correctness": { "primary": { "code": "4", "label": "Highly Correct" }, "secondary": { "code": "5", "label": "Exceptionally Correct" } }, "education_level": { "primary": { "code": "4", "label": "Graduate/Expert Level" }, "secondary": { "code": "3", "label": "Undergraduate Level" } } }
672f1e42c33a7f9846924a2431ea77df
4,689,137,336,218,027,000
Main Content Pick and Place Using RRT for Manipulators Using manipulators to pick and place objects in an environment may require path planning algorithms like the rapidly-exploring random tree planner. The planner explores in the joint-configuration space and searches for a collision-free path between different robot configurations. This example shows how to use the manipulatorRRT object to tune the planner parameters and plan a path between two joint configurations based on a rigidBodyTree robot model of the Franka Emika™ Panda robot. After tuning the planner parameters, the robot manipulator plans a path to move a can from one place to another. Load Robot Model and Environment Load the robot and its environment using the exampleHelperLoadPickAndPlaceRRT function. The function outputs three variables: • franka A Franka Emika Panda robot model as a rigidBodyTree object. The model has been modified to remove some adjacent collision meshes that are always in collision and adjust position limits based on feasibility. • config An initial configuration of joint positions for the robot. • env A set collision objects as a cell array that represent the robot's environment. The path planner checks for self-collisions and collisions with this environment. [franka,config,env] = exampleHelperLoadPickAndPlaceRRT; Visualize the robot model's collision meshes and the environment objects. figure("Name","Pick and Place Using RRT",... "Units","normalized",... "OuterPosition",[0, 0, 1, 1],... "Visible","on"); show(franka,config,"Visuals","off","Collisions","on"); hold on for i = 1:length(env) show(env{i}); end {"String":"Figure Pick and Place Using RRT contains an axes object. The axes object contains 40 objects of type patch, line. These objects represent panda_link0, panda_link1, panda_link2, panda_link3, panda_link4, panda_link5, panda_link6, panda_link7, panda_link8, panda_hand, panda_leftfinger, panda_rightfinger, panda_link1_mesh, panda_link2_mesh, panda_link3_mesh, panda_link4_mesh, panda_link5_mesh, panda_link6_mesh, panda_link7_mesh, panda_hand_mesh, panda_leftfinger_mesh, panda_rightfinger_mesh, panda_link0_mesh, panda_link1_coll_mesh, panda_link2_coll_mesh, panda_link3_coll_mesh, panda_link4_coll_mesh, panda_link5_coll_mesh, panda_link6_coll_mesh, panda_link7_coll_mesh, panda_hand_coll_mesh, panda_leftfinger_coll_mesh, panda_rightfinger_coll_mesh, panda_link0_coll_mesh.","Tex":[],"LaTex":[]} Planner Create the RRT path planner and specify the robot model and the environment. Define some parameters, which are later tuned, and specify the start and goal configuration for the robot. planner = manipulatorRRT(franka, env); planner.SkippedSelfCollisions='parent'; planner.MaxConnectionDistance = 0.3; planner.ValidationDistance = 0.1; startConfig = config; goalConfig = [0.2371 -0.0200 0.0542 -2.2272 0.0013 2.2072 -0.9670 0.0400 0.0400]; Plan the path between configurations. The RRT planner should generate a rapidly-exploring tree of random configurations to explore the space and eventually returns a collision-free path through the environment. Before planning, reset the MATLAB's random number generator for repeatabile results. rng('default'); path = plan(planner,startConfig,goalConfig); To visualize the entire path, interpolate the path into small steps. By default, the interpolate function generates all of the configurations that were checked for feasibility (collision checking) based on the ValidationDistance property. interpStates = interpolate(planner, path); for i = 1:2:size(interpStates,1) show(franka, interpStates(i,:),... "PreservePlot", false,... "Visuals","off",... "Collisions","on"); title("Plan 1: MaxConnectionDistance = 0.3") drawnow; end {"String":"Figure Pick and Place Using RRT contains an axes object. The axes object with title Plan 1: MaxConnectionDistance = 0.3 contains 40 objects of type patch, line. These objects represent panda_link0, panda_link1, panda_link2, panda_link3, panda_link4, panda_link5, panda_link6, panda_link7, panda_link8, panda_hand, panda_leftfinger, panda_rightfinger, panda_link1_mesh, panda_link2_mesh, panda_link3_mesh, panda_link4_mesh, panda_link5_mesh, panda_link6_mesh, panda_link7_mesh, panda_hand_mesh, panda_leftfinger_mesh, panda_rightfinger_mesh, panda_link0_mesh, panda_link1_coll_mesh, panda_link2_coll_mesh, panda_link3_coll_mesh, panda_link4_coll_mesh, panda_link5_coll_mesh, panda_link6_coll_mesh, panda_link7_coll_mesh, panda_hand_coll_mesh, panda_leftfinger_coll_mesh, panda_rightfinger_coll_mesh, panda_link0_coll_mesh.","Tex":"Plan 1: MaxConnectionDistance = 0.3","LaTex":[]} Tuning the Planner Tune the path planner by modifying the MaxConnectionDistance, ValidationDistance, EnableConnectHeuristic properties on the planner object. Setting the MaxConnectionDistance property to a larger value causes longer motions in the planned path, but also enables the planner to greedily explore the space. Use tic and toc functions to time the plan function for reference on how these parameters can affect the execution time. planner.MaxConnectionDistance = 5; tic path = plan(planner,startConfig,goalConfig); toc Elapsed time is 6.629093 seconds. Notice the change in the path. The robot arm swings much higher due to the larger connection distance. interpStates = interpolate(planner, path); for i = 1:2:size(interpStates, 1) show(franka,interpStates(i,:),... "PreservePlot",false,... "Visuals","off",... "Collisions","on"); title("Plan 2: MaxConnectionDistance = 5") drawnow; end {"String":"Figure Pick and Place Using RRT contains an axes object. The axes object with title Plan 2: MaxConnectionDistance = 5 contains 40 objects of type patch, line. These objects represent panda_link0, panda_link1, panda_link2, panda_link3, panda_link4, panda_link5, panda_link6, panda_link7, panda_link8, panda_hand, panda_leftfinger, panda_rightfinger, panda_link1_mesh, panda_link2_mesh, panda_link3_mesh, panda_link4_mesh, panda_link5_mesh, panda_link6_mesh, panda_link7_mesh, panda_hand_mesh, panda_leftfinger_mesh, panda_rightfinger_mesh, panda_link0_mesh, panda_link1_coll_mesh, panda_link2_coll_mesh, panda_link3_coll_mesh, panda_link4_coll_mesh, panda_link5_coll_mesh, panda_link6_coll_mesh, panda_link7_coll_mesh, panda_hand_coll_mesh, panda_leftfinger_coll_mesh, panda_rightfinger_coll_mesh, panda_link0_coll_mesh.","Tex":"Plan 2: MaxConnectionDistance = 5","LaTex":[]} Setting the ValidationDistance to a smaller value enables finer validation of the motion along an edge in the planned path. Increasing the number of configurations to be validated along a path leads to longer planning times. A smaller value is useful in case of a cluttered environment with a lot of collision objects. Because of the small validation distance, interpStates has a larger number of elements. For faster visualization, the for loop skips more states in this step for faster visualization. planner.MaxConnectionDistance = 0.3; planner.ValidationDistance = 0.01; tic path = plan(planner,startConfig,goalConfig); toc Elapsed time is 9.827085 seconds. interpStates = interpolate(planner,path); for i = 1:10:size(interpStates,1) show(franka, interpStates(i,:),... "PreservePlot",false,... "Visuals","off",... "Collisions","on"); title("Plan 3: ValidationDistance = 0.01") drawnow; end {"String":"Figure Pick and Place Using RRT contains an axes object. The axes object with title Plan 3: ValidationDistance = 0.01 contains 40 objects of type patch, line. These objects represent panda_link0, panda_link1, panda_link2, panda_link3, panda_link4, panda_link5, panda_link6, panda_link7, panda_link8, panda_hand, panda_leftfinger, panda_rightfinger, panda_link1_mesh, panda_link2_mesh, panda_link3_mesh, panda_link4_mesh, panda_link5_mesh, panda_link6_mesh, panda_link7_mesh, panda_hand_mesh, panda_leftfinger_mesh, panda_rightfinger_mesh, panda_link0_mesh, panda_link1_coll_mesh, panda_link2_coll_mesh, panda_link3_coll_mesh, panda_link4_coll_mesh, panda_link5_coll_mesh, panda_link6_coll_mesh, panda_link7_coll_mesh, panda_hand_coll_mesh, panda_leftfinger_coll_mesh, panda_rightfinger_coll_mesh, panda_link0_coll_mesh.","Tex":"Plan 3: ValidationDistance = 0.01","LaTex":[]} The connect heuristic allows the planner to greedily join the start and goal trees. In places where the environment is less cluttered, this heuristic is useful for shorter planning times. However, a greedy behavior in a cluttered environment leads to wasted connection attempts. Setting the EnableConnectHeuristic to false may give longer planning times and longer paths, but results in a higher success rate of finding a path given the number of iterations. planner.ValidationDistance = 0.1; planner.EnableConnectHeuristic = false; tic path = plan(planner,startConfig,goalConfig); toc Elapsed time is 1.167672 seconds. interpStates = interpolate(planner,path); for i = 1:2:size(interpStates,1) show(franka, interpStates(i,:), ... "PreservePlot",false,... "Visuals","off",... "Collisions","on"); title("Plan 4: EnableConnectHeuristic = false") drawnow; end {"String":"Figure Pick and Place Using RRT contains an axes object. The axes object with title Plan 4: EnableConnectHeuristic = false contains 40 objects of type patch, line. These objects represent panda_link0, panda_link1, panda_link2, panda_link3, panda_link4, panda_link5, panda_link6, panda_link7, panda_link8, panda_hand, panda_leftfinger, panda_rightfinger, panda_link1_mesh, panda_link2_mesh, panda_link3_mesh, panda_link4_mesh, panda_link5_mesh, panda_link6_mesh, panda_link7_mesh, panda_hand_mesh, panda_leftfinger_mesh, panda_rightfinger_mesh, panda_link0_mesh, panda_link1_coll_mesh, panda_link2_coll_mesh, panda_link3_coll_mesh, panda_link4_coll_mesh, panda_link5_coll_mesh, panda_link6_coll_mesh, panda_link7_coll_mesh, panda_hand_coll_mesh, panda_leftfinger_coll_mesh, panda_rightfinger_coll_mesh, panda_link0_coll_mesh.","Tex":"Plan 4: EnableConnectHeuristic = false","LaTex":[]} Attach the Can to the End-Effector After tuning the planner for the desired behavior, follow the pick-and-place workflow where the robot moves an object through the environment. This example attaches a cylinder, or can, to the end-effector of the robot and moves it to a new location. For each configuration, the planner checks for collisions with the cylinder mesh as well. % Create can as a rigid body cylinder1 = env{3}; canBody = rigidBody("myCan"); canJoint = rigidBodyJoint("canJoint"); % Get current pose of the robot hand. startConfig = path(end, :); endEffectorPose = getTransform(franka,startConfig,"panda_hand"); % Place can into the end effector gripper. setFixedTransform(canJoint,endEffectorPose\cylinder1.Pose); % Add collision geometry to rigid body. addCollision(canBody,cylinder1,inv(cylinder1.Pose)); canBody.Joint = canJoint; % Add rigid body to robot model. addBody(franka,canBody,"panda_hand"); % Remove object from environment. env(3) = []; After the can has been attached to the robot arm, specify a goal configuration for placing the object. Modify the planner parameters. Plan a path from start to goal. Visualize the path. Notice the can clears the wall. goalConfig = [-0.6564 0.2885 -0.3187 -1.5941 0.1103 1.8678 -0.2344 0.04 0.04]; planner.MaxConnectionDistance = 1; planner.ValidationDistance = 0.2; planner.EnableConnectHeuristic = false; path = plan(planner,startConfig,goalConfig); interpStates = interpolate(planner,path); hold off {"String":"Figure Pick and Place Using RRT contains an axes object. The axes object with title Plan 4: EnableConnectHeuristic = false contains 40 objects of type patch, line. These objects represent panda_link0, panda_link1, panda_link2, panda_link3, panda_link4, panda_link5, panda_link6, panda_link7, panda_link8, panda_hand, panda_leftfinger, panda_rightfinger, panda_link1_mesh, panda_link2_mesh, panda_link3_mesh, panda_link4_mesh, panda_link5_mesh, panda_link6_mesh, panda_link7_mesh, panda_hand_mesh, panda_leftfinger_mesh, panda_rightfinger_mesh, panda_link0_mesh, panda_link1_coll_mesh, panda_link2_coll_mesh, panda_link3_coll_mesh, panda_link4_coll_mesh, panda_link5_coll_mesh, panda_link6_coll_mesh, panda_link7_coll_mesh, panda_hand_coll_mesh, panda_leftfinger_coll_mesh, panda_rightfinger_coll_mesh, panda_link0_coll_mesh.","Tex":"Plan 4: EnableConnectHeuristic = false","LaTex":[]} show(franka,config,"Visuals","off","Collisions","on"); hold on for i = 1:length(env) show(env{i}); end for i = 1:size(interpStates,1) show(franka,interpStates(i,:),... "PreservePlot", false,... "Visuals","off",... "Collisions","on"); title("Plan 5: Place the Can") drawnow; if i == (size(interpStates,1)) view([80,7]) end end {"String":"Figure Pick and Place Using RRT contains an axes object. The axes object with title Plan 5: Place the Can contains 42 objects of type patch, line. These objects represent panda_link0, panda_link1, panda_link2, panda_link3, panda_link4, panda_link5, panda_link6, panda_link7, panda_link8, panda_hand, panda_leftfinger, panda_rightfinger, myCan, panda_link1_mesh, panda_link2_mesh, panda_link3_mesh, panda_link4_mesh, panda_link5_mesh, panda_link6_mesh, panda_link7_mesh, panda_hand_mesh, panda_leftfinger_mesh, panda_rightfinger_mesh, panda_link0_mesh, panda_link1_coll_mesh, panda_link2_coll_mesh, panda_link3_coll_mesh, panda_link4_coll_mesh, panda_link5_coll_mesh, panda_link6_coll_mesh, panda_link7_coll_mesh, panda_hand_coll_mesh, panda_leftfinger_coll_mesh, panda_rightfinger_coll_mesh, myCan_coll_mesh, panda_link0_coll_mesh.","Tex":"Plan 5: Place the Can","LaTex":[]} Shorten the Planned Path To shorten your path, use the shorten function and specify a number of iterations. A small value for the ValidationDistance property combined with a large number of iterations can result in large computation times. shortenedPath = shorten(planner,path,20); interpStates = interpolate(planner,shortenedPath); for i = 1:size(interpStates,1) show(franka, ... interpStates(i, :), ... "PreservePlot", false, ... "Visuals", "off", ... "Collisions", "on"); drawnow; title("Plan 6: Shorten the Path") if i > (size(interpStates,1)-2) view([80,7]) end end hold off {"String":"Figure Pick and Place Using RRT contains an axes object. The axes object with title Plan 6: Shorten the Path contains 42 objects of type patch, line. These objects represent panda_link0, panda_link1, panda_link2, panda_link3, panda_link4, panda_link5, panda_link6, panda_link7, panda_link8, panda_hand, panda_leftfinger, panda_rightfinger, myCan, panda_link1_mesh, panda_link2_mesh, panda_link3_mesh, panda_link4_mesh, panda_link5_mesh, panda_link6_mesh, panda_link7_mesh, panda_hand_mesh, panda_leftfinger_mesh, panda_rightfinger_mesh, panda_link0_mesh, panda_link1_coll_mesh, panda_link2_coll_mesh, panda_link3_coll_mesh, panda_link4_coll_mesh, panda_link5_coll_mesh, panda_link6_coll_mesh, panda_link7_coll_mesh, panda_hand_coll_mesh, panda_leftfinger_coll_mesh, panda_rightfinger_coll_mesh, myCan_coll_mesh, panda_link0_coll_mesh.","Tex":"Plan 6: Shorten the Path","LaTex":[]}
{ "url": "https://it.mathworks.com/help/robotics/ug/pick-and-place-using-rrt-for-manipulators.html", "source_domain": "it.mathworks.com", "snapshot_id": "crawl=CC-MAIN-2022-40", "warc_metadata": { "Content-Length": "98983", "Content-Type": "application/http; msgtype=response", "WARC-Block-Digest": "sha1:MRC72HQTARXNNXTJPKFHJG7PD3U763GS", "WARC-Concurrent-To": "<urn:uuid:0ae6722f-faea-4bbd-802b-33637459c436>", "WARC-Date": "2022-09-26T09:54:00Z", "WARC-IP-Address": "104.96.217.125", "WARC-Identified-Payload-Type": "text/html", "WARC-Payload-Digest": "sha1:PPTFGBRMNMCWLWUTJ7H4THYDA3GBHCSO", "WARC-Record-ID": "<urn:uuid:44bd47af-4d17-4de2-bb69-9021e04943a6>", "WARC-Target-URI": "https://it.mathworks.com/help/robotics/ug/pick-and-place-using-rrt-for-manipulators.html", "WARC-Truncated": null, "WARC-Type": "response", "WARC-Warcinfo-ID": "<urn:uuid:88410cb6-0730-47e3-a4f4-7e9ab07dc90d>" }, "warc_info": "isPartOf: CC-MAIN-2022-40\r\npublisher: Common Crawl\r\ndescription: Wide crawl of the web for September/October 2022\r\noperator: Common Crawl Admin ([email protected])\r\nhostname: ip-10-67-67-120\r\nsoftware: Apache Nutch 1.19 (modified, https://github.com/commoncrawl/nutch/)\r\nrobots: checked via crawler-commons 1.4-SNAPSHOT (https://github.com/crawler-commons/crawler-commons)\r\nformat: WARC File Format 1.1\r\nconformsTo: https://iipc.github.io/warc-specifications/specifications/warc-format/warc-1.1/" }
{ "line_start_idx": [ 0, 13, 14, 56, 57, 658, 659, 692, 693, 819, 820, 1038, 1039, 1109, 1110, 1280, 1281, 1337, 1338, 1412, 1413, 1458, 1487, 1524, 1545, 1600, 1608, 1630, 1648, 1652, 1653, 2461, 2462, 2470, 2471, 2655, 2656, 2695, 2735, 2736, 2773, 2807, 2808, 2830, 2933, 2934, 3230, 3231, 3247, 3292, 3293, 3532, 3533, 3576, 3577, 3610, 3649, 3683, 3711, 3739, 3788, 3801, 3805, 3806, 4696, 4697, 4716, 4717, 4856, 4857, 5142, 5143, 5178, 5182, 5227, 5231, 5265, 5266, 5369, 5370, 5413, 5414, 5448, 5486, 5519, 5547, 5575, 5622, 5635, 5639, 5640, 6526, 6527, 7030, 7031, 7068, 7103, 7104, 7108, 7153, 7157, 7191, 7233, 7267, 7306, 7339, 7367, 7395, 7442, 7455, 7459, 7460, 8346, 8347, 8806, 8807, 8841, 8881, 8882, 8886, 8931, 8935, 8969, 9011, 9044, 9084, 9117, 9145, 9173, 9225, 9238, 9242, 9243, 10139, 10140, 10175, 10176, 10516, 10517, 10546, 10566, 10596, 10635, 10636, 10674, 10702, 10767, 10768, 10811, 10872, 10873, 10913, 10966, 10992, 10993, 11026, 11064, 11065, 11099, 11112, 11113, 11331, 11332, 11411, 11412, 11447, 11481, 11521, 11566, 11567, 11609, 11610, 11619, 11620, 12516, 12517, 12572, 12580, 12602, 12620, 12624, 12625, 12656, 12694, 12728, 12756, 12784, 12819, 12832, 12867, 12888, 12896, 12900, 12901, 13787, 13788, 13813, 13814, 14029, 14030, 14072, 14073, 14074, 14125, 14156, 14177, 14210, 14245, 14275, 14304, 14317, 14355, 14391, 14412, 14420, 14424, 14433, 14434 ], "line_end_idx": [ 13, 14, 56, 57, 658, 659, 692, 693, 819, 820, 1038, 1039, 1109, 1110, 1280, 1281, 1337, 1338, 1412, 1413, 1458, 1487, 1524, 1545, 1600, 1608, 1630, 1648, 1652, 1653, 2461, 2462, 2470, 2471, 2655, 2656, 2695, 2735, 2736, 2773, 2807, 2808, 2830, 2933, 2934, 3230, 3231, 3247, 3292, 3293, 3532, 3533, 3576, 3577, 3610, 3649, 3683, 3711, 3739, 3788, 3801, 3805, 3806, 4696, 4697, 4716, 4717, 4856, 4857, 5142, 5143, 5178, 5182, 5227, 5231, 5265, 5266, 5369, 5370, 5413, 5414, 5448, 5486, 5519, 5547, 5575, 5622, 5635, 5639, 5640, 6526, 6527, 7030, 7031, 7068, 7103, 7104, 7108, 7153, 7157, 7191, 7233, 7267, 7306, 7339, 7367, 7395, 7442, 7455, 7459, 7460, 8346, 8347, 8806, 8807, 8841, 8881, 8882, 8886, 8931, 8935, 8969, 9011, 9044, 9084, 9117, 9145, 9173, 9225, 9238, 9242, 9243, 10139, 10140, 10175, 10176, 10516, 10517, 10546, 10566, 10596, 10635, 10636, 10674, 10702, 10767, 10768, 10811, 10872, 10873, 10913, 10966, 10992, 10993, 11026, 11064, 11065, 11099, 11112, 11113, 11331, 11332, 11411, 11412, 11447, 11481, 11521, 11566, 11567, 11609, 11610, 11619, 11620, 12516, 12517, 12572, 12580, 12602, 12620, 12624, 12625, 12656, 12694, 12728, 12756, 12784, 12819, 12832, 12867, 12888, 12896, 12900, 12901, 13787, 13788, 13813, 13814, 14029, 14030, 14072, 14073, 14074, 14125, 14156, 14177, 14210, 14245, 14275, 14304, 14317, 14355, 14391, 14412, 14420, 14424, 14433, 14434, 15325 ] }
{ "red_pajama_v2": { "ccnet_original_length": 15325, "ccnet_original_nlines": 217, "rps_doc_curly_bracket": 0.0014355600578710437, "rps_doc_ldnoobw_words": 0, "rps_doc_lorem_ipsum": 0, "rps_doc_stop_word_fraction": 0.15672515332698822, "rps_doc_ut1_blacklist": 0, "rps_doc_frac_all_caps_words": 0.00662767980247736, "rps_doc_frac_lines_end_with_ellipsis": 0.10091742873191833, "rps_doc_frac_no_alph_words": 0.38362571597099304, "rps_doc_frac_unique_words": 0.26801347732543945, "rps_doc_mean_word_length": 7.8646464347839355, "rps_doc_num_sentences": 144, "rps_doc_symbol_to_word_ratio": 0.008577000349760056, "rps_doc_unigram_entropy": 5.351891040802002, "rps_doc_word_count": 1485, "rps_doc_frac_chars_dupe_10grams": 0.45380598306655884, "rps_doc_frac_chars_dupe_5grams": 0.5210206508636475, "rps_doc_frac_chars_dupe_6grams": 0.4916516840457916, "rps_doc_frac_chars_dupe_7grams": 0.479493111371994, "rps_doc_frac_chars_dupe_8grams": 0.4653652012348175, "rps_doc_frac_chars_dupe_9grams": 0.45380598306655884, "rps_doc_frac_chars_top_2gram": 0.01369979977607727, "rps_doc_frac_chars_top_3gram": 0.010274849832057953, "rps_doc_frac_chars_top_4gram": 0.01369979977607727, "rps_doc_books_importance": -1051.1337890625, "rps_doc_books_importance_length_correction": -1051.1337890625, "rps_doc_openwebtext_importance": -594.8056640625, "rps_doc_openwebtext_importance_length_correction": -594.8056640625, "rps_doc_wikipedia_importance": -498.2057189941406, "rps_doc_wikipedia_importance_length_correction": -498.2057189941406 }, "fasttext": { "dclm": 0.7253586649894714, "english": 0.6297696232795715, "fineweb_edu_approx": 3.3493738174438477, "eai_general_math": 0.9067492485046387, "eai_open_web_math": 0.13780295848846436, "eai_web_code": 0.9248744249343872 } }
{ "free_decimal_correspondence": { "primary": { "code": "004.0285", "labels": { "level_1": "General works, books and libraries, information sciences", "level_2": "", "level_3": "Computers and Computer science" } }, "secondary": { "code": "629.832", "labels": { "level_1": "Industrial arts, Technology, and Engineering", "level_2": "Engineering", "level_3": "" } } }, "bloom_cognitive_process": { "primary": { "code": "3", "label": "Apply" }, "secondary": { "code": "2", "label": "Understand" } }, "bloom_knowledge_domain": { "primary": { "code": "3", "label": "Procedural" }, "secondary": { "code": "2", "label": "Conceptual" } }, "document_type_v1": { "primary": { "code": "3", "label": "Reference/Encyclopedic/Educational" }, "secondary": { "code": "4", "label": "Code/Software" } }, "extraction_artifacts": { "primary": { "code": "0", "label": "No Artifacts" }, "secondary": { "code": "3", "label": "Irrelevant Content" } }, "missing_content": { "primary": { "code": "0", "label": "No missing content" }, "secondary": { "code": "4", "label": "Missing Images or Figures" } }, "document_type_v2": { "primary": { "code": "23", "label": "Tutorial" }, "secondary": { "code": "8", "label": "Documentation" } }, "reasoning_depth": { "primary": { "code": "3", "label": "Intermediate Reasoning" }, "secondary": { "code": "2", "label": "Basic Reasoning" } }, "technical_correctness": { "primary": { "code": "4", "label": "Highly Correct" }, "secondary": { "code": "3", "label": "Mostly Correct" } }, "education_level": { "primary": { "code": "3", "label": "Undergraduate Level" }, "secondary": { "code": "4", "label": "Graduate/Expert Level" } } }
672f1e42c33a7f9846924a2431ea77df
8,539,746,514,128,908,000
Should you cache hash values even for trivial classes? Hash tables are a fundamental data structure in computing, used to implement maps and sets. In software, we use hash values to determine where objects are located within hash tables. In my previous blog post, I showed that the bulk of the running time when checking values in a hash table could be taken up by the computation of the hash values themselves. I got some pushback because I implemented an array of three integers as a Java ArrayList<Integer>. This seems fair to me but there are obviously more efficient ways to represent three integers in Java. So I decided to do as suggested, and use a trivial class: class Triple { int x; int y; int z; public Triple(int mx, int my, int mz) { x = mx; y = my; z = my; } public int hashCode() { return x + 31 * y + 31 * 31 * z; } } I also implemented a version with precomputed hash values: class BufferedTriple { int x; int y; int z; int hashcode; public BufferedTriple(int mx, int my, int mz) { x = mx; y = my; z = my; hashcode = x + 31 * y + 31 * 31 * z; } public int hashCode() { return hashcode; } } Given these two classes, I create large hash tables (10 million entries) and I check 100 different target keys. So is the class with the cached hash value faster? Yes, it is about 25% faster: Triple BufferedTriple 0.4 us 0.3 us My code is available. I should add that Java’s hash tables re-hash the hash values that we provide, so the benefits of the cached hash value are less than they could be. Moreover, Strings have cached hash values by default in Java. I’m definitively not the first to notice that caching hash values can be valuable. I should add that this was not the point that I wanted to make in my original blog post. I do not particularly care whether you cache your hash values. The point I wanted to make is that even something as cheap as the computation of a hash value can be a limiting factor, even when you have lots of data in RAM. Further reading: For greater speed, try batching your out-of-cache data accesses Published by Daniel Lemire A computer science professor at the University of Quebec (TELUQ). 7 thoughts on “Should you cache hash values even for trivial classes?” 1. Daniel, Java Objects are bloated. For most efficient storage of triples, one needs to use type[3], e.g., int[3]. 1. Sorry, I forgot about this. BTW, I tried to keep integers as 3-int array INSIDE an object, but this only slowed things down. In a hindsight, this apparently only creates an extra “lookup layer”. 1. Sorry, I forgot about this. Honestly, I looked it up while preparing this blog post. I could not believe that arrays did not hash properly (as values) in Java. Mind you, I still cannot believe that Java lacks value types. 1. for me it’s not the computation in itself, but accessing data for computing the hash. Zven in L1, accessing data will be slower than performing add and mul operations This can be tested. Lay out the objects in L1 cache and check how many instructions you get per cycle while computing hash values. If you are correct, then you will get far fewer than one instruction per cycle (the CPU will spin empty). I believe you are not correct but I’d be interested in seeing the numbers. 2. Mmm, there must be some big trade off somewhere. To begin, an increase of memory footprint and a decrease in the cache utilization. Mutability will have cost as well. Did you try it in the context of C++? Leave a Reply Your email address will not be published. The comment form expects plain text. If you need to format your text, you can use HTML elements such strong, blockquote, cite, code and em. For formatting code as HTML automatically, I recommend tohtml.com. You may subscribe to this blog by email.
{ "url": "https://lemire.me/blog/2018/03/29/should-you-cache-hash-values-even-for-trivial-classes/", "source_domain": "lemire.me", "snapshot_id": "crawl=CC-MAIN-2022-21", "warc_metadata": { "Content-Length": "98173", "Content-Type": "application/http; msgtype=response", "WARC-Block-Digest": "sha1:EXBV27WDXZRV2IH7WOSN2QPNV5FHGVVC", "WARC-Concurrent-To": "<urn:uuid:f31fb38d-c736-4e76-8028-12b6292d0bc7>", "WARC-Date": "2022-05-21T10:11:17Z", "WARC-IP-Address": "104.21.7.120", "WARC-Identified-Payload-Type": "text/html", "WARC-Payload-Digest": "sha1:MT2QXWUELDKUR7ZGKZ7P5SZ242RLFOCT", "WARC-Record-ID": "<urn:uuid:194413be-60cc-47fd-8abc-6329b3dd9816>", "WARC-Target-URI": "https://lemire.me/blog/2018/03/29/should-you-cache-hash-values-even-for-trivial-classes/", "WARC-Truncated": null, "WARC-Type": "response", "WARC-Warcinfo-ID": "<urn:uuid:2982a82b-d7eb-49dc-bb24-9c5d1c67431a>" }, "warc_info": "isPartOf: CC-MAIN-2022-21\r\npublisher: Common Crawl\r\ndescription: Wide crawl of the web for May 2022\r\noperator: Common Crawl Admin ([email protected])\r\nhostname: ip-10-67-67-222\r\nsoftware: Apache Nutch 1.18 (modified, https://github.com/commoncrawl/nutch/)\r\nrobots: checked via crawler-commons 1.3-SNAPSHOT (https://github.com/crawler-commons/crawler-commons)\r\nformat: WARC File Format 1.1\r\nconformsTo: https://iipc.github.io/warc-specifications/specifications/warc-format/warc-1.1/" }
{ "line_start_idx": [ 0, 55, 56, 239, 240, 616, 617, 675, 676, 691, 700, 709, 718, 719, 761, 773, 785, 797, 801, 802, 828, 865, 869, 871, 872, 931, 932, 955, 964, 973, 982, 998, 999, 1049, 1061, 1073, 1085, 1127, 1131, 1132, 1158, 1179, 1183, 1185, 1186, 1298, 1299, 1379, 1380, 1402, 1416, 1417, 1439, 1440, 1588, 1589, 1734, 1735, 2047, 2048, 2129, 2130, 2143, 2144, 2158, 2159, 2225, 2226, 2297, 2298, 2311, 2312, 2421, 2422, 2626, 2627, 2666, 2667, 2871, 2872, 3046, 3047, 3290, 3291, 3372, 3373, 3583, 3584, 3598, 3599, 3848, 3849 ], "line_end_idx": [ 55, 56, 239, 240, 616, 617, 675, 676, 691, 700, 709, 718, 719, 761, 773, 785, 797, 801, 802, 828, 865, 869, 871, 872, 931, 932, 955, 964, 973, 982, 998, 999, 1049, 1061, 1073, 1085, 1127, 1131, 1132, 1158, 1179, 1183, 1185, 1186, 1298, 1299, 1379, 1380, 1402, 1416, 1417, 1439, 1440, 1588, 1589, 1734, 1735, 2047, 2048, 2129, 2130, 2143, 2144, 2158, 2159, 2225, 2226, 2297, 2298, 2311, 2312, 2421, 2422, 2626, 2627, 2666, 2667, 2871, 2872, 3046, 3047, 3290, 3291, 3372, 3373, 3583, 3584, 3598, 3599, 3848, 3849, 3889 ] }
{ "red_pajama_v2": { "ccnet_original_length": 3889, "ccnet_original_nlines": 91, "rps_doc_curly_bracket": 0.003085630014538765, "rps_doc_ldnoobw_words": 0, "rps_doc_lorem_ipsum": 0, "rps_doc_stop_word_fraction": 0.37142857909202576, "rps_doc_ut1_blacklist": 0, "rps_doc_frac_all_caps_words": 0.03928570821881294, "rps_doc_frac_lines_end_with_ellipsis": 0, "rps_doc_frac_no_alph_words": 0.21547618508338928, "rps_doc_frac_unique_words": 0.4487951695919037, "rps_doc_mean_word_length": 4.344879627227783, "rps_doc_num_sentences": 50, "rps_doc_symbol_to_word_ratio": 0, "rps_doc_unigram_entropy": 5.221088886260986, "rps_doc_word_count": 664, "rps_doc_frac_chars_dupe_10grams": 0.014558060094714165, "rps_doc_frac_chars_dupe_5grams": 0.06655112653970718, "rps_doc_frac_chars_dupe_6grams": 0.06655112653970718, "rps_doc_frac_chars_dupe_7grams": 0.036741770803928375, "rps_doc_frac_chars_dupe_8grams": 0.014558060094714165, "rps_doc_frac_chars_dupe_9grams": 0.014558060094714165, "rps_doc_frac_chars_top_2gram": 0.034662049263715744, "rps_doc_frac_chars_top_3gram": 0.009358749724924564, "rps_doc_frac_chars_top_4gram": 0.012478340417146683, "rps_doc_books_importance": -258.8515930175781, "rps_doc_books_importance_length_correction": -258.8515930175781, "rps_doc_openwebtext_importance": -173.0623016357422, "rps_doc_openwebtext_importance_length_correction": -173.0623016357422, "rps_doc_wikipedia_importance": -118.71924591064453, "rps_doc_wikipedia_importance_length_correction": -118.71924591064453 }, "fasttext": { "dclm": 0.32258206605911255, "english": 0.8899667859077454, "fineweb_edu_approx": 1.8173106908798218, "eai_general_math": 0.7387572526931763, "eai_open_web_math": 0.16729581356048584, "eai_web_code": 0.10803979635238647 } }
{ "free_decimal_correspondence": { "primary": { "code": "005.1", "labels": { "level_1": "General works, books and libraries, information sciences", "level_2": "", "level_3": "Computer programming" } }, "secondary": { "code": "005.72", "labels": { "level_1": "General works, books and libraries, information sciences", "level_2": "", "level_3": "Computer programming" } } }, "bloom_cognitive_process": { "primary": { "code": "4", "label": "Analyze" }, "secondary": { "code": "3", "label": "Apply" } }, "bloom_knowledge_domain": { "primary": { "code": "3", "label": "Procedural" }, "secondary": { "code": "2", "label": "Conceptual" } }, "document_type_v1": { "primary": { "code": "4", "label": "Code/Software" }, "secondary": { "code": "-1", "label": "Abstain" } }, "extraction_artifacts": { "primary": { "code": "0", "label": "No Artifacts" }, "secondary": { "code": "-1", "label": "Abstain" } }, "missing_content": { "primary": { "code": "0", "label": "No missing content" }, "secondary": { "code": "-1", "label": "Abstain" } }, "document_type_v2": { "primary": { "code": "16", "label": "Personal Blog" }, "secondary": { "code": "10", "label": "Knowledge Article" } }, "reasoning_depth": { "primary": { "code": "3", "label": "Intermediate Reasoning" }, "secondary": { "code": "2", "label": "Basic Reasoning" } }, "technical_correctness": { "primary": { "code": "4", "label": "Highly Correct" }, "secondary": { "code": "3", "label": "Mostly Correct" } }, "education_level": { "primary": { "code": "3", "label": "Undergraduate Level" }, "secondary": { "code": "2", "label": "High School Level" } } }
672f1e42c33a7f9846924a2431ea77df
4,025,715,090,714,299,000
Вы находитесь на странице: 1из 48 Ethiopian TVET System Training, Teaching and Learning Material Ethiopian TVET-System INFORMATION TECHNOLOGY SUPPORT SERVICE Level II LEARNING GUIDE # 1 Unit of Competence: Operate Database Application Module Title: Operating Database Application LG Code: ICT ITS2 LO1 01 TTLM Code: ICT ITS2 TTLM 0511 LO 5: Create database Queries TTLM Development Manual Date: 05-2011 Page 1 of 48 3rd Revision Author: ADDIS ABABA TEGBARE-ID Ethiopian TVET System Training, Teaching and Learning Material Introduction Learning Guide # 1  This learning guide was developed to provide you the necessary information regarding the following  Opening and designing DB application and principles  Creating DB objects  This guide will also assist you & you will be able to  Open and design database application incorporating basic design principles  Create database object according to database usage, as well as user requirements  Modify database object as required  Add and modify data in a table according to information requirements  Add, modify and delete records as required & Save and compile database objects Learning Activities 1. Read the specific objectives of this Learning Guide. 2. Read the information written in the “Information Sheets 1”. 3. Accomplish the “Self-check1”. 4. Submit your accomplished Self-check1. This will form part of your training portfolio/range. 5. Read the “Operation Sheet” and try to understand the procedures discussed. 6. Do the “LAP test” (if you are ready) and show your output to your teacher. Your teacher will evaluate your output either satisfactory or unsatisfactory. If unsatisfactory, your teacher shall advice you on additional work. But if satisfactory you can proceed to Learning Outcome 6. TTLM Development Manual Date: 05-2011 Page 2 of 48 3rd Revision Author: ADDIS ABABA TEGBARE-ID Ethiopian TVET System Training, Teaching and Learning Material Information Sheet-1 Creating Database Queries Queries  A query is a request for data results, for action on data, or for both. You can use a query to answer a simple question, to perform calculations, to combine data from different tables, or even to add, change, or delete table data.  Queries that you use to retrieve data from a table or to make calculations are called select queries.  Queries that add, change, or delete data are called action queries.  A query is a derived item in the database meant to answer specific questions that relate to the information in the database.  It is the means to retrieve relevant information in one or more tables.  Queries are useful during data processing. Types of Queries  When it comes time to build a query for your database, you have two ways to go about creating it. 1. Either use the Query Wizard that Microsoft Access provides for you, or 2. Create your own queries from scratch. Select Query  The select query is the simplest type of query and because of that, it is also the most commonly used one in Microsoft Access databases.  A select query is the most common type of query.  It retrieves data from one or more tables and displays the results in a datasheet where you can update the records (with some restrictions).  You can also use a select query to group records and calculate sums, counts, averages, and other types of totals.  It can be used to select and display data from either one table or a series of them depending on what is needed.  In the end, it is the user-determined criteria that tell the database what the selection is to be based on.  After the select query is called, it creates a "virtual" table where the data can be changed, but at no more than one record at a time. TTLM Development Manual Date: 05-2011 Page 3 of 48 3rd Revision Author: ADDIS ABABA TEGBARE-ID Ethiopian TVET System Training, Teaching and Learning Material Action Query  Action queries are very popular in data management because they allow for many records to be changed at one time instead of only single records like in a select query.  When the action query is called, the database undergoes a specific action depending on what was specified in the query itself.  This can include such things as creating new tables, deleting rows from existing ones and updating records or creating entirely new ones.  Four kinds of action queries are: 1. Append Query  An append query adds a group of records from one or more tables to the end of one or more tables.  For example, suppose that you acquire some new customers and a database containing a table of information on those customers.  To avoid typing all this information into your own database, you'd like to append it to your Customers table. 2. Delete Query  A delete query deletes a group of records from one or more tables.  For example, you could use a delete query to remove products that are discontinued or for which there are no orders.  With delete queries, you always delete entire records, not just selected fields within records. 3. Make Table Query  As the name suggests, it creates a table based on the set results of a query.  A make-table query creates a new table from all or part of the data in one or more tables.  Make-table queries are helpful for creating a table to export to other Microsoft Access databases or a history table that contains old records. 4. Update Query  Allows for one or more field in your table to be updated.  An update query makes global changes to a group of records in one or more tables. TTLM Development Manual Date: 05-2011 Page 4 of 48 3rd Revision Author: ADDIS ABABA TEGBARE-ID Ethiopian TVET System Training, Teaching and Learning Material  For example, you can raise prices by 10 percent for all dairy products, or you can raise salaries by 5 percent for the people within a certain job category. With an update query, you can change data in existing tables. Parameter Query  In Microsoft Access, a parameter query works with other types of queries to get whatever results you are after.  This is because, when using this type of query, you are able to pass a parameter to a different query, such as an action or a select query.  It can either be a value or a condition and will essentially tell the other query specifically what you want it to do.  It is often chosen because it allows for a dialog box where the end user can enter whatever parameter value they wish each time the query is run. The parameter query is just a modified select query.  A parameter query is a query that when run displays its own dialog box prompting you for information, such as criteria for retrieving records or a value you want to insert in a field.  You can design the query to prompt you for more than one piece of information; for example, you can design it to prompt you for two dates.  Access can then retrieve all records that fall between those two dates.  Parameter queries are also handy when used as the basis for forms, reports, and data access pages.  For example, you can create a monthly earnings report based on a parameter query.  When you print the report, Access displays a dialog box asking for the month that you want the report to cover.  You enter a month and Access prints the appropriate report. Aggregate Query  A special type of query is known as an aggregate query.  It can work on other queries (such as selection, action or parameter) just like the parameter query does, but instead of passing a parameter to another query it totals up the items by selected groups.  It essentially creates a summation of any selected attribute in your table.  This can be further generated into statistical amounts such as averages and standard deviation, just to name a couple. TTLM Development Manual Date: 05-2011 Page 5 of 48 3rd Revision Author: ADDIS ABABA TEGBARE-ID Ethiopian TVET System Training, Teaching and Learning Material Crosstab queries  You use crosstab queries to calculate and restructure data for easier analysis of your data.  Crosstab queries calculate a sum, average, count, or other type of total for data that is grouped by two types of information— one down the left side of the datasheet and another across the top. Queries may be created by either:  Query wizard  Design view  SQL View Creating Queries using a wizard  Creation of a query by using a wizard is a fastest and easy way to extract information you need from one or more tables. Creating Queries using design view  Creation of a query by design view is a user customized way of setting-up criteria to filter the necessary information you need from one or more tables. Creating Queries using SQL view  Creation of a form by SQL view is another option but not normally being use by Ms Access programmers. It requires an understanding of the Structured Query Language statements. TTLM Development Manual Date: 05-2011 Page 6 of 48 3rd Revision Author: ADDIS ABABA TEGBARE-ID Ethiopian TVET System Training, Teaching and Learning Material Self - Check 1 Creating Database Queries Name: ______________________________________ Date: ________________________________________ Time started: _______________________________ Time finished: _______________________________ Directions: Answer all the questions listed below. 1. List & Discuss the different types Queries? (5 points) 2. List & Discuss the different types Action Queries? (4 Point) 3. What is Query? (2 points) 4. Queries can be creates in to three ways? (3 point) Note: Satisfactory rating – above 12 points Satisfactory & below 11 points Unsatisfactory (You can ask you teacher for the copy of the correct answer) TTLM Development Manual Date: 05-2011 Page 7 of 48 3rd Revision Author: ADDIS ABABA TEGBARE-ID Ethiopian TVET System Training, Teaching and Learning Material Self - Check 1 Opening and designing DB application and principles (Answer Key) 1. Types of Queries a. Select Query b. Action Query c. Parameter Query d. Aggregate Query e. Crosstab queries 2. Types of Action Query a. Append Query: An append query adds a group of records from one or more tables to the end of one or more tables. b. Delete Query : A delete query deletes a group of records from one or more tables. c. Make Table Query: As the name suggests, it creates a table based on the set results of a query. d. Update Query : Allows for one or more field in your table to be updated. 3. Query is:- a. A query is a request for data results, for action on data, or for both. You can use a query to answer a simple question, to perform calculations, to combine data from different tables, or even to add, change, or delete table data. b. Queries are useful during data processing. c. A query is a derived item in the database meant to answer specific questions that relate to the information in the database. 4. Queries can be created using a. Query wizard b. Design view c. SQL View TTLM Development Manual Date: 05-2011 Page 8 of 48 3rd Revision Author: ADDIS ABABA TEGBARE-ID Ethiopian TVET System Training, Teaching and Learning Material Operation Sheet Creating Database Forms Introduction A query is the result of getting or isolating a list of values from a table or another query. Querying is the technique of examining data to create a dependent list. You can perform querying on a table, a form, or a query. Microsoft Access provides various means of performing such an operation visually. As an alternative, Microsoft Access supports SQL that allows you to query a database using code. There are various ways you create a query in Microsoft Access. The Query Wizard The simplest way to create a query is by using the Query Wizard. It presents a list of tables and queries you can select from the current database. To use the Query Wizard, on the Ribbon, you can click the Create tab and, in the Other section, click Query Wizard . This would display the New Query dialog box: On the New Query dialog box, you can click Simple Query Wizard and click OK. The first page of the Simple Query Wizard expects you to choose the origin of the query as a table or an already created query. After selecting the table or query, the second page of the wizard would present the fields of TTLM Development Manual Date: 05-2011 Page 9 of 48 3rd Revision Author: ADDIS ABABA TEGBARE-ID Ethiopian TVET System Training, Teaching and Learning Material that list and you can select those you want: The next page of the wizard allows you to specify the name of the query: Introduction to Query Design TTLM Development Manual Date: 05-2011 Page 10 of 48 3rd Revision Author: ADDIS ABABA TEGBARE-ID Ethiopian TVET System Training, Teaching and Learning Material Like other objects of a database, a query can be designed. You design a query using the Design View:  To display a query in Design View, from the Navigation Pane, you can right-click a query and click Design View  To start designing a new query, in the Other section of the Create tab of the Ribbon, click Query Design This would display the Show Table dialog box that allows you to specify the table or query that holds the fields you want to use in the intended query If the Show Tables dialog box is closed or for any reason you want to display it:  In the Query Setup section of the Design tab of the Ribbon, you can click the Show Table button  You can right-click anywhere on the query window and click Show Table... When a query is displaying in Design View, the Design tab of the Ribbon displays the buttons used for a query: TTLM Development Manual Date: 05-2011 Page 11 of 48 3rd Revision Author: ADDIS ABABA TEGBARE-ID Ethiopian TVET System Training, Teaching and Learning Material The Query Window The Query window allows you to design and manage various aspects of a query. You can right-click the title bar of the Query window to access a menu: One of the operations you can perform on the Query window consists of resizing its top and bottom sections by dragging the splitter bar up or down: TTLM Development Manual Date: 05-2011 Page 12 of 48 3rd Revision Author: ADDIS ABABA TEGBARE-ID Ethiopian TVET System Training, Teaching and Learning Material Selecting the Columns To create the fields for a query, you use the table(s) or query( queries) displayed in the upper section of the window. Once you have decided on the originating object(s), you can select which fields are relevant for your query:  To select one field from the list, just click it  To select many fields on the same range, you can click one of them, press and hold Shift. Then click one field on the other end of the desired range  To select fields at random, click one of the desired fields, press and hold Ctrl; then click each one of the desired fields  To select all fields, you can click the * line on the list of fields To Add Columns To make a field participate in a query, you have various options:  Once you have made your selection on the list in the top part of the query window, you can drag it and drop it in the bottom section of the query window TTLM Development Manual Date: 05-2011 Page 13 of 48 3rd Revision Author: ADDIS ABABA TEGBARE-ID Ethiopian TVET System Training, Teaching and Learning Material  You can also select more than one field and drag them:  Instead of dragging a field or all fields, you can either double-click a field to add it to the query, or double-click the line with * to add all fields to the query  In the bottom part of the query window, click an empty Field box to show a combo box. Then TTLM Development Manual Date: 05-2011 Page 14 of 48 3rd Revision Author: ADDIS ABABA TEGBARE-ID Ethiopian TVET System Training, Teaching and Learning Material click the arrow of that combo box and select an item from the list: Executing a Query To execute a query:  If the query is currently closed, from the Navigation Pane: o You can double-click it o You can right-click it and click Open  If the query is already opened and it is in Design View, on the Ribbon: o You can click the Run button o You can click the View button or you can click the arrow of the View button and click Datasheet View If you manually write a SQL statement and want to execute it, change the view to Datasheet View. Selecting a Column TTLM Development Manual Date: 05-2011 Page 15 of 48 3rd Revision Author: ADDIS ABABA TEGBARE-ID Ethiopian TVET System Training, Teaching and Learning Material Some operations require that you select a column from the bottom section of the query window:  To select a field in the lower section of the view, click the tiny bar of the column header: The whole column will be selected  To select a range of columns, click the column header of one at one end, press and hold Shift, then click the column header at the other end Since selecting a column in the Query window is a visual operation, there is no equivalent in SQL. Removing a Column From a Query As seen above, a query is built by selecting columns from the originating list and adding them. If you do not need a column anymore on a query, which happens regularly during data analysis, you can either delete it or replace it with another column:  To delete a column: o Once it is selected, you can press Delete o Right-click the column header and click Cut  To delete a group of columns, select them and press Delete Replacing a Column To replace a column, click the arrow on the combo box that displays its name and select a different field from the list: TTLM Development Manual Date: 05-2011 Page 16 of 48 3rd Revision Author: ADDIS ABABA TEGBARE-ID Ethiopian TVET System Training, Teaching and Learning Material Moving a Column Columns on a query are positioned incrementally as they are added to it. If you do not like the arrangement, you can move them and apply any sequence of your choice. Before moving a column or a group of columns, you must first select it. Then:  To move a field, click its column header once. Click it again and hold your mouse down, and drag in the direction on your choice TTLM Development Manual Date: 05-2011 Page 17 of 48 3rd Revision Author: ADDIS ABABA TEGBARE-ID Ethiopian TVET System Training, Teaching and Learning Material  To move a group of columns, first select the group and then proceed as if it were one column Data Entry on a Query You can perform data entry on a query. To do this, display it in Datasheet View, click the desired cells and type the information as necessary. Query Printing You can print the record of a query. To do this:  Right-click the query from the Navigation Pane and click Print  Open the query or select it in the Navigation Pane, then use the Office Button to access the Print menu Types of Queries TTLM Development Manual Date: 05-2011 Page 18 of 48 3rd Revision Author: ADDIS ABABA TEGBARE-ID Ethiopian TVET System Training, Teaching and Learning Material Select Query Create a select query in a desktop database For example, in a desktop database with a Products table, to see only the product names and the price for each product, you can create a select using the Query Wizard: On the Create tab, click Query Wizard and let the wizard walk you through the process of creating and running the query. However, if you want to add criteria to your query, use the query designer: 1. Click Create > Query Design. 2. In the Show Table box, double-click the Products table > Close. 3. To add the fields to the design grid, double-click the Product Name and List Price fields. 4. In the criteria row, under List Price add a criteria. For example, >=10 to show a list of products more than or equal to $10.00. 5. To see the query results, on the Design tab, click Run. Example 1 TTLM Development Manual Date: 05-2011 Page 19 of 48 3rd Revision Author: ADDIS ABABA TEGBARE-ID Ethiopian TVET System Training, Teaching and Learning Material Next, highlight the tables that you wish to use in the query. In this example, we've selected the Employees table and clicked on the Add button. When you are done selecting the tables, click on the Close button. Add the fields to the query. You can do this by double-clicking on the field name. In this example, we've added the LastName, FirstName, and Address fields. TTLM Development Manual Date: 05-2011 Page 20 of 48 3rd Revision Author: ADDIS ABABA TEGBARE-ID Ethiopian TVET System Training, Teaching and Learning Material Then click on the Save button at the top left of the window (this is the button with the picture of the disk). The Save As window should appear. Enter the name that you'd like to assign to the query and click on the OK button. In this example, we've saved the query as Query1. Example 2 TTLM Development Manual Date: 05-2011 Page 21 of 48 3rd Revision Author: ADDIS ABABA TEGBARE-ID Ethiopian TVET System Training, Teaching and Learning Material 1. Click the Query Wizard command In the Other group on the Create tab Figure 30. New Query Dialog Box 2. Choose Simple Query Wizard 3. Click OK Figure 31. Query Wizard - Select Tables & Fields 4. Select a Table or Query 5. Move fields from Available Fields to Selected Fields TTLM Development Manual Date: 05-2011 Page 22 of 48 3rd Revision Author: ADDIS ABABA TEGBARE-ID Ethiopian TVET System Training, Teaching and Learning Material 6. Repeat steps 4 and 5 for additional tables for queries 7. Click Next Figure 32. Query Wizard - Query Title 8. If desired, change the Query Title 9. Click Finish Append Query TTLM Development Manual Date: 05-2011 Page 23 of 48 3rd Revision Author: ADDIS ABABA TEGBARE-ID Ethiopian TVET System Training, Teaching and Learning Material 1. Click the QUERY DESIGN icon (located in the OTHER group of the CREATE ribbon). The QUERY DESIGN window then opens along with the SHOW TABLE dialog form. 2. The next step is to add tblMoreNames to the QUERY DESIGN window. Do this by clicking ADD in the SHOW TABLE dialog form. Notice it is the table containing the data to be appended that we have selected. 3. Click the APPEND icon from the QUERY TYPE group of the DESIGN ribbon. As you do this, you will see the APPEND dialog box open. 4. You are now asked to select the name of the original table to which the new data is to be appended. So select tblContacts from the drop down list. 5. You are also asked whether this table is stored in the current database or in an external database. In this exercise both tables are stored in the current database. This is the default button displayed in the option group, so there should not be any need to change it. 6. Click OK to close the dialog box. 7. Next we are going to select the fields from tblMoreNames to be appended. To do this drag and drop the Initials and LastName fields from the table (in the top half of the window) down onto the design grid. 8. Next we are going to tell Access which fields the data from Initials and Lastname will be appended to. To do this go down to the APPEND TO row of the design grid (see figure 3 below), and select FirstName in the Initials column, and Surname in the LastName Column. Figure 3: The Query Design Grid. 9. We could add query criteria at this stage, but this particular exercise does not require any. If we did, however, this is added in the CRITERIA row just like it is with a select query. 10. If you want to view the data that is going to be appended, click the VIEW icon from the RESULTS group of the DESIGN ribbon. It is especially important to do this if any if any criteria is applied in step 9 above. TTLM Development Manual Date: 05-2011 Page 24 of 48 3rd Revision Author: ADDIS ABABA TEGBARE-ID Ethiopian TVET System Training, Teaching and Learning Material 11. Once you are satisfied the correct data is going to be appended, click the RUN icon, again from the RESULTS group of the DESIGN ribbon. 12. A dialog box opens informing us that 10 rows are going to be appended, and asking us to confirm that we want to go ahead with this operation. Click YES to complete. To see the result of our Append Query, re-open tblContacts. Delete Query Example 1 TTLM Development Manual Date: 05-2011 Page 25 of 48 3rd Revision Author: ADDIS ABABA TEGBARE-ID Ethiopian TVET System Training, Teaching and Learning Material 1. Open the Employees table. It should open and look like this: Note that Tom Gumman and Tina Gumman are currently employees. 2. Close the table. 3. Click the Create tab. In the Ribbon, click Query Design. 4. When the Show Table window appears, click Employees. Then click the button. 5. Click the button. 6. In the Ribbon, click Delete. TTLM Development Manual Date: 05-2011 Page 26 of 48 3rd Revision Author: ADDIS ABABA TEGBARE-ID Ethiopian TVET System Training, Teaching and Learning Material Add fields 1. In the field list of the Employees table, click-and-drag the * field to the first column of the design grid. 2. Release the mouse button. The query design grid should look like this: TTLM Development Manual Date: 05-2011 Page 27 of 48 3rd Revision Author: ADDIS ABABA TEGBARE-ID Ethiopian TVET System Training, Teaching and Learning Material 3. In the field list of the Employees table, double-click the Last Name field. The query design grid should now look like this: Add criteria 1. In the Last Name column of the design grid, click in the Criteria row. 2. Type:Gumman then press the ENTER key. It should look like this: 3. In the Ribbon, click Run. When the alert window appears, click the button. TIP: Be careful when designing Delete queries. Once a record is deleted, the deletion cannot be undone. TTLM Development Manual Date: 05-2011 Page 28 of 48 3rd Revision Author: ADDIS ABABA TEGBARE-ID Ethiopian TVET System Training, Teaching and Learning Material You may want to run the query as a regular query first, to verify it's choosing the correct records, and then switch the type to a Delete query. 4. On the Title Bar, click the icon. 5. When the Save As window appears, type: Delete Old Employees in the Query Name box. 6. Click the button. 7. Close the query window. Verify results 1. Open the Employees table.It should look like this: Note that Tom Gumman and Tina Gumman are no longer listed as employees. 2. Exit Microsoft Access. TTLM Development Manual Date: 05-2011 Page 29 of 48 3rd Revision Author: ADDIS ABABA TEGBARE-ID Ethiopian TVET System Training, Teaching and Learning Material Example 2 Stage 1 - Creating the initial Select Query 1. Select the CREATE TAB of the Access Ribbon. 2. Click the QUERY DESIGN icon. It is located in the OTHER group of the CREATE ribbon. 3. Select tblContacts from the SHOW TABLE dialogue box. 4. Drag the asterix (*) from tblContacts down to the first column of the DESIGN GRID. This is a way of getting the query results to display all fields from the table without having to select each one individually. 5. Then Drag the Company field from tblContacts down to the second column of the grid. We have added this field separately because we are going to enter a criteria in this column. 6. Click on the CRITERIA row of the Company column, and add the criteria: "Company 5" The Select Query has now been created. It should look like this: Figure 2: The Select Query created in the first stage of the Delete Query. It is advisable to run the query at this point and check the results are correct. They should look like this: TTLM Development Manual Date: 05-2011 Page 30 of 48 3rd Revision Author: ADDIS ABABA TEGBARE-ID Ethiopian TVET System Training, Teaching and Learning Material Figure 2: The results from the Select Query. As you can see, our select query has found four records from tblContacts matching the criteria of Company 5". Since this is the correct result for the dataset we are working with, we can move onto the second stage of the process: converting the Select Query to a Delete Query. Stage 2 - Converting the Initial Select Query to a Delete Query 1. If you look at the QUERY TYPE group of the DESIGN ribbon, you will notice that the SELECT QUERY icon is highlighted orange. We need to change this to DELETE QUERY. To do this just click the DELETE QUERY icon further along the group. Figure 3: The QUERY TYPE group of the DESIGN ribbon. The DELETE QUERY icon is highlighted orange. 2. After the clicking the DELETE icon, you will notice that the row of SHOW tick boxes disappears from the DESIGN GRID, along with the row for SORT. A new row entitled DELETE has taken their place. Access has filled in the values of FROM and WHERE in the first and second columns respectively. These are SQL Keywords: the FROM keyword indicates the first column contains fields from tblContacts, and WHERE indicates the Company column contains a criteria against the data stored in this field. TTLM Development Manual Date: 05-2011 Page 31 of 48 3rd Revision Author: ADDIS ABABA TEGBARE-ID Ethiopian TVET System Training, Teaching and Learning Material Figure 4: The QUERY DESIGN GRID for our DELETE Query. Notice the new row for DELETE containing the SQL FROM and WHERE Keywords. 3. Click RUN from the QUERY RESULTS group. 4. Click YES when prompted whether we want to delete the number of rows matching our query criteria. This will be four rows for the dataset we have been working with. We can now go back and open the tblContacts table. As you can see from Figure 5 below, all Company 5 contacts have been removed by our DELETE QUERY. TTLM Development Manual Date: 05-2011 Page 32 of 48 3rd Revision Author: ADDIS ABABA TEGBARE-ID Ethiopian TVET System Training, Teaching and Learning Material Make table Query Using our scenario, we will create a make-table query for all customers from the specified area that have ordered products in the past 12 months. We will need to perform the following steps to create the query: 1. Create a new query, use the Customers and Orders tables. 2. From the Query Type button on the toolbar, select Make Table The Make Table dialog box appears, where you should enter the name for the new table. Here we can also select whether we want to create the new table in the current database or in another database. Ensure that the current database is selected and click OK. 3. Select the mailing information fields, in our case CustomerTitle, CustomerName, Address, City, Postcode from the Customers table, and OrderDate from the Orders table. 4. Specify the chosen City criteria in the City field and add the criteria required in the OrderDate field to only show records from the last 12 months - using the DateAdd TTLM Development Manual Date: 05-2011 Page 33 of 48 3rd Revision Author: ADDIS ABABA TEGBARE-ID Ethiopian TVET System Training, Teaching and Learning Material function: >=DateAdd("yyyy",-1,Now()) The query design should appear like the example below: 5. To check that the results are returned that we expect, click on the datasheet button on the toolbar. Once you have verified this, switch back to query design view. 6. In query design, deselect the Show: property for the OrderDate field, as we do not need this to be visible in our new table. 7. Click on the Run button on the toolbar. Microsoft Access now displays a message to indicate how many records will be copied to the new table. 8. Click Yes to complete the query, and create the new table. TTLM Development Manual Date: 05-2011 Page 34 of 48 3rd Revision Author: ADDIS ABABA TEGBARE-ID Ethiopian TVET System Training, Teaching and Learning Material Update Query Example 1 When the Show Table window appears, select the tables that you wish to use in the query and click on the Add button. When you have finished adding your tables, click on the Close button. In this example, we've selected the Big and Bot tables. You can select multiple tables by holding down the Ctrl key while selecting the table names. TTLM Development Manual Date: 05-2011 Page 35 of 48 3rd Revision Author: ADDIS ABABA TEGBARE-ID Ethiopian TVET System Training, Teaching and Learning Material Next, right-click somewhere next to the tables (but not on a table) in the query editor, and select Query Type > Update Query from the popup menu. Next, build the query like the one below: TTLM Development Manual Date: 05-2011 Page 36 of 48 3rd Revision Author: ADDIS ABABA TEGBARE-ID Ethiopian TVET System Training, Teaching and Learning Material This query will update the MFG field in the Bot table with the value in the MFG field in the Big table when the PART values match. The SQL for this query is as follows: UPDATE Big INNER JOIN Bot ON Big.PART = Bot.PART SET Bot.MFG = [Big].[MFG]; Example 2 Here are the steps that we follow to produce the required outcome: 1. Create a new query using the Products table and the Suppliers table. Include the fields that you are going to use to update the data (ProductID, ProductName and UnitPrice from the Products table, and CompanyName from the Suppliers table) We have also included criteria in the CompanyName field to limit the results to only those of the Supplier that we are updating the records for. Also, in the image below, you will see that we have included an additional field, just to test our expression. This will give us a value for the 3% increase, just to check that the results will be returned correctly. This field will be removed before we run the update, but we will use the expression later. Creating a SELECT query, that will later be changed to the UPDATE Query TTLM Development Manual Date: 05-2011 Page 37 of 48 3rd Revision Author: ADDIS ABABA TEGBARE-ID Ethiopian TVET System Training, Teaching and Learning Material 2. To check what results this will produce, run the query by clicking on the datasheet button Checking the results before running the Update Query 3. When we are happy with the resulting data, we can switch back to design view to convert the query to an Update query. From the Query Type button on the toolbar, select Update Query In the curUnitPrice column, in the Update To cell, type in the expression[curUnitPrice]*1.03 and press enter. This expression will update the original Unit Price by 3 Percent. The query design should now look like: The Update Query design 4. We now need to Run this query, using the Run button to update the data in our Products table that meets the criteria that is applied. The warning dialog box indicates the number of records that will be update, click Yes to accept this: TTLM Development Manual Date: 05-2011 Page 38 of 48 3rd Revision Author: ADDIS ABABA TEGBARE-ID Ethiopian TVET System Training, Teaching and Learning Material Example 3 1. Open the Products table. 2. Note the price (to customers) of French Truffles ($8.99) and Blue Mountain Coffee ($9.99). 3. Note the cost (to the store) of French Truffles ($6.15) and Blue Mountain Coffee ($8.00). 4. Close the Products table. 5. Make sure the Create tab is selected. In the Ribbon, click Query Design. 6. When the Show Table window appears, click Products. 7. Click the button. TTLM Development Manual Date: 05-2011 Page 39 of 48 3rd Revision Author: ADDIS ABABA TEGBARE-ID Ethiopian TVET System Training, Teaching and Learning Material The query window should look like this: 8. In the Ribbon, click Update. TTLM Development Manual Date: 05-2011 Page 40 of 48 3rd Revision Author: ADDIS ABABA TEGBARE-ID Ethiopian TVET System Training, Teaching and Learning Material Then click the button. 10. In the field list of the Products table, scroll down and double-click the Price field. The query design grid should now look like this: TTLM Development Manual Date: 05-2011 Page 41 of 48 3rd Revision Author: ADDIS ABABA TEGBARE-ID Ethiopian TVET System Training, Teaching and Learning Material 11. In the Price column, click in the Update To row. 12. Type: [Cost]*1.25 Then press the ENTER Key. TIP: Make sure that you've enabled all content in the database. To do that, click the button if the Security Warning bar is showing. This will raise prices for all products by 25%. TIP: The square brackets [ ] tell Access that Cost is a field. 13. In the Ribbon, click Run. 14. In the Title Bar, click the icon. TTLM Development Manual Date: 05-2011 Page 42 of 48 3rd Revision Author: ADDIS ABABA TEGBARE-ID Ethiopian TVET System Training, Teaching and Learning Material 15. When the Save As window appears, type: Update Product Prices in the Query Name box. Then click the button. 16. Close the query window. Verify results 1. Open the Products table. It should look like this: Note the updated price of French Truffles ($7.69) and Blue Mountain Coffee ($10.00). 2. Close the Products table. TTLM Development Manual Date: 05-2011 Page 43 of 48 3rd Revision Author: ADDIS ABABA TEGBARE-ID Ethiopian TVET System Training, Teaching and Learning Material Parametric Query Example 1 1. View the query in design view 2. In the criteria cell for the appropriate field(s), type in the desired expression within the square brackets([ ]) For example, using a parameter to query for date values: When the query is run, Microsoft Access displays this text to prompt the user for the criteria. The text of the prompt must be different from the field name, although it can include the field name. 3. Run the parameter query 4. When you are prompted to enter a parameter value, enter the value of the data that you want to view and click OK. The Prompt Message When you specify a prompt message, it should be brief but meaningful. Microsoft Access can display up to 50 or so characters in the prompt message. Do not type periods, commas, exclamation points or square brackets within the outer square brackets. When we run the above query, we will see a prompt similar to below: TTLM Development Manual Date: 05-2011 Page 44 of 48 3rd Revision Author: ADDIS ABABA TEGBARE-ID Ethiopian TVET System Training, Teaching and Learning Material Example 2 A parameter query is one of the simplest and most useful advanced queries you can create. It allows you to create a query that can be updated easily to reflect a new search term. When you open a parameter query, Access will prompt you for a search term and then show you query results that reflect that search. When you’re running parameter queries, search terms act as variable criteria, which are query criteria that changeeach time you run the query. For instance, let's say we own a bakery and want to create a query that will quickly look up orders that were placed on a certain date. We could create a parameter query with variable criteria in the Date field. This way, each time we run the query a dialog box will appear prompting us to enter the date we’d like our query to search for. We’ll enter the date we want, then Access will run the query using the date we entered as a search term. 1.1.1.1 To create and run a parameter query: 1. Create a query as you normally would, modifying the table joins if necessary, selecting the fields to include in your query, and adding any nonvariable criteria to the appropriate fields in the Criteria: row. 2. Locate the field or fields where you would like the variable criteria to appear, and place your cursor in theCriteria: row. 3. Type the phrase you would like to appear in the prompt that will pop up every time you run your query. Make sure to enclose the phrase in brackets [ ]. For example, in our parameter query that searches for orders placed on a certain date, we might type our criteria like this: [What date?]. TTLM Development Manual Date: 05-2011 Page 45 of 48 3rd Revision Author: ADDIS ABABA TEGBARE-ID Ethiopian TVET System Training, Teaching and Learning Material 4. On the Query Design tab, click the Run command to run your query. A dialog box will appear with the prompt you specified. Enter your search term, then click OK to view your query results. To run an existing parameter query, simply open it. 1.1.1.2 Tips for writing parameter queries  Ideally, the prompt you create for your query should make it clear what type of information the search term should be, and what format it should be entered in. For example, to guarantee that people enter a search for a date in the format we use in our database, we could write the following in the Criteria: row of the Date field:[What date? (mm/dd/yy)] .  The simplest parameter query will give you an exact-match criteria, meaning the query will search for the exact text you enter in the prompt. However, you can turn any type of criteria into a variable criteria. Simply type your prompt text in brackets in the part of the criteria where you would normally put a search term. TTLM Development Manual Date: 05-2011 Page 46 of 48 3rd Revision Author: ADDIS ABABA TEGBARE-ID Ethiopian TVET System Training, Teaching and Learning Material For example, in a normal query we could find orders that were placed between two dates by using the criteriaBetween x AND y, and replacing the x and y with the first and second dates, respectively. To turn this into a parameter criteria, we would simply replace the x and y with the text we want to appear in the prompt. Our variable criteria might look like this: Between [Enter the start date:] AND [Enter the ending date:]. These prompts would appear: TTLM Development Manual Date: 05-2011 Page 47 of 48 3rd Revision Author: ADDIS ABABA TEGBARE-ID Ethiopian TVET System Training, Teaching and Learning Material LAP Test Creating Database Queries Name: ______________________________________ Date: ________________________________________ Time started: _______________________________ Time finished: _______________________________ 1. Create Query 1.1. Create query using query button 1.2. Create query using query wizard 2. Create Simple Query 3. Create Select Query 4. Create Action Query 4.1. Create Append Query 4.2. Create Update Query 4.3. Create Make table Query 4.4. Create Delete Query 5. Create parametric Query 6. Create Aggregate Query 7. Create Crosstab Query 1.1 1.2 2 3 4.1 4.2 4.3 4.4 5 6 7 TTLM Development Manual Date: 05-2011 Page 48 of 48 3rd Revision Author: ADDIS ABABA TEGBARE-ID
{ "url": "https://ru.scribd.com/document/419327989/LO5-Creating-Database-Queries-docx", "source_domain": "ru.scribd.com", "snapshot_id": "crawl=CC-MAIN-2019-39", "warc_metadata": { "Content-Length": "422026", "Content-Type": "application/http; msgtype=response", "WARC-Block-Digest": "sha1:LCFIGNHIFXVJQBMZKSMG5LYGG6B44J53", "WARC-Concurrent-To": "<urn:uuid:70218bd7-6728-44d6-b771-e2c99769dc2b>", "WARC-Date": "2019-09-21T11:54:55Z", "WARC-IP-Address": "151.101.250.152", "WARC-Identified-Payload-Type": "application/xhtml+xml", "WARC-Payload-Digest": "sha1:2HID2QCVXSQOCYQ3QNKY67IM2N7CM4KG", "WARC-Record-ID": "<urn:uuid:23e2cda1-9911-4113-8b8d-67f8b1bf6085>", "WARC-Target-URI": "https://ru.scribd.com/document/419327989/LO5-Creating-Database-Queries-docx", "WARC-Truncated": null, "WARC-Type": "response", "WARC-Warcinfo-ID": "<urn:uuid:f5aece49-4322-466a-98a1-fea5c1ca9249>" }, "warc_info": "isPartOf: CC-MAIN-2019-39\r\npublisher: Common Crawl\r\ndescription: Wide crawl of the web for September 2019\r\noperator: Common Crawl Admin ([email protected])\r\nhostname: ip-10-67-67-140.ec2.internal\r\nsoftware: Apache Nutch 1.15 (modified, https://github.com/commoncrawl/nutch/)\r\nrobots: checked via crawler-commons 1.1-SNAPSHOT (https://github.com/crawler-commons/crawler-commons)\r\nformat: WARC File Format 1.1\r\nconformsTo: http://iipc.github.io/warc-specifications/specifications/warc-format/warc-1.1/" }
{ "line_start_idx": [ 0, 34, 35, 57, 58, 99, 100, 122, 123, 162, 163, 172, 173, 192, 193, 242, 243, 244, 289, 314, 344, 345, 375, 376, 377, 428, 472, 494, 535, 567, 568, 669, 723, 745, 801, 802, 879, 880, 881, 964, 1001, 1072, 1153, 1154, 1174, 1230, 1231, 1294, 1295, 1328, 1329, 1424, 1425, 1503, 1504, 1609, 1712, 1788, 1789, 1840, 1841, 1842, 1886, 1908, 1949, 1950, 1996, 1997, 2005, 2111, 2208, 2238, 2342, 2412, 2510, 2539, 2613, 2658, 2675, 2676, 2776, 2777, 2851, 2892, 2905, 2906, 3010, 3045, 3046, 3097, 3098, 3099, 3204, 3242, 3347, 3358, 3359, 3463, 3474, 3475, 3585, 3586, 3692, 3724, 3725, 3776, 3777, 3778, 3822, 3844, 3885, 3898, 3899, 3994, 4069, 4070, 4168, 4199, 4200, 4301, 4340, 4341, 4377, 4378, 4394, 4481, 4494, 4581, 4622, 4717, 4734, 4750, 4819, 4911, 4938, 5027, 5036, 5056, 5136, 5229, 5319, 5375, 5391, 5451, 5535, 5536, 5587, 5588, 5589, 5633, 5655, 5696, 5787, 5877, 5917, 5933, 5934, 6037, 6048, 6049, 6154, 6191, 6192, 6298, 6313, 6314, 6418, 6515, 6516, 6607, 6702, 6804, 6843, 6917, 7018, 7102, 7206, 7216, 7278, 7279, 7295, 7353, 7455, 7556, 7557, 7635, 7636, 7742, 7757, 7758, 7809, 7810, 7811, 7855, 7877, 7918, 7935, 7936, 8031, 8138, 8228, 8229, 8263, 8264, 8265, 8280, 8294, 8305, 8337, 8440, 8460, 8495, 8594, 8650, 8682, 8773, 8860, 8861, 8912, 8913, 8914, 8958, 8980, 9021, 9022, 9063, 9064, 9156, 9157, 9250, 9251, 9302, 9303, 9361, 9362, 9426, 9427, 9456, 9457, 9511, 9512, 9602, 9663, 9664, 9715, 9716, 9717, 9761, 9783, 9824, 9825, 9892, 9893, 9894, 9907, 9908, 9928, 9944, 9960, 9979, 9998, 10018, 10043, 10131, 10158, 10243, 10335, 10342, 10418, 10432, 10530, 10620, 10666, 10712, 10807, 10840, 10872, 10888, 10903, 10915, 10916, 10967, 10968, 10969, 11013, 11035, 11076, 11077, 11117, 11118, 11131, 11132, 11226, 11314, 11398, 11491, 11534, 11535, 11598, 11599, 11616, 11617, 11715, 11765, 11766, 11862, 11928, 11929, 12028, 12127, 12228, 12229, 12277, 12278, 12279, 12282, 12326, 12348, 12389, 12434, 12435, 12508, 12509, 12538, 12539, 12588, 12589, 12590, 12593, 12637, 12659, 12700, 12701, 12802, 12803, 12898, 12916, 13016, 13023, 13125, 13174, 13175, 13257, 13258, 13349, 13350, 13357, 13432, 13534, 13543, 13544, 13593, 13594, 13595, 13598, 13642, 13664, 13705, 13706, 13723, 13724, 13821, 13873, 13874, 13972, 14022, 14023, 14072, 14073, 14074, 14077, 14121, 14143, 14184, 14185, 14207, 14208, 14313, 14412, 14437, 14438, 14489, 14490, 14491, 14588, 14642, 14746, 14768, 14839, 14854, 14855, 14921, 14922, 15020, 15077, 15078, 15127, 15128, 15129, 15132, 15176, 15198, 15239, 15240, 15297, 15298, 15403, 15466, 15559, 15560, 15609, 15610, 15611, 15614, 15658, 15680, 15721, 15789, 15790, 15808, 15809, 15829, 15830, 15892, 15893, 15894, 15920, 15960, 16034, 16035, 16066, 16067, 16149, 16170, 16267, 16268, 16287, 16288, 16337, 16338, 16339, 16342, 16386, 16408, 16449, 16450, 16544, 16545, 16640, 16641, 16675, 16676, 16677, 16774, 16820, 16919, 16920, 16951, 16952, 17055, 17150, 17202, 17203, 17225, 17269, 17315, 17376, 17395, 17396, 17496, 17517, 17518, 17567, 17568, 17569, 17572, 17616, 17638, 17679, 17680, 17696, 17697, 17793, 17889, 17941, 17942, 18041, 18073, 18074, 18123, 18124, 18125, 18128, 18172, 18194, 18235, 18236, 18331, 18353, 18354, 18459, 18498, 18499, 18514, 18515, 18564, 18565, 18630, 18731, 18736, 18737, 18754, 18755, 18804, 18805, 18806, 18809, 18853, 18875, 18916, 18929, 18973, 18974, 19070, 19161, 19256, 19263, 19264, 19340, 19341, 19373, 19374, 19375, 19442, 19536, 19643, 19668, 19669, 19728, 19729, 19739, 19788, 19791, 19835, 19857, 19898, 19899, 20017, 20111, 20112, 20228, 20269, 20270, 20319, 20320, 20321, 20324, 20368, 20390, 20431, 20542, 20543, 20659, 20709, 20710, 20720, 20721, 20770, 20771, 20772, 20775, 20819, 20841, 20882, 20953, 20954, 20986, 20987, 20988, 21018, 21019, 21031, 21032, 21081, 21082, 21083, 21110, 21166, 21215, 21218, 21262, 21284, 21325, 21383, 21384, 21398, 21399, 21437, 21438, 21439, 21477, 21493, 21494, 21507, 21556, 21559, 21603, 21625, 21666, 21752, 21822, 21910, 21994, 22026, 22119, 22156, 22249, 22306, 22399, 22494, 22578, 22615, 22707, 22796, 22823, 22916, 23002, 23091, 23092, 23125, 23126, 23226, 23314, 23406, 23503, 23531, 23532, 23581, 23582, 23583, 23586, 23630, 23652, 23693, 23793, 23833, 23925, 24002, 24003, 24063, 24064, 24077, 24078, 24088, 24089, 24138, 24139, 24140, 24143, 24187, 24209, 24250, 24279, 24280, 24315, 24316, 24378, 24379, 24399, 24400, 24425, 24426, 24461, 24462, 24518, 24519, 24542, 24543, 24564, 24565, 24597, 24598, 24647, 24648, 24649, 24652, 24696, 24718, 24759, 24770, 24771, 24866, 24883, 24884, 24913, 24914, 24959, 24960, 25009, 25010, 25011, 25014, 25058, 25080, 25121, 25200, 25201, 25250, 25251, 25264, 25265, 25339, 25340, 25407, 25408, 25437, 25438, 25487, 25488, 25489, 25575, 25593, 25594, 25643, 25644, 25645, 25648, 25692, 25714, 25755, 25847, 25900, 25901, 25938, 25939, 25981, 25982, 26003, 26004, 26005, 26028, 26029, 26050, 26051, 26078, 26079, 26094, 26095, 26149, 26150, 26222, 26223, 26249, 26250, 26299, 26300, 26301, 26304, 26348, 26370, 26411, 26421, 26422, 26466, 26467, 26514, 26515, 26516, 26603, 26659, 26755, 26850, 26873, 26963, 27053, 27139, 27140, 27205, 27206, 27248, 27249, 27250, 27283, 27284, 27388, 27394, 27395, 27444, 27445, 27446, 27449, 27493, 27515, 27556, 27601, 27602, 27697, 27794, 27879, 27880, 27944, 27945, 28038, 28128, 28181, 28182, 28235, 28236, 28237, 28282, 28283, 28370, 28442, 28534, 28618, 28699, 28777, 28778, 28827, 28828, 28829, 28832, 28876, 28898, 28939, 28940, 28980, 28981, 28982, 29026, 29060, 29070, 29071, 29114, 29115, 29116, 29207, 29283, 29284, 29371, 29433, 29434, 29483, 29484, 29485, 29488, 29532, 29554, 29595, 29612, 29613, 29714, 29813, 29824, 29884, 29885, 29949, 29950, 30044, 30138, 30207, 30208, 30306, 30378, 30479, 30550, 30599, 30602, 30646, 30668, 30709, 30746, 30747, 30802, 30803, 30898, 30970, 31072, 31098, 31193, 31243, 31244, 31306, 31307, 31356, 31357, 31358, 31361, 31405, 31427, 31468, 31481, 31482, 31492, 31493, 31591, 31680, 31681, 31784, 31830, 31831, 31880, 31881, 31882, 31885, 31929, 31951, 31992, 32092, 32139, 32140, 32182, 32183, 32232, 32233, 32234, 32237, 32281, 32303, 32344, 32447, 32475, 32476, 32514, 32515, 32564, 32565, 32592, 32593, 32603, 32604, 32671, 32771, 32863, 32912, 32913, 33011, 33058, 33059, 33158, 33256, 33349, 33367, 33368, 33440, 33489, 33492, 33536, 33558, 33599, 33693, 33694, 33747, 33748, 33749, 33844, 33870, 33871, 33934, 33935, 33998, 33999, 34000, 34093, 34113, 34114, 34153, 34154, 34178, 34179, 34180, 34270, 34364, 34419, 34420, 34469, 34470, 34471, 34474, 34518, 34540, 34581, 34591, 34592, 34620, 34621, 34706, 34715, 34716, 34800, 34809, 34810, 34839, 34840, 34881, 34882, 34917, 34918, 34973, 34974, 34995, 34996, 34997, 35046, 35049, 35093, 35115, 35156, 35196, 35197, 35229, 35230, 35279, 35280, 35281, 35284, 35328, 35350, 35391, 35392, 35415, 35416, 35507, 35508, 35557, 35558, 35607, 35608, 35609, 35612, 35656, 35678, 35719, 35772, 35773, 35783, 35784, 35796, 35797, 35823, 35824, 35825, 35907, 35958, 36006, 36069, 36070, 36100, 36101, 36139, 36140, 36189, 36190, 36191, 36194, 36238, 36260, 36301, 36344, 36345, 36367, 36368, 36369, 36392, 36393, 36416, 36417, 36445, 36446, 36461, 36462, 36490, 36491, 36517, 36518, 36603, 36604, 36633, 36634, 36683, 36684, 36685, 36688, 36732, 36754, 36795, 36812, 36813, 36823, 36824, 36857, 36858, 36859, 36962, 36976, 36977, 37034, 37035, 37135, 37233, 37260, 37358, 37377, 37396, 37495, 37595, 37645, 37713, 37714, 37763, 37764, 37765, 37768, 37812, 37834, 37875, 37885, 37886, 37979, 38070, 38157, 38197, 38198, 38286, 38381, 38477, 38573, 38666, 38681, 38682, 38774, 38787, 38788, 38833, 38834, 38925, 39005, 39046, 39140, 39173, 39263, 39348, 39433, 39467, 39468, 39517, 39518, 39519, 39522, 39566, 39588, 39629, 39716, 39800, 39820, 39821, 39873, 39874, 39917, 39918, 40000, 40084, 40172, 40262, 40276, 40277, 40370, 40455, 40550, 40603, 40604, 40653, 40654, 40655, 40658, 40702, 40724, 40765, 40852, 40942, 41033, 41124, 41206, 41220, 41221, 41270, 41271, 41272, 41275, 41319, 41341, 41382, 41383, 41418, 41419, 41511, 41512, 41605, 41606, 41622, 41659, 41696, 41719, 41742, 41765, 41790, 41815, 41844, 41869, 41896, 41922, 41947, 41948, 41982, 41983, 42032, 42033, 42034, 42037 ], "line_end_idx": [ 34, 35, 57, 58, 99, 100, 122, 123, 162, 163, 172, 173, 192, 193, 242, 243, 244, 289, 314, 344, 345, 375, 376, 377, 428, 472, 494, 535, 567, 568, 669, 723, 745, 801, 802, 879, 880, 881, 964, 1001, 1072, 1153, 1154, 1174, 1230, 1231, 1294, 1295, 1328, 1329, 1424, 1425, 1503, 1504, 1609, 1712, 1788, 1789, 1840, 1841, 1842, 1886, 1908, 1949, 1950, 1996, 1997, 2005, 2111, 2208, 2238, 2342, 2412, 2510, 2539, 2613, 2658, 2675, 2676, 2776, 2777, 2851, 2892, 2905, 2906, 3010, 3045, 3046, 3097, 3098, 3099, 3204, 3242, 3347, 3358, 3359, 3463, 3474, 3475, 3585, 3586, 3692, 3724, 3725, 3776, 3777, 3778, 3822, 3844, 3885, 3898, 3899, 3994, 4069, 4070, 4168, 4199, 4200, 4301, 4340, 4341, 4377, 4378, 4394, 4481, 4494, 4581, 4622, 4717, 4734, 4750, 4819, 4911, 4938, 5027, 5036, 5056, 5136, 5229, 5319, 5375, 5391, 5451, 5535, 5536, 5587, 5588, 5589, 5633, 5655, 5696, 5787, 5877, 5917, 5933, 5934, 6037, 6048, 6049, 6154, 6191, 6192, 6298, 6313, 6314, 6418, 6515, 6516, 6607, 6702, 6804, 6843, 6917, 7018, 7102, 7206, 7216, 7278, 7279, 7295, 7353, 7455, 7556, 7557, 7635, 7636, 7742, 7757, 7758, 7809, 7810, 7811, 7855, 7877, 7918, 7935, 7936, 8031, 8138, 8228, 8229, 8263, 8264, 8265, 8280, 8294, 8305, 8337, 8440, 8460, 8495, 8594, 8650, 8682, 8773, 8860, 8861, 8912, 8913, 8914, 8958, 8980, 9021, 9022, 9063, 9064, 9156, 9157, 9250, 9251, 9302, 9303, 9361, 9362, 9426, 9427, 9456, 9457, 9511, 9512, 9602, 9663, 9664, 9715, 9716, 9717, 9761, 9783, 9824, 9825, 9892, 9893, 9894, 9907, 9908, 9928, 9944, 9960, 9979, 9998, 10018, 10043, 10131, 10158, 10243, 10335, 10342, 10418, 10432, 10530, 10620, 10666, 10712, 10807, 10840, 10872, 10888, 10903, 10915, 10916, 10967, 10968, 10969, 11013, 11035, 11076, 11077, 11117, 11118, 11131, 11132, 11226, 11314, 11398, 11491, 11534, 11535, 11598, 11599, 11616, 11617, 11715, 11765, 11766, 11862, 11928, 11929, 12028, 12127, 12228, 12229, 12277, 12278, 12279, 12282, 12326, 12348, 12389, 12434, 12435, 12508, 12509, 12538, 12539, 12588, 12589, 12590, 12593, 12637, 12659, 12700, 12701, 12802, 12803, 12898, 12916, 13016, 13023, 13125, 13174, 13175, 13257, 13258, 13349, 13350, 13357, 13432, 13534, 13543, 13544, 13593, 13594, 13595, 13598, 13642, 13664, 13705, 13706, 13723, 13724, 13821, 13873, 13874, 13972, 14022, 14023, 14072, 14073, 14074, 14077, 14121, 14143, 14184, 14185, 14207, 14208, 14313, 14412, 14437, 14438, 14489, 14490, 14491, 14588, 14642, 14746, 14768, 14839, 14854, 14855, 14921, 14922, 15020, 15077, 15078, 15127, 15128, 15129, 15132, 15176, 15198, 15239, 15240, 15297, 15298, 15403, 15466, 15559, 15560, 15609, 15610, 15611, 15614, 15658, 15680, 15721, 15789, 15790, 15808, 15809, 15829, 15830, 15892, 15893, 15894, 15920, 15960, 16034, 16035, 16066, 16067, 16149, 16170, 16267, 16268, 16287, 16288, 16337, 16338, 16339, 16342, 16386, 16408, 16449, 16450, 16544, 16545, 16640, 16641, 16675, 16676, 16677, 16774, 16820, 16919, 16920, 16951, 16952, 17055, 17150, 17202, 17203, 17225, 17269, 17315, 17376, 17395, 17396, 17496, 17517, 17518, 17567, 17568, 17569, 17572, 17616, 17638, 17679, 17680, 17696, 17697, 17793, 17889, 17941, 17942, 18041, 18073, 18074, 18123, 18124, 18125, 18128, 18172, 18194, 18235, 18236, 18331, 18353, 18354, 18459, 18498, 18499, 18514, 18515, 18564, 18565, 18630, 18731, 18736, 18737, 18754, 18755, 18804, 18805, 18806, 18809, 18853, 18875, 18916, 18929, 18973, 18974, 19070, 19161, 19256, 19263, 19264, 19340, 19341, 19373, 19374, 19375, 19442, 19536, 19643, 19668, 19669, 19728, 19729, 19739, 19788, 19791, 19835, 19857, 19898, 19899, 20017, 20111, 20112, 20228, 20269, 20270, 20319, 20320, 20321, 20324, 20368, 20390, 20431, 20542, 20543, 20659, 20709, 20710, 20720, 20721, 20770, 20771, 20772, 20775, 20819, 20841, 20882, 20953, 20954, 20986, 20987, 20988, 21018, 21019, 21031, 21032, 21081, 21082, 21083, 21110, 21166, 21215, 21218, 21262, 21284, 21325, 21383, 21384, 21398, 21399, 21437, 21438, 21439, 21477, 21493, 21494, 21507, 21556, 21559, 21603, 21625, 21666, 21752, 21822, 21910, 21994, 22026, 22119, 22156, 22249, 22306, 22399, 22494, 22578, 22615, 22707, 22796, 22823, 22916, 23002, 23091, 23092, 23125, 23126, 23226, 23314, 23406, 23503, 23531, 23532, 23581, 23582, 23583, 23586, 23630, 23652, 23693, 23793, 23833, 23925, 24002, 24003, 24063, 24064, 24077, 24078, 24088, 24089, 24138, 24139, 24140, 24143, 24187, 24209, 24250, 24279, 24280, 24315, 24316, 24378, 24379, 24399, 24400, 24425, 24426, 24461, 24462, 24518, 24519, 24542, 24543, 24564, 24565, 24597, 24598, 24647, 24648, 24649, 24652, 24696, 24718, 24759, 24770, 24771, 24866, 24883, 24884, 24913, 24914, 24959, 24960, 25009, 25010, 25011, 25014, 25058, 25080, 25121, 25200, 25201, 25250, 25251, 25264, 25265, 25339, 25340, 25407, 25408, 25437, 25438, 25487, 25488, 25489, 25575, 25593, 25594, 25643, 25644, 25645, 25648, 25692, 25714, 25755, 25847, 25900, 25901, 25938, 25939, 25981, 25982, 26003, 26004, 26005, 26028, 26029, 26050, 26051, 26078, 26079, 26094, 26095, 26149, 26150, 26222, 26223, 26249, 26250, 26299, 26300, 26301, 26304, 26348, 26370, 26411, 26421, 26422, 26466, 26467, 26514, 26515, 26516, 26603, 26659, 26755, 26850, 26873, 26963, 27053, 27139, 27140, 27205, 27206, 27248, 27249, 27250, 27283, 27284, 27388, 27394, 27395, 27444, 27445, 27446, 27449, 27493, 27515, 27556, 27601, 27602, 27697, 27794, 27879, 27880, 27944, 27945, 28038, 28128, 28181, 28182, 28235, 28236, 28237, 28282, 28283, 28370, 28442, 28534, 28618, 28699, 28777, 28778, 28827, 28828, 28829, 28832, 28876, 28898, 28939, 28940, 28980, 28981, 28982, 29026, 29060, 29070, 29071, 29114, 29115, 29116, 29207, 29283, 29284, 29371, 29433, 29434, 29483, 29484, 29485, 29488, 29532, 29554, 29595, 29612, 29613, 29714, 29813, 29824, 29884, 29885, 29949, 29950, 30044, 30138, 30207, 30208, 30306, 30378, 30479, 30550, 30599, 30602, 30646, 30668, 30709, 30746, 30747, 30802, 30803, 30898, 30970, 31072, 31098, 31193, 31243, 31244, 31306, 31307, 31356, 31357, 31358, 31361, 31405, 31427, 31468, 31481, 31482, 31492, 31493, 31591, 31680, 31681, 31784, 31830, 31831, 31880, 31881, 31882, 31885, 31929, 31951, 31992, 32092, 32139, 32140, 32182, 32183, 32232, 32233, 32234, 32237, 32281, 32303, 32344, 32447, 32475, 32476, 32514, 32515, 32564, 32565, 32592, 32593, 32603, 32604, 32671, 32771, 32863, 32912, 32913, 33011, 33058, 33059, 33158, 33256, 33349, 33367, 33368, 33440, 33489, 33492, 33536, 33558, 33599, 33693, 33694, 33747, 33748, 33749, 33844, 33870, 33871, 33934, 33935, 33998, 33999, 34000, 34093, 34113, 34114, 34153, 34154, 34178, 34179, 34180, 34270, 34364, 34419, 34420, 34469, 34470, 34471, 34474, 34518, 34540, 34581, 34591, 34592, 34620, 34621, 34706, 34715, 34716, 34800, 34809, 34810, 34839, 34840, 34881, 34882, 34917, 34918, 34973, 34974, 34995, 34996, 34997, 35046, 35049, 35093, 35115, 35156, 35196, 35197, 35229, 35230, 35279, 35280, 35281, 35284, 35328, 35350, 35391, 35392, 35415, 35416, 35507, 35508, 35557, 35558, 35607, 35608, 35609, 35612, 35656, 35678, 35719, 35772, 35773, 35783, 35784, 35796, 35797, 35823, 35824, 35825, 35907, 35958, 36006, 36069, 36070, 36100, 36101, 36139, 36140, 36189, 36190, 36191, 36194, 36238, 36260, 36301, 36344, 36345, 36367, 36368, 36369, 36392, 36393, 36416, 36417, 36445, 36446, 36461, 36462, 36490, 36491, 36517, 36518, 36603, 36604, 36633, 36634, 36683, 36684, 36685, 36688, 36732, 36754, 36795, 36812, 36813, 36823, 36824, 36857, 36858, 36859, 36962, 36976, 36977, 37034, 37035, 37135, 37233, 37260, 37358, 37377, 37396, 37495, 37595, 37645, 37713, 37714, 37763, 37764, 37765, 37768, 37812, 37834, 37875, 37885, 37886, 37979, 38070, 38157, 38197, 38198, 38286, 38381, 38477, 38573, 38666, 38681, 38682, 38774, 38787, 38788, 38833, 38834, 38925, 39005, 39046, 39140, 39173, 39263, 39348, 39433, 39467, 39468, 39517, 39518, 39519, 39522, 39566, 39588, 39629, 39716, 39800, 39820, 39821, 39873, 39874, 39917, 39918, 40000, 40084, 40172, 40262, 40276, 40277, 40370, 40455, 40550, 40603, 40604, 40653, 40654, 40655, 40658, 40702, 40724, 40765, 40852, 40942, 41033, 41124, 41206, 41220, 41221, 41270, 41271, 41272, 41275, 41319, 41341, 41382, 41383, 41418, 41419, 41511, 41512, 41605, 41606, 41622, 41659, 41696, 41719, 41742, 41765, 41790, 41815, 41844, 41869, 41896, 41922, 41947, 41948, 41982, 41983, 42032, 42033, 42034, 42037, 42080 ] }
{ "red_pajama_v2": { "ccnet_original_length": 42080, "ccnet_original_nlines": 1218, "rps_doc_curly_bracket": 0, "rps_doc_ldnoobw_words": 0, "rps_doc_lorem_ipsum": 0, "rps_doc_stop_word_fraction": 0.3369312882423401, "rps_doc_ut1_blacklist": 0, "rps_doc_frac_all_caps_words": 0.05095171928405762, "rps_doc_frac_lines_end_with_ellipsis": 0.0008203400066122413, "rps_doc_frac_no_alph_words": 0.19730733335018158, "rps_doc_frac_unique_words": 0.12751862406730652, "rps_doc_mean_word_length": 4.540160179138184, "rps_doc_num_sentences": 461, "rps_doc_symbol_to_word_ratio": 0.0003481899912003428, "rps_doc_unigram_entropy": 5.443512916564941, "rps_doc_word_count": 7246, "rps_doc_frac_chars_dupe_10grams": 0.18420572578907013, "rps_doc_frac_chars_dupe_5grams": 0.33622103929519653, "rps_doc_frac_chars_dupe_6grams": 0.29497233033180237, "rps_doc_frac_chars_dupe_7grams": 0.23727886378765106, "rps_doc_frac_chars_dupe_8grams": 0.2104383260011673, "rps_doc_frac_chars_dupe_9grams": 0.1957869827747345, "rps_doc_frac_chars_top_2gram": 0.01185481995344162, "rps_doc_frac_chars_top_3gram": 0.02772204950451851, "rps_doc_frac_chars_top_4gram": 0.03939449042081833, "rps_doc_books_importance": -3803.730712890625, "rps_doc_books_importance_length_correction": -3803.730712890625, "rps_doc_openwebtext_importance": -2356.923583984375, "rps_doc_openwebtext_importance_length_correction": -2356.923583984375, "rps_doc_wikipedia_importance": -932.1099853515625, "rps_doc_wikipedia_importance_length_correction": -932.1099853515625 }, "fasttext": { "dclm": 0.01936882920563221, "english": 0.8297948837280273, "fineweb_edu_approx": 2.458381175994873, "eai_general_math": 0.33614546060562134, "eai_open_web_math": 0.14497298002243042, "eai_web_code": 0.22784441709518433 } }
{ "free_decimal_correspondence": { "primary": { "code": "005.74", "labels": { "level_1": "General works, books and libraries, information sciences", "level_2": "", "level_3": "Computer programming" } }, "secondary": { "code": "005.1", "labels": { "level_1": "General works, books and libraries, information sciences", "level_2": "", "level_3": "Computer programming" } } }, "bloom_cognitive_process": { "primary": { "code": "3", "label": "Apply" }, "secondary": { "code": "2", "label": "Understand" } }, "bloom_knowledge_domain": { "primary": { "code": "3", "label": "Procedural" }, "secondary": { "code": "2", "label": "Conceptual" } }, "document_type_v1": { "primary": { "code": "3", "label": "Reference/Encyclopedic/Educational" }, "secondary": { "code": "-1", "label": "Abstain" } }, "extraction_artifacts": { "primary": { "code": "3", "label": "Irrelevant Content" }, "secondary": { "code": "0", "label": "No Artifacts" } }, "missing_content": { "primary": { "code": "4", "label": "Missing Images or Figures" }, "secondary": { "code": "0", "label": "No missing content" } }, "document_type_v2": { "primary": { "code": "23", "label": "Tutorial" }, "secondary": { "code": "8", "label": "Documentation" } }, "reasoning_depth": { "primary": { "code": "2", "label": "Basic Reasoning" }, "secondary": { "code": "3", "label": "Intermediate Reasoning" } }, "technical_correctness": { "primary": { "code": "4", "label": "Highly Correct" }, "secondary": { "code": "3", "label": "Mostly Correct" } }, "education_level": { "primary": { "code": "2", "label": "High School Level" }, "secondary": { "code": "3", "label": "Undergraduate Level" } } }
672f1e42c33a7f9846924a2431ea77df
-217,503,672,942,531,650
Reset Windows 7 Password Hi there, so you forgot Windows 7 password, and now you are wondering how to unlock your computer? You are in the right place, and to comfort you up a little bit, you are not unique on this matter. There are a lot of people that for some reason forget the login password on their […] Continue reading about How to Unlock Computer When You Forgot Windows 7 Password
{ "url": "http://www.recover-outlook-password.com/tag/reset-windows-7-password/", "source_domain": "www.recover-outlook-password.com", "snapshot_id": "crawl=CC-MAIN-2017-39", "warc_metadata": { "Content-Length": "24012", "Content-Type": "application/http; msgtype=response", "WARC-Block-Digest": "sha1:VECKSKX5XXQOY6ZZYSTBGIMAJ64G6OEJ", "WARC-Concurrent-To": "<urn:uuid:d369662e-8a97-4d72-9c4f-0403fda2766e>", "WARC-Date": "2017-09-24T22:57:27Z", "WARC-IP-Address": "166.62.1.1", "WARC-Identified-Payload-Type": "text/html", "WARC-Payload-Digest": "sha1:233NTGZ5JPUAKDRSVVAMPZEYK6I6Q2TM", "WARC-Record-ID": "<urn:uuid:c042993b-f4e7-4d52-8eee-a2b5fbf2a214>", "WARC-Target-URI": "http://www.recover-outlook-password.com/tag/reset-windows-7-password/", "WARC-Truncated": null, "WARC-Type": "response", "WARC-Warcinfo-ID": "<urn:uuid:c01e9ec4-40c7-47cd-a380-2e0b2d536cf0>" }, "warc_info": "robots: classic\r\nhostname: ip-10-236-162-53.ec2.internal\r\nsoftware: Nutch 1.6 (CC)\r\nisPartOf: CC-MAIN-2017-39\r\noperator: Common Crawl Admin\r\ndescription: Wide crawl of the web for September 2017\r\npublisher: Common Crawl\r\nformat: WARC File Format 1.0\r\nconformsTo: http://bibnum.bnf.fr/WARC/WARC_ISO_28500_version1_latestdraft.pdf" }
{ "line_start_idx": [ 0, 25, 26, 310, 311 ], "line_end_idx": [ 25, 26, 310, 311, 391 ] }
{ "red_pajama_v2": { "ccnet_original_length": 391, "ccnet_original_nlines": 4, "rps_doc_curly_bracket": 0, "rps_doc_ldnoobw_words": 0, "rps_doc_lorem_ipsum": 0, "rps_doc_stop_word_fraction": 0.4556961953639984, "rps_doc_ut1_blacklist": 0, "rps_doc_frac_all_caps_words": 0, "rps_doc_frac_lines_end_with_ellipsis": 0, "rps_doc_frac_no_alph_words": 0.1265822798013687, "rps_doc_frac_unique_words": 0.6438356041908264, "rps_doc_mean_word_length": 4.232876777648926, "rps_doc_num_sentences": 3, "rps_doc_symbol_to_word_ratio": 0.012658230029046535, "rps_doc_unigram_entropy": 3.684910535812378, "rps_doc_word_count": 73, "rps_doc_frac_chars_dupe_10grams": 0, "rps_doc_frac_chars_dupe_5grams": 0.16181230545043945, "rps_doc_frac_chars_dupe_6grams": 0, "rps_doc_frac_chars_dupe_7grams": 0, "rps_doc_frac_chars_dupe_8grams": 0, "rps_doc_frac_chars_dupe_9grams": 0, "rps_doc_frac_chars_top_2gram": 0.0776699036359787, "rps_doc_frac_chars_top_3gram": 0.1553398072719574, "rps_doc_frac_chars_top_4gram": 0.11003235727548599, "rps_doc_books_importance": -33.631900787353516, "rps_doc_books_importance_length_correction": -33.631900787353516, "rps_doc_openwebtext_importance": -17.74225616455078, "rps_doc_openwebtext_importance_length_correction": -17.74225616455078, "rps_doc_wikipedia_importance": -24.828746795654297, "rps_doc_wikipedia_importance_length_correction": -24.828746795654297 }, "fasttext": { "dclm": 0.08317548036575317, "english": 0.9342802166938782, "fineweb_edu_approx": 0.9595798850059509, "eai_general_math": 0.0022573499009013176, "eai_open_web_math": 0.2066320776939392, "eai_web_code": -0.000006790000043110922 } }
{ "free_decimal_correspondence": { "primary": { "code": "005.462", "labels": { "level_1": "General works, books and libraries, information sciences", "level_2": "", "level_3": "Computer programming" } }, "secondary": { "code": "004.67", "labels": { "level_1": "General works, books and libraries, information sciences", "level_2": "", "level_3": "Computers and Computer science" } } }, "bloom_cognitive_process": { "primary": { "code": "3", "label": "Apply" }, "secondary": { "code": "2", "label": "Understand" } }, "bloom_knowledge_domain": { "primary": { "code": "3", "label": "Procedural" }, "secondary": { "code": "2", "label": "Conceptual" } }, "document_type_v1": { "primary": { "code": "3", "label": "Reference/Encyclopedic/Educational" }, "secondary": { "code": "-1", "label": "Abstain" } }, "extraction_artifacts": { "primary": { "code": "3", "label": "Irrelevant Content" }, "secondary": { "code": "-1", "label": "Abstain" } }, "missing_content": { "primary": { "code": "1", "label": "Truncated Snippets" }, "secondary": { "code": "-1", "label": "Abstain" } }, "document_type_v2": { "primary": { "code": "23", "label": "Tutorial" }, "secondary": { "code": "21", "label": "Customer Support" } }, "reasoning_depth": { "primary": { "code": "2", "label": "Basic Reasoning" }, "secondary": { "code": "1", "label": "No Reasoning" } }, "technical_correctness": { "primary": { "code": "6", "label": "Not Applicable/Indeterminate" }, "secondary": { "code": "3", "label": "Mostly Correct" } }, "education_level": { "primary": { "code": "1", "label": "General Audience" }, "secondary": { "code": "2", "label": "High School Level" } } }
672f1e42c33a7f9846924a2431ea77df
-7,161,694,419,928,915,000
Wrong pocket TMs and HMs From Glitch City Wiki Jump to navigation Jump to search PRAMA Initiative a également une page sur Wrong pocket TMs and HMs. Wrong pocket TMs and HMs are TMs and HMs in Pokémon Gold, Silver and Crystal in the bag for when they are outside of the TM/HM pocket (which is normally impossible without glitches). These glitch items when used will result in unintended behavior and/or arbitrary code execution, depending on which TM or HM was used. Examples Hex ID Dec ID Item Version Use CE 206 TM15 Crystal (Japanese) Executes arbitrary code from FA10 (Echo RAM for DA10, which is somewhere in the expanded Balls pocket. If "(item) x195", "Star Piece x 252" (represents C3 84 FB - a jump to FB84, box 4 name character 2) are stored at DA0F, then this will serve as a bootstrap code to stored box Pokémon names. CE 206 TM15 Crystal (English) Executes arbitrary code from FA10 (Echo RAM for DA10, which is somewhere in the expanded Balls pocket. D0 208 TM17 Crystal (English) Executes arbitrary code from FA47 (Echo RAM for DA47, which is somewhere in the expanded Balls pocket. D0 208 TM17 Gold/Silver (English) Executes arbitrary code from FA47 (Echo RAM for DA47, which is the caught data for party Pokémon 1. The player can make a jump to somewhere more useful (via C3 yy xx), using Pokémon 1 as a slide Pokémon if necessary. D4 212 TM21 Crystal (English) Executes arbitrary code from FA69 (Echo RAM for DA69, which is somewhere in the expanded Balls pocket; like TM25. D8 216 TM25 Crystal (English) Executes arbitrary code from FA69 (Echo RAM for DA69, which is somewhere in the expanded Balls pocket; like TM21. D8 216 TM25 Gold/Silver (English) Executes arbitrary code from FA6A (Echo RAM for DA6A, and DA6A is the least significant Defense EV byte of the second Pokémon in the party. The Speed EVs word follows, and the player can make a jump to somewhere more useful (via C3 yy xx). This may be possible without trading, but getting the exact EVs would be tedious and it would be hard to keep track of them without a memory viewer. Through arbitrary code execution in Pokémon Red, Blue, and Yellow, the second Pokémon may be manipulated to have Defense and Speed EV values that the player desires. E1 225 TM33 Crystal (English) Executes arbitrary code from F418 in Echo RAM (essentially D418); the third character of the fifth enemy Pokémon's nickname. Through arbitrary code execution in Pokémon Red, Blue, and Yellow, the player may rename this Pokémon to include a jump instruction (e.g. to C3 F2 D8 bytes which reads jp D8F2, i.e. jumps to the first item in the item storage system). EF 239 TM47 Gold/Silver (English) Allows the player to offset their position on the map, as documented here. How to obtain (bad clone glitch for non-Japanese versions) 1. Get your bad clone. If you have access to an emulator, you can try a breakpoint method for BGB described in the bad clone glitch article (note you must use box 4 and five Pokémon in the box for this). If you don't want to obtain a bad clone, then theoretically you can use the Hall of Fame SRAM glitch or an international Generation II trade (Japanese and English versions) to get a bad clone. 2. Put a Machop with Seismic Toss as move 1 in the party (learned at Level 19). 3. Stabilize your bad clone to get a ?????. Put this at the top of the party and the Machop after it. 4. Move a Pokémon above ????? to get 7 Pokémon. 5. Take the second Pokémon (a ????? that changed from 00 to FF I think) into the Day Care. 6. Deposit the first Pokémon. Machop should now be in the first slot after depositing it. 7. Deposit the second Pokémon. Machop should now be holding a Mystery Egg. 8. Keep depositing the first Pokémon and fix the party. Don't take Machop's item yet. 9. Clone Machop to obtain another Machop holding a Mystery Egg. 10. Take both Mystery Eggs. The key items should now just contain two Mystery Egg stacks. 11. Don't swap them yet, and have in the Balls pocket exactly: Great Ball x1, Poké Ball x2. 12. Swap the two Mystery Eggs. You should now have Master Ball x5 and Ultra Ball x255. 13. Toss from the Ultra Ball stack to equal the TM/HM (which will be forced in the Balls pocket) ID you desire. For example, tossing 47 to get 208 will give TM17, an arbitrary code execution item. 14. Swap the Master Ball x5 with the Ultra Ball x(amount) 15. Deposit a Mystery Egg you should have, and then withdraw it again. This will result in you have the item you desire in slot 1. It doesn't just work with TMs/HMs, for example you can obtain a wrong pocket Rare Candy (quantity of 32) that likely has infinite use instead. However it's a good idea to get arbitrary code execution set up first, and then you can use box names to obtain other wrong pocket items like HM01. How to obtain (Coin Case glitch) This section only applies to Pokémon Gold and Silver. Perform the following codes for Coin Case glitch with the move 1 Return Quagsire in slot 4 holding a TM02 setup: 1. FMK's one-off code: Box 1 : Ap0w'vA55 (XOR A; OR b6; SUB 0x80) Box 2 : é'm2p'v7'v'd Box 3 : éA355555 (LD [80f9], A) Box 4+: 55555555 (Safe filler code) Box 13: 5555péD9 (XOR A; LD [83ff], A) Box 14: 'l'lA'lx'd55 (POP DE; POP DE; INC SP; POP DE; OR A; RET NC) 2. Torchickens' all TMs/HMs code: Requires above one-off code to have been used first: Box 1: Ap'vCé025 Box 2: 'vj'vué♀25 Box 3: 'v.é32p'v9r Box 4: é22pé425 Box 5: 'vué62'v 5 (there is a space after the 'v and before the 5) Box 6: é52'v:é72 Box 7: 55♀55555 Box 8-12: 55555555 Box 13, 14: Same as before, don't change them. 3. Expand the Balls pocket to access stored PC items (including TMs/HMs outside of their pocket) Requires one-off code to have been used first: Box 1: Ap09é6♀5 Box 2-12: 55555555 Box 13, 14: Same as before, don't change them. Upon depositing the TM/HM of your choice, it should appear in the expanded Balls pocket. See also • TMHMDex: Project documenting wrong pocket TMs and HMs
{ "url": "https://glitchcity.wiki/wiki/Wrong_pocket_TMs_and_HMs", "source_domain": "glitchcity.wiki", "snapshot_id": "CC-MAIN-2024-22", "warc_metadata": { "Content-Length": "40593", "Content-Type": "application/http; msgtype=response", "WARC-Block-Digest": "sha1:5H7MHRWJRU5IRXX5DEXASVY7HBEKV3UH", "WARC-Concurrent-To": "<urn:uuid:3e7a1500-ccb4-4235-9c34-7788c885f29c>", "WARC-Date": "2024-05-18T10:15:31Z", "WARC-IP-Address": "38.46.223.205", "WARC-Identified-Payload-Type": "text/html", "WARC-Payload-Digest": "sha1:TPUDZFRXVEDCG25YRELQ2MLZ7CLZRCOV", "WARC-Record-ID": "<urn:uuid:3cf4be10-3db9-4740-b12a-af4bc540761a>", "WARC-Target-URI": "https://glitchcity.wiki/wiki/Wrong_pocket_TMs_and_HMs", "WARC-Truncated": null, "WARC-Type": "response", "WARC-Warcinfo-ID": "<urn:uuid:bf09ba68-b7a2-40d5-a2e6-53d523c231f8>" }, "warc_info": "isPartOf: CC-MAIN-2024-22\r\npublisher: Common Crawl\r\ndescription: Wide crawl of the web for May 2024\r\noperator: Common Crawl Admin ([email protected])\r\nhostname: ip-10-67-67-29\r\nsoftware: Apache Nutch 1.19 (modified, https://github.com/commoncrawl/nutch/)\r\nrobots: checked via crawler-commons 1.5-SNAPSHOT (https://github.com/crawler-commons/crawler-commons)\r\nformat: WARC File Format 1.1\r\nconformsTo: https://iipc.github.io/warc-specifications/specifications/warc-format/warc-1.1/" }
{ "line_start_idx": [ 0, 25, 26, 48, 82, 150, 151, 334, 335, 470, 471, 480, 481, 512, 836, 969, 1102, 1353, 1497, 1641, 2230, 2620, 2729, 2730, 2789, 2790, 2994, 2995, 3188, 3189, 3269, 3270, 3372, 3373, 3421, 3422, 3513, 3514, 3604, 3605, 3680, 3681, 3767, 3768, 3832, 3833, 3923, 3924, 4016, 4017, 4104, 4105, 4302, 4303, 4361, 4362, 4433, 4434, 4785, 4786, 4819, 4820, 4987, 4988, 4989, 5012, 5013, 5056, 5057, 5078, 5079, 5111, 5112, 5148, 5149, 5188, 5189, 5257, 5258, 5292, 5293, 5346, 5347, 5364, 5365, 5383, 5384, 5403, 5404, 5420, 5421, 5488, 5489, 5506, 5507, 5523, 5524, 5543, 5544, 5591, 5592, 5689, 5690, 5737, 5738, 5754, 5755, 5774, 5775, 5822, 5823, 5912, 5913, 5922, 5923 ], "line_end_idx": [ 25, 26, 48, 82, 150, 151, 334, 335, 470, 471, 480, 481, 512, 836, 969, 1102, 1353, 1497, 1641, 2230, 2620, 2729, 2730, 2789, 2790, 2994, 2995, 3188, 3189, 3269, 3270, 3372, 3373, 3421, 3422, 3513, 3514, 3604, 3605, 3680, 3681, 3767, 3768, 3832, 3833, 3923, 3924, 4016, 4017, 4104, 4105, 4302, 4303, 4361, 4362, 4433, 4434, 4785, 4786, 4819, 4820, 4987, 4988, 4989, 5012, 5013, 5056, 5057, 5078, 5079, 5111, 5112, 5148, 5149, 5188, 5189, 5257, 5258, 5292, 5293, 5346, 5347, 5364, 5365, 5383, 5384, 5403, 5404, 5420, 5421, 5488, 5489, 5506, 5507, 5523, 5524, 5543, 5544, 5591, 5592, 5689, 5690, 5737, 5738, 5754, 5755, 5774, 5775, 5822, 5823, 5912, 5913, 5922, 5923, 5980 ] }
{ "red_pajama_v2": { "ccnet_original_length": 5980, "ccnet_original_nlines": 114, "rps_doc_curly_bracket": 0, "rps_doc_ldnoobw_words": 2, "rps_doc_lorem_ipsum": 0, "rps_doc_stop_word_fraction": 0.3159797489643097, "rps_doc_ut1_blacklist": 0, "rps_doc_frac_all_caps_words": 0.07302965223789215, "rps_doc_frac_lines_end_with_ellipsis": 0, "rps_doc_frac_no_alph_words": 0.2494577020406723, "rps_doc_frac_unique_words": 0.36915889382362366, "rps_doc_mean_word_length": 4.2943925857543945, "rps_doc_num_sentences": 74, "rps_doc_symbol_to_word_ratio": 0, "rps_doc_unigram_entropy": 5.383800983428955, "rps_doc_word_count": 1070, "rps_doc_frac_chars_dupe_10grams": 0.18128399550914764, "rps_doc_frac_chars_dupe_5grams": 0.2535364627838135, "rps_doc_frac_chars_dupe_6grams": 0.22002175450325012, "rps_doc_frac_chars_dupe_7grams": 0.1956474334001541, "rps_doc_frac_chars_dupe_8grams": 0.1956474334001541, "rps_doc_frac_chars_dupe_9grams": 0.1956474334001541, "rps_doc_frac_chars_top_2gram": 0.016322089359164238, "rps_doc_frac_chars_top_3gram": 0.03656148165464401, "rps_doc_frac_chars_top_4gram": 0.0435255691409111, "rps_doc_books_importance": -553.3113403320312, "rps_doc_books_importance_length_correction": -553.3113403320312, "rps_doc_openwebtext_importance": -297.91436767578125, "rps_doc_openwebtext_importance_length_correction": -297.91436767578125, "rps_doc_wikipedia_importance": -208.21078491210938, "rps_doc_wikipedia_importance_length_correction": -208.21078491210938 }, "fasttext": { "dclm": 0.19954776763916016, "english": 0.8748235106468201, "fineweb_edu_approx": 1.280424952507019, "eai_general_math": 0.27466946840286255, "eai_open_web_math": 0.39464521408081055, "eai_web_code": 0.07004767656326294 } }
{ "free_decimal_correspondence": { "primary": { "code": "004.656", "labels": { "level_1": "General works, books and libraries, information sciences", "level_2": "", "level_3": "Computers and Computer science" } }, "secondary": { "code": "794.8", "labels": { "level_1": "Arts", "level_2": "Amusements and Recreation", "level_3": "" } } }, "bloom_cognitive_process": { "primary": { "code": "3", "label": "Apply" }, "secondary": { "code": "2", "label": "Understand" } }, "bloom_knowledge_domain": { "primary": { "code": "3", "label": "Procedural" }, "secondary": { "code": "2", "label": "Conceptual" } }, "document_type_v1": { "primary": { "code": "3", "label": "Reference/Encyclopedic/Educational" }, "secondary": { "code": "-1", "label": "Abstain" } }, "extraction_artifacts": { "primary": { "code": "0", "label": "No Artifacts" }, "secondary": { "code": "3", "label": "Irrelevant Content" } }, "missing_content": { "primary": { "code": "0", "label": "No missing content" }, "secondary": { "code": "-1", "label": "Abstain" } }, "document_type_v2": { "primary": { "code": "23", "label": "Tutorial" }, "secondary": { "code": "8", "label": "Documentation" } }, "reasoning_depth": { "primary": { "code": "4", "label": "Advanced Reasoning" }, "secondary": { "code": "3", "label": "Intermediate Reasoning" } }, "technical_correctness": { "primary": { "code": "4", "label": "Highly Correct" }, "secondary": { "code": "3", "label": "Mostly Correct" } }, "education_level": { "primary": { "code": "4", "label": "Graduate/Expert Level" }, "secondary": { "code": "3", "label": "Undergraduate Level" } } }
672f1e42c33a7f9846924a2431ea77df
-3,918,437,573,454,677,000
Generating UPC Check Digits in Bulk Morovia offers a check digit calculator online at http://www.morovia.com/education/utility/upc-ean.asp. With this utility, you can calculate check digit by entering your numbers - no matter it is a UPC, EAN, SSCC, GTIN or BLN.  It is a wonderful utility and has been running for many years since Morovia started. Now, Morovia added another online utility to allow you to calculate check digits in bulk. This is quite useful when you have a range of number. The typical case is the UPC-A numbers you own. For example, you could have a GS1 prefix 123456789, which leaves you 2 digits for your item number. And you want a list of final numbers with check digits.  Repeating 100 times is tedious work on a web page. Now no more. Go to http://www.morovia.com/bulk-check-digit-calculation/index.php and enter 123456789??. Note that the two question marks repsent any digits in the last two positions.  Click on Submit and the web page send out the answer - 123456789005 123456789012 123456789029 123456789036 123456789043 123456789050 123456789067 123456789074 123456789081 123456789098 123456789104 123456789111 123456789128 123456789135 123456789142 123456789159 123456789166 123456789173 123456789180 123456789197 123456789203 123456789210 123456789227 123456789234 123456789241 123456789258 123456789265 123456789272 123456789289 123456789296 123456789302 123456789319 123456789326 123456789333 123456789340 123456789357 123456789364 123456789371 123456789388 123456789395 123456789401 123456789418 123456789425 123456789432 123456789449 123456789456 123456789463 123456789470 123456789487 123456789494 123456789500 123456789517 123456789524 123456789531 123456789548 123456789555 123456789562 123456789579 123456789586 123456789593 123456789609 123456789616 123456789623 123456789630 123456789647 123456789654 123456789661 123456789678 123456789685 123456789692 123456789708 123456789715 123456789722 123456789739 123456789746 123456789753 123456789760 123456789777 123456789784 123456789791 123456789807 123456789814 123456789821 123456789838 123456789845 123456789852 123456789869 123456789876 123456789883 123456789890 123456789906 123456789913 123456789920 123456789937 123456789944 123456789951 123456789968 123456789975 123456789982 123456789999 All 100 numbers are listed in order. Quite neat. Create QRCode Symbols Free Online Morovia has just published a new free utility - online QR Code Maker. This is a not a demo version as it does not add watermark or other text. We intended to make it free online permanently. Visit it today. I made  a QR symbol with 238 characters encoded. Want to know what is inside? Print the page and Scan it to find out -:) example QR code image background image Recently a customer asked us a question - I have an emergency task. I have a spreadsheet with all product codes (tens of thousands of them). I need to print barcodes for them. I do not have an advanced IT system for this task. I want to print them one by one in Avery label papers. Do you have software doing that? Small companies often have needs to print barcodes.  But not many of them have dedicated IT systems for the purpose, or in-house programmers that can come up with a solution at any time. Some examples - • You want to add a barcode into the invoices, so that you can find the invoice quickly. Your current software that prints invoices, Quickbooks, does not have the capability. • You run a retail store and occasionally you bought some items with no UPC barcode on the box. You want to print UPC barcodes and stick them on the boxes by youself. • You want to stick barcodes with special serial number encoded. In this way, you can check if the item returned by your customer was indeed sold from your store. The first option is to use barcode fonts. A typical solution is to add a column in the spreadsheet that holds the barcode strings, and use Word Mail Merge to get the barcodes into the label paper (which is a Word document0. It does not require advanced programming knowledge. Unfortunately, many companies do not use VBA at all nowadays, and it seems to become a formidable task for many people. Fortunately, Monterey Barcode Creator has a “batch printing” feature.  You do not need any programming knowledge at all to print barcodes into label papers with this program. Let’s assume that you have a speadsheet that lists all yor product codes in one column. You want to print sticks with barcodes on it, and each stick corresponds to one product code.  You will then print them to Avery Label Paper 3111 on a laser printer. After stickers are printed, you stick them one by one on the item box. CSV File First of all, Monterey Barcode Creator can’t read Excel file directly. You will have to export to a .csv file.  CSV is the abbreviation of Comma Separated Value, a format used to exchange data between software.  Each line is a record, and column values are separate by commas.  The below line is an example: 1003434323214, 0-20-ABC-2004, Product Item One You can generate a .csv file by choosing “Save As” and specify .csv as the format. As you can see from the example record, a line contain multiple columns. Columns are numbered from 1. Record your column number, you will need it soon. You can also use Copy/Paste to paste the whole column to Notepad, and save it into a text file. This file can be thought as a special type of .CSV, as each record has only one column. Adjusting Barcode Properties Now start Monterey Barcode Creator.  The images created by Barcode Creator can contain three parts - barcode, human readable text and a text comment. We do not need comment here so we uncheck “comment” box. Change the barcode type to “Code128″. You might want to change other properties, such as BarHeight and NarrowBarWidth.  There are quite a number of properties that you can change. If you are interested, read the product manaul at http://mdn.morovia.com/manuals/mbc3/shared.properties_methods.php. Remember that all barcodes will have the same property set (except the barcode and human readable text). After you are satisfied with the barcode size, you can now print them out. From File Menu, choose Print. Batch Printing in Barcode Creator Print Dialog in Monterey Barcode Creator 3 Print Dialog in Monterey Barcode Creator 3 First, make sure that “Use Label Settings” is checked. If it is not checked, you will not see the bottom portion of the dialog. Use “Label Products” and “Product Number” to select the label paper you print on.  The program has most paper listed, but if you can not find the one you have, you can enter it into the database by selecting “New Label…”.  After you are done with the label paper selection, Click on Print Batch. print-batch-datafile-setup Batch Setup (Data File Mode) Monterey Barcode Creator supports two batch modes - Number Sequence and Data File. Number Sequence mode allows you to specify a sequence. For our work, we will use Data File mode. Click on the Data File to switch, and right side of the dialog will change accordingly. You will supply two parameters: 1. File Path. This is the path of your CSV or Text file. You can use the small “” button to navigate to the file, or type it by yourself. 2. Column. The cardinal of the column that the data is from.  If  you pasted the data to a text file, you only have one column and it cardinal is 1. Click OK to dismiss the setup dialog, and click OK on the Print Dialog to print out. If you have hundreds of thounds of records to print, it may take quite some time. The data file we used for this article can be downloaded here. And the print result is here. Creating Barcode Strings Using PHP If you are creating barcode using barcode fonts, you should already know that you can not just type your number and format with a font. Extra characters such as start/stop characters, and often times checksum characters are required to be present in the barcode.  Some formats such as Code 39 and Code25, are easy. But formats like Code 128 are quite difficult. To easy the difficulty creating barcode strings, Morovia provides source code, Windows DLLs and COM classes that you can integrate in your applications. For an overview on these utilities, visit http://www.morovia.com/font/support/font-tool.asp. Many people asked the question - do you have an example with PHP? It turns out that it is not difficult to do it in PHP at all. The following code creates a Code128 barcode string (which becomes a Code128 barcode after being formatted with Morovia Code128 font): <?php $encoder = new COM("cruflMorovia.Barcode"); $result = $encoder->Code128Ex(”PN-NUMBER-13334432″); echo $result; ?> Note that the Prog ID of the encoder COM object is cruflMorovia.Barcode. You can write code for other barcode types. For example, for EAN-128 barcodes, just replace Code128Ex with EAN128Ex. GS1 has published a new coupon format to be used in North America, called DataBar Coupon. This coupon code can encode. The Application Identifier (AI) for this code is 8110, and the value can be from 25 to 39 digits long. You can use Morovia DataBar Fontware to create such a coupon. You can create the code in drawing programs such as Adobe Illustrator, or Word Processing programs such as Microsoft Word. Adobe Illustrator users might want to read this article for a tip on adjusting line heights. 1. The coupon code requires the company prefix and offer code to be printed on top of the barcode. Started with a text box and enter this information on the top. word-textbox 2. Open Morovia DataBar Font Encoder GUI. 3. Choose DataBar Expanded Stacked in the Barcode Type box, and enter the coupon code in the Data to Be Encoded box. Note that you must include both AI and data, and AI must be enclosed with parentheses. Make sure that the Symbols Per Row is set to 6. DataBar Encoder GUI DataBar Encoder GUI 4. Press Make Barcode button and the barcode will appear below. 5. Press Copy to Clipboard to transfer the barcode into clipboard. 6. In Microsoft Word, press Paste to Paste the barcode into the text box. Right click on the barcode and choose Ignore Once to disable spell checking. word-databar-coupon-barcode 7. Now you created the coupon code. According to GS1, before January 2010 you should also include the UPC-A coupon symbol. After that date the UPC-A coupon is no longer necessary. In Morovia DataBar Fontware’s manual, it explains how to overcome the line gap issues in several programs, such as Microsoft Visio. It did not mention Adobe Illustrator, which alos has the same issue. Adobe Illustrator does not present a menu item for you to adjust line height.  That is at least what I found on my computer (Illustrator CS2). Instead, you have to press Ctrl+T to pop up the character palette, and make the line height the same as the font size you selected. illustrator-databar-problem In the character palette, make sure that the line height is the same as the point size. The gap then disappears. illustrator-databar-good Conclusion - in Adobe illustrator you can adjust the line height. To do so, press CTRL+T to pop up character palette and change the line height to be the same as the font size. Free Online Barcode Generator You can now create barcode easily from your browser. Morovia has just announced the availability of Online Barcode Generator. Again the address is http://www.morovia.com/free-online-barcode-generator/. The new coupon code (AI 8110) is required to be printed in GS1 DataBar Expanded Stacked. Its data can be quite long. In fact the length is the longest we have ever seen in GS1-128 - from 25 to 39 digits. GS1 DataBar Expanded Stacked can have multiple rows, depending on the data and the value of SegmentPerRow. The value of SegmentsPerRow can be anywhere between 2 and 22. 22 also produces one-row barcode. In some cases, it is desirable to find out the range of SegmentPerRow, based on the data encoded and the number of rows desired. For example, our coupon codes can be between 25 to 39 digits, but we want all barcodes created two rows. To find out the relationship among the data length, segment per row and the number of rows in the barcode created, we wrote a simple perl program. It calls the encoder DLL included in Morovia DataBar Fontware to get the barcode string, then parses the string to find out how many rows in the result. It generates randomized data as input (based on DataBar Spec, it is not necessary as numeric data with the same length should always create the same length barcodes, all things being equal). It increases the number of digits one by one, and calculates using all possible SegmentsperRow values. Warning: if you are running the trial version, do not run this program - you will have to press OK button hundreds of times to complete the program (each call pops up a ‘trial notice’ message box). We found that the results can be divided into five groups - 25 digits, 26-32, 33-39, 40-46 and 47-49. It seems that each segment encodes 6 digits. Note - the digits we talked here refer to the coupon code, not including the AI portion. coupon code length vs. row For small manufacturers with dedicated IT stuff, it is often a pain to write a trivial program. Numbers such as UPC-A, EAN-13, SCC-14, SSCC-18 and Bill of Lading (BLN) all require a modulo 10 check digit at the end. Although the algorithm is simple, it is easy to make mistaken if done by human. Fortunately you can now calculate check digits online using this free web utility. All you need is an Internet connection. UPC Coupon Value Codes The value codes are the two digits before the last check digit. They have fixed meanings as defined by GS1 organization. The table below lists the meanings effective as of writing.  Occasionally GS1 changed the meanings of some. Fortunately, The changes were rare and the last update happened in year 200. coupon-values-upc
{ "url": "http://www.barcodeschool.com/page/3/", "source_domain": "www.barcodeschool.com", "snapshot_id": "crawl=CC-MAIN-2022-27", "warc_metadata": { "Content-Length": "36184", "Content-Type": "application/http; msgtype=response", "WARC-Block-Digest": "sha1:CJTQZ3TL5X7JZ3DO4FQ232IKSQNXUQ5A", "WARC-Concurrent-To": "<urn:uuid:1861b8a1-1e3a-42c5-aff8-ebfdacccd2c8>", "WARC-Date": "2022-07-04T06:27:25Z", "WARC-IP-Address": "74.208.159.198", "WARC-Identified-Payload-Type": "application/xhtml+xml", "WARC-Payload-Digest": "sha1:3TTKQVX6VVWLRDICPJEM2YXX23AKKSYE", "WARC-Record-ID": "<urn:uuid:48aa47cb-a16c-459d-90f3-5bd0173b579d>", "WARC-Target-URI": "http://www.barcodeschool.com/page/3/", "WARC-Truncated": null, "WARC-Type": "response", "WARC-Warcinfo-ID": "<urn:uuid:8215f2b9-afc1-4d3c-a0c2-a2d657c1c0d0>" }, "warc_info": "isPartOf: CC-MAIN-2022-27\r\npublisher: Common Crawl\r\ndescription: Wide crawl of the web for June/July 2022\r\noperator: Common Crawl Admin ([email protected])\r\nhostname: ip-10-67-67-253\r\nsoftware: Apache Nutch 1.18 (modified, https://github.com/commoncrawl/nutch/)\r\nrobots: checked via crawler-commons 1.3-SNAPSHOT (https://github.com/crawler-commons/crawler-commons)\r\nformat: WARC File Format 1.1\r\nconformsTo: https://iipc.github.io/warc-specifications/specifications/warc-format/warc-1.1/" }
{ "line_start_idx": [ 0, 36, 37, 350, 351, 763, 764, 990, 991, 1004, 1017, 1030, 1043, 1056, 1069, 1082, 1095, 1108, 1121, 1134, 1147, 1160, 1173, 1186, 1199, 1212, 1225, 1238, 1251, 1264, 1277, 1290, 1303, 1316, 1329, 1342, 1355, 1368, 1381, 1394, 1407, 1420, 1433, 1446, 1459, 1472, 1485, 1498, 1511, 1524, 1537, 1550, 1563, 1576, 1589, 1602, 1615, 1628, 1641, 1654, 1667, 1680, 1693, 1706, 1719, 1732, 1745, 1758, 1771, 1784, 1797, 1810, 1823, 1836, 1849, 1862, 1875, 1888, 1901, 1914, 1927, 1940, 1953, 1966, 1979, 1992, 2005, 2018, 2031, 2044, 2057, 2070, 2083, 2096, 2109, 2122, 2135, 2148, 2161, 2174, 2187, 2200, 2213, 2226, 2239, 2252, 2265, 2278, 2291, 2292, 2341, 2342, 2376, 2377, 2584, 2585, 2706, 2707, 2729, 2730, 2747, 2748, 2790, 2791, 3064, 3065, 3268, 3269, 3446, 3615, 3780, 3781, 4177, 4178, 4353, 4354, 4679, 4680, 4689, 4690, 4998, 4999, 5046, 5047, 5130, 5131, 5283, 5284, 5468, 5469, 5498, 5499, 6003, 6004, 6109, 6110, 6215, 6216, 6250, 6251, 6294, 6295, 6338, 6339, 6763, 6764, 6791, 6792, 6821, 6822, 7122, 7123, 7263, 7414, 7415, 7582, 7583, 7676, 7677, 7712, 7713, 8228, 8229, 8322, 8323, 8586, 8587, 8593, 8637, 8638, 8691, 8692, 8706, 8709, 8710, 8900, 8901, 9123, 9124, 9402, 9403, 9567, 9584, 9628, 9882, 9883, 9907, 9908, 9932, 9933, 9999, 10068, 10221, 10253, 10435, 10436, 10637, 10638, 10913, 10914, 10942, 10943, 11056, 11057, 11082, 11083, 11260, 11261, 11291, 11292, 11418, 11419, 11495, 11496, 11903, 11904, 12138, 12139, 12931, 12932, 13168, 13169, 13196, 13197, 13616, 13617, 13640, 13641, 13947, 13948 ], "line_end_idx": [ 36, 37, 350, 351, 763, 764, 990, 991, 1004, 1017, 1030, 1043, 1056, 1069, 1082, 1095, 1108, 1121, 1134, 1147, 1160, 1173, 1186, 1199, 1212, 1225, 1238, 1251, 1264, 1277, 1290, 1303, 1316, 1329, 1342, 1355, 1368, 1381, 1394, 1407, 1420, 1433, 1446, 1459, 1472, 1485, 1498, 1511, 1524, 1537, 1550, 1563, 1576, 1589, 1602, 1615, 1628, 1641, 1654, 1667, 1680, 1693, 1706, 1719, 1732, 1745, 1758, 1771, 1784, 1797, 1810, 1823, 1836, 1849, 1862, 1875, 1888, 1901, 1914, 1927, 1940, 1953, 1966, 1979, 1992, 2005, 2018, 2031, 2044, 2057, 2070, 2083, 2096, 2109, 2122, 2135, 2148, 2161, 2174, 2187, 2200, 2213, 2226, 2239, 2252, 2265, 2278, 2291, 2292, 2341, 2342, 2376, 2377, 2584, 2585, 2706, 2707, 2729, 2730, 2747, 2748, 2790, 2791, 3064, 3065, 3268, 3269, 3446, 3615, 3780, 3781, 4177, 4178, 4353, 4354, 4679, 4680, 4689, 4690, 4998, 4999, 5046, 5047, 5130, 5131, 5283, 5284, 5468, 5469, 5498, 5499, 6003, 6004, 6109, 6110, 6215, 6216, 6250, 6251, 6294, 6295, 6338, 6339, 6763, 6764, 6791, 6792, 6821, 6822, 7122, 7123, 7263, 7414, 7415, 7582, 7583, 7676, 7677, 7712, 7713, 8228, 8229, 8322, 8323, 8586, 8587, 8593, 8637, 8638, 8691, 8692, 8706, 8709, 8710, 8900, 8901, 9123, 9124, 9402, 9403, 9567, 9584, 9628, 9882, 9883, 9907, 9908, 9932, 9933, 9999, 10068, 10221, 10253, 10435, 10436, 10637, 10638, 10913, 10914, 10942, 10943, 11056, 11057, 11082, 11083, 11260, 11261, 11291, 11292, 11418, 11419, 11495, 11496, 11903, 11904, 12138, 12139, 12931, 12932, 13168, 13169, 13196, 13197, 13616, 13617, 13640, 13641, 13947, 13948, 13965 ] }
{ "red_pajama_v2": { "ccnet_original_length": 13965, "ccnet_original_nlines": 249, "rps_doc_curly_bracket": 0, "rps_doc_ldnoobw_words": 0, "rps_doc_lorem_ipsum": 0, "rps_doc_stop_word_fraction": 0.3618108928203583, "rps_doc_ut1_blacklist": 0, "rps_doc_frac_all_caps_words": 0.02738226018846035, "rps_doc_frac_lines_end_with_ellipsis": 0, "rps_doc_frac_no_alph_words": 0.2037239968776703, "rps_doc_frac_unique_words": 0.32653963565826416, "rps_doc_mean_word_length": 4.943287372589111, "rps_doc_num_sentences": 185, "rps_doc_symbol_to_word_ratio": 0.00036509998608380556, "rps_doc_unigram_entropy": 5.731264114379883, "rps_doc_word_count": 2257, "rps_doc_frac_chars_dupe_10grams": 0, "rps_doc_frac_chars_dupe_5grams": 0.01918078027665615, "rps_doc_frac_chars_dupe_6grams": 0.01003853976726532, "rps_doc_frac_chars_dupe_7grams": 0.006453350186347961, "rps_doc_frac_chars_dupe_8grams": 0, "rps_doc_frac_chars_dupe_9grams": 0, "rps_doc_frac_chars_top_2gram": 0.010217799805104733, "rps_doc_frac_chars_top_3gram": 0.011831140145659447, "rps_doc_frac_chars_top_4gram": 0.0032266699708998203, "rps_doc_books_importance": -1359.510498046875, "rps_doc_books_importance_length_correction": -1359.510498046875, "rps_doc_openwebtext_importance": -834.1350708007812, "rps_doc_openwebtext_importance_length_correction": -834.1350708007812, "rps_doc_wikipedia_importance": -581.5333251953125, "rps_doc_wikipedia_importance_length_correction": -581.5333251953125 }, "fasttext": { "dclm": 0.07406502962112427, "english": 0.8433148860931396, "fineweb_edu_approx": 2.21132230758667, "eai_general_math": 0.6559915542602539, "eai_open_web_math": 0.25497907400131226, "eai_web_code": 0.3197975158691406 } }
{ "free_decimal_correspondence": { "primary": { "code": "004.6", "labels": { "level_1": "General works, books and libraries, information sciences", "level_2": "", "level_3": "Computers and Computer science" } }, "secondary": { "code": "658.8", "labels": { "level_1": "Industrial arts, Technology, and Engineering", "level_2": "Business", "level_3": "Management" } } }, "bloom_cognitive_process": { "primary": { "code": "3", "label": "Apply" }, "secondary": { "code": "2", "label": "Understand" } }, "bloom_knowledge_domain": { "primary": { "code": "3", "label": "Procedural" }, "secondary": { "code": "2", "label": "Conceptual" } }, "document_type_v1": { "primary": { "code": "3", "label": "Reference/Encyclopedic/Educational" }, "secondary": { "code": "4", "label": "Code/Software" } }, "extraction_artifacts": { "primary": { "code": "0", "label": "No Artifacts" }, "secondary": { "code": "3", "label": "Irrelevant Content" } }, "missing_content": { "primary": { "code": "0", "label": "No missing content" }, "secondary": { "code": "4", "label": "Missing Images or Figures" } }, "document_type_v2": { "primary": { "code": "23", "label": "Tutorial" }, "secondary": { "code": "8", "label": "Documentation" } }, "reasoning_depth": { "primary": { "code": "2", "label": "Basic Reasoning" }, "secondary": { "code": "3", "label": "Intermediate Reasoning" } }, "technical_correctness": { "primary": { "code": "4", "label": "Highly Correct" }, "secondary": { "code": "3", "label": "Mostly Correct" } }, "education_level": { "primary": { "code": "2", "label": "High School Level" }, "secondary": { "code": "1", "label": "General Audience" } } }
672f1e42c33a7f9846924a2431ea77df
-264,370,345,171,842,400
How to install Kali Linux in USB How to install Kali Linux in USB Image for post Hello guys in this story we are going to see about the installation procedure of Kali Linux in USB with live persistence. Follow each and every steps carefully to avoid unnecessary errors. Herewith i have enclosed a video tutorial, If you have any doubts please refer into that. STEP 1: Download Kali Linux ISO Image from the official Kali Linux website STEP 2: Then Download Power iso, and create a bootable USB. STEP 3: Now your are ready for the installation, Reboot your device and enter into Boot Menu. STEP 4: choose the Bootable USB stick and you will see the Kali Linux installation option, In that choose Live system ( 1st option ). NOTE: While entering to Live Kali Linux generally it won?t ask for username and password. In case if it asked then Type ? root ? as username and ? toor ? as password. STEP 5: Now open up your terminal and type the following commands one by one. ? fdisk -l #the above command will list out all the disk that are available in your Laptop or PC . Now note down the USB name ( mostly it will be /dev/sdb ). fdisk /dev/sdb #now it will ask you for command, type n and hit enter Now press enter 4 times, and type w and hit enter. #we have successfully created the partition. fdisk -l #now check whether the partition has been created or not ( copy the name of the partition that you created now eg: /dev/sdb3 ) mkfs.ext4 -L persistence /dev/sdb3 #wait until it complete and after it completes type, e2label /dev/sdb3 persistence #the above command will label the partition with the name persistence. mkdir -p /mnt/hackison (you can use any name ) mount /dev/sdb3 /mnt/hackison echo ?/ union? > /mnt/hackison/persistence.conf umount /dev/sdb3 exit #now reboot your laptop or Pc. and enter into your boot menu STEP 6: In the Kali Linux Boot Menu choose the 4th option Live system ( Persistence ) STEP 7: Now Let?s check whether the persistence is working good or not. To do that just change your background or create a new folder. STEP 8: Now reboot your device and again enter into Live System ( Persistence) NOTE: Each and every time when you want to go to persistence mode you need to choose that option. Still if you have any issue please refer to the following video tutorial for more detailed explanation. Don?t forget to follow and subscribe?!!!! 9 No Responses Write a response
{ "url": "https://911weknow.com/how-to-install-kali-linux-in-usb", "source_domain": "911weknow.com", "snapshot_id": "CC-MAIN-2023-50", "warc_metadata": { "Content-Length": "43681", "Content-Type": "application/http; msgtype=response", "WARC-Block-Digest": "sha1:MYN3MUCEPRUTGBKCZJOFOQTICWXEZVXH", "WARC-Concurrent-To": "<urn:uuid:aaa9383a-91be-47a1-a0f3-d18e5a016cd2>", "WARC-Date": "2023-11-30T17:33:23Z", "WARC-IP-Address": "104.21.22.173", "WARC-Identified-Payload-Type": "text/html", "WARC-Payload-Digest": "sha1:Q5XXSEQATK6FUIBTCNMTALZI74ZKPPAI", "WARC-Record-ID": "<urn:uuid:139afc82-fc87-48e0-9f1b-aa0c5b65d7ac>", "WARC-Target-URI": "https://911weknow.com/how-to-install-kali-linux-in-usb", "WARC-Truncated": null, "WARC-Type": "response", "WARC-Warcinfo-ID": "<urn:uuid:8810990d-8e28-473c-a740-828ef8e3dd2c>" }, "warc_info": "isPartOf: CC-MAIN-2023-50\r\npublisher: Common Crawl\r\ndescription: Wide crawl of the web for November/December 2023\r\noperator: Common Crawl Admin ([email protected])\r\nhostname: ip-10-67-67-56\r\nsoftware: Apache Nutch 1.19 (modified, https://github.com/commoncrawl/nutch/)\r\nrobots: checked via crawler-commons 1.5-SNAPSHOT (https://github.com/crawler-commons/crawler-commons)\r\nformat: WARC File Format 1.1\r\nconformsTo: https://iipc.github.io/warc-specifications/specifications/warc-format/warc-1.1/" }
{ "line_start_idx": [ 0, 33, 34, 67, 68, 83, 84, 363, 364, 439, 440, 500, 501, 595, 596, 730, 731, 898, 899, 977, 978, 989, 990, 1137, 1138, 1153, 1154, 1209, 1210, 1261, 1262, 1307, 1308, 1317, 1318, 1445, 1446, 1481, 1482, 1535, 1536, 1566, 1567, 1638, 1639, 1686, 1687, 1717, 1718, 1766, 1767, 1784, 1785, 1790, 1791, 1852, 1853, 1939, 1940, 2075, 2076, 2155, 2156, 2254, 2255, 2359, 2360, 2402, 2403, 2405, 2406, 2419, 2420 ], "line_end_idx": [ 33, 34, 67, 68, 83, 84, 363, 364, 439, 440, 500, 501, 595, 596, 730, 731, 898, 899, 977, 978, 989, 990, 1137, 1138, 1153, 1154, 1209, 1210, 1261, 1262, 1307, 1308, 1317, 1318, 1445, 1446, 1481, 1482, 1535, 1536, 1566, 1567, 1638, 1639, 1686, 1687, 1717, 1718, 1766, 1767, 1784, 1785, 1790, 1791, 1852, 1853, 1939, 1940, 2075, 2076, 2155, 2156, 2254, 2255, 2359, 2360, 2402, 2403, 2405, 2406, 2419, 2420, 2436 ] }
{ "red_pajama_v2": { "ccnet_original_length": 2436, "ccnet_original_nlines": 72, "rps_doc_curly_bracket": 0, "rps_doc_ldnoobw_words": 0, "rps_doc_lorem_ipsum": 0, "rps_doc_stop_word_fraction": 0.35687023401260376, "rps_doc_ut1_blacklist": 0, "rps_doc_frac_all_caps_words": 0.03625953942537308, "rps_doc_frac_lines_end_with_ellipsis": 0, "rps_doc_frac_no_alph_words": 0.19656488299369812, "rps_doc_frac_unique_words": 0.4506024122238159, "rps_doc_mean_word_length": 4.501204967498779, "rps_doc_num_sentences": 32, "rps_doc_symbol_to_word_ratio": 0.013358780182898045, "rps_doc_unigram_entropy": 4.849384784698486, "rps_doc_word_count": 415, "rps_doc_frac_chars_dupe_10grams": 0, "rps_doc_frac_chars_dupe_5grams": 0.02783725969493389, "rps_doc_frac_chars_dupe_6grams": 0.02783725969493389, "rps_doc_frac_chars_dupe_7grams": 0.02783725969493389, "rps_doc_frac_chars_dupe_8grams": 0, "rps_doc_frac_chars_dupe_9grams": 0, "rps_doc_frac_chars_top_2gram": 0.03854389861226082, "rps_doc_frac_chars_top_3gram": 0.01766595058143139, "rps_doc_frac_chars_top_4gram": 0.022483939304947853, "rps_doc_books_importance": -212.40139770507812, "rps_doc_books_importance_length_correction": -212.40139770507812, "rps_doc_openwebtext_importance": -131.66712951660156, "rps_doc_openwebtext_importance_length_correction": -131.66712951660156, "rps_doc_wikipedia_importance": -124.58039093017578, "rps_doc_wikipedia_importance_length_correction": -124.58039093017578 }, "fasttext": { "dclm": 0.020146910101175308, "english": 0.849295437335968, "fineweb_edu_approx": 1.7996832132339478, "eai_general_math": 0.025449929758906364, "eai_open_web_math": 0.1775418519973755, "eai_web_code": 0.21590816974639893 } }
{ "free_decimal_correspondence": { "primary": { "code": "005.445", "labels": { "level_1": "General works, books and libraries, information sciences", "level_2": "", "level_3": "Computer programming" } }, "secondary": { "code": "004.028", "labels": { "level_1": "General works, books and libraries, information sciences", "level_2": "", "level_3": "Computers and Computer science" } } }, "bloom_cognitive_process": { "primary": { "code": "3", "label": "Apply" }, "secondary": { "code": "2", "label": "Understand" } }, "bloom_knowledge_domain": { "primary": { "code": "3", "label": "Procedural" }, "secondary": { "code": "2", "label": "Conceptual" } }, "document_type_v1": { "primary": { "code": "3", "label": "Reference/Encyclopedic/Educational" }, "secondary": { "code": "4", "label": "Code/Software" } }, "extraction_artifacts": { "primary": { "code": "0", "label": "No Artifacts" }, "secondary": { "code": "-1", "label": "Abstain" } }, "missing_content": { "primary": { "code": "0", "label": "No missing content" }, "secondary": { "code": "2", "label": "Click Here References" } }, "document_type_v2": { "primary": { "code": "23", "label": "Tutorial" }, "secondary": { "code": "8", "label": "Documentation" } }, "reasoning_depth": { "primary": { "code": "2", "label": "Basic Reasoning" }, "secondary": { "code": "1", "label": "No Reasoning" } }, "technical_correctness": { "primary": { "code": "3", "label": "Mostly Correct" }, "secondary": { "code": "4", "label": "Highly Correct" } }, "education_level": { "primary": { "code": "2", "label": "High School Level" }, "secondary": { "code": "1", "label": "General Audience" } } }
672f1e42c33a7f9846924a2431ea77df
-1,431,506,687,387,278,300
Tell me more × Database Administrators Stack Exchange is a question and answer site for database professionals who wish to improve their database skills and learn from others in the community. It's 100% free, no registration required. When designing a SQL server data schema and the subsequent queries, sprocs, views, etc. does the notion of a clustered index and order of data on disk make any sense to consider for DB designs made explicitly to be deployed on SSD platforms? http://msdn.microsoft.com/en-us/library/aa933131(v=sql.80).aspx "A clustered index determines the physical order of data in a table." On a physical disk platform, the design to consider them makes sense to me as a physical scan of the data to retrieve "sequential" rows can be more performant than a seek through the table. On an SSD platform, all data read access uses an identical seek. There is no concept of "physical order" and data reads are not "sequential" in the sense that bits are stored on the same piece of silicon. So, in the process of designining an application database is the clustered index consideration relevant to this platform? My initial thought is that it is not because the idea of "ordered data" doesn't apply to SSDs storage and seek/retreival optimization. EDIT: I know the SQL Server will create one, I'm just philosophizing about whether it makes sense to think about it during design/optimization. share|improve this question 1   Interesting question; I'd probably include clustered indexes becasue the optimizer is geared to use them. Think about little things like Primary Keys defaulting to use clustered indexes, and how all nonclustered indexes point to clustered indexes (if they exist). In short, there may be no difference in terms of file I/O, but I would think that the database engine would make certain assumptions based on their presence or absence. –  Stuart Ainsworth Nov 28 '11 at 19:43   Not sure what you meant by your edit, but clustered indexes are not required. So no, SQL Server does not have to create a clustered index for each table. –  Nick Chammas Nov 28 '11 at 20:14   @NickChammas I'm referring to the fact that SQL Server automatically created a clustered index on the primary key. –  Matthew Nov 28 '11 at 22:08 1   Some papers on this general area (not specific to your question) Do Query Optimizers Need to be SSD-aware? and Query Processing Techniques for Solid State Drives –  Martin Smith Dec 30 '12 at 11:52 add comment migrated from stackoverflow.com Nov 28 '11 at 19:49 This question came from our site for professional and enthusiast programmers. 3 Answers up vote 24 down vote accepted Ask yourself another question: If the entire database is in memory and I never have to touch the disk, do I want to store my data in an ordered B-tree or do I want to store my data in an unordered heap? The answer to this question will depend on your access pattern. On most cases your access requires single row look-up (ie. seeks) and range scans. These access patterns require a B-Tree, otherwise they are inefficient. Some other access patterns, common in DW and OLAP, are always doing aggregates over the entire table end-to-end always and they do no benefit from range scans. As you drill further other requirements come to light, like the speed of insert and allocation into a heap vs. B-Tree may play a role for huge ETL transfer jobs. But most times the answer really boils down to one question: do you seek or range-scan? The overwhelming number of times the answer is YES. And therefore the overwhelming number of times the design requires a clustered index. In other words: just because is cheap to read it from disk in random order does not imply that you can trash your TLBs and L2 lines in a 64Gb RAM scan bonanza... share|improve this answer 2   Even if a given access pattern requires a B-tree with a certain ordering, that does not necessarily imply that the actual table data needs to be stored in a B-tree with that ordering. (Put another way: just because you need an index, that doesn't necessarily mean you need a clustered index.) –  ruakh Nov 28 '11 at 21:35 1   @ruakh: True. But if the majority of access patterns require an order then is a strong indication that order should be your clustered index. –  Remus Rusanu Nov 28 '11 at 21:46   @RemusRusanu I think that ruakh hits on my question to you. What benefit is there for choosing the clustered index over choosing a standard index; particularly in your example of an in-RAM DB... Is it solely Bill Karwin's example of the coincidence that data will be on the same page? –  Matthew Nov 28 '11 at 22:14 1   @RemusRusanu - Does the read ahead mechanism still work exactly the same for SSDs as for "traditional" discs? –  Martin Smith Dec 1 '11 at 11:22 1   @Martin Smith: read-ahead still occurs on SSDs, as the code will exercise what it knows to do. But the benefits of read-ahead are muted on SSDs: if an IO stall is hit, the wait time for the page to be fetched is usually very small on SSDs. –  Remus Rusanu Dec 1 '11 at 16:12 show 1 more comment If you use a well-chosen clustered index, you are more likely to get all the related data you need in fewer pages of data. That is, you can hold the data you need in less memory. This gives a benefit regardless of whether you use spinning disks or SSD. But you're correct that the other benefit of a clustered index -- to read/write related data sequentially instead of with many disk seeks -- isn't a significant benefit for SSD, where seeks are not such a huge performance overhead as they are with spinning disks. Re @Matthew PK's comment. Of course location A in RAM is just as quick as location B in RAM. That's not the point. I'm talking about the case when all the data you need won't fit in RAM if the data is scattered among many pages. Any given page may contain only a small amount of data you're interested in. So the RDBMS has to keep loading and purging pages as you access A, B, and other rows. That's where you get the performance penalty. It would be better for every page to be full of data you're interested in, in the hopes that all of the subsequent row requests are served from pages in RAM. Using a clustered index is a good way to ensure that your data is grouped together onto fewer pages. share|improve this answer   ...Or in RAM as well. Locality of reference does indeed make a difference in performance –  Conrad Frix Nov 28 '11 at 19:44   Right, by memory I mean RAM. Disks are storage, not memory. –  Bill Karwin Nov 28 '11 at 19:45   I don't understand how a lookup to location A in RAM is faster than a lookup to location B in RAM. Can you elaborate or post a link? I understand that fewer pages will take up less space. –  Matthew Nov 28 '11 at 19:56 2   @MatthewPK - When you seek against a clustered index, any column you could possibly want from the table will, by definition, be in the clustered index. When you seek against a nonclustered index, you will have to perform extra lookups to retrieve the columns you asked for in your SELECT clause that weren't in the index. For a large number of IOPS you will feel the burden of these extra reads, SSDs or not. –  Nick Chammas Nov 28 '11 at 20:08 2   @Matthew PK: lookup location A and lookup location B in RAM can be vastly different in time, see aristeia.com/TalkNotes/ACCU2011_CPUCaches.pdf. However, at a database page granularity level, the impact is restricted to the TLB misses which is not that big of an impact. –  Remus Rusanu Nov 28 '11 at 20:22 show 2 more comments Yes, it absolutely still does make sense. You're thinking too low-level in your approach. SQL Server (in a very very simplified explanation) stores clustered data in a B-tree architecture. This allows for fast data retrieval based on the clustered index key values. A heap (no clustered index) has no sequential order of data. The most important thing to consider here that is in a heap the data pages are not linked in a linked list. So the answer is yes, it still makes sense to have clustered indexes created on tables, even on an SSD. It's all based on how much data SQL Server has to sift through to get to the resulting data. With a clustered index seek, it is minimized. Reference: http://msdn.microsoft.com/en-us/library/ms189051.aspx share|improve this answer   There will be a clustered index. The point was whether or not seeks along it matter on the SSD platform –  Matthew Nov 28 '11 at 19:54 4   Yes, the seeks matter. 3 reads as opposed to 300 reads is faster no matter what the medium you are using. –  Thomas Stringer Nov 28 '11 at 19:56   but I'm not discussing the merits of having no indices, simply questioning the importance of a clustered versus standard index given identical seek times. –  Matthew Nov 28 '11 at 22:14   @MatthewPK What's your definition of a "standard index"? –  Thomas Stringer Nov 28 '11 at 22:26   good point, poor word use on my part, when I say "standard index" I mean all types of non-clustered indices. –  Matthew Nov 28 '11 at 22:37 add comment Your Answer   discard By posting your answer, you agree to the privacy policy and terms of service. Not the answer you're looking for? Browse other questions tagged or ask your own question.
{ "url": "http://dba.stackexchange.com/questions/8496/is-the-concept-of-a-clustered-index-in-a-db-design-sensical-when-using-ssds/8500", "source_domain": "dba.stackexchange.com", "snapshot_id": "crawl=CC-MAIN-2013-48", "warc_metadata": { "Content-Length": "78071", "Content-Type": "application/http; msgtype=response", "WARC-Block-Digest": "sha1:IUWUGAWCOPSKFFDIU4AEMBHP63NARY5W", "WARC-Concurrent-To": "<urn:uuid:fc5fae7d-19fe-4002-bbb3-e61f5d4107f2>", "WARC-Date": "2013-12-09T04:45:53Z", "WARC-IP-Address": "198.252.206.16", "WARC-Identified-Payload-Type": null, "WARC-Payload-Digest": "sha1:AXBRZJEZELV6O4RSHBLLPS7IAAG6FUWM", "WARC-Record-ID": "<urn:uuid:c02a78db-c072-4de7-86fb-1546d1cf1a9e>", "WARC-Target-URI": "http://dba.stackexchange.com/questions/8496/is-the-concept-of-a-clustered-index-in-a-db-design-sensical-when-using-ssds/8500", "WARC-Truncated": null, "WARC-Type": "response", "WARC-Warcinfo-ID": "<urn:uuid:5d3d81f6-fbb5-479c-a508-048cd90161af>" }, "warc_info": "robots: classic\r\nhostname: ip-10-33-133-15.ec2.internal\r\nsoftware: Nutch 1.6 (CC)/CC WarcExport 1.0\r\nisPartOf: CC-MAIN-2013-48\r\noperator: CommonCrawl Admin\r\ndescription: Wide crawl of the web with URLs provided by Blekko for Winter 2013\r\npublisher: CommonCrawl\r\nformat: WARC File Format 1.0\r\nconformsTo: http://bibnum.bnf.fr/WARC/WARC_ISO_28500_version1_latestdraft.pdf" }
{ "line_start_idx": [ 0, 15, 235, 236, 478, 479, 543, 613, 614, 804, 1009, 1010, 1132, 1133, 1268, 1269, 1413, 1414, 1442, 1446, 1919, 1921, 2111, 2113, 2259, 2263, 2461, 2473, 2474, 2526, 2527, 2605, 2606, 2616, 2617, 2647, 2648, 2851, 2852, 3619, 3620, 3782, 3783, 3809, 3813, 4135, 4139, 4316, 4318, 4634, 4638, 4783, 4787, 5062, 5082, 5083, 5336, 5337, 5601, 5602, 5603, 5629, 5630, 6043, 6044, 6303, 6304, 6330, 6332, 6456, 6458, 6553, 6555, 6774, 6778, 7223, 7227, 7533, 7554, 7555, 7821, 7822, 7991, 7992, 8235, 8236, 8301, 8302, 8328, 8330, 8465, 8469, 8614, 8616, 8802, 8804, 8900, 8902, 9042, 9054, 9055, 9067, 9068, 9070, 9078, 9079, 9157, 9158 ], "line_end_idx": [ 15, 235, 236, 478, 479, 543, 613, 614, 804, 1009, 1010, 1132, 1133, 1268, 1269, 1413, 1414, 1442, 1446, 1919, 1921, 2111, 2113, 2259, 2263, 2461, 2473, 2474, 2526, 2527, 2605, 2606, 2616, 2617, 2647, 2648, 2851, 2852, 3619, 3620, 3782, 3783, 3809, 3813, 4135, 4139, 4316, 4318, 4634, 4638, 4783, 4787, 5062, 5082, 5083, 5336, 5337, 5601, 5602, 5603, 5629, 5630, 6043, 6044, 6303, 6304, 6330, 6332, 6456, 6458, 6553, 6555, 6774, 6778, 7223, 7227, 7533, 7554, 7555, 7821, 7822, 7991, 7992, 8235, 8236, 8301, 8302, 8328, 8330, 8465, 8469, 8614, 8616, 8802, 8804, 8900, 8902, 9042, 9054, 9055, 9067, 9068, 9070, 9078, 9079, 9157, 9158, 9248 ] }
{ "red_pajama_v2": { "ccnet_original_length": 9248, "ccnet_original_nlines": 107, "rps_doc_curly_bracket": 0, "rps_doc_ldnoobw_words": 0, "rps_doc_lorem_ipsum": 0, "rps_doc_stop_word_fraction": 0.38308456540107727, "rps_doc_ut1_blacklist": 0, "rps_doc_frac_all_caps_words": 0.03631841018795967, "rps_doc_frac_lines_end_with_ellipsis": 0.009259260259568691, "rps_doc_frac_no_alph_words": 0.20398010313510895, "rps_doc_frac_unique_words": 0.3353846073150635, "rps_doc_mean_word_length": 4.4424614906311035, "rps_doc_num_sentences": 94, "rps_doc_symbol_to_word_ratio": 0.0014925400028005242, "rps_doc_unigram_entropy": 5.543416500091553, "rps_doc_word_count": 1625, "rps_doc_frac_chars_dupe_10grams": 0, "rps_doc_frac_chars_dupe_5grams": 0.07064689695835114, "rps_doc_frac_chars_dupe_6grams": 0.048760220408439636, "rps_doc_frac_chars_dupe_7grams": 0.03075218014419079, "rps_doc_frac_chars_dupe_8grams": 0.006649119779467583, "rps_doc_frac_chars_dupe_9grams": 0.006649119779467583, "rps_doc_frac_chars_top_2gram": 0.010527770034968853, "rps_doc_frac_chars_top_3gram": 0.016484279185533524, "rps_doc_frac_chars_top_4gram": 0.02119407057762146, "rps_doc_books_importance": -1061.9547119140625, "rps_doc_books_importance_length_correction": -1061.9547119140625, "rps_doc_openwebtext_importance": -521.7645874023438, "rps_doc_openwebtext_importance_length_correction": -521.7645874023438, "rps_doc_wikipedia_importance": -403.17822265625, "rps_doc_wikipedia_importance_length_correction": -403.17822265625 }, "fasttext": { "dclm": 0.6941772103309631, "english": 0.9138699769973755, "fineweb_edu_approx": 1.9952913522720337, "eai_general_math": 0.37281763553619385, "eai_open_web_math": 0.2531368136405945, "eai_web_code": 0.02491760067641735 } }
{ "free_decimal_correspondence": { "primary": { "code": "005.746", "labels": { "level_1": "General works, books and libraries, information sciences", "level_2": "", "level_3": "Computer programming" } }, "secondary": { "code": "004.67", "labels": { "level_1": "General works, books and libraries, information sciences", "level_2": "", "level_3": "Computers and Computer science" } } }, "bloom_cognitive_process": { "primary": { "code": "4", "label": "Analyze" }, "secondary": { "code": "5", "label": "Evaluate" } }, "bloom_knowledge_domain": { "primary": { "code": "2", "label": "Conceptual" }, "secondary": { "code": "3", "label": "Procedural" } }, "document_type_v1": { "primary": { "code": "5", "label": "Social/Forum" }, "secondary": { "code": "-1", "label": "Abstain" } }, "extraction_artifacts": { "primary": { "code": "3", "label": "Irrelevant Content" }, "secondary": { "code": "-1", "label": "Abstain" } }, "missing_content": { "primary": { "code": "0", "label": "No missing content" }, "secondary": { "code": "-1", "label": "Abstain" } }, "document_type_v2": { "primary": { "code": "18", "label": "Q&A Forum" }, "secondary": { "code": "8", "label": "Documentation" } }, "reasoning_depth": { "primary": { "code": "3", "label": "Intermediate Reasoning" }, "secondary": { "code": "2", "label": "Basic Reasoning" } }, "technical_correctness": { "primary": { "code": "4", "label": "Highly Correct" }, "secondary": { "code": "3", "label": "Mostly Correct" } }, "education_level": { "primary": { "code": "3", "label": "Undergraduate Level" }, "secondary": { "code": "4", "label": "Graduate/Expert Level" } } }
672f1e42c33a7f9846924a2431ea77df
-5,694,482,629,789,809,000
We're here to help you navigate STEM learning at home while schools and camps are closed due to COVID-19. Here are some resources to guide your at home learning: What variables make a game popular with players, and do boys and girls choose different types of games? Design a survey-based science project this summer and do some statistical analysis of the data you gather. Your results might be eye opening and informative in terms of game design, the gaming industry, and what works and what doesn't depending on the audience. Boys and girls and video games / Student science project There are only three girls, as far as I know, in the clan in one of my current favorite games. With a staggering more than seventy-five thousand clans floating around in the game, and hundreds of thousands of players around the world, clan members come and go. A few other self-identified female players have pitstopped in our clan before moving on, but three of us have been clan members for a long time and seem to be staying—three out of a clan that typically weighs in right around the max of fifty players. That simple statistic—3 out of 50—seems revealing. It seems to support gender stereotypes about who plays video games. But there are other variables to consider. Age and location, for example, throw a possible wrench into the picture. Our clan is global. Twenty-four hours a day there are people in our clan online from all around the world, and there are players of all ages, a strong mix, in fact, of adults, teens, and even younger players. How old might you guess the three girls are? Where do they live? Do age and location have anything to do with which games boys and girls play? Minecraft skin Who is the Hero? In a game like Angry Birds, the gamer remains off screen. You pull the slingshot, but your identity is not part of the game. What matters is what happens between the birds and pigs in response to your aim and launch. In games where players appear on screen as a visible protagonist, choosing between available characters—or enabling customization of one's avatar—is a common game element. Minecraft players, for examples, create and change "skins" to control the appearance of their character (like the one shown above). In story-based games, however, players often take on the role of a predetermined main character, a protagonist who appears in video cutscenes as well as in game play. Some story-based games offer a choice of playable characters, but many do not. Does the gender of the playable character make a difference in terms of who buys and plays a game? Discussion and speculation surrounding previews of new Zelda, Halo, and Assasin's Creed titles suggest that the gender of playable characters is, indeed, a big deal for many gamers. Conducting survey-based science research projects like Do Males and Females Play the Same Types of Games? and Gamers: Myth or Man? can help you better analyze today's gaming scene and make some predictions about the future of game development and design. Survey Says To learn more about setting up and using surveys as part of a science project, see the Designing a Survey and Sample Size: How Many Survey Participants Do I Need? resources from the Science Buddies Project Guide. To view more science project ideas like the ones discussed here, see the Video and Computer Game section at Science Buddies. In other games I play, the balance of male to female players appears more equal or, in some cases, maybe tilted to the "more girls" side. Who plays Words with Friends? Who plays Candy Crush? Who plays Hay Day or Farmville? Who plays Infinity Blade? Who plays Final Fantasy or Elder Scrolls? Who plays Temple Run or Subway Surfer? Who plays Minecraft or Wizard 101? Who plays Pokémon, Zelda, Uncharted, or Assasin's Creed? Or maybe we need to step back and ask, what kinds of games are those listed above—and does that have anything to do with who plays them? Games, Games, Everywhere As an adult gamer, "who plays games" and "what games do they play" is an interesting social puzzle. As a parent of kids who also play video games, I find the gender dynamics fascinating. After all, kids today are kids growing up in an age saturated with video games, mobile apps, social media, and an always-on, always-connected, pervasive tech-based lifestyle and social reality. I often ask my teen "do any of the girls you know play video games?" While most of the boys he knows do play video games, Minecraft, Terraria, and phone-based games like Clash of Clans topping the list in current popularity, his sense is that most of the girls do not. The ones that do appear to be on the fringe. It can't be that clear cut. Or can it? Is it really true that video gamers are still, by and large, male? Or is that stereotype outdated, wrong, and a real misreading of today's gaming scene? What does the type (or genre) of game have to do with the numbers of males and females who play? What trends can be found in different age groups, and how do those age groups compare to one another when you look at gender demographics? These are great questions for a gamer to ask, and a clever gamer can turn questions like these into a really cool science project that does a study of human behavior, social trends, and the video gaming industry—and opens up opportunities for doing some impressive statistical analysis of the results. Surveying the Gaming Scene The Do Males and Females Play the Same Types of Games? science project offers a framework for designing and conducting a survey of gamers to see if girls and boys differ in the genre of games they choose. With summer break here, you could do a social-media or text-based campaign to get friends (and their friends) involved in answering your survey. (While the project outlines a traditional paper-based survey, you might want to set up an electronic survey instead and run it through your social streams to cast a really broad net for responders. The more people who take the survey, the more data you have to help support your findings!) Before you get started, be sure and really look at the games that are on top of the charts today. (Make sure you keep a list of your sources and the dates since top game lists change frequently.) What categories or genres of games do you want to ask about? The list of genres and example games in the project helps get you started, but you will want to spend time editing and adding to the list to make it really fit today's gaming scene. You might also want to create additional categories to study different platforms and the subcategories of games that appear on each platform. You may find that you want to ask about genres (as the project shows) but that you also want to ask about a bunch of specific games, since some games cross genre boundaries or defy easy classification. There are lots of ways to customize and personalize a study like this, but summer is a great time to get started. You may be surprised at what you learn about gaming, gender, game genres, platforms and devices, and how people of different ages approach gaming. With a bit of data crunching down the road, you could crank out a large portion of next year's science fair project without leaving the couch this summer. (If you are thinking that far ahead, it might not hurt to drop your teacher an email first and let her know you are tackling a summer science survey that you hope to turn into your science fair project.) We do advocate leaving the couch, but this kind of study makes it easy to combine something you love with something that can really shine a light on social trends. Not only can a project like this give you better insight into gaming and the personality and profile of gamers, but this kind of data is also critical for aspiring video game designers and developers. The more you understand people who play games, the better you can develop successful games that attract thousands and thousands of players and fans. (For a related science project that compares gamer stereotypes to real gamers, see Gamers: Myth or Man?.) We would love to see the survey you create and hear about your experience with the project! Note: assumptions above about the number of boys vs. girls in games the author plays are based on guessing from user names or avatar photos or based on things said during in-game conversations. Many players do use ambiguous names or adopt a different identity during game play. You Might Also Enjoy These Related Posts: Free science fair projects.
{ "url": "https://www.sciencebuddies.org/blog/girls-boys-video-gaming-and-summer-science", "source_domain": "www.sciencebuddies.org", "snapshot_id": "crawl=CC-MAIN-2020-29", "warc_metadata": { "Content-Length": "47122", "Content-Type": "application/http; msgtype=response", "WARC-Block-Digest": "sha1:J5QGD44EODODOERW4C3R44K47XQNOODQ", "WARC-Concurrent-To": "<urn:uuid:fd242efa-4372-4830-a515-1adcd7af1b24>", "WARC-Date": "2020-07-11T15:04:10Z", "WARC-IP-Address": "18.216.42.187", "WARC-Identified-Payload-Type": "text/html", "WARC-Payload-Digest": "sha1:Q6NTALBIG6WFX2CLCVT6J5UY5IJEKUPY", "WARC-Record-ID": "<urn:uuid:c33c1b76-f4d6-4a16-9adb-0a59abbc36c6>", "WARC-Target-URI": "https://www.sciencebuddies.org/blog/girls-boys-video-gaming-and-summer-science", "WARC-Truncated": null, "WARC-Type": "response", "WARC-Warcinfo-ID": "<urn:uuid:93b19932-3167-4e14-bcd3-091d6a5fd2ba>" }, "warc_info": "isPartOf: CC-MAIN-2020-29\r\npublisher: Common Crawl\r\ndescription: Wide crawl of the web for July 2020\r\noperator: Common Crawl Admin ([email protected])\r\nhostname: ip-10-67-67-198.ec2.internal\r\nsoftware: Apache Nutch 1.17 (modified, https://github.com/commoncrawl/nutch/)\r\nrobots: checked via crawler-commons 1.2-SNAPSHOT (https://github.com/crawler-commons/crawler-commons)\r\nformat: WARC File Format 1.1\r\nconformsTo: http://iipc.github.io/warc-specifications/specifications/warc-format/warc-1.1/" }
{ "line_start_idx": [ 0, 106, 107, 163, 164, 530, 531, 588, 589, 590, 591, 1103, 1104, 1691, 1692, 1707, 1708, 1725, 1726, 2247, 2248, 2593, 2594, 2776, 2777, 3032, 3033, 3034, 3046, 3047, 3260, 3261, 3386, 3387, 3810, 3811, 3948, 3949, 3950, 3975, 3976, 4357, 4358, 4672, 4673, 4712, 4713, 5102, 5103, 5405, 5406, 5407, 5434, 5435, 5640, 5641, 6076, 6077, 6860, 6861, 7481, 7482, 8102, 8103, 8195, 8196, 8197, 8198, 8476, 8477, 8519, 8520 ], "line_end_idx": [ 106, 107, 163, 164, 530, 531, 588, 589, 590, 591, 1103, 1104, 1691, 1692, 1707, 1708, 1725, 1726, 2247, 2248, 2593, 2594, 2776, 2777, 3032, 3033, 3034, 3046, 3047, 3260, 3261, 3386, 3387, 3810, 3811, 3948, 3949, 3950, 3975, 3976, 4357, 4358, 4672, 4673, 4712, 4713, 5102, 5103, 5405, 5406, 5407, 5434, 5435, 5640, 5641, 6076, 6077, 6860, 6861, 7481, 7482, 8102, 8103, 8195, 8196, 8197, 8198, 8476, 8477, 8519, 8520, 8547 ] }
{ "red_pajama_v2": { "ccnet_original_length": 8547, "ccnet_original_nlines": 71, "rps_doc_curly_bracket": 0, "rps_doc_ldnoobw_words": 0, "rps_doc_lorem_ipsum": 0, "rps_doc_stop_word_fraction": 0.42865362763404846, "rps_doc_ut1_blacklist": 0, "rps_doc_frac_all_caps_words": 0.004602990113198757, "rps_doc_frac_lines_end_with_ellipsis": 0, "rps_doc_frac_no_alph_words": 0.12715764343738556, "rps_doc_frac_unique_words": 0.3537323474884033, "rps_doc_mean_word_length": 4.577000617980957, "rps_doc_num_sentences": 79, "rps_doc_symbol_to_word_ratio": 0, "rps_doc_unigram_entropy": 5.462628364562988, "rps_doc_word_count": 1487, "rps_doc_frac_chars_dupe_10grams": 0.01175433024764061, "rps_doc_frac_chars_dupe_5grams": 0.0226270891726017, "rps_doc_frac_chars_dupe_6grams": 0.01175433024764061, "rps_doc_frac_chars_dupe_7grams": 0.01175433024764061, "rps_doc_frac_chars_dupe_8grams": 0.01175433024764061, "rps_doc_frac_chars_dupe_9grams": 0.01175433024764061, "rps_doc_frac_chars_top_2gram": 0.012929770164191723, "rps_doc_frac_chars_top_3gram": 0.005289449822157621, "rps_doc_frac_chars_top_4gram": 0.0061710299924016, "rps_doc_books_importance": -685.3143310546875, "rps_doc_books_importance_length_correction": -685.3143310546875, "rps_doc_openwebtext_importance": -374.01080322265625, "rps_doc_openwebtext_importance_length_correction": -374.01080322265625, "rps_doc_wikipedia_importance": -177.366455078125, "rps_doc_wikipedia_importance_length_correction": -177.366455078125 }, "fasttext": { "dclm": 0.15412843227386475, "english": 0.957709550857544, "fineweb_edu_approx": 2.32187557220459, "eai_general_math": 0.03851873055100441, "eai_open_web_math": 0.1291465163230896, "eai_web_code": 0.020763220265507698 } }
{ "free_decimal_correspondence": { "primary": { "code": "004.0194", "labels": { "level_1": "General works, books and libraries, information sciences", "level_2": "", "level_3": "Computers and Computer science" } }, "secondary": { "code": "305.23", "labels": { "level_1": "Social sciences", "level_2": "", "level_3": "Social sciences — Periodicals" } } }, "bloom_cognitive_process": { "primary": { "code": "4", "label": "Analyze" }, "secondary": { "code": "3", "label": "Apply" } }, "bloom_knowledge_domain": { "primary": { "code": "2", "label": "Conceptual" }, "secondary": { "code": "3", "label": "Procedural" } }, "document_type_v1": { "primary": { "code": "3", "label": "Reference/Encyclopedic/Educational" }, "secondary": { "code": "-1", "label": "Abstain" } }, "extraction_artifacts": { "primary": { "code": "0", "label": "No Artifacts" }, "secondary": { "code": "-1", "label": "Abstain" } }, "missing_content": { "primary": { "code": "0", "label": "No missing content" }, "secondary": { "code": "-1", "label": "Abstain" } }, "document_type_v2": { "primary": { "code": "10", "label": "Knowledge Article" }, "secondary": { "code": "23", "label": "Tutorial" } }, "reasoning_depth": { "primary": { "code": "3", "label": "Intermediate Reasoning" }, "secondary": { "code": "2", "label": "Basic Reasoning" } }, "technical_correctness": { "primary": { "code": "6", "label": "Not Applicable/Indeterminate" }, "secondary": { "code": "3", "label": "Mostly Correct" } }, "education_level": { "primary": { "code": "2", "label": "High School Level" }, "secondary": { "code": "1", "label": "General Audience" } } }
672f1e42c33a7f9846924a2431ea77df
6,003,508,217,361,231,000
EP0112186A2 - Modular high-speed multipliers, and integrated circuit chip modules for such multipliers - Google Patents Modular high-speed multipliers, and integrated circuit chip modules for such multipliers Download PDF Info Publication number EP0112186A2 EP0112186A2 EP19830307696 EP83307696A EP0112186A2 EP 0112186 A2 EP0112186 A2 EP 0112186A2 EP 19830307696 EP19830307696 EP 19830307696 EP 83307696 A EP83307696 A EP 83307696A EP 0112186 A2 EP0112186 A2 EP 0112186A2 Authority EP Grant status Application Patent type Prior art keywords bit parity multiple multiplier multiply Prior art date Legal status (The legal status is an assumption and is not a legal conclusion. Google has not performed a legal analysis and makes no representation as to the accuracy of the status listed.) Granted Application number EP19830307696 Other languages German (de) French (fr) Other versions EP0112186B1 (en ) EP0112186A3 (en ) Inventor John Ronald Schomburg Current Assignee (The listed assignees may be inaccurate. Google has not performed a legal analysis and makes no representation or warranty as to the accuracy of the list.) Unisys Corp Original Assignee Unisys Corp Sperry Corp Priority date (The priority date is an assumption and is not a legal conclusion. Google has not performed a legal analysis and makes no representation as to the accuracy of the date listed.) Filing date Publication date Links Images Classifications • GPHYSICS • G06COMPUTING; CALCULATING; COUNTING • G06FELECTRIC DIGITAL DATA PROCESSING • G06F7/00Methods or arrangements for processing data by operating upon the order or content of the data handled • G06F7/38Methods or arrangements for performing computations using exclusively denominational number representation, e.g. using binary, ternary, decimal representation • G06F7/48Methods or arrangements for performing computations using exclusively denominational number representation, e.g. using binary, ternary, decimal representation using non-contact-making devices, e.g. tube, solid state device; using unspecified devices • G06F7/52Multiplying; Dividing • G06F7/523Multiplying only • G06F7/53Multiplying only in parallel-parallel fashion, i.e. both operands being entered in parallel • G06F7/5324Multiplying only in parallel-parallel fashion, i.e. both operands being entered in parallel partitioned, i.e. using repetitively a smaller parallel parallel multiplier or using an array of such smaller multipliers • GPHYSICS • G06COMPUTING; CALCULATING; COUNTING • G06FELECTRIC DIGITAL DATA PROCESSING • G06F11/00Error detection; Error correction; Monitoring • G06F11/07Responding to the occurrence of a fault, e.g. fault tolerance • G06F11/08Error detection or correction by redundancy in data representation, e.g. by using checking codes • G06F11/10Adding special bits or symbols to the coded information, e.g. parity check, casting out 9's or 11's • GPHYSICS • G06COMPUTING; CALCULATING; COUNTING • G06FELECTRIC DIGITAL DATA PROCESSING • G06F2207/00Indexing scheme relating to methods or arrangements for processing data by operating upon the order or content of the data handled • G06F2207/38Indexing scheme relating to groups G06F7/38 - G06F7/575 • G06F2207/3804Details • G06F2207/386Special constructional features • G06F2207/3884Pipelining Abstract @ In a high-speed multiplier the array of partial products shown in Figure 7 is generated and reduced by means of similar integrated circuit chips of a first type each handling a square section (714, 716, 718, 720, 722, 724) of the array. The chip contains thirty-six AND gates, each receiving one digit of the multiplier and one digit of the multiplicand for its inputs, and whose outputs are fed to the first rank of a tree of pseudoadders, each of which is arranged to sum three input numbers and produce sum and carry outputs. The partial products produced by these chips are fed to a further rank of chips of a second type which further reduce the partial products to two in number, and these are finally combined by a full adder to produce the full product. Each chip contains means for generating the parity of its internal carries, thereby allowing the parity of the sums and carries produced by the array of chips to be generated and used for parity checking. Description • In the past, multiplication was most often performed in digital data processors by repetitively adding the multiplicand to itself the number of times indicated by the multiplier. As speed increased, this repetitive addition by the machine's adder gave way to the use of individual multiplier circuits. Today, most contemporary large scale processors utilize specialized networks to accomplish faster multiply operations. These networks are designed to simultaneously process several multiplier bits. Further, a number of specialized algorithms have been developed to simplify the logic used in the building of these arrays. Generally, this simplification involves the decoding of multiplier groups and interaction between the multiplier groups to control the array. • This approach, of course, complicates the control aspect, while simplifying the array. Such arrays are commonly built of a combination of adders and multiplexers, with multiplier decoders used for control. The adders used may be of the conventional type or they may take the carry/save form. • Another common feature in past digital data processors has been the inclusion of error detecting networks in all data paths of the machine. However, present day large scale integrated (LSI) devices do not provide for the inclusion of these error detecting networks in the usual manner. This leaves duplication and result comparison as the only available method of error detection for these LSI devices. • One specific earlier example of a fast multiplier was suggested in the article published in IEEE TRANSACTIONS ON ELECTRONIC COMPUTERS in an article by C. S. Wallace, entitled "A Suggestion for a Fast Multiplier", on pages 14-17 of the February, 1964 issue. The article suggests that the economics of large scale scientific computers of that day could benefit from greater investment in hardware to mechanize multiplication that was common in computers of a still earlier vintage. • Multiplication of binary fractions is normally implemented as the addition of a number of summands, each some simple multiple of the multiplicand, chosen from a limited set of available multiples on the basis of one or more multiplier digits. No good reason is known to depart from this general scheme. Acceleration of the process must then be based on one or more of the following expedients: (I) reduction in the number of summands; (2) acceleration of the formation of summands; (3) acceleration of the addition of summands. Acceleration of addition will be discussed initially. • The basic addition processes usually employed in computers add two numbers together. The possibility exists of adding together more than two numbers in a single adder to produce a single sum. However, the logical complexity of the adder required appears to grow quite disproportionately to the resulting increase in speed, consequently there appears to be no advantage in trying to sum even three numbers at a time into a single sum. • Another expedient, now quite commonly used, is to employ a pseudoadder which adds together three numbers, but rather than producing a single sum, produces two numbers whose sum equals that of the original three. In the context of the basic problem of adding together many summands, one pass through such an adder reduces the number of summands left to be summed by one, as does a pass through a conventional adder. The advantage of the pseudoadder is that it can operate without carry propagation along its digital stages and hence is much faster than the conventional adder. Thus, rather than allowing the possibility of a carry propagation from the lowest-order digit to the highest-order digit, carry propagation is allowed only to the next-highest-order digit. A simple form for such an adder is a string of full adder circuits of the normal sort, where the carry inputs are used for the third input number, and the carry outputs for the second output number. In multiplication, one pseudoadder is usually used, and storage is provided for two numbers. On each pass through the adder, the two stored numbers and one multiple of the multiplicand are added, and the resulting two numbers returned to storage. • In any scheme employing pseudoadders, the number of adder passes occurring in a multiplication before the product is reduced to the sum of two numbers will be two less than the number of summands, since each pass through an adder converts three numbers to two, reducing the count of numbers by one. To improve the speed of the multiplication, one must arrange many of these passes to occur simultaneously by providing several pseudoadders. • Assuming that all summands are generated simultaneously, the best possible first step is to group the summands into threes, and introduce each group into its own pseudoadder, thus reducing the count of numbers by a factor of 1.5 (or a little less, if the number of summands is not a multiple of three). The best possible second step is to group the numbers resulting from the first step into threes and again add each group in its own pseudoadder. By continuing such steps until only two numbers remain, the addition is completed in a time proportional to the logarithm of the number of summands. • Successive steps may use the same set of pseudoadders (using progressively fewer of the set in each step) by using temporary storage registers for the outputs of the pseudoadders. However, if we use separate adders the equipment cost is little, if at all increased, since the additional pseudoadders required will not need many more components than the flip- flop registers eliminated, and the control circuitry is greatly simplified. Further consideration of this approach will reveal that the purely combinational adder would have a considerable speed advantage. In the simplest form of multiplication, there are as many summands as multiplier digits, each either 0 or 1 times the multiplicand. A wide range of schemes involving recoding the multiplier into a new (possibly redundant) form using some negative digits have been developed to reduce the number of summands. Since all summands are to be generated simultaneously, and then summed very quickly, it is desirable that the recoding scheme used should (1) require only multiples of the multiplicand obtainable by shifting and complementing, and (2) be a local recoding in which each recoded digit depends only on a small group of original multiplier digits. The best system found gives base-four recoded multiplier digits which can be +2, +1,0 -1 or -2, and each is determined entirely by three adjacent original binary multiplier digits. Considering the process as a base-four recoding, digits 0, 1, 2, 3 are recoded into digits 0, 1, -2, -1, respectively, if the next less significant original base-four digit is 0 or 1, and into 1, 2, -1, 0 if the next less significant original digit is 2 or 3. The number of summands is half the number of binary multiplier digits. Attempts to reduce the number further appear to require multiples not obtainable by shifting. Some complications arise in the pseudoadder structure because of the negative multiplier digits, which, in a two's complement system, require correction digits to be added in. However, detailed examination shows that these problems are surmountable without loss of speed or undue circuit cost. • Additional schemes for parallel multipliers which are based on different principles and having some advantages over those presented above by C. S. Wallace, have been proposed by L. Dadda in his article entitled Some Schemes for Parallel Adders published in"Alta Frequenza" in March 1965 at pages 349 to 356. • In his conclusions, Mr. L. Dadda suggests that having established the possibility of a parallel digital multiplier, some considerations can now be made about the important aspects of speed and cost that can lie encountered in a practical design. • He suggests that if one assumes that a third of all arithmetic operations in scientific computers are multiplications and that these, at present, take about four times as long as additions, the use of a fast multiplier allowing a multiplication in a memory cycle time, would approximately double the speed of computation. • There is therefore a chance that a parallel multiplier could become a convenient means to improve the value of a computer, owing to the fact that its cost can be shown to be only a few percent of the total computer cost. • Let us first note that the total multiplication time is composed of two parts: the first is the time elapsed from the application of the signals representing the two factors to the inputs of the multiplier, to the availability of the inputs to the carry-propagating adder; the second part is the delay proper of the adder, mainly consisting in the carry propagation delay. • In the design of a practical multiplier, one can assume as a goal to obtain a total delay equal or less than the cycle time of the high-speed memory, so that the computer can work at its maximum speed, limited only by the memory speed. The choice of the type of circuits depends therefore on the memory cycle time of present day computers. The increasing use of large scale integration appears to suggest it as the prime path for multipliers as well. • Among the different type of full adders, the most suitable for the application in parallel multipliers, from the point of view of economy and speed, are those which require input variables of one form only (natural or complemented) so that output variables of the same form only must be generated. If such condition is satisfied, outputs of one stage can be used directly as inputs to the next stages, without the need of inverters, leading in general to a considerable saving in components and to a reduction of stage delay. • It must be noted that the above restriction can be partially relieved by allowing the use of counters producing outputs of only one form but different from the input's form. • Another similar multiplier circuit is disclosed in co-pending U.S. patent application Serial No. 387,644, entitled "Fast Error Checked Multibit Multiplier", by Glen R. Kregness filed June 8, 1982 and assigned to the present assignee. The differences and improvements in the present application will be more clearly understood as one reads and undestands the present description. However, generally that earlier application is primarily concerned with the use of a gated carry/save network. Through the use of that network, the need for the decoding of the multiplier bits was eliminated. Although the array itself is increased in complexity, the multiplexers required by earlier systems have been eliminated. In addition, interaction between multiplier groups is no longer required. The net effect is a reduction in logic with faster operation because of the elimination of the decoding. • Even though it is well known, indeed, that many important features of today's fast computers depend on the fact that during operations that last longer than one memory cycle (typically, during multiplication or division) memory can be made available for other operations (e.g. input- output) it is still desirable to process in parallel. Thus, a unique approach for the construction of an LSI multiplier will now be described which provides a unique logic block for use in creating a multiplication pipeline of arbitrary size. • Accordingly it is an object of this invention to provide an improved multiplier for use in a large scale digital data processing system, and integrated circuit chips which may be used for building such a network. Preferably each chip includes provision for parity generation. • A logical building block for use in a binary digital multiplier circuit is disclosed wherein the logical building block may be used to create a multiplication pipeline with parity, which pipeline may be of arbitrary size. The multiply pipeline with parity configuration includes circuits for generating and checking parity. The parity of the output carries leaving the component chips of the multiply pipeline are explicitly generated internally to those chips. By generating output carry parity on-chip, all single-bit errors except for those caused by on-chip internal gate or metallization faults will be detected. Thus, means are provided for detection of single-bit errors in the multiplication circuitry. The proposed multiply pipeline with parity circuit includes a pair of gate arrays, or chips, which participate as follows. A 6-bit by 6-bit multiply with parity chip is used as a building block in a portion of the pipeline in which all logical products are initially generated, and reduction of these logical products, or partial products, commences. A 6-bit by 4-bit adder/register with parity chip is used as a building block in subsequent sections of the pipeline, which are used to further reduce the partial products. This latter chip type also contains a latch register to provide orderly and controlled staging of the intermediate partial products. • The invention will be futher described with reference to the accompanying drawings, in which: • Fig. 1 is a functional block diagram of a multiplier according to the invention. • Fig. 2 is a detailed block diagram of the 6x6 multiply-with-parity gate array used in the multiplier of Fig. 1. • Figure 3 illustrates an individual carry/save adder used in the logical diagram of Fig. 2. • Fig. 4 illustrates the function of the 6x6 multiply-with-parity gate array. • Fig. 5 is a detailed logical block diagram of the 6x4 adder tree/register with parity gate array used in the multiplier of Fig. 1. • Fig. 6 illustrates the logical interconnection of the 6x6 multiplier gate array and the 6x4 adder tree/register gate array to form a 12-bit by 12-bit multiplier. • Fig. 7 is a table showing the formation of logical products in the 6x6 multiplier gate arrays for the multiplier of Fig. 6. • Fig. 8 shows the use of 12-bit by 48-bit multipliers, adder tree/registers, and full binary adders to form a 48-bit by 48-bit multiplication pipeline. • Fig. 1 shows a simplified block diagram of a 12-bit by n-bit multiplier. The value of n is assumed to be a multiple of six, which may be achieved by appending zeros to the data which is to be used as the multiplicand. The inputs to the multiplier consist of a 12-bit multiplier held in the Multiplier Register 110, and a n-bit multiplicand, held in the Multiplicand Register 112. These registers are clocked simultaneously via clock signal A. The bits in these registers are numbered starting at 1, from left to right. The main outputs of the multiplier consist of a n+12 bit Sums word and a n+12 bit Carries word. These two outputs, when added together after shifting the Carries left one position, will form the desired product. Parity information is included with all input and output data and parity checking of the multiplier is performed. • Each of the lower ranks (ø and 1) 114 and 118, of the multiplier consists of 6-bit by 6-bit multiplier chips, numbering n/6 + 1 chips. The outputs of these ranks 114, 118 consist of Sums and Carries similar in principle to the overall multiplier outputs, and combined Parity of the Internal, (PIC) and Output (POC) Carries. Internal carries are any carries produced in the process of reducing the summands by addition, but which are not accessible to the pins of the individual chips because of pin limitations. • The Upper Rank 122 of the multiplier consists of 6-bit-wide by 4-bit adder/register chips, numbering n/6 + 2 chips. This rank 122 reduces the four inputs from the lower ranks 114, 118 to the Sums and Carries which form the final product, and it also produces the Parities of the Internal (PIC) and Output (POC) Carries separately for error detection. • Lower rank A 118 is fed the most significant six bits of the Multiplier, bits 1 through 6, and Lower Rank 1, 114 is fed the least significant six bits of the Multiplier, bits 7 through 12. The entire Multiplicand 112 is fed to both of the Lower Ranks 114, 118. • When the Sum and Carry outputs of the Lower Ranks 114, 118 are connected to the Upper Rank 122, the Carries must be logically left-shifted by one bit position relative to the Sums. In addition, the Sum and Carry outputs of Lower Rank Ø, 118 must be connected to the Upper Rank 122 six bit positions to the left relative to the respective Lower Rank 1, 114 connections. • The scheme used for error detection and parity generation will now be described. Let p(x) represent the parity of x, *represent the logical . AND operator, and ⊕ represent the logical exclusive-OR operator. Equations for parity generation are developed as follows. • For the outputs of Lower Ranks Ø and 1: Figure imgb0001 Figure imgb0002 Figure imgb0003 Figure imgb0004 when Multr stands for Multiplier, Mpcd for Multiplicand, IC for Internal Carries and OC for Output Carries. • For the Upper Rank, and overall parity: Figure imgb0005 Figure imgb0006 where AC stands for combined internal and Output Carries. • The parity of any of the carries, internal or external, is not readily predicted, and is instead generated directly from the carries inside each chip. External parity generators 124, 126 are labelled PG in Fig. 1, and parity checkers 128, 130 which detect single bit errors, are labelled PC. The PC and PG functions are logically identical, and are circuits which perform multiple-bit exclusive-OR functions. • Fig. 2 shows the internal structure 200 of the 6-bit by 6-bit multiply circuits which are used to build structures 114, 118 of Fig. 1. The triangular structures 250, 254, 258, 262, 268, 272, 276, 278 and 280 represent parity generators with the same function as in the previous discussion. These circuits are well known in the art and require no further discussion. The trapezoidal structures 300 represent carry-save adders, each of whose function will be later described in conjunction with Fig. 3. • In the simplest form of binary multiplication, as in ordinary pencil-and-paper decimal multiplication, the entire multiplicand is multiplied by each digit of the multiplier. This causes the formation of as many partial products as there are digits in the multiplier. In binary multiplication, the multiplicand is merely copied for non-zero multiplier bits, and all zeros are copied for multiplier bits equal to zero. When the resulting partial products are arranged in a column with respect to the order of the multiplier bits used and in such a manner that all of the bits in a column have the same mathematical weight, a parallelogram is formed, the sum of whose rows produce the desired product. • The circuit of Fig. 2 generates a 6-bit square of this parallelogram for binary multiplication, and reduces the six rows to two rows termed sums and carries using the network of carry-save adders. There are six columns of four carry-save adders each. The partial product bits are all formed at the inputs to the lowest row of the carry-save adders via AND gates at each adder input. These are not explicitly shown in Fig. 2, but are implied by the structure labelled Adder Control 212. The bits being AND-ed at each input will be later detailed in Fig. 4. • The main inputs to the circuit chip are the 6-bit Multiplier 208 entering Adder Control 212 which implies the generation of six rows of partial products, and the 11-bit Multiplicand 210. Eleven bits are required since eleven different Multiplier bits participate in a 6-bit wide slice of the partial product parallelogram as the Multiplicand is left-shifted five times. This is also illustrated in Fig. 4. • The main outputs of the circuit chip are six Sum bits, (noted as out sum 1 through out sum 6) and six Carry bits, (out cyl through out cy6). When added together as shown in the lower portion of Fig. 4 below the summation bar, along with the Stage Carries to be discussed later, these form the sum of the six original rows of partial products. • Fig. 3 illustrates the function of a single carrry-save adder 300. The function of this structure is to add together three input bits of equal weight x, y, and z to produce a two-bit output sum. The most significant bit of this sum is termed the Carry bit, and the least significant bit is termed the Sum bit. Generalized Boolean equations for these bits are also shown in Fig. 3. • As shown in Fig. 4, each column of four carry-save adders reduces a column of six partial product bits to two bits, a Sum and a Carry Sn, Cn. Internal carries are allowed to propagate only to the next-highest order column when passing through each of the three ranks of adders in the circuit chip. Carries leaving the highest-order column, and those entering the lowest-order column, are termed Stage Carries. These are provided as pins on the circuit chip to allow unlimited extension of the three adder ranks for wider multiplicands. The parity of the internal carries and output stage carries, the parity of the output carries and their combined parity is also generated and sent to output pins for error checking. • As previously stated, the function of the 6-bit by 6-bit multiplier chip is illustrated in Fig. 4. The generation of the 6-bit square slice of the partial product parallelogram is also shown in Fig. 4. The Multiplier and Multiplicand bits are numbered to correspond with Fig. 2. Each Multiplier bit is AND-ed with each of the Multiplicand bits on its row. Fig. 4 illustrates the manner in which the output Sums, Carries, and Stage Carries are aligned before addition to produce the sum of the six partial product rows. • Fig. 5 shows the internal organization of the 6-bit wide by 4-bit adder/register circuit chip 500. This circuit chip 500 reduces four summands A, B, C, and D to two numbers, also termed Sums and Carries, using a network of carry-save adders 512 to 534, as in the multiplier circuit chip. In this case, only four rows A, B, C and D of data are being reduced, so that two ranks of adders are sufficient. Again, carries from the lower rank are allowed to propagate only to the next highest-order column of adders. • Carries leaving the first rank of adders from the highest-order column 512 and entering the second rank of adders in the lowest-order column 534 are termed Stage Carries. Pins are provided for the Stage Carries to allow unlimited extension of the adder/register structure. • As previously discussed, the main inputs to the adder/register chip 500 are the four Summands, named A, B, C, and D, each consisting of six bits numbered 1-6. The like-numbered bits of each Summand are applied to the same column of adders. The outputs of the second rank of adders 524, 526, 528, 530, 532 and 534 are six sum bits and six carry bits which are captured in twelve bits of a latch register 540 located on the circuit chip. • Parity of the internal carries and the output Stage Carry is generated and latched in the remaining bit of the 13-bit latch register 540.. • The main outputs of the circuit chip are the Sum and Carry output signals and the parity of the output Carries which is generated on the circuit chip directly from the register outputs via parity generator 542. The Sum and Carry outputs 544, 546 and internal carry parity 548 come directly from the latch register 540. • Fig. 6 illustrates the detailed interconnection of the Multiplier 610 and Multiplicand 612 registers, the two Lower Ranks of 6-bit by 6-bit multiplier chips 200, the Upper Rank of 6-bit by 4-bit adder/register chips 500, and external error detection gating necessary to build a 12-bit by 12- bit multiplier. To form the resulting 24-bit product, a 24-bit full binary adder may be used to add the output Sums from the Upper Rank to the left-shifted output Carries from the Upper Rank. This adder is not shown. • Fig. 6 further clearly illustrates how the Carry outputs from each of the Lower Ranks fJ and 1 are, in effect, left-shifted before being fed forward to the Upper Rank. Several important points should also be noted. The first point is that bit 1 of the Carries is not connected to any subsequent logic for any of the ranks. This is because these Carry bits will always be zero if the multiply array is connected in this scheme. If these bits were non-zero, the product would be forced to be one bit wider than is mathematically possible for a given rank. Another point is that the Stage Carries out of the left end of any given rank are also left unconnected for the same reason. The relationship of the Lower Ranks Ø and 1 to the Upper Rank and to each other is also clearly shown. The Lower Ranks Ø and 1 are offset from each other by six bit positions, since each takes a separate but adjacent six-bit slice of the Multiplier. Each of the Lower Ranks θ and 1 takes identical Multiplicand connections as shown. The scheme of these connections will be better understood when studying Fig. 7. • Fig. 7 shows a chart of how the partial products are formed in each of the Lower Ranks and the necessary Multiplier and Multiplicand connections for the 12-bit by 12-bit multiplier of Fig. 6. This chart can be extended to show the necessary connections for a 12-bit by n-bit multiplier where n is a multiple of 6. Note that this chart is reversed top-for-bottom with respect to the logical connection scheme of Fig. 6, to present its information in a natural mathematical sense. • Fig. 7 illustrates how the multiplication operation may be thought of as taking shifted copies of the Multiplicand 710, ANDing each of the shifted copies with a particular bit of the Multiplier 712, and summing the resultant logical products. Each of the six-bit square areas 714, 716, 718, 720, 722, 724 outlined in Fig. 7 represents the area of logical products covered by a single 6-bit by 6-bit multiply chip. By inspecting the centre square 716 of the top row, the need for eleven different bits of Multiplicand to be applied to each multiply chip is clear. Note that roughly one-half of each of the end multiply chips is wasted due to zero-filling. As the width of the Multiplicand increases, the fraction of the multiplication array wasted due to zero-filling decreases proportionally. • A multiplication pipeline built from the chips and structures previously discussed is presented in Fig. 8. The pipeline implementation shown is designed to multiply a 48-bit Multiplier 810 and a 48-bit Multiplicand 812 to form a 96-bit Product. Since this is a pipeline, a new Product may be formed at the clock cycle rate of the pipeline. There are four structure types identified besides the latch registers holding the Multiplier, Multiplicand, and Product previously described. • Each of the first structure type A's included in dotted area 801 consists of two ranks of 6-bit by 6-bit multiply circuit chips and one rank of 6-bit by 4-bit adder/register chips as shown in Figs. 1 and 6. There are four copies of structures type A, 814, 816, 818, 820, all sharing the same Multiplier and Multiplicand registers 810, 812. The ranks of multiply circuit chips are nine chips wide, and the rank of adder/register chips is ten chips wide. These structures form the original 48 partial product rows, and reduce them to 8 partial product rows. • Each of the second structure type B's included in dotted area 802 consists of a rank of twelve 6-bit by 4-bit adder/register chips, interconnected as shown in Fig. 6. These structures reduce the 8 rows of partial products to 4 rows. Structure type C 826 shown in dotted area 803 is a similar arrangement of sixteen adder/register chips, and reduces the 4 remaining partial product rows to 2 rows. • Structure type D, 828 shown in dotted area 804 is an ordinary full binary adder, which reduces the two remaining partial products to the final product. • Note that the Carries are left-shifted by one position as they are fed forward and that none of the highest-order Carries are connected as discussed earlier. Also observe that the A type structures are offset from each other by twelve bits since each takes a separate twelve-bit slice of the Multiplier. Although error detection logic is not shown in this figure, it may easily be extended from the schemes shown in Figs. 1 and 6 by those skilled in this art. • In summary, the inputs to a multiplier consists of a 12-bit multiplier (MULTR) and an n-bit multiplicand (MPCD); outputs consist of an n+12 bit sum word (SUM) and an n+12 bit carry word (CY). Parity is included with all input and output data. • Ranks Ø and 1 gate arrays each consist of (n+6)/6 chips, and perform n-bit X 6-bit multiplies. Outputs consist of an n+6 bit sum word and an n+6 bit carry word, and the parity of all carries generated (PIOC). • The higher rank of gate arrays consists of (n+12)/6 chips, and sums four n+12 bit input words resulting in an n+12 bit sum word and an n+12 bit carry word, which are then latched. Parity of the internal carries (PIC) and of the output carry word (POC) are also provided. • The inputs to the higher rank of gate arrays consists of the sum and carry outputs of ranks a and 1 of lower gate arrays. The outputs of rank #are logically shifted left six positions relative to those from rank 1 before being fed to the next higher rank. • The sum and carry words from this higher rank may be summed using a full binary adder to completely resolve the n+12 bit product. If a multiplier of arbitrary size is desired, subsequent ranks of these chips followed by a full binary adder may be used. • The scheme for checking the parities of the outputs of the multiplier was also presented. Let PX represent the parity of X. • For the outputs of rank Ø of the lower gate arrays: Figure imgb0007 • For the rank of upper gate arrays: Figure imgb0008 Figure imgb0009 • The carry parity (internal or output) is not predictable at any point, but is generated directly from the carries inside each circuit chip. Parity may be generated directly from data using a parity generator (PG) and checked using a parity checker (PC). These functions are logically identical, and are exclusive-OR functions. • The lower level gate array forms a six bit by six bit square slice of the parallelogram of logical products formed in a binary multiplication. Inputs include 11 bits of multiplicand and six bits of multiplier; outputs include a six bit sum word and a six bit carry word. Internal to the circuit chip is a tree of carry/save adders. Logical products are formed at the inputs to a first rank of carry-save adders. Stage carry inputs and outputs for extending the carry-save adder array beyond circuit chip boundaries are provided. The parity of the internal carries, output carries and their combination are provided for error checking. • We have also illustrated the upper level circuit chip referred to earlier which is the 6x4 adder tree register with parity gate array. This circuit chip reduces four six-bit summands to two six-bit outputs. Inputs include four six-bit summands: A1-A6, B1-B6, C1-C6 and D1-D6. Outputs include a six-bit sum with a six-bit carry. The circuit consists of a six-bit wide, four-bit deep carry-save adder tree followed by a latch which captures the six-bit sum and six-bit carry outputs of the adder tree and the parity of the carries generated in the adder tree. • The internal carry parity (latched) and the output carry parity (taken from the latch outputs) are again provided for error checking. Claims (17) 1. A multiply pipeline building block comprising: (a) a multiplicand register; (b) a multiplier register; (c) multiple ranks of multiply gate arrays commonly connected to said multiplicand and multiplier register; (d) a single rank of adder tree/register gate arrays commonly connected to each of said multiple ranks of multiply gate arrays. (e) first and second parity generating means connected to the output of said single rank of adder tree/register gate arrays, and (f) first and second parity checking means correspondingly connected to said parity generating means and to said output of said single rank of adder tree/register gate arrays; (g) said first parity checking means further connected to receive parity signals from said multiplicand register, said multiplier register, and each of said multiple ranks of multiply gate arrays to provide circuitry for generating and checking parity within the multiplication circuitry and by internally generating output carry parity signals to thereby internally detect single-bit errors therein. 2. The multiply pipeline building block as set forth in claim 1 wherein each of said multiple ranks of multiply gate arrays are identical gate arrays of a first gate array type and said single rank adder tree/register gate array is of a second gate array type. 3. The multiply pipeline building block as set forth in claim 2 wherein said multiple rank of multiply gate arrays are a pair of identical gate arrays of a first type. 4. The multiply pipeline building block as set forth in claim 2 wherein each of said multiple ranks of multiply gate arrays of identical first gate array types comprises: (a) means including a tree of carry-save adders for forming a six-bit by six-bit square slice of the parallelogram of logical products formed in a binary multiplication; (b) input means connected to the input thereof, which includes eleven bits of multiplicand and six bits of a multiplier for application to said forming means; and (d) output means connected to the output of said forming means which includes a six-bit word sum and a six-bit carry word. wherein logical products are formed at the inputs of the first level of said carry-save adders of said tree. 5. The multiply pipeline building block as set forth in claim 3, wherein each of said multiple ranks of multiply gate arrays of identical first gate array types comprises further means for including stage carry inputs and outputs for extending the carry-save adder array beyond the boundaries of said circuit chip. 6. The multiply pipeline building block as set forth in claim 2 wherein said single rank adder tree/register gate array of the second gate array type includes an adder circuit capable of summing four numbers and producing two numbers as a result. 7. The invention as set forth in claim 5, wherein the second gate array type further includes latching means provided directly at the adder outputs on the circuit chip to thereby eliminate chip crossing delay. 8. The invention as set forth in claim 6 wherein the second gate array type further includes means for providing carries which do not ripple more than one bit position thereby minimizing propagation delay. 9. A multiply pipeline with parity comprising: (a) multiplication circuitry for receiving a multiplier and a multiplicand; (b) Parity generating circuits included therewith for explicitly generating internally to the circuit chip the parities of the output carries leaving said circuit chip of the multiply pipeline; and (c) Parity checking circuits further included with said multiplication circuitry for checking parity internally to said circuit chip. 10. The invention as set forth in claim 9 above wherein said parity generating and checking means also includes means for providing detection of single-bit errors in the multiplication circuitry wherein by providing output carry parity on said circuit chip, all single-bit errors, except those caused by on-chip gate faults, will be detected. 11. A multiply with parity gate array to be used as a building block in a binary digital multiplier circuit comprising: (a) means including a tree of carry-save adders forming multiple bit by multiple bit square slice of the parallelogram of logical products formed in a binary multilplication; (b) input means connected to the input thereof which includes multiple bits of a multiplicand and multiple bits of a multiplier for application to said forming means; and (c) output means connected to the output of said forming means which includes a multiple bit word sum and a multiple bit carry word, wherein logical products are formed at the inputs at the first level of said carry-save adders of said tree. 12. The invention as set forth in claim 11 wherein said multiple bits of said square slice of the parallelogram is a six-bit by six-bit slice. 13. The invention as set forth in claim 11 wherein said multiple bits of the multiplicand comprise eleven bits and said multiple bits of said multiplier comprise six bits. 14. The invention as set forth in claim 11 wherein said multiple bit word sum is a six-bit word and said multiple bit carry word is also a six-bit word. 15. The invention as set forth in claim 11 wherein said multiply with parity gate array is a six by six multiply with parity gate array. 16. A multiple bit adder tree/register with parity gate array comprising: (a) an adder circuit capable of summing multiple numbers and producing a reduced multiple of numbers as a result; and (b) a latching means connected directly to the output of the adder on the circuit chip to thereby eliminate chip crossing delay. 1.7. The invention as set forth in claim 16 wherein said multiple adder tree/register with parity is a six-bit by four-bit adder tree/register with parity gate array and said multiple bit summed numbers to produce reduced multiple bits results in an adder circuit capable of summing four numbers and producing two numbers as a result. EP19830307696 1982-12-20 1983-12-19 Modular high-speed multipliers, and integrated circuit chip modules for such multipliers Expired - Lifetime EP0112186B1 (en) Priority Applications (2) Application Number Priority Date Filing Date Title US06451130 US4549280A (en) 1982-12-20 1982-12-20 Apparatus for creating a multiplication pipeline of arbitrary size US451130 1982-12-20 Publications (3) Publication Number Publication Date EP0112186A2 true true EP0112186A2 (en) 1984-06-27 EP0112186A3 true EP0112186A3 (en) 1986-10-15 EP0112186B1 EP0112186B1 (en) 1990-02-28 Family ID=23790932 Family Applications (1) Application Number Title Priority Date Filing Date EP19830307696 Expired - Lifetime EP0112186B1 (en) 1982-12-20 1983-12-19 Modular high-speed multipliers, and integrated circuit chip modules for such multipliers Country Status (4) Country Link US (1) US4549280A (en) EP (1) EP0112186B1 (en) JP (1) JPS59116853A (en) DE (1) DE3381262D1 (en) Families Citing this family (3) * Cited by examiner, † Cited by third party Publication number Priority date Publication date Assignee Title JPH0317132B2 (en) * 1983-12-28 1991-03-07 Hitachi Ltd JPH0584530B2 (en) * 1984-11-09 1993-12-02 Hitachi Ltd JPH04502677A (en) * 1989-01-13 1992-05-14 Citations (2) * Cited by examiner, † Cited by third party Publication number Priority date Publication date Assignee Title US3596072A (en) * 1968-05-24 1971-07-27 Hitachi Ltd Error-detecting circuitry in adder system US3670956A (en) * 1968-09-26 1972-06-20 Hughes Aircraft Co Digital binary multiplier employing sum of cross products technique Family Cites Families (5) * Cited by examiner, † Cited by third party Publication number Priority date Publication date Assignee Title US3508038A (en) * 1966-08-30 1970-04-21 Ibm Multiplying apparatus for performing division using successive approximate reciprocals of a divisor US3986015A (en) * 1975-06-23 1976-10-12 International Business Machines Corporation Arithmetic unit for use in a digital data processor and having an improved system for parity check bit generation and error detection JPS5395540A (en) * 1977-02-01 1978-08-21 Nec Corp Multi-input adder JPS576617B2 (en) * 1978-08-02 1982-02-05 US4314349A (en) * 1979-12-31 1982-02-02 Goodyear Aerospace Corporation Processing element for parallel array processors Patent Citations (2) * Cited by examiner, † Cited by third party Publication number Priority date Publication date Assignee Title US3596072A (en) * 1968-05-24 1971-07-27 Hitachi Ltd Error-detecting circuitry in adder system US3670956A (en) * 1968-09-26 1972-06-20 Hughes Aircraft Co Digital binary multiplier employing sum of cross products technique Non-Patent Citations (3) * Cited by examiner, † Cited by third party Title IEEE JOURNAL OF SOLID STATE CIRCUITS, vol. SC-16, no. 3, June 1981, pages 174-179, IEEE, New York, US; J.-L. RAINARD et al.: "A 16-bit self-testing multiplier" * IEEE TRANSACTIONS ON COMPUTERS, vol. C-22, no. 2, February 1973, pages 113-120, IEEE, New York, US; S. SINGH et al.: "Multiple operand addition and multiplication" * THE RADIO AND ELECTRONIC ENGINEER, vol. 49, no. 5, May 1979, pages 250-254; F. DEMMELMEIER: "A fast multiplier module" * Also Published As Publication number Publication date Type DE3381262D1 (en) 1990-04-05 grant JPS59116853A (en) 1984-07-05 application US4549280A (en) 1985-10-22 grant EP0112186B1 (en) 1990-02-28 grant EP0112186A3 (en) 1986-10-15 application Similar Documents Publication Publication Date Title Anderson et al. The IBM system/360 model 91: floating-point execution unit US6230179B1 (en) Finite field multiplier with intrinsic modular reduction US4839847A (en) N-clock, n-bit-serial multiplier US5524090A (en) Apparatus for multiplying long integers US4075704A (en) Floating point data processor for high speech operation US6523055B1 (en) Circuit and method for multiplying and accumulating the sum of two products in a single cycle US4251875A (en) Sequential Galois multiplication in GF(2n) with GF(2m) Galois multiplication gates US6438569B1 (en) Sums of production datapath US4825401A (en) Functional dividable multiplier array circuit for multiplication of full words or simultaneous multiplication of two half words US5289397A (en) High-speed modulo exponentiator device US4901270A (en) Four-to-two adder cell for parallel multiplication US4972362A (en) Method and apparatus for implementing binary multiplication using booth type multiplication US5153848A (en) Floating point processor with internal free-running clock US5790446A (en) Floating point multiplier with reduced critical paths using delay matching techniques Shen et al. The design of easily testable VLSI array multipliers Irwin et al. Digit pipelined processors US4791590A (en) High performance signal processor EP0428327A1 (en) Processor array system US5880985A (en) Efficient combined array for 2n bit n bit multiplications US5347481A (en) Method and apparatus for multiplying denormalized binary floating point numbers without additional delay US6029187A (en) Fast regular multiplier architecture US4737926A (en) Optimally partitioned regenerative carry lookahead adder US4489393A (en) Monolithic discrete-time digital convolution circuit US4769779A (en) Systolic complex multiplier US4982352A (en) Methods and apparatus for determining the absolute value of the difference between binary operands Legal Events Date Code Title Description AK Designated contracting states: Designated state(s): DE FR GB IT AK Designated contracting states: Kind code of ref document: A3 Designated state(s): DE FR GB IT 17P Request for examination filed Effective date: 19870413 17Q First examination report Effective date: 19881013 RAP1 Transfer of rights of an ep published application Owner name: UNISYS CORPORATION AK Designated contracting states: Kind code of ref document: B1 Designated state(s): DE FR GB IT REF Corresponds to: Ref document number: 3381262 Country of ref document: DE Date of ref document: 19900405 Format of ref document f/p: P ITF It: translation for a ep patent filed Owner name: ING. C. GREGORJ S.P.A. ET Fr: translation filed 26N No opposition filed ITTA It: last paid annual fee PGFP Postgrant: annual fees paid to national office Ref country code: GB Payment date: 19961108 Year of fee payment: 14 PGFP Postgrant: annual fees paid to national office Ref country code: FR Payment date: 19961211 Year of fee payment: 14 PGFP Postgrant: annual fees paid to national office Ref country code: DE Payment date: 19961227 Year of fee payment: 14 PG25 Lapsed in a contracting state announced via postgrant inform. from nat. office to epo Ref country code: GB Free format text: LAPSE BECAUSE OF NON-PAYMENT OF DUE FEES Effective date: 19971219 PG25 Lapsed in a contracting state announced via postgrant inform. from nat. office to epo Ref country code: FR Free format text: THE PATENT HAS BEEN ANNULLED BY A DECISION OF A NATIONAL AUTHORITY Effective date: 19971231 GBPC Gb: european patent ceased through non-payment of renewal fee Effective date: 19971219 PG25 Lapsed in a contracting state announced via postgrant inform. from nat. office to epo Ref country code: DE Free format text: LAPSE BECAUSE OF NON-PAYMENT OF DUE FEES Effective date: 19980901 REG Reference to a national code Ref country code: FR Ref legal event code: ST
{ "url": "https://patents.google.com/patent/EP0112186A2/en", "source_domain": "patents.google.com", "snapshot_id": "crawl=CC-MAIN-2018-47", "warc_metadata": { "Content-Length": "125193", "Content-Type": "application/http; msgtype=response", "WARC-Block-Digest": "sha1:E7Q7XND4UUJNNUQLVGHN3KV5TIJATPC5", "WARC-Concurrent-To": "<urn:uuid:df0b6191-efaa-4400-8720-e9dc95713b2f>", "WARC-Date": "2018-11-14T07:18:02Z", "WARC-IP-Address": "172.217.5.238", "WARC-Identified-Payload-Type": "text/html", "WARC-Payload-Digest": "sha1:WOE5L7AIGXWZBPC32QIOCLCRHOZPQ7EW", "WARC-Record-ID": "<urn:uuid:6fb077a4-c3c4-438f-a0a1-490fc5108a23>", "WARC-Target-URI": "https://patents.google.com/patent/EP0112186A2/en", "WARC-Truncated": null, "WARC-Type": "response", "WARC-Warcinfo-ID": "<urn:uuid:2bfdc486-52f4-4441-9bda-6a260214e8c0>" }, "warc_info": "isPartOf: CC-MAIN-2018-47\r\npublisher: Common Crawl\r\ndescription: Wide crawl of the web for November 2018\r\noperator: Common Crawl Admin ([email protected])\r\nhostname: ip-10-67-168-131.ec2.internal\r\nsoftware: Apache Nutch 1.15 (modified, https://github.com/commoncrawl/nutch/)\r\nrobots: checked via crawler-commons 0.11-SNAPSHOT (https://github.com/crawler-commons/crawler-commons)\r\nformat: WARC File Format 1.1\r\nconformsTo: http://iipc.github.io/warc-specifications/specifications/warc-format/warc-1.1/" }
{ "line_start_idx": [ 0, 120, 121, 223, 224, 229, 230, 249, 261, 475, 485, 488, 501, 513, 525, 544, 548, 555, 564, 575, 584, 599, 790, 798, 817, 831, 847, 859, 871, 886, 904, 922, 931, 953, 1126, 1138, 1156, 1168, 1180, 1371, 1383, 1400, 1401, 1407, 1408, 1415, 1416, 1432, 1433, 1448, 1490, 1533, 1650, 1823, 2087, 2123, 2155, 2261, 2491, 2506, 2548, 2591, 2652, 2729, 2841, 2956, 2971, 3013, 3056, 3204, 3277, 3304, 3354, 3384, 3385, 3394, 3395, 3634, 4159, 4364, 4365, 4377, 4378, 5148, 5444, 5851, 6335, 6921, 7359, 8574, 9018, 9619, 11740, 12052, 12302, 12628, 12853, 13230, 13685, 14215, 14393, 15285, 15816, 16096, 17467, 17565, 17652, 17770, 17867, 17949, 18086, 18254, 18384, 18541, 19390, 19906, 20261, 20526, 20899, 21168, 21212, 21232, 21252, 21272, 21292, 21404, 21448, 21468, 21488, 21550, 21963, 22468, 23171, 23731, 24141, 24488, 24873, 25595, 26118, 26633, 26910, 27350, 27493, 27816, 28329, 29425, 29908, 30705, 31191, 31751, 32152, 32308, 32772, 33019, 33232, 33507, 33767, 34024, 34152, 34208, 34228, 34267, 34287, 34307, 34638, 35277, 35839, 35977, 35978, 35990, 35991, 36041, 36070, 36097, 36205, 36333, 36462, 36638, 37039, 37300, 37468, 37639, 37809, 37972, 38204, 38519, 38766, 38976, 39182, 39229, 39305, 39503, 39637, 39980, 40100, 40275, 40446, 40579, 40580, 40689, 40832, 41004, 41157, 41294, 41368, 41486, 41615, 41950, 42111, 42112, 42138, 42139, 42190, 42306, 42326, 42327, 42344, 42345, 42381, 42431, 42476, 42516, 42517, 42524, 42525, 42537, 42538, 42562, 42563, 42614, 42775, 42776, 42795, 42796, 42809, 42832, 42856, 42881, 42905, 42906, 42938, 42939, 42983, 43048, 43102, 43156, 43198, 43199, 43213, 43214, 43258, 43323, 43417, 43544, 43545, 43571, 43572, 43616, 43681, 43825, 44043, 44111, 44152, 44272, 44273, 44294, 44295, 44339, 44404, 44498, 44625, 44626, 44651, 44652, 44696, 44702, 44864, 45030, 45151, 45152, 45170, 45171, 45212, 45246, 45287, 45320, 45354, 45394, 45395, 45413, 45414, 45449, 45524, 45598, 45647, 45703, 45775, 45886, 45985, 46030, 46174, 46229, 46296, 46404, 46478, 46580, 46645, 46685, 46735, 46775, 46849, 46970, 47023, 47096, 47165, 47209, 47324, 47325, 47338, 47339, 47367, 47401, 47402, 47435, 47436, 47470, 47471, 47501, 47502, 47535, 47536, 47570, 47571, 47596, 47597, 47626, 47627, 47652, 47653, 47708, 47709, 47740, 47741, 47775, 47776, 47806, 47807, 47840, 47841, 47861, 47862, 47891, 47892, 47920, 47921, 47952, 47953, 47983, 47984, 48026, 48027, 48062, 48063, 48088, 48112, 48142, 48194, 48195, 48216, 48217, 48240, 48241, 48265, 48266, 48318, 48319, 48340, 48341, 48364, 48365, 48389, 48390, 48442, 48443, 48464, 48465, 48488, 48489, 48513, 48514, 48605, 48606, 48627, 48628, 48687, 48688, 48713, 48714, 48805, 48806, 48827, 48828, 48913, 48914, 48939, 48940, 49007, 49008, 49033, 49034, 49125, 49126, 49147, 49148, 49207, 49208, 49233, 49234, 49267, 49268, 49289, 49290 ], "line_end_idx": [ 120, 121, 223, 224, 229, 230, 249, 261, 475, 485, 488, 501, 513, 525, 544, 548, 555, 564, 575, 584, 599, 790, 798, 817, 831, 847, 859, 871, 886, 904, 922, 931, 953, 1126, 1138, 1156, 1168, 1180, 1371, 1383, 1400, 1401, 1407, 1408, 1415, 1416, 1432, 1433, 1448, 1490, 1533, 1650, 1823, 2087, 2123, 2155, 2261, 2491, 2506, 2548, 2591, 2652, 2729, 2841, 2956, 2971, 3013, 3056, 3204, 3277, 3304, 3354, 3384, 3385, 3394, 3395, 3634, 4159, 4364, 4365, 4377, 4378, 5148, 5444, 5851, 6335, 6921, 7359, 8574, 9018, 9619, 11740, 12052, 12302, 12628, 12853, 13230, 13685, 14215, 14393, 15285, 15816, 16096, 17467, 17565, 17652, 17770, 17867, 17949, 18086, 18254, 18384, 18541, 19390, 19906, 20261, 20526, 20899, 21168, 21212, 21232, 21252, 21272, 21292, 21404, 21448, 21468, 21488, 21550, 21963, 22468, 23171, 23731, 24141, 24488, 24873, 25595, 26118, 26633, 26910, 27350, 27493, 27816, 28329, 29425, 29908, 30705, 31191, 31751, 32152, 32308, 32772, 33019, 33232, 33507, 33767, 34024, 34152, 34208, 34228, 34267, 34287, 34307, 34638, 35277, 35839, 35977, 35978, 35990, 35991, 36041, 36070, 36097, 36205, 36333, 36462, 36638, 37039, 37300, 37468, 37639, 37809, 37972, 38204, 38519, 38766, 38976, 39182, 39229, 39305, 39503, 39637, 39980, 40100, 40275, 40446, 40579, 40580, 40689, 40832, 41004, 41157, 41294, 41368, 41486, 41615, 41950, 42111, 42112, 42138, 42139, 42190, 42306, 42326, 42327, 42344, 42345, 42381, 42431, 42476, 42516, 42517, 42524, 42525, 42537, 42538, 42562, 42563, 42614, 42775, 42776, 42795, 42796, 42809, 42832, 42856, 42881, 42905, 42906, 42938, 42939, 42983, 43048, 43102, 43156, 43198, 43199, 43213, 43214, 43258, 43323, 43417, 43544, 43545, 43571, 43572, 43616, 43681, 43825, 44043, 44111, 44152, 44272, 44273, 44294, 44295, 44339, 44404, 44498, 44625, 44626, 44651, 44652, 44696, 44702, 44864, 45030, 45151, 45152, 45170, 45171, 45212, 45246, 45287, 45320, 45354, 45394, 45395, 45413, 45414, 45449, 45524, 45598, 45647, 45703, 45775, 45886, 45985, 46030, 46174, 46229, 46296, 46404, 46478, 46580, 46645, 46685, 46735, 46775, 46849, 46970, 47023, 47096, 47165, 47209, 47324, 47325, 47338, 47339, 47367, 47401, 47402, 47435, 47436, 47470, 47471, 47501, 47502, 47535, 47536, 47570, 47571, 47596, 47597, 47626, 47627, 47652, 47653, 47708, 47709, 47740, 47741, 47775, 47776, 47806, 47807, 47840, 47841, 47861, 47862, 47891, 47892, 47920, 47921, 47952, 47953, 47983, 47984, 48026, 48027, 48062, 48063, 48088, 48112, 48142, 48194, 48195, 48216, 48217, 48240, 48241, 48265, 48266, 48318, 48319, 48340, 48341, 48364, 48365, 48389, 48390, 48442, 48443, 48464, 48465, 48488, 48489, 48513, 48514, 48605, 48606, 48627, 48628, 48687, 48688, 48713, 48714, 48805, 48806, 48827, 48828, 48913, 48914, 48939, 48940, 49007, 49008, 49033, 49034, 49125, 49126, 49147, 49148, 49207, 49208, 49233, 49234, 49267, 49268, 49289, 49290, 49314 ] }
{ "red_pajama_v2": { "ccnet_original_length": 49314, "ccnet_original_nlines": 421, "rps_doc_curly_bracket": 0, "rps_doc_ldnoobw_words": 0, "rps_doc_lorem_ipsum": 0, "rps_doc_stop_word_fraction": 0.32588455080986023, "rps_doc_ut1_blacklist": 0, "rps_doc_frac_all_caps_words": 0.03548521175980568, "rps_doc_frac_lines_end_with_ellipsis": 0, "rps_doc_frac_no_alph_words": 0.2056693583726883, "rps_doc_frac_unique_words": 0.1807502806186676, "rps_doc_mean_word_length": 4.996589660644531, "rps_doc_num_sentences": 364, "rps_doc_symbol_to_word_ratio": 0.00010345999908167869, "rps_doc_unigram_entropy": 5.933711051940918, "rps_doc_word_count": 7917, "rps_doc_frac_chars_dupe_10grams": 0.12199807912111282, "rps_doc_frac_chars_dupe_5grams": 0.23461751639842987, "rps_doc_frac_chars_dupe_6grams": 0.199100062251091, "rps_doc_frac_chars_dupe_7grams": 0.16411344707012177, "rps_doc_frac_chars_dupe_8grams": 0.14143788814544678, "rps_doc_frac_chars_dupe_9grams": 0.12930381298065186, "rps_doc_frac_chars_top_2gram": 0.016178779304027557, "rps_doc_frac_chars_top_3gram": 0.0060670399107038975, "rps_doc_frac_chars_top_4gram": 0.003943580202758312, "rps_doc_books_importance": -4159.62841796875, "rps_doc_books_importance_length_correction": -4159.62841796875, "rps_doc_openwebtext_importance": -3226.13037109375, "rps_doc_openwebtext_importance_length_correction": -3226.13037109375, "rps_doc_wikipedia_importance": -2510.17822265625, "rps_doc_wikipedia_importance_length_correction": -2510.17822265625 }, "fasttext": { "dclm": 0.04940849915146828, "english": 0.89829421043396, "fineweb_edu_approx": 2.7936768531799316, "eai_general_math": 0.8471310138702393, "eai_open_web_math": 0.4308807849884033, "eai_web_code": 0.07650011777877808 } }
{ "free_decimal_correspondence": { "primary": { "code": "004.16", "labels": { "level_1": "General works, books and libraries, information sciences", "level_2": "", "level_3": "Computers and Computer science" } }, "secondary": { "code": "621.395", "labels": { "level_1": "Industrial arts, Technology, and Engineering", "level_2": "Engineering", "level_3": "Mechanical engineering and Machinery" } } }, "bloom_cognitive_process": { "primary": { "code": "4", "label": "Analyze" }, "secondary": { "code": "-1", "label": "Abstain" } }, "bloom_knowledge_domain": { "primary": { "code": "3", "label": "Procedural" }, "secondary": { "code": "-1", "label": "Abstain" } }, "document_type_v1": { "primary": { "code": "11", "label": "Legal/Regulatory" }, "secondary": { "code": "-1", "label": "Abstain" } }, "extraction_artifacts": { "primary": { "code": "0", "label": "No Artifacts" }, "secondary": { "code": "0", "label": "No Artifacts" } }, "missing_content": { "primary": { "code": "0", "label": "No missing content" }, "secondary": { "code": "0", "label": "No missing content" } }, "document_type_v2": { "primary": { "code": "8", "label": "Documentation" }, "secondary": { "code": "10", "label": "Knowledge Article" } }, "reasoning_depth": { "primary": { "code": "4", "label": "Advanced Reasoning" }, "secondary": { "code": "3", "label": "Intermediate Reasoning" } }, "technical_correctness": { "primary": { "code": "4", "label": "Highly Correct" }, "secondary": { "code": "3", "label": "Mostly Correct" } }, "education_level": { "primary": { "code": "4", "label": "Graduate/Expert Level" }, "secondary": { "code": "3", "label": "Undergraduate Level" } } }
672f1e42c33a7f9846924a2431ea77df
7,828,134,324,842,875,000
12 I am given the task to design a lengthy web form that asks a user to enter a lot of information on a technical installation he planned/built. Given the nature of the required data (there are different chunks, nearly all fields are required, and there are some dependencies that suggest a certain order) I organized into a wizard. However, after dividing the process into clear main steps (the easy part) I was still left with enough input fields to justify further grouping. In this "second level" of data input, however, chunks are not necessarily ordered or dependent and I expect there to be more going back and forth (although most users will just go ahead from group to group). In consequence, I considered a) a tabbed structure which allows "free" jumps, while still hinting at a from-top-to-bottom approach as the standard procedure and b) a secondary wizard-thingy with another set of "substeps", which is more rigid EDIT: c) another possibility would be to create a secondary level right inside the wizard to illustrate the substeps. It would work just as b), but communicate this with different visuals. Mockups for suggestions a and b Mockup for suggestion c This is a similar question to this one, and basically my suggestion is a combination of the two approaches in the accepted answer, however, they are there presented as alternatives, not in combination. It is also one approach to answer this question about long wizards, but one that wasn't discussed there. My questions are: • Does anybody have experience with this kind of two-tiered approach? Are there possible pitfalls I have to be aware of, or is there something completely inadvisable about this? (some reading suggests not to mix tabs and wizards, but this refers mostly to "don't use tabs when the functionality is wizard") • Instead of tabs I also considered an accordion style for the secondary level (much like this jQuery example or this answer here). However, other comments I read around here state that accordions are "lame" or "old school" or "only for mobile" without much detail beyond that. So what would be actual drawbacks in this given situation? • I currently lean towards the rather rigid two-level wizard. As I said, most fields are required and important and I have not yet a found a need to jump ahead. In this case I wonder how to design the forward/backward navigation. The current Next button is supposed to jump to the next substep, while the last step contains the "to the next main step" button. However, I'm not quite satisfied with this yet as it may confuse users. Any ideas, experiences? • Funny tidbit: I managed to not notice the humongous red alert that said you can't post images, and only got suspicious when my question didn't show up in the user profile... Thank god there's a back button... And so much for users and warning alerts. – Louise Jun 26 '12 at 12:46 • Do you allow moving back and forth in the process? Perhaps you can take some inspiration from the way complicated electronic tax forms like the one we have in the Netherlands work. I find them easy to use here, even though they are complicated and require many fields to be entered in some cases. – André Jun 26 '12 at 13:00 • @André: The requirements do not specify this issue. From some of the dependencies it is clear that we (as designers) have trouble in deciding what to display should the user skip ahead > hence the strict wizard pattern. Going back to correct something has to be possible, however. Are the forms you mention available online somewhere? (from Germany ;) ) – Louise Jun 26 '12 at 15:42 • You should be able to download the software from belastingdienst.nl/wps/wcm/connect/bldcontentnl/belastingdienst/… – André Jun 27 '12 at 8:55 • The only difference between your three examples is the design. Personally I'd prefer a combination of the 2nd and the 3rd (the 2nd with checkmarks to the left of the tab headers). – Danny Varod Jun 27 '12 at 13:52 2 If I had to choose I would vote for the third of the presented options. It has more intuitive progress indicator that feels the most natural (in the set of proposals you presented). If I had to suggest an approach to a long wizard problem I would propose to reverse the relation: do not force them to fill all the required information, just allow to add an installation info entry and mark it as not complete ("your installation info is 10% complete"). This allows a user to enter the information in piece-by-piece style while she is pulled towards completion of the information, rather than forcing to fill all the info at once. It also simplifies a design of the installation "profile" page. | improve this answer | | • Your suggestion sounds great. However, the client explicitly rejected a sign up/login solution, as the process is one you would go through every other year (or even only once) and only for one project at a time. Hence there will be no "member profile" where one has an overview over various data records and their completion status. If we had, I would definitely agree with your approach to loosen the step-by-step-rule. – Louise Jun 27 '12 at 11:36 • Once-a-year condition definitely changes the situation :) Just remember about a "back" button. – Bartosz Rakowski Jun 27 '12 at 11:52 1 Who says accordian are oldstyle? I am very much in favour of accordion style wizards that gradually reveal the questions and also provide feedback about the answers already given. I did one of these for a financial questionaire for a client: enter image description here They are also used in a lot of well designed checkouts like the one at harrods and John Lewis. Benefits are: • You can progressively disclose questions depending on answers (so you can change progression through the wizard. • Users have a good understanding of where they are at all times. • Users have an overview of the answers already given and can easily return to them. I think this covers your forward/backward navigation issue. • It makes the length of the wizard feel shorter, as questions are all presented on one page. | improve this answer | | • I would question the assumption that an accordion "makes the length of the wizard feel shorter, as questions are all presented on one page." Presenting questions piece-by-piece as in a tabbed wizard can make the process seem less burdensome, and also submitting the first step in a tabbed wizard creates a sense of commitment to the process (resulting in decreased abandonment). See The Secret Objectives of Queues - the point "It commits guests to the attraction". – Mike Eng Jul 9 '12 at 16:20 • Please feel free to add an answer to my bounty question about when not to use an accordion (: Sounds like you have an interesting perspective. – Lisa Tweedie Jul 9 '12 at 16:31 0 Based on your wire-frames above, I believe you are already on the right track. I would suggest though to create a smaller similar sub-wizard underneath the primary wizard bar to keep page consistence. With some slight tweaks that can be seen below, you can supply additional clarity of where a user currently is in the process with a simple arrow. To allow navigation between the different steps just turn the title of each step into a link. mockup download bmml source – Wireframes created with Balsamiq Mockups | improve this answer | | • 4 I find this design confusing. It isn't imeadiately obvious which step the substeps relate to. – Lisa Tweedie Jun 27 '12 at 15:02 • 1 @Lisa: Agreed. I think the secondary level should deliberately look different so that it is not confused with the primary one. For this approach I'd prefer my third mockup, which marks the subsets as subordinate and puts them right under the corresponding main step. But thanks for your thoughts! – Louise Jun 28 '12 at 11:20 Your Answer By clicking “Post Your Answer”, you agree to our terms of service, privacy policy and cookie policy Not the answer you're looking for? Browse other questions tagged or ask your own question.
{ "url": "https://ux.stackexchange.com/questions/22867/if-and-how-to-break-a-long-wizard-into-main-steps-and-substeps?noredirect=1", "source_domain": "ux.stackexchange.com", "snapshot_id": "crawl=CC-MAIN-2020-40", "warc_metadata": { "Content-Length": "180373", "Content-Type": "application/http; msgtype=response", "WARC-Block-Digest": "sha1:WNVH2GSZWTRJXIVFQVNODAXQVQE7F32L", "WARC-Concurrent-To": "<urn:uuid:5e06de9c-6081-4baa-9848-bdcde42aae88>", "WARC-Date": "2020-09-30T05:51:58Z", "WARC-IP-Address": "151.101.1.69", "WARC-Identified-Payload-Type": "text/html", "WARC-Payload-Digest": "sha1:IXNC6CL52JFQNK3ZOYZNFH26UTHAHJA2", "WARC-Record-ID": "<urn:uuid:f24b6461-4a62-4818-8a10-73356cd69b3a>", "WARC-Target-URI": "https://ux.stackexchange.com/questions/22867/if-and-how-to-break-a-long-wizard-into-main-steps-and-substeps?noredirect=1", "WARC-Truncated": null, "WARC-Type": "response", "WARC-Warcinfo-ID": "<urn:uuid:0d57cf68-128f-45b2-8d63-35a74a70b005>" }, "warc_info": "isPartOf: CC-MAIN-2020-40\r\npublisher: Common Crawl\r\ndescription: Wide crawl of the web for September 2020\r\noperator: Common Crawl Admin ([email protected])\r\nhostname: ip-10-67-67-108.ec2.internal\r\nsoftware: Apache Nutch 1.17 (modified, https://github.com/commoncrawl/nutch/)\r\nrobots: checked via crawler-commons 1.2-SNAPSHOT (https://github.com/crawler-commons/crawler-commons)\r\nformat: WARC File Format 1.1\r\nconformsTo: http://iipc.github.io/warc-specifications/specifications/warc-format/warc-1.1/" }
{ "line_start_idx": [ 0, 3, 4, 479, 480, 717, 718, 850, 851, 932, 933, 1122, 1123, 1179, 1180, 1487, 1488, 1506, 1507, 1816, 2155, 2613, 2897, 3226, 3613, 3759, 3977, 3979, 3980, 4162, 4163, 4675, 4676, 4702, 5156, 5294, 5296, 5297, 5477, 5478, 5569, 5570, 5665, 5666, 5680, 5681, 5798, 5866, 6013, 6109, 6135, 6635, 6816, 6818, 6819, 7261, 7262, 7269, 7270, 7334, 7335, 7361, 7367, 7500, 7506, 7836, 7837, 7849, 7850, 7950, 7951 ], "line_end_idx": [ 3, 4, 479, 480, 717, 718, 850, 851, 932, 933, 1122, 1123, 1179, 1180, 1487, 1488, 1506, 1507, 1816, 2155, 2613, 2897, 3226, 3613, 3759, 3977, 3979, 3980, 4162, 4163, 4675, 4676, 4702, 5156, 5294, 5296, 5297, 5477, 5478, 5569, 5570, 5665, 5666, 5680, 5681, 5798, 5866, 6013, 6109, 6135, 6635, 6816, 6818, 6819, 7261, 7262, 7269, 7270, 7334, 7335, 7361, 7367, 7500, 7506, 7836, 7837, 7849, 7850, 7950, 7951, 8041 ] }
{ "red_pajama_v2": { "ccnet_original_length": 8041, "ccnet_original_nlines": 70, "rps_doc_curly_bracket": 0, "rps_doc_ldnoobw_words": 0, "rps_doc_lorem_ipsum": 0, "rps_doc_stop_word_fraction": 0.4271903336048126, "rps_doc_ut1_blacklist": 0, "rps_doc_frac_all_caps_words": 0.018731119111180305, "rps_doc_frac_lines_end_with_ellipsis": 0, "rps_doc_frac_no_alph_words": 0.18912386894226074, "rps_doc_frac_unique_words": 0.3898181915283203, "rps_doc_mean_word_length": 4.607999801635742, "rps_doc_num_sentences": 66, "rps_doc_symbol_to_word_ratio": 0.0018126900540664792, "rps_doc_unigram_entropy": 5.582308292388916, "rps_doc_word_count": 1375, "rps_doc_frac_chars_dupe_10grams": 0.022411620244383812, "rps_doc_frac_chars_dupe_5grams": 0.03377525135874748, "rps_doc_frac_chars_dupe_6grams": 0.027462119236588478, "rps_doc_frac_chars_dupe_7grams": 0.022411620244383812, "rps_doc_frac_chars_dupe_8grams": 0.022411620244383812, "rps_doc_frac_chars_dupe_9grams": 0.022411620244383812, "rps_doc_frac_chars_top_2gram": 0.008680559694766998, "rps_doc_frac_chars_top_3gram": 0.005523989908397198, "rps_doc_frac_chars_top_4gram": 0.007102270144969225, "rps_doc_books_importance": -605.2092895507812, "rps_doc_books_importance_length_correction": -605.2092895507812, "rps_doc_openwebtext_importance": -368.7077331542969, "rps_doc_openwebtext_importance_length_correction": -368.7077331542969, "rps_doc_wikipedia_importance": -278.66717529296875, "rps_doc_wikipedia_importance_length_correction": -278.66717529296875 }, "fasttext": { "dclm": 0.0367741584777832, "english": 0.9485025405883789, "fineweb_edu_approx": 1.4870160818099976, "eai_general_math": 0.6234839558601379, "eai_open_web_math": 0.4139953851699829, "eai_web_code": 0.05394821986556053 } }
{ "free_decimal_correspondence": { "primary": { "code": "005.857", "labels": { "level_1": "General works, books and libraries, information sciences", "level_2": "", "level_3": "Computer programming" } }, "secondary": { "code": "005.1", "labels": { "level_1": "General works, books and libraries, information sciences", "level_2": "", "level_3": "Computer programming" } } }, "bloom_cognitive_process": { "primary": { "code": "4", "label": "Analyze" }, "secondary": { "code": "3", "label": "Apply" } }, "bloom_knowledge_domain": { "primary": { "code": "2", "label": "Conceptual" }, "secondary": { "code": "3", "label": "Procedural" } }, "document_type_v1": { "primary": { "code": "5", "label": "Social/Forum" }, "secondary": { "code": "-1", "label": "Abstain" } }, "extraction_artifacts": { "primary": { "code": "3", "label": "Irrelevant Content" }, "secondary": { "code": "0", "label": "No Artifacts" } }, "missing_content": { "primary": { "code": "4", "label": "Missing Images or Figures" }, "secondary": { "code": "0", "label": "No missing content" } }, "document_type_v2": { "primary": { "code": "18", "label": "Q&A Forum" }, "secondary": { "code": "23", "label": "Tutorial" } }, "reasoning_depth": { "primary": { "code": "3", "label": "Intermediate Reasoning" }, "secondary": { "code": "2", "label": "Basic Reasoning" } }, "technical_correctness": { "primary": { "code": "6", "label": "Not Applicable/Indeterminate" }, "secondary": { "code": "3", "label": "Mostly Correct" } }, "education_level": { "primary": { "code": "2", "label": "High School Level" }, "secondary": { "code": "3", "label": "Undergraduate Level" } } }
672f1e42c33a7f9846924a2431ea77df
6,933,377,339,255,727,000
Common Magento Errors and How to Fix Them Common Magento Errors and How to Fix Them Magento errors are one of the main reasons for an undesired ecommerce experience for your customers. You may have encountered various error messages if you're a Magento store owner or developer. Magento is a popular choice for most ecommerce store owners, and errors can sometimes be inevitable. But you can fix them for an improved user experience. This post outlines some common errors your Magento site may incur. It will offer practical solutions to help streamline your website. Key Takeaways • Learn how to diagnose and fix common Magento errors. • Understand error log handling and error reporting for improving website performance and user experience. • Stay up-to-date with the latest Magento version for bug fixes and performance improvements. • Choose reliable Magento hosting for faster issue resolution and optimized site performance. What are Magento errors? Magento errors refer to the issues within a Magento site. Errors cause disruptions or malfunctions in the site's functionality. Error logs in Magento 2 You can find error logs in the Magento 2 browser by enabling the developer mode. After installing Magento 2, set the developer mode using the following command: php bin/magento deploy:mode: set developer You can check critical errors in the var/log and var/reports folder. Also, PHP errors are visible in the browser. You can enable error reporting from index.php in the Magento root directory index.php. Adding the following code after the starting of <?php should help: ini_set('display_errors', 1); error_reporting(E_ALL); Common Magento 2 Server Errors 1. Internal Server Error 500 Magento internal server error is one of the most common Magento issues. It appears when the server can not reach the requested page. To solve the issue, you can reload the page. You must access the server logs to find more information if it doesn't work. Magento Internal Server Error 500 displayed on screen To solve the issue, you can: • Change file permissions. • Check the htaccess file for errors. Rename the file. • Increase PHP memory_limit in php.ini. • Disable third-party Magento extensions 2. 301 Redirect Error When you change the URL of a page on your website, setting up a 301 redirect is crucial. It allows redirecting visitors and search engines to the new URL. Prevent 301 errors by reviewing and correcting your URL rewrites in Magento admin. To create a 301 redirect in Magento, you can use the built-in Redirects feature. 3. Magento Access Denied HTTP Error 403 Magento Access Denied HTTP Error 403 restricts access to certain resources, disrupting the website. It occurs due to incorrect file or folder permissions. Magento Access Denied HTTP Error 403 warning To fix it, ensure correct permissions (typically 755 for directories and 644 for files). Also, verify file and folder permissions in your Magento directory. 4. PHP Error (Memory Limit Exceeded) PHP errors in Magento 2 can cause website malfunctions. You must set aside at least 2GB of memory for PHP processes and debugging. To fix the exceeded memory limit, review error logs and increase memory_limit in php.ini. 1. Log in to your Adobe Commerce server. 2. Find the php.ini file using the following command: bash $ php --ini 1. Use a text editor to open the php.ini specified by Loaded Configuration File. 2. Find the memory_limit option. 3. Change it to 2GB for normal use and debugging. 4. Save the changes to php.ini and exit the text editor. 5. Restart your web server. 6. Try the installation again. Common Magento Backend Errors 1. Error Processing your Request You can fix the ‘error processing request’ issue in the following ways: • Flush Magento Cache • Manually read the log files in /var/report • Enable exception printing errors 2. Invalid Form Key An Invalid form key error appears when you update Magento, create configurable products, or add a third-party extension. It happens when you want to pass huge amounts of data to the server to submit a form. To solve the error, you can: • Increase the max_input_vars in PHP. • Change the base URL of your local environment to 127.0.0.1. • Improve Magento speed so all Javascripts are loaded fast enough. 3. No Such Entity Occurs if you try to load a non-existing object in the Magento 2 Repository Class. To debug the error, open the following file: vendor/magento/framework/Exception/NoSuchEntityException.php Then add the debugging debug backtrace code: php public function __construct (Phrase $phrase = null, \Exception $cause = null, $code = 0) { foreach (debug_backtrace() as $_stack) { echo ($_stack["file"] ? $_stack["file"] : '') . ':'. ($_stack["line"] ? $_stack["line"] : '') . ' - '. ($_stack["function"] ? $_stack["function"] : ''); } exit(); if ($phrase === null) { $phrase = new Phrase('No such entity.'); } 4. 503 Service Temporarily Unavailable Error Error 503 Service Temporarily Unavailable is an error that may occur when you try to access the Magento admin panel or storefront after installation. It can result from installing or updating Magento 2 extensions or a new version of Magento. The web server can only handle your request after some time, or it might be misconfigured. As a result, the following message appears on your screen: "The server is temporarily unable to service your request due to maintenance downtime or capacity problems. Please try again later". 503 Service Temporarily Unavailable Error in Magento To fix the error, go to your Magento root directory and Clean your Magento cache by running the following command: php bin/magento cache: flush 5. Permission Denied Errors Permission denied occurs when you run a script or file that does not have “execute” permissions. Follow the steps to fix the error: 1. Open terminal (shell) 2. Find the folder with the script 3. Run the command and change file permission settings: chmod +x path_to_file/file_name 6. Compilation Errors You can fix compilation errors in Magento 2 by disabling the compilation mode in the admin panel: 1. Go to System > Tools > Cache Management> Compiled Configuration. 2. Click "Disable" and then refresh the cache. Compilation Errors in Magento admin panel 7. Invalid XML Configuration Invalid XML Configuration Issues in Magento can disrupt the website's functionality. It leads to misconfigurations that cause site malfunctions. Fix the issue by reviewing and correcting XML configuration files for syntax errors and compliance. 8. Magento Error for Outdated Modules An error for outdated modules occurs when modules are incompatible with your current Magento version. To fix it, update or replace outdated modules with versions that match your Magento version. 9. Backend Session Timeout By default, Magento’s backend session lifetime is set at 900 seconds. Update it to access the backend panel longer while maintaining Magento security. Backend session timeout message in Magento Fixing Magento 2 Errors to Optimize Site Speed 1. SSL/TLS Certificate Issues: Outdated SSL/TLS Certificates introduce latency due to the Magento encryption process. They also impact a Magento website's SEO performance. You can fix the error by renewing or reinstalling SSL/TLS certificates. Choose a robust Magento hosting solution to ensure optimized website performace. 2. Fix Broken or Missing JavaScript Broken Javascript and CSS files affect your Magento store’s performance. Fix the issue to improve the loading time of your website and user experience. To fix the issue, you can: • Minify the Javascript and CSS files by removing unnecessary spaces and characters. • Combine multiple Javascript or CSS files into one, reducing the number of requests made by the browser. 3. Enable Varnish Cache Enabling a varnish cache helps store a copy of frequently requested pages in memory. When a user visits the site again, Varnish serves the cached version instead of generating it from scratch. It reduces server load and response time and provides a smoother browsing experience for your customers. To enable the Varnish cache in Magento 2, configure it in the admin panel: Go to Stores > Configuration > Advanced > System > Full Page Cache > Varnish. Enabling Varnish Cache in Magento settings 4. Enable Flat Tables By enabling flat tables, you can optimize database queries and reduce the loading time for product pages. It is especially helpful if you have a large catalog with many products. To enable flat tables in Magento, you can: 1. Go to the admin panel. 2. Navigate to Stores > Configuration > Catalog > Catalog > Storefront. 3. In the "Use Flat Catalog Category" and "Use Flat Catalog Product" settings, set them to "Yes." Activating flat tables in Magento admin panel 1. Enabling flat tables may require reindexing your data for the changes to take effect. 2. Go to System > Index Management and select all indexes related to catalog data. 3. Click "Reindex Data" from the Actions dropdown menu and wait for the process to complete. 5. Update to the Latest Magento Version Upgrading to the latest Magento version fixes bugs and issues from older versions. Updating the Magento version can also make your store faster. 1. Go to your admin panel and click 'System.' Updating to the latest Magento version via admin panel 1. Then, choose 'Web Setup Wizard.' 2. Next, select 'System Upgrade'. Pick the newest version. Click 'Next' till you see 'Start Update.’ Create a backup of your data before updating. 3. Click ‘Start update.’ FAQs 1. How can I fix a Magento error reporting message? To resolve a Magento error message, find the exception logs. Then, enable error reporting in Magento and troubleshoot the issue. You should also check error log files and troubleshoot broken content. 2. How can I resolve permission issues in admin configuration to avoid Magento 2 errors? When working with Magento, ensure proper permissions for files and directories. You can also switch Magento to developer mode. If you encounter a Magento error, run PHP with error reporting to display error messages. Consider optimizing Magento 2 cache settings. 3. How can I find the error log record number for a white page issue in Magento's open-source ecommerce platform? To diagnose a white page problem in Magento, check the error log in the root folder. The Magento issue's error message, record number, and HTTP status code can be found there. Summary Magento errors might adversely affect a website’s performance and brand reputation. Check your Magento 2 admin panel for website errors after installing or upgrading extensions. You can fix the backend errors with solutions provided by Adobe Commerce and the information from this post. Staying updated with errors and resolving them is crucial for a smooth website experience. A robust hosting solution assures faster resolution of service-side errors. Choose a reliable Magento hosting service and explore your Magento store’s potential. Vaishali Sharma Vaishali Sharma Technical Writer As a seasoned writer, Vaishali consistently expands her knowledge in Magento hosting. She has four years of professional experience, with a focus on crafting diverse Magento-related content. Get the fastest Magento Hosting! Get Started
{ "url": "https://www.mgt-commerce.com/tutorial/magento-errors/", "source_domain": "www.mgt-commerce.com", "snapshot_id": "CC-MAIN-2024-18", "warc_metadata": { "Content-Length": "36253", "Content-Type": "application/http; msgtype=response", "WARC-Block-Digest": "sha1:IW3IAYGX4J6K5PTJTNDKAJON7AISWGNL", "WARC-Concurrent-To": "<urn:uuid:4130fb53-4d19-4f1b-8a83-cae40bc66ca9>", "WARC-Date": "2024-04-22T15:34:39Z", "WARC-IP-Address": "13.32.151.109", "WARC-Identified-Payload-Type": "text/html", "WARC-Payload-Digest": "sha1:GQDYRRF7VW3BG7MJTZ4CSU2WYO2B5JMA", "WARC-Record-ID": "<urn:uuid:897be3ae-2687-4f4e-8124-07816386baf4>", "WARC-Target-URI": "https://www.mgt-commerce.com/tutorial/magento-errors/", "WARC-Truncated": null, "WARC-Type": "response", "WARC-Warcinfo-ID": "<urn:uuid:ce376890-c381-46bb-8b71-5b9923d4f7b4>" }, "warc_info": "isPartOf: CC-MAIN-2024-18\r\npublisher: Common Crawl\r\ndescription: Wide crawl of the web for April 2024\r\noperator: Common Crawl Admin ([email protected])\r\nhostname: ip-10-67-67-82\r\nsoftware: Apache Nutch 1.19 (modified, https://github.com/commoncrawl/nutch/)\r\nrobots: checked via crawler-commons 1.5-SNAPSHOT (https://github.com/crawler-commons/crawler-commons)\r\nformat: WARC File Format 1.1\r\nconformsTo: https://iipc.github.io/warc-specifications/specifications/warc-format/warc-1.1/" }
{ "line_start_idx": [ 0, 42, 43, 85, 86, 570, 571, 585, 586, 643, 752, 848, 944, 945, 970, 971, 1099, 1100, 1124, 1125, 1286, 1287, 1288, 1331, 1332, 1600, 1601, 1602, 1656, 1657, 1688, 1689, 1718, 1719, 1974, 1975, 2029, 2030, 2059, 2060, 2089, 2090, 2147, 2148, 2190, 2191, 2234, 2235, 2257, 2258, 2577, 2578, 2618, 2619, 2774, 2775, 2820, 2821, 2978, 2979, 3016, 3017, 3148, 3149, 3239, 3240, 3283, 3284, 3340, 3341, 3358, 3359, 3442, 3443, 3478, 3479, 3531, 3532, 3591, 3592, 3622, 3623, 3656, 3657, 3687, 3688, 3721, 3722, 3794, 3795, 3819, 3820, 3867, 3868, 3905, 3906, 3926, 3927, 4163, 4164, 4204, 4205, 4269, 4270, 4339, 4340, 4358, 4359, 4487, 4488, 4489, 4550, 4551, 4552, 4597, 4598, 4599, 4692, 4694, 4739, 4800, 4862, 4924, 4930, 4942, 4943, 4971, 5020, 5026, 5027, 5072, 5073, 5465, 5466, 5467, 5600, 5601, 5602, 5655, 5656, 5657, 5772, 5773, 5802, 5803, 5831, 5832, 5964, 5965, 5992, 5993, 6030, 6031, 6089, 6090, 6122, 6123, 6145, 6146, 6244, 6245, 6315, 6316, 6365, 6366, 6408, 6409, 6438, 6439, 6684, 6685, 6723, 6724, 6919, 6920, 6947, 6948, 7099, 7100, 7143, 7144, 7191, 7192, 7223, 7224, 7518, 7519, 7555, 7556, 7708, 7709, 7736, 7737, 7824, 7825, 7933, 7934, 7958, 7959, 8257, 8258, 8259, 8334, 8335, 8413, 8414, 8457, 8458, 8480, 8481, 8660, 8661, 8704, 8705, 8733, 8734, 8808, 8809, 8909, 8910, 8956, 8957, 9048, 9049, 9134, 9135, 9230, 9231, 9271, 9272, 9417, 9418, 9466, 9467, 9522, 9523, 9561, 9562, 9711, 9712, 9739, 9740, 9745, 9746, 9798, 9799, 9999, 10000, 10001, 10090, 10091, 10354, 10355, 10356, 10470, 10471, 10647, 10648, 10656, 10657, 10944, 10945, 10946, 11199, 11200, 11216, 11232, 11249, 11250, 11441, 11442, 11443 ], "line_end_idx": [ 42, 43, 85, 86, 570, 571, 585, 586, 643, 752, 848, 944, 945, 970, 971, 1099, 1100, 1124, 1125, 1286, 1287, 1288, 1331, 1332, 1600, 1601, 1602, 1656, 1657, 1688, 1689, 1718, 1719, 1974, 1975, 2029, 2030, 2059, 2060, 2089, 2090, 2147, 2148, 2190, 2191, 2234, 2235, 2257, 2258, 2577, 2578, 2618, 2619, 2774, 2775, 2820, 2821, 2978, 2979, 3016, 3017, 3148, 3149, 3239, 3240, 3283, 3284, 3340, 3341, 3358, 3359, 3442, 3443, 3478, 3479, 3531, 3532, 3591, 3592, 3622, 3623, 3656, 3657, 3687, 3688, 3721, 3722, 3794, 3795, 3819, 3820, 3867, 3868, 3905, 3906, 3926, 3927, 4163, 4164, 4204, 4205, 4269, 4270, 4339, 4340, 4358, 4359, 4487, 4488, 4489, 4550, 4551, 4552, 4597, 4598, 4599, 4692, 4694, 4739, 4800, 4862, 4924, 4930, 4942, 4943, 4971, 5020, 5026, 5027, 5072, 5073, 5465, 5466, 5467, 5600, 5601, 5602, 5655, 5656, 5657, 5772, 5773, 5802, 5803, 5831, 5832, 5964, 5965, 5992, 5993, 6030, 6031, 6089, 6090, 6122, 6123, 6145, 6146, 6244, 6245, 6315, 6316, 6365, 6366, 6408, 6409, 6438, 6439, 6684, 6685, 6723, 6724, 6919, 6920, 6947, 6948, 7099, 7100, 7143, 7144, 7191, 7192, 7223, 7224, 7518, 7519, 7555, 7556, 7708, 7709, 7736, 7737, 7824, 7825, 7933, 7934, 7958, 7959, 8257, 8258, 8259, 8334, 8335, 8413, 8414, 8457, 8458, 8480, 8481, 8660, 8661, 8704, 8705, 8733, 8734, 8808, 8809, 8909, 8910, 8956, 8957, 9048, 9049, 9134, 9135, 9230, 9231, 9271, 9272, 9417, 9418, 9466, 9467, 9522, 9523, 9561, 9562, 9711, 9712, 9739, 9740, 9745, 9746, 9798, 9799, 9999, 10000, 10001, 10090, 10091, 10354, 10355, 10356, 10470, 10471, 10647, 10648, 10656, 10657, 10944, 10945, 10946, 11199, 11200, 11216, 11232, 11249, 11250, 11441, 11442, 11443, 11487 ] }
{ "red_pajama_v2": { "ccnet_original_length": 11487, "ccnet_original_nlines": 271, "rps_doc_curly_bracket": 0.000435269990703091, "rps_doc_ldnoobw_words": 0, "rps_doc_lorem_ipsum": 0, "rps_doc_stop_word_fraction": 0.25663313269615173, "rps_doc_ut1_blacklist": 0, "rps_doc_frac_all_caps_words": 0.016010979190468788, "rps_doc_frac_lines_end_with_ellipsis": 0, "rps_doc_frac_no_alph_words": 0.21225984394550323, "rps_doc_frac_unique_words": 0.3059023916721344, "rps_doc_mean_word_length": 5.098751544952393, "rps_doc_num_sentences": 177, "rps_doc_symbol_to_word_ratio": 0, "rps_doc_unigram_entropy": 5.4656171798706055, "rps_doc_word_count": 1762, "rps_doc_frac_chars_dupe_10grams": 0, "rps_doc_frac_chars_dupe_5grams": 0.04808548837900162, "rps_doc_frac_chars_dupe_6grams": 0.022595729678869247, "rps_doc_frac_chars_dupe_7grams": 0.007569009903818369, "rps_doc_frac_chars_dupe_8grams": 0.007569009903818369, "rps_doc_frac_chars_dupe_9grams": 0, "rps_doc_frac_chars_top_2gram": 0.010685659945011139, "rps_doc_frac_chars_top_3gram": 0.005008900072425604, "rps_doc_frac_chars_top_4gram": 0.010240430012345314, "rps_doc_books_importance": -1049.87255859375, "rps_doc_books_importance_length_correction": -1049.87255859375, "rps_doc_openwebtext_importance": -645.3983154296875, "rps_doc_openwebtext_importance_length_correction": -645.3983154296875, "rps_doc_wikipedia_importance": -569.7548828125, "rps_doc_wikipedia_importance_length_correction": -569.7548828125 }, "fasttext": { "dclm": 0.20448803901672363, "english": 0.7732802033424377, "fineweb_edu_approx": 1.3395934104919434, "eai_general_math": 0.07596033811569214, "eai_open_web_math": 0.07694519311189651, "eai_web_code": 0.5319820046424866 } }
{ "free_decimal_correspondence": { "primary": { "code": "005.467", "labels": { "level_1": "General works, books and libraries, information sciences", "level_2": "", "level_3": "Computer programming" } }, "secondary": { "code": "658.85", "labels": { "level_1": "Industrial arts, Technology, and Engineering", "level_2": "Business", "level_3": "Management" } } }, "bloom_cognitive_process": { "primary": { "code": "3", "label": "Apply" }, "secondary": { "code": "2", "label": "Understand" } }, "bloom_knowledge_domain": { "primary": { "code": "3", "label": "Procedural" }, "secondary": { "code": "2", "label": "Conceptual" } }, "document_type_v1": { "primary": { "code": "3", "label": "Reference/Encyclopedic/Educational" }, "secondary": { "code": "-1", "label": "Abstain" } }, "extraction_artifacts": { "primary": { "code": "0", "label": "No Artifacts" }, "secondary": { "code": "3", "label": "Irrelevant Content" } }, "missing_content": { "primary": { "code": "0", "label": "No missing content" }, "secondary": { "code": "2", "label": "Click Here References" } }, "document_type_v2": { "primary": { "code": "23", "label": "Tutorial" }, "secondary": { "code": "21", "label": "Customer Support" } }, "reasoning_depth": { "primary": { "code": "2", "label": "Basic Reasoning" }, "secondary": { "code": "3", "label": "Intermediate Reasoning" } }, "technical_correctness": { "primary": { "code": "4", "label": "Highly Correct" }, "secondary": { "code": "3", "label": "Mostly Correct" } }, "education_level": { "primary": { "code": "3", "label": "Undergraduate Level" }, "secondary": { "code": "2", "label": "High School Level" } } }
672f1e42c33a7f9846924a2431ea77df
9,157,579,293,903,791,000
Take the 2-minute tour × Super User is a question and answer site for computer enthusiasts and power users. It's 100% free, no registration required. I've tried following the instructions here and here. Currently, I can read email from my GMail account, but not send—this is on a personal computer, so even if I configured Sendmail, I doubt any of the mail would survive the spam filter gauntlet. Both guides seem to require the --enable-smtp option for mutt, which I don't think is being exposed by ports. I've tried the mutt and mutt-devel ports and neither seem to give me that option. On a far less important note, I also haven't been able to change folders, e.g., I get told that Sent Mail is not a valid folder when I attempt to go into it. If you can troubleshoot this problem, it's pure gravy. I really only care about sending. Thanks, Hank UPDATE: For those who are interested, the mutt wiki has a GMailOverIMAP UserStory. UPDATE 2: Although I finally solved my original problem (enabling SMTP support for mutt in FreeBSD), @grawity convinced me to use a relay and avoid mutt's native SMTP support. With that in mind, I went with msmtp. At that point, The Quick-N-Dirty Guide to Using mutt with gmail was very useful, including tips on how to enable OpenSSL for msmtp (so it can talk to GMail's SMTP servers, which use TLS). share|improve this question 3 Answers 3 up vote 5 down vote accepted I wouldn't use the SMTP support in Mutt. <subjective> It's just not the Unix way. Much better is </subjective> to configure something like esmtp or msmtp or maybe nullmailer to relay email through Gmail's servers, and then let Mutt use the standard /usr/bin/sendmail way of sending messages. share|improve this answer 1   +1 for good suggestions, and for the tricky full-stop-spanning subjective tag. –  coneslayer Apr 21 '10 at 18:54 1   How likely is it that I'm going to get nailed as spam because I don't have a PTR record, my IP changes, etc.? –  Hank Gay Apr 21 '10 at 19:07      Hank: You're still sending through Gmail's servers (that's what "relay" means). You'll just let a separate program handle that, instead of trying to make mutt do everything. So to answer the question, it won't affect spam score at all. –  grawity Apr 21 '10 at 19:54      Do you also recommend using an MRA? –  Hank Gay Apr 21 '10 at 20:17      Hank: Depends on whether you prefer all mail to be stored locally (~/mail/) or in Gmail's servers. Most MRAs do not support two-way synchronization, so using one would mean deleting mail from gmail.com after retrieval. Exception: offlineimap can work in both directions, and is useful because mutt doesn't cache IMAP messages. So you choose this time. –  grawity Apr 21 '10 at 20:47 If that config option is the issue (I don't know one way or the other), try building the mail/mutt port with: $ make clean $ make MUTT_CONFIGURE_ARGS=--enable-smtp $ make install share|improve this answer      I still get errors telling me the smtp_url etc. are invalid after following these steps. I found the --enable-smtp option in the comments to shreevatsa.wordpress.com/2007/07/31/… but I could just be barking up the wrong tree. –  Hank Gay Apr 21 '10 at 15:52 The following sequence seems to work (hat-tip to The Quick-N-Dirty Guide to Using mutt with gmail, where I learned the -Doption trick for ports). 1. cd /usr/ports/mail/mutt-devel 2. make -DWITH_MUTT_SMTP install clean share|improve this answer Your Answer   discard By posting your answer, you agree to the privacy policy and terms of service. Not the answer you're looking for? Browse other questions tagged or ask your own question.
{ "url": "http://superuser.com/questions/133316/has-anyone-successfully-configured-mutt-for-gmail-on-freebsd-if-so-how/133324", "source_domain": "superuser.com", "snapshot_id": "crawl=CC-MAIN-2014-49", "warc_metadata": { "Content-Length": "82110", "Content-Type": "application/http; msgtype=response", "WARC-Block-Digest": "sha1:EGMG3BW2NIJ3KPA6SP3Z5FOSWEO3CKOQ", "WARC-Concurrent-To": "<urn:uuid:0961d2af-82cf-4ea2-a513-a11262b75826>", "WARC-Date": "2014-11-28T17:19:48Z", "WARC-IP-Address": "198.252.206.16", "WARC-Identified-Payload-Type": null, "WARC-Payload-Digest": "sha1:BRF67JE3HAUNKY4N6XH77DN7KDMJB7ZD", "WARC-Record-ID": "<urn:uuid:5b61e069-a07c-4d15-b1ba-1074bd5f54ce>", "WARC-Target-URI": "http://superuser.com/questions/133316/has-anyone-successfully-configured-mutt-for-gmail-on-freebsd-if-so-how/133324", "WARC-Truncated": null, "WARC-Type": "response", "WARC-Warcinfo-ID": "<urn:uuid:46af3605-87d9-433b-9792-5ff65bb06db2>" }, "warc_info": "robots: classic\r\nhostname: ip-10-235-23-156.ec2.internal\r\nsoftware: Nutch 1.6 (CC)/CC WarcExport 1.0\r\nisPartOf: CC-MAIN-2014-49\r\noperator: CommonCrawl Admin\r\ndescription: Wide crawl of the web with URLs provided by Blekko for November 2014\r\npublisher: CommonCrawl\r\nformat: WARC File Format 1.0\r\nconformsTo: http://bibnum.bnf.fr/WARC/WARC_ISO_28500_version1_latestdraft.pdf" }
{ "line_start_idx": [ 0, 25, 150, 151, 590, 591, 838, 839, 847, 848, 853, 854, 937, 938, 1340, 1341, 1369, 1370, 1382, 1383, 1412, 1413, 1705, 1706, 1732, 1736, 1849, 1853, 1995, 2000, 2267, 2272, 2340, 2345, 2728, 2729, 2839, 2840, 2853, 2894, 2909, 2935, 2940, 3198, 3199, 3345, 3346, 3381, 3422, 3448, 3449, 3461, 3462, 3464, 3472, 3473, 3551, 3552 ], "line_end_idx": [ 25, 150, 151, 590, 591, 838, 839, 847, 848, 853, 854, 937, 938, 1340, 1341, 1369, 1370, 1382, 1383, 1412, 1413, 1705, 1706, 1732, 1736, 1849, 1853, 1995, 2000, 2267, 2272, 2340, 2345, 2728, 2729, 2839, 2840, 2853, 2894, 2909, 2935, 2940, 3198, 3199, 3345, 3346, 3381, 3422, 3448, 3449, 3461, 3462, 3464, 3472, 3473, 3551, 3552, 3642 ] }
{ "red_pajama_v2": { "ccnet_original_length": 3642, "ccnet_original_nlines": 57, "rps_doc_curly_bracket": 0, "rps_doc_ldnoobw_words": 0, "rps_doc_lorem_ipsum": 0, "rps_doc_stop_word_fraction": 0.34569376707077026, "rps_doc_ut1_blacklist": 0, "rps_doc_frac_all_caps_words": 0.04186603054404259, "rps_doc_frac_lines_end_with_ellipsis": 0, "rps_doc_frac_no_alph_words": 0.24760766327381134, "rps_doc_frac_unique_words": 0.5057096481323242, "rps_doc_mean_word_length": 4.553018093109131, "rps_doc_num_sentences": 40, "rps_doc_symbol_to_word_ratio": 0.0011961699929088354, "rps_doc_unigram_entropy": 5.33980655670166, "rps_doc_word_count": 613, "rps_doc_frac_chars_dupe_10grams": 0, "rps_doc_frac_chars_dupe_5grams": 0.058402009308338165, "rps_doc_frac_chars_dupe_6grams": 0.058402009308338165, "rps_doc_frac_chars_dupe_7grams": 0.04621998965740204, "rps_doc_frac_chars_dupe_8grams": 0.027946969494223595, "rps_doc_frac_chars_dupe_9grams": 0, "rps_doc_frac_chars_top_2gram": 0.010748839937150478, "rps_doc_frac_chars_top_3gram": 0.015048369765281677, "rps_doc_frac_chars_top_4gram": 0.01934790052473545, "rps_doc_books_importance": -418.4952087402344, "rps_doc_books_importance_length_correction": -418.4952087402344, "rps_doc_openwebtext_importance": -267.17474365234375, "rps_doc_openwebtext_importance_length_correction": -267.17474365234375, "rps_doc_wikipedia_importance": -172.916015625, "rps_doc_wikipedia_importance_length_correction": -172.916015625 }, "fasttext": { "dclm": 0.10162199288606644, "english": 0.9259519577026367, "fineweb_edu_approx": 1.2597095966339111, "eai_general_math": 0.007397470064461231, "eai_open_web_math": 0.22881489992141724, "eai_web_code": 0.0002676800068002194 } }
{ "free_decimal_correspondence": { "primary": { "code": "005.442", "labels": { "level_1": "General works, books and libraries, information sciences", "level_2": "", "level_3": "Computer programming" } }, "secondary": { "code": "004.67", "labels": { "level_1": "General works, books and libraries, information sciences", "level_2": "", "level_3": "Computers and Computer science" } } }, "bloom_cognitive_process": { "primary": { "code": "3", "label": "Apply" }, "secondary": { "code": "2", "label": "Understand" } }, "bloom_knowledge_domain": { "primary": { "code": "3", "label": "Procedural" }, "secondary": { "code": "2", "label": "Conceptual" } }, "document_type_v1": { "primary": { "code": "5", "label": "Social/Forum" }, "secondary": { "code": "3", "label": "Reference/Encyclopedic/Educational" } }, "extraction_artifacts": { "primary": { "code": "3", "label": "Irrelevant Content" }, "secondary": { "code": "0", "label": "No Artifacts" } }, "missing_content": { "primary": { "code": "0", "label": "No missing content" }, "secondary": { "code": "2", "label": "Click Here References" } }, "document_type_v2": { "primary": { "code": "18", "label": "Q&A Forum" }, "secondary": { "code": "23", "label": "Tutorial" } }, "reasoning_depth": { "primary": { "code": "3", "label": "Intermediate Reasoning" }, "secondary": { "code": "2", "label": "Basic Reasoning" } }, "technical_correctness": { "primary": { "code": "4", "label": "Highly Correct" }, "secondary": { "code": "3", "label": "Mostly Correct" } }, "education_level": { "primary": { "code": "3", "label": "Undergraduate Level" }, "secondary": { "code": "2", "label": "High School Level" } } }
672f1e42c33a7f9846924a2431ea77df
5,269,375,549,467,735,000
HTTP önbelleği This translation is incomplete. Please help translate this article from English The performance of web sites and applications can be significantly improved by reusing previously fetched resources. Web caches reduce latency and network traffic and thus lessen the time needed to display a representation of a resource. By making use of HTTP caching, Web sites become more responsive. Önbellek çeşitleri Caching is a technique that stores a copy of a given resource and serves it back when requested. When a web cache has a requested resource in its store, it intercepts the request and returns its copy instead of re-downloading from the originating server. This achieves several goals: it eases the load of the server that doesn’t need to serve all clients itself, and it improves performance by being closer to the client, i.e., it takes less time to transmit the resource back. For a web site, it is a major component in achieving high performance. On the other side, it has to be configured properly as not all resources stay identical forever: it is important to cache a resource only until it changes, not longer. There are several kinds of caches: these can be grouped into two main categories: private or shared caches. A shared cache is a cache that stores responses for reuse by more than one user. A private cache is dedicated to a single user. This page will mostly talk about browser and proxy caches, but there are also gateway caches, CDN, reverse proxy caches and load balancers that are deployed on web servers for better reliability, performance and scaling of web sites and web applications. What a cache provide, advantages/disadvantages of shared/private caches. Özel tarayıcı önbellekleri A private cache is dedicated to a single user. You might have seen "caching" in your browser's settings already. A browser cache holds all documents downloaded via HTTP by the user. This cache is used to make visited documents available for back/forward navigation, saving, viewing-as-source, etc. without requiring an additional trip to the server. It likewise improves offline browsing of cached content. Paylaşılan vekil sunucu önbellekleri A shared cache is a cache that stores responses to be reused by more than one user. For example, an ISP or your company might have set up a web proxy as part of its local network infrastructure to serve many users so that popular resources are reused a number of times, reducing network traffic and latency. Önbellekleme işlemlerinin hedefleri HTTP caching is optional, but reusing a cached resource is usually desirable. However, common HTTP caches are typically limited to caching responses to GET and may decline other methods. The primary cache key consists of the request method and target URI (oftentimes only the URI is used as only GET requests are caching targets). Common forms of caching entries are: • Successful results of a retrieval request: a 200 (OK) response to a GET request containing a resource like HTML documents, images or files. • Permanent redirects: a 301 (Moved Permanently) response. • Error responses: a 404 (Not Found) result page. • Incomplete results: a 206 (Partial Content) response. • Responses other than GET if something suitable for use as a cache key is defined. A cache entry might also consist of multiple stored responses differentiated by a secondary key, if the request is target of content negotiation. For more details see the information about the Vary header below. Önbellek kontrolü Cache-control başlığı The Cache-Control HTTP/1.1 general-header field is used to specify directives for caching mechanisms in both requests and responses. Use this header to define your caching policies with the variety of directives it provides. Önbellek yok The cache should not store anything about the client request or server response. A request is sent to the server and a full response is downloaded each and every time. Cache-Control: no-store Önbellekleme yap ama yeniden onaylat A cache will send the request to the origin server for validation before releasing a cached copy. Cache-Control: no-cache Özel ve genel önbellekler The "public" directive indicates that the response may be cached by any cache. This can be useful, if pages with HTTP authentication or response status codes that aren't normally cacheable, should now be cached. On the other hand, "private" indicates that the response is intended for a single user only and must not be stored by a shared cache. A private browser cache may store the response in this case. Cache-Control: private Cache-Control: public Tükenme The most important directive here is "max-age=<seconds>" which is the maximum amount of time a resource will be considered fresh. Contrary to Expires, this directive is relative to the time of the request. For the files in the application that will not change, you can usually add aggressive caching. This includes static files such as images, CSS files and JavaScript files, for example. For more details, see also the Freshness section below. Cache-Control: max-age=31536000 Onaylama When using the "must-revalidate" directive, the cache must verify the status of the stale resources before using it and expired ones should not be used. For more details, see the Validation section below. Cache-Control: must-revalidate Pragma başlığı Pragma is a HTTP/1.0 header, it is not specified for HTTP responses and is therefore not a reliable replacement for the general HTTP/1.1 Cache-Control header, although it does behave the same as Cache-Control: no-cache, if the Cache-Control header field is omitted in a request. Use Pragma only for backwards compatibility with HTTP/1.0 clients. Tazelik Once a resource is stored in a cache, it could theoretically be served by the cache forever. Caches have finite storage so items are periodically removed from storage. This process is called cache eviction. On the other side, some resources may change on the server so the cache should be updated. As HTTP is a client-server protocol, servers can't contact caches and clients when a resource changes; they have to communicate an expiration time for the resource. Before this expiration time, the resource is fresh; after the expiration time, the resource is stale. Eviction algorithms often privilege fresh resources over stale resources. Note that a stale resource is not evicted or ignored; when the cache receives a request for a stale resource, it forwards this request with a If-None-Match to check if it is in fact still fresh. If so, the server returns a 304 (Not Modified) header without sending the body of the requested resource, saving some bandwidth. Here is an example of this process with a shared cache proxy: Show how a proxy cache acts when a doc is not cache, in the cache and fresh, in the cache and stale. The freshness lifetime is calculated based on several headers. If a "Cache-control: max-age=N" header is specified, then the freshness lifetime is equal to N. If this header is not present, which is very often the case, it is checked if an Expires header is present. If an Expires header exists, then its value minus the value of the Date header determines the freshness lifetime. Finally, if neither header is present, look for a Last-Modified header. If this header is present, then the cache's freshness lifetime is equal to the value of the Date header minus the value of the Last-modified header divided by 10. The expiration time is computed as follows: expirationTime = responseTime + freshnessLifetime - currentAge where responseTime is the time at which the response was received according to the browser. Yenilenmiş kaynaklar The more we use cached resources, the better the responsiveness and the performance of a Web site will be. To optimize this, good practices recommend to set expiration times as far in the future as possible. This is possible on resources that are regularly updated, or often, but is problematic for resources that are rarely and infrequently updated. They are the resources that would benefit the most from caching resources, yet this makes them very difficult to update. This is typical of the technical resources included and linked from each Web pages: JavaScript and CSS files change infrequently, but when they change you want them to be updated quickly. Web developers invented a technique that Steve Souders called revving[1]. Infrequently updated files are named in specific way: in their URL, usually in the filename, a revision (or version) number is added. That way each new revision of this resource is considered as a resource on its own that never changes and that can have an expiration time very far in the future, usually one year or even more. In order to have the new versions, all the links to them must be changed, that is the drawback of this method: additional complexity that is usually taken care of by the tool chain used by Web developers. When the infrequently variable resources change they induce an additional change to often variable resources. When these are read, the new versions of the others are also read. This technique has an additional benefit: updating two cached resources at the same time will not lead to the situation where the out-dated version of one resource is used in combination with the new version of the other one. This is very important when web sites have CSS stylesheets or JS scripts that have mutual dependencies, i.e., they depend on each other because they refer to the same HTML elements. The revision version added to revved resources doesn't need to be a classical revision string like 1.1.3, or even a monotonously growing suite of number. It can be anything that prevent collisions, like a hash or a date. Önbellek doğrulama When a cached document's expiration time has been reached, it is either validated or fetched again. Validation can only occur if the server provided either a strong validator or a weak validator. Revalidation is triggered when the user presses the reload button. It is also triggered under normal browsing if the cached response includes the "Cache-control: must-revalidate" header. Another factor is the cache validation preferences in the Advanced->Cache preferences panel. There is an option to force a validation each time a document is loaded. ETag'lar The ETag response header is an opaque-to-the-useragent value that can be used as a strong validator. That means that a HTTP user-agent, such as the browser, does not know what this string represents and can't predict what its value would be. If the ETag header was part of the response for a resource, the client can issue an If-None-Match in the header of future requests – in order to validate the cached resource. The Last-Modified response header can be used as a weak validator. It is considered weak because it only has 1-second resolution. If the Last-Modified header is present in a response, then the client can issue an If-Modified-Since request header to validate the cached document. When a validation request is made, the server can either ignore the validation request and response with a normal 200 OK, or it can return 304 Not Modified (with an empty body) to instruct the browser to use its cached copy. The latter response can also include headers that update the expiration time of the cached document. Değişen yanıtlar The Vary HTTP response header determines how to match future request headers to decide whether a cached response can be used rather than requesting a fresh one from the origin server. When a cache receives a request that can be satisfied by a cached response that has a Vary header field, it must not use that cached response unless all header fields as nominated by the Vary header match in both the original (cached) request and the new request. The Vary header leads cache to use more HTTP headers as key for the cache. This can be useful for serving content dynamically, for example. When using the Vary: User-Agent header, caching servers should consider the user agent when deciding whether to serve the page from cache. If you are serving different content to mobile users, it can help you to avoid that a cache may mistakenly serve a desktop version of your site to your mobile users. In addition, it can help Google and other search engines to discover the mobile version of a page, and might also tell them that no Cloaking is intended. Vary: User-Agent Because the User-Agent header value is different ("varies") for mobile and desktop clients, caches will not be used to serve mobile content mistakenly to desktop users or vice versa. Bakınız
{ "url": "https://developer.mozilla.org/tr/docs/Web/HTTP/Onbellek", "source_domain": "developer.mozilla.org", "snapshot_id": "crawl=CC-MAIN-2020-10", "warc_metadata": { "Content-Length": "378423", "Content-Type": "application/http; msgtype=response", "WARC-Block-Digest": "sha1:VT7GDDM7SG6YNUENGY3U7GI4EXERJZVM", "WARC-Concurrent-To": "<urn:uuid:63314649-46b1-4d34-be68-3b6b817ab25d>", "WARC-Date": "2020-02-29T03:05:41Z", "WARC-IP-Address": "52.85.148.95", "WARC-Identified-Payload-Type": "text/html", "WARC-Payload-Digest": "sha1:TUPVFPWBBRIYKQIPMSEYW7SKNAFTE6QM", "WARC-Record-ID": "<urn:uuid:e9535c66-d104-48d6-86fe-1bbcca42020a>", "WARC-Target-URI": "https://developer.mozilla.org/tr/docs/Web/HTTP/Onbellek", "WARC-Truncated": null, "WARC-Type": "response", "WARC-Warcinfo-ID": "<urn:uuid:05e6f176-06e7-492e-9406-70cae949a29d>" }, "warc_info": "isPartOf: CC-MAIN-2020-10\r\npublisher: Common Crawl\r\ndescription: Wide crawl of the web for February 2020\r\noperator: Common Crawl Admin ([email protected])\r\nhostname: ip-10-67-67-123.ec2.internal\r\nsoftware: Apache Nutch 1.16 (modified, https://github.com/commoncrawl/nutch/)\r\nrobots: checked via crawler-commons 1.1-SNAPSHOT (https://github.com/crawler-commons/crawler-commons)\r\nformat: WARC File Format 1.1\r\nconformsTo: http://iipc.github.io/warc-specifications/specifications/warc-format/warc-1.1/" }
{ "line_start_idx": [ 0, 15, 16, 96, 97, 400, 401, 420, 421, 1138, 1139, 1630, 1631, 1704, 1705, 1732, 1733, 2140, 2141, 2178, 2179, 2487, 2488, 2524, 2525, 2893, 2894, 3038, 3099, 3151, 3209, 3295, 3296, 3508, 3509, 3527, 3528, 3550, 3551, 3776, 3777, 3790, 3791, 3959, 3960, 3984, 3985, 4022, 4023, 4121, 4122, 4146, 4147, 4173, 4174, 4386, 4387, 4582, 4583, 4606, 4628, 4629, 4637, 4638, 5027, 5028, 5084, 5085, 5117, 5118, 5127, 5128, 5333, 5334, 5365, 5366, 5381, 5382, 5728, 5729, 5737, 5738, 6701, 6702, 6764, 6765, 6866, 6867, 7483, 7527, 7528, 7591, 7592, 7684, 7685, 7706, 7707, 8367, 8368, 9152, 9153, 9561, 9562, 9783, 9784, 9803, 9804, 10000, 10001, 10354, 10355, 10364, 10365, 10782, 10783, 11062, 11063, 11389, 11390, 11407, 11408, 11592, 11593, 11857, 11858, 11933, 11934, 12458, 12459, 12476, 12477, 12660, 12661 ], "line_end_idx": [ 15, 16, 96, 97, 400, 401, 420, 421, 1138, 1139, 1630, 1631, 1704, 1705, 1732, 1733, 2140, 2141, 2178, 2179, 2487, 2488, 2524, 2525, 2893, 2894, 3038, 3099, 3151, 3209, 3295, 3296, 3508, 3509, 3527, 3528, 3550, 3551, 3776, 3777, 3790, 3791, 3959, 3960, 3984, 3985, 4022, 4023, 4121, 4122, 4146, 4147, 4173, 4174, 4386, 4387, 4582, 4583, 4606, 4628, 4629, 4637, 4638, 5027, 5028, 5084, 5085, 5117, 5118, 5127, 5128, 5333, 5334, 5365, 5366, 5381, 5382, 5728, 5729, 5737, 5738, 6701, 6702, 6764, 6765, 6866, 6867, 7483, 7527, 7528, 7591, 7592, 7684, 7685, 7706, 7707, 8367, 8368, 9152, 9153, 9561, 9562, 9783, 9784, 9803, 9804, 10000, 10001, 10354, 10355, 10364, 10365, 10782, 10783, 11062, 11063, 11389, 11390, 11407, 11408, 11592, 11593, 11857, 11858, 11933, 11934, 12458, 12459, 12476, 12477, 12660, 12661, 12668 ] }
{ "red_pajama_v2": { "ccnet_original_length": 12668, "ccnet_original_nlines": 132, "rps_doc_curly_bracket": 0, "rps_doc_ldnoobw_words": 0, "rps_doc_lorem_ipsum": 0, "rps_doc_stop_word_fraction": 0.40997567772865295, "rps_doc_ut1_blacklist": 0, "rps_doc_frac_all_caps_words": 0.017437150701880455, "rps_doc_frac_lines_end_with_ellipsis": 0, "rps_doc_frac_no_alph_words": 0.14355230331420898, "rps_doc_frac_unique_words": 0.29830509424209595, "rps_doc_mean_word_length": 4.949636936187744, "rps_doc_num_sentences": 115, "rps_doc_symbol_to_word_ratio": 0, "rps_doc_unigram_entropy": 5.484044551849365, "rps_doc_word_count": 2065, "rps_doc_frac_chars_dupe_10grams": 0, "rps_doc_frac_chars_dupe_5grams": 0.04921241104602814, "rps_doc_frac_chars_dupe_6grams": 0.01937188021838665, "rps_doc_frac_chars_dupe_7grams": 0.014871340245008469, "rps_doc_frac_chars_dupe_8grams": 0.014871340245008469, "rps_doc_frac_chars_dupe_9grams": 0.014871340245008469, "rps_doc_frac_chars_top_2gram": 0.006359459832310677, "rps_doc_frac_chars_top_3gram": 0.004696209914982319, "rps_doc_frac_chars_top_4gram": 0.004989730194211006, "rps_doc_books_importance": -1235.9462890625, "rps_doc_books_importance_length_correction": -1235.9462890625, "rps_doc_openwebtext_importance": -643.5481567382812, "rps_doc_openwebtext_importance_length_correction": -643.5481567382812, "rps_doc_wikipedia_importance": -451.1155700683594, "rps_doc_wikipedia_importance_length_correction": -451.1155700683594 }, "fasttext": { "dclm": 0.1628764271736145, "english": 0.9119728803634644, "fineweb_edu_approx": 2.7530341148376465, "eai_general_math": 0.689841091632843, "eai_open_web_math": 0.308940052986145, "eai_web_code": 0.9536883234977722 } }
{ "free_decimal_correspondence": { "primary": { "code": "004.678", "labels": { "level_1": "General works, books and libraries, information sciences", "level_2": "", "level_3": "Computers and Computer science" } }, "secondary": { "code": "005.72", "labels": { "level_1": "General works, books and libraries, information sciences", "level_2": "", "level_3": "Computer programming" } } }, "bloom_cognitive_process": { "primary": { "code": "2", "label": "Understand" }, "secondary": { "code": "3", "label": "Apply" } }, "bloom_knowledge_domain": { "primary": { "code": "2", "label": "Conceptual" }, "secondary": { "code": "3", "label": "Procedural" } }, "document_type_v1": { "primary": { "code": "3", "label": "Reference/Encyclopedic/Educational" }, "secondary": { "code": "-1", "label": "Abstain" } }, "extraction_artifacts": { "primary": { "code": "0", "label": "No Artifacts" }, "secondary": { "code": "3", "label": "Irrelevant Content" } }, "missing_content": { "primary": { "code": "0", "label": "No missing content" }, "secondary": { "code": "-1", "label": "Abstain" } }, "document_type_v2": { "primary": { "code": "8", "label": "Documentation" }, "secondary": { "code": "10", "label": "Knowledge Article" } }, "reasoning_depth": { "primary": { "code": "3", "label": "Intermediate Reasoning" }, "secondary": { "code": "2", "label": "Basic Reasoning" } }, "technical_correctness": { "primary": { "code": "4", "label": "Highly Correct" }, "secondary": { "code": "3", "label": "Mostly Correct" } }, "education_level": { "primary": { "code": "3", "label": "Undergraduate Level" }, "secondary": { "code": "2", "label": "High School Level" } } }
672f1e42c33a7f9846924a2431ea77df
5,308,198,906,787,063,000
Exception Handling in LLVM Introduction This document is the central repository for all information pertaining to exception handling in LLVM. It describes the format that LLVM exception handling information takes, which is useful for those interested in creating front-ends or dealing directly with the information. Further, this document provides specific examples of what exception handling information is used for in C and C++. Itanium ABI Zero-cost Exception Handling Exception handling for most programming languages is designed to recover from conditions that rarely occur during general use of an application. To that end, exception handling should not interfere with the main flow of an application’s algorithm by performing checkpointing tasks, such as saving the current pc or register state. The Itanium ABI Exception Handling Specification defines a methodology for providing outlying data in the form of exception tables without inlining speculative exception handling code in the flow of an application’s main algorithm. Thus, the specification is said to add “zero-cost” to the normal execution of an application. A more complete description of the Itanium ABI exception handling runtime support of can be found at Itanium C++ ABI: Exception Handling. A description of the exception frame format can be found at Exception Frames, with details of the DWARF 4 specification at DWARF 4 Standard. A description for the C++ exception table formats can be found at Exception Handling Tables. Setjmp/Longjmp Exception Handling Setjmp/Longjmp (SJLJ) based exception handling uses LLVM intrinsics llvm.eh.sjlj.setjmp and llvm.eh.sjlj.longjmp to handle control flow for exception handling. For each function which does exception processing — be it try/catch blocks or cleanups — that function registers itself on a global frame list. When exceptions are unwinding, the runtime uses this list to identify which functions need processing. Landing pad selection is encoded in the call site entry of the function context. The runtime returns to the function via llvm.eh.sjlj.longjmp, where a switch table transfers control to the appropriate landing pad based on the index stored in the function context. In contrast to DWARF exception handling, which encodes exception regions and frame information in out-of-line tables, SJLJ exception handling builds and removes the unwind frame context at runtime. This results in faster exception handling at the expense of slower execution when no exceptions are thrown. As exceptions are, by their nature, intended for uncommon code paths, DWARF exception handling is generally preferred to SJLJ. Windows Runtime Exception Handling LLVM supports handling exceptions produced by the Windows runtime, but it requires a very different intermediate representation. It is not based on the “landingpad” instruction like the other two models, and is described later in this document under Exception Handling using the Windows Runtime. Overview When an exception is thrown in LLVM code, the runtime does its best to find a handler suited to processing the circumstance. The runtime first attempts to find an exception frame corresponding to the function where the exception was thrown. If the programming language supports exception handling (e.g. C++), the exception frame contains a reference to an exception table describing how to process the exception. If the language does not support exception handling (e.g. C), or if the exception needs to be forwarded to a prior activation, the exception frame contains information about how to unwind the current activation and restore the state of the prior activation. This process is repeated until the exception is handled. If the exception is not handled and no activations remain, then the application is terminated with an appropriate error message. Because different programming languages have different behaviors when handling exceptions, the exception handling ABI provides a mechanism for supplying personalities. An exception handling personality is defined by way of a personality function (e.g. __gxx_personality_v0 in C++), which receives the context of the exception, an exception structure containing the exception object type and value, and a reference to the exception table for the current function. The personality function for the current compile unit is specified in a common exception frame. The organization of an exception table is language dependent. For C++, an exception table is organized as a series of code ranges defining what to do if an exception occurs in that range. Typically, the information associated with a range defines which types of exception objects (using C++ type info) that are handled in that range, and an associated action that should take place. Actions typically pass control to a landing pad. A landing pad corresponds roughly to the code found in the catch portion of a try/catch sequence. When execution resumes at a landing pad, it receives an exception structure and a selector value corresponding to the type of exception thrown. The selector is then used to determine which catch should actually process the exception. LLVM Code Generation From a C++ developer’s perspective, exceptions are defined in terms of the throw and try/catch statements. In this section we will describe the implementation of LLVM exception handling in terms of C++ examples. Throw Languages that support exception handling typically provide a throw operation to initiate the exception process. Internally, a throw operation breaks down into two steps. 1. A request is made to allocate exception space for an exception structure. This structure needs to survive beyond the current activation. This structure will contain the type and value of the object being thrown. 2. A call is made to the runtime to raise the exception, passing the exception structure as an argument. In C++, the allocation of the exception structure is done by the __cxa_allocate_exception runtime function. The exception raising is handled by __cxa_throw. The type of the exception is represented using a C++ RTTI structure. Try/Catch A call within the scope of a try statement can potentially raise an exception. In those circumstances, the LLVM C++ front-end replaces the call with an invoke instruction. Unlike a call, the invoke has two potential continuation points: 1. where to continue when the call succeeds as per normal, and 2. where to continue if the call raises an exception, either by a throw or the unwinding of a throw The term used to define the place where an invoke continues after an exception is called a landing pad. LLVM landing pads are conceptually alternative function entry points where an exception structure reference and a type info index are passed in as arguments. The landing pad saves the exception structure reference and then proceeds to select the catch block that corresponds to the type info of the exception object. The LLVM ‘landingpad’ Instruction is used to convey information about the landing pad to the back end. For C++, the landingpad instruction returns a pointer and integer pair corresponding to the pointer to the exception structure and the selector value respectively. The landingpad instruction looks for a reference to the personality function to be used for this try/catch sequence in the parent function’s attribute list. The instruction contains a list of cleanup, catch, and filter clauses. The exception is tested against the clauses sequentially from first to last. The clauses have the following meanings: • catch <type> @ExcType • This clause means that the landingpad block should be entered if the exception being thrown is of type @ExcType or a subtype of @ExcType. For C++, @ExcType is a pointer to the std::type_info object (an RTTI object) representing the C++ exception type. • If @ExcType is null, any exception matches, so the landingpad should always be entered. This is used for C++ catch-all blocks (“catch (...)”). • When this clause is matched, the selector value will be equal to the value returned by “@llvm.eh.typeid.for(i8* @ExcType)”. This will always be a positive value. • filter <type> [<type> @ExcType1, ..., <type> @ExcTypeN] • This clause means that the landingpad should be entered if the exception being thrown does not match any of the types in the list (which, for C++, are again specified as std::type_info pointers). • C++ front-ends use this to implement C++ exception specifications, such as “void foo() throw (ExcType1, ..., ExcTypeN) { ... }”. • When this clause is matched, the selector value will be negative. • The array argument to filter may be empty; for example, “[0 x i8**] undef”. This means that the landingpad should always be entered. (Note that such a filter would not be equivalent to “catch i8* null”, because filter and catch produce negative and positive selector values respectively.) • cleanup • This clause means that the landingpad should always be entered. • C++ front-ends use this for calling objects’ destructors. • When this clause is matched, the selector value will be zero. • The runtime may treat “cleanup” differently from “catch <type> null”. In C++, if an unhandled exception occurs, the language runtime will call std::terminate(), but it is implementation-defined whether the runtime unwinds the stack and calls object destructors first. For example, the GNU C++ unwinder does not call object destructors when an unhandled exception occurs. The reason for this is to improve debuggability: it ensures that std::terminate() is called from the context of the throw, so that this context is not lost by unwinding the stack. A runtime will typically implement this by searching for a matching non-cleanup clause, and aborting if it does not find one, before entering any landingpad blocks. Once the landing pad has the type info selector, the code branches to the code for the first catch. The catch then checks the value of the type info selector against the index of type info for that catch. Since the type info index is not known until all the type infos have been gathered in the backend, the catch code must call the llvm.eh.typeid.for intrinsic to determine the index for a given type info. If the catch fails to match the selector then control is passed on to the next catch. Finally, the entry and exit of catch code is bracketed with calls to __cxa_begin_catch and __cxa_end_catch. • __cxa_begin_catch takes an exception structure reference as an argument and returns the value of the exception object. • __cxa_end_catch takes no arguments. This function: 1. Locates the most recently caught exception and decrements its handler count, 2. Removes the exception from the caught stack if the handler count goes to zero, and 3. Destroys the exception if the handler count goes to zero and the exception was not re-thrown by throw. Note a rethrow from within the catch may replace this call with a __cxa_rethrow. Cleanups A cleanup is extra code which needs to be run as part of unwinding a scope. C++ destructors are a typical example, but other languages and language extensions provide a variety of different kinds of cleanups. In general, a landing pad may need to run arbitrary amounts of cleanup code before actually entering a catch block. To indicate the presence of cleanups, a ‘landingpad’ Instruction should have a cleanup clause. Otherwise, the unwinder will not stop at the landing pad if there are no catches or filters that require it to. Note Do not allow a new exception to propagate out of the execution of a cleanup. This can corrupt the internal state of the unwinder. Different languages describe different high-level semantics for these situations: for example, C++ requires that the process be terminated, whereas Ada cancels both exceptions and throws a third. When all cleanups are finished, if the exception is not handled by the current function, resume unwinding by calling the resume instruction, passing in the result of the landingpad instruction for the original landing pad. Throw Filters C++ allows the specification of which exception types may be thrown from a function. To represent this, a top level landing pad may exist to filter out invalid types. To express this in LLVM code the ‘landingpad’ Instruction will have a filter clause. The clause consists of an array of type infos. landingpad will return a negative value if the exception does not match any of the type infos. If no match is found then a call to __cxa_call_unexpected should be made, otherwise _Unwind_Resume. Each of these functions requires a reference to the exception structure. Note that the most general form of a landingpad instruction can have any number of catch, cleanup, and filter clauses (though having more than one cleanup is pointless). The LLVM C++ front-end can generate such landingpad instructions due to inlining creating nested exception handling scopes. Restrictions The unwinder delegates the decision of whether to stop in a call frame to that call frame’s language-specific personality function. Not all unwinders guarantee that they will stop to perform cleanups. For example, the GNU C++ unwinder doesn’t do so unless the exception is actually caught somewhere further up the stack. In order for inlining to behave correctly, landing pads must be prepared to handle selector results that they did not originally advertise. Suppose that a function catches exceptions of type A, and it’s inlined into a function that catches exceptions of type B. The inliner will update the landingpad instruction for the inlined landing pad to include the fact that B is also caught. If that landing pad assumes that it will only be entered to catch an A, it’s in for a rude awakening. Consequently, landing pads must test for the selector results they understand and then resume exception propagation with the resume instruction if none of the conditions match. Exception Handling Intrinsics In addition to the landingpad and resume instructions, LLVM uses several intrinsic functions (name prefixed with llvm.eh) to provide exception handling information at various points in generated code. llvm.eh.typeid.for i32 @llvm.eh.typeid.for(i8* %type_info) This intrinsic returns the type info index in the exception table of the current function. This value can be used to compare against the result of landingpad instruction. The single argument is a reference to a type info. Uses of this intrinsic are generated by the C++ front-end. llvm.eh.begincatch void @llvm.eh.begincatch(i8* %ehptr, i8* %ehobj) This intrinsic marks the beginning of catch handling code within the blocks following a landingpad instruction. The exact behavior of this function depends on the compilation target and the personality function associated with the landingpad instruction. The first argument to this intrinsic is a pointer that was previously extracted from the aggregate return value of the landingpad instruction. The second argument to the intrinsic is a pointer to stack space where the exception object should be stored. The runtime handles the details of copying the exception object into the slot. If the second parameter is null, no copy occurs. Uses of this intrinsic are generated by the C++ front-end. Many targets will use implementation-specific functions (such as __cxa_begin_catch) instead of this intrinsic. The intrinsic is provided for targets that require a more abstract interface. When used in the native Windows C++ exception handling implementation, this intrinsic serves as a placeholder to delimit code before a catch handler is outlined. When the handler is outlined, this intrinsic will be replaced by instructions that retrieve the exception object pointer from the frame allocation block. llvm.eh.endcatch void @llvm.eh.endcatch() This intrinsic marks the end of catch handling code within the current block, which will be a successor of a block which called llvm.eh.begincatch''. The exact behavior of this function depends on the compilation target and the personality function associated with the corresponding ``landingpad instruction. There may be more than one call to llvm.eh.endcatch for any given call to llvm.eh.begincatch with each llvm.eh.endcatch call corresponding to the end of a different control path. All control paths following a call to llvm.eh.begincatch must reach a call to llvm.eh.endcatch. Uses of this intrinsic are generated by the C++ front-end. Many targets will use implementation-specific functions (such as __cxa_begin_catch) instead of this intrinsic. The intrinsic is provided for targets that require a more abstract interface. When used in the native Windows C++ exception handling implementation, this intrinsic serves as a placeholder to delimit code before a catch handler is outlined. After the handler is outlined, this intrinsic is simply removed. llvm.eh.exceptionpointer i8 addrspace(N)* @llvm.eh.padparam.pNi8(token %catchpad) This intrinsic retrieves a pointer to the exception caught by the given catchpad. SJLJ Intrinsics The llvm.eh.sjlj intrinsics are used internally within LLVM’s backend. Uses of them are generated by the backend’s SjLjEHPrepare pass. llvm.eh.sjlj.setjmp i32 @llvm.eh.sjlj.setjmp(i8* %setjmp_buf) For SJLJ based exception handling, this intrinsic forces register saving for the current function and stores the address of the following instruction for use as a destination address by llvm.eh.sjlj.longjmp. The buffer format and the overall functioning of this intrinsic is compatible with the GCC __builtin_setjmp implementation allowing code built with the clang and GCC to interoperate. The single parameter is a pointer to a five word buffer in which the calling context is saved. The front end places the frame pointer in the first word, and the target implementation of this intrinsic should place the destination address for a llvm.eh.sjlj.longjmp in the second word. The following three words are available for use in a target-specific manner. llvm.eh.sjlj.longjmp void @llvm.eh.sjlj.longjmp(i8* %setjmp_buf) For SJLJ based exception handling, the llvm.eh.sjlj.longjmp intrinsic is used to implement __builtin_longjmp(). The single parameter is a pointer to a buffer populated by llvm.eh.sjlj.setjmp. The frame pointer and stack pointer are restored from the buffer, then control is transferred to the destination address. llvm.eh.sjlj.lsda i8* @llvm.eh.sjlj.lsda() For SJLJ based exception handling, the llvm.eh.sjlj.lsda intrinsic returns the address of the Language Specific Data Area (LSDA) for the current function. The SJLJ front-end code stores this address in the exception handling function context for use by the runtime. llvm.eh.sjlj.callsite void @llvm.eh.sjlj.callsite(i32 %call_site_num) For SJLJ based exception handling, the llvm.eh.sjlj.callsite intrinsic identifies the callsite value associated with the following invoke instruction. This is used to ensure that landing pad entries in the LSDA are generated in matching order. Asm Table Formats There are two tables that are used by the exception handling runtime to determine which actions should be taken when an exception is thrown. Exception Handling Frame An exception handling frame eh_frame is very similar to the unwind frame used by DWARF debug info. The frame contains all the information necessary to tear down the current frame and restore the state of the prior frame. There is an exception handling frame for each function in a compile unit, plus a common exception handling frame that defines information common to all functions in the unit. The format of this call frame information (CFI) is often platform-dependent, however. ARM, for example, defines their own format. Apple has their own compact unwind info format. On Windows, another format is used for all architectures since 32-bit x86. LLVM will emit whatever information is required by the target. Exception Tables An exception table contains information about what actions to take when an exception is thrown in a particular part of a function’s code. This is typically referred to as the language-specific data area (LSDA). The format of the LSDA table is specific to the personality function, but the majority of personalities out there use a variation of the tables consumed by __gxx_personality_v0. There is one exception table per function, except leaf functions and functions that have calls only to non-throwing functions. They do not need an exception table. Exception Handling using the Windows Runtime Background on Windows exceptions Interacting with exceptions on Windows is significantly more complicated than on Itanium C++ ABI platforms. The fundamental difference between the two models is that Itanium EH is designed around the idea of “successive unwinding,” while Windows EH is not. Under Itanium, throwing an exception typically involves allocating thread local memory to hold the exception, and calling into the EH runtime. The runtime identifies frames with appropriate exception handling actions, and successively resets the register context of the current thread to the most recently active frame with actions to run. In LLVM, execution resumes at a landingpad instruction, which produces register values provided by the runtime. If a function is only cleaning up allocated resources, the function is responsible for calling _Unwind_Resume to transition to the next most recently active frame after it is finished cleaning up. Eventually, the frame responsible for handling the exception calls __cxa_end_catch to destroy the exception, release its memory, and resume normal control flow. The Windows EH model does not use these successive register context resets. Instead, the active exception is typically described by a frame on the stack. In the case of C++ exceptions, the exception object is allocated in stack memory and its address is passed to __CxxThrowException. General purpose structured exceptions (SEH) are more analogous to Linux signals, and they are dispatched by userspace DLLs provided with Windows. Each frame on the stack has an assigned EH personality routine, which decides what actions to take to handle the exception. There are a few major personalities for C and C++ code: the C++ personality (__CxxFrameHandler3) and the SEH personalities (_except_handler3, _except_handler4, and __C_specific_handler). All of them implement cleanups by calling back into a “funclet” contained in the parent function. Funclets, in this context, are regions of the parent function that can be called as though they were a function pointer with a very special calling convention. The frame pointer of the parent frame is passed into the funclet either using the standard EBP register or as the first parameter register, depending on the architecture. The funclet implements the EH action by accessing local variables in memory through the frame pointer, and returning some appropriate value, continuing the EH process. No variables live in to or out of the funclet can be allocated in registers. The C++ personality also uses funclets to contain the code for catch blocks (i.e. all user code between the braces in catch (Type obj) { ... }). The runtime must use funclets for catch bodies because the C++ exception object is allocated in a child stack frame of the function handling the exception. If the runtime rewound the stack back to frame of the catch, the memory holding the exception would be overwritten quickly by subsequent function calls. The use of funclets also allows __CxxFrameHandler3 to implement rethrow without resorting to TLS. Instead, the runtime throws a special exception, and then uses SEH (__try / __except) to resume execution with new information in the child frame. In other words, the successive unwinding approach is incompatible with Visual C++ exceptions and general purpose Windows exception handling. Because the C++ exception object lives in stack memory, LLVM cannot provide a custom personality function that uses landingpads. Similarly, SEH does not provide any mechanism to rethrow an exception or continue unwinding. Therefore, LLVM must use the IR constructs described later in this document to implement compatible exception handling. SEH filter expressions The SEH personality functions also use funclets to implement filter expressions, which allow executing arbitrary user code to decide which exceptions to catch. Filter expressions should not be confused with the filter clause of the LLVM landingpad instruction. Typically filter expressions are used to determine if the exception came from a particular DLL or code region, or if code faulted while accessing a particular memory address range. LLVM does not currently have IR to represent filter expressions because it is difficult to represent their control dependencies. Filter expressions run during the first phase of EH, before cleanups run, making it very difficult to build a faithful control flow graph. For now, the new EH instructions cannot represent SEH filter expressions, and frontends must outline them ahead of time. Local variables of the parent function can be escaped and accessed using the llvm.localescape and llvm.localrecover intrinsics. New exception handling instructions The primary design goal of the new EH instructions is to support funclet generation while preserving information about the CFG so that SSA formation still works. As a secondary goal, they are designed to be generic across MSVC and Itanium C++ exceptions. They make very few assumptions about the data required by the personality, so long as it uses the familiar core EH actions: catch, cleanup, and terminate. However, the new instructions are hard to modify without knowing details of the EH personality. While they can be used to represent Itanium EH, the landingpad model is strictly better for optimization purposes. The following new instructions are considered “exception handling pads”, in that they must be the first non-phi instruction of a basic block that may be the unwind destination of an EH flow edge: catchswitch, catchpad, and cleanuppad. As with landingpads, when entering a try scope, if the frontend encounters a call site that may throw an exception, it should emit an invoke that unwinds to a catchswitch block. Similarly, inside the scope of a C++ object with a destructor, invokes should unwind to a cleanuppad. New instructions are also used to mark the points where control is transferred out of a catch/cleanup handler (which will correspond to exits from the generated funclet). A catch handler which reaches its end by normal execution executes a catchret instruction, which is a terminator indicating where in the function control is returned to. A cleanup handler which reaches its end by normal execution executes a cleanupret instruction, which is a terminator indicating where the active exception will unwind to next. Each of these new EH pad instructions has a way to identify which action should be considered after this action. The catchswitch instruction is a terminator and has an unwind destination operand analogous to the unwind destination of an invoke. The cleanuppad instruction is not a terminator, so the unwind destination is stored on the cleanupret instruction instead. Successfully executing a catch handler should resume normal control flow, so neither catchpad nor catchret instructions can unwind. All of these “unwind edges” may refer to a basic block that contains an EH pad instruction, or they may unwind to the caller. Unwinding to the caller has roughly the same semantics as the resume instruction in the landingpad model. When inlining through an invoke, instructions that unwind to the caller are hooked up to unwind to the unwind destination of the call site. Putting things together, here is a hypothetical lowering of some C++ that uses all of the new IR instructions: struct Cleanup { Cleanup(); ~Cleanup(); int m; }; void may_throw(); int f() noexcept { try { Cleanup obj; may_throw(); } catch (int e) { may_throw(); return e; } return 0; } define i32 @f() nounwind personality i32 (...)* @__CxxFrameHandler3 { entry: %obj = alloca %struct.Cleanup, align 4 %e = alloca i32, align 4 %call = invoke %struct.Cleanup* @"??0Cleanup@@QEAA@XZ"(%struct.Cleanup* nonnull %obj) to label %invoke.cont unwind label %lpad.catch invoke.cont: ; preds = %entry invoke void @"?may_throw@@YAXXZ"() to label %invoke.cont.2 unwind label %lpad.cleanup invoke.cont.2: ; preds = %invoke.cont call void @"??_DCleanup@@QEAA@XZ"(%struct.Cleanup* nonnull %obj) nounwind br label %return return: ; preds = %invoke.cont.3, %invoke.cont.2 %retval.0 = phi i32 [ 0, %invoke.cont.2 ], [ %3, %invoke.cont.3 ] ret i32 %retval.0 lpad.cleanup: ; preds = %invoke.cont.2 %0 = cleanuppad within none [] call void @"??1Cleanup@@QEAA@XZ"(%struct.Cleanup* nonnull %obj) nounwind cleanupret %0 unwind label %lpad.catch lpad.catch: ; preds = %lpad.cleanup, %entry %1 = catchswitch within none [label %catch.body] unwind label %lpad.terminate catch.body: ; preds = %lpad.catch %catch = catchpad within %1 [%rtti.TypeDescriptor2* @"??_R0H@8", i32 0, i32* %e] invoke void @"?may_throw@@YAXXZ"() to label %invoke.cont.3 unwind label %lpad.terminate invoke.cont.3: ; preds = %catch.body %3 = load i32, i32* %e, align 4 catchret from %catch to label %return lpad.terminate: ; preds = %catch.body, %lpad.catch cleanuppad within none [] call void @"?terminate@@YAXXZ" unreachable } Funclet parent tokens In order to produce tables for EH personalities that use funclets, it is necessary to recover the nesting that was present in the source. This funclet parent relationship is encoded in the IR using tokens produced by the new “pad” instructions. The token operand of a “pad” or “ret” instruction indicates which funclet it is in, or “none” if it is not nested within another funclet. The catchpad and cleanuppad instructions establish new funclets, and their tokens are consumed by other “pad” instructions to establish membership. The catchswitch instruction does not create a funclet, but it produces a token that is always consumed by its immediate successor catchpad instructions. This ensures that every catch handler modelled by a catchpad belongs to exactly one catchswitch, which models the dispatch point after a C++ try. Here is an example of what this nesting looks like using some hypothetical C++ code: void f() { try { throw; } catch (...) { try { throw; } catch (...) { } } } define void @f() #0 personality i8* bitcast (i32 (...)* @__CxxFrameHandler3 to i8*) { entry: invoke void @_CxxThrowException(i8* null, %eh.ThrowInfo* null) #1 to label %unreachable unwind label %catch.dispatch catch.dispatch: ; preds = %entry %0 = catchswitch within none [label %catch] unwind to caller catch: ; preds = %catch.dispatch %1 = catchpad within %0 [i8* null, i32 64, i8* null] invoke void @_CxxThrowException(i8* null, %eh.ThrowInfo* null) #1 to label %unreachable unwind label %catch.dispatch2 catch.dispatch2: ; preds = %catch %2 = catchswitch within %1 [label %catch3] unwind to caller catch3: ; preds = %catch.dispatch2 %3 = catchpad within %2 [i8* null, i32 64, i8* null] catchret from %3 to label %try.cont try.cont: ; preds = %catch3 catchret from %1 to label %try.cont6 try.cont6: ; preds = %try.cont ret void unreachable: ; preds = %catch, %entry unreachable } The “inner” catchswitch consumes %1 which is produced by the outer catchswitch. Funclet transitions The EH tables for personalities that use funclets make implicit use of the funclet nesting relationship to encode unwind destinations, and so are constrained in the set of funclet transitions they can represent. The related LLVM IR instructions accordingly have constraints that ensure encodability of the EH edges in the flow graph. A catchswitch, catchpad, or cleanuppad is said to be “entered” when it executes. It may subsequently be “exited” by any of the following means: • A catchswitch is immediately exited when none of its constituent catchpads are appropriate for the in-flight exception and it unwinds to its unwind destination or the caller. • A catchpad and its parent catchswitch are both exited when a catchret from the catchpad is executed. • A cleanuppad is exited when a cleanupret from it is executed. • Any of these pads is exited when control unwinds to the function’s caller, either by a call which unwinds all the way to the function’s caller, a nested catchswitch marked “unwinds to caller”, or a nested cleanuppad’s cleanupret marked “unwinds to caller". • Any of these pads is exited when an unwind edge (from an invoke, nested catchswitch, or nested cleanuppad’s cleanupret) unwinds to a destination pad that is not a descendant of the given pad. Note that the ret instruction is not a valid way to exit a funclet pad; it is undefined behavior to execute a ret when a pad has been entered but not exited. A single unwind edge may exit any number of pads (with the restrictions that the edge from a catchswitch must exit at least itself, and the edge from a cleanupret must exit at least its cleanuppad), and then must enter exactly one pad, which must be distinct from all the exited pads. The parent of the pad that an unwind edge enters must be the most-recently-entered not-yet-exited pad (after exiting from any pads that the unwind edge exits), or “none” if there is no such pad. This ensures that the stack of executing funclets at run-time always corresponds to some path in the funclet pad tree that the parent tokens encode. All unwind edges which exit any given funclet pad (including cleanupret edges exiting their cleanuppad and catchswitch edges exiting their catchswitch) must share the same unwind destination. Similarly, any funclet pad which may be exited by unwind to caller must not be exited by any exception edges which unwind anywhere other than the caller. This ensures that each funclet as a whole has only one unwind destination, which EH tables for funclet personalities may require. Note that any unwind edge which exits a catchpad also exits its parent catchswitch, so this implies that for any given catchswitch, its unwind destination must also be the unwind destination of any unwind edge that exits any of its constituent catchpads. Because catchswitch has no nounwind variant, and because IR producers are not required to annotate calls which will not unwind as nounwind, it is legal to nest a call or an “unwind to callercatchswitch within a funclet pad that has an unwind destination other than caller; it is undefined behavior for such a call or catchswitch to unwind. Finally, the funclet pads’ unwind destinations cannot form a cycle. This ensures that EH lowering can construct “try regions” with a tree-like structure, which funclet-based personalities may require. Exception Handling support on the target In order to support exception handling on particular target, there are a few items need to be implemented. • CFI directives First, you have to assign each target register with a unique DWARF number. Then in TargetFrameLowering’s emitPrologue, you have to emit CFI directives to specify how to calculate the CFA (Canonical Frame Address) and how register is restored from the address pointed by the CFA with an offset. The assembler is instructed by CFI directives to build .eh_frame section, which is used by th unwinder to unwind stack during exception handling. • getExceptionPointerRegister and getExceptionSelectorRegister TargetLowering must implement both functions. The personality function passes the exception structure (a pointer) and selector value (an integer) to the landing pad through the registers specified by getExceptionPointerRegister and getExceptionSelectorRegister respectively. On most platforms, they will be GPRs and will be the same as the ones specified in the calling convention. • EH_RETURN The ISD node represents the undocumented GCC extension __builtin_eh_return (offset, handler), which adjusts the stack by offset and then jumps to the handler. __builtin_eh_return is used in GCC unwinder (libgcc), but not in LLVM unwinder (libunwind). If you are on the top of libgcc and have particular requirement on your target, you have to handle EH_RETURN in TargetLowering. If you don’t leverage the existing runtime (libstdc++ and libgcc), you have to take a look on libc++ and libunwind to see what have to be done there. For libunwind, you have to do the following • __libunwind_config.h Define macros for your target. • include/libunwind.h Define enum for the target registers. • src/Registers.hpp Define Registers class for your target, implement setter and getter functions. • src/UnwindCursor.hpp Define dwarfEncoding and stepWithCompactEncoding for your Registers class. • src/UnwindRegistersRestore.S Write an assembly function to restore all your target registers from the memory. • src/UnwindRegistersSave.S Write an assembly function to save all your target registers on the memory.
{ "url": "https://releases.llvm.org/13.0.0/docs/ExceptionHandling.html", "source_domain": "releases.llvm.org", "snapshot_id": "crawl=CC-MAIN-2022-05", "warc_metadata": { "Content-Length": "78409", "Content-Type": "application/http; msgtype=response", "WARC-Block-Digest": "sha1:FJDMSRF32MAXYR5LF5BCPBWWRAI4UBO6", "WARC-Concurrent-To": "<urn:uuid:25237bd1-8f5a-42fc-a032-b8f12afc8ccb>", "WARC-Date": "2022-01-27T02:17:38Z", "WARC-IP-Address": "146.75.34.49", "WARC-Identified-Payload-Type": "application/xhtml+xml", "WARC-Payload-Digest": "sha1:JEFZKZQEZYR4DA57PJAZWJOHLRUFDK5W", "WARC-Record-ID": "<urn:uuid:ee82c1cd-7fad-4150-a5c7-53cad066c893>", "WARC-Target-URI": "https://releases.llvm.org/13.0.0/docs/ExceptionHandling.html", "WARC-Truncated": null, "WARC-Type": "response", "WARC-Warcinfo-ID": "<urn:uuid:97a8d2b1-4e8f-452f-891c-940d74156da6>" }, "warc_info": "isPartOf: CC-MAIN-2022-05\r\npublisher: Common Crawl\r\ndescription: Wide crawl of the web for January 2022\r\noperator: Common Crawl Admin ([email protected])\r\nhostname: ip-10-67-67-45\r\nsoftware: Apache Nutch 1.18 (modified, https://github.com/commoncrawl/nutch/)\r\nrobots: checked via crawler-commons 1.3-SNAPSHOT (https://github.com/crawler-commons/crawler-commons)\r\nformat: WARC File Format 1.1\r\nconformsTo: https://iipc.github.io/warc-specifications/specifications/warc-format/warc-1.1/" }
{ "line_start_idx": [ 0, 27, 28, 41, 42, 433, 434, 475, 476, 807, 808, 1134, 1135, 1507, 1508, 1542, 1543, 1703, 1704, 1951, 1952, 2216, 2217, 2650, 2651, 2686, 2687, 2983, 2984, 2993, 2994, 3119, 3120, 3852, 3853, 4412, 4413, 4845, 4846, 5178, 5179, 5200, 5201, 5413, 5414, 5420, 5421, 5592, 5593, 5810, 5811, 5918, 5919, 6145, 6146, 6156, 6157, 6394, 6395, 6460, 6461, 6563, 6564, 6985, 6986, 7253, 7254, 7600, 7601, 7627, 7628, 7886, 7887, 8036, 8037, 8205, 8206, 8266, 8267, 8469, 8470, 8605, 8606, 8678, 8679, 8974, 8975, 8987, 8988, 9058, 9059, 9123, 9124, 9192, 9193, 9269, 9270, 9922, 9923, 10417, 10418, 10526, 10527, 10650, 10651, 10706, 10707, 10791, 10792, 10882, 10883, 10993, 10994, 11003, 11004, 11084, 11085, 11094, 11095, 11627, 11628, 11633, 11634, 11960, 11961, 12184, 12185, 12199, 12200, 13061, 13062, 13075, 13076, 13397, 13398, 14061, 14062, 14092, 14093, 14294, 14295, 14314, 14315, 14355, 14356, 14578, 14579, 14638, 14639, 14658, 14659, 14708, 14709, 14964, 14965, 15346, 15347, 15595, 15596, 15912, 15913, 15930, 15931, 15956, 15957, 16266, 16267, 16542, 16543, 16791, 16792, 17019, 17020, 17045, 17046, 17103, 17104, 17186, 17187, 17203, 17204, 17339, 17340, 17360, 17361, 17403, 17404, 17795, 17796, 18158, 18159, 18180, 18181, 18225, 18226, 18540, 18541, 18559, 18560, 18585, 18586, 18852, 18853, 18875, 18876, 18924, 18925, 19169, 19170, 19188, 19189, 19330, 19331, 19356, 19357, 19753, 19754, 20070, 20071, 20088, 20089, 20642, 20643, 20688, 20689, 20722, 20723, 20980, 20981, 21791, 21792, 22632, 22633, 23209, 23210, 23909, 23910, 24393, 24394, 24417, 24418, 25377, 25378, 25414, 25415, 26036, 26037, 26552, 26553, 27070, 27071, 27943, 27944, 28055, 28056, 28073, 28086, 28100, 28109, 28112, 28130, 28149, 28157, 28174, 28191, 28211, 28228, 28242, 28246, 28258, 28260, 28330, 28337, 28378, 28405, 28493, 28550, 28551, 28618, 28655, 28716, 28717, 28790, 28866, 28885, 28886, 28977, 29045, 29065, 29066, 29141, 29174, 29249, 29290, 29291, 29373, 29453, 29454, 29526, 29609, 29646, 29709, 29710, 29782, 29816, 29856, 29857, 29942, 29970, 30003, 30017, 30019, 30020, 30042, 30043, 30426, 30427, 30874, 30875, 30960, 30961, 30972, 30980, 30991, 31009, 31019, 31032, 31052, 31058, 31062, 31064, 31150, 31157, 31225, 31286, 31287, 31354, 31417, 31418, 31494, 31549, 31617, 31679, 31680, 31747, 31809, 31810, 31887, 31942, 31980, 31981, 32049, 32088, 32089, 32159, 32170, 32171, 32246, 32260, 32262, 32263, 32343, 32344, 32364, 32365, 32699, 32700, 32844, 32845, 33024, 33025, 33130, 33131, 33197, 33198, 33459, 33460, 33656, 33657, 33815, 33816, 34445, 34446, 35517, 35518, 35719, 35720, 35761, 35762, 35869, 35870, 35889, 35890, 36334, 36335, 36400, 36401, 36787, 36788, 36802, 36803, 37186, 37187, 37381, 37382, 37407, 37408, 37443, 37444, 37468, 37469, 37511, 37512, 37534, 37535, 37618, 37619, 37644, 37645, 37724, 37725, 37758, 37759, 37844, 37845, 37875, 37876 ], "line_end_idx": [ 27, 28, 41, 42, 433, 434, 475, 476, 807, 808, 1134, 1135, 1507, 1508, 1542, 1543, 1703, 1704, 1951, 1952, 2216, 2217, 2650, 2651, 2686, 2687, 2983, 2984, 2993, 2994, 3119, 3120, 3852, 3853, 4412, 4413, 4845, 4846, 5178, 5179, 5200, 5201, 5413, 5414, 5420, 5421, 5592, 5593, 5810, 5811, 5918, 5919, 6145, 6146, 6156, 6157, 6394, 6395, 6460, 6461, 6563, 6564, 6985, 6986, 7253, 7254, 7600, 7601, 7627, 7628, 7886, 7887, 8036, 8037, 8205, 8206, 8266, 8267, 8469, 8470, 8605, 8606, 8678, 8679, 8974, 8975, 8987, 8988, 9058, 9059, 9123, 9124, 9192, 9193, 9269, 9270, 9922, 9923, 10417, 10418, 10526, 10527, 10650, 10651, 10706, 10707, 10791, 10792, 10882, 10883, 10993, 10994, 11003, 11004, 11084, 11085, 11094, 11095, 11627, 11628, 11633, 11634, 11960, 11961, 12184, 12185, 12199, 12200, 13061, 13062, 13075, 13076, 13397, 13398, 14061, 14062, 14092, 14093, 14294, 14295, 14314, 14315, 14355, 14356, 14578, 14579, 14638, 14639, 14658, 14659, 14708, 14709, 14964, 14965, 15346, 15347, 15595, 15596, 15912, 15913, 15930, 15931, 15956, 15957, 16266, 16267, 16542, 16543, 16791, 16792, 17019, 17020, 17045, 17046, 17103, 17104, 17186, 17187, 17203, 17204, 17339, 17340, 17360, 17361, 17403, 17404, 17795, 17796, 18158, 18159, 18180, 18181, 18225, 18226, 18540, 18541, 18559, 18560, 18585, 18586, 18852, 18853, 18875, 18876, 18924, 18925, 19169, 19170, 19188, 19189, 19330, 19331, 19356, 19357, 19753, 19754, 20070, 20071, 20088, 20089, 20642, 20643, 20688, 20689, 20722, 20723, 20980, 20981, 21791, 21792, 22632, 22633, 23209, 23210, 23909, 23910, 24393, 24394, 24417, 24418, 25377, 25378, 25414, 25415, 26036, 26037, 26552, 26553, 27070, 27071, 27943, 27944, 28055, 28056, 28073, 28086, 28100, 28109, 28112, 28130, 28149, 28157, 28174, 28191, 28211, 28228, 28242, 28246, 28258, 28260, 28330, 28337, 28378, 28405, 28493, 28550, 28551, 28618, 28655, 28716, 28717, 28790, 28866, 28885, 28886, 28977, 29045, 29065, 29066, 29141, 29174, 29249, 29290, 29291, 29373, 29453, 29454, 29526, 29609, 29646, 29709, 29710, 29782, 29816, 29856, 29857, 29942, 29970, 30003, 30017, 30019, 30020, 30042, 30043, 30426, 30427, 30874, 30875, 30960, 30961, 30972, 30980, 30991, 31009, 31019, 31032, 31052, 31058, 31062, 31064, 31150, 31157, 31225, 31286, 31287, 31354, 31417, 31418, 31494, 31549, 31617, 31679, 31680, 31747, 31809, 31810, 31887, 31942, 31980, 31981, 32049, 32088, 32089, 32159, 32170, 32171, 32246, 32260, 32262, 32263, 32343, 32344, 32364, 32365, 32699, 32700, 32844, 32845, 33024, 33025, 33130, 33131, 33197, 33198, 33459, 33460, 33656, 33657, 33815, 33816, 34445, 34446, 35517, 35518, 35719, 35720, 35761, 35762, 35869, 35870, 35889, 35890, 36334, 36335, 36400, 36401, 36787, 36788, 36802, 36803, 37186, 37187, 37381, 37382, 37407, 37408, 37443, 37444, 37468, 37469, 37511, 37512, 37534, 37535, 37618, 37619, 37644, 37645, 37724, 37725, 37758, 37759, 37844, 37845, 37875, 37876, 37955 ] }
{ "red_pajama_v2": { "ccnet_original_length": 37955, "ccnet_original_nlines": 426, "rps_doc_curly_bracket": 0.0006850200006738305, "rps_doc_ldnoobw_words": 0, "rps_doc_lorem_ipsum": 0, "rps_doc_stop_word_fraction": 0.3571428656578064, "rps_doc_ut1_blacklist": 0, "rps_doc_frac_all_caps_words": 0.026397520676255226, "rps_doc_frac_lines_end_with_ellipsis": 0, "rps_doc_frac_no_alph_words": 0.1794184148311615, "rps_doc_frac_unique_words": 0.1900797188282013, "rps_doc_mean_word_length": 5.269087791442871, "rps_doc_num_sentences": 446, "rps_doc_symbol_to_word_ratio": 0.0016939600463956594, "rps_doc_unigram_entropy": 5.726980686187744, "rps_doc_word_count": 5645, "rps_doc_frac_chars_dupe_10grams": 0.04407611861824989, "rps_doc_frac_chars_dupe_5grams": 0.10983727872371674, "rps_doc_frac_chars_dupe_6grams": 0.08327730000019073, "rps_doc_frac_chars_dupe_7grams": 0.06300429999828339, "rps_doc_frac_chars_dupe_8grams": 0.057053521275520325, "rps_doc_frac_chars_dupe_9grams": 0.04407611861824989, "rps_doc_frac_chars_top_2gram": 0.02857718989253044, "rps_doc_frac_chars_top_3gram": 0.003294779919087887, "rps_doc_frac_chars_top_4gram": 0.004370629787445068, "rps_doc_books_importance": -2728.049072265625, "rps_doc_books_importance_length_correction": -2728.049072265625, "rps_doc_openwebtext_importance": -1473.30224609375, "rps_doc_openwebtext_importance_length_correction": -1473.30224609375, "rps_doc_wikipedia_importance": -956.0343627929688, "rps_doc_wikipedia_importance_length_correction": -956.0343627929688 }, "fasttext": { "dclm": 0.513968825340271, "english": 0.8432032465934753, "fineweb_edu_approx": 2.710308313369751, "eai_general_math": 0.8815242052078247, "eai_open_web_math": 0.1363736391067505, "eai_web_code": 0.9818041324615479 } }
{ "free_decimal_correspondence": { "primary": { "code": "005.4", "labels": { "level_1": "General works, books and libraries, information sciences", "level_2": "", "level_3": "Computer programming" } }, "secondary": { "code": "005.1", "labels": { "level_1": "General works, books and libraries, information sciences", "level_2": "", "level_3": "Computer programming" } } }, "bloom_cognitive_process": { "primary": { "code": "2", "label": "Understand" }, "secondary": { "code": "3", "label": "Apply" } }, "bloom_knowledge_domain": { "primary": { "code": "2", "label": "Conceptual" }, "secondary": { "code": "3", "label": "Procedural" } }, "document_type_v1": { "primary": { "code": "3", "label": "Reference/Encyclopedic/Educational" }, "secondary": { "code": "-1", "label": "Abstain" } }, "extraction_artifacts": { "primary": { "code": "0", "label": "No Artifacts" }, "secondary": { "code": "3", "label": "Irrelevant Content" } }, "missing_content": { "primary": { "code": "0", "label": "No missing content" }, "secondary": { "code": "-1", "label": "Abstain" } }, "document_type_v2": { "primary": { "code": "8", "label": "Documentation" }, "secondary": { "code": "10", "label": "Knowledge Article" } }, "reasoning_depth": { "primary": { "code": "4", "label": "Advanced Reasoning" }, "secondary": { "code": "3", "label": "Intermediate Reasoning" } }, "technical_correctness": { "primary": { "code": "4", "label": "Highly Correct" }, "secondary": { "code": "3", "label": "Mostly Correct" } }, "education_level": { "primary": { "code": "4", "label": "Graduate/Expert Level" }, "secondary": { "code": "3", "label": "Undergraduate Level" } } }
672f1e42c33a7f9846924a2431ea77df
6,377,116,125,749,535,000
17 $\begingroup$ In how many of the integer numbers between $0$ and $10\,000$ does the digit $3$ appear some place to the left of the digit $4$? This would include, for example, the numbers $34$, $374$, $4384$ and $3874$, but would not include $27$, $43$, $3650$ or $4333$. So I am thinking you do $$8\cdot9\cdot1\cdot1+1\cdot9\cdot1+8\cdot1\cdot9\cdot1+1\cdot9\cdot9\cdot1 = 234$$ But I feel like that is too low. $\endgroup$ 2 • $\begingroup$ You have created confusion between the header : in how many numbers 3 appears, and in the main part: how many numbers 3 appears some place to the left of 4. Please clarify ! $\endgroup$ – true blue anil Mar 14 '16 at 5:00 • 2 $\begingroup$ ZERO if the numbers are binary.... $\endgroup$ – CiaPan Mar 15 '16 at 8:47 19 $\begingroup$ Call such an integer good. Pad with initial $0$'s if necessary to make a $4$-digit string from $0000$ to $9999$. The easy ones are the ones that begin with $3$. Any such string is good if the $3$-digit string after the initial $3$ contains a $4$. There are $10^3$ $3$-digit strings, of which $9^3$ have no $4$, giving $271$ with at least one $4$. Now we count the "other" good strings. There are $9$ times as many as there are good strings that begin with $0$, which is the same as the number of $3$-digit good strings. How many good $3$-digit strings are there? There are the ones that begin with $3$. The remaining $2$-digit string must contain a $4$. There are $19$ of these. Then there are the ones that begin with something else. How many? $9$ times as many as there are good $4$-digit strings that begin with $00$. There is only $1$ of these. We get a total of $271+9(19+(9)(1))$. Remark: The same idea will take care of the count from say $000000$ to $999999$. $\endgroup$ 1 • 3 $\begingroup$ Sounds good to me. And a brute force count confirms that 523 is the correct number. $\endgroup$ – PM 2Ring Mar 14 '16 at 7:19 30 $\begingroup$ We can describe the valid numbers with a FSA using the digits as alphabet (the upper bound on the number is not yet included): FSA Here the black numbers at the arrows denote the symbol used for the transition. If there is no such number every symbol that isn't used with another transition is valid. The red numbers denote the number of symbols that that result in the given target state. States A: No 3 found yet (initial state) B: 3 found but no 4 found to the right of the first 3. C: 4 found to the right of a 3 (final state) We can represent this as a graph with the following adjacency matrix (weighted with number of symbols that can be used for each transition): $$A=\begin{bmatrix}9&1&0\\0&9&1\\0&0&10\\\end{bmatrix}$$ Note that any path in this graph starting at vertex 1 corresponds to a number and every path also ending at vertex 3 is one of the numbers we want to count. We only need to consider numbers 0, ..., 9999 since 10000 obviously doesn't fit the pattern. Therefore we only need to consider words with exactly 4 symbols ($\rightarrow$ path of length 4 in the graph). Since we've chosen the edge weights of the graph appropriately the number to calculate can be directly read from last entry in the first row of the matrix $$A^4=\begin{bmatrix}6561&2916&523\\0&6561&3439\\0&0&10000\\\end{bmatrix}$$ $\endgroup$ 5 • 1 $\begingroup$ Might I suggest relabeling the nodes in the FSA with letters instead of numbers? This would be a bit more clear. $\endgroup$ – jpmc26 Mar 15 '16 at 2:04 • $\begingroup$ Using A as both a state and a matrix is a bit confusing too. $\endgroup$ – Paul Evans Mar 15 '16 at 14:58 • $\begingroup$ A good tool for a good job. $\endgroup$ – Orace Mar 15 '16 at 15:03 • $\begingroup$ Essentially a finite markov chain, but using 0-10 rather than probabilities. Very cool and the perfect tool for the job. $\endgroup$ – Mat Mar 15 '16 at 15:56 • $\begingroup$ And, of course, you can put $A$ into Jordan normal form to get the general count for any number of digits $n$, which turns out to be $10^n-9^{n-1}(n+9)$. That looks like it would be easier to count the numbers that do not have this property. $\endgroup$ – Thomas Andrews Feb 1 '17 at 22:03 26 $\begingroup$ Revised answer Marking the first $3$ and the first $4$ after $3$, there are only $6$ cases: $$\begin{array}{} 3&4&\bullet&\bullet&\implies& 10\cdot10 = 100\\ 3&\bullet&4&\bullet&\implies& 9\cdot 10 = 90\\ 3&\bullet&\bullet&4&\implies& 9\cdot9 =81\\ \bullet&3&4&\bullet&\implies& 9\cdot10 = 90\\ \bullet&3&\bullet&4&\implies& 9\cdot 9 = 81\\ \bullet&\bullet&3&4&\implies& 9\cdot9 = 81 \end{array}$$ Adding up, $523$. Note: The first $4$ after the first $3$ has been marked, but this doesn't preclude a $4$ coming before the first $3$. $\endgroup$ 7 • $\begingroup$ There are a little more, since some will have a $3$ before a $4$ and also a $4$ before a $3$. $\endgroup$ – André Nicolas Mar 14 '16 at 5:18 • $\begingroup$ Yea, thanks and (+1). Deleting for the nonce to examine if this route can lead somewhere ! $\endgroup$ – true blue anil Mar 14 '16 at 5:48 • $\begingroup$ The idea will work. Let $w$ be the number where a $3$ occurs before a $4$ but not the other way, and let $b$ be the number of ways both happen. Then we want $w+b$, and $2w+b$ is your $974$. If we can find $b$, we will be finished, and finding $b$ is not hard, one just has to be careful. $\endgroup$ – André Nicolas Mar 14 '16 at 5:54 • $\begingroup$ So this isn't correct? I feel like it is. $\endgroup$ – user322290 Mar 14 '16 at 7:29 • 2 $\begingroup$ 523 is the correct answer as first given by Andre, I am also getting the same answer after correcting for the fact that the first digit can be $0$ if the number of digits is less than $4$. $\endgroup$ – true blue anil Mar 14 '16 at 7:36 6 $\begingroup$ Let $a_n$ denote the number of strings of $n$ digits that contain at least one $3$ somewhere to the left of at least one $4$. You want to calculate $a_4$. If $b = 0, 1, 2, 4, 5, \dots 9$, there are exactly $a_{n-1}$ such strings whose first digit is $b$. The number of such strings whose first digit is $3$ is the same as the number of $(n-1)$-digit strings containing at least one $4$, or $10^{n-1} - 9^{n-1}$. Therefore $a_n$ can be calculated using the recurrence relation $$a_n = 9a_{n-1} + 10^{n-1} - 9^{n-1}, \quad a_1 = 0.$$ We have $a_2 = 1$, $a_3 = 28$, $a_4 = 523$. Or: We can solve this linear recurrence relation to find $$a_n = 10^n - (n+9)9^{n-1}.$$ Then $a_4 = 10^4 - 13 \times 9^3 = 523.$ $\endgroup$ 2 $\begingroup$ Here is my non-math answer: 523. I wonder if breaking the answer out into 6 cases yields duplicates. i.e. the same number could be counted in more than one case. SYS@dbtstmra AS SYSDBA> SELECT COUNT(*) FROM ( SELECT LPAD( rownum, 4, '0') mynum FROM dba_tab_columns WHERE rownum < 10000 ) WHERE mynum LIKE '%3%4%'; COUNT(*) ---------- 523 1 row selected. Elapsed: 00:00:00.25 $\endgroup$ 6 • 1 $\begingroup$ I think it's WHERE mynum LIKE '%3%4%'. $\endgroup$ – Alexander Mar 14 '16 at 15:22 • 4 $\begingroup$ ... or just seq 10000 | grep '3.*4' | wc -l -> 523. $\endgroup$ – Doorknob Mar 14 '16 at 16:00 • $\begingroup$ @Doorknob: you could probably save a few bytes my omitting the spaces around the pipes. Oh, this is not PCG? Well, then... :-) $\endgroup$ – Thomas Weller Mar 14 '16 at 16:25 • 1 $\begingroup$ @Thomas hahaha. Pyth, 16 bytes: lf:`T"3.*4"0U^T4 $\endgroup$ – Doorknob Mar 14 '16 at 16:28 • 1 $\begingroup$ @Doorknob seq 1e4|grep -c 3.\*4, if you're golfing :) $\endgroup$ – Digital Trauma Mar 14 '16 at 21:28 2 $\begingroup$ Python: >>> len([l for l in map(list, map(str, range(10001))) if '3' in l and '4' in l[l.index('3'):]]) 523 Update: The list map is not needed: >>> len([l for l in map(str, range(10001)) if '3' in l and '4' in l[l.index('3'):]]) 523 $\endgroup$ 1 • 1 $\begingroup$ Minor: you could avoid materializing the list at all, e.g. sum('3' in l and '4' in l[l.index("3"):] for l in map(str, range(10001))). $\endgroup$ – DSM Mar 15 '16 at 15:49 1 $\begingroup$ As David shows, the formula for this with $d$ digits is $$10^d - (d+9)9^{d-1}$$ We can prove this without the recurrence by showing that there are $9^d + d9^{d-1}$ ($=(d+9)9^{d-1}$) numbers where $3$ does not appear before $4$. There are $9^d$ $d$-digit string with no $4$ at all, giving us $9^d$. Every other $d$-digit string has at least one $4$. Suppose the rightmost occurrence of $4$ is at digit $i \in [1, d]$, then every number before it has $9$ choices (anything but $3$, by assumption) and every number after it has $9$ choices (anything but $4$). This gives us our $d9^{d-1}$. Adding the two together gives us all numbers where $3$ is not before $4$, and since there are $10^d$ digits, the answer above is shown to be correct. $\endgroup$ 0 $\begingroup$ Answer: 523 In C#: using System.Text; namespace ThreeBeforeFour { class Program { private const char firstDigit = '3'; private const char secondDigit = '4'; private const ulong upperLimit = 10000; static void Main(string[] args) { ulong count = 0UL; for( ulong i = 0; i <= upperLimit; ++i ) { string s = i.ToString(); int firstPos = s.IndexOf( firstDigit ); if( ( firstPos >= 0 ) && ( s.IndexOf( secondDigit, firstPos + 1 ) >= 0 ) ) { Console.Write( s + ' ' ); count++; } } Console.WriteLine(); Console.WriteLine( "There are " + count + " integers between 0 and " + upperLimit + " that contain the digit " + firstDigit + " followed by the digit " + secondDigit + "." ); Console.WriteLine( "Press return." ); Console.ReadLine(); } } } The output of this program is: 34 134 234 304 314 324 334 340 341 342 343 344 345 346 347 348 349 354 364 374 3 84 394 434 534 634 734 834 934 1034 1134 1234 1304 1314 1324 1334 1340 1341 1342 1343 1344 1345 1346 1347 1348 1349 1354 1364 1374 1384 1394 1434 1534 1634 1734 1834 1934 2034 2134 2234 2304 2314 2324 2334 2340 2341 2342 2343 2344 2345 2346 2347 2348 2349 2354 2364 2374 2384 2394 2434 2534 2634 2734 2834 2934 3004 3014 3024 3034 3040 3041 3042 3043 3044 3045 3046 3047 3048 3049 3054 3064 3074 3084 3094 3104 3114 3124 3134 3140 3141 3142 3143 3144 3145 3146 3147 3148 3149 3154 3164 3174 3184 3194 3204 3214 3224 3234 3240 3241 3242 3243 3244 3245 3246 3247 3248 3249 3254 3264 3274 3284 3294 3304 3314 3324 3334 3340 3341 3342 3343 3344 3345 3346 3347 3348 3349 3354 3364 3374 3384 3394 3400 3401 3402 3403 3404 3405 3406 3407 3408 3409 3410 3411 3412 3413 3414 3415 3416 3417 3418 3419 3420 3421 3422 3423 3424 3425 3426 3427 3428 3429 3430 3431 3432 3433 3434 3435 3436 3437 3438 3439 3440 3441 3442 3443 3444 3445 3446 3447 3448 3449 3450 3451 3452 3453 3454 3455 3456 3457 3458 3459 3460 3461 3462 3463 3464 3465 3466 3467 3468 3469 3470 3471 3472 3473 3474 3475 3476 3477 3478 3479 3480 3481 3482 3483 3484 3485 3486 3487 3488 3489 3490 3491 3492 3493 3494 3495 3496 3497 3498 3499 3504 3514 3524 3534 3540 3541 3542 3543 3544 3545 3546 3547 3548 3549 3554 3564 3574 3584 3594 3604 3614 3624 3634 3640 3641 3642 3643 3644 3645 3646 3647 3648 3649 3654 3664 3674 3684 3694 3704 3714 3724 3734 3740 3741 3742 3743 3744 3745 3746 3747 3748 3749 3754 3764 3774 3784 3794 3804 3814 3824 3834 3840 3841 3842 3843 3844 3845 3846 3847 3848 3849 3854 3864 3874 3884 3894 3904 3914 3924 3934 3940 3941 3942 3943 3944 3945 3946 3947 3948 3949 3954 3964 3974 3984 3994 4034 4134 4234 4304 4314 4324 4334 4340 4341 4342 4343 4344 4345 4346 4347 4348 4349 4354 4364 4374 4384 4394 4434 4534 4634 4734 4834 4934 5034 5134 5234 5304 5314 5324 5334 5340 5341 5342 5343 5344 5345 5346 5347 5348 5349 5354 5364 5374 5384 5394 5434 5534 5634 5734 5834 5934 6034 6134 6234 6304 6314 6324 6334 6340 6341 6342 6343 6344 6345 6346 6347 6348 6349 6354 6364 6374 6384 6394 6434 6534 6634 6734 6834 6934 7034 7134 7234 7304 7314 7324 7334 7340 7341 7342 7343 7344 7345 7346 7347 7348 7349 7354 7364 7374 7384 7394 7434 7534 7634 7734 7834 7934 8034 8134 8234 8304 8314 8324 8334 8340 8341 8342 8343 8344 8345 8346 8347 8348 8349 8354 8364 8374 8384 8394 8434 8534 8634 8734 8834 8934 9034 9134 9234 9304 9314 9324 9334 9340 9341 9342 9343 9344 9345 9346 9347 9348 9349 9354 9364 9374 9384 9394 9434 9534 9634 9734 9834 9934 There are 523 integers between 0 and 10000 that contain the digit 3 followed by the digit 4. Press return. $\endgroup$ Your Answer By clicking “Post Your Answer”, you agree to our terms of service, privacy policy and cookie policy
{ "url": "https://math.stackexchange.com/questions/1696577/in-how-many-of-the-integer-numbers-between-0-and-10-000-does-the-digit-3-appear/1697349", "source_domain": "math.stackexchange.com", "snapshot_id": "crawl=CC-MAIN-2021-10", "warc_metadata": { "Content-Length": "234401", "Content-Type": "application/http; msgtype=response", "WARC-Block-Digest": "sha1:SHKTZEI2HWG2VVHXEDXSFJWA4AXJCFXH", "WARC-Concurrent-To": "<urn:uuid:644f5989-be33-4948-9d11-2992a0e73c2e>", "WARC-Date": "2021-02-28T10:52:15Z", "WARC-IP-Address": "151.101.1.69", "WARC-Identified-Payload-Type": "text/html", "WARC-Payload-Digest": "sha1:BFVFA7ZLPEKF7HCTOUF6QIIR5JNTBFWT", "WARC-Record-ID": "<urn:uuid:4aaf0f2c-6789-4821-8d17-5cc42b919017>", "WARC-Target-URI": "https://math.stackexchange.com/questions/1696577/in-how-many-of-the-integer-numbers-between-0-and-10-000-does-the-digit-3-appear/1697349", "WARC-Truncated": null, "WARC-Type": "response", "WARC-Warcinfo-ID": "<urn:uuid:df6f830f-bf27-43b6-9840-129b1928f72f>" }, "warc_info": "isPartOf: CC-MAIN-2021-10\r\npublisher: Common Crawl\r\ndescription: Wide crawl of the web for February/March 2021\r\noperator: Common Crawl Admin ([email protected])\r\nhostname: ip-10-67-67-229.ec2.internal\r\nsoftware: Apache Nutch 1.18 (modified, https://github.com/commoncrawl/nutch/)\r\nrobots: checked via crawler-commons 1.2-SNAPSHOT (https://github.com/crawler-commons/crawler-commons)\r\nformat: WARC File Format 1.1\r\nconformsTo: http://iipc.github.io/warc-specifications/specifications/warc-format/warc-1.1/" }
{ "line_start_idx": [ 0, 3, 17, 18, 275, 276, 417, 418, 430, 432, 672, 678, 771, 774, 788, 789, 902, 903, 1137, 1138, 1311, 1312, 1471, 1472, 1642, 1643, 1681, 1682, 1763, 1764, 1776, 1778, 1784, 1928, 1931, 1945, 1946, 2073, 2074, 2078, 2079, 2338, 2339, 2346, 2347, 2381, 2382, 2437, 2438, 2483, 2484, 2625, 2626, 2683, 2684, 2841, 2842, 3046, 3047, 3202, 3203, 3279, 3280, 3292, 3294, 3300, 3471, 3595, 3681, 3858, 4166, 4169, 4183, 4184, 4199, 4200, 4277, 4278, 4584, 4585, 4603, 4604, 4722, 4723, 4735, 4737, 4896, 5053, 5406, 5510, 5516, 5771, 5773, 5787, 5788, 5943, 5944, 6201, 6202, 6322, 6323, 6367, 6368, 6497, 6498, 6510, 6512, 6526, 6527, 6689, 6690, 6842, 6843, 6854, 6865, 6876, 6877, 6893, 6894, 6915, 6927, 6929, 6935, 7036, 7042, 7155, 7348, 7354, 7464, 7470, 7591, 7593, 7607, 7608, 7616, 7617, 7713, 7717, 7718, 7726, 7727, 7755, 7756, 7841, 7845, 7857, 7859, 7865, 8055, 8057, 8071, 8072, 8128, 8129, 8153, 8154, 8302, 8303, 8373, 8374, 8663, 8664, 8814, 8815, 8827, 8829, 8843, 8844, 8856, 8857, 8864, 8865, 8884, 8885, 8913, 8914, 8934, 8935, 8980, 9026, 9074, 9075, 9117, 9148, 9203, 9244, 9300, 9393, 9439, 9468, 9486, 9500, 9533, 9564, 9674, 9749, 9764, 9814, 9846, 9856, 9857, 9863, 9864, 9866, 9867, 9898, 9899, 9980, 10061, 10142, 10223, 10304, 10385, 10466, 10547, 10628, 10709, 10790, 10871, 10952, 11033, 11114, 11195, 11276, 11357, 11438, 11519, 11600, 11681, 11762, 11843, 11924, 12005, 12086, 12167, 12248, 12329, 12410, 12491, 12517, 12597, 12610, 12624, 12636, 12637, 12649, 12650 ], "line_end_idx": [ 3, 17, 18, 275, 276, 417, 418, 430, 432, 672, 678, 771, 774, 788, 789, 902, 903, 1137, 1138, 1311, 1312, 1471, 1472, 1642, 1643, 1681, 1682, 1763, 1764, 1776, 1778, 1784, 1928, 1931, 1945, 1946, 2073, 2074, 2078, 2079, 2338, 2339, 2346, 2347, 2381, 2382, 2437, 2438, 2483, 2484, 2625, 2626, 2683, 2684, 2841, 2842, 3046, 3047, 3202, 3203, 3279, 3280, 3292, 3294, 3300, 3471, 3595, 3681, 3858, 4166, 4169, 4183, 4184, 4199, 4200, 4277, 4278, 4584, 4585, 4603, 4604, 4722, 4723, 4735, 4737, 4896, 5053, 5406, 5510, 5516, 5771, 5773, 5787, 5788, 5943, 5944, 6201, 6202, 6322, 6323, 6367, 6368, 6497, 6498, 6510, 6512, 6526, 6527, 6689, 6690, 6842, 6843, 6854, 6865, 6876, 6877, 6893, 6894, 6915, 6927, 6929, 6935, 7036, 7042, 7155, 7348, 7354, 7464, 7470, 7591, 7593, 7607, 7608, 7616, 7617, 7713, 7717, 7718, 7726, 7727, 7755, 7756, 7841, 7845, 7857, 7859, 7865, 8055, 8057, 8071, 8072, 8128, 8129, 8153, 8154, 8302, 8303, 8373, 8374, 8663, 8664, 8814, 8815, 8827, 8829, 8843, 8844, 8856, 8857, 8864, 8865, 8884, 8885, 8913, 8914, 8934, 8935, 8980, 9026, 9074, 9075, 9117, 9148, 9203, 9244, 9300, 9393, 9439, 9468, 9486, 9500, 9533, 9564, 9674, 9749, 9764, 9814, 9846, 9856, 9857, 9863, 9864, 9866, 9867, 9898, 9899, 9980, 10061, 10142, 10223, 10304, 10385, 10466, 10547, 10628, 10709, 10790, 10871, 10952, 11033, 11114, 11195, 11276, 11357, 11438, 11519, 11600, 11681, 11762, 11843, 11924, 12005, 12086, 12167, 12248, 12329, 12410, 12491, 12517, 12597, 12610, 12624, 12636, 12637, 12649, 12650, 12749 ] }
{ "red_pajama_v2": { "ccnet_original_length": 12749, "ccnet_original_nlines": 246, "rps_doc_curly_bracket": 0.003765000030398369, "rps_doc_ldnoobw_words": 0, "rps_doc_lorem_ipsum": 0, "rps_doc_stop_word_fraction": 0.22844693064689636, "rps_doc_ut1_blacklist": 0, "rps_doc_frac_all_caps_words": 0.013071900233626366, "rps_doc_frac_lines_end_with_ellipsis": 0, "rps_doc_frac_no_alph_words": 0.5508870482444763, "rps_doc_frac_unique_words": 0.4806886911392212, "rps_doc_mean_word_length": 4.135411739349365, "rps_doc_num_sentences": 104, "rps_doc_symbol_to_word_ratio": 0.001556179951876402, "rps_doc_unigram_entropy": 6.225252628326416, "rps_doc_word_count": 2149, "rps_doc_frac_chars_dupe_10grams": 0.008776869624853134, "rps_doc_frac_chars_dupe_5grams": 0.08641836047172546, "rps_doc_frac_chars_dupe_6grams": 0.07381568849086761, "rps_doc_frac_chars_dupe_7grams": 0.05164847895503044, "rps_doc_frac_chars_dupe_8grams": 0.0317317396402359, "rps_doc_frac_chars_dupe_9grams": 0.01890401914715767, "rps_doc_frac_chars_top_2gram": 0.019241590052843094, "rps_doc_frac_chars_top_3gram": 0.009452009573578835, "rps_doc_frac_chars_top_4gram": 0.012152579613029957, "rps_doc_books_importance": -1324.6148681640625, "rps_doc_books_importance_length_correction": -1324.6148681640625, "rps_doc_openwebtext_importance": -579.7258911132812, "rps_doc_openwebtext_importance_length_correction": -579.7258911132812, "rps_doc_wikipedia_importance": -385.70452880859375, "rps_doc_wikipedia_importance_length_correction": -385.70452880859375 }, "fasttext": { "dclm": 0.339629590511322, "english": 0.6266905665397644, "fineweb_edu_approx": 2.386153221130371, "eai_general_math": 0.8867950439453125, "eai_open_web_math": 0.5769010782241821, "eai_web_code": 0.1452007293701172 } }
{ "free_decimal_correspondence": { "primary": { "code": "511", "labels": { "level_1": "Science and Natural history", "level_2": "Mathematics", "level_3": "Arithmetic" } }, "secondary": { "code": "005.1", "labels": { "level_1": "General works, books and libraries, information sciences", "level_2": "", "level_3": "Computer programming" } } }, "bloom_cognitive_process": { "primary": { "code": "4", "label": "Analyze" }, "secondary": { "code": "3", "label": "Apply" } }, "bloom_knowledge_domain": { "primary": { "code": "2", "label": "Conceptual" }, "secondary": { "code": "3", "label": "Procedural" } }, "document_type_v1": { "primary": { "code": "5", "label": "Social/Forum" }, "secondary": { "code": "3", "label": "Reference/Encyclopedic/Educational" } }, "extraction_artifacts": { "primary": { "code": "1", "label": "Leftover HTML" }, "secondary": { "code": "3", "label": "Irrelevant Content" } }, "missing_content": { "primary": { "code": "0", "label": "No missing content" }, "secondary": { "code": "-1", "label": "Abstain" } }, "document_type_v2": { "primary": { "code": "18", "label": "Q&A Forum" }, "secondary": { "code": "23", "label": "Tutorial" } }, "reasoning_depth": { "primary": { "code": "4", "label": "Advanced Reasoning" }, "secondary": { "code": "3", "label": "Intermediate Reasoning" } }, "technical_correctness": { "primary": { "code": "4", "label": "Highly Correct" }, "secondary": { "code": "3", "label": "Mostly Correct" } }, "education_level": { "primary": { "code": "3", "label": "Undergraduate Level" }, "secondary": { "code": "2", "label": "High School Level" } } }
672f1e42c33a7f9846924a2431ea77df
701,913,708,339,266,200
Featured The 2022 Raku Advent Posts (in chronological order, with comment references) Featured All the blog posts of 2021 Featured It’s that time of the year When we start all over again with advent calendars, publishing one article a day until Christmas. This is going to be the first full year with Raku being called Raku, and the second year we have moved to this new site. However, it’s going to be the 12th year (after this first article) in a row with a Perl 6 or Raku calendar, previously published in the Perl 6 Advent Calendar blog. And also the 5th year since the Christmas release, which was announced in the advent calendar of that year. Anyway. Here we go again! We have lined a up a full (or eventually full by the time the Advent Calendar is finished) set of articles on many different topics, but all of them about our beloved Raku. So, enjoy, stay healthy, and have -Ofun reading this nice list of articles this year will be bringing. Day 25: Rakudo 2022 Review In a year as eventful as 2022 was in the real world, it is a good idea to look back to see what one might have missed while life was messing with your (Raku) plans. Rakudo saw about 1500 commits this year, about the same as the year before that. Many of these were bug fixes and performance improvements, which you would normally not notice. But there were also commits that actually added features to the Raku Programming Language. So it feels like a good idea to actually mention those more in depth. So here goes! Unless otherwise noted, all of these changes are in language level 6.d, and available thanks to several Rakudo compiler releases during 2022. New REPL functionality It is now possible to refer to values that were produced earlier, using the $*N syntax, where N is a number greater than or equal to 0. $ raku To exit type 'exit' or '^D' [0] > 42 42 [1] > 666 666 [2] > $*0 + $*1 708 Note that the number before the prompt indicates the index with which the value that is going to be produced, can be obtained. New MAIN options You can now affect the interpretation of command line arguments to MAIN by setting these options in the %*SUB-MAIN-OPTS hash: allow-no Allow negation of a named argument to be specified as --no-foo instead of --/foo. numeric-suffix-as-value Allow specification of a numeric value together with the name of a single letter named argument. So -j2 being the equivalent of --j=2. So for example, by putting: my %*SUB-MAIN-OPTS = :allow-no, :numeric-suffix-as-value; at the top of your script, you would enable these features in the command-line argument parsing. New types Native unsigned integers (both in scalar, as well as a (shaped) array) have finally become first class citizens. This means that a native unsigned integer can now hold the value 18446744073709551615 as the largest positive value, from 9223372036854775807 before. This also allowed for a number of internal optimisations as the check for negative values could be removed. As simple as this sounds, this was quite an undertaking to get support for this on all VM backends. my uint $foo = 42; my uint8 $bar = 255; my int8 $baz = 255; say $foo; # 42 say $bar; # 255 say $baz; # -1 say ++$foo; # 43 say ++$bar; # 0 say ++$baz; # 0 And yes, all of the other explicitly sized types, such as uint16uint32 and uint64, are now also supported! New subroutines A number of subroutines entered the global namespace this year. Please note that they will not interfere with any subroutines in your code with the same name, as these will always take precedence. NYI() The NYI subroutine takes a string to indicate a feature not yet implemented, and turns that into a Failure with the X::NYI exception at its core. You could consider this short for ... with feedback, rather than just the “Stub code executed”. say NYI "Frobnication"; # Frobnication not yet implemented. Sorry. chown() The chown subroutine takes zero or more filenames, and changes the UID (with the :uid argument) and/or the GID (with the :gid argument) if possible. Returns the filenames that were successfully changed. There is also a IO::Path.chown method version. my @files = ...; my $uid = +$*USER; my changed = chown @files, :$uid; say "Converted UID of $changed / @files.elems() files"; Also available as a method on IO::Path, but then only applicable to a single path. head(), skip(), tail() The .head.skip and .tail methods got their subroutine counterparts. say head 3, ^10; # (0 1 2) say skip 3, ^10; # (3,4,5,6,7,8,9) say tail 3, ^10; # (7 8 9) Note that the number of elements is always the first positional argument. New methods Any.are The .are method returns the type object that all of the values of the invocant have in common. This can be either a class or a role. say (1, 42e0, .137).are; # (Real) say (1, 42e0, .137, "foo").are; # (Cool) say (42, DateTime.now).are; # (Any) In some languages this functionality appears to be called infer, but this name was deemed to be too ComputerSciency for Raku. IO::Path.inode|dev|devtype|created|chown Some low level IO features were added to the IO::Path class, in the form of 5 new methods. Note that they may not actually work on your OS and/or filesystem. Looking at you there, Windows 🙂 • .inode – the inode of the path (if available) • .dev – the device number of the filesystem (if available) • .devtype – the device identifier of the filesystem (if available) • .created – DateTime object when path got created (if available) • .chown – change uid and/or gid of path (if possible, method version of chown()) (Date|DateTime).days-in-year The Date and DateTime classes already provide many powerfule date and time manipulation features. But a few features were considered missing this year, and so they were added. A new .days-in-year class method was added to the Date and DateTime classes. It takes a year as positional argument: say Date.days-in-year(2023); # 365 say Date.days-in-year(2024); # 366 This behaviour was also expanded to the .days-in-month method, when called as a class method: say Date.days-in-month(2023, 2); # 28 say Date.days-in-month(2024, 2); # 29 They can also be called as instance methods, in which case the parameters default to the associated values in the object: given Date.today { .say; # 2022-12-25 say .days-in-year; # 365 say .days-in-month; # 31 } New Dynamic Variables Dynamic variables provide a very powerful way to keep “global” variables. A number of them are provided by the Raku Programming Language. And now there is one more of them! $*RAT-OVERFLOW Determine the behaviour of rational numbers (aka Rats) if they run out of precision. More specifically when the denominator no longer fits in a native 64-bit integer. By default, Rats will be downgraded to floating point values (aka Nums). By setting the $*RAT-OVERFLOW dynamic variable, you can influence this behaviour. The $*RAT-OVERFLOW dynamic variable is expected to contain a class (or an object) on which an UPGRADE-RAT method will be called. This method is expected to take the numerator and denominator as positional arguments, and is expected to return whatever representation one wants for the given arguments. The following type objects can be specified using core features: Num Default. Silently convert to floating point. Sacrifies precision for speed. CX::Warn Downgrade to floating point, but issue a warning. Sacrifies precision for speed. FatRat Silently upgrade to FatRat, aka rational numbers with arbitrary precision. Sacrifies speed by conserving precision. Failure Return an appropriate Failure object, rather than doing a conversion. This will most likely throw an exception unless specifically handled. Exception Throw an appropriate exception. Note that you can introduce any custom behaviour by creating a class with an UPGRADE-RAT method in it, and setting that class in the $*RAT-OVERFLOW dynamic variable. class Meh { method UPGRADE-RAT($num, $denom) is hidden-from-backtrace { die "$num / $denom is meh" } } my $*RAT-OVERFLOW = Meh; my $a = 1 / 0xffffffffffffffff; say $a; # 0.000000000000000000054 say $a / 2; # 1 / 36893488147419103230 is meh Note that the is hidden-from-backtrace is only added so that any backtrace will show the location of where the offending calculation was done, rather than inside the UPGRADE-RAT method itself. New Environment Variables Quite a few environment variables are already checked by Rakudo whenever it starts. Two more were added in the past year: RAKUDO_MAX_THREADS This environment variable can be set to indicate the maximum number of OS-threads that Rakudo may use for its thread pool. The default is 64, or the number of CPU-cores times 8, whichever is larger. Apart from a numerical value, you can also specify "Inf” or "unlimited" to indicate that Rakudo should use as many OS-threads as it can. These same values can also be used in a call to ThreadPoolScheduler.new with the :max_threads named argument. my $*SCHEDULER = ThreadPoolScheduler.new(:max_threads<unlimited>); INSIDE_EMACS This environment variable can be set to a true value if you do not want the REPL to check for installed modules to handle editing of lines. When set, it will fallback to the behaviour as if none of the supported line editing modules are installed. This appears to be handy for Emacs users, as the name implies 🙂 New experimental features Some Raku features are not yet cast in stone yet, so there’s no guarantee that any code written by using these experimental features, will continue to work in the future. Two new experimental features have been added in the past year: :will-complain If you add a use experimental :will-complain to your code, you can customize typecheck errors by specifying a will complain trait. The trait expects a Callable that will be given the offending value in question, and is expected to return a string to be added to the error message. For example: use experimental :will-complain; my Int $a will complain { "You cannot use -$_-, dummy!" } $a = "foo"; # Type check failed in assignment to $a; You cannot use -foo-, dummy! The will complain trait can be used anywhere you can specify a type constraint in Raku, so that includes parameters and attributes. :rakuast The RakuAST classes allow you to dynamically build an AST (Abstract Syntax Tree programmatically, and have that converted to executable code. What was previously only possible by programmatically creating a piece of Raku source code (with all of its escaping issues), and then calling EVAL on it. But RakuAST not only allows you to build code programmatically (as seen in yesterday’s blog post), it also allows you to introspect the AST, which opens up all sorts of syntax / lintifying possibilities. There is an associated effort to compile the Raku core itself using a grammar that uses RakuAST to build executable code. This effort is now capable of passing 585/1355 test-files in roast completely, and 83/131 of the Rakudo test-files completely. So still a lot of work to do, although it has now gotten to the point that implementation of a single Raku feature in the new grammar, often creates an avalanche of now passing test-files. So, if you add a use experimental :rakuast to your code, you will be able to use all of the currently available RakuAST classes to build code programmatically. This is an entire new area of Raku development, which will be covered by many blog posts in the coming year. As of now, there is only some internal documentation. A small example, showing how to build the expression "foo" ~ "bar": use experimental :rakuast; my $left = RakuAST::StrLiteral.new("foo"); my $infix = RakuAST::Infix.new("~"); my $right = RakuAST::StrLiteral.new("bar"); my $ast = RakuAST::ApplyInfix.new(:$left, :$infix, :$right); dd $ast; # "foo" ~ "bar" This is very verbose, agreed. Syntactic sugar for making this easier will certainly be developed, either in core or in module space. Note how each element of the expression can be created separately, and then combined together. And that you can call dd to show the associated Raku source code (handy when debugging your ASTs). For the very curious, you can check out a proof-of-concept of the use of RakuAST classes in the Rakudo core in the Formatter class, that builds executable code out of an sprintf format. New arguments to existing functionality roundrobin(…, :slip) The roundrobin subroutine now also accepts a :slip named argument. When specified, it will produce all values as a single, flattened list. say roundrobin (1,2,3), <a b c>; # ((1 a) (2 b) (3 c)) say roundrobin (1,2,3), <a b c>, :slip; # (1 a 2 b 3 c) This is functionally equivalent to: say roundrobin((1,2,3), <a b c>).map: *.Slip; but many times more efficient. Cool.chomp($needle) The .chomp method by default any logical newline from the end of a string. It is now possible to specify a specific needle as a positional argument: only when that is equal to the end of the string, will it be removed. say "foobar".chomp("foo"); # foobar say "foobar".chomp("bar"); # foo It actually works on all Cool values, but the return value will always be a string: say 427.chomp(7); # 42 DateTime.posix DateTime value has better than millisecond precision. Yet, the .posix method always returned an integer value. Now it can also return a Num with the fractional part of the second by specifying the :real named argument. given DateTime.now { say .posix; # 1671733988 say .posix(:real); # 1671733988.4723697 } Additional meaning to existing arguments Day from end of month The day parameter to Date.new and DateTime.new (whether named or positional) can now be specified as either a Whatever to indicate the last day of the month, or as a Callable indicating number of days from the end of the month. say Date.new(2022,12,*); # 2022-12-31 say Date.new(2022,12,*-6); # 2022-12-25 Additions in v6.e.PREVIEW You can already access new v6.e language features by specifying use v6.e.PREVIEW at the top of your compilation unit. Several additions were made the past year! term nano nano term is now available. It returns the number of nanoseconds since midnight UTC on 1 January 1970. It is similar to the time term but one billion times more accurate. It is intended for very accurate timekeeping / logging. use v6.e.PREVIEW; say time; # 1671801948 say nano; # 1671801948827918628 With current 64-bit native unsigned integer precision, this should roughly be enough for another 700 years 🙂 prefix // You can now use // as a prefix as well as an infix. It will return whatever the .defined method returns on the given argument). use v6.e PREVIEW; my $foo; say //$foo; # False $foo = 42; say //$foo; # True Basically //$foo is syntactic sugar for $foo.defined. snip() and Any.snip The new snip subroutine and method allows one to cut up a list into sublists according the given specification. The specification consists of one or more smartmatch targets. Each value of the list will be smartmatched with the given target: as soon as it returns False, will all the values before that be produced as a List. use v6.e.PREVIEW; say (2,5,13,9,6,20).snip(* < 10); # ((2 5) (13 9 6 20)) Multiple targets can also be specified. say (2,5,13,9,6,20).snip(* < 10, * < 20); # ((2 5) (13 9 6) (20)) The argument can also be an Iterable. To split a list consisting of integers and strings into sublists of just integers and just strings, you can do: say (2,"a","b",5,8,"c").snip(|(Int,Str) xx *); # ((2) (a b) (5 8) (c)) Inspired by Haskell’s span function. Any.snitch The new .snitch method is a debugging tool that will show its invocant with note by default, and return the invocant. So you can insert a .snitch in a sequence of method calls and see what’s happening “half-way” as it were. $ raku -e 'use v6.e.PREVIEW;\ say (^10).snitch.map(* + 1).snitch.map(* * 2)' ^10 (1 2 3 4 5 6 7 8 9 10) (2 4 6 8 10 12 14 16 18 20) You can also insert your own “reporter” in there: the .snitch method takes a Callable. An easy example of this, is using dd for snitching: $ raku -e 'use v6.e.PREVIEW;\ say (^10).snitch(&dd).map(*+1).snitch(&dd).map(* * 2)' ^10 (1, 2, 3, 4, 5, 6, 7, 8, 9, 10).Seq (2 4 6 8 10 12 14 16 18 20) Any.skip(produce,skip,…) You can now specify more than one argument to the .skip method. Before, you could only specify a single (optional) argument. my @a = <a b c d e f g h i j>; say @a.skip; # (b c d e f g h i j) say @a.skip(3); # (d e f g h i j) say @a.skip(*-3); # (h i j) On v6.e.PREVIEW, you can now specify any number of arguments in the order: produce, skip, produce, etc. Some examples: use v6.e.PREVIEW; my @a = <a b c d e f g h i j>; # produce 2, skip 5, produce rest say @a.skip(2, 5); # (a b h i j) # produce 0, skip 3, then produce 2, skip rest say @a.skip(0, 3, 2); # (d e) # same, but be explicit about skipping rest say @a.skip(0, 3, 2, *); # (d e) In fact, any Iterable can now be specified as the argument to .skip. my @b = 3,5; # produce 3, skip 5, then produce rest say @a.skip(@b); # (a b c i j) # produce 1, then skip 2, repeatedly until the end say @a.skip(|(1,2) xx *); # (a d g j) Cool.comb(Pair) On v6.e.PREVIEW, the .comb method will also accept a Pair as an argument to give it .rotor_-like capabilities. For instance, to produce trigrams of a string, one can now do: use v6.e.PREVIEW; say "foobar".comb(3 => -2); # (foo oob oba bar) This is the functional equivalent of "foobar".comb.rotor(3 => -2)>>.join, but about 10x as fast. Changed semantics on Int.roll|pick To pick a number from 0 till N-1, one no longer has to specify a range, but can use just the integer value as the invocant: use v6.e.PREVIEW; say (^10).roll; # 5 say 10.roll; # 7 say (^10).pick(*); # (2 0 6 9 4 1 5 7 8 3) say 10.pick(*); # (4 6 1 0 2 9 8 3 5 7) Of course, all of these values are examples, as each run will, most likely, produce different results. More interesting stuff There were some more new things and changes the past year. I’ll just mention them very succinctly here: New methods on CompUnit::Repository::Staging .deploy.remove-artifacts, and .self-destruct. :!precompile flag on CompUnit::Repository::Installation.install Install module but precompile on first loading rather than at installation. New methods on Label .file and .line where the Label was created. .Failure coercer Convert a Cool object or an Exception to a Failure. Mainly intended to reduce binary size of hot paths that do some error checking. Cool.Order coercer Coerce the given value to an Int, then convert to Less if less than 0, to Same if 0, and More if more than 0. Allow semi-colon Now allow for the semi-colon in my :($a,$b) = 42,666 because the left-hand side is really a Signature rather than a List. Summary I guess we’ve seen one big change in the past year, namely having experimental support for RakuAST become available. And many smaller goodies and tweaks and features. Now that RakuAST has become “mainstream” as it were, we can think of having certain optimizations. Such as making sprintf with a fixed format string about 30x as fast! Exciting times ahead! Hopefully you will all be able to enjoy the Holiday Season with sufficient R&R. The next Raku Advent Blog is only 340 days away! Day 24: He’s making a list… (part 2) In our last edition, we learned about some of the work that Santa’s elves put into automating how they make their lists. What you probably didn’t know is that the elves stay on top of the latest and greatest technology. Being well-known avid Raku programmers, the elves were excited to hear about RakuAST and decided to see how they might be able to use it. One of the elves decided to rework the list formatting code to use RakuAST. What follows is the story of how she upgraded their current technology to use RakuAST. Background The current code that the elves had is fairly straight forward (check out part one for a full explanation) sub format-list( +@items, :$language 'en', :$type = 'and', :$length = 'standard' ) { state %formatters; my $code = "$language/$type/$length"; # Get a formatter, generate if it's not been requested before my &formatter = %cache{$code} // %cache{$code} = generate-list-formatter($language, $type, $length); formatter @items; } sub generate-list-formatter($language, $type, $length --> Sub ) { # Get CLDR information my $format = cldr{$language}.list-format{$type}{$length}; my ($start, $middle, $end, $two) = $format<start middle end two>.map: *.substr(3,*-3).raku; # Generate code my $code = q:s:to/FORMATCODE/; sub format-list(+@items) { if @items > 2 { @items[0] ~ $start ~ @items[1..*-2].join($middle) ~ $end ~ @items[*-1] } elsif @items == 2 { @items[0] ~ $two ~ @items[1] } elsif @items == 1 { @items[0] } else { '' } } FORMATCODE # compile and return use MONKEY-SEE-NO-EVAL; EVAL $code } While the caching technique is rudimentary and technically not thread-safe, it works (a different elf will probably revisit the code to make it so). Now, when creating all the lists for, say, children in Georgia, the data for Georgian list formatters in CLDR will only need to be accessed a single time. For the next half a million or so calls, the code will be run practically as fast as if it had been hard coded (since, in effect, it has been). The problem is how the generate-list-formatter code works. The code block uses a heredoc-style :to string, but it’s interpolated. There are numerous ways to accomplish this but all of them require having to use proper escapes. That’s…. risky. Another elf, having seen the performance improvements that this new EVAL code brought, wanted to find a way to avoid the risky string evaluation. She had heard about the new RakuAST and decided to give it a whirl. While it initially looked more daunting, she quickly realized that RakuAST was very powerful. What is RakuAST RakuAST is an object-based representation of Raku’s abstract syntax tree, or roughly what you might get if you parsed Raku’s code into its individual elements. For instance, a string literal might be represented as 'foo' in code, but once parsed, becomes a string literal. That string literal, by the way, can be created by using RakuAST::StrLiteral.new(…). Remember how the elf had to worry about how the string might be interpolated? By creating a the string literal directly via a RakuAST node, that whole process is safely bypassed. No RakuAST::StrLiteral node can be created that will result in a string injection! Every single construct in the Raku language has an associated RakuAST node. When creating nodes, you might frequently pass in another node, which means you can build up code objects in a piece-by-piece fashion, and again, without ever worrying about string interpolation, escaping, or injection attacks. So let’s see how the elf eventually created the safer RakuAST version of the formatter method. The elf works her AST off To ease her transition into RakuAST, the elf decided to go from the simplest to the most complex part of the code. The simplest is the value for the final else block: my $none = RakuAST::StrLiteral.new('');  Okay. That was easy. Now she wanted to tackle the single element value. In the original code, that was @list.head. Although we don’t normally think of it as such, . is a special infix for method calling. Operators can be used creating an RakuAST::Apply___fix node, where ___ is the type of operator. Depending on the node, there are different arguments. In the case of RakuAST::ApplyPostfix, the arguments are operand (the list), and postfix which is the actual operator. These aren’t as simple as typing in some plain text, but when looking at the code the elf came up with, it’s quite clear what’s going on: my $operand = RakuAST::Var::Lexical.new('@list'); my $postfix = RakuAST::Call::Method.new( name => RakuAST::Name.from-identifier('head') ); my $one = RakuAST::ApplyPostfix.new(:$operand, :$postfix)  The operand isn’t a literal, but a variable. Specifically, it’s a lexical variable, so we create a node that will reference it. The call method operator needs a name as well, so we do that as well. This involves a lot of assignment statements. Sometimes that can be helpful, but for something this simple, the elf decided it was easier to write it as one “line”: my $one = RakuAST::ApplyPostfix.new( operand => RakuAST::Var::Lexical.new('@list'), postfix => RakuAST::Call::Method.new( name => RakuAST::Name.from-identifier('head') ) ); Alright, so the first two cases are done. How might she create the result for when the list has two items? Almost exactly like the last time, except now she’d provide an argument. While you might think it would be as simple as adding args => RakuAST::StrLiteral($two-infix), it’s actually a tiny bit more complicated because in Raku, argument lists are handled somewhat specially, so we actually need a RakuAST::ArgList node. So the equivalent of @list.join($two-infix) is my $two = RakuAST::ApplyPostfix.new( operand => RakuAST::Var::Lexical.new('@list'), postfix => RakuAST::Call::Method.new( name => RakuAST::Name.from-identifier('join'), args => RakuAST::ArgList.new( RakuAST::StrLiteral.new($two-infix) ) ) );   The RakuAST::ArgList takes in a list of arguments — be they positional or named (named applied by way of a RakuAST::FatComma). Finally, the elf decided to tackle what likely would be the most complicated bit: the code for 3 or more items. This code makes multiple method calls (including a chained one), as well as combining everything with a chained infix operator. The method calls were fairly straightforward, but she thought about what the multiple ~ operators would be handled. As it turns out, it would actually require being set up as if (($a ~ $b) ~ $c) ~ $d, etc., and the elf didn’t really like the idea of having ultimately intending her code that much. She also thought about just using join on a list that she could make, but she already knew how to do method calls, so she thought she’d try something cool: reduction operators (think [~] $a, $b, $c, $d for the previous). This uses the RakuAST::Term::Reduce node that takes a simple list of arguments. For the * - 2 syntax, to avoid getting too crazy, she treated it as if it had been written as the functionally identical @list - 2. Becaused that reduction bit has some many elements, she ending up breaking things into pieces: the initial item, the special first infix, a merged set of the second to penultimate items joined with the common infix, the special final infix, and the final item. For a list like [1,2,3,4,5] in English, that amounts to 1 (initial item), , (first infix), 2, 3, 4 (second to penultimate, joined with , ), , and (final infix) and 5 (final item). In other languages, the first and repeated infixes may be different, and in others, all three may be identical. # @list.head my $more-first-item = RakuAST::ApplyPostfix.new( operand => RakuAST::Var::Lexical.new('@list'), postfix => RakuAST::Call::Method.new( name => RakuAST::Name.from-identifier('head') ) ); # @list[1, * - 2].join($more-middle-infix) my $more-mid-items = RakuAST::ApplyPostfix.new( # @list[1, @list - 2 operand => RakuAST::ApplyPostfix.new( operand => RakuAST::Var::Lexical.new('@list'), postfix => RakuAST::Postcircumfix::ArrayIndex.new( # (1 .. @list - 2) RakuAST::SemiList.new( RakuAST::ApplyInfix.new( left => RakuAST::IntLiteral.new(1), infix => RakuAST::Infix.new('..'), # @list - 2 right => RakuAST::ApplyInfix.new( left => RakuAST::Var::Lexical.new('@list'), infix => RakuAST::Infix.new('-'), right => RakuAST::IntLiteral.new(2) ) ) ) ) ), # .join($more-middle-infix) postfix => RakuAST::Call::Method.new( name => RakuAST::Name.from-identifier('join'), args => RakuAST::ArgList.new( RakuAST::StrLiteral.new($more-middle-infix) ) ) ); # @list.tail my $more-final-item = RakuAST::ApplyPostfix.new( operand => RakuAST::Var::Lexical.new('@list'), postfix => RakuAST::Call::Method.new( name => RakuAST::Name.from-identifier('tail') ) ); # [~] ... my $more = RakuAST::Term::Reduce.new( infix => RakuAST::Infix.new('~'), args => RakuAST::ArgList.new( $more-first-item, RakuAST::StrLiteral.new($more-first-infix), $more-mid-items, RakuAST::StrLiteral.new($more-final-infix), $more-final-item, ) ); As one can note, as RakuAST code starts getting more complex, it can be extremely helpful to store interim pieces into variables. For complex programs, some RakuAST users will create functions that do some of the verbose stuff for them. For instance, one might get tired of the code for an infix, and write a sub like sub rast-infix($left, $infix, $right) { RakuAST::ApplyInfix.new: left => $left, infix => RakuAST::Infix.new($infix), right => $right } to enable code like rast-infix($value, '+', $value) which ends up being much less bulky. Depending on what they’re doing, they might make a sub just for adding two values, or maybe making a list more compactly. In any case, the hard working elf had now programmatically defined all of the formatter code. All that was left was for her to piece together the number logic and she’d be done. That logic was, in practice, quite simple: if @list > 2 { $more } elsif @list == 2 { $two } elsif @list == 1 { $one } else { $none }  In practice, there was still a bit of a learning curve. Why? As it turns out, the [els]if statements are actually officially expressions, and need to be wrapped up in an expression block. That’s easy enough, she could just use RakuAST::Statement::Expression. Her conditions end up being coded as # @list > 2 my $more-than-two = RakuAST::Statement::Expression.new( expression => RakuAST::ApplyInfix.new( left => RakuAST::Var::Lexical.new('@list'), infix => RakuAST::Infix.new('>'), right => RakuAST::IntLiteral.new(2) ) ); # @list == 2 my $exactly-two = RakuAST::Statement::Expression.new( expression => RakuAST::ApplyInfix.new( left => RakuAST::Var::Lexical.new('@list'), infix => RakuAST::Infix.new('=='), right => RakuAST::IntLiteral.new(2) ) ); # @list == 1 my $exactly-one = RakuAST::Statement::Expression.new( expression => RakuAST::ApplyInfix.new( left => RakuAST::Var::Lexical.new('@list'), infix => RakuAST::Infix.new('=='), right => RakuAST::IntLiteral.new(1) ) );   That was simple enough. But now sure realized that the then statements were not just the simple code she had made, but were actually a sort of block! She would need to wrap them with a RakuAST::Block. A block has a required RakuAST::Blockoid element, which in turn has a required RakuAST::Statement::List element, and this in turn will contain a list of statements, the simplest of which is a RakuAST::Statement::Expression that she had already seen. She decided to try out the technique of writing a helper sub to do this: sub wrap-in-block($expression) { RakuAST::Block.new( body => RakuAST::Blockoid.new( RakuAST::StatementList.new( RakuAST::Statement::Expression.new(:$expression) ) ) ) } $more = wrap-in-block $more; $two = wrap-in-block $two; $one = wrap-in-block $one; $none = wrap-in-block $none;  Phew, that was a pretty easy way to handle some otherwise very verbose coding. Who knew Raku hid away so much complex stuff in such simple syntax?! Now that she had both the if and then statements finished, she was ready to finish the full conditional: my $if = RakuAST::Statement::If.new( condition => $more-than-two, then => $more, elsifs => [ RakuAST::Statement::Elsif.new( condition => $exactly-two, then => $two ), RakuAST::Statement::Elsif.new( condition => $exactly-one, then => $one ) ], else => $none );  All that was left was for her to wrap it up into a Routine and she’d be ready to go! She decided to put it into a PointyBlock, since that’s a sort of anonymous function that still takes arguments. Her fully-wrapped code block ended up as: my $code = RakuAST::PointyBlock.new( signature => RakuAST::Signature.new( parameters => ( RakuAST::Parameter.new( target => RakuAST::ParameterTarget::Var.new('@list'), slurpy => RakuAST::Parameter::Slurpy::SingleArgument ), ), ), body => RakuAST::Blockoid.new( RakuAST::StatementList.new( RakuAST::Statement::Expression.new( expression => $if ) ) ) );  Working with RakuAST, she really got a feel for how things worked internally in Raku. It was easy to see that a runnable code block like a pointy block consisted of a signature and a body. That signature had a list of parameters, and the body a list of statements. Seems obvious, but it can be enlightening to see it spread out like she had it. The final step was for her actually evaluate this (now much safer!) code. For that, nothing changed. In fact, the entire rest of her block was simply sub generate-list-formatter($language, $type, $length) { use Intl::CLDR; my $pattern = cldr{$lang}.list-patterns{$type}{$length}; my $two-infix = $pattern.two.substr: 3, *-3; my $more-first-infix = $pattern.start.substr: 3, *-3; my $more-middle-infix = $pattern.middle.substr: 3, *-3; my $more-final-infix = $pattern.end.substr: 3, *-3; ... use MONKEY-SEE-NO-EVAL; EVAL $code } Was her code necessarily faster than the older method? Not necessarily. It didn’t require a parse phase, which probably saved a bit, but once compiled, the speed would be the same. So why would she bother doing all this extra work when some string manipulation could have produced the same result? A number of reasons. To begin, she learned the innards of RakuAST, which helped her learn the innards of Raku a bit better. But for us non-elf programmers, RakuAST is important for many other reasons. For instance, at every stage of this process, everything was fully introspectable! If your mind jumped to writing optimizers, besides being a coding masochist, you’ve actually thought about something that will likely come about. Macros is another big feature that’s coming in Raku and will rely heavily on RakuAST. Rather than just do text replacement in the code like macros in many other languages, macros will run off of RakuAST nodes. This means an errant quote will never cause problems, and likely enable far more complex macro development. DSL developers can seamlessly integrate with Raku by just compiling down to RakuAST. The future So what is the status of RakuAST? When can you use it? As of today, you will need to build the most recent main branch of Rakudo to use it. Then, in your code, include the statement use experimental :rakuast;. Yours truly will be updating a number of his formatting modules to use RakuAST very shortly which will make them far more maintainable and thus easier to add new features. For more updates on the progress of RakuAST, check out the Rakudo Weekly, where Elizabeth Mattijsen gives regular updates on RakuAST and all things Raku. Day 23: Sigils followup: semantics and language design Until a few days ago, I’d intended for this post to be an update on the Raku persistent data structures I’m developing. And I have included a (very brief) status update at the end of this post. But something more pressing has come to my attention: Someone on the Internet was wrong — and that someone was me. xkcd_386 Specifically, in my post about sigils the other day, I significantly misdescribed the semantics that Raku applies to sigiled-variables. Considering that the post was about sigils, the final third focused on Raku’s sigils, and much of that section discussed the semantics of those sigils – being wrong about the semantics of Raku’s sigils isn’t exactly a trivial mistake. Oops! In partial mitigation, I’ll mention one thing: no one pointed out my incorrect description of the relevant semantics, even though the post generated over two hundred comments of discussion, most of it thoughtful. Now, it could be no one read all the way to Part 3 of a 7,000 word post (an understandable choice!). But, considering the well-known popularity of correcting people on the Internet, I view the lack of any correction as some evidence that my misunderstanding wasn’t obvious to others either. In fact, I only discovered the issue when I decided, while replying to a comment on that post, to write an an oddly-designed Raku class to illustrate the semantics I’d described; much to my suprise, it showed that I’d gotten those semantics wrong. Clearly, that calls for a followup post, which you’re now reading. My goal for this post is, first of all, to explain what I got wrong about Raku’s semantics, how I made that error, and why neither I nor anyone else noticed. Then we’ll turn to some broader lessons about language design, both in Raku and in programming languages generally.  Finally, with the benefit of correctly understanding of Raku’s semantics, we’ll reevaluate Raku’s sigils, and the expressive power they provide. What I got wrong – and what I got right In that post, I said that the @ sigil can only be used for types that implement the Positional (“array-like”) role; that the % sigil can only be used for types that implement the Associative (“hash-like”) role; and that the & sigil can only be used for types that implement the Callable (“function-like”) role. All of that is right (and pretty much straight from the language docs). Where I went wrong was when I described the requirements that a type must satisfy in order to implement those roles. I described the Positional role as requiring an iterable, ordered collection that can be indexed positionally (e.g., with @foo[5]); I described the Associative role as requiring an iterable, unordered collection of Pairs that can be indexed associatively (e.g., with %foo<key>); and I described the Callable role as requiring a type to support being called as a function (e.g., with &foo()). That, however, was an overstatement. The requirements for implementing those three roles are actually: absolutely nothing. That’s right, they’re entirely “marker roles”, the Raku equivalent of Rust’s marker traits. Oh sure, the Raku docs provide lists of methods that you should implement, but those are just suggestions. There’s absolutely nothing stopping us from writing classes that are Associative, Positional, or Callable, or – why not? – all three if we want to. Or, for that matter, since Raku supports runtime composition, the following is perfectly valid: my @pos := 'foo' but Positional; my %asc := 90000 but Associative; my &cal := False but Callable; Yep, we can have a Positional string, an Associative number, and a Callable How did we miss that? So, here’s the thing: I’ve written quite a bit of Raku code while operating under the mistaken belief that those roles had the type constraints I described – which are quite a bit stricter than “none at all”. And I don’t think I’m alone in that; in fact, the most frequent comment I got on the previous post was surprise/confusion that @ and % weren’t constrained to concrete Arrays and Hashes (a sentiment I’ve heard before). And I don’t think any of us were crazy to think those sorts of things – when you first start out in Raku, the vast majority (maybe all) of the @– and %-sigiled things you see are Arrays and Hashes. And I don’t believe I’ve ever seen an @-sigiled variable in Raku that wasn’t an ordered collection of some sort. So maybe people thinking that the type constraints are stricter makes a certain amount of sense. But that, in turn, just raises two more questions: First, given the unconstrained nature of those sigils, why haven’t I seen some Positional strings in the wild? After all, relying on programmer discipline instead of tool-enforcement is usually a recipe for quick and painful disaster. And, second, given that @– and % Good defaults > programmer discipline Let’s address those questions in order: Why haven’t I seen @-sigiled strings or %-sigiled numbers? Because Raku isn’t relying on programmer discipline to prevent those things; it’s relying on programmer laziness – a much stronger force. Writing my @pos := 'foo' but Positional seems very easy, but it has three different elements that would dissuade a new Rakoon from writing it: the := bind operator (most programmers are more familiar with assignment, and = is overwhelmingly more common in Raku code examples); the but operator (runtime composition is relatively uncommon in the wider programming world, and it’s not a tool Raku code turns to all that often) and Positional (roles in general aren’t really a Raku 101 topic, and Positional/Associative/Callable even less so – after all, all the built-in types that should implement those roles already do so). Let’s contrast that line with the version that a new Rakoon would be more likely to write – indeed, the version that every Rakoon must have written over and over: my @pos = 'foo'. That removes all three of the syntactic stumbling blocks from the preceding code. More importantly, it works. Because the @-sigil provides a default Array container, that line creates the Array ['foo'] – which is much more likely to be what the user wanted in the first place. Of course, that’s just one example, but the general pattern holds: Raku very rarely prohibits users from doing something (even something as bone-headed as a Positional string) but it’s simultaneously good at making the default/easiest path one that avoids those issues. If there’s an easy-but-less-rigorous option available, then no amount of “programmer discipline” will prevent everyone from taking it. But when the safer/saner thing is also by far the easier thing, then we’re not relying on programmer discipline. We’re removing the temptation entirely. And then by the time someone has written enough Raku that :=, but, and Positional wouldn’t give them any pause, they probably have the “@ means “array-like, but maybe not an Array” concept so deeply ingrained that they wouldn’t consider creating a wacky Positional Being stricter What about the second question we posed earlier: Why doesn’t Raku enforce a tighter type constraint? It certainly could: Raku has the language machinery to really tighten down the requirements for a role. It would be straightforward to mandate that any type implementing the Positional role must also implement the methods for positional indexing. And, since Raku already has an Iterable role, requiring Positional types to be iterable would also be trivial. So why not? Well, because – even if the vast majority of Positional types should allow indexing and should be iterable, there will be some that have good reasons not to be. And Raku could turn the “why not?” question around and ask “why?” Providing guarantees versus communicating intent All of this brings a question into focus – a question that goes right to the heart of Raku’s design philosophy and is an important one for any language designer to consider. That question is: Is your language more interested in providing guarantees or in communicating intent Guarantees are great When I’m not writing Raku (or long blog posts), the programming language I spend the most time with is Rust. And Rust is very firmly on the providing guarantees side of that issue. And it’s genuinely great. There’s something just absolutely incredible and freeing about having the Rust compiler and a strong static type system at your back, of knowing that you just absolutely, 100% don’t need to worry about certain categories of bugs or errors. With that guarantee, you can drop those considerations from your mental cache altogether (you know, to free up space for the things that are cognitively complex in Rust – which isn’t a tiny list). So, yes, I saw the appeal when primarily writing Rust and I see it again every time I return to the language. Indeed, I think Rust’s guarantees are 100% the right choice – for Rust. I believe that the strength of those guarantees was a great fit for Rust’s original use case (working on Firefox) and are a huge part of why Facebook, Microsoft, Amazon, and Google have all embraced Rust: when you’re collaborating on a team with the scope of a huge open-source project or a big tech company, guarantees become even more valuable. When some people leave, new ones join, and there’s no longer a way to get everyone on the same page, it’s great to have a language that says “you don’t have to trust their code, just trust me”. But the thing about guarantees is that they have to be absolute. If something is “90% guaranteed”, then it’s not Coding as a collaborative, asynchronous communication Guarantees-versus-communication is one trade off where Raku makes the other choice, in a big way. Raku is vastly more interested in helping programmers to communicate their intent than in enforcing rules strictly enough to make guarantees. If Rust’s fundamental metaphor for code is the deductive proof – each step depends on the correctness of the previous ones, so we’d better be as sure as possible that they’re right – Raku’s fundamental metaphor is, unsurprisingly, more linguistic. Raku’s metaphor for coding is an asynchronous conversation between friends: an email exchange, maybe, or — better yet – a series of letters. How is writing code like emailing a friend? Well, we talked last time about the three-way conversation between author, reader, and compiler, but that’s a bit of a simplification. Most of the time, we’re simultaneously reading previously-written code and writing additional code, which turns the three-way conversation into a four-way one. True, the “previous author”, “current reader/author”, and “future reader” might all be you, but the fact that you’re talking to yourself doesn’t make it any less of a conversation: either way, the goal is to understand the previous author’s meaning as well as possible, decide what you want to add to the conversation, and then express yourself as clearly as possible – subject to the constraint that the compiler also needs to understand your code. A few words on that last point. From inside a code-as-proof metaphor, a strict compiler is a clear win. Being confident in the correctness of anything is hard enough, but it’s vastly harder as you increase the possibility space. But from a code-as-communication metaphor, there’s a real drawback to compilers (or formatters) that limit your ability to say the same thing in multiple ways. What shirt you wear stops being an expressive choice if you’re required to wear a uniform. In the same way, when there’s exactly one way to do something, then doing it that way doesn’t communicate anything. But when there’s more than one way to do it, then suddenly it makes sense to ask, “Okay, but why did they do it that way?”. This is deeply evident in Raku: there are multiple ways to write code that does the same thing, but those different ways don’t say the same thing – they allow you to place the emphasis in different points, depending on where you’d like to draw the reader’s attention. Raku’s large “vocabulary” plays the same role as increasing your vocabulary in a natural language: it makes it easier to pick just the right word. When code is communication, rules become suggestions When emailing a friend, neither of you can set “rules” that the other person must follow. You can make an argument for why they shouldn’t do something, you can express clearly and unequivocally that doing that would be a mistake, but you can’t stop them. You are friends – equals – and neither the email’s author nor its reader can overrule the other. And the same is true of Raku: Raku makes it very difficult (frequently impossible) for the author of some code to 100% prevent someone from using their code in a particular way. Raku provides many ways to express – with all the intensity of an ALL CAPS EMAIL – that doing something is a really, really bad idea. But if you are determined to misuse code and knowledgeable enough, there’s pretty much no stopping you. Coming from Rust, this took me a while to notice, because (at least in intro materials) Raku presents certain things as absolute rules (“private attributes cannot be accessed outside the class!”) when, in reality, they turn out to be strongly worded suggestions (”…unless you’re messing with the Meta Object Protocol in ways that you really shouldn’t”). From a Rust perspective, that just wouldn’t fly – private implementations should be private, But it fits perfectly with Raku’s overall design philosophy. Communicating through sigils Applying this design philosophy to sigils, I’ve come around to believing that making Possitional, Associative, and Callable marker roles was entirely the correct choice. After all, marker roles are entirely about communicating through code – even in Rust, the entire purpose of marker traits is to communicate some property that the Rust compiler can’t verify. This is a perfect fit for sigils. What does @ mean? It means that the variable is Positional. Okay, what does Positional mean? It means “array-like”… Okay. What does “array-like” mean? Well, that’s up to you to decide, as part of the collaborative dialogue (trialogue?) with the past and future authors. That doesn’t mean you’re on your own, crafting meaning from the void: Raku keeps us on the same general page by ensuring that every Rakoon has extensive experience with Arrays, which creates a shared understanding for what “array-like” means. And the language documentation provides clear explanations of how to make your custom types behave like Raku’s Array. But – as I now realize – Raku isn’t going to stomp its foot and say that @-sigiled variables must behave a particular way. If it makes sense – in your code base, in the context of your multilateral conversation – to have an @-sigiled variable that is neither ordered nor iterable, then you can. So, I’m disappointed that I was mistaken about Raku’s syntax when I wrote my previous post. And I’m especially sorry if anyone was confused by the uncorrected version of that post. But I’m really glad to realize Raku’s actual semantics for sigils, because it fits perfectly with Raku as a whole.  Moreover, these semantics not only fit better with Raku’s design, they make Raku’s sigil’s even more better-suited for their primary purpose: helping someone writing code to clearly and concisely communicate their intent to someone reading that code In keeping with my earlier post, I’ll include a table with the semantics of the three sigils we discussed: Sigil Meaning @ Someone intentionally marked the variable Positional % Someone intentionally marked the variable Associative & Someone intentionally marked the variable Callable These semantics are perfect because, in the end, that’s what @, %, &, and $ really are: signs of what someone else intended. Little, semantically dense, magic signs. Day 22: He’s making a list… (part 1) If there’s anything that Santa and his elves ought to know, it’s how to make a list. After all, they’re reading lists that children send in, and Santa maintains his very famous list. Another thing we know is that Santa and his elves are quite multilingual. So one day one of the elfs decided that, rather than hand typing out a list of gifts based on the data they received (requiring elves that spoke all the world’s languages), they’d take advantage of the power of Unicode’s CLDR (Common Linguistic Data Repository). This is Unicode’s lesser-known project. As luck would have it, Raku has a module providing access to the data, called Intl::CLDR. One elf decided that he could probably use some of the data in it to automate their list formatting. He began by installing Intl::CLDR and played around with it in the terminal. The module was designed to allow some degree of exploration in a REPL, so the elf did the following after reading the provided read me: # Repl response use Intl::CLDR; # Nil my $english = cldr<en> # [CLDR::Language: characters,context-transforms, # dates,delimiters,grammar,layout,list-patterns, # locale-display-names,numbers,posix,units] The module loaded up the data for English and the object returned had a neat gist that provides information about the elements it contains. For a variety of reasons, Intl::CLDR objects can be referenced either as attributes or as keys. Most of the time, the attribute reference is faster in performance, but the key reference is more flexible (because let’s be honest, $english{$foo} looks nicer than $english."$foo"(), and it also enables listy assignment via e.g. $english<grammar numbers>). In any case, the elf saw that one of the data points is list-patterns, so he explored further: # Repl response $english.list-patterns; # [CLDR::ListPatterns: and,or,unit] $english.list-patterns.and; # [CLDR::ListPattern: narrow,short,standard] $english.list-patterns.standard; # [CLDR::ListPatternWidth: end,middle,start,two] $english.list-patterns.standard.start; # {0}, {1} $english.list-patterns.standard.middle; # {0}, {1} $english.list-patterns.standard.end; # {0}, and {1} $english.list-patterns.standard.two; # {0} and {1} Aha! He found the data he needed. List patterns are catalogued by their function (and-ing them, or-ing them, and a unit one designed for formatting conjoined units such as 2ft 1in or similar). Each pattern has three different lengths. Standard is what one would use most of the time, but if space is a concern, some languages might allow for even slimmer formatting. Lastly, each of those widths has four forms. The two form combines, well, two elements. The other three are used to collectively join three or more: start combines the first and second element, end combines the penultimate and final element, and middle combines all second to penultimate elements. He then wondered what this might look like for other languages. Thankfully, testing this out in the repl was easy enough: my &and-pattern = { cldr{$^language}.list-patterns-standard<start middle end two>.join: "\t"'" } # Repl response (RTL corrected, s/\t/' '+/) and-pattern 'es' # {0}, {1} {0}, {1} {0} y {1} {0} y {1} and-pattern 'ar' # ‮{0} و{1} {0} و{1} {0} و{1} {0} و{1} and-pattern 'ko' # {0}, {1} {0}, {1} {0} 및 {1} {0} 및 {1} and-pattern 'my' # {0} - {1} {0} - {1} {0}နှင့် {1} {0}နှင့် {1} and-pattern 'th' # {0} {1} {0} {1} {0} และ{1} {0}และ{1} He quickly saw that there was quite a bit of variation! Thank goodness someone else had already catalogued all of this for him. So he went about trying to create a simple formatting routine. To begin, he created a very detailed signature and then imported the modules he’d need. #| Lengths for list format. Valid values are 'standard', 'short', and 'narrow'. subset ListFormatLength of Str where <standard short narrow>; #| Lengths for list format. Valid values are 'and', 'or', and 'unit'. subset ListFormatType of Str where <standard short narrow>; use User::Language; # obtains default languages for a system use Intl::LanguageTag; # use standardized language tags use Intl::CLDR; # accesses international data #| Formats a list of items in an internationally-aware manner sub format-list( +@items, #= The items to be formatted into a list LanguageTag() :$language = user-language #= The language to use for formatting ListFormatLength :$length = 'standard', #= The formatting width ListFormatType :$type = 'and' #= The type of list to create ) { ... ... ... } That’s a bit of a big bite, but it’s worth taking a look at. First, the elf opted to use declarator POD wherever it’s possible. This can really help out people who might want to use his eventual module in an IDE, for autogenerating documentation, or for curious users in the REPL. (If you type in ListFormatLength.WHY, the text “Lengths for list format … and ‘narrow’” will be returned.) For those unaware of declarator POD, you can use either #| to apply a comment to the following symbol declaration (in the example, for the subset and the sub itself), or #= to apply it to the preceeding symbol declaration (most common with attributes). Next, he imported two modules that will be useful. User::Language detects the system language, and he used it to provide sane defaults. Intl::LanguageTag is one of the most fundamental modules in the international ecosystem. While he wouldn’t strictly need it (we’ll see he’ll ultimately only use them in string-like form), it helps to ensure at least a plausible language tag is passed. If you’re wondering what the +@items means, it applies a DWIM logic to the positional arguments. If one does format-list @foo, presumably the list is @foo, and so @items will be set to @foo. On the other hand, if someone does format-list $foo, $bar, $xyz, presumably the list isn’t $foo, but all three items. Since the first item isn’t a Positional, Raku assumes that $foo is just the first item and the remaining positional arguments are the rest of the items. The extra () in LanguageTag() means that it will take either a LanguageTag or anything that can be coerced into one (like a string). Okay, so with that housekeeping stuff out of the way, he got to coding the actual formatting, which is devilishly simple: my $format = cldr{$language}.list-format{$type}{$length}; my ($start, $middle, $end, $two) = $format<start middle end two>; if @items > 2 { ... } elsif @items == 2 { @items[0] ~ $two ~ @items[1] } elsif @items == 1 { @items.head } else { '' } He paused here to check and see if stuff would work. So he ran his script and added in the following tests: # output format-list <>, :language<en>; # '' format-list <a>, :language<en>; # 'a' format-list <a b>, :language<en>; # 'a{0} and {1}b' While the simplest two cases were easy, the first one to use CLDR data didn’t work quite as expected. The elf realized he’d need to actually replace the {0} and {1} with the item. While technically he should use subst or similar, after going through the CLDR, he realized that all of them begin with {0} and end with {1}. So he cheated and changed the initial assignment line to my $format = cldr{$language}.list-format{$type}{$length}; my ($start, $middle, $end, $two) = $format<start middle end two>.map: *.substr(3, *-3); Now he his two-item function worked well. For the three-or-more condition though, he had to think a bit harder how to combine things. There are actually quite a few different ways to do it! The simplest way for him was to take the first item, then the $start combining text, then join the second through penutimate, and then finish off with the $end and final item: if @items > 2 { ~ $items[0] ~ $start ~ $items[1..*-2].join($middle) ~ $end ~ $items[*-1] } elsif @items == 2 { @items[0] ~ $two ~ @items[1] } elsif @items == 1 { @items.head } else { '' } Et voilà! His formatting function was ready for prime-time! # output format-list <>, :language<en>; # '' format-list <a>, :language<en>; # 'a' format-list <a b>, :language<en>; # 'a and b' format-list <a b c>, :language<en>; # 'a, b, and c' format-list <a b c d>, :language<en>; # 'a, b, c, and d' Perfect! Except for one small problem. When they actually started using this, the computer systems melted some of the snow away because it overheated. Every single time they called the function, the CLDR database needed to be queried and the strings would need to be clipped. The elf had to come up with something to be a slight bit more efficient. He searched high and wide for a solution, and eventually found himself in the dangerous lands of Here Be Dragons™, otherwise known in Raku as EVAL. He knew that EVAL could potentially be dangerous, but that for his purposes, he could avoid those pitfalls. What he would do is query CLDR just once, and then produce a compilable code block that would do the simple logic based on the number of items in the list. The string values could probably be hard coded, sparing some variable look ups too. There be dragons here 🐉🦋 EVAL should be used with great caution. All it takes is one errant unescaped string being accepted from an unknown source and your system could be taken. This is why it requires you to affirmatively type use MONKEY-SEE-NO-EVAL in a scope that needs EVAL. However, in situations like this, where we control all inputs going in, things are much safer. In tomorrow’s article, we’ll discuss ways to do this in an even more safer manner, although it adds a small degree of complexity. Back to the regularly scheduled program To begin, the elf imagined his formatting function. sub format-list(+@items) { if @items > 2 { @items[0] ~ $start ~ @items[1..*-2].join($middle) ~ $end ~ @items[*-1] } elsif @items == 2 { @items[0] ~ $two ~ @items[1] } elsif @items == 1 { @items[0] } else { '' } } That was … really simple! But he needed this in a string format. One way to do that would be to just use straight string interpolation, but he decided to use Raku’s equivalent of a heredoc, q:to. For those unfamiliar, in Raku, quotation marks are actually just a form of syntactic sugar to enter into the Q (for quoting) sublanguage. Using quotation marks, you only get a few options: ' ' means no escaping except for \\, and using " " means interpolating blocks and $-sigiled variables. If we manually enter the Q-language (using q or Q), we get a LOT more options. If you’re more interested in those, you can check out Elizabeth Mattijsen’s 2014 Advent Calendar post on the topic. Our little elf decided to use the q:s:to option to enable him to keep his code as is, with the exception of having scalar variables interpolated. (The rest of his code only used positional variables, so he didn’t need to escape!) my $format = cldr{$language}.list-format{$type}{$length}; my ($start, $middle, $end, $two) = $format<start middle end two>; my $code = q:s:to/FORMATCODE/; sub format-list(+@items) { if @items > 2 { @items[0] ~ $start ~ @items[1..*-2].join($middle) ~ $end ~ @items[*-1] } elsif @items == 2 { @items[0] ~ $two ~ @items[1] } elsif @items == 1 { @items[0] } else { '' } } FORMATCODE EVAL $code; The only small catch is that he’d need to get a slightly different version of the text from CLDR. If the text and were placed verbatim where $two is, that block would end up being @items[0] ~ and ~ @items[1] which would cause a compile error. Luckily, Raku has a command here to help out! By using the .raku function, we get a Raku code form for most any object. For instance: # REPL output 'abc'.raku # "abc" "abc".raku # "abc" <a b c>.raku # ("a", "b", "c") So he just changed his initial assignment line to chain one more method (.raku): my ($start, $middle, $end, $two) = $format<start middle end two>.map: *.substr(3,*-3).raku; Now his code worked. His last step was to find a way to reuse it to benefit from this initial extra work.He made a very rudimentary caching set up (rudimentary because it’s not theoretically threadsafe, but even in this case, since values are only added, and will be identically produced, there’s not a huge problem). This is what he came up with (declarator pod and type information removed): sub format-list (+@items, :$language 'en', :$type = 'and', :$length = 'standard') { state %formatters; my $code = "$language/$type/$length"; # Get a formatter, generating it if it's not been requested before my &formatter = %cache{$code} // %cache{$code} = generate-list-formatter($language, $type, $length); formatter @items; } sub generate-list-formatter($language, $type, $length --> Sub ) { # Get CLDR information my $format = cldr{$language}.list-format{$type}{$length}; my ($start, $middle, $end, $two) = $format<start middle end two>.map: *.substr(3,*-3).raku; # Generate code my $code = q:s:to/FORMATCODE/; sub format-list(+@items) { if @items > 2 { @items[0] ~ $start ~ @items[1..*-2].join($middle) ~ $end ~ @items[*-1] } elsif @items == 2 { @items[0] ~ $two ~ @items[1] } elsif @items == 1 { @items[0] } else { '' } } FORMATCODE # compile and return use MONKEY-SEE-NO-EVAL; EVAL $code; } And there he was! His function was all finished. He wrapped it up into a module and sent it off to the other elves for testing: format-list <apples bananas kiwis>, :language<en>; # apples, bananas, and kiwis format-list <apples bananas>, :language<en>, :type<or>; # apples or bananas format-list <manzanas plátanos>, :language<es>; # manzanas y plátanos format-list <انارها زردآلو تاریخ>, :language<fa>; # انارها، زردآلو، و تاریخ Hooray! Shortly thereafter, though, another elf took up his work and decided to go even crazier! Stay tuned for more of the antics from Santa’s elves how they took his lists to another level. Day 21: Raku and I: Journey begin … It has been ages since I last blogged about Raku. The only time, I have blogged about when I took part in The Weekly Challenge. But then this also changed recently as I finally found time to contribute to the weekly fun challenges but no blogging still. I would say it is all about my mental state, since I have so much to talk about. Recently I was approached by a very dear friend and senior member of Raku Community if I am interested in contributing to the Raku Advent Calendar 2022. So as you guessed it rightly so, I have a compelling reason to get back to blogging. But then you know …. I always have too many things on my plate, so getting done something new is always tricky. However I had made up my mind, no matter what I would give it my best shot. Here I am … So what am I going to talk about then? Those who know me personally, are aware that I am a Perl guy by nature. Having said, I started playing with other languages recently thanks to the vibrant group of Team PWC. In this blog post, I would like to talk about some of my contributions to the weekly challenge in the new found love Raku language. 1: Prime Sum You are given a number $N. Write a script to find the minimum number of prime numbers required, whose summation gives you $N. With the power of Raku built-in features, it is a piece of cake, if you know what you are doing. For me, the official Raku Documentation is the answer to all my questions. And if I can’t find what I am looking for then I ask my friends on various social platform. 9 out of 10 times, I get the answer instantly. So in this case, all the hard work is done by is-prime. I am a big fan of method chaining as you can see below. Just for the sake of the reader, I go through the list 2..$sum and grep everything that is-prime. Isn’t beautiful? For me, it is. sub find-prime-upto(Int $sum) { return (2..$sum).grep: { .is-prime }; } Now with the handy subroutine ready, we are read to solve the task as below: For my Perl friends new to Raku, the only thing might trouble you is the use of [+], right? It is the Reduction Operator [] that works on lists of values. sub prime-sum(Int $sum) { my @prime = find-prime-upto($sum); my @prime-sum = Empty; for 1..$sum -> $i { for @prime.combinations: $i -> $j { my $_sum = [+] $j; @prime-sum.push: $j if $_sum == $sum; } } return @prime-sum; } Now glue together everything as below: use v6.d; sub MAIN(Int $SUM where $SUM > 0) { prime-sum($SUM).join("\n").say; } Still not done yet as unit test is nice to have. use Test; is-deeply prime-sum(6).<>, [], "prime sum = 6"; is-deeply prime-sum(9).<>, [(2, 7),], "prime sum = 9"; is-deeply prime-sum(12).<>, [(5, 7), (2, 3, 7)], "prime sum = 12"; done-testing; 2: Fibonacci Sum Write a script to find out all possible combination of Fibonacci Numbers required to get $N on addition. You are NOT allowed to repeat a number. Print 0 if none found. You may find the solution below somewhat similar to the above work but there is something new for Perl fans. In Perl we can get the last element of a list $list[-1] but in Raku it is slightly different as you see below. One more thing, if you look at closely the parameter checks done in the signature itself which we don’t have in Perl. Raku rocks !!! sub fibonacci-series-upto(Int $num where $num > 0) { my @fibonacci = (1, 2); while @fibonacci.[*-1] + @fibonacci.[*-2] <= $num { @fibonacci.push: @fibonacci.[*-1] + @fibonacci.[*-2]; } return @fibonacci; } Now we are ready to solve the task as below: Did you notice something special here? Yes, .combinations. Again all built-in, no need to import any library. It generates all possible combinations of given size. sub fibonacci-sum(Int $sum where $sum > 0) { my @fibonacci = fibonacci-series-upto($sum); my @fibonacci_sum = Empty; for 1 .. $sum -> $i { last if $i > @fibonacci.elems; for @fibonacci.combinations: $i -> $comb { my $_sum = [+] $comb; @fibonacci_sum.push: $comb if $_sum == $sum; } } return |@fibonacci_sum; } Final application. use v6.d; sub MAIN(Int :$N where $N > 0) { fibonacci-sum($N).join("\n").say; } Time for some unit test too. use Test; is-deeply fibonacci-sum(6), ((1,5), (1,2,3)), "fibonacci sum = 6"; is-deeply fibonacci-sum(9), ((1,8), (1,3,5)), "fibonacci sum = 9"; done-testing; 3: Count Set Bits You are given a positive number $N. Write a script to count the total numbrer of set bits of the binary representations of all numbers from 1 to $N and return $total_count_set_bit % 1000000007. For this task, Raku has most of the funtions built-in, so nothing to be invented. As you see, it is one-liner, (1..$n).map( -> $i { $c += [+] $i.base(2).comb; }); where all the work is done. .map() works same as in Perl. In this case each element gets assigned to $i. Further on $i gets converted to base 2 i.e. binary form then finally split into individual digits using .comb. How can you not fall in love with Raku? sub count-set-bits(Int $n) { my $c = 0; (1..$n).map( -> $i { $c += [+] $i.base(2).comb; }); return $c % 1000000007; } Unit test to go with it. use Test; is count-set-bits(4), 5, "testing example 1"; is count-set-bits(3), 4, "testing example 2"; done-testing; 4: Smallest Positive Number You are given unsorted list of integers @N. Write a script to find out the smallest positive number missing. This task introduced me something new that I wasn’t aware of earlier. I always wanted to put check on the elements of input list. In this task, I am checking every elements in the given input list is integer. Also the return value is of type integer too. All these done with one line @n where .all ~~ Int --> Int. This is the power of Raku we can have the power in our script. Also to sort a list, just use .sort together with .grep makes it very powerful. The .elems gives me the total number of elements in the list. sub smallest-positive-number(@n where .all ~~ Int --> Int) { my @positive-numbers = @n.sort.grep: { $_ > 0 }; return 1 unless @positive-numbers.elems; my Int $i = 0; (1 .. @positive-numbers.tail).map: -> $n { return $n if $n < @positive-numbers[$i++] }; return [email protected]; } Final application looks like below. Did you see something new? Well, it shows how to set the default parameter values. use v6.d; sub MAIN(:@N where .all ~~ Int = (2, 3, 7, 6, 8, -1, -10, 15)) { say smallest-positive-number(@N); } Time for unit test again. use Test; is smallest-positive-number((5, 2, -2, 0)), 1, "testing (5, 2, -2, 0)"; is smallest-positive-number((1, 8, -1)), 2, "testing (1, 8, -1)"; is smallest-positive-number((2, 0, -1)), 1, "testing (2, 0, -1)"; done-testing; CONCLUSION Learning Raku is an ongoing journey and I am loving it. I haven’t shared everything to be honest. If you are interested then you can checkout the rest in my collections. Enjoy the break and stay safe. Day 20: Sigils are an underappreciated programming technology Sigils – those odd, non-alphabetic prefix characters that many programmers associate with Bash scripting; the $ in echo $USER – have a bit of a bad reputation. Some programmers view them as “old fashioned”, perhaps because sigils are used in several languages that first gained popularity last millennium (e.g. BASIC, Bash, Perl, and PHP). Other programmers just view sigils as rather pointless, as “just a way of encoding type information” in variable names – basically a glorified version of systems Hungarian notation (which isn’t even the good kind of Hungarian notation). Maybe sigils served a purpose in the bad old days, these critics say, but modern IDEs and editors give us all the type information we could want, and these tools made sigils obsolete. Now that we have VS Code, we don’t have any reason to take the risk that someone might use sigils to write code that bears a suspicious resemblance to line noise, or perhaps to an extremely angry comic strip character. A cropped panel of a newspaper comic showing one character's head. A speech bubble from that head has the text 'Awww… ?%$X☹#©!!!' This is a family-friendly post, I swear! Or so they say. But I disagree – as do many of the hackers whose perspectives and insights I value most. This post represents my attempt to convince you that sigils are a powerful tool for writing clear, expressive code. No, strike that, I’ll go further – this post is my argument that sigils are a powerful tool for clear communication in general; sigils being useful for programming is just an application of the more general rule. To investigate this claim, we’ll start with three non-programming situations where sigils let us communicate more clearly and expressively. Second, we’ll use these examples to dig into how sigils work: Where does their expressive power come from? And what makes particular sigils good or bad at their job? Once we’ve wrestled with sigils in general, we’ll turn to the specific case of programming-language sigils. We’ll investigate whether the general power of sigils carries over to the project of writing clear, expressive code, where our goal is to express ourselves in ways that our computer and our readers can both understand. We’ll also consider the unique challenges – and extra powers – that are relevant to programming-language sigils. And we’ll examine some sigils in action, to judge how helpful they really are. By the time you reach the end of this post, I believe that you’ll have a better understanding of how sigils work, a new mental tool to apply to your communication (programming and non-programming), and – just possibly – a bit more appreciation for the languages that seem like they’re swearing at you. Three non-programming sigils that aid clarity 1. Chatting about sigils If you’ve ever talked with programmers online, you’ve probably seen someone say something like We had that problem at $day_job: our code was a mess, but everyone thought $framework would magically fix it. I don’t mean that you’ve seen someone express that sentiment (though, let’s face it, you probably have). Instead, I mean that you’ve probably seen someone use terms like $day_job or $framework. And, even if you haven’t, you can probably tell what they mean. In both cases, the $ sigil marks a placeholder word – in $day_job, probably because the speaker doesn’t want to reveal where they work; in $framework, to avoid turning the conversation into a debate about the merits of a particular framework. Of course, neither sigil is indispensable; you could reword the sentence to use phrases like “at my day job” or “a new framework”. So using a sigil saves us a couple of words, which you might view as a trivial benefit. But, in the context of synchronous communication, it really isn’t trivial – anything that reduces the time that an entire team spends looking a $boss is typing... message (see what I did there?) is a clear win. The more important benefit of the sigil is increased clarity: “a new framework” communicates that the speaker is talking about some indefinite framework, no more. If you want to know which framework, it’d be natural to ask. In contrast, $framework communicates that the speaker has consciously decided to omit the name of the framework. Someone might still ask, but they’d be aware that they’re asking for a detail that was deliberately withheld. 2. Now that you mention it… And, of course, if sigils can communicate something to humans, they can also communicate something to computers. Which brings us to the way that everyone uses sigils these days (even if they don’t know the term): mentions and/or hashtags xkcd comic 1306. The comic depicts a graph with 'time' on the horizontal axis and 'odds that a word I type will start with some weird symbol' on the vertical axis. A line oscillates up and down, similar to a sine wave, with three peaks. The peaks are labeled '$QBASIC', '$Bash @$Perl', and '+Google @twitter #hashtags'. Every time a programmer @s someone on GitHub – for that matter, every time someone describes themselves as #blessed or tags a post as #nofilter – they’re using a sigil. Maybe you have a reaction to that? While we’re on the subject of hashtags and @mentions, let’s briefly digress to something that isn’t technically a sigil but that’s closely related: reactions – those emoji-style responses to SMS messages, GitHub issues, and Slack chats. Love them or hate them, I’m sure you’ve seen them. They’re not sigils (they’re used alone, not as a prefix to a word). But they’re worth talking about here because – like sigils – they take advantage of an inherent element of human nature: we’re visual, and think in symbols. GitHub could trivially replace all of their reactions with words. In fact, they list the word equivalent of each symbol: 👍 ⇒ +1; 👎 ⇒ -1; 😀 ⇒ smile; 😕 ⇒ confused;  ⇒ heart; and 🎉 ⇒ tada. So why does GitHub use symbols instead of the equivalent words? Because when you react to project milestone with 🎉, that’s a better way to communicate excitement/congratulations than using words – and much better than saying “tada”. Similarly, symbols like 👍 and 👎 are direct ways to communicate approval or disapproval to someone who understands the symbol. Of course, some cultures ascribe a very different meaning to a thumbs up gesture; I’m not at all claiming that 👍 magically avoids ambiguity. But if someone does understand 👍, then they understand it directly as a single symbol. They’re not translating 👍 into the words “I approve and/or agree” any more than a math-literate person translates “a ÷ (b × c)” into the words “the quotient of ‘a’ divided by the product of ‘b’ and ‘c’”. In both cases, they’re reasoning with the symbols directly; no translation needed. (See also the APL-family programming languages, which take the insight about the power of symbolic reasoning to its (il)logical extreme.) 3. In email, tags > folders – but sigils are my secret weapon So far, we’ve discussed two non-programming examples where you’ve probably already encountered the power of sigils and symbolic communication. For our third example, lets look at a context that won’t be as familiar to you: the system I use to manage my email inbox. For the past decade, I essentially didn’t organize my emails. Instead, I followed the recommendation of a 2011 study of email re-finding: dump everything into a single Archive and get better at searching. Maybe that was correct at the time or maybe I just wanted an excuse to hit the Archive All button. But by last year, that system had clearly stopped working – between my different projects, committees, mailing lists, and patchlists, I simply get too much relevant email to be able to effectively search a single Archive. So I turned to Thunderbird filters. But unlike many people, I’m not using filters to make emails skip my inbox; I like to see the stream of incoming messages. Instead, I use filters to programmatically apply labels to incoming emails (e.g., emails from/to the Raku Steering Council are labeled “RSC”). And when I archive emails, other filters move the email to the correct folder based on their labels. But this left me with a decision: should my labels have folder semantics (each email is in exactly one folder) or tag semantics (emails can have any number of tags, including zero)? The issue is a fairly contentious one – it’s been debated for years, but that post still generated 140+ comments of debate. The merits of the two approaches aren’t relevant here; I’ll just say that I eventually decided to use some of both. Specifically, I decided to give four labels folder semantics: Work, Life, List, and Bulk. Every email in my inbox should be automatically assigned exactly one of these labels – if it has more or less than one, something has gone wrong and I need to fix my filtering rules. And when an email is archived, it should be moved to a folder that corresponds to one of these folder-labels. But every label other than those four gets tag semantics: they’re optional, and emails can have any number of these tag-labels. Examples include Raku, RSC, TPRC, Family, Rust, conference, guix, and blogs. So far, so good – but also irrelevant to sigils. How do sigils come into this picture? Well, I wound up with two different types of labels (folder-labels and tag-labels), each with different semantics. Further, I need to be able to quickly distinguish between the two types of labels so that I can notice emails that don’t have exactly one folder-label. This is a job for sigils. I added the sigil to my folder-semantic labels (⋄Work, ⋄Life, ⋄List, and ⋄Bulk) and the sigil to my tag-semantic labels (e.g., •Raku or •Family). Now, if I see an email that’s labeled ⋄Work, •Raku, •rainbow-butterfly, •RSC, I can instantly see that it has just one folder-semantic label. But if I saw one with •Family, •Parents, •conference, I’d know that it was missing its folder-semantic label. Why do the sigils in ⋄Work and •Raku matter? Using ⋄Work and •Raku solves a problem – one without similarly good, non-sigil solutions. When we were talking about $day_job, the sigil was helpful but not essential: saying “my day job” would work maybe 90% as well. But here, there isn’t a similar 90% solution. We can’t leave the folder-labels and tag-labels undifferentiated; that’s a recipe for confusion. What about the use-your-words approach we employed with $day_job? The equivalent substitution here would be to use labels like primary_Work in place of ⋄Work. But even though primary_Work does communicate that label has folder semantics, the substitution costs far more than the 10% we estimated for $day_job. Indeed, primary_Work is a poor substitute for ⋄Work for exactly the same reason that “confused” is a poor substitute for 😕: in both cases, switching from a symbol to text dramatically ups the cognitive load involved and correspondingly slows reading. Put differently, ⋄Work is (mentally) pronounced “work”, not “primary work”. ⋄Work conveys information symbolically, which makes understanding that info easier and faster. In turn, faster understanding means that reading ⋄Work, •Raku, •rainbow-butterfly, •RSC in a glance is practical, but reading primary_Work, secondary_Raku, secondary_rainbow-butterfly, secondary_RSC isn’t. There’s far more than a 10% improvement in the readability of ⋄Work, and that difference is 100% sigils. Learning from non-programming sigils We’ve looked at 3½ examples where sigils helped in non-programming contexts: chat messages, hashtags, and email labels (plus ½ credit for emoji reactions). In each case, using sigils lets us communicate more clearly. But can we generalize from these examples into an explanation for sigils’ power? I think so. Sigils are powerful because they use semantically dense symbols to quickly and easily communicate meaningful, low-context information to the reader. That was a mouthful, so let’s unpack it, one term at a time. 2. Sigils use semantically dense symbols The defining feature of sigils is that they’re symbolic: ⋄Work uses a sigil; primary_Work doesn’t. The symbolic nature of sigils is key to their power: because they encode an entire phrase’s worth of information into a single glyph, they have a much higher semantic density. Put differently, they let you say more, with less. Recognizing value of code that packs a lot of meaning into a small package isn’t a novel insight, of course. And neither is the observation that symbols are extraordinarily good at concise communication. Hillel Wayne made a similar point a couple of years ago: A tweet by @hillelogram with the text 'I think sigils (like the dollar sign) in programming are underrated. We recognize they're bad for readability and you should use more descriptive names, but we also use 'i' as an iterator variable name, so there's something more legible to us about terse names when we can get away with them' Indeed, APL programmers have been practically shouting this insight from the rooftops for over 30 years: Ken Iverson, the designer of APL, opened his famous 1979 Turing Award talk, Notation as a Tool of Thought, with exactly this point: The quantity of meaning compressed into small space by algebraic signs, is another circumstance that facilitates the reasonings we are accustomed to carry on by their aid. But those weren’t Iverson’s words: he was quoting a book published in 1826 by Charles Babbage (the “father of the computer”, if that’s even a meaningful title). And then, just to complete the cycle of quotation – and drive home the point that a focus on semantic density is widespread – Paul Graham quoted Iverson (quoting Babbage) in his 2002 essay Succinctness is Power. I might not value succinctness quite as highly as Graham’s essay did, but it’s hard to deny that sigils’ expressive concision provides quite a bit of power. And, indeed, we can see evidence of that expressive power in one of the non-programming sigils we discussed: The hashtag is so expressive that it’s even starting to make its way into spoken language. 2. Sigils can be used quickly and easily Given the immense power of symbols to create expressive, semantically dense code, why shouldn’t we use them for everything? Should everyone be programming APL? Is there such a thing as code that’s too semantically dense? A page of APL code. The code contains almost entirely symbols and is impossible to read for anyone unfamiliar with APL Why, yes, yes it is. Despite my admiration for APL, I think it gets the balance wrong. I appreciate symbols, but I also like words (I know, you’re shocked, shocked to learn that about the person responsible for the ~2.5k words you’ve just read). So a few words in defense of words: although symbols offer tremendous semantic density, they sacrifice flexibility; symbols are best when they play a supporting role to words. They’re the punctuation, while words are, well, the words. Another downside to fully embracing symbols can be seen in APL’s overwhelming number of symbols. The problem with APL’s symbolic abundance isn’t the learning curve – that takes time, but veteran APLers have long mastered the vocabulary. Instead, the problem with APL’s extremely large symbol vocabulary is that it crowds out user-created vocabulary. This leaves little space for users to grow their language or to solve specific problems with specific languages; that is, it discourages DSLs. And, indeed, some of the best APL programmers aren’t a fan of DSLs. I respect this view but respectfully disagree. So, if we don’t want to fully embrace symbols, APL-style, where should we draw the line with sigils? Well, we want our sigils to be both memorable and quickly recognizable. This will both help new users learn them faster and allow experienced users to read sigils without expending any cognitive effort. Or, put slightly differently, good sigils should be easy to use – but easy in the very specific sense from Rich Hickey’s Simple Made Easy talk. Hickey distinguishes between “simple” (an objective measure of how intertwined/“complected” something is) from “easy” (a subjective measure of how familiar and practiced/“near to hand” something is). Hickey distinguish between simple and easy to argue in favor of simplicity. In general, I agree. But in the specific case of sigils (or symbols more broadly) I think that making them easy – in the “near to hand” sense – is crucially important. Being easy matters because sigils derive much of their power from their ability to communicate to experienced readers almost for free. For example, when I see @codesections, I perceive that I’ve been mentioned without devoting any conscious thought to the @. The @ communicates to me in same way that the capital letters in “the White House” communicates that we’re talking about the U.S. president without me ever thinking “oh, capital W means a proper noun”. But sigils can only get that meaning-for-free effect when the sigils are very near to hand indeed. One way to make sigils easy is to make them visually distinct. The sigils we’ve seen $, #, @, ⋄, and pass this bar. In contrast, using 😀, 😃, and 😄 as sigils would never be easy. Additionally, sigils are easier to use if users read them frequently. So there probably shouldn’t be many sigils and every sigil should be used often. For an example of this done well, consider social media’s use of # and @ – just two sigils, both used daily. The same goes for my and sigils: they’re applied to all my emails, so I use both every day. Finally, sigils will be easier to use if users practice using them, so sigils should be convenient to type. Of course, since “easy” is a subjective question, “convenient to type” depends on the sigil’s users. So and were good sigils for their target user – me – because the compose key lets me type them painlessly. But they’d be the wrong sigil to choose for a user group that finds typing non-ASCII characters difficult. That description probably applies to enough programmers that, at least right now, programming sigils are probably better off sticking to ASCII … even though that is a bit ☹. When sigils follow those rules – they’re visually distinct, few in number, and read and used frequently – they’re able to communicate practically for free. Which is pretty powerful. 3. Sigils must communicate meaningful, low-context information Earlier, we said that sigils have high semantic density because they’re short. But, of course, that implicitly assumes that they convey some useful meaning. If they don’t mean anything, then they really are low-semantic-density characters that just get in the way – that is, line noise. Going back to our starting example, if someone said We had that problem at $day_job: our $code was a $mess, but everyone thought $framework would $magically fix it. …then that’s not using sigils, it’s just being confusing. Moreover, sigils should do more than communicate some useful info; they should communicate that information in a low-context way. By low-context, I mean that the reader should be able to grasp the sigil’s full meaning using purely local reasoning. When a word has a good sigil – like ⋄Work – you can look at the sigiled-word in isolation and fully grasp its meaning. In contrast, consider GitHub’s #-sigiled issues and pull requests (e.g., #1066). I view these as miserable sigils. But the problem isn’t that #1066 fails to communicate useful information: it communicates that 1066 refers to an issue or PR in the current repo (and not, say, a year). And, by cross referencing that number with a list of the issues and PRs, you can learn what it was about. But that info requires cross referencing with data outside the immediate context – that is, data that’s not locally available. Relying on external context tanks the sigil’s usefulness because humans really struggle to remember more than a few things at a time – a fact of which programmers are frequently and forcibly reminded whenever we try to exceed that threshold. So we really don’t want sigils that require the reader to keep additional, non-local context in their short-term memory. A good sigil should avoid that; it’s meaning should be immediately and locally clear. Programming language sigils Sigils for programming languages are a lot like sigils in other contexts. They use a single symbol to say a lot, concisely; to do so, they must convey low-context information that’s easy to understand. But programming differs from other communication contexts in one fundamental respect, and this difference means that programming sigils face unique challenges and opportunities. The key difference between programming and other forms of communication is that code always has two audiences: human readers and the computer. Some programmers aim to write for people to read, and only incidentally for the computer; others might be happier writing in unadorned hexadecimal numbers, with no human readers at all. Nevertheless, as Donald Knuth observed, programs must always be written both for computers to execute and for humans to understand. This duality applies just as strongly to sigils: when an author puts a sigil in their code, they’re simultaneously communicating to the compiler and to readers of that code. And since the human readers depend on their ability to accurately model the computer, the semantics given to sigils must at all costs avoid giving inconsistent messages to those two audiences. But there’s a subtle-but-crucial nuance: often, the information that a sigil is communicating to the reader is about information that it conveyed to the computer. As an analogy, think back to the hashtag and @mention sigils. Using # tells readers that the following word is a hashtag – but the following word only is a hashtag because the # told computers to treat it that way. The # didn’t communicate inconsistently; it communicated something to the computer that caused a change and simultaneously communicated that change to readers. This same pattern plays out in many programming sigils and is a key source of their expressive power. The three-way conversation between the author, computer, and reader works in the other direction as well: just as the code author is communicating with both reader and computer, the computer can communicate with author and reader. (Sadly, this symmetry doesn’t extend to the reader communicating back in time to the author, though that would greatly simplify software maintenance.) One consequence of the computer → author communication is that the computer intervene if the author tries to use an invalid sigil. Thus, even the weakest version of sigils wouldn’t reduce down to Hungarian notation – at worst, it would be compiler verified Hungarian notation. But encoding a variable’s type isn’t a good use for sigils, anyway, due to the computer → reader communication. Specifically, the computer can communicate type info to the reader, and the sigil-skeptics are correct that IDEs/editors have gotten pretty good at doing so. But what those skeptics seem to miss is that sigils can communicate far more meaningful sorts of information anyway. Using a sigil just to denote the type would be a waste of a perfectly good sigil, so IDE-supplied type info is entirely irrelephant. If a sigil shouldn’t convey type info, what should a sigil communicate? Well, that question is unanswerable in the abstract – it depends on the needs of the particular programming language that supplies the sigil. I’ll discuss the language I’m most familiar with, Raku – which you must have expected would eventually make an appearance on this Raku Advent Calendar post. How Raku uses sigils This isn’t the time or place to introduce you to Raku, but in case you’re not familiar with it, I’ll just say that you should be able to follow everything below without any Raku-specific knowledge. Well, and that Raku is, by far, my favorite programming language; I believe it’s pretty much the ideal language for writing free software – in large part because Raku provides the expressive power needed for a small team to keep up with a much larger big-tech team. And some of that power comes from Raku’s sigils. 1. Raku without sigils Before we look at Raku’s sigils, I’ll mention that you can write Raku without any sigils at all; they’re a feature, not a requirement. For example, the JavaScript code const foo = 42 can be written in Raku as my \foo = 42. Rakoons choose to use these sigils because they – we – believe that they make for clearer code. Let’s take a look at why. 2. Raku with sigils Raku provides sigils that let you communicate (to the computer and the reader) what interface a variable supports – that is, how you can use that variable. Raku has four sigils; here’s what each communicates: • @ says “Use me with an array-like interface” • % says “Use me with a hash-like interface” • & says “Use me with a function-like interface” • $ says “I won’t tell you what interface you can use, but treat me as a single item” What does it mean to treat a variable as a single entity? Well, imagine I’ve got a grocery list with five ingredients on it. Saying that I’ve got one thing (a list) is true, but saying that I’ve got five things (the foods) is also true from a certain point of view. Using $ versus @ or % expresses this difference in Raku. Thus, if I use @grocery-list with a for loop, the body of the loop will be executed five times. But if I use $grocery-list, the loop will get executed just once (with the list as its argument). This matters for more than just for loops; Raku has many places where it can operate on either an entire collection or on each item in a collection (this often comes up with variadic functions). The sigil tells Raku how to behave in those cases: it treats $grocery-list as one item, but operates on each food in @grocery-list. We can temporarily opt into the other behavior if needed, but the sigil provides a reminder to keep us and Raku on the same page. Bonus powers and defaults Raku’s sigils have a couple of extra powers that aren’t directly part of their interface – they’re just bonus perks. The first of these perks is interpolation. In Raku, every sigiled variable is eligible for interpolation in the right sort of string. The exact details depend on the sigil and aren’t worth getting into here (mostly based on how the characters are typically used in strings – it’s kind of nice that “[email protected]’t interpolate by default). You can selectively enable/disable interpolation for specific sigials or temporarily enable interpolation in strings that normally don’t allow it with \qq[ ] (like JavaScript’s ${ }). Between this, its Unicode support, and rationalized regex DSL system, I’m prepared to confidently claim that Raku’s text manipulation facilities significantly outdo any language I’ve tried. The second perk is a bit of syntax sugar that only applies to &-sigiled variables but that’s responsible for a fair bit of Raku’s distinctive look. We already said that you can invoke &-sigiled variables with syntax like &foo(). But due to sugar associated with &, you can also invoke them with by omitting both the & and the parentheses. Thus, in Raku code, you typically only see a & when someone is doing something with a function other than calling it (such as passing it to a higher order function). I’ve previously blogged about how you can write Raku with extra parens to give it a syntax and semantics surprisingly close to lisp’s, so it’s only fair to point out that, thanks to this & perk, it’s possible to write Raku with basically no parentheses at all. Finally, the @ and % sigils provide a default type. I’ve mentioned a few times that @ does not mean that a variable is an Array, just that it provides an array-like interface. New Rakoons sometimes get confused about this, maybe because many @-sigiled variables you see (especially starting out) happen to be Arrays, and many of the %-sigiled variables happen to be Hashs. That’s not too surprising; ordered-mutable-list and key-value-hashmap are both useful, general abstractions – there’s a reason JavaScript was able to survive so many years with just hashes and objects. To support the common use case of @-sigiled variables being Arrays and %-sigiled variables being Hashes, Raku provides them as default types when you declare a variable with @ or %. That is, when you assign into an uninitialized @-sigiled variable, Raku provides a default Array (and the same for % and Hash). So we can write my @a = 1, 2 to create an Array with 1 and 2; or we can write my %h = k => "v" to create a Hash. But this is just a default – you’re entirely free to bind any type that provides the correct interface. At this point, we’ve covered powers that the four Raku sigils provide. Here’s a table with a summary before we move on: sigil @ % & $ Interface Positional Associative Callable Scalar Default Array Hash (none) (none) Iteration One at a time; fixed order One pair at a time; random order Entire container Entire container Guarantee Positional indexing Associative indexing invokable (none) 3. Raku sigils in practice To really judge their power, let’s look at Raku’s sigils in a more practical setting. Imagine that we’re working with FiveThirtyEight’s World Cup predictions – specifically, their pre-game prediction of the final: A depiction of Five Thirty Eight's prediction for the World Cup final. It assigns Argentina a 53% chance of victory and France a 47% chance Where would we start? Our data consists of two key–value pairs. Let’s represent them with two of Raku’s Pairs: Argentina => .53 and France => .47. Next, we’ll want to store these pairs into a variable. But what sigil should we use? Well, we could use no sigil at all, but that would sacrifice all the perks that come with sigils in Raku. No thanks. Or we could use a &-sigiled variable by writing my &f = {(Argentina => .53, France => .47)}. But this would be a pretty odd – ok, bizarre – choice. By using & we get a function that takes no arguments and, when invoked, returns the two pairs – which seems strictly inferior to working with the two pairs directly. I mention the possibility of using & only to emphasize that it’s our choice: we choose the sigil (and thus the semantics). With those two out of the way, let’s consider the three viable options: @, %, and $. Using the @ sigil creates an Array containing two Pairs; we could do that with my @win‑predictions = Argentina => .53, France => .47. This keeps the pairs in order, so it might be a good choice if we care about order (maybe we’re planning to display teams ranked by win chance?). The @-sigiled Array also lets us iterate through the teams one at a time. Alternately, using the % sigil (my %win‑predictions = Argentina => .53, France => .47) gives us a Hash with team names for our keys and predicted odds as our values, which lets us access a team’s odds by providing their name: e.g., %win‑predictions<France> returns .53. This might be the way to go if we’ll need to access an arbitrary team’s odds of winning (maybe we’re building a search box where you can enter a name to see that team’s predicted odds). The %-sigiled Hash still lets us iterate through the teams one at a time but this time in a random order. What about the $ sigil? Well, we actually have a few options. $ tells Raku (and the reader) that we’re treating the predictions as a single item (not a collection). This means that my $win‑predictions = Argentina => .53, France => .47 isn’t the syntax we want – since $-sigiled variables are always a single item, that would assign the pair Argentina => .53 to $win‑predictions and discard the second pair. (If we did this, Raku helpfully warns that we might not have meant to.) To store both Pairs in $win-predictions, we’ll need to group them in some way. For example, we could group them with square brackets, which creates an Array. Or we could group them with curly brackets, which creates a. These two options would look like my $win‑predictions = [Argentina => .53, France => .47] and my $win‑predictions = {Argentina => .53, France => .47}, respectively. But hold on, if we end up storing an Array or Hash in our $-sigiled variable, how is using $ different from using the @ or % sigils? It’s different that the $ communicates to Raku and to readers that we’re treating the entire Array/Hash as a single item – and that Raku should too. This has a few effects, most notably that iterating over a $-sigiled Array/Hash will take the entire container at once, rather than one Pair at a time. This “item” semantics might best fit our mental model if we’re thinking of “matches” as a single entity (instead of collection of teams–odds pairs). Looking at them this way makes a lot of sense – after all, the statement “France had a 47% chance to win” doesn’t mean much without knowing that we’re talking about their match against Argentina. If we do use a $-sigiled variable, then we’ll still need to decide between using an array or a hash. The considerations here are basically the same as in our choice between @ and %: do we care more about preserving order or about indexing by team name? In sum, we can pick between three sigils. Choosing @ communicates that we’re using an ordered array of Pairs; choosing % communicates that we’re focused on the key–value relationship; and choosing $ communicates that we’re treating the match as a single item. And, crucially, our choice of sigil communicates that entirely locally: every time a reader (which could be us in a few weeks!) sees win-predictions with a certain sigil, it tells the reader whether they’re dealing with an ordered collection, a collection that associates keys with values, or a conceptually single item. There’s never a need to scroll up to where the variable was defined – and, as the functional programmers keep reminding us, it’s far easier to understand code when we can do so without relying on any remote context. Finally, it’s important to note that the information we get from the sigil is not the variable’s type: my @pos = 1,2, and my $scalar = [1,2] both create Arrays and if you (or your IDE) ask @pos or $scalar for their type, they’ll both honestly report that they’re Arrays. And, as we discussed, @– and %-sigiled variables aren’t guaranteed to be Arrays and Hashes. The questions “what type is this variable” and “what interface does this variable provide” are orthogonal: answering one doesn’t answer the other. So Raku’s sigils definitely aren’t “just a way of encoding type information that could be displayed by an IDE” – they’re a way to create and document a variable’s interface. At least in my book, that’s quite a bit of information for a single character to communicate. I’m more than happy to conclude that Raku’s sigils communicate meaningful, low-context information. All in all, I believe we’ve seen that Raku’s sigils can be pretty powerful – and that’s without even mentioning Raku’s nine “twigils” (secondary sigils)! Conclusion and next steps If you’ve made it this far, thanks – you’re clearly my kind of weird! …Unless you skipped down to the conclusion in the naive hope that I’d offer some sort of cogent tl;dr ಠ_ಠ In this post, we’ve talked about sigils generally and seen how – even in non-programming contexts – they can be a powerful tool to concisely communicate with your reader. We also looked at how Raku uses sigils and saw that sigils let Rakoons communicate what interface our variables have both to Raku and to readers of our code. I hope that I’ve convinced you that sigils, used thoughtfully, can make communication easier and code more readable. In my ideal world, I also hope to have tempted you into taking a closer look at Raku. Realistically, though, this post wasn’t really optimized for doing so – as cool as sigils are, they might not make my list of Top 10 Raku Features (number 7 will shock you!). But even if they’re not my favorite Raku feature, I wanted to talk about sigils – and specifically Raku’s sigils – because they are a strength of the language. And yet, people sometimes mention sigils as the reason that they or others avoid a language – which strikes me as entirely backwards. So, whether it’s Raku, Bash, Perl, PHP, or any of the other languages that use sigils, I hope you’ll never again pass on a language because it uses a few more $s than some others. Sigils can be a powerful tool. According to Wikipedia, the word “sigil” derives from the Latin for “little sign with magical power”. And, yeah, “magical power” seems about right to me. Just, you know, not top-ten-level magic. Day 19: A few modules to ease working with databases in Raku applications There’s no single big Raku application I work on regularly at the moment, but there’s plenty of smaller ones that I need to do a bit of work on now and then. Nearly all of them involve using a database for persistence; I tend to reach for Postgres. This year I put together a few Raku modules to ease my work with databases on these projects. All of them are available for installation via zef; maybe some will make nice Christmas gifts for others using Raku and databases together. Just let me develop this thing locally! How should I run an application I’m developing on my local machine when it uses a database? I could use the system Postgres, creating a database, user, and so forth. Of course, this isn’t a task I do every day, and not even every month, so I have to go and look up the syntax every time. Then what if the Postgres version in the deployment environment is different from the one on my system? Not likely to be an issue often in such a mature product, but maybe some day it’d be a tripping hazard. Thankfully, container technology has made it super easy to spin up most things at the version of one’s choice. Postgres containers are readily available. That still leaves a little scripting and plumbing to do to get the container up, create the database and user required, and have the environment variables injected before running the application. I’ve done it a bunch of times, in a bunch of ways. It’s easy, but boring. What’s less boring? Writing a Raku module to take away the repetitive work, of course! Thus Dev::ContainerizedService. Now I need only write a devenv.raku like this: #!/usr/bin/env raku use Dev::ContainerizedService; service 'postgres', :tag<13.0>, -> (:$conninfo, *%) { env 'DB_CONN_INFO', $conninfo; } This suffices to have a Postgres 13.0 docker container pulled (if needed), then started up, with a database and user created. These are then injected into the environment for the application; in this case, my application was expected a Postgres connection string in %*ENV<DB_CONN_INFO>. I can then run my application via this script: ./devenv.raku run raku -I. service.raku The Postgres instance is run on the host network, and a free port is chosen, which is just great for when I’ve got a couple of different projects that I’m working on at once. If I’m using cro and the development runner that restarts the service on changes, that’d be: ./devenv.raku run cro run By default, it creates a throwaway database each time I run it. If I want to have the database persisted between runs, I need to add a project name and specify that it should store the data persistently: #!/usr/bin/env raku use Dev::ContainerizedService; project 'my-app'; store; service 'postgres', :tag<13.0>, -> (:$conninfo, *%) { env 'DB_CONN_INFO', $conninfo; } Sometimes, for debugging reasons, I’ll want a psql shell to poke at the database. That’s available with: ./devenv.raku tool postgres client See the docs to learn about some of the more advanced features, and how to add support for further services (I’ve done Postgres and Redis, since those are what I had immediate use for). I want integration tests that hit the database! Unit tests that stub out the database, perhaps using something like Test::Mock, are all well and good, but do I really think my data access code is somehow going to be perfect? Of course not; it needs testing too. Of course, that means some plumbing. Setting up a test database. Doing the same in the CI environment. I’ve done it a dozen times before. It’s easy. It’s boring. Why can’t I have a Raku module to take away the tedium? Well, I could if I wrote it. Thus Test::ContainerizedService, the testy descendant of Dev::ContainerizedService. It’s actually a small wrapper around the core of Dev::ContainerizedService, meaning that one only has to add support for a database or queue in Dev::ContainerizedService, and then it’s available in Test::ContainerizedService too. Using it looks like this: use Test; use Test::ContainerizedService; use DB::Pg; # Either receive a formed connection string: test-service 'postgres', :tag<14.4> -> (:$conninfo, *%) { my $pg = DB::Pg.new(:$conninfo); # And now there's a connection to a throwaway test database } In a nutshell, wrap the tests up in a test-service block, which does what is needed to get a Postgres container up and running, and then passes in the connection information. If docker is not available, the tests are skipped instead. What about migrations? The two previous gifts are ready for unwrapping this Christmas. I’ve also been working on one that is probably only unwrappable for the adventurous at this point: DB::Migration::Declare. It’s not the first Raku effort at database migrations – that is, the idea of having an ordered, append-only list of database change that together bring the database schema up to the current state. The Red ORM has some work in that direction, for those using Red. There’s also a module where one writes the SQL DDL up and down steps, and it applies them. I’ve used it, it works. But inspired by the benefits and shortcomings of Knex.js migrations, which I’ve been using quite a bit this year at a customer, I decided to set about building something sort of similar in Raku. The idea is relatively simple: use a Raku DSL for specifying the migrations, and have the SQL to put the changes into effect generated. Supposing we want a database table to track the tallest skyscrapers, we could write this: use DB::Migration::Declare; migration 'Setup', { create-table 'skyscrapers', { add-column 'id', integer(), :increments, :primary; add-column 'name', text(), :!null, :unique; add-column 'height', integer(), :!null; } } Assuming it’s in a file migrations.raku alongside the application entrypoint script, we could add this code: use DB::Migration::Declare::Applicator; use DB::Migration::Declare::Database::Postgres; use DB::Pg; my $conn = $pg.new(:conninfo(%*ENV<DB_CONN_INFO>)); my $applicator = DB::Migration::Declare::Applicator.new: schema-id => 'my-project', source => $*PROGRAM.parent.add('migrations.raku'), database => DB::Migration::Declare::Database::Postgres.new, connection => $conn; my $status = $applicator.to-latest; note "Applied $status.migrations.elems() migration(s)"; At application startup, it will check if the migration we wrote has been applied to the database yet, and if not, translated it to SQL and apply it. If a little later we realized that we also wanted to know what country each skyscraper was in, we could write a second migration after this first one: migration 'Add countries', { create-table 'countries', { add-column 'id', integer(), :increments, :primary; add-column 'name', varchar(255), :!null, :unique; } alter-table 'skyscrapers',{ add-column 'country', integer(); foreign-key table => 'countries', from => 'country', to => 'id'; } } On starting up our application, it would detect that the latest migration had not yet been applied and do so. DB::Migration::Declare doesn’t just produce schema change SQL from Raku code, however. It also maintains a model of the current state of the database. Thus if my previous migration had a typo like this: alter-table 'skyskrapers',{ add-column 'country', integer(); foreign-key table => 'countries', from => 'country', to => 'id'; } It would detect it and let me know, before it even gets so far as trying to build the SQL: Migration at migrations.raku:11 has problems: Cannot alter non-existent table 'skyskrapers' It detects a range of such mistakes – not only typos, but also semantic issues such as trying to drop a table that was already dropped, or adding a duplicate primary key. Development environment, test environment, and migrations Not quite gold, frankincense, and myrrh, but my hope is that somebody else might find these useful too. Cheers! Day 18: Something else Santa was absent-mindedly going through the Rakudo commits of the past weeks, after hearing about the new 2022.12 release of the Rakudo compiler. And noticed that there were no commits after that release anymore. Had all the elves been too busy doing other stuff in the Holiday Season, he wondered. But, in other years, the Raku core elves had always been very busy in December. He recalled December 2015 with a bit of a smile on his face: my, my, had the elves been busy then! A little worried, he asked Lizzybel to come in again. “So, why is nobody working on Rakudo anymore”, he asked. “Ah, that!”, Lizzybel said. “Not to worry, we changed the default branch of Rakudo to ‘main'”, she said. “Why would you do that?”, Santa asked, showing a bit of grumpiness. “Was the old default branch not good enough?”. Lizzybel feared a bit of a long discussion (again), and said: “It’s the new default on Github, so us Raku core elves thought it would be a good idea to follow that, as many tools now assume ‘main’ as the default branch”. “Hmmrph”, said Santa, while he switched to the ‘main’ branch’. “Wow!, more than 780 commits since the 2022.12 release, how is that possible?”, he exclamed. “Don’t the elves have nothing better to do in this time of the year?” he said, while raising his voice a bit. Lizzybel noticed his cheeks turning a little redder than usual. “Ah that!”, said Lizzybel again. RakuAST And she continued, again. Remember the RakuAST project that was started by the main MoarVM elf about two and a half year ago? It’s been in off-and-on development since then, and now the core elves deemed it ready enough to make that work available in this new ‘main’ branch. So that other core and non-core elves could easily try out some of the new features that it is providing. “So, it’s now done, this RakuAST project?”, said Santa, with a little glimmer of hope in his eyes. “Ah, no, you could say that the project is now more than halfway”, Lizzybel said, hoping it would be enough for Santa. “Remind me again what that project was all about?”, Santa said, destroying Lizzybel’s hope for a quit exit. While sitting down, Lizzybel said: “An AST can be thought of as a document object model for a programming language. The goal of RakuAST is to provide an AST that is part of the Raku language specification, and thus can be relied upon by the language user. Such an AST is a prerequisite for a useful implementation of macros that actually solve practical problems, but also offers further powerful opportunities for the module developer. RakuAST will also become the initial internal representation of Raku programs used by Rakudo itself. That in turn gives an opportunity to improve the compiler.” “I bet you had ChatGPT type that out for you to memorize”, Santa said with a twinkle in his eye. “Eh, no, actually, this is from the MoarVM’s elf grant proposal in 2020, confessed Lizzybel. “Ok, so tell me what are the deliverables of that project? I don’t have all day to look through grant proposals, you know”, said Santa. Lizzybel peeked at her elfpad, took a deep breath and said: “Well, firstly: class and role implementations defining an document object model for the Raku language and its sub-languages, constructable and introspectable from within the Raku language. Secondly, the generation of QAST, the backend-independent intermediate representation, from RakuAST nodes, such that one can execute an AST. Thirdly, tests that cover the running of RakuAST nodes. And lastly, integration of RakuAST into the compilation process”. “Interesting”, said Santa, “and how much of that is done already?” “Enough to make more than 60% of the Rakudo test files pass completely, and more than 40% of the Raku test files pass completely if you use RakuAST for compiling your Raku code”, said Lizzybel. Use it now Santa continued what was now feeling like an interrogation. “So what use is RakuAST now?” “Well, it allows module developers to start playing with RakuAST features”, said Lizzybel. “But are you sure that RakuAST is stable enough for module developers to depend upon?”, Santa said, frowning. “No, the core elves are not sure enough about it yet, so that is why module developers will need to add a use experimental :rakuast to their code.” “Is there any documentation of these RakUAST classes?” “No, not really, but there are test files in the t/12-rakuast subdirectory. And there is a proof-of-concept of a module that converts sprintf format strings into executable code in the new Formatter class, that will be up to 30x faster”, Lizzybel blurted out. “Ok, that’s a start”, said Santa, with a lighter shade of red on his cheeks. Then Santa was distracted by the snow outside again and mumbled: “Are the reindeer prepared now?” Day 17: How to clarify which parts of the documentation change Using Pod::To::HTML2 a new custom FormatCode, D<> (D for deprecation), can be made to help with the Raku Documentation process. The new FormatCode should show a span of documentation that is deprecated in some way. This happens a lot when Rakudo is being upgraded. However, people using older versions of Rakudo need to understand what has changed, as well as what has been added. So it is not a good idea to delete older information, but it is not efficient to re-generate the entire Documentation suite for each new version of Rakudo. Perhaps it would be good for a span of words to be highlighted in some way, and then for a deprecation string to appear when a mouse hovers over it. For example D<function moveover( $x, $y, $z) { … } | Not expected to work in Rakudo-H > would be used to cover the function definition, and the deprecation string is after the |. First install the module using zef install Raku::Pod::Render which will install Pod::To:HTML2. You will need at least version 4.2.0. A default directory is created with some distribution plugins. To see examples of the distribution plugins, type Rakudoc-to-html Example in an empty directory. Then serve the file Samples.html using some html serving system. However, this is about making a bespoke plugin to implement a new Formatting Code. Pod::To::HTML2 interprets specified local sub-directories whose name does not contain the character _ after the first character of the name to contain plugin information. Pod::To::HTML2 is a sub-class of ProcessedPod, so below I shall mention instances of ProcessedPod, though possibly I should be saying instances of Pod::To::HTML2. Lets start with an empty directory ‘test’ (this article is written for Ubuntu linux, apologies for those on other systems that differ significantly). Now we enter the directory and create a Rakudoc file (eg. ‘test-d.rakudoc’) with the following text: =begin pod This is some text to test a new format code. For example, D<function moveover( $x, $y, $z) { ... } | Not expected to work in Rakudo-H > should have a highlight and a deprecation string. =end pod Now if you run Rakudoc-to-html test-d.rakudoc in the test/ directory you will get an html file test-d.html together with a directory asset_files containing some CSS files and the icon images. Note how ‘asset_files’ has a ‘_’ in it so that it will not be interpreted in the future as a plugin. The file test-d.html can be served to a browser. I have the excellent Comma IDE, which allows a project-root-directory file to be served to a brower simply by opening it in that browser. I am sure everyone reading this article will have some favourite way of serving a file. The FormatCode is not known to the Renderer, so the unknown-name template is triggered for a FormatCode. To create a plugin, we need to: • tell the renderer that a custom Block is available. However, the Pod-Block for a FormatCode already exists, so we only need to provide a template for D. (I wrote about this in case you want to experiment with new Custom Blocks). • tell the renderer what HTML needs to be created for the FormatCode-D, that is provide a template. • provide Pod::To::HTML2 with a name for the CSS to be associated with the HTML containers, which we need to get the highlighting effect. We create a sub-directory of test/ called deprecation-span. The name is not too important but it contains a ‘-‘ rather than ‘_’, though a name without ‘-‘ is possible. Inside deprecation-span we create a file called config.raku. The name is important and a plugin must have a config.raku file. A config.raku is a Raku program that ends with a hash value. The following is a possible minimal content %( :custom-raku(), # this key is mandatory, so we need it to exist and have Nil value :template-raku<deprecation-template.raku>, #:add-css<deprecate-span.css>, ) You will see that this a hash in Raku idiom. One could call it RakuON by analogy with JSON. But you will also see that because it is normal Raku code, we can include comments as well. I have also commented-out the CSS line, as we will discuss CSS below. The template is provided by ‘deprecate-template.raku’. Although multiple templating engines, such as RakuClosure and Mustache, can also be used with ProcessedPod, I have not yet had enough time to develop the HTML2 plugins to use more than one. So I will use the default RakuClosureTemplates system here. Basically all RakuClosure templates are contained in a Raku program that returns a Hash (like config.raku). The keys of the Hash are the names of the Pod-Block. The values for the keys are closures, viz., a sub that accepts two Hash parameters (conventionally %prm and %tml). The first (%prm) contains all the parameters passed by ProcessedPod to the template, and the second (%tml) contains all the templates known to ProcessedPod. So any template can call any template. (Currently, circularity is not detected). The sub must return a Str, which is inserted into the final html file. Plugins create a template Hash whose keys (new templates) are added to the default keys. The ProcessedPod renderer passes (at least) two parameters to a template for a FormatCode in the %prm hash. These are contents, which is the first part of the FormatCode, and meta, which is the part after the |. So we create a file called deprecate-template.raku with the following contents: %( format-d => sub (%prm, %tml) { # note that the format letter is lower case '<span class="raku-deprecation" title="' ~ %prm<meta> ~ '">' ~ %prm<contents> ~ '</span>' }, ) We also have to tell that there is a new plugin, so we run Rakudoc-to-html --add-plugins='deprecate-span' test-d.rakudoc (add-plugins can take a space delimited list of plugins) Now we have the correct text without an error, and if we put a mouse over the word ‘function’, we will get the deprecation string. In order to highlight the span so that the user can be prompted to hover a mouse over the text, we need to have some CSS. By way of example, put the following CSS in the file deprecate-span.css (remember the HTML class raku-deprecation was included in the template): .raku-deprecation { background-color: bisque; border-block-color: blue; border-width: 1px; border-style: dashed; border-radius: 5px; } We need to uncomment the :add-css line in config.raku. deprecate-span.css is assumed to be a valid CSS file and it will be transferred to test/asset_files/css/ by Pod::To::HTML2. Pod::To::HTML2 also creates the stylesheet reference in the HTML header so that it is served too. Run the file again Rakudoc-to-html --add-plugins='deprecate-span' test-d.rakudoc and the CSS has been added. Obviously, a lot more CSS tricks can be played, but I just wanted to show some CSS. There is much more to the plugin process, including the ability to add JQuery and images. In order to examine copies of the distributed plugins into your local test directory, run the following in that directory. Rakudoc-to-html get-local Rendered from newplugin at 2022-12-12T22:11:57Z
{ "url": "https://raku-advent.blog/blog-feed/", "source_domain": "raku-advent.blog", "snapshot_id": "CC-MAIN-2023-06", "warc_metadata": { "Content-Length": "437249", "Content-Type": "application/http; msgtype=response", "WARC-Block-Digest": "sha1:LRACMDI6YKMX75C6PYWTCXWBGEHSG7AP", "WARC-Concurrent-To": "<urn:uuid:434a00ea-42c7-4217-b220-ae93e2c9ac31>", "WARC-Date": "2023-01-31T12:49:30Z", "WARC-IP-Address": "192.0.78.25", "WARC-Identified-Payload-Type": "text/html", "WARC-Payload-Digest": "sha1:MM7EJCIHUI33YB66IR7OQN2DYU4BT3UF", "WARC-Record-ID": "<urn:uuid:965aac79-6cf5-42fb-a676-38ca3bfd6b31>", "WARC-Target-URI": "https://raku-advent.blog/blog-feed/", "WARC-Truncated": null, "WARC-Type": "response", "WARC-Warcinfo-ID": "<urn:uuid:060f390f-fecd-4984-ba55-5060406d6185>" }, "warc_info": "isPartOf: CC-MAIN-2023-06\r\npublisher: Common Crawl\r\ndescription: Wide crawl of the web for January/February 2023\r\noperator: Common Crawl Admin ([email protected])\r\nhostname: ip-10-67-67-43\r\nsoftware: Apache Nutch 1.19 (modified, https://github.com/commoncrawl/nutch/)\r\nrobots: checked via crawler-commons 1.4-SNAPSHOT (https://github.com/crawler-commons/crawler-commons)\r\nformat: WARC File Format 1.1\r\nconformsTo: https://iipc.github.io/warc-specifications/specifications/warc-format/warc-1.1/" }
{ "line_start_idx": [ 0, 9, 10, 37, 38, 88, 89, 98, 99, 126, 127, 136, 137, 164, 165, 657, 658, 857, 858, 961, 962, 989, 990, 1155, 1156, 1494, 1495, 1651, 1652, 1675, 1676, 1812, 1813, 1820, 1848, 1857, 1860, 1870, 1874, 1890, 1894, 1895, 2022, 2023, 2040, 2041, 2167, 2168, 2177, 2178, 2260, 2261, 2285, 2286, 2421, 2422, 2450, 2451, 2509, 2510, 2607, 2608, 2618, 2619, 3090, 3091, 3111, 3132, 3153, 3154, 3169, 3185, 3200, 3201, 3218, 3234, 3250, 3251, 3358, 3359, 3375, 3376, 3573, 3574, 3580, 3581, 3823, 3824, 3848, 3891, 3892, 3900, 3901, 4151, 4152, 4170, 4192, 4226, 4282, 4283, 4366, 4367, 4390, 4391, 4459, 4460, 4487, 4522, 4549, 4550, 4624, 4625, 4637, 4638, 4646, 4647, 4780, 4781, 4822, 4863, 4903, 4904, 5030, 5031, 5072, 5073, 5263, 5264, 5314, 5376, 5446, 5514, 5598, 5599, 5628, 5629, 5805, 5806, 5923, 5924, 5960, 5996, 5997, 6091, 6092, 6131, 6170, 6171, 6293, 6294, 6313, 6351, 6382, 6412, 6414, 6415, 6437, 6438, 6611, 6612, 6627, 6628, 6950, 6951, 7252, 7253, 7318, 7319, 7323, 7324, 7400, 7401, 7410, 7411, 7492, 7493, 7500, 7501, 7617, 7618, 7626, 7627, 7767, 7768, 7778, 7779, 7811, 7812, 7978, 7979, 7991, 8055, 8090, 8096, 8098, 8123, 8155, 8193, 8239, 8240, 8433, 8434, 8460, 8461, 8583, 8584, 8603, 8604, 8940, 8941, 9051, 9052, 9069, 9121, 9122, 9135, 9136, 9448, 9449, 9475, 9476, 9711, 9712, 9727, 9728, 10022, 10023, 10056, 10114, 10126, 10196, 10197, 10329, 10330, 10339, 10340, 10841, 10842, 11280, 11281, 11604, 11605, 11673, 11674, 11701, 11702, 11746, 11783, 11827, 11828, 11889, 11915, 11916, 12049, 12050, 12244, 12245, 12431, 12432, 12472, 12473, 12494, 12495, 12634, 12635, 12697, 12753, 12754, 12790, 12791, 12837, 12838, 12869, 12870, 12890, 12891, 13110, 13111, 13147, 13180, 13181, 13265, 13266, 13289, 13290, 13305, 13306, 13525, 13526, 13547, 13583, 13627, 13629, 13630, 13671, 13672, 13694, 13695, 13923, 13924, 13964, 14004, 14005, 14031, 14032, 14193, 14194, 14204, 14205, 14432, 14433, 14451, 14474, 14506, 14507, 14616, 14617, 14627, 14628, 14756, 14757, 14775, 14784, 14804, 14815, 14834, 14835, 14889, 14890, 14910, 14911, 15236, 15237, 15255, 15289, 15311, 15312, 15352, 15353, 15395, 15419, 15420, 15570, 15571, 15618, 15642, 15643, 15680, 15681, 15692, 15693, 15917, 15918, 15948, 15995, 15999, 16022, 16050, 16051, 16190, 16191, 16221, 16276, 16280, 16316, 16344, 16345, 16370, 16371, 16496, 16497, 16528, 16569, 16606, 16635, 16636, 16755, 16756, 16774, 16805, 16839, 16879, 16926, 16960, 17004, 17038, 17039, 17108, 17109, 17122, 17161, 17202, 17253, 17292, 17293, 17309, 17310, 17484, 17485, 17503, 17552, 17553, 17650, 17651, 17686, 17687, 17811, 17812, 17830, 17854, 17878, 17922, 17966, 17967, 18070, 18071, 18094, 18095, 18199, 18200, 18245, 18246, 18292, 18293, 18357, 18358, 18434, 18435, 18456, 18457, 18502, 18503, 18520, 18521, 18653, 18654, 18673, 18674, 18784, 18785, 18802, 18803, 18925, 18926, 18934, 18935, 19102, 19103, 19293, 19294, 19423, 19424, 19461, 19462, 19983, 19984, 19995, 19996, 20103, 20104, 20121, 20132, 20151, 20169, 20193, 20197, 20220, 20262, 20266, 20332, 20385, 20443, 20447, 20469, 20471, 20475, 20541, 20568, 20630, 20669, 20732, 20736, 20756, 20791, 20826, 20854, 20880, 20907, 20956, 20981, 21013, 21027, 21059, 21104, 21118, 21150, 21176, 21190, 21209, 21228, 21242, 21252, 21267, 21271, 21296, 21324, 21339, 21341, 21342, 21790, 21791, 22034, 22035, 22343, 22344, 22360, 22361, 22981, 22982, 23286, 23287, 23382, 23383, 23409, 23410, 23577, 23578, 23619, 23620, 24230, 24231, 24281, 24322, 24370, 24373, 24432, 24433, 24631, 24632, 24797, 24798, 24835, 24884, 24924, 24974, 24978, 24981, 24982, 25455, 25456, 25493, 25542, 25582, 25633, 25667, 25709, 25715, 25719, 25725, 25726, 25853, 25854, 26094, 26095, 26826, 26827, 27380, 27381, 27394, 27443, 27492, 27532, 27582, 27586, 27589, 27590, 27633, 27681, 27704, 27744, 27795, 27850, 27875, 27904, 27937, 27983, 28028, 28050, 28094, 28150, 28196, 28244, 28256, 28266, 28274, 28280, 28285, 28315, 28355, 28406, 28440, 28490, 28496, 28500, 28503, 28505, 28518, 28567, 28616, 28656, 28706, 28710, 28713, 28717, 28727, 28765, 28801, 28833, 28855, 28903, 28924, 28972, 28994, 28998, 29001, 29002, 29320, 29321, 29361, 29390, 29411, 29454, 29476, 29478, 29479, 29690, 29691, 29912, 29913, 29936, 29962, 29988, 30004, 30005, 30301, 30302, 30314, 30370, 30411, 30459, 30497, 30537, 30541, 30544, 30548, 30561, 30615, 30656, 30704, 30743, 30783, 30787, 30790, 30794, 30807, 30861, 30902, 30950, 30989, 31029, 31033, 31038, 31039, 31563, 31564, 31597, 31621, 31658, 31694, 31753, 31763, 31771, 31777, 31779, 31783, 31812, 31840, 31868, 31898, 31899, 32152, 32153, 32190, 32221, 32238, 32252, 32287, 32320, 32339, 32346, 32381, 32414, 32433, 32439, 32444, 32460, 32464, 32465, 32704, 32705, 32742, 32781, 32801, 32831, 32893, 32948, 32957, 32964, 32969, 33002, 33034, 33076, 33102, 33110, 33116, 33120, 33124, 33125, 33470, 33471, 33621, 33622, 33679, 33699, 33760, 33809, 33867, 33927, 33983, 33987, 33995, 33999, 34027, 34042, 34044, 34045, 34226, 34227, 34774, 34775, 35178, 35179, 35190, 35191, 35727, 35728, 35783, 35784, 36093, 36094, 36103, 36104, 36240, 36241, 36482, 36483, 37235, 37236, 37303, 37304, 37724, 37725, 37765, 37766, 38149, 38150, 38659, 38660, 38875, 38876, 39227, 39228, 39263, 39299, 39332, 39333, 39409, 39410, 39432, 39433, 40268, 40269, 40588, 40589, 40627, 40628, 41490, 41491, 41948, 41949, 42507, 42508, 42773, 42774, 42789, 42790, 43261, 43262, 43489, 43490, 43539, 43540, 43714, 43715, 43817, 43818, 43839, 43840, 44594, 44595, 45208, 45209, 45322, 45323, 45377, 45378, 46007, 46008, 46797, 46798, 47933, 47934, 47987, 47988, 48340, 48341, 48757, 48758, 49266, 49267, 49296, 49297, 49658, 49659, 49963, 49964, 50620, 50621, 51168, 51169, 51276, 51277, 51291, 51346, 51402, 51455, 51456, 51622, 51623, 51660, 51661, 51918, 51919, 52413, 52414, 52627, 52628, 52644, 52666, 52739, 52788, 52832, 52833, 53327, 53328, 53423, 53424, 53440, 53500, 53573, 53655, 53705, 53756, 53808, 53859, 53860, 53894, 53895, 54526, 54527, 54649, 54650, 54747, 54791, 54848, 54904, 54961, 55026, 55082, 55083, 55362, 55363, 55443, 55505, 55575, 55635, 55696, 55752, 55798, 55860, 55877, 55927, 56006, 56070, 56130, 56134, 56138, 56142, 56146, 56148, 56149, 56790, 56791, 57179, 57180, 57775, 57776, 57898, 57899, 57957, 58023, 58045, 58096, 58130, 58142, 58143, 58251, 58252, 58261, 58297, 58335, 58387, 58388, 58767, 58768, 58826, 58914, 58915, 59281, 59282, 59298, 59310, 59319, 59350, 59357, 59371, 59373, 59424, 59458, 59470, 59471, 59531, 59532, 59541, 59577, 59615, 59661, 59713, 59770, 59771, 60120, 60121, 60617, 60618, 60643, 60644, 61124, 61125, 61165, 61166, 61218, 61219, 61246, 61335, 61386, 61418, 61430, 61432, 61433, 62346, 62347, 62405, 62471, 62502, 62529, 62618, 62669, 62701, 62713, 62715, 62726, 62738, 62739, 63116, 63117, 63131, 63150, 63169, 63200, 63201, 63282, 63283, 63375, 63376, 63770, 63771, 63855, 63874, 63912, 63979, 64009, 64080, 64098, 64100, 64166, 64189, 64247, 64339, 64355, 64386, 64413, 64502, 64553, 64585, 64597, 64599, 64610, 64631, 64655, 64667, 64669, 64670, 64798, 64799, 64879, 64955, 65025, 65101, 65102, 65110, 65111, 65295, 65296, 65332, 65333, 65587, 65588, 65907, 65908, 65929, 65930, 66097, 66098, 66110, 66111, 66150, 66151, 66457, 66458, 66471, 66472, 66598, 66599, 66600, 66911, 66912, 67122, 67123, 67155, 67156, 67188, 67230, 67232, 67233, 67310, 67311, 67403, 67404, 67467, 67468, 67494, 67495, 67534, 67561, 67585, 67629, 67660, 67710, 67720, 67726, 67727, 67750, 67752, 67753, 67792, 67793, 67803, 67804, 67840, 67876, 67878, 67879, 67928, 67929, 67939, 67940, 68006, 68072, 68139, 68140, 68154, 68155, 68172, 68173, 68278, 68279, 68342, 68343, 68344, 68564, 68565, 68683, 68684, 68699, 68700, 68753, 68781, 68837, 68899, 68905, 68906, 68929, 68931, 68932, 68977, 68978, 69017, 69018, 69143, 69144, 69189, 69190, 69243, 69274, 69300, 69339, 69390, 69424, 69481, 69491, 69497, 69498, 69526, 69528, 69529, 69548, 69549, 69559, 69560, 69593, 69631, 69633, 69634, 69663, 69664, 69674, 69675, 69742, 69809, 69810, 69824, 69825, 69843, 69844, 69880, 69881, 70039, 70040, 70041, 70123, 70124, 70233, 70234, 70422, 70423, 70463, 70464, 70493, 70508, 70564, 70592, 70594, 70595, 70620, 70621, 70631, 70632, 70678, 70724, 70725, 70739, 70740, 70768, 70769, 70813, 70814, 70879, 70880, 70881, 70951, 70952, 71259, 71260, 71340, 71341, 71403, 71404, 71465, 71466, 71519, 71564, 71565, 71584, 71631, 71681, 71688, 71689, 71726, 71728, 71729, 71765, 71766, 71793, 71794, 71850, 71851, 71861, 71862, 71927, 71965, 71967, 71968, 71994, 71995, 72005, 72006, 72052, 72083, 72129, 72157, 72203, 72231, 72232, 72246, 72247, 72258, 72259, 72429, 72430, 72461, 72462, 72524, 72525, 73102, 73103, 73506, 73507, 73638, 73679, 73680, 73785, 73786, 74115, 74116, 74422, 74423, 74942, 74943, 75245, 75246, 75292, 75293, 75318, 75319, 75414, 75415, 75525, 75526, 75784, 75785, 76159, 76160, 76459, 76460, 76907, 76908, 76936, 76937, 77175, 77176, 77177, 77500, 77501, 77670, 77671, 77706, 77707, 77995, 77996, 78471, 78472, 78908, 78909, 79421, 79422, 79484, 79485, 79751, 79752, 80278, 80279, 80682, 80683, 81105, 81106, 81489, 81490, 81695, 81696, 81783, 81784, 82051, 82052, 82476, 82477, 82522, 82523, 82741, 82742, 83195, 83196, 83523, 83524, 83930, 83931, 83968, 83969, 84267, 84268, 84490, 84491, 84532, 84533, 84859, 84860, 85121, 85122, 85123, 85455, 85456, 85693, 85694, 85866, 85867, 86240, 86241, 86598, 86599, 86640, 86641, 86862, 86863, 86983, 87004, 87005, 87465, 87466, 87816, 87817, 88075, 88076, 88177, 88178, 88381, 88382, 88726, 88727, 89106, 89107, 89532, 89533, 89711, 89712, 90064, 90065, 90662, 90663, 90845, 90846, 90909, 90910, 91197, 91198, 91250, 91251, 91364, 91365, 91423, 91424, 91791, 91792, 92309, 92310, 92673, 92674, 92760, 92761, 92789, 92790, 93170, 93171, 93632, 93633, 94000, 94001, 94641, 94642, 95024, 95025, 95302, 95303, 95823, 95824, 96195, 96196, 96217, 96218, 96731, 96732, 96755, 96756, 97101, 97102, 97122, 97123, 97332, 97333, 97382, 97429, 97480, 97568, 97569, 98086, 98087, 98544, 98545, 98571, 98572, 98689, 98690, 99419, 99420, 100186, 100187, 100762, 100763, 101290, 101291, 101411, 101412, 101426, 101475, 101508, 101612, 101680, 101707, 101708, 101922, 101923, 102064, 102065, 102297, 102298, 102852, 102853, 102938, 102939, 103293, 103294, 103856, 103857, 104336, 104337, 104721, 104722, 104855, 104856, 105157, 105158, 105756, 105757, 106017, 106018, 106555, 106556, 107240, 107241, 107589, 107590, 107616, 107617, 107793, 107794, 108123, 108124, 108502, 108503, 108797, 108798, 109163, 109164, 109205, 109206, 109207, 109281, 109282, 109765, 109766, 109806, 109807, 110303, 110304, 110728, 110729, 110895, 110896, 110916, 110947, 110948, 111002, 111037, 111039, 111040, 111327, 111328, 111375, 111376, 111416, 111417, 111685, 111686, 111712, 111713, 111917, 111918, 111938, 111969, 111970, 111988, 111995, 111996, 112050, 112085, 112087, 112088, 112193, 112194, 112229, 112230, 112416, 112417, 112465, 112466, 112680, 112681, 112899, 112900, 113243, 113244, 113270, 113271, 113281, 113313, 113325, 113326, 113371, 113429, 113466, 113530, 113532, 113533, 113767, 113768, 113791, 113792, 113979, 113980, 114553, 114554, 114780, 114781, 114809, 114810, 114831, 114865, 114924, 114976, 115024, 115030, 115032, 115033, 115142, 115143, 115183, 115231, 115243, 115244, 115296, 115297, 115354, 115389, 115448, 115516, 115545, 115581, 115637, 115638, 115787, 115788, 115939, 115940, 115969, 116001, 116060, 116118, 116124, 116125, 116157, 116198, 116271, 116277, 116279, 116280, 116390, 116391, 116594, 116595, 116627, 116668, 116741, 116747, 116748, 116839, 116840, 116886, 116934, 116935, 117106, 117107, 117165, 117166, 117278, 117279, 117302, 117303, 117781, 117782, 118334, 118335, 118665, 118666, 118699, 118700, 118708, 118709, 119416, 119417, 120112, 120113, 120342, 120343, 121117, 121118, 121129, 121130, 121884, 121885, 121962, 121963, 122061, 122062, 122125, 122126, 122663, 122664, 122813, 122814, 122993, 122994, 123352, 123353, 123607, 123608, 123771, 123772, 123922, 123923, 124024, 124025, 124040, 124041, 124181, 124235, 124236, 124249, 124250, 124251, 124544, 124545, 124820, 124821, 124926, 124927, 124959, 124960, 125193, 125295, 125435, 125436, 125604, 125605, 125836, 125837, 125840, 125927, 125974, 126009, 126011, 126012, 126013, 126267, 126268, 126323, 126324, 126574, 126575, 127249, 127250, 127462, 127463, 127543, 127544, 127547, 127626, 127695, 127720, 127740, 127747, 127749, 127750, 127751, 127810, 127811, 127873, 127874, 127931, 127932, 128185, 128186, 128331, 128332, 128352, 128379, 128406, 128426, 128449, 128470, 128472, 128473, 128474, 128751, 128752, 128771, 128772, 128834, 128835, 128947, 128948, 129161, 129162, 129188, 129189, 129190 ], "line_end_idx": [ 9, 10, 37, 38, 88, 89, 98, 99, 126, 127, 136, 137, 164, 165, 657, 658, 857, 858, 961, 962, 989, 990, 1155, 1156, 1494, 1495, 1651, 1652, 1675, 1676, 1812, 1813, 1820, 1848, 1857, 1860, 1870, 1874, 1890, 1894, 1895, 2022, 2023, 2040, 2041, 2167, 2168, 2177, 2178, 2260, 2261, 2285, 2286, 2421, 2422, 2450, 2451, 2509, 2510, 2607, 2608, 2618, 2619, 3090, 3091, 3111, 3132, 3153, 3154, 3169, 3185, 3200, 3201, 3218, 3234, 3250, 3251, 3358, 3359, 3375, 3376, 3573, 3574, 3580, 3581, 3823, 3824, 3848, 3891, 3892, 3900, 3901, 4151, 4152, 4170, 4192, 4226, 4282, 4283, 4366, 4367, 4390, 4391, 4459, 4460, 4487, 4522, 4549, 4550, 4624, 4625, 4637, 4638, 4646, 4647, 4780, 4781, 4822, 4863, 4903, 4904, 5030, 5031, 5072, 5073, 5263, 5264, 5314, 5376, 5446, 5514, 5598, 5599, 5628, 5629, 5805, 5806, 5923, 5924, 5960, 5996, 5997, 6091, 6092, 6131, 6170, 6171, 6293, 6294, 6313, 6351, 6382, 6412, 6414, 6415, 6437, 6438, 6611, 6612, 6627, 6628, 6950, 6951, 7252, 7253, 7318, 7319, 7323, 7324, 7400, 7401, 7410, 7411, 7492, 7493, 7500, 7501, 7617, 7618, 7626, 7627, 7767, 7768, 7778, 7779, 7811, 7812, 7978, 7979, 7991, 8055, 8090, 8096, 8098, 8123, 8155, 8193, 8239, 8240, 8433, 8434, 8460, 8461, 8583, 8584, 8603, 8604, 8940, 8941, 9051, 9052, 9069, 9121, 9122, 9135, 9136, 9448, 9449, 9475, 9476, 9711, 9712, 9727, 9728, 10022, 10023, 10056, 10114, 10126, 10196, 10197, 10329, 10330, 10339, 10340, 10841, 10842, 11280, 11281, 11604, 11605, 11673, 11674, 11701, 11702, 11746, 11783, 11827, 11828, 11889, 11915, 11916, 12049, 12050, 12244, 12245, 12431, 12432, 12472, 12473, 12494, 12495, 12634, 12635, 12697, 12753, 12754, 12790, 12791, 12837, 12838, 12869, 12870, 12890, 12891, 13110, 13111, 13147, 13180, 13181, 13265, 13266, 13289, 13290, 13305, 13306, 13525, 13526, 13547, 13583, 13627, 13629, 13630, 13671, 13672, 13694, 13695, 13923, 13924, 13964, 14004, 14005, 14031, 14032, 14193, 14194, 14204, 14205, 14432, 14433, 14451, 14474, 14506, 14507, 14616, 14617, 14627, 14628, 14756, 14757, 14775, 14784, 14804, 14815, 14834, 14835, 14889, 14890, 14910, 14911, 15236, 15237, 15255, 15289, 15311, 15312, 15352, 15353, 15395, 15419, 15420, 15570, 15571, 15618, 15642, 15643, 15680, 15681, 15692, 15693, 15917, 15918, 15948, 15995, 15999, 16022, 16050, 16051, 16190, 16191, 16221, 16276, 16280, 16316, 16344, 16345, 16370, 16371, 16496, 16497, 16528, 16569, 16606, 16635, 16636, 16755, 16756, 16774, 16805, 16839, 16879, 16926, 16960, 17004, 17038, 17039, 17108, 17109, 17122, 17161, 17202, 17253, 17292, 17293, 17309, 17310, 17484, 17485, 17503, 17552, 17553, 17650, 17651, 17686, 17687, 17811, 17812, 17830, 17854, 17878, 17922, 17966, 17967, 18070, 18071, 18094, 18095, 18199, 18200, 18245, 18246, 18292, 18293, 18357, 18358, 18434, 18435, 18456, 18457, 18502, 18503, 18520, 18521, 18653, 18654, 18673, 18674, 18784, 18785, 18802, 18803, 18925, 18926, 18934, 18935, 19102, 19103, 19293, 19294, 19423, 19424, 19461, 19462, 19983, 19984, 19995, 19996, 20103, 20104, 20121, 20132, 20151, 20169, 20193, 20197, 20220, 20262, 20266, 20332, 20385, 20443, 20447, 20469, 20471, 20475, 20541, 20568, 20630, 20669, 20732, 20736, 20756, 20791, 20826, 20854, 20880, 20907, 20956, 20981, 21013, 21027, 21059, 21104, 21118, 21150, 21176, 21190, 21209, 21228, 21242, 21252, 21267, 21271, 21296, 21324, 21339, 21341, 21342, 21790, 21791, 22034, 22035, 22343, 22344, 22360, 22361, 22981, 22982, 23286, 23287, 23382, 23383, 23409, 23410, 23577, 23578, 23619, 23620, 24230, 24231, 24281, 24322, 24370, 24373, 24432, 24433, 24631, 24632, 24797, 24798, 24835, 24884, 24924, 24974, 24978, 24981, 24982, 25455, 25456, 25493, 25542, 25582, 25633, 25667, 25709, 25715, 25719, 25725, 25726, 25853, 25854, 26094, 26095, 26826, 26827, 27380, 27381, 27394, 27443, 27492, 27532, 27582, 27586, 27589, 27590, 27633, 27681, 27704, 27744, 27795, 27850, 27875, 27904, 27937, 27983, 28028, 28050, 28094, 28150, 28196, 28244, 28256, 28266, 28274, 28280, 28285, 28315, 28355, 28406, 28440, 28490, 28496, 28500, 28503, 28505, 28518, 28567, 28616, 28656, 28706, 28710, 28713, 28717, 28727, 28765, 28801, 28833, 28855, 28903, 28924, 28972, 28994, 28998, 29001, 29002, 29320, 29321, 29361, 29390, 29411, 29454, 29476, 29478, 29479, 29690, 29691, 29912, 29913, 29936, 29962, 29988, 30004, 30005, 30301, 30302, 30314, 30370, 30411, 30459, 30497, 30537, 30541, 30544, 30548, 30561, 30615, 30656, 30704, 30743, 30783, 30787, 30790, 30794, 30807, 30861, 30902, 30950, 30989, 31029, 31033, 31038, 31039, 31563, 31564, 31597, 31621, 31658, 31694, 31753, 31763, 31771, 31777, 31779, 31783, 31812, 31840, 31868, 31898, 31899, 32152, 32153, 32190, 32221, 32238, 32252, 32287, 32320, 32339, 32346, 32381, 32414, 32433, 32439, 32444, 32460, 32464, 32465, 32704, 32705, 32742, 32781, 32801, 32831, 32893, 32948, 32957, 32964, 32969, 33002, 33034, 33076, 33102, 33110, 33116, 33120, 33124, 33125, 33470, 33471, 33621, 33622, 33679, 33699, 33760, 33809, 33867, 33927, 33983, 33987, 33995, 33999, 34027, 34042, 34044, 34045, 34226, 34227, 34774, 34775, 35178, 35179, 35190, 35191, 35727, 35728, 35783, 35784, 36093, 36094, 36103, 36104, 36240, 36241, 36482, 36483, 37235, 37236, 37303, 37304, 37724, 37725, 37765, 37766, 38149, 38150, 38659, 38660, 38875, 38876, 39227, 39228, 39263, 39299, 39332, 39333, 39409, 39410, 39432, 39433, 40268, 40269, 40588, 40589, 40627, 40628, 41490, 41491, 41948, 41949, 42507, 42508, 42773, 42774, 42789, 42790, 43261, 43262, 43489, 43490, 43539, 43540, 43714, 43715, 43817, 43818, 43839, 43840, 44594, 44595, 45208, 45209, 45322, 45323, 45377, 45378, 46007, 46008, 46797, 46798, 47933, 47934, 47987, 47988, 48340, 48341, 48757, 48758, 49266, 49267, 49296, 49297, 49658, 49659, 49963, 49964, 50620, 50621, 51168, 51169, 51276, 51277, 51291, 51346, 51402, 51455, 51456, 51622, 51623, 51660, 51661, 51918, 51919, 52413, 52414, 52627, 52628, 52644, 52666, 52739, 52788, 52832, 52833, 53327, 53328, 53423, 53424, 53440, 53500, 53573, 53655, 53705, 53756, 53808, 53859, 53860, 53894, 53895, 54526, 54527, 54649, 54650, 54747, 54791, 54848, 54904, 54961, 55026, 55082, 55083, 55362, 55363, 55443, 55505, 55575, 55635, 55696, 55752, 55798, 55860, 55877, 55927, 56006, 56070, 56130, 56134, 56138, 56142, 56146, 56148, 56149, 56790, 56791, 57179, 57180, 57775, 57776, 57898, 57899, 57957, 58023, 58045, 58096, 58130, 58142, 58143, 58251, 58252, 58261, 58297, 58335, 58387, 58388, 58767, 58768, 58826, 58914, 58915, 59281, 59282, 59298, 59310, 59319, 59350, 59357, 59371, 59373, 59424, 59458, 59470, 59471, 59531, 59532, 59541, 59577, 59615, 59661, 59713, 59770, 59771, 60120, 60121, 60617, 60618, 60643, 60644, 61124, 61125, 61165, 61166, 61218, 61219, 61246, 61335, 61386, 61418, 61430, 61432, 61433, 62346, 62347, 62405, 62471, 62502, 62529, 62618, 62669, 62701, 62713, 62715, 62726, 62738, 62739, 63116, 63117, 63131, 63150, 63169, 63200, 63201, 63282, 63283, 63375, 63376, 63770, 63771, 63855, 63874, 63912, 63979, 64009, 64080, 64098, 64100, 64166, 64189, 64247, 64339, 64355, 64386, 64413, 64502, 64553, 64585, 64597, 64599, 64610, 64631, 64655, 64667, 64669, 64670, 64798, 64799, 64879, 64955, 65025, 65101, 65102, 65110, 65111, 65295, 65296, 65332, 65333, 65587, 65588, 65907, 65908, 65929, 65930, 66097, 66098, 66110, 66111, 66150, 66151, 66457, 66458, 66471, 66472, 66598, 66599, 66600, 66911, 66912, 67122, 67123, 67155, 67156, 67188, 67230, 67232, 67233, 67310, 67311, 67403, 67404, 67467, 67468, 67494, 67495, 67534, 67561, 67585, 67629, 67660, 67710, 67720, 67726, 67727, 67750, 67752, 67753, 67792, 67793, 67803, 67804, 67840, 67876, 67878, 67879, 67928, 67929, 67939, 67940, 68006, 68072, 68139, 68140, 68154, 68155, 68172, 68173, 68278, 68279, 68342, 68343, 68344, 68564, 68565, 68683, 68684, 68699, 68700, 68753, 68781, 68837, 68899, 68905, 68906, 68929, 68931, 68932, 68977, 68978, 69017, 69018, 69143, 69144, 69189, 69190, 69243, 69274, 69300, 69339, 69390, 69424, 69481, 69491, 69497, 69498, 69526, 69528, 69529, 69548, 69549, 69559, 69560, 69593, 69631, 69633, 69634, 69663, 69664, 69674, 69675, 69742, 69809, 69810, 69824, 69825, 69843, 69844, 69880, 69881, 70039, 70040, 70041, 70123, 70124, 70233, 70234, 70422, 70423, 70463, 70464, 70493, 70508, 70564, 70592, 70594, 70595, 70620, 70621, 70631, 70632, 70678, 70724, 70725, 70739, 70740, 70768, 70769, 70813, 70814, 70879, 70880, 70881, 70951, 70952, 71259, 71260, 71340, 71341, 71403, 71404, 71465, 71466, 71519, 71564, 71565, 71584, 71631, 71681, 71688, 71689, 71726, 71728, 71729, 71765, 71766, 71793, 71794, 71850, 71851, 71861, 71862, 71927, 71965, 71967, 71968, 71994, 71995, 72005, 72006, 72052, 72083, 72129, 72157, 72203, 72231, 72232, 72246, 72247, 72258, 72259, 72429, 72430, 72461, 72462, 72524, 72525, 73102, 73103, 73506, 73507, 73638, 73679, 73680, 73785, 73786, 74115, 74116, 74422, 74423, 74942, 74943, 75245, 75246, 75292, 75293, 75318, 75319, 75414, 75415, 75525, 75526, 75784, 75785, 76159, 76160, 76459, 76460, 76907, 76908, 76936, 76937, 77175, 77176, 77177, 77500, 77501, 77670, 77671, 77706, 77707, 77995, 77996, 78471, 78472, 78908, 78909, 79421, 79422, 79484, 79485, 79751, 79752, 80278, 80279, 80682, 80683, 81105, 81106, 81489, 81490, 81695, 81696, 81783, 81784, 82051, 82052, 82476, 82477, 82522, 82523, 82741, 82742, 83195, 83196, 83523, 83524, 83930, 83931, 83968, 83969, 84267, 84268, 84490, 84491, 84532, 84533, 84859, 84860, 85121, 85122, 85123, 85455, 85456, 85693, 85694, 85866, 85867, 86240, 86241, 86598, 86599, 86640, 86641, 86862, 86863, 86983, 87004, 87005, 87465, 87466, 87816, 87817, 88075, 88076, 88177, 88178, 88381, 88382, 88726, 88727, 89106, 89107, 89532, 89533, 89711, 89712, 90064, 90065, 90662, 90663, 90845, 90846, 90909, 90910, 91197, 91198, 91250, 91251, 91364, 91365, 91423, 91424, 91791, 91792, 92309, 92310, 92673, 92674, 92760, 92761, 92789, 92790, 93170, 93171, 93632, 93633, 94000, 94001, 94641, 94642, 95024, 95025, 95302, 95303, 95823, 95824, 96195, 96196, 96217, 96218, 96731, 96732, 96755, 96756, 97101, 97102, 97122, 97123, 97332, 97333, 97382, 97429, 97480, 97568, 97569, 98086, 98087, 98544, 98545, 98571, 98572, 98689, 98690, 99419, 99420, 100186, 100187, 100762, 100763, 101290, 101291, 101411, 101412, 101426, 101475, 101508, 101612, 101680, 101707, 101708, 101922, 101923, 102064, 102065, 102297, 102298, 102852, 102853, 102938, 102939, 103293, 103294, 103856, 103857, 104336, 104337, 104721, 104722, 104855, 104856, 105157, 105158, 105756, 105757, 106017, 106018, 106555, 106556, 107240, 107241, 107589, 107590, 107616, 107617, 107793, 107794, 108123, 108124, 108502, 108503, 108797, 108798, 109163, 109164, 109205, 109206, 109207, 109281, 109282, 109765, 109766, 109806, 109807, 110303, 110304, 110728, 110729, 110895, 110896, 110916, 110947, 110948, 111002, 111037, 111039, 111040, 111327, 111328, 111375, 111376, 111416, 111417, 111685, 111686, 111712, 111713, 111917, 111918, 111938, 111969, 111970, 111988, 111995, 111996, 112050, 112085, 112087, 112088, 112193, 112194, 112229, 112230, 112416, 112417, 112465, 112466, 112680, 112681, 112899, 112900, 113243, 113244, 113270, 113271, 113281, 113313, 113325, 113326, 113371, 113429, 113466, 113530, 113532, 113533, 113767, 113768, 113791, 113792, 113979, 113980, 114553, 114554, 114780, 114781, 114809, 114810, 114831, 114865, 114924, 114976, 115024, 115030, 115032, 115033, 115142, 115143, 115183, 115231, 115243, 115244, 115296, 115297, 115354, 115389, 115448, 115516, 115545, 115581, 115637, 115638, 115787, 115788, 115939, 115940, 115969, 116001, 116060, 116118, 116124, 116125, 116157, 116198, 116271, 116277, 116279, 116280, 116390, 116391, 116594, 116595, 116627, 116668, 116741, 116747, 116748, 116839, 116840, 116886, 116934, 116935, 117106, 117107, 117165, 117166, 117278, 117279, 117302, 117303, 117781, 117782, 118334, 118335, 118665, 118666, 118699, 118700, 118708, 118709, 119416, 119417, 120112, 120113, 120342, 120343, 121117, 121118, 121129, 121130, 121884, 121885, 121962, 121963, 122061, 122062, 122125, 122126, 122663, 122664, 122813, 122814, 122993, 122994, 123352, 123353, 123607, 123608, 123771, 123772, 123922, 123923, 124024, 124025, 124040, 124041, 124181, 124235, 124236, 124249, 124250, 124251, 124544, 124545, 124820, 124821, 124926, 124927, 124959, 124960, 125193, 125295, 125435, 125436, 125604, 125605, 125836, 125837, 125840, 125927, 125974, 126009, 126011, 126012, 126013, 126267, 126268, 126323, 126324, 126574, 126575, 127249, 127250, 127462, 127463, 127543, 127544, 127547, 127626, 127695, 127720, 127740, 127747, 127749, 127750, 127751, 127810, 127811, 127873, 127874, 127931, 127932, 128185, 128186, 128331, 128332, 128352, 128379, 128406, 128426, 128449, 128470, 128472, 128473, 128474, 128751, 128752, 128771, 128772, 128834, 128835, 128947, 128948, 129161, 129162, 129188, 129189, 129190, 129237 ] }
{ "red_pajama_v2": { "ccnet_original_length": 129237, "ccnet_original_nlines": 1802, "rps_doc_curly_bracket": 0.002460600109770894, "rps_doc_ldnoobw_words": 2, "rps_doc_lorem_ipsum": 0, "rps_doc_stop_word_fraction": 0.37658533453941345, "rps_doc_ut1_blacklist": 0, "rps_doc_frac_all_caps_words": 0.01637027971446514, "rps_doc_frac_lines_end_with_ellipsis": 0.004437049850821495, "rps_doc_frac_no_alph_words": 0.2654052972793579, "rps_doc_frac_unique_words": 0.17118977010250092, "rps_doc_mean_word_length": 4.827237129211426, "rps_doc_num_sentences": 1404, "rps_doc_symbol_to_word_ratio": 0.005996690131723881, "rps_doc_unigram_entropy": 6.581557273864746, "rps_doc_word_count": 20340, "rps_doc_frac_chars_dupe_10grams": 0.02909783087670803, "rps_doc_frac_chars_dupe_5grams": 0.06027336046099663, "rps_doc_frac_chars_dupe_6grams": 0.05053674057126045, "rps_doc_frac_chars_dupe_7grams": 0.0401584692299366, "rps_doc_frac_chars_dupe_8grams": 0.0309718307107687, "rps_doc_frac_chars_dupe_9grams": 0.029321899637579918, "rps_doc_frac_chars_top_2gram": 0.0042266701348125935, "rps_doc_frac_chars_top_3gram": 0.0007944100070744753, "rps_doc_frac_chars_top_4gram": 0.0009777400409802794, "rps_doc_books_importance": -14679.873046875, "rps_doc_books_importance_length_correction": -14679.873046875, "rps_doc_openwebtext_importance": -8382.767578125, "rps_doc_openwebtext_importance_length_correction": -8382.767578125, "rps_doc_wikipedia_importance": -6003.69921875, "rps_doc_wikipedia_importance_length_correction": -6003.69921875 }, "fasttext": { "dclm": 0.19367587566375732, "english": 0.8956151008605957, "fineweb_edu_approx": 2.146923780441284, "eai_general_math": 0.8948882222175598, "eai_open_web_math": 0.3626837134361267, "eai_web_code": 0.7977452278137207 } }
{ "free_decimal_correspondence": { "primary": { "code": "005.1332", "labels": { "level_1": "General works, books and libraries, information sciences", "level_2": "", "level_3": "Computer programming" } }, "secondary": { "code": "005.133", "labels": { "level_1": "General works, books and libraries, information sciences", "level_2": "", "level_3": "Computer programming" } } }, "bloom_cognitive_process": { "primary": { "code": "2", "label": "Understand" }, "secondary": { "code": "3", "label": "Apply" } }, "bloom_knowledge_domain": { "primary": { "code": "2", "label": "Conceptual" }, "secondary": { "code": "3", "label": "Procedural" } }, "document_type_v1": { "primary": { "code": "3", "label": "Reference/Encyclopedic/Educational" }, "secondary": { "code": "-1", "label": "Abstain" } }, "extraction_artifacts": { "primary": { "code": "0", "label": "No Artifacts" }, "secondary": { "code": "3", "label": "Irrelevant Content" } }, "missing_content": { "primary": { "code": "0", "label": "No missing content" }, "secondary": { "code": "-1", "label": "Abstain" } }, "document_type_v2": { "primary": { "code": "16", "label": "Personal Blog" }, "secondary": { "code": "8", "label": "Documentation" } }, "reasoning_depth": { "primary": { "code": "3", "label": "Intermediate Reasoning" }, "secondary": { "code": "2", "label": "Basic Reasoning" } }, "technical_correctness": { "primary": { "code": "4", "label": "Highly Correct" }, "secondary": { "code": "3", "label": "Mostly Correct" } }, "education_level": { "primary": { "code": "3", "label": "Undergraduate Level" }, "secondary": { "code": "2", "label": "High School Level" } } }
672f1e42c33a7f9846924a2431ea77df
3,728,438,064,643,827,700
Unlock the Secrets of Decentralized Technology with Algha_Porthos' Blog Are you fascinated by the world of decentralized technology? Do you want to stay up-to-date with the latest developments and explore new possibilities in blockchain and cryptocurrency? Then look no further than Algha_Porthos' Blog, a comprehensive resource for anyone interested in this exciting and rapidly-evolving field. Led by Algha_Porthos, a seasoned expert in decentralized technology, this blog provides valuable insights, analysis, and commentary on the latest trends, innovations, and challenges in the world of blockchain and cryptocurrency. With years of experience in the field, Algha_Porthos is uniquely positioned to provide authoritative and insightful content for both novice and expert readers. Whether you're interested in the technical aspects of blockchain, the latest trends in cryptocurrency, or the broader implications of decentralized technology for society and the economy, Algha_Porthos' Blog has something for you. With regular updates and in-depth analysis, you'll always be on top of the latest developments and have the knowledge you need to make informed decisions and stay ahead of the curve. But don't take our word for it – check out the blog for yourself and see what all the fuss is about. With engaging and informative articles on a wide range of topics, from smart contracts to DeFi to NFTs and more, you're sure to find something that piques your interest and expands your knowledge. So what are you waiting for? Visit Algha_Porthos' Blog today and start exploring the fascinating world of decentralized technology. Link to blog: https://53058.blog.luogu.org/
{ "url": "https://isab.run/paste/5d8a512c", "source_domain": "isab.run", "snapshot_id": "CC-MAIN-2024-10", "warc_metadata": { "Content-Length": "5087", "Content-Type": "application/http; msgtype=response", "WARC-Block-Digest": "sha1:DMOT33AX322AMIFXCFUOZGO4HG6WC2FU", "WARC-Concurrent-To": "<urn:uuid:94a05354-84b6-48ae-947b-8260bad6eafc>", "WARC-Date": "2024-02-27T21:03:01Z", "WARC-IP-Address": "172.67.194.98", "WARC-Identified-Payload-Type": "text/html", "WARC-Payload-Digest": "sha1:SSBDYONFJFTQ3BONP7SJZKNDKP7WLASZ", "WARC-Record-ID": "<urn:uuid:4a94b6c3-912a-460a-b5c9-c9af446113b1>", "WARC-Target-URI": "https://isab.run/paste/5d8a512c", "WARC-Truncated": null, "WARC-Type": "response", "WARC-Warcinfo-ID": "<urn:uuid:5e2e5fc0-5d92-4bcd-958c-f6654127dfbe>" }, "warc_info": "isPartOf: CC-MAIN-2024-10\r\npublisher: Common Crawl\r\ndescription: Wide crawl of the web for February/March 2024\r\noperator: Common Crawl Admin ([email protected])\r\nhostname: ip-10-67-67-79\r\nsoftware: Apache Nutch 1.19 (modified, https://github.com/commoncrawl/nutch/)\r\nrobots: checked via crawler-commons 1.5-SNAPSHOT (https://github.com/crawler-commons/crawler-commons)\r\nformat: WARC File Format 1.1\r\nconformsTo: https://iipc.github.io/warc-specifications/specifications/warc-format/warc-1.1/" }
{ "line_start_idx": [ 0, 72, 73, 397, 398, 787, 788, 1202, 1203, 1501, 1502, 1634, 1635 ], "line_end_idx": [ 72, 73, 397, 398, 787, 788, 1202, 1203, 1501, 1502, 1634, 1635, 1678 ] }
{ "red_pajama_v2": { "ccnet_original_length": 1678, "ccnet_original_nlines": 12, "rps_doc_curly_bracket": 0, "rps_doc_ldnoobw_words": 0, "rps_doc_lorem_ipsum": 0, "rps_doc_stop_word_fraction": 0.4131147563457489, "rps_doc_ut1_blacklist": 0, "rps_doc_frac_all_caps_words": 0, "rps_doc_frac_lines_end_with_ellipsis": 0, "rps_doc_frac_no_alph_words": 0.14754098653793335, "rps_doc_frac_unique_words": 0.5440000295639038, "rps_doc_mean_word_length": 5.48799991607666, "rps_doc_num_sentences": 15, "rps_doc_symbol_to_word_ratio": 0, "rps_doc_unigram_entropy": 4.492733955383301, "rps_doc_word_count": 250, "rps_doc_frac_chars_dupe_10grams": 0, "rps_doc_frac_chars_dupe_5grams": 0, "rps_doc_frac_chars_dupe_6grams": 0, "rps_doc_frac_chars_dupe_7grams": 0, "rps_doc_frac_chars_dupe_8grams": 0, "rps_doc_frac_chars_dupe_9grams": 0, "rps_doc_frac_chars_top_2gram": 0.08381924033164978, "rps_doc_frac_chars_top_3gram": 0.07288630306720734, "rps_doc_frac_chars_top_4gram": 0.04373177886009216, "rps_doc_books_importance": -157.8178253173828, "rps_doc_books_importance_length_correction": -144.8969268798828, "rps_doc_openwebtext_importance": -85.129638671875, "rps_doc_openwebtext_importance_length_correction": -85.129638671875, "rps_doc_wikipedia_importance": -75.27091217041016, "rps_doc_wikipedia_importance_length_correction": -64.55638122558594 }, "fasttext": { "dclm": 0.022653400897979736, "english": 0.9165005087852478, "fineweb_edu_approx": 1.1388516426086426, "eai_general_math": 0.0016342999879270792, "eai_open_web_math": 0.04535878077149391, "eai_web_code": 0.0006175599992275238 } }
{ "free_decimal_correspondence": { "primary": { "code": "004.0285", "labels": { "level_1": "General works, books and libraries, information sciences", "level_2": "", "level_3": "Computers and Computer science" } }, "secondary": { "code": "332.8", "labels": { "level_1": "Social sciences", "level_2": "Economics", "level_3": "Finance" } } }, "bloom_cognitive_process": { "primary": { "code": "2", "label": "Understand" }, "secondary": { "code": "3", "label": "Apply" } }, "bloom_knowledge_domain": { "primary": { "code": "2", "label": "Conceptual" }, "secondary": { "code": "3", "label": "Procedural" } }, "document_type_v1": { "primary": { "code": "6", "label": "Promotional/Advertisement" }, "secondary": { "code": "3", "label": "Reference/Encyclopedic/Educational" } }, "extraction_artifacts": { "primary": { "code": "0", "label": "No Artifacts" }, "secondary": { "code": "-1", "label": "Abstain" } }, "missing_content": { "primary": { "code": "0", "label": "No missing content" }, "secondary": { "code": "2", "label": "Click Here References" } }, "document_type_v2": { "primary": { "code": "16", "label": "Personal Blog" }, "secondary": { "code": "10", "label": "Knowledge Article" } }, "reasoning_depth": { "primary": { "code": "2", "label": "Basic Reasoning" }, "secondary": { "code": "1", "label": "No Reasoning" } }, "technical_correctness": { "primary": { "code": "6", "label": "Not Applicable/Indeterminate" }, "secondary": { "code": "3", "label": "Mostly Correct" } }, "education_level": { "primary": { "code": "1", "label": "General Audience" }, "secondary": { "code": "2", "label": "High School Level" } } }
672f1e42c33a7f9846924a2431ea77df
6,913,800,171,819,810,000
EVERQUEST The original game!   While Everquest was not the first MMORPG (Massive Multi-player On-Line Role Playing Game), it certainly reinvented the genre and is the inspiration behind all of the MMORPG's out there today, including the World of Warcraft. Everquest took MMORPG's into the 3D realm (360-degree environment). It was originally released March 16, 1999, developed by 989 Studios and Verant Studio's (later fully acquired by Sony On-Line Entertainment, aka SOE). I recall when this first came out. I was highly annoyed because they not only wanted me to buy the game, but they also wanted me to pay a monthly fee in order to be able to play! Theses days, this is a standard practice! I returned the game to the store because I was so peeved by the idea of having to BUY the game and then PAY a monthly fee on top of that! However... I was later lured back to the game by a friend and I have been addicted to it ever since! The game is so addictive that players started calling it "Ever Crack." The game has recieved much love and much hate since it's inception... being blamed for things on both ends of the spectrum... from divorces to marriages. Back in this era, the world of Norrath included Antonica, Odus, Faydwer and the "original" planes (Hate, Fear, Sky). Most players, today, literally don't know how easy they have it, compared to how it was then. The top level was 50; there was no tutorial; you were born naked and penniless; there was no compass; no maps; no task window; no fellowships; no guild windows; no raid windows (or raid grouping); no clickies; no familiars; no alternate advancement; blocky primitive graphics; no mercernaries; copper was valuable; coins could be dropped (they were actual items); no instanced zones; you had to recover your corpse (even at Level 1); you lost experience with deaths (even at Level 1); there were no task windows; and on and on... I recall the days of killing rats just outside Freeport, so I could run to the guards for help; looting and selling every item; and then saving every copper just to buy myself cloth armor... that is, after buying rations and water! My first weapon was, in fact, a rusty sword that I looted from a skeleton... and I was thrilled to have it! How many remember when Befallen was overrun with dark elves? That retrieving your corpse was a major undertaking? When 1 Silver or 1 Gold was actually a LOT of money? When Copper, Silver, Gold and Platinum was droppable and could become very, very heavy? When those rats by the city gates were, in fact, a challenge to kill? When the graphics were boxy and primitive? When sitting to meditate, the spell book would take up and covere the entire screen so you couldn't see a thing? When the player trade market was via "shout" in the East Commonlands tunnel (or near the Freeport bank)? To view a players wares they had to open a trade window with you so you could look at it (you could not link items)? When there were no portals and you either had to run zone-to-zone, or you had to pay for a ride from a Wizard or Druid? Good times! This page will cover topics related to the original game, or things that are basic or general game mechanics that are not necessarily related to a single expansion (i.e. abilities, skills...). . . . TOPIC LINKS . The below links are either related to the original Everquest, the original Everquest zones or are somewhat "all-expansion" generic links (such as game feature links, even if those features were updated with later expansions).  PROGRESSION RELATED   WEAPONS & ITEMS 100 Percent Weight Reduction Bags  ARMOR Qeynos Badge of Nobility SPELLS AND TOMES     MISC. Alcohol Tolerance Begging Bind Wound Foraging Sense Editing Swimming Taunt EXPANSION LINKS Bonzz's Home Page --> EVERQUEST (ORIGINAL) Ruins of Kunark (1st) Scars of Velious (2nd) Shadows of Luclin (3rd) Planes of Power (4th) Legacy of Ykesha (5th) Lost Dungeons of Norrath (6th) Gates of Discord (7th) Omens of War (8th) Dragons of Norrath (9th) Depth of Darkhollow (10th) Prophecy of Ro (11th) The Serpents Spine (12th) The Buried Sea (13th) Secrets of Faydwer (14th) Seeds of Destruction (15th) Underfoot (16th) House of Thule (17th) Veil of Alaris (18th) Rain of Fear (19th) . If you note any errors, misspellings, item name errors, misinformation or anything that needs addressing on this page -- PLEASE let me know! E-Mail me here (click here)! HOME This Page Last updated August 2, 2013
{ "url": "http://www.bonzz.com/everquest.htm", "source_domain": "www.bonzz.com", "snapshot_id": "crawl=CC-MAIN-2017-34", "warc_metadata": { "Content-Length": "20181", "Content-Type": "application/http; msgtype=response", "WARC-Block-Digest": "sha1:GIAWLUXZTAPTNZY4DABLMOPZGNTTC6HB", "WARC-Concurrent-To": "<urn:uuid:40e26581-0cc5-461e-b40a-0f4b81bfc41d>", "WARC-Date": "2017-08-17T03:32:16Z", "WARC-IP-Address": "69.65.3.245", "WARC-Identified-Payload-Type": "application/xhtml+xml", "WARC-Payload-Digest": "sha1:6H4HWUJSNJZZLON7Z347JVE4O7W4RXHS", "WARC-Record-ID": "<urn:uuid:c080da3a-69fa-4c86-ba00-75046526c3af>", "WARC-Target-URI": "http://www.bonzz.com/everquest.htm", "WARC-Truncated": null, "WARC-Type": "response", "WARC-Warcinfo-ID": "<urn:uuid:1d1e9e23-c90a-420f-8842-7fdc624875c3>" }, "warc_info": "robots: classic\r\nhostname: ip-10-185-153-145.ec2.internal\r\nsoftware: Nutch 1.6 (CC)\r\nisPartOf: CC-MAIN-2017-34\r\noperator: Common Crawl Admin\r\ndescription: Wide crawl of the web for August 2017\r\npublisher: Common Crawl\r\nformat: WARC File Format 1.0\r\nconformsTo: http://bibnum.bnf.fr/WARC/WARC_ISO_28500_version1_latestdraft.pdf" }
{ "line_start_idx": [ 0, 10, 11, 30, 31, 33, 34, 259, 260, 479, 480, 839, 840, 1012, 1013, 1167, 1168, 1285, 1286, 1380, 1381, 1911, 1912, 2252, 2253, 3076, 3077, 3089, 3090, 3283, 3284, 3286, 3287, 3289, 3290, 3292, 3293, 3305, 3306, 3308, 3309, 3536, 3537, 3557, 3558, 3560, 3561, 3577, 3578, 3612, 3613, 3620, 3621, 3646, 3647, 3664, 3665, 3667, 3668, 3670, 3671, 3677, 3678, 3696, 3697, 3705, 3706, 3717, 3718, 3727, 3728, 3742, 3743, 3752, 3753, 3759, 3760, 3776, 3777, 3795, 3796, 3821, 3822, 3844, 3845, 3868, 3869, 3893, 3894, 3916, 3917, 3940, 3941, 3972, 3973, 3996, 3997, 4016, 4017, 4042, 4043, 4070, 4071, 4093, 4094, 4120, 4121, 4143, 4144, 4170, 4171, 4199, 4200, 4217, 4218, 4240, 4241, 4263, 4264, 4284, 4285, 4287, 4288, 4458, 4459, 4464, 4465 ], "line_end_idx": [ 10, 11, 30, 31, 33, 34, 259, 260, 479, 480, 839, 840, 1012, 1013, 1167, 1168, 1285, 1286, 1380, 1381, 1911, 1912, 2252, 2253, 3076, 3077, 3089, 3090, 3283, 3284, 3286, 3287, 3289, 3290, 3292, 3293, 3305, 3306, 3308, 3309, 3536, 3537, 3557, 3558, 3560, 3561, 3577, 3578, 3612, 3613, 3620, 3621, 3646, 3647, 3664, 3665, 3667, 3668, 3670, 3671, 3677, 3678, 3696, 3697, 3705, 3706, 3717, 3718, 3727, 3728, 3742, 3743, 3752, 3753, 3759, 3760, 3776, 3777, 3795, 3796, 3821, 3822, 3844, 3845, 3868, 3869, 3893, 3894, 3916, 3917, 3940, 3941, 3972, 3973, 3996, 3997, 4016, 4017, 4042, 4043, 4070, 4071, 4093, 4094, 4120, 4121, 4143, 4144, 4170, 4171, 4199, 4200, 4217, 4218, 4240, 4241, 4263, 4264, 4284, 4285, 4287, 4288, 4458, 4459, 4464, 4465, 4502 ] }
{ "red_pajama_v2": { "ccnet_original_length": 4502, "ccnet_original_nlines": 126, "rps_doc_curly_bracket": 0, "rps_doc_ldnoobw_words": 0, "rps_doc_lorem_ipsum": 0, "rps_doc_stop_word_fraction": 0.3636363744735718, "rps_doc_ut1_blacklist": 0, "rps_doc_frac_all_caps_words": 0.038662489503622055, "rps_doc_frac_lines_end_with_ellipsis": 0.007874020375311375, "rps_doc_frac_no_alph_words": 0.19853709638118744, "rps_doc_frac_unique_words": 0.5203145742416382, "rps_doc_mean_word_length": 4.520314693450928, "rps_doc_num_sentences": 42, "rps_doc_symbol_to_word_ratio": 0.007314519956707954, "rps_doc_unigram_entropy": 5.470211029052734, "rps_doc_word_count": 763, "rps_doc_frac_chars_dupe_10grams": 0, "rps_doc_frac_chars_dupe_5grams": 0, "rps_doc_frac_chars_dupe_6grams": 0, "rps_doc_frac_chars_dupe_7grams": 0, "rps_doc_frac_chars_dupe_8grams": 0, "rps_doc_frac_chars_dupe_9grams": 0, "rps_doc_frac_chars_top_2gram": 0.012177440337836742, "rps_doc_frac_chars_top_3gram": 0.011597559787333012, "rps_doc_frac_chars_top_4gram": 0.006958540063351393, "rps_doc_books_importance": -476.05694580078125, "rps_doc_books_importance_length_correction": -476.05694580078125, "rps_doc_openwebtext_importance": -244.8661651611328, "rps_doc_openwebtext_importance_length_correction": -244.8661651611328, "rps_doc_wikipedia_importance": -185.66680908203125, "rps_doc_wikipedia_importance_length_correction": -185.66680908203125 }, "fasttext": { "dclm": 0.036840979009866714, "english": 0.9658003449440002, "fineweb_edu_approx": 1.5074585676193237, "eai_general_math": 0.06401556730270386, "eai_open_web_math": 0.27060776948928833, "eai_web_code": 0.013163980096578598 } }
{ "free_decimal_correspondence": { "primary": { "code": "004.67", "labels": { "level_1": "General works, books and libraries, information sciences", "level_2": "", "level_3": "Computers and Computer science" } }, "secondary": { "code": "794.8", "labels": { "level_1": "Arts", "level_2": "Amusements and Recreation", "level_3": "" } } }, "bloom_cognitive_process": { "primary": { "code": "2", "label": "Understand" }, "secondary": { "code": "1", "label": "Remember" } }, "bloom_knowledge_domain": { "primary": { "code": "2", "label": "Conceptual" }, "secondary": { "code": "1", "label": "Factual" } }, "document_type_v1": { "primary": { "code": "3", "label": "Reference/Encyclopedic/Educational" }, "secondary": { "code": "9", "label": "Personal/Misc" } }, "extraction_artifacts": { "primary": { "code": "3", "label": "Irrelevant Content" }, "secondary": { "code": "0", "label": "No Artifacts" } }, "missing_content": { "primary": { "code": "0", "label": "No missing content" }, "secondary": { "code": "1", "label": "Truncated Snippets" } }, "document_type_v2": { "primary": { "code": "16", "label": "Personal Blog" }, "secondary": { "code": "6", "label": "Content Listing" } }, "reasoning_depth": { "primary": { "code": "2", "label": "Basic Reasoning" }, "secondary": { "code": "1", "label": "No Reasoning" } }, "technical_correctness": { "primary": { "code": "6", "label": "Not Applicable/Indeterminate" }, "secondary": { "code": "3", "label": "Mostly Correct" } }, "education_level": { "primary": { "code": "1", "label": "General Audience" }, "secondary": { "code": "2", "label": "High School Level" } } }
672f1e42c33a7f9846924a2431ea77df
6,192,634,814,556,493,000
This Is Me--2024 A to Z Theme My A to Z Themes in the past have covered a range of topics and for 2024 the theme is a personal retrospective that I call "I Coulda Been" which is in reference to my job and career arc over my lifetime. I'll be looking at all sorts of occupations that I have done or could have done. Maybe you've done some of these too! Monday, March 26, 2012 Can't You Please Just Make It Simple? Spitzer - base do telescopioSpitzer - base do telescopio (Photo credit: Wikipedia)            Maybe it has something to do with Amazon wanting to keep you on their site longer.  And of course Amazon's not the only one.    A lot of web sites seem to want to make you jump through hoops to find things or do things on their sites.  Internet hell is what I've found sometimes.  It's like being locked inside a store until you buy something.          On the blog site bridgetstraub this question was raised in relation to finding a particular book on Amazon.  She related some of the problems she ran into finding a particular book and wanting to leave a review.  As she admits, her techie skills might not be the best, but I can certainly relate to that.  However the point she raised made me start contemplating some of the internet hassles I've dealt with and so have many of you I would guess.            Fortunately, I've learned to navigate Amazon and some of those sites pretty well, but now and then I still get caught in a frustrating quagmire.  I've even managed to get pretty adept at leaving comments on WordPress and the annoying systems like CommentLuv, Intense Debate, and Disqus and the like.  Oh sure I'm a big blog baby some of you may say.   Well, that's what I think when I hear complaints about commenting on Blogspot sites. What's so difficult about leaving a comment on a Blogger site?           I guess it's a matter of what we're used to.  I'd be fine if Blogger stopped changing stuff so I didn't have to stop and think and relearn new processes.    It's like the automated phone systems that keep asking you to press this button or that and then keep you on line waiting for a long time.         And then there's those automated phone systems that say, "Please listen closely as some of our options have changed."   My health care provider has had that message on their system for like seven years now.  Can you please stop changing the options already people?  Either that or change the message.          Don't even get me started on the crazy lady giving directions on those navigation systems.   Have you ever driven in circles with that lady scolding you about missing your turn and telling you to go back.           Time-saving technology sure seems to waste a lot of time.         What website loop-de-loops particularly annoy you?   What about those automated phone systems?   How often do you actually talk to real people anymore?  I mean people that are actually helpful? Enhanced by Zemanta 40 comments: 1. The thing that still really annoys me is word verification! http://duncaninkuantan.blogspot.com/2012/03/z-fever-hits-our-home-called-kuantan.html ReplyDelete 2. Hi Lee - I still can't comment via Chrome on embedded comments, but now can as long as I use the IE plug in within Chrome. I don't get comments from others blogs eg this one .. and that's a little frustrating .. At least I can comment for the A - Z .. and like you I've found my way round .. Cheers everyone - hate telephone systems!! and complicated websites - I escape ... and have been known to switch off .. looking forward to seeing you all next Sunday onwards - Hilary ReplyDelete 3. I'm with you completely on this Lee. Just like you I find it disconcerting some of these comment systems that we see on various people's Wordpresses and such. I also believe in the old adage that if it's not broken then don't fix it. As we know Blogger is one of the world's most popular websites and I just can't understand why the people in charge believe messing around with it and changing things can benefit such an awesome website though I guess I've been privileged to do this blogging thing for free so I can't and shall not complain! ReplyDelete 4. I tried chrome but gave up, yet chrome is the only explorer i can use on Face Book without getting it freezing up. Yvonne. ReplyDelete 5. I wasn't aware anything had changed on my blog until someone told me in strong words. Then I had to check to see if my chosen setting had been change and low and behold Google/Blogger had change my setting without me knowing. A quiet, gentle word in my ear would have made me check. I hope I've made it easier for others to leave comments on my blog, but who know Google/Blogger may have change things again ReplyDelete 6. voice recognition systems give me the willies ReplyDelete 7. I agree with you on the Satnav part, I always get "Make a U-turn when possible" even though most of the time I'm in a place where it would be impossible to do a U-turn. Either that or the voice would tell me to turn into a road that doesn't exist, that's happened to me quite a lot. ReplyDelete 8. I'd have to say for me it's the variety of devices to leave comments. I upload a few different ones over the past two years, but eventually returned to the simplicity of Blogger commenting--well, before they just messed it up again. lol I would love it if Blogger had a feature where a blogger replies directly from the blog comments to a follower. That would be cool. ReplyDelete 9. I got frustrated a few weeks ago at the ...PHONE compnay. I could not beleive the amount of hoops I had to jump through to finally get to talk to someone only to then WAIT. ReplyDelete 10. I used to work in a call centre, it's all a total scam. Most of the time the callers would come through to the same centre regardless of the options they had pressed, then we would pass them on to the correct place! :) ReplyDelete 11. I'm tired of the current trend that has patients jumping through hoops to get a message to the doctor. It has been going on for years but is becoming more prevalent. When I call my doctor's office and can't leave voicemail for the nurse anymore because the new system makes me go through someone who is unlikely to relay the message properly, that gets on my last nerve. Ellie ReplyDelete 12. I don't think they ever change those prompts - they're just forcing you to listen to everything so it takes you longer to reach the right option. I'm good with Disqus. It's the best embedded comment box I've seen. ReplyDelete 13. There are EPs (EVIL PROGRAMMERS) that have been unleashed into this cyberworld and their sole purpose is to keep the bloggers busy figuring out how to post, how to find information and how to read those #!@%!!! word verifications. Then when bloggers figure it all out, the EPs change the way to do it! Very simple. There is a conspiracy afloat. ReplyDelete 14. so true... i am one of those guys who thinks he knows computer stuff, amazon i really only check a few things, so i know what i am doing. i am afraid to anything on the phone with all the internet options, i am lost... my problem is my computer is protected by programs to prevent crud getting onto my computer. what about the phone, what protects my data or who is following me. if you all before downloading an app, read what info you are giving out... what the app makers are taking from you. sorry i steered off course again, but in my defense the navigation lady told me to go this way. Jeremy [Retro-Zombie] Visit The Madness: iZOMBIE ReplyDelete 15. Duncan --Keep spreading the word and we may continue to see less WV. Hilary -- There seems to be a definite inconsistency among different users. I try to leave responses to comments but you not being notified is disappointing. I leave my comments on the pop-out boxes but I usually forget to go back to see if there was a response. I still don't know how to approach this. Matthew YW-- It's frustrating for sure, but like you say it's free. Yvonne -- Wish I could help. Chrome works very well for me. Jamara -- I'm sure Google will keep changing things if for no other reason than to annoy us. Delores -- Voice recognition is strange, but when it works it's kind of cool. It's another job for people companies can scrap. Loopyloo -- More than once have I dead-ended at a road and looked across a freeway at my destination. S.A.L.-- Anything that would save time and be more effective would be nice. Jennifer -- I've been there too. What a waste of our time. Anna -- It's a labyrinth callers need to navigate through to sometimes get nowhere. Ellie -- Our doctors' network would prefer that we use email, but it doesn't seem that effective when there are questions and answers involved. Karen -- The real people are home unemployed. They should ban the automated systems and give more people jobs. Alex -- I seem to get through Disqus now, but I still find it to be a hassle. Lee ReplyDelete 16. C.Lee-- I think it's mostly a case of programmer's who are bored and just want to watch users squirm. They are definitely masochists. Jeremy -- I'm not sure why I even have a cell phone. I only use it for phone calls. Not even sure about apps or texting. Lee ReplyDelete 17. Not too long ago, I was somehow getting add-ons that stole my browser and that really irritated me. I don't remember how I got rid of all that, but am glad that I did. I don't really buy much online, so my biggest complaint now is easily leaving comments. As for the phone stuff, I try to out-wait those messages to get to a real person. Sometimes if they cycle through twice that will get you over to somebody live, because they think you might be a dinosaur without a touch tone. Looking forward to A-Z ... I've been prepping my posts and am so excited. Kathy M. ReplyDelete 18. Hello Lee. The only thing I've had to struggle with on Blogger is scheduling my blog posts. I spent the whole day with someone from England trying to sort it out. We did eventually but it still took hours of back and forth. The other thing that I have difficulty with is word verification, I can't read the words, if that's what they are, and find when having to struggle for too long I will give up and just move on. I'll have to see about the A-z, last year all hell broke out with my computer when I did take part. We'll see. Blessings, Geoff. ReplyDelete 19. The automated phone systems I hate are the ones that call me. ReplyDelete 20. Kathy -- I've found that on those phone systems usually if I just press "0" I'll get a person, but not always. Geoff -- With practice I've gotten pretty at those Word Verifications. Actually I'm thinking that if you get close it will sometimes accept whatever you put in--I'm pretty sure I don't always get it right and it usually goes through. Andrew -- At least I know if an automated phone service calls me it's probably not important and I can hang up on them. Lee ReplyDelete 21. Blogger works fine if the person leaving the comment is logged into their own Blogger or gmail account. For those outside that system, seems to be problematic. Auto-phone systems are the worst, IMO. When I have to make "those kinds of calls," at work (Several every week), I line up some other project I can work on while on hold and passing through. Thanks for sharing your pain in a funny way. :-) ReplyDelete 22. The phone loops drive me the nuttiest! I looped myself right out of the system twice last week trying to contact someone in an organization! ReplyDelete 23. I have a difficult time trying to find where to comment sometimes. Do the options ever really change? I think it is a conspiracy. ReplyDelete 24. You pretty much hit it on target when you mentioned it all being a matter of what we're accustomed to using. Me personally, I happen to like CommentLUV, Intense Debate and Disqus but not necessarily in that order, yet, there are many other bloggers who loathe those services with regards to comments. Commenting on Blogger blogs are relatively simple but I've noticed that it can be a challenge for people who may not have a Google account or any of the other comment options....especially if the blogger has disabled the Anonymous or the Name/URL options. The Home Depot and Kmart websites annoy me as well as the websites for wireless carriers and cable TV companies....yeah, any on of them, you name it, I've had issues with their site. I mean, really, it shouldn't be that difficult to shop for a phone or wireless service or upgrade my plans and I find myself in a web of frustration every time I shop through those websites. Thanks for bringing up the hell that is those automated phone systems. I know you mentioned "internet hell" in this post but I'd say it would be fair to just lump the websites in with the automated phone systems and call it all Telecommunications Hell! I rarely get to talk to real people unless I'm seeking customer service in person, at a brick and mortar location. Otherwise, I'm on hold with bad elevator music and sporadic check-ins from computer systems that claim "someone will be with you shortly." Yeah, gotta love it. Oh and "People that are actually helpful," you ask? You're kidding right? I think the human helpful service reps went the way of beepers/pagers, VHS tapes and Britney Spears' teenybopper stardom! ;) ~Nicole Blog: The Madlab Post @MadlabPost on Twitter ReplyDelete 25. I left a long comment on this post earlier this evening and now it's missing. Strange. Anydoodle....I like Disqus and CommentLUV but those automated phone systems are annoying with a capital 'A' for sure. The cell phone and cable companies have difficult websites that are not easy to navigate and Home Depot and Kmart are both up there in the annoying category as well. ~Nicole Blog: The Madlab Post @MadlabPost on Twitter ReplyDelete 26. I'm a techno-phobe... not by choice though... CommentLuv? Intense Debate? Disqus? Phew... sonds Greek to me LOL! But I'll get there... I actually came face-to-face with an iPad for the FIRST time, about 2 weeks ago - and you know what? It seemed simple enough... my hubby said he's going to get one for me! Imagine that? So there is hope after all... ReplyDelete 27. I just started using ebay. I am way behind on the times. The ebay craze is over, I know that. It will be years before I get to Amazon. ReplyDelete 28. Nothing aggravates me as much as an automated phone system with no option to speak with a living, breathing person. ReplyDelete 29. It is the face of the future...being annoyed! I miss real people ;D Word verification and automated calls are the worst, so far...for me. The military pharmacy is a real hoot! You have to keep entering the last 4 of your social security number followed by the # or nothing happens. Then the locations for pick up are numbered...mine is the last number. It is an annoying MALE voice, drone robotic like. Great post Lee! I do have to say, when I get real person, it isn't always better. It is if they can answer your question. Most of the time I am tossed into another automated list ;D ReplyDelete 30. A new one I've encountered for customer support/service is "leave your number an we will call you back". The automated service calls you back and then hangs up on you. ReplyDelete 31. I used to work in customer service for a company that also had an automated line for quick issues. It used to frustrate me when people would refuse to use the automated system and opt to talk to a live person instead. Our automated system was so easy! But I realize that it really is what you're used to and some people just like to do things in whatever way they feel is easiest. Amazon is one of the most confusing sites ever. ReplyDelete 32. Beverly --I always try to keep reading material or some activity to keep me busy when I'm waiting for the phone systems or for my computer to do something. Jemi -- The phone systems can become a vicious circle. Ciara -- Comments can be tricky sometimes and then other times there really isn't a place to leave comments. I guess they're not interested in interaction. Nicole -- I hate that you would have spent time on a comment that disappeared. I've had that happen too. Just another part of the problem. Michelle -- Once you get used to something it can be okay, but it's that initial breaking in period that is troubling. Nellie -- I've been thinking about getting onto EBay. It's seems like some things would be easier to sell there than on Craig's List. Don't tell be the craze is over. Brinda -- Usually if I do get somebody they aren't of much help. Quality customer service has become a problem. Ella -- That's funny--the face of the future is being annoyed. Well, maybe not funny, but it sounds funny. Southpaw -- Maybe nobody really wants to talk to us anymore. Lee ReplyDelete 33. Karen -- Some automated sites are easy and I'd rather deal with them than a real person, but sometimes more explanation is involved and you can get into a lot of frustrating back and forth that makes little progress. Lee ReplyDelete 34. Banking on the phone drives me crazy with the press this, now press that, now remember this etc.. And, social media sites that are ever-evolving, changing formats and privacy rules - all to justify (IMHO) some tech's reason for existence. ReplyDelete 35. You caught me in just the right mood. I have been dealing with Verizon for the last 36 hours. I've called them four times and each time I have to answer a computer by keying in numbers 12 times before I can go on hold to speak to a person. Grrrrr ReplyDelete 36. You have definitely hit a nerve here. I'm learning the ropes with the automated phone situation, how to get around it, but it's an ongoing education. My latest issue is with blogger messing up the option for follow-up comments. (I think we've discussed this before.) Where is it!!! Example: The only way that I NOW can see if you respond to me is to come back to the post. I'm not a consistent blogger, so if I don't note somewhere that I commented on this post, I won't remember to come back to see if you responded, or to hear what others are saying. It totally messes up the conversation to not have that "follow-up comments" option. I've googled and read that there is something to do to get it, but I'm afraid I'll mess up something else if I try it. Can you tell I'm urked? ReplyDelete 37. BOIDMAN ~ Have you seen Blogger's new message on your Dashboard? It says words to the effect of: Blogger is getting a makeover in April. Upgrade now by clicking here. Uh-Oh! You know what that means! The website programmers have felt a need to justify their jobs again, and so they are about to fix the unbroken and in the process, break things that are currently working just fine. Wait for it... Wait for it, people... New Blogger bugs are being created for us as we speak... ~ D-FensDogg 'Loyal American Underground' ReplyDelete 38. I sometimes struggle just to find the "X comments" button! Sometimes it's in a really small font, hidden amongst a whole lot of other really small words at the bottom of someone's post. I have to search for a while before I find it! ReplyDelete 39. After some relentless research, I have figured out my inability to receive follow-up comments from your posts and those of others. There are 3 options for the way that comment screens are displayed. Yours is a pop-up. Mine was a full screen. These 2 options do not have a phrase "subscribe by email" in the comment area. If you change your comment screen to be displayed under the comments (the Embedded format), there will be the option to "subscribe by email." I would be able to click on it and therefore, all future comments on the post will be sent to my email inbox. I changed mine, and I hope not to have any issues. To do so, you go to dashboard, then settings, then comments, and you'll see the 3 options. Don't forget to click "Save settings" at the bottom. I wondered why some people have "reply" options under each comment on their post. Of course, this is your decision. However, I'm pushing, because I like to hear what your readers respond back to your posts. :) ReplyDelete 40. cool ill await for your a-z hope you visit mine too here http://pa-ul.blogspot.com/2012/03/z-artists.html ReplyDelete Go ahead and say something. Don't be afraid to speak your mind. I normally try to respond to all comments in the comment section so please remember to check the "Email follow-up comments" box if you want to participate in the comment conversation. For Battle of the Bands voting the "Anonymous" commenting option has been made available though this version is the least preferred. If voting using "anonymous" please include in your comment your name (first only is okay) and city you are voting from and the reason you chose the artist you did. If you know me and want to comment but don't want to do it here, then you can send me an email @ jacksonlee51 at aol dot com. Lee
{ "url": "https://tossingitout.blogspot.com/2012/03/cant-you-please-just-make-it-simple.html", "source_domain": "tossingitout.blogspot.com", "snapshot_id": "CC-MAIN-2024-22", "warc_metadata": { "Content-Length": "236234", "Content-Type": "application/http; msgtype=response", "WARC-Block-Digest": "sha1:RW2I7JHNJXN2TJAFHF2HV4EXRDRHFGBQ", "WARC-Concurrent-To": "<urn:uuid:66a1df06-b115-4bc4-8c97-e16e1a49ffbf>", "WARC-Date": "2024-05-29T20:38:02Z", "WARC-IP-Address": "142.251.111.132", "WARC-Identified-Payload-Type": "application/xhtml+xml", "WARC-Payload-Digest": "sha1:2KS3VIM42BTNEULIW3H7UVMS3VASQKWQ", "WARC-Record-ID": "<urn:uuid:6d9bb994-33e3-463d-b0e9-bdb910a8095d>", "WARC-Target-URI": "https://tossingitout.blogspot.com/2012/03/cant-you-please-just-make-it-simple.html", "WARC-Truncated": null, "WARC-Type": "response", "WARC-Warcinfo-ID": "<urn:uuid:d95b127a-0a3a-4e76-ad71-0007a116598a>" }, "warc_info": "isPartOf: CC-MAIN-2024-22\r\npublisher: Common Crawl\r\ndescription: Wide crawl of the web for May 2024\r\noperator: Common Crawl Admin ([email protected])\r\nhostname: ip-10-67-67-204\r\nsoftware: Apache Nutch 1.19 (modified, https://github.com/commoncrawl/nutch/)\r\nrobots: checked via crawler-commons 1.5-SNAPSHOT (https://github.com/crawler-commons/crawler-commons)\r\nformat: WARC File Format 1.1\r\nconformsTo: https://iipc.github.io/warc-specifications/specifications/warc-format/warc-1.1/" }
{ "line_start_idx": [ 0, 30, 31, 353, 354, 377, 378, 416, 417, 500, 501, 856, 857, 1313, 1314, 1825, 1826, 2132, 2133, 2442, 2443, 2659, 2660, 2726, 2727, 2929, 2930, 2931, 2932, 2933, 2953, 2954, 2967, 2968, 3033, 3034, 3124, 3125, 3141, 3269, 3270, 3363, 3364, 3448, 3449, 3636, 3637, 3653, 4201, 4202, 4218, 4338, 4339, 4351, 4352, 4368, 4599, 4600, 4786, 4787, 4803, 4854, 4855, 4871, 5159, 5160, 5176, 5418, 5419, 5555, 5556, 5572, 5750, 5751, 5767, 5992, 5993, 6009, 6386, 6387, 6397, 6398, 6414, 6566, 6638, 6639, 6655, 7006, 7007, 7023, 7525, 7526, 7626, 7627, 7653, 7676, 7688, 7689, 7705, 7780, 7781, 8089, 8090, 8162, 8163, 8227, 8228, 8325, 8326, 8457, 8458, 8564, 8565, 8645, 8646, 8709, 8710, 8798, 8799, 8947, 8948, 9063, 9064, 9146, 9147, 9155, 9156, 9172, 9312, 9313, 9438, 9439, 9447, 9448, 9464, 9638, 9639, 9731, 9732, 9962, 9963, 10041, 10042, 10055, 10056, 10072, 10089, 10306, 10504, 10619, 10641, 10642, 10658, 10726, 10727, 10743, 10860, 10861, 11099, 11100, 11224, 11225, 11233, 11234, 11250, 11416, 11417, 11612, 11613, 11666, 11667, 11683, 11830, 11831, 11847, 11983, 11984, 12000, 12307, 12308, 12568, 12569, 12947, 12948, 13205, 13206, 13464, 13465, 13490, 13491, 13694, 13695, 13707, 13733, 13760, 13761, 13777, 13988, 13989, 14159, 14160, 14172, 14198, 14225, 14226, 14242, 14294, 14365, 14391, 14567, 14615, 14616, 14632, 14773, 14774, 14790, 14912, 14913, 14929, 15003, 15077, 15119, 15229, 15370, 15540, 15541, 15557, 15731, 15732, 15748, 16135, 16136, 16188, 16189, 16205, 16367, 16368, 16427, 16428, 16588, 16589, 16732, 16733, 16856, 16857, 17028, 17029, 17145, 17146, 17257, 17258, 17323, 17324, 17332, 17333, 17349, 17572, 17573, 17581, 17582, 17598, 17702, 17703, 17848, 17849, 17865, 18118, 18119, 18135, 18291, 18292, 18428, 18429, 18788, 18789, 18936, 18937, 18953, 18969, 19060, 19061, 19135, 19136, 19356, 19357, 19456, 19457, 19474, 19507, 19508, 19524, 19763, 19764, 19780, 19917, 19918, 20112, 20113, 20259, 20260, 20425, 20426, 20574, 20575, 20661, 20662, 20794, 20795, 20811, 20923, 20924, 20940, 20941, 21005, 21189, 21190, 21487, 21488, 21614, 21615 ], "line_end_idx": [ 30, 31, 353, 354, 377, 378, 416, 417, 500, 501, 856, 857, 1313, 1314, 1825, 1826, 2132, 2133, 2442, 2443, 2659, 2660, 2726, 2727, 2929, 2930, 2931, 2932, 2933, 2953, 2954, 2967, 2968, 3033, 3034, 3124, 3125, 3141, 3269, 3270, 3363, 3364, 3448, 3449, 3636, 3637, 3653, 4201, 4202, 4218, 4338, 4339, 4351, 4352, 4368, 4599, 4600, 4786, 4787, 4803, 4854, 4855, 4871, 5159, 5160, 5176, 5418, 5419, 5555, 5556, 5572, 5750, 5751, 5767, 5992, 5993, 6009, 6386, 6387, 6397, 6398, 6414, 6566, 6638, 6639, 6655, 7006, 7007, 7023, 7525, 7526, 7626, 7627, 7653, 7676, 7688, 7689, 7705, 7780, 7781, 8089, 8090, 8162, 8163, 8227, 8228, 8325, 8326, 8457, 8458, 8564, 8565, 8645, 8646, 8709, 8710, 8798, 8799, 8947, 8948, 9063, 9064, 9146, 9147, 9155, 9156, 9172, 9312, 9313, 9438, 9439, 9447, 9448, 9464, 9638, 9639, 9731, 9732, 9962, 9963, 10041, 10042, 10055, 10056, 10072, 10089, 10306, 10504, 10619, 10641, 10642, 10658, 10726, 10727, 10743, 10860, 10861, 11099, 11100, 11224, 11225, 11233, 11234, 11250, 11416, 11417, 11612, 11613, 11666, 11667, 11683, 11830, 11831, 11847, 11983, 11984, 12000, 12307, 12308, 12568, 12569, 12947, 12948, 13205, 13206, 13464, 13465, 13490, 13491, 13694, 13695, 13707, 13733, 13760, 13761, 13777, 13988, 13989, 14159, 14160, 14172, 14198, 14225, 14226, 14242, 14294, 14365, 14391, 14567, 14615, 14616, 14632, 14773, 14774, 14790, 14912, 14913, 14929, 15003, 15077, 15119, 15229, 15370, 15540, 15541, 15557, 15731, 15732, 15748, 16135, 16136, 16188, 16189, 16205, 16367, 16368, 16427, 16428, 16588, 16589, 16732, 16733, 16856, 16857, 17028, 17029, 17145, 17146, 17257, 17258, 17323, 17324, 17332, 17333, 17349, 17572, 17573, 17581, 17582, 17598, 17702, 17703, 17848, 17849, 17865, 18118, 18119, 18135, 18291, 18292, 18428, 18429, 18788, 18789, 18936, 18937, 18953, 18969, 19060, 19061, 19135, 19136, 19356, 19357, 19456, 19457, 19474, 19507, 19508, 19524, 19763, 19764, 19780, 19917, 19918, 20112, 20113, 20259, 20260, 20425, 20426, 20574, 20575, 20661, 20662, 20794, 20795, 20811, 20923, 20924, 20940, 20941, 21005, 21189, 21190, 21487, 21488, 21614, 21615, 21618 ] }
{ "red_pajama_v2": { "ccnet_original_length": 21618, "ccnet_original_nlines": 319, "rps_doc_curly_bracket": 0, "rps_doc_ldnoobw_words": 0, "rps_doc_lorem_ipsum": 0, "rps_doc_stop_word_fraction": 0.4588055908679962, "rps_doc_ut1_blacklist": 0, "rps_doc_frac_all_caps_words": 0.04337402060627937, "rps_doc_frac_lines_end_with_ellipsis": 0.012500000186264515, "rps_doc_frac_no_alph_words": 0.16150827705860138, "rps_doc_frac_unique_words": 0.27925747632980347, "rps_doc_mean_word_length": 4.364810466766357, "rps_doc_num_sentences": 307, "rps_doc_symbol_to_word_ratio": 0.005013079848140478, "rps_doc_unigram_entropy": 5.908409118652344, "rps_doc_word_count": 3717, "rps_doc_frac_chars_dupe_10grams": 0, "rps_doc_frac_chars_dupe_5grams": 0.018121300265192986, "rps_doc_frac_chars_dupe_6grams": 0.010971399955451488, "rps_doc_frac_chars_dupe_7grams": 0.0065335300751030445, "rps_doc_frac_chars_dupe_8grams": 0.0065335300751030445, "rps_doc_frac_chars_dupe_9grams": 0.0065335300751030445, "rps_doc_frac_chars_top_2gram": 0.0046227797865867615, "rps_doc_frac_chars_top_3gram": 0.009060650132596493, "rps_doc_frac_chars_top_4gram": 0.00641026021912694, "rps_doc_books_importance": -2247.6982421875, "rps_doc_books_importance_length_correction": -2247.6982421875, "rps_doc_openwebtext_importance": -1219.778564453125, "rps_doc_openwebtext_importance_length_correction": -1219.778564453125, "rps_doc_wikipedia_importance": -757.4375610351562, "rps_doc_wikipedia_importance_length_correction": -757.4375610351562 }, "fasttext": { "dclm": 0.04813193902373314, "english": 0.9691693782806396, "fineweb_edu_approx": 0.9532648324966431, "eai_general_math": 0.041209518909454346, "eai_open_web_math": 0.24895095825195312, "eai_web_code": 0.01875448040664196 } }
{ "free_decimal_correspondence": { "primary": { "code": "004.67", "labels": { "level_1": "General works, books and libraries, information sciences", "level_2": "", "level_3": "Computers and Computer science" } }, "secondary": { "code": "005.1", "labels": { "level_1": "General works, books and libraries, information sciences", "level_2": "", "level_3": "Computer programming" } } }, "bloom_cognitive_process": { "primary": { "code": "5", "label": "Evaluate" }, "secondary": { "code": "2", "label": "Understand" } }, "bloom_knowledge_domain": { "primary": { "code": "2", "label": "Conceptual" }, "secondary": { "code": "3", "label": "Procedural" } }, "document_type_v1": { "primary": { "code": "9", "label": "Personal/Misc" }, "secondary": { "code": "-1", "label": "Abstain" } }, "extraction_artifacts": { "primary": { "code": "3", "label": "Irrelevant Content" }, "secondary": { "code": "0", "label": "No Artifacts" } }, "missing_content": { "primary": { "code": "0", "label": "No missing content" }, "secondary": { "code": "-1", "label": "Abstain" } }, "document_type_v2": { "primary": { "code": "16", "label": "Personal Blog" }, "secondary": { "code": "5", "label": "Comment Section" } }, "reasoning_depth": { "primary": { "code": "2", "label": "Basic Reasoning" }, "secondary": { "code": "3", "label": "Intermediate Reasoning" } }, "technical_correctness": { "primary": { "code": "6", "label": "Not Applicable/Indeterminate" }, "secondary": { "code": "2", "label": "Partially Correct" } }, "education_level": { "primary": { "code": "1", "label": "General Audience" }, "secondary": { "code": "2", "label": "High School Level" } } }
672f1e42c33a7f9846924a2431ea77df
2,148,833,083,045,104,600
onfire247 Enthusiast Enthusiast How to install Zoom correctly in an instant clone environment Since there seems to be a bit of confusion on how to do Zoom correctly in VDI, I thought I'd post how I successfully setup Zoom in an instant clone Horizon environment. 1. In DEM, click on the Download Config Template to load the pre-configured Zoom setting into DEM. 2. Browse to the VMwareDEMConfig\Config\general\Applications folder. 3. Rename all three Zoom icons, changing "Zoom" to "ZoomVDI" in all three icons. 4. In DEM, hit Refresh Tree. You should now see a ZoomVDI option under Applications. 5. In the ZoomVDI Import/Export tab, change [IncludeFolderTrees] <AppData>\Zoom to <AppData>\ZoomVDI 6. In the DirectFlex Tab, Add a new executable: C:\Program Files (x86)\ZoomVDI\bin\Zoom.exe 7. Delete the other executable until the previous line is the only executable. DirectFlex should still be checked at this point. 8. Click the Save Config File button 9. In the Golden Image, install the corporate version of Zoom for VDI (https://support.zoom.us/hc/en-us/articles/360041602711) [File name: ZoomInstallerVDI.msi] 10. Snapshot the image and push it to a test pool.   After following these steps, you should have a start menu icon called ZoomVDI in your test Windows 10 pool. Launch it and save your credentials. When you log out and back in, it should allow you to run the ZoomVDI icon with saved credentials. The profile size is about 13MB.   Have I missed any steps or configurations that I should make to improve my install? 0 Kudos 4 Replies mrkasius Enthusiast Enthusiast Hi onfire247, The DEM Zoom for VDI Config Template remembers out-of-the-box my saved credentials at the next login. 0 Kudos Mickeybyte Contributor Contributor @onfire247  As @mrkasius already pointed out, I recently published a config for Zoom for VDI in the communities. It has some extra exclusions to prevent the more than 2.000 emoticons to be zipped/unzipped everytime Zoom is started. It also contains an extra setting to be able to let the user choose "run at startup". Besides the DEM config, also don't forget if you want to use Zoom on VDI, there's also a client plugin that needs to be installed on the local computer, depending on what version you have installed in the VDI. On this page VDI Release Version 3.3.3 – Zoom Help Center you can find the installed for the VDI on the left column and the corresponding client plugins in the second column.   Regards, Mickeybyte onfire247 Enthusiast Enthusiast Thanks for the information guys. I saw the plugin but I was confused where it goes. We mostly use zero clients, but we do have a few of the Wyse 3000 series thin clients. Are you saying the plugin needs to be installed on those thin clients, or just people's personal PCs who use the Horizon client remotely? 0 Kudos Mickeybyte Contributor Contributor I've only worked with Windows computers, who need the plugin installed to make the optimized mode for Zoom work. I'm guessing the same goes for other types of clients: they need the local plugin installed or the plugin needs to be build into the firmware or something like that.     Regards, Mickeybyte 0 Kudos
{ "url": "https://communities.vmware.com/t5/Dynamic-Environment-Manager/How-to-install-Zoom-correctly-in-an-instant-clone-environment/m-p/2831643", "source_domain": "communities.vmware.com", "snapshot_id": "crawl=CC-MAIN-2021-10", "warc_metadata": { "Content-Length": "206498", "Content-Type": "application/http; msgtype=response", "WARC-Block-Digest": "sha1:HD6PJH7DVMYZXCWSUEKZN2SXQZAQSRFS", "WARC-Concurrent-To": "<urn:uuid:62ddcadb-c3d4-4f60-b69b-976994b5a117>", "WARC-Date": "2021-03-03T00:10:50Z", "WARC-IP-Address": "99.84.176.36", "WARC-Identified-Payload-Type": "text/html", "WARC-Payload-Digest": "sha1:VXKHKHZB46ZZWGSEBDQ6XCXLYNPHO77M", "WARC-Record-ID": "<urn:uuid:d90a6bdf-24d4-48e5-8e24-4310ee752785>", "WARC-Target-URI": "https://communities.vmware.com/t5/Dynamic-Environment-Manager/How-to-install-Zoom-correctly-in-an-instant-clone-environment/m-p/2831643", "WARC-Truncated": null, "WARC-Type": "response", "WARC-Warcinfo-ID": "<urn:uuid:8dfbac2e-e998-4644-853e-81f3651898d3>" }, "warc_info": "isPartOf: CC-MAIN-2021-10\r\npublisher: Common Crawl\r\ndescription: Wide crawl of the web for February/March 2021\r\noperator: Common Crawl Admin ([email protected])\r\nhostname: ip-10-67-67-128.ec2.internal\r\nsoftware: Apache Nutch 1.18 (modified, https://github.com/commoncrawl/nutch/)\r\nrobots: checked via crawler-commons 1.2-SNAPSHOT (https://github.com/crawler-commons/crawler-commons)\r\nformat: WARC File Format 1.1\r\nconformsTo: http://iipc.github.io/warc-specifications/specifications/warc-format/warc-1.1/" }
{ "line_start_idx": [ 0, 10, 21, 32, 33, 95, 96, 265, 266, 367, 438, 521, 608, 711, 805, 936, 975, 1138, 1191, 1192, 1194, 1195, 1470, 1471, 1473, 1474, 1558, 1559, 1567, 1577, 1586, 1597, 1608, 1609, 1623, 1624, 1726, 1727, 1735, 1746, 1758, 1770, 1771, 1783, 1784, 2090, 2091, 2476, 2477, 2479, 2480, 2489, 2500, 2510, 2521, 2532, 2533, 2842, 2843, 2851, 2862, 2874, 2886, 2887, 3166, 3167, 3169, 3170, 3172, 3173, 3182, 3193 ], "line_end_idx": [ 10, 21, 32, 33, 95, 96, 265, 266, 367, 438, 521, 608, 711, 805, 936, 975, 1138, 1191, 1192, 1194, 1195, 1470, 1471, 1473, 1474, 1558, 1559, 1567, 1577, 1586, 1597, 1608, 1609, 1623, 1624, 1726, 1727, 1735, 1746, 1758, 1770, 1771, 1783, 1784, 2090, 2091, 2476, 2477, 2479, 2480, 2489, 2500, 2510, 2521, 2532, 2533, 2842, 2843, 2851, 2862, 2874, 2886, 2887, 3166, 3167, 3169, 3170, 3172, 3173, 3182, 3193, 3200 ] }
{ "red_pajama_v2": { "ccnet_original_length": 3200, "ccnet_original_nlines": 71, "rps_doc_curly_bracket": 0, "rps_doc_ldnoobw_words": 0, "rps_doc_lorem_ipsum": 0, "rps_doc_stop_word_fraction": 0.3520485460758209, "rps_doc_ut1_blacklist": 0, "rps_doc_frac_all_caps_words": 0.037936270236968994, "rps_doc_frac_lines_end_with_ellipsis": 0, "rps_doc_frac_no_alph_words": 0.20030349493026733, "rps_doc_frac_unique_words": 0.4874274730682373, "rps_doc_mean_word_length": 4.866537570953369, "rps_doc_num_sentences": 44, "rps_doc_symbol_to_word_ratio": 0, "rps_doc_unigram_entropy": 5.064014911651611, "rps_doc_word_count": 517, "rps_doc_frac_chars_dupe_10grams": 0, "rps_doc_frac_chars_dupe_5grams": 0.05683625116944313, "rps_doc_frac_chars_dupe_6grams": 0, "rps_doc_frac_chars_dupe_7grams": 0, "rps_doc_frac_chars_dupe_8grams": 0, "rps_doc_frac_chars_dupe_9grams": 0, "rps_doc_frac_chars_top_2gram": 0.009538950398564339, "rps_doc_frac_chars_top_3gram": 0.01192369032651186, "rps_doc_frac_chars_top_4gram": 0.01271860022097826, "rps_doc_books_importance": -295.29852294921875, "rps_doc_books_importance_length_correction": -295.29852294921875, "rps_doc_openwebtext_importance": -164.41220092773438, "rps_doc_openwebtext_importance_length_correction": -164.41220092773438, "rps_doc_wikipedia_importance": -85.61676788330078, "rps_doc_wikipedia_importance_length_correction": -85.61676788330078 }, "fasttext": { "dclm": 0.02761220932006836, "english": 0.8499037623405457, "fineweb_edu_approx": 1.4609876871109009, "eai_general_math": 0.37710320949554443, "eai_open_web_math": 0.18432879447937012, "eai_web_code": 0.12570470571517944 } }
{ "free_decimal_correspondence": { "primary": { "code": "004.678", "labels": { "level_1": "General works, books and libraries, information sciences", "level_2": "", "level_3": "Computers and Computer science" } }, "secondary": { "code": "005.44", "labels": { "level_1": "General works, books and libraries, information sciences", "level_2": "", "level_3": "Computer programming" } } }, "bloom_cognitive_process": { "primary": { "code": "3", "label": "Apply" }, "secondary": { "code": "2", "label": "Understand" } }, "bloom_knowledge_domain": { "primary": { "code": "3", "label": "Procedural" }, "secondary": { "code": "2", "label": "Conceptual" } }, "document_type_v1": { "primary": { "code": "3", "label": "Reference/Encyclopedic/Educational" }, "secondary": { "code": "5", "label": "Social/Forum" } }, "extraction_artifacts": { "primary": { "code": "0", "label": "No Artifacts" }, "secondary": { "code": "-1", "label": "Abstain" } }, "missing_content": { "primary": { "code": "0", "label": "No missing content" }, "secondary": { "code": "-1", "label": "Abstain" } }, "document_type_v2": { "primary": { "code": "23", "label": "Tutorial" }, "secondary": { "code": "18", "label": "Q&A Forum" } }, "reasoning_depth": { "primary": { "code": "2", "label": "Basic Reasoning" }, "secondary": { "code": "3", "label": "Intermediate Reasoning" } }, "technical_correctness": { "primary": { "code": "3", "label": "Mostly Correct" }, "secondary": { "code": "4", "label": "Highly Correct" } }, "education_level": { "primary": { "code": "3", "label": "Undergraduate Level" }, "secondary": { "code": "4", "label": "Graduate/Expert Level" } } }
672f1e42c33a7f9846924a2431ea77df
-8,390,564,376,048,380,000
0 $\begingroup$ Sometime the distribution of a variable is not normal, either left or right skewed, and people tend to transform the distribution by doing x^2 or log(x), etc to make it look more normal. Question is why this would help model accuracy? Thanks, $\endgroup$ 5 • 1 $\begingroup$ As tagging implies, there are many threads here on transformations. This may not be an exact duplicate of any existing thread, but at the same time I don't think there is anything new here. stats.stackexchange.com/questions/107610/… is good on the most common single case. $\endgroup$ – Nick Cox Jun 3, 2017 at 14:45 • $\begingroup$ This question is a little out of focus, because transformations are not explicitly to "help model accuracy," nor are they generally to create Normal distributions. Their purposes include linearizing relationships, achieving symmetric or homoscedastic residuals, improving goodness of fit, expressing relationships additively, and much more. $\endgroup$ – whuber Jun 3, 2017 at 15:53 • $\begingroup$ Thanks @NickCox for the link. I was puzzled at why people are doing this transformation and sometimes improves model accuracy. Both you and Whuber's comments help me understand that the transformation does make relationship linear, symmetric, homoscedstic, which sometimes improve linear models's accuracy because they are based on these assumptions. However If we don't use linear models, these don't quite matter. $\endgroup$ – frank Dec 1, 2017 at 15:07 • $\begingroup$ And, it only works sometimes because beside linearity, accuracy depends on many other things, (For example model can be perfect linear and have good training score but may give poor predictions if it ignored important confounding factors ). I wish to keep this post for its broader scope. $\endgroup$ – frank Dec 1, 2017 at 15:07 • $\begingroup$ The other question answers why log transformation help skwed distribution, but this question further answers after transformation, why a symmetric distribution is helpful sometimes to linear models. $\endgroup$ – frank Dec 1, 2017 at 15:16 1 Answer 1 0 $\begingroup$ The reason is that you are assuming a normal distribution. To explain a bit: in your model you are assuming that some change in X yields a change in Y. Such that increasing X by some value yields a change in Y by some value. But that's not the way real life works in many cases. The idea of "too much of a good thing" is common in real data. Take this example: the relationship between the height of a corn stalk and the amount of water it is given. This relationship might be linear where I give the corn plant 100 ml of water and this increases the height of the corn by 1 cm. In this case my beta coefficient would probably be .01 (i.e. 1 milliliter yields a .01 centimeter increase in height). But that is not how things in life work. The relationship between the height of the corn and the amount of water it gets is not linear. In other words, I can't keep giving a corn plant water to make it grown infinitely tall. At a certain point, no matter how much water I give it, it will not grow any taller (suggest a logarithmic relationship). Further, there can be a point where I give the corn plant too much water causing it to wilt and die. If I were to model the relationship between watering a plant and plant height as linear, that would mean that my model would be misleading. I transform the variable to more accurately depict the relationship between my predictor and dependent variable. One final thing to caution: your transformation should make sense in the context of the theory of your field. Just transforming a variable so that your model is more elegant is not appropriate. The different relationship might just be due to your sample rather than the true relationship within the population. $\endgroup$ 4 • 3 $\begingroup$ This answer asserts that we are assuming a normal distribution (of what, precisely) but then morphs into an example where a nonlinear relationship is better off treated by logging the predictor. At least that's what you seem to be suggesting. The question about a normal distribution is never really addressed. I don't think this is answering the question directly. A much shorter answer is that often normality is not a key assumption (better explained as "ideal condition") at all, but rather transformations may help with linearity, additivity and homoscedasticity. $\endgroup$ – Nick Cox Jun 3, 2017 at 14:41 • $\begingroup$ @NickCox +1 to your comment. But can you please elaborate and expand on how "transformation may help with linearity...". OP's question is specifically about model accuracy, which I interpret to mean better fit of the response. $\endgroup$ – horaceT Jun 3, 2017 at 15:17 • 1 $\begingroup$ @horaceT That's what the thread I linked to to my comment on the question discusses. Simple examples: if the real pattern for outcome $y$ and predictor $x$ is more like $\exp(a + bx)$ or $a x^b$ consider working on the logarithmic scale (in the second example with $\log x$ as well). (I am interpreting accuracy loosely as meaning finding a better or more appropriate model.) $\endgroup$ – Nick Cox Jun 3, 2017 at 15:26 • $\begingroup$ @NickCox Agree with your comment. The answer wasn't to the point. $\endgroup$ – frank Dec 1, 2017 at 15:11 Not the answer you're looking for? Browse other questions tagged or ask your own question.
{ "url": "https://stats.stackexchange.com/questions/283383/why-transform-distribution-could-help-model-accuracy?noredirect=1", "source_domain": "stats.stackexchange.com", "snapshot_id": "CC-MAIN-2024-10", "warc_metadata": { "Content-Length": "149994", "Content-Type": "application/http; msgtype=response", "WARC-Block-Digest": "sha1:QOK2FNFRHOCHVX2YHNYUKDETT6RDP4LE", "WARC-Concurrent-To": "<urn:uuid:a99c10fd-2fc4-4567-9e80-5bf3920a1baa>", "WARC-Date": "2024-02-21T08:10:24Z", "WARC-IP-Address": "172.64.144.30", "WARC-Identified-Payload-Type": "text/html", "WARC-Payload-Digest": "sha1:3HHKBSJQQXIJFR6MODKI7ITNSR2YF6UG", "WARC-Record-ID": "<urn:uuid:ab5b83f7-93b1-45d9-9406-cb4a75c4608e>", "WARC-Target-URI": "https://stats.stackexchange.com/questions/283383/why-transform-distribution-could-help-model-accuracy?noredirect=1", "WARC-Truncated": null, "WARC-Type": "response", "WARC-Warcinfo-ID": "<urn:uuid:5c15e712-ed4b-46a4-b5e7-3636ae9334d3>" }, "warc_info": "isPartOf: CC-MAIN-2024-10\r\npublisher: Common Crawl\r\ndescription: Wide crawl of the web for February/March 2024\r\noperator: Common Crawl Admin ([email protected])\r\nhostname: ip-10-67-67-183\r\nsoftware: Apache Nutch 1.19 (modified, https://github.com/commoncrawl/nutch/)\r\nrobots: checked via crawler-commons 1.5-SNAPSHOT (https://github.com/crawler-commons/crawler-commons)\r\nformat: WARC File Format 1.1\r\nconformsTo: https://iipc.github.io/warc-specifications/specifications/warc-format/warc-1.1/" }
{ "line_start_idx": [ 0, 2, 16, 17, 204, 205, 253, 254, 262, 263, 275, 277, 283, 586, 601, 626, 997, 1010, 1035, 1481, 1493, 1518, 1837, 1849, 1874, 2103, 2115, 2140, 2141, 2152, 2153, 2155, 2169, 2170, 2512, 2513, 2910, 2911, 3318, 3319, 3572, 3573, 3884, 3885, 3897, 3899, 3905, 4504, 4519, 4544, 4801, 4815, 4840, 4846, 5252, 5267, 5292, 5388, 5400, 5425, 5426 ], "line_end_idx": [ 2, 16, 17, 204, 205, 253, 254, 262, 263, 275, 277, 283, 586, 601, 626, 997, 1010, 1035, 1481, 1493, 1518, 1837, 1849, 1874, 2103, 2115, 2140, 2141, 2152, 2153, 2155, 2169, 2170, 2512, 2513, 2910, 2911, 3318, 3319, 3572, 3573, 3884, 3885, 3897, 3899, 3905, 4504, 4519, 4544, 4801, 4815, 4840, 4846, 5252, 5267, 5292, 5388, 5400, 5425, 5426, 5516 ] }
{ "red_pajama_v2": { "ccnet_original_length": 5516, "ccnet_original_nlines": 60, "rps_doc_curly_bracket": 0, "rps_doc_ldnoobw_words": 0, "rps_doc_lorem_ipsum": 0, "rps_doc_stop_word_fraction": 0.39134275913238525, "rps_doc_ut1_blacklist": 0, "rps_doc_frac_all_caps_words": 0.01678445003926754, "rps_doc_frac_lines_end_with_ellipsis": 0, "rps_doc_frac_no_alph_words": 0.23851589858531952, "rps_doc_frac_unique_words": 0.38691794872283936, "rps_doc_mean_word_length": 4.72505521774292, "rps_doc_num_sentences": 53, "rps_doc_symbol_to_word_ratio": 0.0017667800420895219, "rps_doc_unigram_entropy": 5.3151750564575195, "rps_doc_word_count": 902, "rps_doc_frac_chars_dupe_10grams": 0.01830127090215683, "rps_doc_frac_chars_dupe_5grams": 0.11098075658082962, "rps_doc_frac_chars_dupe_6grams": 0.08352886140346527, "rps_doc_frac_chars_dupe_7grams": 0.047864850610494614, "rps_doc_frac_chars_dupe_8grams": 0.036602530628442764, "rps_doc_frac_chars_dupe_9grams": 0.01830127090215683, "rps_doc_frac_chars_top_2gram": 0.01900516077876091, "rps_doc_frac_chars_top_3gram": 0.009385270066559315, "rps_doc_frac_chars_top_4gram": 0.011731579899787903, "rps_doc_books_importance": -514.2546997070312, "rps_doc_books_importance_length_correction": -514.2546997070312, "rps_doc_openwebtext_importance": -244.5938262939453, "rps_doc_openwebtext_importance_length_correction": -244.5938262939453, "rps_doc_wikipedia_importance": -245.9239501953125, "rps_doc_wikipedia_importance_length_correction": -245.9239501953125 }, "fasttext": { "dclm": 0.11237359046936035, "english": 0.9318466782569885, "fineweb_edu_approx": 2.1018447875976562, "eai_general_math": 0.694741427898407, "eai_open_web_math": 0.48155951499938965, "eai_web_code": 0.04596894979476929 } }
{ "free_decimal_correspondence": { "primary": { "code": "519.5", "labels": { "level_1": "Science and Natural history", "level_2": "Mathematics", "level_3": "Probabilities; or, Mathematical statistics" } }, "secondary": { "code": "-1", "labels": { "level_1": "", "level_2": "", "level_3": "" } } }, "bloom_cognitive_process": { "primary": { "code": "2", "label": "Understand" }, "secondary": { "code": "4", "label": "Analyze" } }, "bloom_knowledge_domain": { "primary": { "code": "2", "label": "Conceptual" }, "secondary": { "code": "3", "label": "Procedural" } }, "document_type_v1": { "primary": { "code": "5", "label": "Social/Forum" }, "secondary": { "code": "-1", "label": "Abstain" } }, "extraction_artifacts": { "primary": { "code": "1", "label": "Leftover HTML" }, "secondary": { "code": "3", "label": "Irrelevant Content" } }, "missing_content": { "primary": { "code": "0", "label": "No missing content" }, "secondary": { "code": "-1", "label": "Abstain" } }, "document_type_v2": { "primary": { "code": "18", "label": "Q&A Forum" }, "secondary": { "code": "10", "label": "Knowledge Article" } }, "reasoning_depth": { "primary": { "code": "3", "label": "Intermediate Reasoning" }, "secondary": { "code": "2", "label": "Basic Reasoning" } }, "technical_correctness": { "primary": { "code": "4", "label": "Highly Correct" }, "secondary": { "code": "3", "label": "Mostly Correct" } }, "education_level": { "primary": { "code": "3", "label": "Undergraduate Level" }, "secondary": { "code": "2", "label": "High School Level" } } }
672f1e42c33a7f9846924a2431ea77df
-6,473,883,755,976,292,000
Friday, November 02, 2007 iPlayer DRM Ben Laurie is not impressed at the BBC's complaints about criticism of the iPlayer. "An interesting podcast with Ashley Highfield, Director Future Media & Technology. We’re not doing enough [about open source] and it is something I want to turn up the heat on Well, that’s a good start, but he then goes on to say The problem at the moment, there is no open source DRM. It’s almost a contradiction in terms, if you have DRM how can you have it open source? Because open source people will be able to find out how it works and get round it. Oh, dear. Because, of course, no-one will work out how the Microsoft DRM works, just like they haven’t worked out all the other DRMs out there. Not. In any case, this entirely misses the point: there is no DRM on the broadcast signal, nor was there on old-fashioned video tapes. Why are downloads different? Why is it not sufficient to rely on the law, as has always happened in the past? Why not assume that your users are mostly honest rather than treat them like criminals?" Well said Ben. Not surpisingly Cory has something to say about this as well. "Ashley Highfield, the BBC's Director Future Media & Technology, has done an interview with the BBC Backstage podcast about the BBC's new DRM-based net-delivery system, iPlayer, which delivers a slim fraction of the functionality available to people who watch their TV over the air. Highfield defends the company's DRM in an incoherent way, attacking straw-men ("The rightsholders need DRM to protect their rights" and "we need open source DRM, but that may be a contradiction in terms," "Rightsholders are scary," "We need a fictional technology that will let us insert ads but only when American eyeballs are present") but without addressing the really meaty questions. The BBC broadcasts the entirety of its programming at the speed of light, in digital form, without DRM, to every corner of the UK. The net is flooded with every single show the BBC transmits. The BBC has previously stood up to rightsholders who insisted DRM (removing DRM from its satellite feeds, despite an entertainment industry boycott that lasted a year). Adding DRM to its downloads just makes the downloads suck, traps Britons into using Microsoft OSes, shuts out one in four license-paying households who don't have the right combination, bans open source -- but it has nothing to do with stopping infringing downloads... I like Highfield -- I know him personally and think he's smarter than this. I'd love to see him interviewed by someone who actually walked him through the real implications of what he's proposing here." No comments:
{ "url": "http://b2fxxx.blogspot.com/2007/11/iplayer-drm.html", "source_domain": "b2fxxx.blogspot.com", "snapshot_id": "crawl=CC-MAIN-2017-34", "warc_metadata": { "Content-Length": "258442", "Content-Type": "application/http; msgtype=response", "WARC-Block-Digest": "sha1:CFV3SICB4UFX6KVBQPKMAFVAQT7VKPYA", "WARC-Concurrent-To": "<urn:uuid:7a2da4f8-aec5-41b6-a268-c2081d669303>", "WARC-Date": "2017-08-21T04:36:51Z", "WARC-IP-Address": "172.217.8.1", "WARC-Identified-Payload-Type": "text/html", "WARC-Payload-Digest": "sha1:VNFN5JGCAXZCJCTUAUZQEFFZRYY4FLBQ", "WARC-Record-ID": "<urn:uuid:6e71b7ad-4a64-44b7-891a-9f1712679c3d>", "WARC-Target-URI": "http://b2fxxx.blogspot.com/2007/11/iplayer-drm.html", "WARC-Truncated": null, "WARC-Type": "response", "WARC-Warcinfo-ID": "<urn:uuid:d9a838b7-382c-461d-911c-6819eff6349e>" }, "warc_info": "robots: classic\r\nhostname: ip-10-147-224-2.ec2.internal\r\nsoftware: Nutch 1.6 (CC)\r\nisPartOf: CC-MAIN-2017-34\r\noperator: Common Crawl Admin\r\ndescription: Wide crawl of the web for August 2017\r\npublisher: Common Crawl\r\nformat: WARC File Format 1.0\r\nconformsTo: http://bibnum.bnf.fr/WARC/WARC_ISO_28500_version1_latestdraft.pdf" }
{ "line_start_idx": [ 0, 26, 27, 39, 40, 124, 125, 208, 209, 302, 303, 357, 358, 584, 585, 734, 735, 1064, 1065, 1080, 1081, 1143, 1144, 1427, 1428, 1817, 1818, 2448, 2449, 2652, 2653 ], "line_end_idx": [ 26, 27, 39, 40, 124, 125, 208, 209, 302, 303, 357, 358, 584, 585, 734, 735, 1064, 1065, 1080, 1081, 1143, 1144, 1427, 1428, 1817, 1818, 2448, 2449, 2652, 2653, 2665 ] }
{ "red_pajama_v2": { "ccnet_original_length": 2665, "ccnet_original_nlines": 30, "rps_doc_curly_bracket": 0, "rps_doc_ldnoobw_words": 1, "rps_doc_lorem_ipsum": 0, "rps_doc_stop_word_fraction": 0.45027124881744385, "rps_doc_ut1_blacklist": 0, "rps_doc_frac_all_caps_words": 0.04701627045869827, "rps_doc_frac_lines_end_with_ellipsis": 0.032258059829473495, "rps_doc_frac_no_alph_words": 0.1591320037841797, "rps_doc_frac_unique_words": 0.5545657277107239, "rps_doc_mean_word_length": 4.688196182250977, "rps_doc_num_sentences": 23, "rps_doc_symbol_to_word_ratio": 0.0018083200557157397, "rps_doc_unigram_entropy": 5.1428632736206055, "rps_doc_word_count": 449, "rps_doc_frac_chars_dupe_10grams": 0, "rps_doc_frac_chars_dupe_5grams": 0, "rps_doc_frac_chars_dupe_6grams": 0, "rps_doc_frac_chars_dupe_7grams": 0, "rps_doc_frac_chars_dupe_8grams": 0, "rps_doc_frac_chars_dupe_9grams": 0, "rps_doc_frac_chars_top_2gram": 0.02850355952978134, "rps_doc_frac_chars_top_3gram": 0.01805225946009159, "rps_doc_frac_chars_top_4gram": 0.027553439140319824, "rps_doc_books_importance": -254.5838623046875, "rps_doc_books_importance_length_correction": -254.5838623046875, "rps_doc_openwebtext_importance": -163.3961181640625, "rps_doc_openwebtext_importance_length_correction": -163.3961181640625, "rps_doc_wikipedia_importance": -109.96505737304688, "rps_doc_wikipedia_importance_length_correction": -109.96505737304688 }, "fasttext": { "dclm": 0.07230603694915771, "english": 0.9603540897369385, "fineweb_edu_approx": 1.4632529020309448, "eai_general_math": 0.09595561027526855, "eai_open_web_math": 0.3226968050003052, "eai_web_code": 0.043877121061086655 } }
{ "free_decimal_correspondence": { "primary": { "code": "004.67", "labels": { "level_1": "General works, books and libraries, information sciences", "level_2": "", "level_3": "Computers and Computer science" } }, "secondary": { "code": "384.3", "labels": { "level_1": "Social sciences", "level_2": "Commerce and Communication and traffic", "level_3": "Telecommunication" } } }, "bloom_cognitive_process": { "primary": { "code": "5", "label": "Evaluate" }, "secondary": { "code": "2", "label": "Understand" } }, "bloom_knowledge_domain": { "primary": { "code": "2", "label": "Conceptual" }, "secondary": { "code": "3", "label": "Procedural" } }, "document_type_v1": { "primary": { "code": "1", "label": "News/Editorial" }, "secondary": { "code": "-1", "label": "Abstain" } }, "extraction_artifacts": { "primary": { "code": "0", "label": "No Artifacts" }, "secondary": { "code": "-1", "label": "Abstain" } }, "missing_content": { "primary": { "code": "0", "label": "No missing content" }, "secondary": { "code": "-1", "label": "Abstain" } }, "document_type_v2": { "primary": { "code": "16", "label": "Personal Blog" }, "secondary": { "code": "10", "label": "Knowledge Article" } }, "reasoning_depth": { "primary": { "code": "3", "label": "Intermediate Reasoning" }, "secondary": { "code": "2", "label": "Basic Reasoning" } }, "technical_correctness": { "primary": { "code": "6", "label": "Not Applicable/Indeterminate" }, "secondary": { "code": "3", "label": "Mostly Correct" } }, "education_level": { "primary": { "code": "2", "label": "High School Level" }, "secondary": { "code": "1", "label": "General Audience" } } }
672f1e42c33a7f9846924a2431ea77df
7,658,281,218,052,199,000
ProcessStartInfo.RedirectStandardOutput ProcessStartInfo.RedirectStandardOutput ProcessStartInfo.RedirectStandardOutput ProcessStartInfo.RedirectStandardOutput Property Definition Gets or sets a value that indicates whether the textual output of an application is written to the StandardOutput stream. public: property bool RedirectStandardOutput { bool get(); void set(bool value); }; public bool RedirectStandardOutput { get; set; } member this.RedirectStandardOutput : bool with get, set Public Property RedirectStandardOutput As Boolean Property Value true if output should be written to StandardOutput; otherwise, false. The default is false. Examples // Run "cl.exe /cld stdstr.cpp /link /out:sample.exe". UseShellExecute is false because we're specifying // an executable directly and in this case depending on it being in a PATH folder. By setting // RedirectStandardOutput to true, the output of cl.exe is directed to the Process.StandardOutput stream // which is then displayed in this console window directly. Process^ compiler = gcnew Process; compiler->StartInfo->FileName = "cl.exe"; compiler->StartInfo->Arguments = "/clr stdstr.cpp /link /out:sample.exe"; compiler->StartInfo->UseShellExecute = false; compiler->StartInfo->RedirectStandardOutput = true; compiler->Start(); Console::WriteLine( compiler->StandardOutput->ReadToEnd() ); compiler->WaitForExit(); // Run "csc.exe /r:System.dll /out:sample.exe stdstr.cs". UseShellExecute is false because we're specifying // an executable directly and in this case depending on it being in a PATH folder. By setting // RedirectStandardOutput to true, the output of csc.exe is directed to the Process.StandardOutput stream // which is then displayed in this console window directly. Process compiler = new Process(); compiler.StartInfo.FileName = "csc.exe"; compiler.StartInfo.Arguments = "/r:System.dll /out:sample.exe stdstr.cs"; compiler.StartInfo.UseShellExecute = false; compiler.StartInfo.RedirectStandardOutput = true; compiler.Start(); Console.WriteLine(compiler.StandardOutput.ReadToEnd()); compiler.WaitForExit(); ' Run "vbc.exe /reference:Microsoft.VisualBasic.dll /out:sample.exe stdstr.vb". UseShellExecute is False ' because we're specifying an executable directly and in this case depending on it being in a PATH folder. ' By setting RedirectStandardOutput to True, the output of csc.exe is directed to the Process.StandardOutput ' stream which is then displayed in this console window directly. Dim compiler As New Process() compiler.StartInfo.FileName = "vbc.exe" compiler.StartInfo.Arguments = "/reference:Microsoft.VisualBasic.dll /out:sample.exe stdstr.vb" compiler.StartInfo.UseShellExecute = False compiler.StartInfo.RedirectStandardOutput = True compiler.Start() Console.WriteLine(compiler.StandardOutput.ReadToEnd()) compiler.WaitForExit() Remarks When a Process writes text to its standard stream, that text is typically displayed on the console. By setting RedirectStandardOutput to true to redirect the StandardOutput stream, you can manipulate or suppress the output of a process. For example, you can filter the text, format it differently, or write the output to both the console and a designated log file. Note You must set UseShellExecute to false if you want to set RedirectStandardOutput to true. Otherwise, reading from the StandardOutput stream throws an exception. The redirected StandardOutput stream can be read synchronously or asynchronously. Methods such as Read, ReadLine, and ReadToEnd perform synchronous read operations on the output stream of the process. These synchronous read operations do not complete until the associated Process writes to its StandardOutput stream, or closes the stream. In contrast, BeginOutputReadLine starts asynchronous read operations on the StandardOutput stream. This method enables a designated event handler (see OutputDataReceived) for the stream output and immediately returns to the caller, which can perform other work while the stream output is directed to the event handler. Note The application that is processing the asynchronous output should call the WaitForExit method to ensure that the output buffer has been flushed. Synchronous read operations introduce a dependency between the caller reading from the StandardOutput stream and the child process writing to that stream. These dependencies can cause deadlock conditions. When the caller reads from the redirected stream of a child process, it is dependent on the child. The caller waits for the read operation until the child writes to the stream or closes the stream. When the child process writes enough data to fill its redirected stream, it is dependent on the parent. The child process waits for the next write operation until the parent reads from the full stream or closes the stream. The deadlock condition results when the caller and child process wait for each other to complete an operation, and neither can continue. You can avoid deadlocks by evaluating dependencies between the caller and child process. For example, the following C# code shows how to read from a redirected stream and wait for the child process to exit. Process p = new Process(); p.StartInfo.UseShellExecute = false; p.StartInfo.RedirectStandardOutput = true; p.StartInfo.FileName = "Write500Lines.exe"; p.Start(); // To avoid deadlocks, always read the output stream first and then wait. string output = p.StandardOutput.ReadToEnd(); p.WaitForExit(); The code example avoids a deadlock condition by calling p.StandardOutput.ReadToEnd before p.WaitForExit. A deadlock condition can result if the parent process calls p.WaitForExit before p.StandardOutput.ReadToEnd and the child process writes enough text to fill the redirected stream. The parent process would wait indefinitely for the child process to exit. The child process would wait indefinitely for the parent to read from the full StandardOutput stream. There is a similar issue when you read all text from both the standard output and standard error streams. For example, the following C# code performs a read operation on both streams. // To avoid deadlocks, use asynchronous read operations on at least one of the streams. // Do not perform a synchronous read to the end of both redirected streams. p.BeginOutputReadLine(); string error = p.StandardError.ReadToEnd(); p.WaitForExit(); The code example avoids the deadlock condition by performing asynchronous read operations on the StandardOutput stream. A deadlock condition results if the parent process calls p.StandardOutput.ReadToEnd followed by p.StandardError.ReadToEnd and the child process writes enough text to fill its error stream. The parent process would wait indefinitely for the child process to close its StandardOutput stream. The child process would wait indefinitely for the parent to read from the full StandardError stream. You can use asynchronous read operations to avoid these dependencies and their deadlock potential. Alternately, you can avoid the deadlock condition by creating two threads and reading the output of each stream on a separate thread. Applies to See Also
{ "url": "https://docs.microsoft.com/en-us/dotnet/api/system.diagnostics.processstartinfo.redirectstandardoutput?redirectedfrom=MSDN&view=netframework-4.7.2", "source_domain": "docs.microsoft.com", "snapshot_id": "crawl=CC-MAIN-2018-47", "warc_metadata": { "Content-Length": "40090", "Content-Type": "application/http; msgtype=response", "WARC-Block-Digest": "sha1:MMUSUNKWYOW2SYCNZXZDR5K7YIPFV6RT", "WARC-Concurrent-To": "<urn:uuid:969c339f-9eff-46d3-8826-4ddd52290a82>", "WARC-Date": "2018-11-12T23:17:11Z", "WARC-IP-Address": "23.1.125.170", "WARC-Identified-Payload-Type": "text/html", "WARC-Payload-Digest": "sha1:O5ZOS7TWENME2ULVOSDVVSTBEOVF5WTL", "WARC-Record-ID": "<urn:uuid:420ffc84-b986-4c4e-bb7f-964bcf0cd365>", "WARC-Target-URI": "https://docs.microsoft.com/en-us/dotnet/api/system.diagnostics.processstartinfo.redirectstandardoutput?redirectedfrom=MSDN&view=netframework-4.7.2", "WARC-Truncated": null, "WARC-Type": "response", "WARC-Warcinfo-ID": "<urn:uuid:eb933693-b798-4f7d-be4c-af56e862da40>" }, "warc_info": "isPartOf: CC-MAIN-2018-47\r\npublisher: Common Crawl\r\ndescription: Wide crawl of the web for November 2018\r\noperator: Common Crawl Admin ([email protected])\r\nhostname: ip-10-181-86-109.ec2.internal\r\nsoftware: Apache Nutch 1.15 (modified, https://github.com/commoncrawl/nutch/)\r\nrobots: checked via crawler-commons 0.11-SNAPSHOT (https://github.com/crawler-commons/crawler-commons)\r\nformat: WARC File Format 1.1\r\nconformsTo: http://iipc.github.io/warc-specifications/specifications/warc-format/warc-1.1/" }
{ "line_start_idx": [ 0, 169, 170, 181, 182, 304, 305, 313, 390, 439, 495, 545, 546, 561, 562, 654, 655, 664, 665, 770, 864, 969, 1033, 1068, 1110, 1184, 1230, 1282, 1301, 1302, 1363, 1364, 1389, 1497, 1591, 1697, 1761, 1795, 1836, 1910, 1954, 2004, 2026, 2027, 2083, 2084, 2108, 2214, 2322, 2432, 2502, 2532, 2572, 2668, 2711, 2760, 2777, 2778, 2833, 2834, 2857, 2858, 2866, 2867, 3232, 3233, 3238, 3239, 3399, 3400, 3739, 3740, 4059, 4060, 4065, 4066, 4211, 4212, 5064, 5065, 5183, 5184, 5213, 5252, 5297, 5343, 5356, 5357, 5433, 5481, 5500, 5501, 5962, 5963, 6147, 6148, 6238, 6316, 6343, 6389, 6408, 6409, 6920, 6921, 7154, 7155, 7166, 7167 ], "line_end_idx": [ 169, 170, 181, 182, 304, 305, 313, 390, 439, 495, 545, 546, 561, 562, 654, 655, 664, 665, 770, 864, 969, 1033, 1068, 1110, 1184, 1230, 1282, 1301, 1302, 1363, 1364, 1389, 1497, 1591, 1697, 1761, 1795, 1836, 1910, 1954, 2004, 2026, 2027, 2083, 2084, 2108, 2214, 2322, 2432, 2502, 2532, 2572, 2668, 2711, 2760, 2777, 2778, 2833, 2834, 2857, 2858, 2866, 2867, 3232, 3233, 3238, 3239, 3399, 3400, 3739, 3740, 4059, 4060, 4065, 4066, 4211, 4212, 5064, 5065, 5183, 5184, 5213, 5252, 5297, 5343, 5356, 5357, 5433, 5481, 5500, 5501, 5962, 5963, 6147, 6148, 6238, 6316, 6343, 6389, 6408, 6409, 6920, 6921, 7154, 7155, 7166, 7167, 7175 ] }
{ "red_pajama_v2": { "ccnet_original_length": 7175, "ccnet_original_nlines": 107, "rps_doc_curly_bracket": 0.00055748998420313, "rps_doc_ldnoobw_words": 0, "rps_doc_lorem_ipsum": 0, "rps_doc_stop_word_fraction": 0.2902979254722595, "rps_doc_ut1_blacklist": 0, "rps_doc_frac_all_caps_words": 0.005347589962184429, "rps_doc_frac_lines_end_with_ellipsis": 0, "rps_doc_frac_no_alph_words": 0.22307105362415314, "rps_doc_frac_unique_words": 0.27161863446235657, "rps_doc_mean_word_length": 6.419068813323975, "rps_doc_num_sentences": 134, "rps_doc_symbol_to_word_ratio": 0.0015278799692168832, "rps_doc_unigram_entropy": 4.853862285614014, "rps_doc_word_count": 902, "rps_doc_frac_chars_dupe_10grams": 0.17789292335510254, "rps_doc_frac_chars_dupe_5grams": 0.36269429326057434, "rps_doc_frac_chars_dupe_6grams": 0.27841106057167053, "rps_doc_frac_chars_dupe_7grams": 0.26424869894981384, "rps_doc_frac_chars_dupe_8grams": 0.19170984625816345, "rps_doc_frac_chars_dupe_9grams": 0.19170984625816345, "rps_doc_frac_chars_top_2gram": 0.026943009346723557, "rps_doc_frac_chars_top_3gram": 0.025906739756464958, "rps_doc_frac_chars_top_4gram": 0.022797929123044014, "rps_doc_books_importance": -568.5556640625, "rps_doc_books_importance_length_correction": -568.5556640625, "rps_doc_openwebtext_importance": -295.0416259765625, "rps_doc_openwebtext_importance_length_correction": -295.0416259765625, "rps_doc_wikipedia_importance": -213.28387451171875, "rps_doc_wikipedia_importance_length_correction": -213.28387451171875 }, "fasttext": { "dclm": 0.3083825707435608, "english": 0.781907320022583, "fineweb_edu_approx": 3.21706223487854, "eai_general_math": 0.819837212562561, "eai_open_web_math": 0.09770982712507248, "eai_web_code": 0.9491022825241089 } }
{ "free_decimal_correspondence": { "primary": { "code": "005.1", "labels": { "level_1": "General works, books and libraries, information sciences", "level_2": "", "level_3": "Computer programming" } }, "secondary": { "code": "004.02", "labels": { "level_1": "General works, books and libraries, information sciences", "level_2": "", "level_3": "Computers and Computer science" } } }, "bloom_cognitive_process": { "primary": { "code": "3", "label": "Apply" }, "secondary": { "code": "2", "label": "Understand" } }, "bloom_knowledge_domain": { "primary": { "code": "3", "label": "Procedural" }, "secondary": { "code": "2", "label": "Conceptual" } }, "document_type_v1": { "primary": { "code": "3", "label": "Reference/Encyclopedic/Educational" }, "secondary": { "code": "4", "label": "Code/Software" } }, "extraction_artifacts": { "primary": { "code": "0", "label": "No Artifacts" }, "secondary": { "code": "3", "label": "Irrelevant Content" } }, "missing_content": { "primary": { "code": "0", "label": "No missing content" }, "secondary": { "code": "4", "label": "Missing Images or Figures" } }, "document_type_v2": { "primary": { "code": "8", "label": "Documentation" }, "secondary": { "code": "23", "label": "Tutorial" } }, "reasoning_depth": { "primary": { "code": "3", "label": "Intermediate Reasoning" }, "secondary": { "code": "2", "label": "Basic Reasoning" } }, "technical_correctness": { "primary": { "code": "4", "label": "Highly Correct" }, "secondary": { "code": "3", "label": "Mostly Correct" } }, "education_level": { "primary": { "code": "3", "label": "Undergraduate Level" }, "secondary": { "code": "4", "label": "Graduate/Expert Level" } } }
672f1e42c33a7f9846924a2431ea77df
-2,152,183,356,947,535,600
使用 json-server 搭建 api mock 服务 (一) 在前端开发过程中,如果后端接口还没有提供,前端拿不到数据一些交互行为的代码可能就没法继续写,这时我们通常自己造一些数据来让页面流程走下去,最近项目切换到vue框架开发,发现json-server能很好的解决接口mock的问题 json-server官方地址 安装 $ npm install json-server -g 启动json-server $ json-server --watch db.json 通过官方的例子你可以发现 json-server其实是在你访问接口时,返回db.json里面的对应的key的值 例如:你访问 http://localhost:3000/posts/ 返回db.json里面的json.posts 那么问题来了 1.如果我们要模拟的接口非常多,要一个一个的往db.json里面添加吗,其他前端人员也会修改到这个文件,每次合并代码都要考虑冲突问题,而且这个文件会变得非常庞大,难以维护 2.如果我的接口是http://localhost:3000/a/bhttp://localhost:3000/a/b/c 怎么解决 本文就主要探讨下这两个问题的解决方案: 1,修改package.json里面的npm run mock 对应的命令为 json-server mock/index.js 在项目中建立mock文件夹,文件夹下建立index.js(名字随意)文件, index.js module.exports = function () { return { a: ['接口a的返回数据'], b: ['接口b的返回数据'] } } 此时启动npm run mock,访问http://localhost:3000/a,可以获得想要的结果 2,在mock文件夹下新建几个js文件,例如我新建了 └─ mock │─ test │ ├─ a.js │ └─ b.js └─ test2 ├─ c.js └─ d.js 举例其中一个a.js module.exports = { url: 'a', title: '', type: 'GET', decs: '', query: { a: '1' }, res: { ret: 1, result: [ { a: '2', b: '3', c: '4' } ] } } 修改index.js let Path = require('path') let glob = require('glob') // 读取所有API文件 const apiFiles = glob.sync(Path.resolve(__dirname, './') + '/**/*.js', { nodir: true }) let data = {} // 输出所有api文件 i从1开始 跳过index.js for (let i = 1, l = apiFiles.length; i < l; i++) { let api = require(apiFiles[i]) if (api.url) { data[api.url] = api.res } } module.exports = function () { return data } 然后启动mock,你会看到控制台打印 Resources http://localhost:8083/a http://localhost:8083/b http://localhost:8083/c http://localhost:8083/d 成功的实现了每个api分离,添加一个api我们只需要复制一个js文件,删除和修改也只是改动我们自己的文件,不会影响到团队其他成员 第二个问题:如果我的api路径类似 a/ba/b/c怎么办 修改index.js let Path = require('path') let glob = require('glob') const apiFiles = glob.sync(Path.resolve(__dirname, './') + '/**/*.js', { nodir: true }) let data = {} for (let i = 1, l = apiFiles.length; i < l; i++) { let api = require(apiFiles[i]) if (api.url) { data[api.url.replace(/\//g, '_')] = api.res } } module.exports = function () { return data } 启动mock服务,我们会看到 Resources http://localhost:8083/a http://localhost:8083/a_b http://localhost:8083/a_b_c http://localhost:8083/a_b_c_d 然后在项目根目录下添加json-server.json文件 { "port": "8888", "routes": "./mock/routes.json" } 在mock文件夹下添加routes.json文件 { "/*/*/*/*/*": "/$1_$2_$3_$4_$5", "/*/*/*/*": "/$1_$2_$3_$4", "/*/*/*": "/$1_$2_$3", "/*/*": "/$1_$2" } 这样我们就将每次请求的路径类似 a/b/c/d/e转换成了a_b_c_d_e 启动mock服务,然后访问路径localhost:8888/a/b/c/d/e,完美 最后贴一下本文中所用到的文件的目录结构 └─ mock │ │─ test # 文件夹1 │ │ ├─ a.js # api1 | │ └─ b.js # api2 | ├─ test2 # 文件夹2 | │ ├─ c.js # api3 | │ └─ d.js # api4 | ├─ index.js # 出口文件 | └─ routers.json # 路径转换配置文件 ├─ json-server.json # 端口等配置 └─ package.json # 项目配置 本文系作者搭建mock服务的一点心得,如有关于搭建mock服务的优雅的解决方案,欢迎各路大神与作者沟通交流,欢迎指正本文中的错误     原文作者:Jaywin     原文地址: https://segmentfault.com/a/1190000010629595     本文转自网络文章,转载此文章仅为分享知识,如有侵权,请联系博主进行删除。 点赞
{ "url": "http://ddrv.cn/a/474334", "source_domain": "ddrv.cn", "snapshot_id": "crawl=CC-MAIN-2020-50", "warc_metadata": { "Content-Length": "39189", "Content-Type": "application/http; msgtype=response", "WARC-Block-Digest": "sha1:UJAGAYMHXGAGKLFXIDLKMSP5FNXWM6DW", "WARC-Concurrent-To": "<urn:uuid:88302112-14f2-491e-a117-d8c74e1be824>", "WARC-Date": "2020-12-01T08:25:10Z", "WARC-IP-Address": "47.94.80.105", "WARC-Identified-Payload-Type": "text/html", "WARC-Payload-Digest": "sha1:U3UXKHQYBYXPP673EVHBX6JN4GVT4IF2", "WARC-Record-ID": "<urn:uuid:13b672f3-431a-4dff-92be-65c2e21af5da>", "WARC-Target-URI": "http://ddrv.cn/a/474334", "WARC-Truncated": null, "WARC-Type": "response", "WARC-Warcinfo-ID": "<urn:uuid:fa9decab-bd55-4151-a7f4-78471896043d>" }, "warc_info": "isPartOf: CC-MAIN-2020-50\r\npublisher: Common Crawl\r\ndescription: Wide crawl of the web for November/December 2020\r\noperator: Common Crawl Admin ([email protected])\r\nhostname: ip-10-67-67-245.ec2.internal\r\nsoftware: Apache Nutch 1.17 (modified, https://github.com/commoncrawl/nutch/)\r\nrobots: checked via crawler-commons 1.2-SNAPSHOT (https://github.com/crawler-commons/crawler-commons)\r\nformat: WARC File Format 1.1\r\nconformsTo: http://iipc.github.io/warc-specifications/specifications/warc-format/warc-1.1/" }
{ "line_start_idx": [ 0, 34, 35, 148, 149, 165, 166, 169, 170, 199, 200, 214, 244, 245, 258, 301, 360, 361, 368, 454, 521, 522, 542, 543, 608, 646, 655, 656, 687, 698, 719, 739, 743, 745, 746, 799, 826, 827, 835, 864, 878, 892, 904, 918, 932, 933, 944, 945, 964, 976, 989, 1004, 1016, 1027, 1038, 1043, 1052, 1064, 1078, 1086, 1102, 1118, 1133, 1141, 1147, 1151, 1153, 1154, 1165, 1166, 1193, 1220, 1221, 1234, 1307, 1321, 1324, 1338, 1368, 1419, 1452, 1469, 1497, 1501, 1503, 1534, 1548, 1550, 1551, 1570, 1571, 1581, 1607, 1633, 1659, 1685, 1686, 1751, 1752, 1782, 1783, 1794, 1795, 1822, 1849, 1850, 1923, 1937, 1940, 1954, 2005, 2038, 2055, 2103, 2107, 2109, 2140, 2154, 2156, 2157, 2172, 2173, 2183, 2209, 2237, 2267, 2299, 2300, 2330, 2331, 2333, 2353, 2388, 2390, 2391, 2416, 2417, 2419, 2454, 2484, 2509, 2528, 2530, 2531, 2570, 2571, 2614, 2615, 2635, 2636, 2644, 2685, 2715, 2745, 2775, 2805, 2835, 2865, 2899, 2930, 2960, 2961, 3026, 3027, 3043, 3097, 3138 ], "line_end_idx": [ 34, 35, 148, 149, 165, 166, 169, 170, 199, 200, 214, 244, 245, 258, 301, 360, 361, 368, 454, 521, 522, 542, 543, 608, 646, 655, 656, 687, 698, 719, 739, 743, 745, 746, 799, 826, 827, 835, 864, 878, 892, 904, 918, 932, 933, 944, 945, 964, 976, 989, 1004, 1016, 1027, 1038, 1043, 1052, 1064, 1078, 1086, 1102, 1118, 1133, 1141, 1147, 1151, 1153, 1154, 1165, 1166, 1193, 1220, 1221, 1234, 1307, 1321, 1324, 1338, 1368, 1419, 1452, 1469, 1497, 1501, 1503, 1534, 1548, 1550, 1551, 1570, 1571, 1581, 1607, 1633, 1659, 1685, 1686, 1751, 1752, 1782, 1783, 1794, 1795, 1822, 1849, 1850, 1923, 1937, 1940, 1954, 2005, 2038, 2055, 2103, 2107, 2109, 2140, 2154, 2156, 2157, 2172, 2173, 2183, 2209, 2237, 2267, 2299, 2300, 2330, 2331, 2333, 2353, 2388, 2390, 2391, 2416, 2417, 2419, 2454, 2484, 2509, 2528, 2530, 2531, 2570, 2571, 2614, 2615, 2635, 2636, 2644, 2685, 2715, 2745, 2775, 2805, 2835, 2865, 2899, 2930, 2960, 2961, 3026, 3027, 3043, 3097, 3138, 3140 ] }
{ "red_pajama_v2": { "ccnet_original_length": 3140, "ccnet_original_nlines": 166, "rps_doc_curly_bracket": 0.01146497018635273, "rps_doc_ldnoobw_words": 0, "rps_doc_lorem_ipsum": 0, "rps_doc_stop_word_fraction": 0.08672799170017242, "rps_doc_ut1_blacklist": 0, "rps_doc_frac_all_caps_words": 0.002628119895234704, "rps_doc_frac_lines_end_with_ellipsis": 0, "rps_doc_frac_no_alph_words": 0.6162943243980408, "rps_doc_frac_unique_words": 0.6060606241226196, "rps_doc_mean_word_length": 8.640692710876465, "rps_doc_num_sentences": 52, "rps_doc_symbol_to_word_ratio": 0.013140600174665451, "rps_doc_unigram_entropy": 4.694735527038574, "rps_doc_word_count": 231, "rps_doc_frac_chars_dupe_10grams": 0.05611221864819527, "rps_doc_frac_chars_dupe_5grams": 0.19539077579975128, "rps_doc_frac_chars_dupe_6grams": 0.15831662714481354, "rps_doc_frac_chars_dupe_7grams": 0.15831662714481354, "rps_doc_frac_chars_dupe_8grams": 0.11322645097970963, "rps_doc_frac_chars_dupe_9grams": 0.05611221864819527, "rps_doc_frac_chars_top_2gram": 0.031563129276037216, "rps_doc_frac_chars_top_3gram": 0.04058115929365158, "rps_doc_frac_chars_top_4gram": 0.027054110541939735, "rps_doc_books_importance": -310.4776916503906, "rps_doc_books_importance_length_correction": -310.4776916503906, "rps_doc_openwebtext_importance": -169.53492736816406, "rps_doc_openwebtext_importance_length_correction": -169.53492736816406, "rps_doc_wikipedia_importance": -51.195274353027344, "rps_doc_wikipedia_importance_length_correction": -51.195274353027344 }, "fasttext": { "dclm": 0.9988725185394287, "english": 0.04355879873037338, "fineweb_edu_approx": 3.2897303104400635, "eai_general_math": 0.8914922475814819, "eai_open_web_math": 0.21190685033798218, "eai_web_code": 0.9115996956825256 } }
{ "free_decimal_correspondence": { "primary": { "code": "005.1", "labels": { "level_1": "General works, books and libraries, information sciences", "level_2": "", "level_3": "Computer programming" } }, "secondary": { "code": "004.02", "labels": { "level_1": "General works, books and libraries, information sciences", "level_2": "", "level_3": "Computers and Computer science" } } }, "bloom_cognitive_process": { "primary": { "code": "3", "label": "Apply" }, "secondary": { "code": "2", "label": "Understand" } }, "bloom_knowledge_domain": { "primary": { "code": "3", "label": "Procedural" }, "secondary": { "code": "2", "label": "Conceptual" } }, "document_type_v1": { "primary": { "code": "3", "label": "Reference/Encyclopedic/Educational" }, "secondary": { "code": "4", "label": "Code/Software" } }, "extraction_artifacts": { "primary": { "code": "0", "label": "No Artifacts" }, "secondary": { "code": "3", "label": "Irrelevant Content" } }, "missing_content": { "primary": { "code": "0", "label": "No missing content" }, "secondary": { "code": "-1", "label": "Abstain" } }, "document_type_v2": { "primary": { "code": "23", "label": "Tutorial" }, "secondary": { "code": "8", "label": "Documentation" } }, "reasoning_depth": { "primary": { "code": "2", "label": "Basic Reasoning" }, "secondary": { "code": "3", "label": "Intermediate Reasoning" } }, "technical_correctness": { "primary": { "code": "4", "label": "Highly Correct" }, "secondary": { "code": "3", "label": "Mostly Correct" } }, "education_level": { "primary": { "code": "3", "label": "Undergraduate Level" }, "secondary": { "code": "2", "label": "High School Level" } } }
672f1e42c33a7f9846924a2431ea77df
9,082,386,357,006,125,000
package com.menthoven.arduinoandroid; import android.app.AlarmManager; import android.app.PendingIntent; import android.app.TimePickerDialog; import android.bluetooth.BluetoothAdapter; import android.bluetooth.BluetoothDevice; import android.content.BroadcastReceiver; import android.content.Context; import android.content.DialogInterface; import android.content.Intent; import android.content.IntentFilter; import android.graphics.drawable.Drawable; import android.os.Build; import android.os.Bundle; import android.os.Handler; import android.os.Message; import android.os.PowerManager; import android.support.design.widget.CoordinatorLayout; import android.support.design.widget.Snackbar; import android.support.v4.app.NavUtils; import android.support.v4.content.ContextCompat; import android.support.v7.app.AlertDialog; import android.support.v7.app.AppCompatActivity; import android.support.v7.widget.Toolbar; import android.util.Log; import android.view.Menu; import android.view.MenuItem; import android.view.View; import android.widget.ArrayAdapter; import android.widget.ProgressBar; import android.widget.TimePicker; import android.widget.Toast; import com.menthoven.arduinoandroid.Recievers.AlarmReceiver; import com.menthoven.arduinoandroid.utils.AppUtils; import com.menthoven.arduinoandroid.utils.Constant; import java.lang.ref.WeakReference; import java.util.Calendar; import java.util.HashMap; import butterknife.Bind; import butterknife.ButterKnife; import info.hoang8f.widget.FButton; public class BluetoothActivity extends AppCompatActivity { BluetoothService bluetoothService; BluetoothDevice device; @Bind(R.id.toolbar) Toolbar toolbar; @Bind(R.id.toolbar_progress_bar) ProgressBar toolbalProgressBar; @Bind(R.id.coordinator_layout_bluetooth) CoordinatorLayout coordinatorLayout; @Bind(R.id.button1) FButton button1; @Bind(R.id.button2) FButton button2; @Bind(R.id.button3) FButton button3; @Bind(R.id.button4) FButton button4; @Bind(R.id.button5) FButton button5; @Bind(R.id.button6) FButton button6; @Bind(R.id.button7) FButton button7; @Bind(R.id.button8) FButton button8; @Bind(R.id.button9) FButton button9; @Bind(R.id.button10) FButton button10; MenuItem reconnectButton; Snackbar snackTurnOn; private boolean showMessagesIsChecked = true; private boolean autoScrollIsChecked = true; public static boolean showTimeIsChecked = true; Drawable bulb_on, bulb_off, fan_on, fan_off, plug_on, plug_off; public static String State; TimePickerDialog timePickerDialog; final static int RQS_1 = 1; PowerManager.WakeLock mWakeLock; public String buttons_state = "abcdefgh"; public HashMap<String,Drawable> buttons_on,buttons_off; @Override protected void onCreate(Bundle savedInstanceState) { super.onCreate(savedInstanceState); setContentView(R.layout.activity_bluetooth); ButterKnife.bind(this); snackTurnOn = Snackbar.make(coordinatorLayout, "Bluetooth turned off", Snackbar.LENGTH_INDEFINITE) .setAction("Turn On", new View.OnClickListener() { @Override public void onClick(View v) { enableBluetooth(); } }); setSupportActionBar(toolbar); myHandler handler = new myHandler(BluetoothActivity.this); assert getSupportActionBar() != null; // won't be null, lint error getSupportActionBar().setDisplayHomeAsUpEnabled(true); device = getIntent().getExtras().getParcelable(Constants.EXTRA_DEVICE); State = getIntent().getStringExtra(Constants.STATE_DEVICE); // Log.d("mmeessaaggee","Activity recieved state "+State); // Log.d("mmeessaaggee","Shared pref "+AppUtils.getAlarmState(getApplicationContext(),Constant.BUTTON_ALARM_STATE)); bluetoothService = new BluetoothService(handler, device); bulb_on = ContextCompat.getDrawable(getApplicationContext(), R.drawable.bulb_on); bulb_off = ContextCompat.getDrawable(getApplicationContext(), R.drawable.bulb_off); bulb_off.setBounds(0, 0, 60, 60); bulb_on.setBounds(0, 0, 60, 60); fan_on = ContextCompat.getDrawable(getApplicationContext(), R.drawable.fan_on); fan_off = ContextCompat.getDrawable(getApplicationContext(), R.drawable.fan_off); fan_off.setBounds(0, 0, 60, 60); fan_on.setBounds(0, 0, 60, 60); plug_on = ContextCompat.getDrawable(getApplicationContext(), R.drawable.plug_on); plug_off = ContextCompat.getDrawable(getApplicationContext(), R.drawable.plug_off); plug_on.setBounds(0, 0, 60, 60); plug_off.setBounds(0, 0, 60, 60); buttons_off=new HashMap<String, Drawable>(); buttons_on=new HashMap<String, Drawable>(); populateHashMap( Constant.BUTTON_1); populateHashMap( Constant.BUTTON_2); populateHashMap( Constant.BUTTON_3); populateHashMap( Constant.BUTTON_4); populateHashMap( Constant.BUTTON_5); populateHashMap( Constant.BUTTON_6); populateHashMap( Constant.BUTTON_7); populateHashMap( Constant.BUTTON_8); setLongClickListener(button1, Constant.BUTTON_1); setLongClickListener(button2, Constant.BUTTON_2); setLongClickListener(button3, Constant.BUTTON_3); setLongClickListener(button4, Constant.BUTTON_4); setLongClickListener(button5, Constant.BUTTON_5); setLongClickListener(button6, Constant.BUTTON_6); setLongClickListener(button7, Constant.BUTTON_7); setLongClickListener(button8, Constant.BUTTON_8); buttonOff(button1, Constant.BUTTON_1); buttonOff(button2, Constant.BUTTON_2); buttonOff(button3, Constant.BUTTON_3); buttonOff(button4, Constant.BUTTON_4); buttonOff(button5, Constant.BUTTON_5); buttonOff(button6, Constant.BUTTON_6); buttonOff(button7, Constant.BUTTON_7); buttonOff(button8, Constant.BUTTON_8); setTitle(device.getName()); button1.setOnClickListener(new View.OnClickListener() { @Override public void onClick(View view) { if (button1.getText().toString().equalsIgnoreCase("OFF")) { sendMessage("A"); buttonOn(button1, Constant.BUTTON_1); } else { sendMessage("a"); buttonOff(button1, Constant.BUTTON_1); } } }); button2.setOnClickListener(new View.OnClickListener() { @Override public void onClick(View view) { if (button2.getText().toString().equalsIgnoreCase("OFF")) { sendMessage("B"); buttonOn(button2, Constant.BUTTON_2); } else { sendMessage("b"); buttonOff(button2, Constant.BUTTON_2); } } }); button3.setOnClickListener(new View.OnClickListener() { @Override public void onClick(View view) { if (button3.getText().toString().equalsIgnoreCase("OFF")) { sendMessage("C"); buttonOn(button3, Constant.BUTTON_3); } else { sendMessage("c"); buttonOff(button3, Constant.BUTTON_3); } } }); button4.setOnClickListener(new View.OnClickListener() { @Override public void onClick(View view) { if (button4.getText().toString().equalsIgnoreCase("OFF")) { sendMessage("D"); buttonOn(button4, Constant.BUTTON_4); } else { sendMessage("d"); buttonOff(button4, Constant.BUTTON_4); } } }); button5.setOnClickListener(new View.OnClickListener() { @Override public void onClick(View view) { if (button5.getText().toString().equalsIgnoreCase("OFF")) { sendMessage("E"); buttonOn(button5, Constant.BUTTON_5); } else { sendMessage("e"); buttonOff(button5, Constant.BUTTON_5); } } }); button6.setOnClickListener(new View.OnClickListener() { @Override public void onClick(View view) { if (button6.getText().toString().equalsIgnoreCase("OFF")) { sendMessage("F"); buttonOn(button6, Constant.BUTTON_6); } else { sendMessage("f"); buttonOff(button6, Constant.BUTTON_6); } } }); button7.setOnClickListener(new View.OnClickListener() { @Override public void onClick(View view) { if (button7.getText().toString().equalsIgnoreCase("OFF")) { sendMessage("G"); buttonOn(button7, Constant.BUTTON_7); } else { sendMessage("g"); buttonOff(button7, Constant.BUTTON_7); } } }); button8.setOnClickListener(new View.OnClickListener() { @Override public void onClick(View view) { if (button8.getText().toString().equalsIgnoreCase("OFF")) { sendMessage("H"); buttonOn(button8, Constant.BUTTON_8); } else { sendMessage("h"); buttonOff(button8, Constant.BUTTON_8); } } }); button9.setOnClickListener(new View.OnClickListener() { @Override public void onClick(View view) { buttonOn(button1, Constant.BUTTON_1); buttonOn(button2, Constant.BUTTON_2); buttonOn(button3, Constant.BUTTON_3); buttonOn(button4, Constant.BUTTON_4); buttonOn(button5, Constant.BUTTON_5); buttonOn(button6, Constant.BUTTON_6); buttonOn(button7, Constant.BUTTON_7); buttonOn(button8, Constant.BUTTON_8); sendMessage("ABCDEFGH"); } }); button10.setOnClickListener(new View.OnClickListener() { @Override public void onClick(View view) { buttonOff(button1, Constant.BUTTON_1); buttonOff(button2, Constant.BUTTON_2); buttonOff(button3, Constant.BUTTON_3); buttonOff(button4, Constant.BUTTON_4); buttonOff(button5, Constant.BUTTON_5); buttonOff(button6, Constant.BUTTON_6); buttonOff(button7, Constant.BUTTON_7); buttonOff(button8, Constant.BUTTON_8); sendMessage("abcdefgh"); } }); } private void populateHashMap(String buttonKey) { String key=device.getName()+"_"+buttonKey; int button_icon=AppUtils.getButtonIcon(getApplicationContext(),key); if(button_icon==3){ buttons_off.put(key,plug_off); buttons_on.put(key,plug_on); }else if(button_icon==2){ buttons_off.put(key,fan_off); buttons_on.put(key,fan_on); }else if(button_icon==1){ buttons_off.put(key,bulb_off); buttons_on.put(key,bulb_on); }else { buttons_off.put(key,null); buttons_on.put(key,null); } } public void buttonOn(FButton button, String buttonKey) { String key=device.getName()+"_"+buttonKey; button.setText("ON"); button.setCompoundDrawables(buttons_on.get(key), null, null, null); button.setButtonColor(getResources().getColor(R.color.colorOn)); } public void buttonOff(FButton button, String buttonKey) { String key=device.getName()+"_"+buttonKey; button.setText("OFF"); button.setCompoundDrawables(buttons_off.get(key), null, null, null); button.setButtonColor(getResources().getColor(R.color.colorItem)); } public void setLongClickListener(FButton button, final String key) { button.setOnLongClickListener(new View.OnLongClickListener() { @Override public boolean onLongClick(View view) { final FButton btn = (FButton) view; final AlertDialog.Builder builderSingle = new AlertDialog.Builder(BluetoothActivity.this); builderSingle.setTitle("Select Tag"); final ArrayAdapter<String> arrayAdapter = new ArrayAdapter<String>(BluetoothActivity.this, android.R.layout.select_dialog_singlechoice); arrayAdapter.add("Light"); arrayAdapter.add("Fan"); arrayAdapter.add("Socket"); arrayAdapter.add("None"); builderSingle.setNegativeButton("cancel", new DialogInterface.OnClickListener() { @Override public void onClick(DialogInterface dialog, int which) { dialog.dismiss(); } }); builderSingle.setAdapter(arrayAdapter, new DialogInterface.OnClickListener() { @Override public void onClick(DialogInterface dialog, int which) { String strName = arrayAdapter.getItem(which); if (strName.equals("Light")) { AppUtils.saveButtonIcon(getApplicationContext(), device.getName()+"_"+key, 1); if (btn.getText().toString().equalsIgnoreCase("ON")) { btn.setCompoundDrawables(bulb_on, null, null, null); } else { btn.setCompoundDrawables(bulb_off, null, null, null); } populateHashMap(key); } else if (strName.equals("Fan")) { AppUtils.saveButtonIcon(getApplicationContext(), device.getName()+"_"+key, 2); if (btn.getText().toString().equalsIgnoreCase("ON")) { btn.setCompoundDrawables(fan_on, null, null, null); } else { btn.setCompoundDrawables(fan_off, null, null, null); } populateHashMap(key); } else if (strName.equals("Socket")) { AppUtils.saveButtonIcon(getApplicationContext(), device.getName()+"_"+key, 3); if (btn.getText().toString().equalsIgnoreCase("ON")) { btn.setCompoundDrawables(plug_on, null, null, null); } else { btn.setCompoundDrawables(plug_off, null, null, null); } populateHashMap(key); } else { AppUtils.saveButtonIcon(getApplicationContext(), device.getName()+"_"+key, 0); btn.setCompoundDrawables(null, null, null, null); populateHashMap(key); } } }); builderSingle.show(); return true; } }); } private void openTimePickerDialog(boolean is24r) { Calendar calendar = Calendar.getInstance(); timePickerDialog = new TimePickerDialog(BluetoothActivity.this, onTimeSetListener, calendar.get(Calendar.HOUR_OF_DAY), calendar.get(Calendar.MINUTE), is24r); timePickerDialog.setTitle("Set Alarm Time"); timePickerDialog.show(); } TimePickerDialog.OnTimeSetListener onTimeSetListener = new TimePickerDialog.OnTimeSetListener() { @Override public void onTimeSet(TimePicker view, int hourOfDay, int minute) { Calendar calNow = Calendar.getInstance(); Calendar calSet = (Calendar) calNow.clone(); calSet.set(Calendar.HOUR_OF_DAY, hourOfDay); calSet.set(Calendar.MINUTE, minute); calSet.set(Calendar.SECOND, 0); calSet.set(Calendar.MILLISECOND, 0); if (calSet.compareTo(calNow) <= 0) { // Today Set time passed, count to tomorrow calSet.add(Calendar.DATE, 1); } setAlarm(calSet); } }; private void setAlarm(Calendar targetCal) { // Calendar calNow = ; long timeInMilis = targetCal.getTimeInMillis() - Calendar.getInstance().getTimeInMillis(); long seconds = timeInMilis / 1000; long minutes = seconds / 60; long hours = minutes / 60; String time = hours % 24 + " hours," + minutes % 60 + " minutes," + seconds % 60 + " seconds"; Toast.makeText(getApplicationContext(), "Alarm set to " + (time) + " from now", Toast.LENGTH_LONG).show(); Intent intent = new Intent(getBaseContext(), AlarmReceiver.class); intent.putExtra(Constants.EXTRA_DEVICE, device); intent.putExtra(Constants.STATE_DEVICE, buttons_state); // Log.d("mmeessaaggee","buttons alarm state "+buttons_state); AppUtils.saveAlarmState(getApplicationContext(), Constant.BUTTON_ALARM_STATE, buttons_state); PendingIntent pendingIntent = PendingIntent.getBroadcast( getBaseContext(), RQS_1, intent, 0); AlarmManager alarmManager = (AlarmManager) getSystemService(Context.ALARM_SERVICE); alarmManager.set(AlarmManager.RTC_WAKEUP, targetCal.getTimeInMillis(), pendingIntent); } @Override protected void onStart() { super.onStart(); IntentFilter filter = new IntentFilter(); filter.addAction(BluetoothAdapter.ACTION_STATE_CHANGED); registerReceiver(mReceiver, filter); Log.d("Alarm", "Onstart"); bluetoothService.connect(); Log.d(Constants.TAG, "Connecting"); PowerManager pm = (PowerManager) getApplicationContext().getSystemService(Context.POWER_SERVICE); mWakeLock = pm.newWakeLock((PowerManager.PARTIAL_WAKE_LOCK | PowerManager.ACQUIRE_CAUSES_WAKEUP), "TAG"); mWakeLock.acquire(); // KeyguardManager manager = (KeyguardManager) this.getSystemService(Context.KEYGUARD_SERVICE); // KeyguardManager.KeyguardLock lock = manager.newKeyguardLock("abc"); // lock.disableKeyguard(); } public void stopActivity() { // mWakeLock.release(); if (Build.VERSION.SDK_INT >= Build.VERSION_CODES.LOLLIPOP) { bluetoothService.stop(); finishAndRemoveTask(); } else { bluetoothService.stop(); finish(); } } @Override protected void onStop() { super.onStop(); if (bluetoothService != null) { bluetoothService.stop(); Log.d(Constants.TAG, "Stopping"); } unregisterReceiver(mReceiver); } @Override protected void onActivityResult(int requestCode, int resultCode, Intent data) { super.onActivityResult(requestCode, resultCode, data); if (requestCode == Constants.REQUEST_ENABLE_BT) { if (resultCode == RESULT_OK) { setStatus("None"); } else { setStatus("Error"); Snackbar.make(coordinatorLayout, "Failed to enable bluetooth", Snackbar.LENGTH_INDEFINITE) .setAction("Try Again", new View.OnClickListener() { @Override public void onClick(View v) { enableBluetooth(); } }).show(); } } } private void sendMessage(String message) { if (message.length() > 1) { /////string is of 8 characters buttons_state = message; // Log.d("mmeessaaggee","Alarm "+message); } else { //single character string if (Character.isUpperCase(message.charAt(0))) { buttons_state = buttons_state.replace(message.toLowerCase(), message.toUpperCase()); } else { buttons_state = buttons_state.replace(message.toUpperCase(), message.toLowerCase()); } } // Check that we're actually connected before trying anything if (bluetoothService.getState() != Constants.STATE_CONNECTED) { Snackbar.make(coordinatorLayout, "You are not connected", Snackbar.LENGTH_LONG) .setAction("Connect", new View.OnClickListener() { @Override public void onClick(View v) { reconnect(); } }).show(); return; } else { // Snackbar.make(coordinatorLayout, message, Snackbar.LENGTH_LONG) // .setAction("Connect", new View.OnClickListener() { // @Override public void onClick(View v) { // reconnect(); // } // }).show(); byte[] send = message.getBytes(); bluetoothService.write(send); } } private static class myHandler extends Handler { private final WeakReference<BluetoothActivity> mActivity; public myHandler(BluetoothActivity activity) { mActivity = new WeakReference<>(activity); } @Override public void handleMessage(Message msg) { final BluetoothActivity activity = mActivity.get(); switch (msg.what) { case Constants.MESSAGE_STATE_CHANGE: switch (msg.arg1) { case Constants.STATE_CONNECTED: activity.setStatus("Connected"); activity.reconnectButton.setVisible(false); activity.toolbalProgressBar.setVisibility(View.GONE); // Toast.makeText(activity.getApplicationContext(),State,Toast.LENGTH_LONG).show(); if (!activity.State.equals("null")) { activity.sendMessage(AppUtils.getAlarmState(activity.getApplicationContext(), Constant.BUTTON_ALARM_STATE)); activity.State = "null"; AppUtils.saveAlarmState(activity.getApplicationContext(), Constant.BUTTON_ALARM_STATE, "null"); try { Thread.sleep(3000); } catch (InterruptedException e) { e.printStackTrace(); } activity.stopActivity(); } break; case Constants.STATE_CONNECTING: activity.setStatus("Connecting"); activity.toolbalProgressBar.setVisibility(View.VISIBLE); break; case Constants.STATE_NONE: activity.setStatus("Not Connected"); activity.toolbalProgressBar.setVisibility(View.GONE); break; case Constants.STATE_ERROR: try { activity.setStatus("Error"); activity.reconnectButton.setVisible(true); activity.toolbalProgressBar.setVisibility(View.GONE); } catch (Exception e) { } if (!activity.State.equals("null")) { activity.reconnect(); } break; } break; case Constants.MESSAGE_WRITE: byte[] writeBuf = (byte[]) msg.obj; // construct a string from the buffer String writeMessage = new String(writeBuf); // ChatMessage messageWrite = new ChatMessage("Me", writeMessage); //activity.addMessageToAdapter(messageWrite); break; case Constants.MESSAGE_READ: String readMessage = (String) msg.obj; if (readMessage != null && activity.showMessagesIsChecked) { ChatMessage messageRead = new ChatMessage(activity.device.getName(), readMessage.trim()); activity.addMessageToAdapter(messageRead); } break; case Constants.MESSAGE_SNACKBAR: try { Snackbar.make(activity.coordinatorLayout, msg.getData().getString(Constants.SNACKBAR), Snackbar.LENGTH_LONG) .setAction("Connect", new View.OnClickListener() { @Override public void onClick(View v) { activity.reconnect(); } }).show(); } catch (Exception e) { e.printStackTrace(); } break; } } } private void addMessageToAdapter(ChatMessage chatMessage) { // Snackbar.make(coordinatorLayout, chatMessage.getMessage(), Snackbar.LENGTH_LONG) // .setAction("Connect", new View.OnClickListener() { // @Override public void onClick(View v) { // reconnect(); // } // }).show(); } private void scrollChatListViewToBottom() { // chatListView.post(new Runnable() { // @Override // public void run() { // // Select the last row so it will scroll into view... // chatListView.smoothScrollToPosition(chatAdapter.getCount() - 1); // } // }); } @Override public boolean onCreateOptionsMenu(Menu menu) { // Inflate the menu; this adds items to the action bar if it is present. getMenuInflater().inflate(R.menu.bluetooth_menu, menu); reconnectButton = menu.findItem(R.id.action_reconnect); menu.findItem(R.id.action_clear).setVisible(false); menu.findItem(R.id.checkable_auto_scroll).setVisible(false); menu.findItem(R.id.checkable_show_messages).setVisible(true); menu.findItem(R.id.checkable_show_time).setVisible(false); return true; } @Override public boolean onOptionsItemSelected(MenuItem item) { switch (item.getItemId()) { // Respond to the action bar's Up/Home button case android.R.id.home: bluetoothService.stop(); NavUtils.navigateUpFromSameTask(this); return true; case R.id.action_reconnect: reconnect(); return true; case R.id.action_clear: // chatAdapter.clear(); return true; case R.id.checkable_auto_scroll: autoScrollIsChecked = !item.isChecked(); item.setChecked(autoScrollIsChecked); return true; case R.id.checkable_show_messages: openTimePickerDialog(false); return true; case R.id.checkable_show_time: showTimeIsChecked = !item.isChecked(); item.setChecked(showTimeIsChecked); // chatAdapter.notifyDataSetChanged(); return true; } return super.onOptionsItemSelected(item); } private final BroadcastReceiver mReceiver = new BroadcastReceiver() { public void onReceive(Context context, Intent intent) { String action = intent.getAction(); if (action.equals(BluetoothAdapter.ACTION_STATE_CHANGED)) { int state = intent.getIntExtra(BluetoothAdapter.EXTRA_STATE, BluetoothAdapter.ERROR); switch (state) { case BluetoothAdapter.STATE_OFF: snackTurnOn.show(); break; case BluetoothAdapter.STATE_TURNING_ON: if (snackTurnOn.isShownOrQueued()) snackTurnOn.dismiss(); break; case BluetoothAdapter.STATE_ON: reconnect(); } } } }; private void setStatus(String status) { toolbar.setSubtitle(status); } private void enableBluetooth() { setStatus("Enabling Bluetooth"); Intent enableBtIntent = new Intent(BluetoothAdapter.ACTION_REQUEST_ENABLE); startActivityForResult(enableBtIntent, Constants.REQUEST_ENABLE_BT); } private void reconnect() { try { reconnectButton.setVisible(false); } catch (Exception e) { e.printStackTrace(); } try { bluetoothService.stop(); bluetoothService.connect(); } catch (Exception e) { e.printStackTrace(); } } }
{ "url": "https://www.programcreek.com/java-api-examples/?code=bilal-rashid/Lazy-Switches/Lazy-Switches-master/app/src/main/java/com/menthoven/arduinoandroid/BluetoothActivity.java", "source_domain": "www.programcreek.com", "snapshot_id": "crawl=CC-MAIN-2020-29", "warc_metadata": { "Content-Length": "76949", "Content-Type": "application/http; msgtype=response", "WARC-Block-Digest": "sha1:ND6NQVOI2ENGTSSTGBETT7QCRH3TB6WR", "WARC-Concurrent-To": "<urn:uuid:493fb8ad-9d60-45cd-8dce-1ddc10204000>", "WARC-Date": "2020-07-03T09:36:32Z", "WARC-IP-Address": "104.26.7.70", "WARC-Identified-Payload-Type": "text/html", "WARC-Payload-Digest": "sha1:NPADXFTEJJPRZ6FC5L34WLQF4AMTWPVH", "WARC-Record-ID": "<urn:uuid:18881be1-8f3a-4a1e-a03b-d928b5efced1>", "WARC-Target-URI": "https://www.programcreek.com/java-api-examples/?code=bilal-rashid/Lazy-Switches/Lazy-Switches-master/app/src/main/java/com/menthoven/arduinoandroid/BluetoothActivity.java", "WARC-Truncated": null, "WARC-Type": "response", "WARC-Warcinfo-ID": "<urn:uuid:e9e2358a-dd1c-4a3f-9329-d23351d7abca>" }, "warc_info": "isPartOf: CC-MAIN-2020-29\r\npublisher: Common Crawl\r\ndescription: Wide crawl of the web for July 2020\r\noperator: Common Crawl Admin ([email protected])\r\nhostname: ip-10-67-67-204.ec2.internal\r\nsoftware: Apache Nutch 1.17 (modified, https://github.com/commoncrawl/nutch/)\r\nrobots: checked via crawler-commons 1.2-SNAPSHOT (https://github.com/crawler-commons/crawler-commons)\r\nformat: WARC File Format 1.1\r\nconformsTo: http://iipc.github.io/warc-specifications/specifications/warc-format/warc-1.1/" }
{ "line_start_idx": [ 0, 38, 39, 72, 106, 143, 186, 228, 270, 302, 342, 373, 410, 453, 478, 504, 531, 558, 590, 646, 693, 733, 782, 825, 874, 916, 941, 967, 997, 1023, 1059, 1094, 1128, 1157, 1158, 1219, 1271, 1323, 1324, 1360, 1387, 1413, 1414, 1439, 1471, 1507, 1508, 1567, 1568, 1569, 1608, 1636, 1660, 1681, 1718, 1754, 1799, 1840, 1864, 1885, 1909, 1930, 1954, 1975, 1999, 2020, 2044, 2065, 2089, 2110, 2134, 2155, 2179, 2200, 2224, 2245, 2270, 2292, 2322, 2348, 2398, 2446, 2498, 2566, 2598, 2637, 2669, 2670, 2671, 2708, 2754, 2814, 2828, 2885, 2929, 2982, 2983, 2984, 3016, 3017, 3124, 3191, 3221, 3271, 3314, 3336, 3356, 3357, 3358, 3396, 3397, 3464, 3465, 3466, 3541, 3604, 3605, 3685, 3753, 3821, 3947, 3948, 4014, 4015, 4016, 4106, 4198, 4240, 4281, 4369, 4459, 4500, 4540, 4630, 4722, 4763, 4805, 4806, 4859, 4911, 4956, 5001, 5046, 5091, 5136, 5181, 5226, 5271, 5272, 5330, 5388, 5446, 5504, 5562, 5620, 5678, 5736, 5737, 5784, 5831, 5878, 5925, 5972, 6019, 6066, 6113, 6114, 6150, 6214, 6236, 6281, 6357, 6395, 6453, 6454, 6479, 6517, 6576, 6577, 6595, 6596, 6610, 6622, 6623, 6687, 6709, 6754, 6830, 6868, 6926, 6927, 6952, 6990, 7049, 7050, 7068, 7082, 7094, 7095, 7159, 7181, 7226, 7302, 7340, 7398, 7399, 7424, 7462, 7521, 7522, 7540, 7554, 7566, 7567, 7631, 7653, 7698, 7774, 7812, 7870, 7871, 7896, 7934, 7993, 7994, 8012, 8026, 8038, 8039, 8103, 8125, 8170, 8246, 8284, 8342, 8343, 8368, 8406, 8465, 8466, 8484, 8498, 8510, 8511, 8575, 8597, 8642, 8718, 8756, 8814, 8815, 8840, 8878, 8937, 8938, 8956, 8970, 8982, 8983, 9047, 9069, 9114, 9190, 9228, 9286, 9287, 9312, 9350, 9409, 9410, 9428, 9442, 9454, 9455, 9519, 9541, 9586, 9662, 9700, 9758, 9759, 9784, 9822, 9881, 9882, 9900, 9914, 9926, 9990, 10012, 10057, 10111, 10165, 10219, 10273, 10327, 10381, 10435, 10489, 10530, 10544, 10556, 10621, 10643, 10688, 10743, 10798, 10853, 10908, 10963, 11018, 11073, 11128, 11169, 11183, 11195, 11196, 11202, 11203, 11256, 11307, 11384, 11412, 11455, 11496, 11530, 11572, 11612, 11646, 11689, 11730, 11746, 11785, 11823, 11833, 11834, 11840, 11841, 11902, 11953, 11983, 12059, 12132, 12138, 12139, 12201, 12252, 12283, 12360, 12435, 12441, 12442, 12515, 12516, 12587, 12609, 12661, 12713, 12820, 12874, 12875, 12933, 13052, 13095, 13136, 13180, 13222, 13223, 13321, 13351, 13428, 13470, 13492, 13512, 13513, 13608, 13638, 13715, 13785, 13840, 13947, 14030, 14115, 14152, 14238, 14268, 14318, 14378, 14485, 14568, 14652, 14689, 14774, 14804, 14854, 14917, 15024, 15107, 15192, 15229, 15315, 15345, 15395, 15428, 15535, 15613, 15663, 15689, 15690, 15712, 15732, 15770, 15799, 15813, 15825, 15826, 15827, 15833, 15834, 15889, 15941, 15942, 16014, 16085, 16140, 16193, 16194, 16227, 16228, 16234, 16235, 16337, 16338, 16356, 16432, 16433, 16487, 16544, 16545, 16602, 16651, 16695, 16744, 16745, 16794, 16854, 16900, 16914, 16915, 16945, 16955, 16962, 16963, 17011, 17041, 17140, 17183, 17220, 17255, 17358, 17359, 17360, 17475, 17476, 17551, 17608, 17672, 17743, 17845, 17846, 17912, 17965, 18057, 18136, 18168, 18169, 18175, 18176, 18190, 18221, 18246, 18296, 18361, 18406, 18407, 18442, 18478, 18522, 18628, 18742, 18771, 18874, 18952, 18986, 18992, 18993, 19026, 19057, 19126, 19163, 19164, 19199, 19216, 19253, 19275, 19285, 19291, 19292, 19306, 19336, 19360, 19400, 19437, 19483, 19493, 19494, 19533, 19539, 19540, 19554, 19638, 19701, 19759, 19802, 19837, 19858, 19894, 20001, 20078, 20116, 20174, 20225, 20255, 20290, 20304, 20314, 20315, 20321, 20322, 20323, 20370, 20406, 20449, 20486, 20541, 20558, 20596, 20656, 20757, 20758, 20779, 20880, 20894, 20904, 20974, 21046, 21138, 21209, 21243, 21297, 21338, 21364, 21395, 21415, 21432, 21510, 21583, 21649, 21692, 21720, 21753, 21754, 21755, 21801, 21843, 21844, 21854, 21860, 21861, 21862, 21915, 21981, 21982, 22037, 22092, 22102, 22103, 22121, 22170, 22171, 22235, 22236, 22268, 22321, 22361, 22417, 22478, 22550, 22632, 22743, 22809, 22919, 22990, 23047, 23166, 23215, 23253, 23309, 23376, 23433, 23467, 23524, 23554, 23555, 23590, 23647, 23709, 23794, 23829, 23880, 23945, 24027, 24062, 24114, 24148, 24149, 24210, 24285, 24371, 24423, 24453, 24454, 24520, 24521, 24575, 24605, 24640, 24662, 24689, 24735, 24791, 24849, 24913, 25001, 25067, 25094, 25139, 25140, 25199, 25200, 25281, 25395, 25462, 25463, 25485, 25512, 25513, 25562, 25588, 25721, 25804, 25850, 25916, 25978, 26016, 26059, 26103, 26148, 26170, 26171, 26198, 26212, 26222, 26223, 26224, 26230, 26231, 26295, 26296, 26297, 26388, 26457, 26519, 26558, 26582, 26611, 26612, 26618, 26619, 26667, 26712, 26736, 26770, 26842, 26925, 26941, 26955, 26961, 26962, 26976, 27028, 27109, 27173, 27237, 27297, 27366, 27436, 27503, 27524, 27530, 27531, 27532, 27546, 27604, 27640, 27698, 27734, 27775, 27830, 27859, 27899, 27928, 27957, 27993, 28033, 28062, 28107, 28164, 28218, 28247, 28294, 28339, 28368, 28411, 28466, 28518, 28573, 28602, 28612, 28662, 28668, 28669, 28743, 28807, 28855, 28856, 28928, 29030, 29063, 29116, 29160, 29191, 29251, 29333, 29364, 29416, 29453, 29471, 29485, 29495, 29502, 29503, 29547, 29584, 29590, 29591, 29628, 29669, 29753, 29830, 29836, 29837, 29868, 29882, 29929, 29930, 29962, 29995, 30005, 30019, 30056, 30096, 30128, 30161, 30171, 30172, 30178, 30179 ], "line_end_idx": [ 38, 39, 72, 106, 143, 186, 228, 270, 302, 342, 373, 410, 453, 478, 504, 531, 558, 590, 646, 693, 733, 782, 825, 874, 916, 941, 967, 997, 1023, 1059, 1094, 1128, 1157, 1158, 1219, 1271, 1323, 1324, 1360, 1387, 1413, 1414, 1439, 1471, 1507, 1508, 1567, 1568, 1569, 1608, 1636, 1660, 1681, 1718, 1754, 1799, 1840, 1864, 1885, 1909, 1930, 1954, 1975, 1999, 2020, 2044, 2065, 2089, 2110, 2134, 2155, 2179, 2200, 2224, 2245, 2270, 2292, 2322, 2348, 2398, 2446, 2498, 2566, 2598, 2637, 2669, 2670, 2671, 2708, 2754, 2814, 2828, 2885, 2929, 2982, 2983, 2984, 3016, 3017, 3124, 3191, 3221, 3271, 3314, 3336, 3356, 3357, 3358, 3396, 3397, 3464, 3465, 3466, 3541, 3604, 3605, 3685, 3753, 3821, 3947, 3948, 4014, 4015, 4016, 4106, 4198, 4240, 4281, 4369, 4459, 4500, 4540, 4630, 4722, 4763, 4805, 4806, 4859, 4911, 4956, 5001, 5046, 5091, 5136, 5181, 5226, 5271, 5272, 5330, 5388, 5446, 5504, 5562, 5620, 5678, 5736, 5737, 5784, 5831, 5878, 5925, 5972, 6019, 6066, 6113, 6114, 6150, 6214, 6236, 6281, 6357, 6395, 6453, 6454, 6479, 6517, 6576, 6577, 6595, 6596, 6610, 6622, 6623, 6687, 6709, 6754, 6830, 6868, 6926, 6927, 6952, 6990, 7049, 7050, 7068, 7082, 7094, 7095, 7159, 7181, 7226, 7302, 7340, 7398, 7399, 7424, 7462, 7521, 7522, 7540, 7554, 7566, 7567, 7631, 7653, 7698, 7774, 7812, 7870, 7871, 7896, 7934, 7993, 7994, 8012, 8026, 8038, 8039, 8103, 8125, 8170, 8246, 8284, 8342, 8343, 8368, 8406, 8465, 8466, 8484, 8498, 8510, 8511, 8575, 8597, 8642, 8718, 8756, 8814, 8815, 8840, 8878, 8937, 8938, 8956, 8970, 8982, 8983, 9047, 9069, 9114, 9190, 9228, 9286, 9287, 9312, 9350, 9409, 9410, 9428, 9442, 9454, 9455, 9519, 9541, 9586, 9662, 9700, 9758, 9759, 9784, 9822, 9881, 9882, 9900, 9914, 9926, 9990, 10012, 10057, 10111, 10165, 10219, 10273, 10327, 10381, 10435, 10489, 10530, 10544, 10556, 10621, 10643, 10688, 10743, 10798, 10853, 10908, 10963, 11018, 11073, 11128, 11169, 11183, 11195, 11196, 11202, 11203, 11256, 11307, 11384, 11412, 11455, 11496, 11530, 11572, 11612, 11646, 11689, 11730, 11746, 11785, 11823, 11833, 11834, 11840, 11841, 11902, 11953, 11983, 12059, 12132, 12138, 12139, 12201, 12252, 12283, 12360, 12435, 12441, 12442, 12515, 12516, 12587, 12609, 12661, 12713, 12820, 12874, 12875, 12933, 13052, 13095, 13136, 13180, 13222, 13223, 13321, 13351, 13428, 13470, 13492, 13512, 13513, 13608, 13638, 13715, 13785, 13840, 13947, 14030, 14115, 14152, 14238, 14268, 14318, 14378, 14485, 14568, 14652, 14689, 14774, 14804, 14854, 14917, 15024, 15107, 15192, 15229, 15315, 15345, 15395, 15428, 15535, 15613, 15663, 15689, 15690, 15712, 15732, 15770, 15799, 15813, 15825, 15826, 15827, 15833, 15834, 15889, 15941, 15942, 16014, 16085, 16140, 16193, 16194, 16227, 16228, 16234, 16235, 16337, 16338, 16356, 16432, 16433, 16487, 16544, 16545, 16602, 16651, 16695, 16744, 16745, 16794, 16854, 16900, 16914, 16915, 16945, 16955, 16962, 16963, 17011, 17041, 17140, 17183, 17220, 17255, 17358, 17359, 17360, 17475, 17476, 17551, 17608, 17672, 17743, 17845, 17846, 17912, 17965, 18057, 18136, 18168, 18169, 18175, 18176, 18190, 18221, 18246, 18296, 18361, 18406, 18407, 18442, 18478, 18522, 18628, 18742, 18771, 18874, 18952, 18986, 18992, 18993, 19026, 19057, 19126, 19163, 19164, 19199, 19216, 19253, 19275, 19285, 19291, 19292, 19306, 19336, 19360, 19400, 19437, 19483, 19493, 19494, 19533, 19539, 19540, 19554, 19638, 19701, 19759, 19802, 19837, 19858, 19894, 20001, 20078, 20116, 20174, 20225, 20255, 20290, 20304, 20314, 20315, 20321, 20322, 20323, 20370, 20406, 20449, 20486, 20541, 20558, 20596, 20656, 20757, 20758, 20779, 20880, 20894, 20904, 20974, 21046, 21138, 21209, 21243, 21297, 21338, 21364, 21395, 21415, 21432, 21510, 21583, 21649, 21692, 21720, 21753, 21754, 21755, 21801, 21843, 21844, 21854, 21860, 21861, 21862, 21915, 21981, 21982, 22037, 22092, 22102, 22103, 22121, 22170, 22171, 22235, 22236, 22268, 22321, 22361, 22417, 22478, 22550, 22632, 22743, 22809, 22919, 22990, 23047, 23166, 23215, 23253, 23309, 23376, 23433, 23467, 23524, 23554, 23555, 23590, 23647, 23709, 23794, 23829, 23880, 23945, 24027, 24062, 24114, 24148, 24149, 24210, 24285, 24371, 24423, 24453, 24454, 24520, 24521, 24575, 24605, 24640, 24662, 24689, 24735, 24791, 24849, 24913, 25001, 25067, 25094, 25139, 25140, 25199, 25200, 25281, 25395, 25462, 25463, 25485, 25512, 25513, 25562, 25588, 25721, 25804, 25850, 25916, 25978, 26016, 26059, 26103, 26148, 26170, 26171, 26198, 26212, 26222, 26223, 26224, 26230, 26231, 26295, 26296, 26297, 26388, 26457, 26519, 26558, 26582, 26611, 26612, 26618, 26619, 26667, 26712, 26736, 26770, 26842, 26925, 26941, 26955, 26961, 26962, 26976, 27028, 27109, 27173, 27237, 27297, 27366, 27436, 27503, 27524, 27530, 27531, 27532, 27546, 27604, 27640, 27698, 27734, 27775, 27830, 27859, 27899, 27928, 27957, 27993, 28033, 28062, 28107, 28164, 28218, 28247, 28294, 28339, 28368, 28411, 28466, 28518, 28573, 28602, 28612, 28662, 28668, 28669, 28743, 28807, 28855, 28856, 28928, 29030, 29063, 29116, 29160, 29191, 29251, 29333, 29364, 29416, 29453, 29471, 29485, 29495, 29502, 29503, 29547, 29584, 29590, 29591, 29628, 29669, 29753, 29830, 29836, 29837, 29868, 29882, 29929, 29930, 29962, 29995, 30005, 30019, 30056, 30096, 30128, 30161, 30171, 30172, 30178, 30179, 30180 ] }
{ "red_pajama_v2": { "ccnet_original_length": 30180, "ccnet_original_nlines": 787, "rps_doc_curly_bracket": 0.008482440374791622, "rps_doc_ldnoobw_words": 0, "rps_doc_lorem_ipsum": 0, "rps_doc_stop_word_fraction": 0.041099000722169876, "rps_doc_ut1_blacklist": 0, "rps_doc_frac_all_caps_words": 0.03996367007493973, "rps_doc_frac_lines_end_with_ellipsis": 0.0012690400471910834, "rps_doc_frac_no_alph_words": 0.5022706389427185, "rps_doc_frac_unique_words": 0.44512662291526794, "rps_doc_mean_word_length": 12.780506134033203, "rps_doc_num_sentences": 605, "rps_doc_symbol_to_word_ratio": 0.0002270700060762465, "rps_doc_unigram_entropy": 5.710509777069092, "rps_doc_word_count": 1303, "rps_doc_frac_chars_dupe_10grams": 0.0476190485060215, "rps_doc_frac_chars_dupe_5grams": 0.11607518047094345, "rps_doc_frac_chars_dupe_6grams": 0.10778839141130447, "rps_doc_frac_chars_dupe_7grams": 0.09721972048282623, "rps_doc_frac_chars_dupe_8grams": 0.07127845287322998, "rps_doc_frac_chars_dupe_9grams": 0.06323184818029404, "rps_doc_frac_chars_top_2gram": 0.015612799674272537, "rps_doc_frac_chars_top_3gram": 0.02269860915839672, "rps_doc_frac_chars_top_4gram": 0.03554914891719818, "rps_doc_books_importance": -2590.26611328125, "rps_doc_books_importance_length_correction": -2590.26611328125, "rps_doc_openwebtext_importance": -1372.4202880859375, "rps_doc_openwebtext_importance_length_correction": -1372.4202880859375, "rps_doc_wikipedia_importance": -1471.6536865234375, "rps_doc_wikipedia_importance_length_correction": -1471.6536865234375 }, "fasttext": { "dclm": 0.9985021352767944, "english": 0.1746702492237091, "fineweb_edu_approx": 2.318868637084961, "eai_general_math": 0.30876290798187256, "eai_open_web_math": 0.008463320322334766, "eai_web_code": 0.9787327647209167 } }
{ "free_decimal_correspondence": { "primary": { "code": "004.0285636", "labels": { "level_1": "General works, books and libraries, information sciences", "level_2": "", "level_3": "Computers and Computer science" } }, "secondary": { "code": "621.392", "labels": { "level_1": "Industrial arts, Technology, and Engineering", "level_2": "Engineering", "level_3": "Mechanical engineering and Machinery" } } }, "bloom_cognitive_process": { "primary": { "code": "3", "label": "Apply" }, "secondary": { "code": "2", "label": "Understand" } }, "bloom_knowledge_domain": { "primary": { "code": "3", "label": "Procedural" }, "secondary": { "code": "2", "label": "Conceptual" } }, "document_type_v1": { "primary": { "code": "4", "label": "Code/Software" }, "secondary": { "code": "3", "label": "Reference/Encyclopedic/Educational" } }, "extraction_artifacts": { "primary": { "code": "0", "label": "No Artifacts" }, "secondary": { "code": "3", "label": "Irrelevant Content" } }, "missing_content": { "primary": { "code": "0", "label": "No missing content" }, "secondary": { "code": "-1", "label": "Abstain" } }, "document_type_v2": { "primary": { "code": "8", "label": "Documentation" }, "secondary": { "code": "23", "label": "Tutorial" } }, "reasoning_depth": { "primary": { "code": "3", "label": "Intermediate Reasoning" }, "secondary": { "code": "2", "label": "Basic Reasoning" } }, "technical_correctness": { "primary": { "code": "4", "label": "Highly Correct" }, "secondary": { "code": "3", "label": "Mostly Correct" } }, "education_level": { "primary": { "code": "3", "label": "Undergraduate Level" }, "secondary": { "code": "4", "label": "Graduate/Expert Level" } } }
672f1e42c33a7f9846924a2431ea77df
-8,357,870,260,348,939,000
SQLServerCentral.com / Administering / SQL Server 2005 / DBCC CHECKDB - how use it in a maintenance plan ? / Latest PostsInstantForum.NET v2.9.0SQLServerCentral.comhttp://www.sqlservercentral.com/Forums/[email protected], 01 Apr 2015 04:59:26 GMT20RE: DBCC CHECKDB - how use it in a maintenance plan ?http://www.sqlservercentral.com/Forums/Topic469988-146-1.aspxHi, woops - thought the other reply disappeared.Fri, 14 Mar 2014 02:11:31 GMTDavidB 36561RE: DBCC CHECKDB - how use it in a maintenance plan ?http://www.sqlservercentral.com/Forums/Topic469988-146-1.aspxHi, (Caveat - I am only a very part time self taught SQL Admin and have been relying on things read recently.)If the Maintenance tasks are linked within the one maintenance plan and you start with the verify then the back up will not proceed unless the dbcc checkdb suceeds. What I haven't yet determined is what the DBCC check task classes as a success - presumably no errors.But you may still want the backup to proceed even if there are errors - but what you definitely do not want is the old backups to be removed as they may be needed if your database is indeed nastily corrupted?So it seems that doing the backup first, then the checkdb, then the cleanup of old .bak files then trn files is a good option. The backup still happens but the checkdb will, if it fails, prevent the bak and trn files that you might need from being removed. Obviously space could become an issue if it is left unresolved for a period. Also this relies on no other maintenance plan deleting the .bak or .trn files. Does this sounds sensible to those with lots of experience?Fri, 14 Mar 2014 00:26:44 GMTDavidB 36561RE: DBCC CHECKDB - how use it in a maintenance plan ?http://www.sqlservercentral.com/Forums/Topic469988-146-1.aspxPlease rather post new questions in a new thread. ThanksMon, 27 Sep 2010 12:06:10 GMTGilaMonsterRE: DBCC CHECKDB - how use it in a maintenance plan ?http://www.sqlservercentral.com/Forums/Topic469988-146-1.aspxI know this is an old thread, but a question.If you have a SQL job running DBCC checks, and a check finds an error, does it raise an error so the job reports failure, or do you need to parse through the logs it generates each time it runs and look for errors manually?Mon, 27 Sep 2010 11:59:22 GMTThe Redneck DBARE: DBCC CHECKDB - how use it in a maintenance plan ?http://www.sqlservercentral.com/Forums/Topic469988-146-1.aspx[quote][b]Oksana March (11/29/2008)[/b][hr]Hi Gail,"Stop SQL" you mean stop SQL Server Agent in the Control Panel--Services?[/quote]No. I mean stop SQL server. The SQL service must be stopped before you can access the data files.[quote] And I just want to confirm: if I create a database to test this *corrupting on purpose*, this will not touch any other database on the same server? I will try it :)[/quote]It won't. However... Do [b]not [/b]do this on a production server. Do not do this anywhere except on a local server on your local machine.This is not something you should be playing with on a SQL instance that's of any importance to anyone but you [quote]Then when I accomplish corrupting it, what is the best exercises to try? Thanks![/quote]See if you can recover it...See these posts for some things to do and try and techniques to recover:[url]http://www.sqlskills.com/BLOGS/PAUL/category/Corruption.aspx[/url]Again, just for emphasise (and for anyone else reading), do not do this on any server of any importance. Preferably install SQL on your local machine (express, or developer edition if you have it) and play thereSat, 29 Nov 2008 09:28:55 GMTGilaMonsterRE: DBCC CHECKDB - how use it in a maintenance plan ?http://www.sqlservercentral.com/Forums/Topic469988-146-1.aspxThank you for answering this one, by the way![quote][b]GilaMonster (11/29/2008)[/b][hr][quote][b]Oksana March (11/29/2008)[/b][hr]I have a question about corruption. How often does it really happen? The previous DBA said it never happen to us in 7 years that she was here and we have a big database, not as big like a bank but it's big, we store information on raw metals from many warehouses and all orders of them, and much more.[/quote]It depends on your hardware. Corruption's typically a hardware problem, so if you have a rock-solid IO subsystem, you shouldn't see much, if any corruption. If, however, the disks are near failure, the SAN controller's buggy or there's a misbehaving filter driver (like anti-virus) you may see a great deal of corruption.For what it's worth, I saw corruption twice in the four years I worked at the bank. Once was a corrupted log record which required a full restore of the DB. The other time wasn't my system (fortunately). For reasons I won't go into, there was no offsite backup. The san controller glitched and spewed garbage across two LUNs, corrupting the only backup and the file header of one of the files in the primary file group.The database was a complete and total loss. They never got it back.[/quote]Sat, 29 Nov 2008 08:43:36 GMTOksana MarchRE: DBCC CHECKDB - how use it in a maintenance plan ?http://www.sqlservercentral.com/Forums/Topic469988-146-1.aspxHi Gail,"Stop SQL" you mean stop SQL Server Agent in the Control Panel--Services? And I just want to confirm: if I create a database to test this *corrupting on purpose*, this will not touch any other database on the same server? I will try it :)Then when I accomplish corrupting it, what is the best exercises to try? Thanks![quote][b]GilaMonster (10/16/2008)[/b][hr][quote]Do you know how I can go about deliberately corrupting a database. I'd like to test some different things, but I'm not sure how to go about it. [/quote]Sure. First though, a disclaimer for anyone else who wants to try this. Do not do this to a DB you care about. Do not do this to a database that you're not willing to discard. Do not ever consider doing it to a production database. Stop SQL. Open the mdf file in a hex editor. Go at least 800kb into the database (to avoid corrupting the system tables) and then randomly change values. Save the file and exit, then restart SQLOr, if you're feeling adventurous, you can use SQL Server as a hex editor and get it to corrupt one of its own databases.[url]http://sqlblogcasts.com/blogs/tonyrogerson/archive/2007/03/10/how-to-create-a-corrupt-database-using-bulk-insert-update-and-bcp-sql-server-as-a-hex-editor.aspx[/url][/quote]Sat, 29 Nov 2008 08:33:57 GMTOksana MarchRE: DBCC CHECKDB - how use it in a maintenance plan ?http://www.sqlservercentral.com/Forums/Topic469988-146-1.aspx[quote][b]Oksana March (11/29/2008)[/b][hr]I have a question about corruption. How often does it really happen? The previous DBA said it never happen to us in 7 years that she was here and we have a big database, not as big like a bank but it's big, we store information on raw metals from many warehouses and all orders of them, and much more.[/quote]It depends on your hardware. Corruption's typically a hardware problem, so if you have a rock-solid IO subsystem, you shouldn't see much, if any corruption. If, however, the disks are near failure, the SAN controller's buggy or there's a misbehaving filter driver (like anti-virus) you may see a great deal of corruption.For what it's worth, I saw corruption twice in the four years I worked at the bank. Once was a corrupted log record which required a full restore of the DB. The other time wasn't my system (fortunately). For reasons I won't go into, there was no offsite backup. The san controller glitched and spewed garbage across two LUNs, corrupting the only backup and the file header of one of the files in the primary file group.The database was a complete and total loss. They never got it back.Sat, 29 Nov 2008 03:38:12 GMTGilaMonsterRE: DBCC CHECKDB - how use it in a maintenance plan ?http://www.sqlservercentral.com/Forums/Topic469988-146-1.aspxI have a question about corruption. How often does it really happen? The previous DBA said it never happen to us in 7 years that she was here and we have a big database, not as big like a bank but it's big, we store information on raw metals from many warehouses and all orders of them, and much more.Sat, 29 Nov 2008 02:49:28 GMTOksana MarchRE: DBCC CHECKDB - how use it in a maintenance plan ?http://www.sqlservercentral.com/Forums/Topic469988-146-1.aspx[quote][b]herbiechin (11/28/2008)[/b][hr]You don't and as far as I am aware, the backup will happen disregard if any error/corruption were found in the database. Not sure if there is an easy way to catch this (others may have ideas on this) [/quote]In 2005 and higher, if the backup is done with the checksum option then SQL will recalculate the page checksums (providing page checksum is turned on, which it should be) as it takes the backup. If any page fails a checksum check, the backup is aborted.[quote]"Use the REPAIR options only as a last resort."[/quote]Absolutely!I'd go as far as saying if you're not 100% sure what to do when you get corruption, ask someone who does know. Maybe ask here, maybe ask on another forum, maybe mail a more-experienced DBA, maybe call CSSBetter yet, research now when you don't have corruption and practice various recovery options so that if you do get it, you know exactly what to do.Fri, 28 Nov 2008 12:56:17 GMTGilaMonsterRE: DBCC CHECKDB - how use it in a maintenance plan ?http://www.sqlservercentral.com/Forums/Topic469988-146-1.aspx[quote][b]Leo (10/16/2008)[/b][hr]Hi Gila,According from 'dbcc checkdb (dbname) with no_infomsgs.' , you said runthat command before backup run. But my questions is, what do we do if I already setup a job like21:00 - dbcc checkdb (dbname) with no_infomsgs22:00 - DBCC BackupDB(DBname)23:00 - DBCC RebuildIndex 1. How do we know database is corrupt before the Backup at 22:00? Job is already setup and backup the corrupt database anyway according from the JOB. Any way to stop that, NOT to backup?2. Which command can fix and repair the Database if 'dbcc checkdb' result is failed.3. Do you have any sample code to check database, run the db repair if needed then backup and rebuild DB?Thanks.[/quote]You don't and as far as I am aware, the backup will happen disregard if any error/corruption were found in the database. Not sure if there is an easy way to catch this (others may have ideas on this) In a wider context, I do not see if there is any need to do so. If setup properly, I would recommend the maintenance task results (Success or Fail) to be confirmed by email to the Administrator. Failing that, a matter of screening through the error logs may be sufficient but it can be a real pain if you have dozens of database to administer.As far as the fix and repair command goes, here is the SQL Server 2005 Books Online recommendation:"Use the REPAIR options only as a last resort. To repair errors, we recommend restoring from a backup. Repair operations do not consider any of the constraints that may exist on or between tables. If the specified table is involved in one or more constraints, we recommend running DBCC CHECKCONSTRAINTS after a repair operation. If you must use REPAIR, run DBCC CHECKDB to find the repair level to use. If you are going to use the REPAIR_ALLOW_DATA_LOSS level, we recommend that you back up the database before you run DBCC CHECKDB."Note: The command here is referring to DBCC CHECKTABLE with REPAIR_ALLOW_DATA_LOSS levelFri, 28 Nov 2008 08:06:06 GMTherbiechinRE: DBCC CHECKDB - how use it in a maintenance plan ?http://www.sqlservercentral.com/Forums/Topic469988-146-1.aspxyour re-index script will also cause yout tlogs to grow.Thu, 27 Nov 2008 05:58:07 GMTNazier.vdrheedeRE: DBCC CHECKDB - how use it in a maintenance plan ?http://www.sqlservercentral.com/Forums/Topic469988-146-1.aspxThanks Gila and Jeff.....LeoSun, 19 Oct 2008 11:20:19 GMTLeo-696352RE: DBCC CHECKDB - how use it in a maintenance plan ?http://www.sqlservercentral.com/Forums/Topic469988-146-1.aspx[quote][b]Leo (10/19/2008)[/b][hr] But what do I do if transaction log is getting bigger, for example -Transaction log initial size 4GB [that is okay and I am happy with this size], During the day or when I check the database at night, Transaction Log is about 40GB . What do I do?[/quote]In simple recovery it shouldn't grow that much. It's growing in full recovery because it has to retain all of the log records until the log is backed up or truncated. In simple, it just has to retain the log records until an automatic checkpoint occurs.If the log grows (and it may if there are long running transactions) then, as Jeffrey said, it needs to be that size.People tend to panic when the log gets large, but it's not necessary to worry about a large log file if the size is constant. The last place I worked had a 220GB log file for the main DB. Most of the time it was nearly empty, but we could use up to 75% easily during the overnight processing.Sun, 19 Oct 2008 11:12:42 GMTGilaMonsterRE: DBCC CHECKDB - how use it in a maintenance plan ?http://www.sqlservercentral.com/Forums/Topic469988-146-1.aspx[quote][b]Leo (10/19/2008)[/b][hr]Hi Gila,Sorry, I get back to you a bit late. I am not fully understanding about Transaction Log. Let say I switched my DB Model to 'SIMPLE'. I don't need to worry about transaction log anymore as you said. But what do I do if transaction log is getting bigger, for example -Transaction log initial size 4GB [that is okay and I am happy with this size], During the day or when I check the database at night, Transaction Log is about 40GB . What do I do?I know, you will hate me but I really want to understand this which mean I want to learn from your experience. Last question.......Insert or Reading the record from Database is very slow sometime, is that anything to do with transaction log? How to solve it?Thanks again.[/quote]On your first question, if your database is in SIMPLE recovery model you don't have to worry about BACKING UP the transaction log. The transaction log is still required and will grow to the size needed to manage the largest transaction that will process on your system.If your transaction log is growing to 40GB (in SIMPLE model), then your transaction log needs to be 40GB to handle the workload. If that is not acceptable, then you need to identify the process(es) that are using that much space in the transaction log and fix those processes.One example would be an ETL process that is loading millions of rows of data in a single transaction. A process like this could easily cause the transaction log to grow that large.Second question - I would not suspect the transaction log has anything to do with your performance issues. The only way the transaction log would really affect your performance is if the file is on the same disks as the data files and you are seeing I/O contention.Sun, 19 Oct 2008 09:06:37 GMTJeffrey Williams 3188RE: DBCC CHECKDB - how use it in a maintenance plan ?http://www.sqlservercentral.com/Forums/Topic469988-146-1.aspxHi Gila,Sorry, I get back to you a bit late. I am not fully understanding about Transaction Log. Let say I switched my DB Model to 'SIMPLE'. I don't need to worry about transaction log anymore as you said. But what do I do if transaction log is getting bigger, for example -Transaction log initial size 4GB [that is okay and I am happy with this size], During the day or when I check the database at night, Transaction Log is about 40GB . What do I do?I know, you will hate me but I really want to understand this which mean I want to learn from your experience. Last question.......Insert or Reading the record from Database is very slow sometime, is that anything to do with transaction log? How to solve it?Thanks again.Sun, 19 Oct 2008 08:55:09 GMTLeo-696352RE: DBCC CHECKDB - how use it in a maintenance plan ?http://www.sqlservercentral.com/Forums/Topic469988-146-1.aspx[quote][b]Leo (10/18/2008)[/b][hr]I never backup the transaction log even database model is -FULL. transaction log backup is not required for my environment at the moment.[/quote]You need transaction log backups in full recovery because if you don't, your transaction log will grow without bound, as you're seeing. If you don't need point-in-time recovery, then switch the database to simple and you won't have to worry about the transaction logs. If you do need point-in-time recovery, then set up regular log backups.See - [url]http://sqlinthewild.co.za/index.php/2008/07/23/recovery-model-and-transaction-logs/[/url][quote]if you suggest to do that, 'Before' or 'After' my Maintenance Plan?*****BACKUP LOG t_log_name WITH TRUNCATE_ONLYDBCC SHRINKFILE (t_log_name,amount,retain )*****[/quote]Neither. Switch to simple recovery if you have no need of log backups. Then you won't have to worry about doing that at all.Sat, 18 Oct 2008 04:27:10 GMTGilaMonsterRE: DBCC CHECKDB - how use it in a maintenance plan ?http://www.sqlservercentral.com/Forums/Topic469988-146-1.aspxHi Jef and Gila,Now I understand why I shouldn't shrink the Data file, I will take 'Shrink Database' Off from Maintenance Plan. All information you guys posted are very useful. Thanks.I got another query, on my production server, all the transaction log files are very big, about 45GB but initial size is 4GB. My question is, shall I shrink the transaction log ONLY so that Log not gow too much. Shall I do that every night? I never backup the transaction log even database model is -FULL. transaction log backup is not required for my environment at the moment.if you suggest to do that, 'Before' or 'After' my Maintenance Plan?*****BACKUP LOG t_log_name WITH TRUNCATE_ONLYDBCC SHRINKFILE (t_log_name,amount,retain )*****Sat&lt;&lt;=== Here ?1.Check DB Integrity [All Database]2.Rebuild Index [Expect System Database]3.Backup Database [All Database]4.Clean Up Maintenance Plan5.Clean Up History&lt;&lt;== Here?ThanksSat, 18 Oct 2008 04:05:54 GMTLeo-696352RE: DBCC CHECKDB - how use it in a maintenance plan ?http://www.sqlservercentral.com/Forums/Topic469988-146-1.aspx[quote]2.Rebuild Index [Expect System Database]3.Shrink Database [Expect System Database][/quote]That's a combination I generally refer to as a waste of time. The shrink will cause massive fragmentation of your indexes, easily up to 90%. You're spending time, CPU and IOs to put the indexes in order (possibly growing the database) and then you're spending more time, cpu and IOs shrinking the file and shuffling the indexes. Next time any data is added, SQL will spend even more time, CPU and IOs growing the file so it has space to work.Also, repeated shrinks and grows can easily cause fragmentation at a file system level, requiring a disk defragment to fixSee - [url]http://sqlinthewild.co.za/index.php/2007/09/08/shrinking-databases/[/url]Sat, 18 Oct 2008 03:08:31 GMTGilaMonsterRE: DBCC CHECKDB - how use it in a maintenance plan ?http://www.sqlservercentral.com/Forums/Topic469988-146-1.aspxYou should not shrink your database on a regular scheduled basis. You should always maintain enough free space in the database to allow for index rebuilds and reorgs - as well as day to day growth.By shrinking your database every day - you are causing index fragmentation and file fragmentation as the database grows again.Fri, 17 Oct 2008 16:38:59 GMTJeffrey Williams 3188RE: DBCC CHECKDB - how use it in a maintenance plan ?http://www.sqlservercentral.com/Forums/Topic469988-146-1.aspxHi Everyone,I bought LiteSpeed Backup, I am using it for SQL 2000 Server. Interesting thing is some function are similar with SQL 2005 maintenance plan.However, let me get to the point...I setup 3 difference maintenance plan in Litespeed - Please analyse and suggest me if I am doing something wrongMon-Fri1.Check DB Integrity [All Database]2.Shrink Database [Expect System Database]3.Backup Database [All Database]4.Clean Up Maintenance Plan5.Clean Up HistorySat1.Check DB Integrity [All Database]2.Rebuild Index [Expect System Database]3.Shrink Database [Expect System Database]4.Backup Database [All Database]5.Clean Up Maintenance Plan6.Clean Up HistorySun1.Check DB Integrity [All Database]2.Reorganise Index [Expect System Database]3.Shrink Database [Expect System Database]4.Backup Database [All Database]5.Clean Up Maintenance Plan6.Clean Up HistoryOther Reason I am doing 'Rebuild' and 'Reorganise' on difference day becasue it take long time to index and another reason is-Reorganise an index when the degree of fragmentation is between 5 and 30% Rebuild an index when the degree of fragmentation is over 30%Is that right setup?Thanks.LeoFri, 17 Oct 2008 16:31:24 GMTLeo-696352RE: DBCC CHECKDB - how use it in a maintenance plan ?http://www.sqlservercentral.com/Forums/Topic469988-146-1.aspx[quote][b]Leo (10/16/2008)[/b][hr]1. How do we know database is corrupt before the Backup at 22:00? Job is already setup and backup the corrupt database anyway according from the JOB. Any way to stop that, NOT to backup?[/quote]You could write the results of checkDB to a table and have the backup job check that. [quote]2. Which command can fix and repair the Database if 'dbcc checkdb' result is failed.[/quote]Don't repair. If all the corruption is within nonclustered indexes, you rebuild (or drop and recreate) those indexes.If the corruption is elsewhere, you restore the last clean backup and then roll the transaction log backups forward.Repair is a last-resort when there's no good backup and no other way to get data back. It doesn't always work.Thu, 16 Oct 2008 15:46:37 GMTGilaMonsterRE: DBCC CHECKDB - how use it in a maintenance plan ?http://www.sqlservercentral.com/Forums/Topic469988-146-1.aspx[quote]Do you know how I can go about deliberately corrupting a database. I'd like to test some different things, but I'm not sure how to go about it. [/quote]Sure. First though, a disclaimer for anyone else who wants to try this. Do not do this to a DB you care about. Do not do this to a database that you're not willing to discard. Do not ever consider doing it to a production database. Stop SQL. Open the mdf file in a hex editor. Go at least 800kb into the database (to avoid corrupting the system tables) and then randomly change values. Save the file and exit, then restart SQLOr, if you're feeling adventurous, you can use SQL Server as a hex editor and get it to corrupt one of its own databases.[url]http://sqlblogcasts.com/blogs/tonyrogerson/archive/2007/03/10/how-to-create-a-corrupt-database-using-bulk-insert-update-and-bcp-sql-server-as-a-hex-editor.aspx[/url]Thu, 16 Oct 2008 15:42:32 GMTGilaMonsterRE: DBCC CHECKDB - how use it in a maintenance plan ?http://www.sqlservercentral.com/Forums/Topic469988-146-1.aspxHere you go:[url]http://blogs.msdn.com/sqlserverstorageengine/archive/2007/03/22/easy-way-to-create-a-corrupt-database-for-testing.aspx[/url]Thu, 16 Oct 2008 15:37:41 GMTJeffrey Williams 3188RE: DBCC CHECKDB - how use it in a maintenance plan ?http://www.sqlservercentral.com/Forums/Topic469988-146-1.aspxHi Gila,According from 'dbcc checkdb (dbname) with no_infomsgs.' , you said runthat command before backup run. But my questions is, what do we do if I already setup a job like21:00 - dbcc checkdb (dbname) with no_infomsgs22:00 - DBCC BackupDB(DBname)23:00 - DBCC RebuildIndex 1. How do we know database is corrupt before the Backup at 22:00? Job is already setup and backup the corrupt database anyway according from the JOB. Any way to stop that, NOT to backup?2. Which command can fix and repair the Database if 'dbcc checkdb' result is failed.3. Do you have any sample code to check database, run the db repair if needed then backup and rebuild DB?Thanks.Thu, 16 Oct 2008 15:29:51 GMTLeo-696352RE: DBCC CHECKDB - how use it in a maintenance plan ?http://www.sqlservercentral.com/Forums/Topic469988-146-1.aspxThanks, everyone! At my last place, we included the integrity checks in the maint plan. I'm not sure why they don't do that here. Next question... Do you know how I can go about deliberately corrupting a database. I'd like to test some different things, but I'm not sure how to go about it.Thu, 16 Oct 2008 15:29:07 GMTMarcia QRE: DBCC CHECKDB - how use it in a maintenance plan ?http://www.sqlservercentral.com/Forums/Topic469988-146-1.aspxI run the integrity check nightly using the PHYSICAL_ONLY option, and weekly perform a full integrity check.Thu, 16 Oct 2008 15:11:38 GMTJeffrey Williams 3188RE: DBCC CHECKDB - how use it in a maintenance plan ?http://www.sqlservercentral.com/Forums/Topic469988-146-1.aspxInto maintenance plan put "Check database integrity task". this will run DBCC Checkdb.Thu, 16 Oct 2008 14:20:39 GMTRPSqlRE: DBCC CHECKDB - how use it in a maintenance plan ?http://www.sqlservercentral.com/Forums/Topic469988-146-1.aspx[quote][b]Marcia Q (10/16/2008)[/b][hr]If you run the checkdb command without any repair arguments, does it actually DO anything to the database? Is it correct to say that without a repair argument, it's only going to provide information?[/quote]Indeed. It checks for corruption and will write results into the error log. CheckDB shouldn't be run with any of the repair options. They are there as a last resort if the corruption can't be fixed by rebuilding indexes or by restoring a clean backup.[quote]What benefit would there be from running this with no_infomsgs on a nightly basis? And if even if I left off that argument, what benefit would there be to running it as nightly job? Does it write the checkdb results to the log? [/quote] With that option, it just reports on corruption. Without that, it will give you a list of tables that it's checked and the number of rows/pages each has. Personally I find that a waste of space and unnecessary. There are easier ways to see the rowcount/pagecount of a table.Nightly may be a little more often than you need. I usually suggest once a week. If you have the time overnight, there's no harm in running it though.Thu, 16 Oct 2008 11:33:16 GMTGilaMonsterRE: DBCC CHECKDB - how use it in a maintenance plan ?http://www.sqlservercentral.com/Forums/Topic469988-146-1.aspxI have another question about checkdb. I just started at a new employer and am wondering about a nightly job that I found. Here's the only thing in it: [i]dbcc checkdb (dbname) with no_infomsgs.[/i]If you run the checkdb command without any repair arguments, does it actually DO anything to the database? Is it correct to say that without a repair argument, it's only going to provide information?What benefit would there be from running this with no_infomsgs on a nightly basis? And if even if I left off that argument, what benefit would there be to running it as nightly job? Does it write the checkdb results to the log? Thu, 16 Oct 2008 11:00:05 GMTMarcia QRE: DBCC CHECKDB - how use it in a maintenance plan ?http://www.sqlservercentral.com/Forums/Topic469988-146-1.aspxI certainly reccommend running CHECKDB, but I think it should be run at the most convenient time of the day which may not be when you do a backup.At my old place the optimum time for backup was before the overnight batch work started, which put the backups under time pressure to complete. After the bulk of the batch work had finished at about 4AM, we had another window where we ran CHECKDB.I understand the issues of checking if your DB is good before doing a backup, but if we did find corruption there was always a good backup we could go back to and transaction log backups to bring us up to the point of failure if not the curent point in time. On good disk systems it is very unlikely you will get a corrupted database, so you need to balance the risks of doing or not doing a CHECKDB before a backup.Back in the 1990s I worked on mainframe DB2. There were good vendor backup utilities that included the equivalent of a CHECKDB in the backup processing. Because the I-O needed to check if the database was OK was combined with the I-O needed to run the backup, doing the DB check in-line with the bacukp only added about 5% to backup completion time. It would be great if one of the backup vendors for SQL Server could do this type of thing, it would be a major selling point!Tue, 18 Mar 2008 03:50:41 GMTEdVassieRE: DBCC CHECKDB - how use it in a maintenance plan ?http://www.sqlservercentral.com/Forums/Topic469988-146-1.aspxI would always run this regularly. If you can run it before every backup, do that and page out errors. As mentioned, if there are problems, then your backup might be worthless.At least run it weeklySun, 16 Mar 2008 21:35:09 GMTSteve Jones - SSC EditorRE: DBCC CHECKDB - how use it in a maintenance plan ?http://www.sqlservercentral.com/Forums/Topic469988-146-1.aspxSure it's necessary. CheckDB checks to see if there's any corruption of any form in the database. Invalid checksums, bad page linkage, out of range column values, etcI would suggest you run it before the backup, and maybe not backup the DB is there's corruption. A backup of a corrupt database is not very usefulSun, 16 Mar 2008 14:01:17 GMTGilaMonsterDBCC CHECKDB - how use it in a maintenance plan ?http://www.sqlservercentral.com/Forums/Topic469988-146-1.aspxhello,do you think it necessary to use DBCC CHECKDB ? how use it in sql server agent ? before backup ?thanks to answer.Yann.Sun, 16 Mar 2008 07:21:08 GMTyann.larvor
{ "url": "http://www.sqlservercentral.com/Forums/RssFeed146-469988-0-2.aspx", "source_domain": "www.sqlservercentral.com", "snapshot_id": "crawl=CC-MAIN-2015-14", "warc_metadata": { "Content-Length": "33626", "Content-Type": "application/http; msgtype=response", "WARC-Block-Digest": "sha1:GUF3TYWSL2D6X3CFIBEANPCBIS55L4HH", "WARC-Concurrent-To": "<urn:uuid:2f6e9c22-8568-43ea-bf6d-592a196976c2>", "WARC-Date": "2015-04-01T10:59:26Z", "WARC-IP-Address": "162.13.37.189", "WARC-Identified-Payload-Type": null, "WARC-Payload-Digest": "sha1:HKK3XU6TRNQA2EQP46FWTXBI5I2VZZXB", "WARC-Record-ID": "<urn:uuid:2fa6275b-7b50-4017-b180-5b7361af4589>", "WARC-Target-URI": "http://www.sqlservercentral.com/Forums/RssFeed146-469988-0-2.aspx", "WARC-Truncated": null, "WARC-Type": "response", "WARC-Warcinfo-ID": "<urn:uuid:4557e5eb-9c1a-40c4-a57e-cea9d3403f23>" }, "warc_info": "robots: classic\r\nhostname: ip-10-168-14-71.ec2.internal\r\nsoftware: Nutch 1.6 (CC)/CC WarcExport 1.0\r\nisPartOf: CC-MAIN-2015-14\r\noperator: CommonCrawl Admin\r\ndescription: Wide crawl of the web with URLs provided by Blekko for March 2015\r\npublisher: CommonCrawl\r\nformat: WARC File Format 1.0\r\nconformsTo: http://bibnum.bnf.fr/WARC/WARC_ISO_28500_version1_latestdraft.pdf" }
{ "line_start_idx": [ 0 ], "line_end_idx": [ 29152 ] }
{ "red_pajama_v2": { "ccnet_original_length": 29152, "ccnet_original_nlines": 0, "rps_doc_curly_bracket": 0, "rps_doc_ldnoobw_words": 0, "rps_doc_lorem_ipsum": 0, "rps_doc_stop_word_fraction": 0.35361891984939575, "rps_doc_ut1_blacklist": 0, "rps_doc_frac_all_caps_words": 0.04431315138936043, "rps_doc_frac_lines_end_with_ellipsis": 0, "rps_doc_frac_no_alph_words": 0.27075332403182983, "rps_doc_frac_unique_words": 0.21139498054981232, "rps_doc_mean_word_length": 5.17567253112793, "rps_doc_num_sentences": 450, "rps_doc_symbol_to_word_ratio": 0.00132938998285681, "rps_doc_unigram_entropy": 5.724906921386719, "rps_doc_word_count": 4423, "rps_doc_frac_chars_dupe_10grams": 0.4664074778556824, "rps_doc_frac_chars_dupe_5grams": 0.4894286096096039, "rps_doc_frac_chars_dupe_6grams": 0.4879433810710907, "rps_doc_frac_chars_dupe_7grams": 0.4850166141986847, "rps_doc_frac_chars_dupe_8grams": 0.477546751499176, "rps_doc_frac_chars_dupe_9grams": 0.47287261486053467, "rps_doc_frac_chars_top_2gram": 0.021142760291695595, "rps_doc_frac_chars_top_3gram": 0.02044381946325302, "rps_doc_frac_chars_top_4gram": 0.023589029908180237, "rps_doc_books_importance": -3229.552978515625, "rps_doc_books_importance_length_correction": -3229.552978515625, "rps_doc_openwebtext_importance": -1787.08935546875, "rps_doc_openwebtext_importance_length_correction": -1787.08935546875, "rps_doc_wikipedia_importance": -1049.392822265625, "rps_doc_wikipedia_importance_length_correction": -1049.392822265625 }, "fasttext": { "dclm": 0.0827842429280281, "english": 0.8833855390548706, "fineweb_edu_approx": 1.7180343866348267, "eai_general_math": 0.3987422585487366, "eai_open_web_math": 0.1922532320022583, "eai_web_code": 0.3425353765487671 } }
{ "free_decimal_correspondence": { "primary": { "code": "005.746", "labels": { "level_1": "General works, books and libraries, information sciences", "level_2": "", "level_3": "Computer programming" } }, "secondary": { "code": "005.456", "labels": { "level_1": "General works, books and libraries, information sciences", "level_2": "", "level_3": "Computer programming" } } }, "bloom_cognitive_process": { "primary": { "code": "3", "label": "Apply" }, "secondary": { "code": "2", "label": "Understand" } }, "bloom_knowledge_domain": { "primary": { "code": "3", "label": "Procedural" }, "secondary": { "code": "2", "label": "Conceptual" } }, "document_type_v1": { "primary": { "code": "5", "label": "Social/Forum" }, "secondary": { "code": "3", "label": "Reference/Encyclopedic/Educational" } }, "extraction_artifacts": { "primary": { "code": "3", "label": "Irrelevant Content" }, "secondary": { "code": "0", "label": "No Artifacts" } }, "missing_content": { "primary": { "code": "0", "label": "No missing content" }, "secondary": { "code": "0", "label": "No missing content" } }, "document_type_v2": { "primary": { "code": "18", "label": "Q&A Forum" }, "secondary": { "code": "8", "label": "Documentation" } }, "reasoning_depth": { "primary": { "code": "3", "label": "Intermediate Reasoning" }, "secondary": { "code": "2", "label": "Basic Reasoning" } }, "technical_correctness": { "primary": { "code": "4", "label": "Highly Correct" }, "secondary": { "code": "3", "label": "Mostly Correct" } }, "education_level": { "primary": { "code": "3", "label": "Undergraduate Level" }, "secondary": { "code": "2", "label": "High School Level" } } }
672f1e42c33a7f9846924a2431ea77df
8,684,537,788,195,565,000
Backing up Spotipo Create a backup of database Spotipo uses MySQL as the DB server to store all its data in a database named spotipo. mysqldump -uroot -p spotipo > spotipo.sql Create a backup of files Backup following directories as well /usr/share/nginx/spotipo/instance /usr/share/nginx/spotipo/unifispot/static/uploads /usr/share/nginx/spotipo/unifispot/templates/guest You can also use the below script which will let you make a full backup in linux. #!/bin/bash CONFFILE=/usr/share/nginx/spotipo/instance/config.py USERNAME=$(grep "SQLALCHEMY" $CONFFILE | sed -r 's/SQLALCHEMY_DATABASE_URI="mysql:\/\/(.+):(.+)@.+\/(.+)"/\1/' ) PASSWORD=$(grep "SQLALCHEMY" $CONFFILE | sed -r 's/SQLALCHEMY_DATABASE_URI="mysql:\/\/(.+):(.+)@.+\/(.+)"/\2/' ) DBNAME=$(grep "SQLALCHEMY" $CONFFILE | sed -r 's/SQLALCHEMY_DATABASE_URI="mysql:\/\/(.+):(.+)@.+\/(.+)"/\3/' ) now=$(date +"%m%d%Y") mysqldump -u$USERNAME -p$PASSWORD $DBNAME > spotipo_$now.sql tar -czvf uploads_$now.tar.gz /usr/share/nginx/spotipo/unifispot/static/uploads/ tar -czvf instance_$now.tar.gz /usr/share/nginx/spotipo/instance/ tar -czvf guesttemplates_$now.tar.gz /usr/share/nginx/spotipo/unifispot/templates/guest Restoring from a backup First step is to intstall a fresh copy of Spotipo on the new server. Make sure to install the same version of spotipo from which backup was created Copy the contents of uploads folder into /usr/share/nginx/spotipo/unifispot/static/uploads Copy the contents of guest templates into /usr/share/nginx/spotipo/unifispot/templates/guest Now restore the database using below command mysql -u root -p SPOTIPODBNAME < spotipo.sql After restoring the database, you should reinitialize migrations. Use below commands for the same. mysql -u root -p -e "DROP TABLE spotipo.alembic_version;" cd /usr/share/nginx/spotipo rm -rf migrations source .env/bin/activate python manage.py db init python manage.py db migrate python manage.py db upgrade How did we do?
{ "url": "https://docs.spotipo.com/article/l2iincnx9x-backing-up-spotipo", "source_domain": "docs.spotipo.com", "snapshot_id": "crawl=CC-MAIN-2019-13", "warc_metadata": { "Content-Length": "25140", "Content-Type": "application/http; msgtype=response", "WARC-Block-Digest": "sha1:J3SFZDCBTVRZ5QXF5FVKK7N44DMOE5AN", "WARC-Concurrent-To": "<urn:uuid:fdf0c3b7-7383-4c05-86b5-e5ea724c1bf9>", "WARC-Date": "2019-03-23T23:06:35Z", "WARC-IP-Address": "104.27.138.107", "WARC-Identified-Payload-Type": "text/html", "WARC-Payload-Digest": "sha1:M5T2EWZI74JZDBFNCP7W7BYH4J24CRXA", "WARC-Record-ID": "<urn:uuid:f7b843d0-4fda-4e33-bcd7-7c596491ba65>", "WARC-Target-URI": "https://docs.spotipo.com/article/l2iincnx9x-backing-up-spotipo", "WARC-Truncated": null, "WARC-Type": "response", "WARC-Warcinfo-ID": "<urn:uuid:c24988c8-ca97-468a-8463-6493c4ad74b9>" }, "warc_info": "isPartOf: CC-MAIN-2019-13\r\npublisher: Common Crawl\r\ndescription: Wide crawl of the web for March 2019\r\noperator: Common Crawl Admin ([email protected])\r\nhostname: ip-10-69-221-252.ec2.internal\r\nsoftware: Apache Nutch 1.15 (modified, https://github.com/commoncrawl/nutch/)\r\nrobots: checked via crawler-commons 0.11-SNAPSHOT (https://github.com/crawler-commons/crawler-commons)\r\nformat: WARC File Format 1.1\r\nconformsTo: http://iipc.github.io/warc-specifications/specifications/warc-format/warc-1.1/" }
{ "line_start_idx": [ 0, 19, 20, 48, 49, 136, 137, 179, 204, 205, 242, 243, 277, 327, 378, 379, 461, 462, 474, 527, 640, 753, 864, 886, 887, 948, 949, 950, 1031, 1097, 1185, 1209, 1210, 1358, 1359, 1450, 1451, 1544, 1545, 1590, 1591, 1636, 1637, 1736, 1737, 1796, 1824, 1842, 1867, 1892, 1920, 1948, 1949 ], "line_end_idx": [ 19, 20, 48, 49, 136, 137, 179, 204, 205, 242, 243, 277, 327, 378, 379, 461, 462, 474, 527, 640, 753, 864, 886, 887, 948, 949, 950, 1031, 1097, 1185, 1209, 1210, 1358, 1359, 1450, 1451, 1544, 1545, 1590, 1591, 1636, 1637, 1736, 1737, 1796, 1824, 1842, 1867, 1892, 1920, 1948, 1949, 1963 ] }
{ "red_pajama_v2": { "ccnet_original_length": 1963, "ccnet_original_nlines": 52, "rps_doc_curly_bracket": 0, "rps_doc_ldnoobw_words": 0, "rps_doc_lorem_ipsum": 0, "rps_doc_stop_word_fraction": 0.18584071099758148, "rps_doc_ut1_blacklist": 0, "rps_doc_frac_all_caps_words": 0.04646018147468567, "rps_doc_frac_lines_end_with_ellipsis": 0, "rps_doc_frac_no_alph_words": 0.35176992416381836, "rps_doc_frac_unique_words": 0.5902438759803772, "rps_doc_mean_word_length": 7.160975456237793, "rps_doc_num_sentences": 24, "rps_doc_symbol_to_word_ratio": 0.0022123900707811117, "rps_doc_unigram_entropy": 4.583720684051514, "rps_doc_word_count": 205, "rps_doc_frac_chars_dupe_10grams": 0, "rps_doc_frac_chars_dupe_5grams": 0, "rps_doc_frac_chars_dupe_6grams": 0, "rps_doc_frac_chars_dupe_7grams": 0, "rps_doc_frac_chars_dupe_8grams": 0, "rps_doc_frac_chars_dupe_9grams": 0, "rps_doc_frac_chars_top_2gram": 0.014305179938673973, "rps_doc_frac_chars_top_3gram": 0.04291553050279617, "rps_doc_frac_chars_top_4gram": 0.04495913162827492, "rps_doc_books_importance": -184.2155303955078, "rps_doc_books_importance_length_correction": -184.2155303955078, "rps_doc_openwebtext_importance": -98.22637176513672, "rps_doc_openwebtext_importance_length_correction": -98.22637176513672, "rps_doc_wikipedia_importance": -59.3609619140625, "rps_doc_wikipedia_importance_length_correction": -59.3609619140625 }, "fasttext": { "dclm": 0.10999107360839844, "english": 0.5640659928321838, "fineweb_edu_approx": 2.374657154083252, "eai_general_math": 0.3819630742073059, "eai_open_web_math": 0.07529156655073166, "eai_web_code": 0.7974130511283875 } }
{ "free_decimal_correspondence": { "primary": { "code": "005.74", "labels": { "level_1": "General works, books and libraries, information sciences", "level_2": "", "level_3": "Computer programming" } }, "secondary": { "code": "005.1", "labels": { "level_1": "General works, books and libraries, information sciences", "level_2": "", "level_3": "Computer programming" } } }, "bloom_cognitive_process": { "primary": { "code": "3", "label": "Apply" }, "secondary": { "code": "2", "label": "Understand" } }, "bloom_knowledge_domain": { "primary": { "code": "3", "label": "Procedural" }, "secondary": { "code": "2", "label": "Conceptual" } }, "document_type_v1": { "primary": { "code": "3", "label": "Reference/Encyclopedic/Educational" }, "secondary": { "code": "4", "label": "Code/Software" } }, "extraction_artifacts": { "primary": { "code": "0", "label": "No Artifacts" }, "secondary": { "code": "3", "label": "Irrelevant Content" } }, "missing_content": { "primary": { "code": "0", "label": "No missing content" }, "secondary": { "code": "-1", "label": "Abstain" } }, "document_type_v2": { "primary": { "code": "23", "label": "Tutorial" }, "secondary": { "code": "8", "label": "Documentation" } }, "reasoning_depth": { "primary": { "code": "2", "label": "Basic Reasoning" }, "secondary": { "code": "3", "label": "Intermediate Reasoning" } }, "technical_correctness": { "primary": { "code": "4", "label": "Highly Correct" }, "secondary": { "code": "3", "label": "Mostly Correct" } }, "education_level": { "primary": { "code": "3", "label": "Undergraduate Level" }, "secondary": { "code": "4", "label": "Graduate/Expert Level" } } }
672f1e42c33a7f9846924a2431ea77df
-5,187,839,800,782,556,000
Cargo Features [dependencies] miden-client = { version = "0.4.1", default-features = false, features = ["std", "async", "concurrent", "idxdb", "sqlite", "tonic", "web-tonic", "testing", "thiserror", "tokio"] } default = std The std feature is set by default whenever miden-client is added without default-features = false somewhere in the dependency tree. std default concurrent? sqlite? tonic? Enables std of miden-objects async idxdb? Enables async of miden-tx concurrent = std Enables concurrent of miden-lib, miden-objects, and miden-tx idxdb = async Enables base64 ^0.13, serde-wasm-bindgen, wasm-bindgen, and wasm-bindgen-futures Affects store::web_store sqlite = std Enables lazy_static, rusqlite ^0.30, and rusqlite_migration Affects store::sqlite_store tonic = std Enables hex and prost ^0.12, transport of tonic ^0.11 web-tonic Enables getrandom, hex, prost ^0.12, tonic ^0.11, and tonic-web-wasm-client testing Enables testing of miden-lib and miden-objects miden-lib: the testing feature is required to enable the account creation pow patch Affects miden-client::testing Features from optional dependencies In crates that don't use the dep: syntax, optional dependencies automatically become Cargo features. These features may have been created by mistake, and this functionality may be removed in the future. getrandom web-tonic? thiserror implicit feature Enables thiserror thiserror: derive(Error) tokio implicit feature Enables tokio tokio: An event-driven, non-blocking I/O platform for writing asynchronous I/O backed applications
{ "url": "https://lib.rs/crates/miden-client/features", "source_domain": "lib.rs", "snapshot_id": "CC-MAIN-2024-30", "warc_metadata": { "Content-Length": "13978", "Content-Type": "application/http; msgtype=response", "WARC-Block-Digest": "sha1:NMWAYNSHJHB35EUU2OSJQSM6VEXZIFLB", "WARC-Concurrent-To": "<urn:uuid:1b3ce613-a75d-433c-992e-50c53dc15711>", "WARC-Date": "2024-07-25T18:35:06Z", "WARC-IP-Address": "172.67.139.52", "WARC-Identified-Payload-Type": "text/html", "WARC-Payload-Digest": "sha1:JVEYN4AIKGOVGVM2MEZ5OLZ4BEZQHM2V", "WARC-Record-ID": "<urn:uuid:c9fe8e3e-934e-4d83-b545-7614a94eae63>", "WARC-Target-URI": "https://lib.rs/crates/miden-client/features", "WARC-Truncated": null, "WARC-Type": "response", "WARC-Warcinfo-ID": "<urn:uuid:b77af105-1f33-4ef7-8ad2-6326edb3f245>" }, "warc_info": "isPartOf: CC-MAIN-2024-30\r\npublisher: Common Crawl\r\ndescription: Wide crawl of the web for July 2024\r\noperator: Common Crawl Admin ([email protected])\r\nhostname: ip-10-67-67-128\r\nsoftware: Apache Nutch 1.20 (modified, https://github.com/commoncrawl/nutch/)\r\nrobots: checked via crawler-commons 1.5-SNAPSHOT (https://github.com/crawler-commons/crawler-commons)\r\nformat: WARC File Format 1.1\r\nconformsTo: https://iipc.github.io/warc-specifications/specifications/warc-format/warc-1.1/" }
{ "line_start_idx": [ 0, 15, 16, 31, 211, 225, 226, 358, 359, 398, 399, 428, 429, 442, 443, 469, 470, 487, 488, 549, 550, 564, 565, 646, 647, 672, 673, 686, 687, 747, 748, 776, 777, 789, 790, 844, 845, 855, 856, 932, 933, 941, 942, 989, 990, 1001, 1002, 1075, 1076, 1106, 1107, 1143, 1144, 1347, 1348, 1369, 1396, 1397, 1415, 1416, 1427, 1428, 1442, 1443, 1466, 1467, 1481, 1482, 1489, 1490 ], "line_end_idx": [ 15, 16, 31, 211, 225, 226, 358, 359, 398, 399, 428, 429, 442, 443, 469, 470, 487, 488, 549, 550, 564, 565, 646, 647, 672, 673, 686, 687, 747, 748, 776, 777, 789, 790, 844, 845, 855, 856, 932, 933, 941, 942, 989, 990, 1001, 1002, 1075, 1076, 1106, 1107, 1143, 1144, 1347, 1348, 1369, 1396, 1397, 1415, 1416, 1427, 1428, 1442, 1443, 1466, 1467, 1481, 1482, 1489, 1490, 1581 ] }
{ "red_pajama_v2": { "ccnet_original_length": 1581, "ccnet_original_nlines": 69, "rps_doc_curly_bracket": 0.0012650199932977557, "rps_doc_ldnoobw_words": 0, "rps_doc_lorem_ipsum": 0, "rps_doc_stop_word_fraction": 0.11988303810358047, "rps_doc_ut1_blacklist": 0, "rps_doc_frac_all_caps_words": 0.01169591024518013, "rps_doc_frac_lines_end_with_ellipsis": 0, "rps_doc_frac_no_alph_words": 0.37134504318237305, "rps_doc_frac_unique_words": 0.5132275223731995, "rps_doc_mean_word_length": 6.439153671264648, "rps_doc_num_sentences": 17, "rps_doc_symbol_to_word_ratio": 0, "rps_doc_unigram_entropy": 4.314769744873047, "rps_doc_word_count": 189, "rps_doc_frac_chars_dupe_10grams": 0, "rps_doc_frac_chars_dupe_5grams": 0, "rps_doc_frac_chars_dupe_6grams": 0, "rps_doc_frac_chars_dupe_7grams": 0, "rps_doc_frac_chars_dupe_8grams": 0, "rps_doc_frac_chars_dupe_9grams": 0, "rps_doc_frac_chars_top_2gram": 0.024650780484080315, "rps_doc_frac_chars_top_3gram": 0.036154478788375854, "rps_doc_frac_chars_top_4gram": 0, "rps_doc_books_importance": -132.3787841796875, "rps_doc_books_importance_length_correction": -119.150390625, "rps_doc_openwebtext_importance": -40.27219772338867, "rps_doc_openwebtext_importance_length_correction": -40.27219772338867, "rps_doc_wikipedia_importance": -45.84786605834961, "rps_doc_wikipedia_importance_length_correction": -32.031124114990234 }, "fasttext": { "dclm": 0.05340975895524025, "english": 0.7873414158821106, "fineweb_edu_approx": 2.422004461288452, "eai_general_math": 0.20388758182525635, "eai_open_web_math": 0.12454109638929367, "eai_web_code": 0.7070233821868896 } }
{ "free_decimal_correspondence": { "primary": { "code": "005.1", "labels": { "level_1": "General works, books and libraries, information sciences", "level_2": "", "level_3": "Computer programming" } }, "secondary": { "code": "004.02", "labels": { "level_1": "General works, books and libraries, information sciences", "level_2": "", "level_3": "Computers and Computer science" } } }, "bloom_cognitive_process": { "primary": { "code": "2", "label": "Understand" }, "secondary": { "code": "3", "label": "Apply" } }, "bloom_knowledge_domain": { "primary": { "code": "3", "label": "Procedural" }, "secondary": { "code": "2", "label": "Conceptual" } }, "document_type_v1": { "primary": { "code": "4", "label": "Code/Software" }, "secondary": { "code": "3", "label": "Reference/Encyclopedic/Educational" } }, "extraction_artifacts": { "primary": { "code": "0", "label": "No Artifacts" }, "secondary": { "code": "3", "label": "Irrelevant Content" } }, "missing_content": { "primary": { "code": "0", "label": "No missing content" }, "secondary": { "code": "-1", "label": "Abstain" } }, "document_type_v2": { "primary": { "code": "8", "label": "Documentation" }, "secondary": { "code": "10", "label": "Knowledge Article" } }, "reasoning_depth": { "primary": { "code": "2", "label": "Basic Reasoning" }, "secondary": { "code": "1", "label": "No Reasoning" } }, "technical_correctness": { "primary": { "code": "4", "label": "Highly Correct" }, "secondary": { "code": "3", "label": "Mostly Correct" } }, "education_level": { "primary": { "code": "4", "label": "Graduate/Expert Level" }, "secondary": { "code": "3", "label": "Undergraduate Level" } } }
672f1e42c33a7f9846924a2431ea77df
8,472,447,437,100,044,000
Topic: Update frequency of mobile application Conference4me (C4me)  (Read 27191 times) We are changing some dates in the schedule of our event and want to make the updated information available quickly. How long does it take until the updates are visible in C4me? By default, the agenda data of ConfTool Pro will be synchronized with the C4me servers twice a day (in the early morning and around lunch time [CET]). The process itself takes about 10 minutes. By request of the organizer, the providers of C4me can increase the update frequency or run an extra update at a certain time. The application itself checks for updates in the database of the Conference4me servers every 15 minutes.
{ "url": "https://www.conftool.net/ctforum/index.php/topic,325.0/prev_next,next.html", "source_domain": "www.conftool.net", "snapshot_id": "crawl=CC-MAIN-2018-43", "warc_metadata": { "Content-Length": "12568", "Content-Type": "application/http; msgtype=response", "WARC-Block-Digest": "sha1:RKQXCCZYRNXNRCPKGAA2XICBZIGAMS6C", "WARC-Concurrent-To": "<urn:uuid:1ba2097d-ccfd-4436-bcde-e65bf3f6f075>", "WARC-Date": "2018-10-23T14:23:52Z", "WARC-IP-Address": "88.99.71.145", "WARC-Identified-Payload-Type": "application/xhtml+xml", "WARC-Payload-Digest": "sha1:CH3PTXPQEDM2DSBZZVSOYBGR4OX3XZOX", "WARC-Record-ID": "<urn:uuid:d360fde5-7726-454d-a657-a7ddb70af663>", "WARC-Target-URI": "https://www.conftool.net/ctforum/index.php/topic,325.0/prev_next,next.html", "WARC-Truncated": null, "WARC-Type": "response", "WARC-Warcinfo-ID": "<urn:uuid:9de1bed0-bb58-42ca-bc26-260e4428cdad>" }, "warc_info": "isPartOf: CC-MAIN-2018-43\r\npublisher: Common Crawl\r\ndescription: Wide crawl of the web for October 2018\r\noperator: Common Crawl Admin ([email protected])\r\nhostname: ip-10-69-127-157.ec2.internal\r\nsoftware: Apache Nutch 1.15 (modified, https://github.com/commoncrawl/nutch/)\r\nrobots: checked via crawler-commons 0.11-SNAPSHOT (https://github.com/crawler-commons/crawler-commons)\r\nformat: WARC File Format 1.1\r\nconformsTo: http://iipc.github.io/warc-specifications/specifications/warc-format/warc-1.1/" }
{ "line_start_idx": [ 0, 87, 88, 265, 266, 417, 460, 461, 588, 589 ], "line_end_idx": [ 87, 88, 265, 266, 417, 460, 461, 588, 589, 693 ] }
{ "red_pajama_v2": { "ccnet_original_length": 693, "ccnet_original_nlines": 9, "rps_doc_curly_bracket": 0, "rps_doc_ldnoobw_words": 0, "rps_doc_lorem_ipsum": 0, "rps_doc_stop_word_fraction": 0.3893129825592041, "rps_doc_ut1_blacklist": 0, "rps_doc_frac_all_caps_words": 0.007633590139448643, "rps_doc_frac_lines_end_with_ellipsis": 0, "rps_doc_frac_no_alph_words": 0.13740457594394684, "rps_doc_frac_unique_words": 0.681034505367279, "rps_doc_mean_word_length": 4.793103218078613, "rps_doc_num_sentences": 6, "rps_doc_symbol_to_word_ratio": 0, "rps_doc_unigram_entropy": 4.1060333251953125, "rps_doc_word_count": 116, "rps_doc_frac_chars_dupe_10grams": 0, "rps_doc_frac_chars_dupe_5grams": 0, "rps_doc_frac_chars_dupe_6grams": 0, "rps_doc_frac_chars_dupe_7grams": 0, "rps_doc_frac_chars_dupe_8grams": 0, "rps_doc_frac_chars_dupe_9grams": 0, "rps_doc_frac_chars_top_2gram": 0.026978420093655586, "rps_doc_frac_chars_top_3gram": 0, "rps_doc_frac_chars_top_4gram": 0, "rps_doc_books_importance": -61.981143951416016, "rps_doc_books_importance_length_correction": -64.88990783691406, "rps_doc_openwebtext_importance": -24.90535545349121, "rps_doc_openwebtext_importance_length_correction": -27.81412124633789, "rps_doc_wikipedia_importance": -23.334470748901367, "rps_doc_wikipedia_importance_length_correction": -26.243236541748047 }, "fasttext": { "dclm": 0.11878973245620728, "english": 0.8737983107566833, "fineweb_edu_approx": 1.2255445718765259, "eai_general_math": 0.07835298776626587, "eai_open_web_math": 0.13506299257278442, "eai_web_code": 0.04457474127411842 } }
{ "free_decimal_correspondence": { "primary": { "code": "004.6", "labels": { "level_1": "General works, books and libraries, information sciences", "level_2": "", "level_3": "Computers and Computer science" } }, "secondary": { "code": "658.403", "labels": { "level_1": "Industrial arts, Technology, and Engineering", "level_2": "Business", "level_3": "Management" } } }, "bloom_cognitive_process": { "primary": { "code": "2", "label": "Understand" }, "secondary": { "code": "3", "label": "Apply" } }, "bloom_knowledge_domain": { "primary": { "code": "1", "label": "Factual" }, "secondary": { "code": "3", "label": "Procedural" } }, "document_type_v1": { "primary": { "code": "5", "label": "Social/Forum" }, "secondary": { "code": "-1", "label": "Abstain" } }, "extraction_artifacts": { "primary": { "code": "0", "label": "No Artifacts" }, "secondary": { "code": "3", "label": "Irrelevant Content" } }, "missing_content": { "primary": { "code": "0", "label": "No missing content" }, "secondary": { "code": "-1", "label": "Abstain" } }, "document_type_v2": { "primary": { "code": "18", "label": "Q&A Forum" }, "secondary": { "code": "21", "label": "Customer Support" } }, "reasoning_depth": { "primary": { "code": "2", "label": "Basic Reasoning" }, "secondary": { "code": "1", "label": "No Reasoning" } }, "technical_correctness": { "primary": { "code": "4", "label": "Highly Correct" }, "secondary": { "code": "3", "label": "Mostly Correct" } }, "education_level": { "primary": { "code": "1", "label": "General Audience" }, "secondary": { "code": "2", "label": "High School Level" } } }
672f1e42c33a7f9846924a2431ea77df
-7,202,356,465,477,986,000
nikola tesla 3 6 9 The Secrets Behind the Numbers 3, 6, and 9 Nikola Tesla did countless mysterious experiments, but he was a whole other mystery on his own. Almost all genius minds have a certain obsession. Nikola Tesla had a pretty big one! nikola tesla 3 6 9 He was walking around a block repeatedly for three times before entering a building, he would clean his plates with 18 napkins, he lived in hotel rooms only with a number devisable by 3. He would make calculations about things in his immediate environment to make sure the result is devisable by 3 and base his choices upon the results. He would do everything in sets of 3. Some say he had OCD, some say he was very superstitious. However, the truth is a lot deeper. “If you knew the magnificence of the three, six and nine, you would have a key to the universe.”Nikola Tesla His obsession was not simply with numbers, but especially with these numbers: 3, 6, 9! He did have an extreme case of OCD and he was superstitious, however, he chose those numbers for a reason. Tesla claimed that these numbers were extremely important. Nobody listened. He even calculated nodal points around the planet linked to the numbers three, six and nine! But why these numbers? What did Nikola Tesla tried to make the world understand? NOTE: Things will get a lot stranger below! First, we must understand that we didn’t create math, we discovered it. It’s The Universal language and law. No matter where you are in The Universe 1 + 2 will always equal to 3! Everything in The Universe obeys this law! the golden ratio There are patterns that naturally occur in The Universe, patterns we’ve discovered in life, galaxies, star formations, evolution, and almost all natural systems. Some of these patterns are The Golden Ratio and Sacred Geometry. One really important system that nature seems to obey is “The Powers of 2 Binary System” in which the pattern start from one and continues by doubling the numbers. Cells and embryos develop following this sacred pattern: 1, 2, 4, 8, 16, 32, 64, 128, 256… binary system Some call these patterns The Blueprint of God. Math, by this analogy, would be God’s Thumbprint. (leaving all religion aside!) In vortex math (the science of torus anatomy) there is a pattern that repeats itself: 1, 2, 4, 8, 7, and 5, and so on 1, 2, 4, 8, 7, 5, 1, 2, 4, 8, 7, 5, 1, 2, 4… torus As you can see 3, 6, and 9 are not in this pattern. Scientist Marko Rodin believes these numbers represent a vector from the third to fourth dimension which he calls a “flux field.” This field is supposed to be a higher dimensional energy that influences the energy circuit of the other six points. Randy Powell, a student of Marko Rodin says that this is the secret key to free energy, something which we all know Tesla mastered. vortex coil Let me explain! Let’s start from 1, doubled it is 2; 2 doubled is 4; 4 doubled is 8; 8 doubled is 16 which means 1 + 6 and that equals to 7; 16 doubled is 32 resulting in 3 + 2 equals 5 (you can do 7 doubled if you want to which you would get 14 resulting in 5); 32 doubled is 64 (5 doubled is 10) resulting in total of 1; If we continue we will keep following the same pattern: 1, 2, 4, 8, 7, 5, 1, 2… If we start from 1 in reverse we will still get the same pattern only in reverse: Half of one is 0.5 (0+5) equaling 5. Half of 5 is 2.5 (2+5) equaling 7, and so on. As you can see there is no mention of 3, 6, and 9! It’s like they are beyond this pattern, free from it. vortex based mathematics vbm 5 However, there is something strange once you start doubling them. 3 doubled is 6; 6 doubled is 12 which would result in 3; in this pattern there is no mention of 9! It’s like 9 is beyond, completely free from both patterns. But if you start doubling 9 it will always result in 9: 18, 36, 72, 144, 288, 576… This is called The Symbol of Enlightenment! If we go to the Great Pyramid of Giza, not only are there the three larger pyramids at Giza, all side by side, mirroring the positions of the stars in Orion’s Belt, but we also see a group of three smaller pyramids immediately away from the three larger pyramids. the symbol of enlightenment We find lots of evidence that nature uses threefold and sixfold symmetry, including the hexagonal tile shape of the common honeycomb. These shapes are in nature, and the ancients emulated these shapes in the building of their sacred architecture. Is it possible that there is something special about the mysterious number three? is it possible that Tesla uncovered this profound secret and used this knowledge to push the boundaries of science and technology? The magnificence of 9! number 3 ying yang Let’s say there are 2 opposites, call them light and dark if you want to. They are like the North and the South poles of a magnet. One side is 1, 2, and 4; the other side is 8, 7 and 5; Just like electricity, everything in The Universe is a stream between these 2 polar sides, like a swinging pendulum: 1, 2, 4, 8, 7, 5, 1, 2… (and if you imagine the movement it’s something like the symbol for infinity). vortex math 600x600 However, these 2 sides are governed by 3 and 6; 3 governs 1, 2, and 4 while 6 governs 8, 7, and 5; and if you look the pattern closely it gets even more mindboggling: 1 and 2 equals 3; 2 and 4 equals 6; 4 and 8 equals 3; 8 and 7 equals 6; 7 and 5 equals 3; 5 and 1 equals 6; 1 and 2 equals 3… The same pattern on a higher scale is actually 3, 6, 3, 6, 3, 6… But even these two sides, 3 and 6 are governed by 9 which shows something spectacular. Looking closely at the pattern of 3 and 6 you realize that 3 and 6 equals 9, 6 and 3 equals 9, all the numbers together equal 9, both ways excluding and including 3 and 6! So 9 means unity of the both sides. 9 is The Universe itself! The vibration, the energy and the frequency! 3,6 and 9! “If you want to find the secrets of the universe, think in terms of energy, frequency and vibration.”Nikola Tesla There is a deeper philosophical truth in this! Just imagine what we can accomplish if we apply this sacred knowledge in everyday science… “The day science begins to study non-physical phenomena, it will make more progress in one decade than in all the previous centuries of its existence.”Nikola Tesla Reference: LifeCoachCode.com
{ "url": "https://humansbefree.com/2021/11/secrets-numbers-3-6-9.html", "source_domain": "humansbefree.com", "snapshot_id": "CC-MAIN-2023-06", "warc_metadata": { "Content-Length": "218336", "Content-Type": "application/http; msgtype=response", "WARC-Block-Digest": "sha1:334FX3ULSP7KN3CFGFGTBVAGWA5IEZWF", "WARC-Concurrent-To": "<urn:uuid:58a941cb-2ced-4b2a-a3cc-f06358388b8e>", "WARC-Date": "2023-02-04T19:14:51Z", "WARC-IP-Address": "172.67.206.228", "WARC-Identified-Payload-Type": "text/html", "WARC-Payload-Digest": "sha1:NW336LU43QA7M4K5V4RNTWUYCZVFX56T", "WARC-Record-ID": "<urn:uuid:aba18074-af78-45ba-9609-12d5f2b3ab84>", "WARC-Target-URI": "https://humansbefree.com/2021/11/secrets-numbers-3-6-9.html", "WARC-Truncated": null, "WARC-Type": "response", "WARC-Warcinfo-ID": "<urn:uuid:36d7607e-fd31-4df4-a2ee-048141cfd355>" }, "warc_info": "isPartOf: CC-MAIN-2023-06\r\npublisher: Common Crawl\r\ndescription: Wide crawl of the web for January/February 2023\r\noperator: Common Crawl Admin ([email protected])\r\nhostname: ip-10-67-67-225\r\nsoftware: Apache Nutch 1.19 (modified, https://github.com/commoncrawl/nutch/)\r\nrobots: checked via crawler-commons 1.4-SNAPSHOT (https://github.com/crawler-commons/crawler-commons)\r\nformat: WARC File Format 1.1\r\nconformsTo: https://iipc.github.io/warc-specifications/specifications/warc-format/warc-1.1/" }
{ "line_start_idx": [ 0, 19, 20, 63, 64, 245, 246, 265, 266, 453, 454, 641, 642, 699, 700, 736, 737, 846, 847, 934, 935, 1042, 1043, 1119, 1120, 1213, 1214, 1237, 1238, 1296, 1297, 1341, 1342, 1564, 1565, 1582, 1583, 1810, 1811, 2066, 2067, 2081, 2082, 2129, 2130, 2210, 2211, 2374, 2375, 2381, 2382, 2681, 2682, 2814, 2815, 2827, 2828, 2844, 2845, 3232, 3233, 3398, 3399, 3504, 3505, 3536, 3537, 3761, 3762, 3845, 3846, 3890, 3891, 4155, 4156, 4184, 4185, 4319, 4320, 4433, 4434, 4647, 4648, 4671, 4672, 4691, 4692, 4823, 4824, 5099, 5100, 5120, 5121, 5414, 5415, 5480, 5481, 5568, 5569, 5741, 5742, 5804, 5805, 5850, 5851, 5862, 5863, 5977, 5978, 6025, 6026, 6117, 6118, 6282, 6283 ], "line_end_idx": [ 19, 20, 63, 64, 245, 246, 265, 266, 453, 454, 641, 642, 699, 700, 736, 737, 846, 847, 934, 935, 1042, 1043, 1119, 1120, 1213, 1214, 1237, 1238, 1296, 1297, 1341, 1342, 1564, 1565, 1582, 1583, 1810, 1811, 2066, 2067, 2081, 2082, 2129, 2130, 2210, 2211, 2374, 2375, 2381, 2382, 2681, 2682, 2814, 2815, 2827, 2828, 2844, 2845, 3232, 3233, 3398, 3399, 3504, 3505, 3536, 3537, 3761, 3762, 3845, 3846, 3890, 3891, 4155, 4156, 4184, 4185, 4319, 4320, 4433, 4434, 4647, 4648, 4671, 4672, 4691, 4692, 4823, 4824, 5099, 5100, 5120, 5121, 5414, 5415, 5480, 5481, 5568, 5569, 5741, 5742, 5804, 5805, 5850, 5851, 5862, 5863, 5977, 5978, 6025, 6026, 6117, 6118, 6282, 6283, 6311 ] }
{ "red_pajama_v2": { "ccnet_original_length": 6311, "ccnet_original_nlines": 114, "rps_doc_curly_bracket": 0, "rps_doc_ldnoobw_words": 0, "rps_doc_lorem_ipsum": 0, "rps_doc_stop_word_fraction": 0.3741307258605957, "rps_doc_ut1_blacklist": 0, "rps_doc_frac_all_caps_words": 0.0020862300880253315, "rps_doc_frac_lines_end_with_ellipsis": 0.060869570821523666, "rps_doc_frac_no_alph_words": 0.30180805921554565, "rps_doc_frac_unique_words": 0.3395744562149048, "rps_doc_mean_word_length": 4.131915092468262, "rps_doc_num_sentences": 62, "rps_doc_symbol_to_word_ratio": 0.005563280079513788, "rps_doc_unigram_entropy": 5.259363174438477, "rps_doc_word_count": 1175, "rps_doc_frac_chars_dupe_10grams": 0, "rps_doc_frac_chars_dupe_5grams": 0.027806390076875687, "rps_doc_frac_chars_dupe_6grams": 0.006591139826923609, "rps_doc_frac_chars_dupe_7grams": 0.006591139826923609, "rps_doc_frac_chars_dupe_8grams": 0.006591139826923609, "rps_doc_frac_chars_dupe_9grams": 0, "rps_doc_frac_chars_top_2gram": 0.004119460005313158, "rps_doc_frac_chars_top_3gram": 0.0037075199652463198, "rps_doc_frac_chars_top_4gram": 0.004943360108882189, "rps_doc_books_importance": -559.7183227539062, "rps_doc_books_importance_length_correction": -559.7183227539062, "rps_doc_openwebtext_importance": -379.99835205078125, "rps_doc_openwebtext_importance_length_correction": -379.99835205078125, "rps_doc_wikipedia_importance": -247.11190795898438, "rps_doc_wikipedia_importance_length_correction": -247.11190795898438 }, "fasttext": { "dclm": 0.799472987651825, "english": 0.9490891695022583, "fineweb_edu_approx": 2.3184735774993896, "eai_general_math": 0.9618894457817078, "eai_open_web_math": 0.47169119119644165, "eai_web_code": 0.48022860288619995 } }
{ "free_decimal_correspondence": { "primary": { "code": "510", "labels": { "level_1": "Science and Natural history", "level_2": "Mathematics", "level_3": "" } }, "secondary": { "code": "530", "labels": { "level_1": "Science and Natural history", "level_2": "Physics", "level_3": "" } } }, "bloom_cognitive_process": { "primary": { "code": "2", "label": "Understand" }, "secondary": { "code": "4", "label": "Analyze" } }, "bloom_knowledge_domain": { "primary": { "code": "2", "label": "Conceptual" }, "secondary": { "code": "3", "label": "Procedural" } }, "document_type_v1": { "primary": { "code": "3", "label": "Reference/Encyclopedic/Educational" }, "secondary": { "code": "-1", "label": "Abstain" } }, "extraction_artifacts": { "primary": { "code": "0", "label": "No Artifacts" }, "secondary": { "code": "-1", "label": "Abstain" } }, "missing_content": { "primary": { "code": "0", "label": "No missing content" }, "secondary": { "code": "-1", "label": "Abstain" } }, "document_type_v2": { "primary": { "code": "16", "label": "Personal Blog" }, "secondary": { "code": "10", "label": "Knowledge Article" } }, "reasoning_depth": { "primary": { "code": "2", "label": "Basic Reasoning" }, "secondary": { "code": "3", "label": "Intermediate Reasoning" } }, "technical_correctness": { "primary": { "code": "1", "label": "Technically Flawed" }, "secondary": { "code": "6", "label": "Not Applicable/Indeterminate" } }, "education_level": { "primary": { "code": "1", "label": "General Audience" }, "secondary": { "code": "2", "label": "High School Level" } } }
672f1e42c33a7f9846924a2431ea77df
-2,611,022,897,707,865,600
Mimikatz DCSync Usage, Exploitation, and Detection Note: I presented on this AD persistence method at DerbyCon (2015). A major feature added to Mimkatz in August 2015 is “DCSync” which effectively “impersonates” a Domain Controller and requests account password data from the targeted Domain Controller. DCSync was written by Benjamin Delpy and Vincent Le Toux. The exploit method prior to DCSync was to run Mimikatz or Invoke-Mimikatz on a Domain Controller to get the KRBTGT password hash to create Golden Tickets. With Mimikatz’s DCSync and the appropriate rights, the attacker can pull the password hash, as well as previous password hashes, from a Domain Controller over the network without requiring interactive logon or copying off the Active Directory database file (ntds.dit). Special rights are required to run DCSync. Any member of Administrators, Domain Admins, or Enterprise Admins as well as Domain Controller computer accounts are able to run DCSync to pull password data. Note that Read-Only Domain Controllers are not  allowed to pull password data for users by default. Mimikatz-DCSync-UserRights-DCR-Administrator-500-Dump2 The credentials section in the graphic above shows the current NTLM hashes as well as the password history. This information can be valuable to an attacker since it can provide password creation strategies for users (if cracked). Will’s post has great information on Red Team usage of Mimikatz DCSync: Mimikatz and DCSync and ExtraSids, Oh My   How DCSync works: 1. Discovers Domain Controller in the specified domain name. 2. Requests the Domain Controller replicate the user credentials via GetNCChanges (leveraging Directory Replication Service (DRS) Remote Protocol) I have previously done some packet captures for Domain Controller replication and identified the intra-DC communication flow regarding how Domain Controllers replicate. The Samba Wiki describes the DSGetNCChanges function: “The client DC sends a DSGetNCChanges request to the server when the first one wants to get AD objects updates from the second one. The response contains a set of updates that the client has to apply to its NC replica. It is possible that the set of updates is too large for only one response message. In those cases, multiple DSGetNCChanges requests and responses are done. This process is called replication cycle or simply cycle.” “When a DC receives a DSReplicaSync Request, then for each DC that it replicates from (stored in RepsFrom data structure) it performs a replication cycle where it behaves like a client and makes DSGetNCChanges requests to that DC. So it gets up-to-date AD objects from each of the DC’s which it replicates from.” From MSDN: The IDL_DRSGetNCChanges method replicates updates from an NC replica on the server.  ULONG IDL_DRSGetNCChanges(   [in, ref] DRS_HANDLE hDrs,   [in] DWORD dwInVersion,   [in, ref, switch_is(dwInVersion)]   DRS_MSG_GETCHGREQ* pmsgIn,   [out, ref] DWORD* pdwOutVersion,   [out, ref, switch_is(*pdwOutVersion)]   DRS_MSG_GETCHGREPLY* pmsgOut  ); hDrs: The RPC context handle returned by the IDL_DRSBind method. dwInVersion: Version of the request message. pmsgIn: A pointer to the request message. pdwOutVersion: A pointer to the version of the response message. pmsgOut: A pointer to the response message. Return Values: 0 if successful, otherwise a Windows error code. Exceptions Thrown: This method might throw the following exceptions beyond those thrown by the underlying RPC protocol (as specified in [MS-RPCE]): ERROR_INVALID_HANDLE, ERROR_DS_DRS_EXTENSIONS_CHANGED, ERROR_DS_DIFFERENT_REPL_EPOCHS, and  ERROR_INVALID_PARAMETER.   Delegating Rights to Pull Account data: It is possible to use a regular domain user account to run DCSync. The combination of the following three rights need to be delegated at the domain level in order for the user account to successfully retrieve the password data with DCSync: • Replicating Directory Changes (DS-Replication-Get-Changes) Extended right needed to replicate only those changes from a given NC that are also replicated to the Global Catalog (which excludes secret domain data). This constraint is only meaningful for Domain NCs. • Replicating Directory Changes All (DS-Replication-Get-Changes-All) Control access right that allows the replication of all data in a given replication NC, including secret domain data. • Replicating Directory Changes In Filtered Set (rare, only required in some environments) Mimikatz-DCSync-UserRights-DCR-DomainPermissionsConfig Note that members of the Administrators and Domain Controller groups have these rights by default. DCSync-ADDomain-DefaultRights Pulling Password Data Using DCSync Once the account is delegated the ability to replicate objects, the account can run Mimikatz DCSync: mimikatz “lsadump::dcsync /domain:rd.adsecurity.org /user:krbtgt”   Mimikatz-DCSync-UserRights-DCR-KRBTGT-Dump Targeting an admin account with DCSync can also provide the account’s password history (in hash format). Since there are LMHashes listed it may be possible to crack these and gain insight into the password strategy the admin uses. This may provide the attacker to guess the next password the admin uses if access is lost. mimikatz “lsadump::dcsync /domain:rd.adsecurity.org /user:Administrator” Mimikatz-DCSync-UserRights-DCR-Administrator-500-Dump2-02   Detecting DCSync usage While there may be event activity that could be used to identify DCSync usage, the best detection method is through network monitoring. Step 1: Identify all Domain Controller IP addresses and add to “Replication Allow List”. PowerShell Active Directory module cmdlet: Get-ADDomainController -filter * | select IPv4Address PowerShell: [System.DirectoryServices.ActiveDirectory.Domain]::GetCurrentDomain().DomainControllers | select IPAddress Nslookup (if DC runs DNS): nslookup Set type=all _ldap._tcp.dc._msdcs.DOMAIN.COM   Step 2: Configure IDS to trigger if DsGetNCChange request originates an IP not on the “Replication Allow List” (list of DC IPs). Mimikatz-DCSync-KRBTGT-PacketCapture-DSGetNCChanges Mimikatz-DCSync-KRBTGT-PacketCapture-DSGetNCChanges02 There are other tools that perform this same process so it’s better to focus on detecting the method instead of specific artifacts. Other tools that leverage GetNCChanges Note that Full Control rights at the domain provides these rights as well, so limit who has domain-level admin rights.   References:   (Visited 81,420 times, 13 visits today)
{ "url": "https://adsecurity.org/?p=1729", "source_domain": "adsecurity.org", "snapshot_id": "crawl=CC-MAIN-2020-40", "warc_metadata": { "Content-Length": "55910", "Content-Type": "application/http; msgtype=response", "WARC-Block-Digest": "sha1:2VQB5AF6WVKTOMCSBMC4XNUVCWBUZRSQ", "WARC-Concurrent-To": "<urn:uuid:4233ceb8-eaee-4d22-a457-a7a2c716b489>", "WARC-Date": "2020-09-19T09:48:35Z", "WARC-IP-Address": "74.208.236.132", "WARC-Identified-Payload-Type": "text/html", "WARC-Payload-Digest": "sha1:FVEH2ZVRCYASQGK25LMWHYMCEBHUWESS", "WARC-Record-ID": "<urn:uuid:b0a08c17-4d29-4684-9688-78fca1cf1d54>", "WARC-Target-URI": "https://adsecurity.org/?p=1729", "WARC-Truncated": null, "WARC-Type": "response", "WARC-Warcinfo-ID": "<urn:uuid:b307dd7d-38b4-4872-b5f8-6400c6a5cc56>" }, "warc_info": "isPartOf: CC-MAIN-2020-40\r\npublisher: Common Crawl\r\ndescription: Wide crawl of the web for September 2020\r\noperator: Common Crawl Admin ([email protected])\r\nhostname: ip-10-67-67-59.ec2.internal\r\nsoftware: Apache Nutch 1.17 (modified, https://github.com/commoncrawl/nutch/)\r\nrobots: checked via crawler-commons 1.2-SNAPSHOT (https://github.com/crawler-commons/crawler-commons)\r\nformat: WARC File Format 1.1\r\nconformsTo: http://iipc.github.io/warc-specifications/specifications/warc-format/warc-1.1/" }
{ "line_start_idx": [ 0, 51, 52, 120, 121, 364, 365, 789, 790, 1092, 1093, 1148, 1149, 1379, 1380, 1452, 1493, 1494, 1496, 1497, 1515, 1516, 1579, 1728, 1729, 1898, 1899, 1953, 1954, 2173, 2174, 2389, 2390, 2703, 2704, 2715, 2716, 2800, 2801, 2829, 2859, 2886, 2924, 2956, 2992, 3034, 3068, 3072, 3073, 3138, 3139, 3184, 3185, 3227, 3228, 3293, 3294, 3338, 3339, 3403, 3404, 3669, 3670, 3672, 3673, 3713, 3714, 3954, 3955, 4018, 4227, 4298, 4420, 4513, 4514, 4569, 4570, 4669, 4670, 4700, 4701, 4736, 4737, 4838, 4839, 4905, 4906, 4908, 4909, 4952, 5274, 5275, 5348, 5349, 5407, 5408, 5410, 5411, 5434, 5435, 5571, 5572, 5661, 5662, 5705, 5706, 5760, 5761, 5762, 5774, 5775, 5882, 5883, 5884, 5911, 5912, 5921, 5934, 5966, 5967, 5969, 5970, 6099, 6100, 6152, 6153, 6207, 6208, 6340, 6341, 6380, 6381, 6500, 6501, 6503, 6504, 6516, 6517, 6519, 6520 ], "line_end_idx": [ 51, 52, 120, 121, 364, 365, 789, 790, 1092, 1093, 1148, 1149, 1379, 1380, 1452, 1493, 1494, 1496, 1497, 1515, 1516, 1579, 1728, 1729, 1898, 1899, 1953, 1954, 2173, 2174, 2389, 2390, 2703, 2704, 2715, 2716, 2800, 2801, 2829, 2859, 2886, 2924, 2956, 2992, 3034, 3068, 3072, 3073, 3138, 3139, 3184, 3185, 3227, 3228, 3293, 3294, 3338, 3339, 3403, 3404, 3669, 3670, 3672, 3673, 3713, 3714, 3954, 3955, 4018, 4227, 4298, 4420, 4513, 4514, 4569, 4570, 4669, 4670, 4700, 4701, 4736, 4737, 4838, 4839, 4905, 4906, 4908, 4909, 4952, 5274, 5275, 5348, 5349, 5407, 5408, 5410, 5411, 5434, 5435, 5571, 5572, 5661, 5662, 5705, 5706, 5760, 5761, 5762, 5774, 5775, 5882, 5883, 5884, 5911, 5912, 5921, 5934, 5966, 5967, 5969, 5970, 6099, 6100, 6152, 6153, 6207, 6208, 6340, 6341, 6380, 6381, 6500, 6501, 6503, 6504, 6516, 6517, 6519, 6520, 6559 ] }
{ "red_pajama_v2": { "ccnet_original_length": 6559, "ccnet_original_nlines": 139, "rps_doc_curly_bracket": 0, "rps_doc_ldnoobw_words": 0, "rps_doc_lorem_ipsum": 0, "rps_doc_stop_word_fraction": 0.289166659116745, "rps_doc_ut1_blacklist": 0, "rps_doc_frac_all_caps_words": 0.04500000178813934, "rps_doc_frac_lines_end_with_ellipsis": 0, "rps_doc_frac_no_alph_words": 0.2058333307504654, "rps_doc_frac_unique_words": 0.439461886882782, "rps_doc_mean_word_length": 5.921524524688721, "rps_doc_num_sentences": 57, "rps_doc_symbol_to_word_ratio": 0, "rps_doc_unigram_entropy": 5.405595302581787, "rps_doc_word_count": 892, "rps_doc_frac_chars_dupe_10grams": 0, "rps_doc_frac_chars_dupe_5grams": 0, "rps_doc_frac_chars_dupe_6grams": 0, "rps_doc_frac_chars_dupe_7grams": 0, "rps_doc_frac_chars_dupe_8grams": 0, "rps_doc_frac_chars_dupe_9grams": 0, "rps_doc_frac_chars_top_2gram": 0.030291559174656868, "rps_doc_frac_chars_top_3gram": 0.009655429981648922, "rps_doc_frac_chars_top_4gram": 0.007383570075035095, "rps_doc_books_importance": -602.8956909179688, "rps_doc_books_importance_length_correction": -602.8956909179688, "rps_doc_openwebtext_importance": -270.4044189453125, "rps_doc_openwebtext_importance_length_correction": -270.4044189453125, "rps_doc_wikipedia_importance": -247.67587280273438, "rps_doc_wikipedia_importance_length_correction": -247.67587280273438 }, "fasttext": { "dclm": 0.06236064061522484, "english": 0.8096925020217896, "fineweb_edu_approx": 2.265559673309326, "eai_general_math": 0.05254077911376953, "eai_open_web_math": 0.056806620210409164, "eai_web_code": 0.17537593841552734 } }
{ "free_decimal_correspondence": { "primary": { "code": "005.822", "labels": { "level_1": "General works, books and libraries, information sciences", "level_2": "", "level_3": "Computer programming" } }, "secondary": { "code": "005.82", "labels": { "level_1": "General works, books and libraries, information sciences", "level_2": "", "level_3": "Computer programming" } } }, "bloom_cognitive_process": { "primary": { "code": "3", "label": "Apply" }, "secondary": { "code": "2", "label": "Understand" } }, "bloom_knowledge_domain": { "primary": { "code": "3", "label": "Procedural" }, "secondary": { "code": "2", "label": "Conceptual" } }, "document_type_v1": { "primary": { "code": "3", "label": "Reference/Encyclopedic/Educational" }, "secondary": { "code": "-1", "label": "Abstain" } }, "extraction_artifacts": { "primary": { "code": "0", "label": "No Artifacts" }, "secondary": { "code": "3", "label": "Irrelevant Content" } }, "missing_content": { "primary": { "code": "4", "label": "Missing Images or Figures" }, "secondary": { "code": "-1", "label": "Abstain" } }, "document_type_v2": { "primary": { "code": "8", "label": "Documentation" }, "secondary": { "code": "23", "label": "Tutorial" } }, "reasoning_depth": { "primary": { "code": "4", "label": "Advanced Reasoning" }, "secondary": { "code": "3", "label": "Intermediate Reasoning" } }, "technical_correctness": { "primary": { "code": "4", "label": "Highly Correct" }, "secondary": { "code": "3", "label": "Mostly Correct" } }, "education_level": { "primary": { "code": "4", "label": "Graduate/Expert Level" }, "secondary": { "code": "3", "label": "Undergraduate Level" } } }
672f1e42c33a7f9846924a2431ea77df
-7,501,426,422,925,652,000
AWI cursor jumps to end of line when updating Path in File Event Text field search cancel AWI cursor jumps to end of line when updating Path in File Event Text field book Article ID: 85100 calendar_today Updated On: Products CA Automic Workload Automation - Automation Engine Issue/Introduction Error Message : N/A When using Automic’s Web Interface (AWI) in version 12, the cursor will jump to the end of the field when updating the path of File Events.   Investigation   1. Create a File Event  and define the path as:  C:\test\test.txt  1. Put your cursor right after C:\test\ and update the path to:  C:\test\test2\test.txt   Results   Actual:  The cursor will bounce to the end of the text field when you attempt to type a character.   Expected:  The cursor will stay where you put it while you type the changes.   Environment OS Version: N/A Cause Cause type: Defect Root Cause: N/A Resolution Update to a fix version listed below or a newer version if available. Fix Status: Released Fix Version(s): Component(s): User Interface Automic Web Interface 12.1.0 Automic Web Interface 12.0.3 Additional Information Workaround : Either 1. Write the entire text in the field initially as it should be. OR 1. Use another editor to type the text and then paste it into the text field.
{ "url": "https://knowledge.broadcom.com/external/article/85100/awi-cursor-jumps-to-end-of-line-when-upd.html", "source_domain": "knowledge.broadcom.com", "snapshot_id": "CC-MAIN-2023-14", "warc_metadata": { "Content-Length": "50059", "Content-Type": "application/http; msgtype=response", "WARC-Block-Digest": "sha1:UB442ERRWZSJLUUQGOO6KM33YUQQ45JY", "WARC-Concurrent-To": "<urn:uuid:b5113598-e7bc-4e61-9a9d-fcf0b2f25287>", "WARC-Date": "2023-03-27T00:31:19Z", "WARC-IP-Address": "104.17.239.92", "WARC-Identified-Payload-Type": "text/html", "WARC-Payload-Digest": "sha1:GSNYFQCLFEVNUWHGLCOFK3QDJE6VG5PO", "WARC-Record-ID": "<urn:uuid:e5f538b4-da8d-4ce0-ba67-525cc8539a9f>", "WARC-Target-URI": "https://knowledge.broadcom.com/external/article/85100/awi-cursor-jumps-to-end-of-line-when-upd.html", "WARC-Truncated": null, "WARC-Type": "response", "WARC-Warcinfo-ID": "<urn:uuid:e2f1fe0a-4b3e-41d2-b154-edd828c15729>" }, "warc_info": "isPartOf: CC-MAIN-2023-14\r\npublisher: Common Crawl\r\ndescription: Wide crawl of the web for March/April 2023\r\noperator: Common Crawl Admin ([email protected])\r\nhostname: ip-10-67-67-159\r\nsoftware: Apache Nutch 1.19 (modified, https://github.com/commoncrawl/nutch/)\r\nrobots: checked via crawler-commons 1.4-SNAPSHOT (https://github.com/crawler-commons/crawler-commons)\r\nformat: WARC File Format 1.1\r\nconformsTo: https://iipc.github.io/warc-specifications/specifications/warc-format/warc-1.1/" }
{ "line_start_idx": [ 0, 76, 90, 91, 167, 168, 173, 174, 192, 193, 208, 209, 221, 222, 231, 232, 283, 284, 303, 304, 320, 324, 325, 465, 467, 481, 483, 534, 552, 619, 642, 644, 652, 654, 753, 755, 832, 833, 835, 836, 848, 849, 865, 866, 872, 873, 885, 892, 908, 909, 920, 921, 991, 992, 1013, 1014, 1030, 1059, 1060, 1089, 1118, 1119, 1142, 1143, 1156, 1163, 1230, 1233 ], "line_end_idx": [ 76, 90, 91, 167, 168, 173, 174, 192, 193, 208, 209, 221, 222, 231, 232, 283, 284, 303, 304, 320, 324, 325, 465, 467, 481, 483, 534, 552, 619, 642, 644, 652, 654, 753, 755, 832, 833, 835, 836, 848, 849, 865, 866, 872, 873, 885, 892, 908, 909, 920, 921, 991, 992, 1013, 1014, 1030, 1059, 1060, 1089, 1118, 1119, 1142, 1143, 1156, 1163, 1230, 1233, 1312 ] }
{ "red_pajama_v2": { "ccnet_original_length": 1312, "ccnet_original_nlines": 67, "rps_doc_curly_bracket": 0, "rps_doc_ldnoobw_words": 0, "rps_doc_lorem_ipsum": 0, "rps_doc_stop_word_fraction": 0.256317675113678, "rps_doc_ut1_blacklist": 0, "rps_doc_frac_all_caps_words": 0.05776172876358032, "rps_doc_frac_lines_end_with_ellipsis": 0, "rps_doc_frac_no_alph_words": 0.2166064977645874, "rps_doc_frac_unique_words": 0.5095238089561462, "rps_doc_mean_word_length": 4.757143020629883, "rps_doc_num_sentences": 16, "rps_doc_symbol_to_word_ratio": 0, "rps_doc_unigram_entropy": 4.36013126373291, "rps_doc_word_count": 210, "rps_doc_frac_chars_dupe_10grams": 0.12212211638689041, "rps_doc_frac_chars_dupe_5grams": 0.14814814925193787, "rps_doc_frac_chars_dupe_6grams": 0.12212211638689041, "rps_doc_frac_chars_dupe_7grams": 0.12212211638689041, "rps_doc_frac_chars_dupe_8grams": 0.12212211638689041, "rps_doc_frac_chars_dupe_9grams": 0.12212211638689041, "rps_doc_frac_chars_top_2gram": 0.0200200192630291, "rps_doc_frac_chars_top_3gram": 0.039039041846990585, "rps_doc_frac_chars_top_4gram": 0.03203203156590462, "rps_doc_books_importance": -115.84977722167969, "rps_doc_books_importance_length_correction": -115.84977722167969, "rps_doc_openwebtext_importance": -71.05986022949219, "rps_doc_openwebtext_importance_length_correction": -71.05986022949219, "rps_doc_wikipedia_importance": -23.625001907348633, "rps_doc_wikipedia_importance_length_correction": -23.624954223632812 }, "fasttext": { "dclm": 0.10279583930969238, "english": 0.7027341723442078, "fineweb_edu_approx": 2.543802499771118, "eai_general_math": 0.014715549536049366, "eai_open_web_math": 0.30610519647598267, "eai_web_code": 0.01357061043381691 } }
{ "free_decimal_correspondence": { "primary": { "code": "005.4", "labels": { "level_1": "General works, books and libraries, information sciences", "level_2": "", "level_3": "Computer programming" } }, "secondary": { "code": "004.67", "labels": { "level_1": "General works, books and libraries, information sciences", "level_2": "", "level_3": "Computers and Computer science" } } }, "bloom_cognitive_process": { "primary": { "code": "2", "label": "Understand" }, "secondary": { "code": "3", "label": "Apply" } }, "bloom_knowledge_domain": { "primary": { "code": "3", "label": "Procedural" }, "secondary": { "code": "1", "label": "Factual" } }, "document_type_v1": { "primary": { "code": "3", "label": "Reference/Encyclopedic/Educational" }, "secondary": { "code": "-1", "label": "Abstain" } }, "extraction_artifacts": { "primary": { "code": "0", "label": "No Artifacts" }, "secondary": { "code": "3", "label": "Irrelevant Content" } }, "missing_content": { "primary": { "code": "0", "label": "No missing content" }, "secondary": { "code": "-1", "label": "Abstain" } }, "document_type_v2": { "primary": { "code": "10", "label": "Knowledge Article" }, "secondary": { "code": "21", "label": "Customer Support" } }, "reasoning_depth": { "primary": { "code": "2", "label": "Basic Reasoning" }, "secondary": { "code": "3", "label": "Intermediate Reasoning" } }, "technical_correctness": { "primary": { "code": "4", "label": "Highly Correct" }, "secondary": { "code": "3", "label": "Mostly Correct" } }, "education_level": { "primary": { "code": "2", "label": "High School Level" }, "secondary": { "code": "3", "label": "Undergraduate Level" } } }
672f1e42c33a7f9846924a2431ea77df
-8,462,593,475,153,746,000
spring-security Spring Security config with java (not XML) 30% OFF - 9th Anniversary discount on Entity Framework Extensions until December 15 with code: ZZZANNIVERSARY9 Introduction Typical database backed, annotation base spring security setup. Syntax 1. configureGlobal() configure the auth object. 2. The later two SQLs may be optional. 3. configure() method tells spring mvc how to authenticate request 4. some url we do not need to authenticate 5. others will redirect to /login if not yet authenticated. Got any spring-security Question?
{ "url": "https://riptutorial.com/spring-security/topic/8700/spring-security-config-with-java--not-xml-", "source_domain": "riptutorial.com", "snapshot_id": "CC-MAIN-2023-50", "warc_metadata": { "Content-Length": "24318", "Content-Type": "application/http; msgtype=response", "WARC-Block-Digest": "sha1:53ABIKVLSJOTBXU3D2C4J2QZUC6WIWDQ", "WARC-Concurrent-To": "<urn:uuid:8c1ac53f-660e-4596-836b-bd64ecc971bf>", "WARC-Date": "2023-12-08T01:38:26Z", "WARC-IP-Address": "40.83.160.29", "WARC-Identified-Payload-Type": "text/html", "WARC-Payload-Digest": "sha1:WBN3C35JHG7S6YZX4Q5V4MMTRD5UCWQB", "WARC-Record-ID": "<urn:uuid:21887428-003a-4301-a5e3-ae591b7dbc58>", "WARC-Target-URI": "https://riptutorial.com/spring-security/topic/8700/spring-security-config-with-java--not-xml-", "WARC-Truncated": null, "WARC-Type": "response", "WARC-Warcinfo-ID": "<urn:uuid:e99a9ab7-6a4e-4c9e-b285-d5485449aa8f>" }, "warc_info": "isPartOf: CC-MAIN-2023-50\r\npublisher: Common Crawl\r\ndescription: Wide crawl of the web for November/December 2023\r\noperator: Common Crawl Admin ([email protected])\r\nhostname: ip-10-67-67-117\r\nsoftware: Apache Nutch 1.19 (modified, https://github.com/commoncrawl/nutch/)\r\nrobots: checked via crawler-commons 1.5-SNAPSHOT (https://github.com/crawler-commons/crawler-commons)\r\nformat: WARC File Format 1.1\r\nconformsTo: https://iipc.github.io/warc-specifications/specifications/warc-format/warc-1.1/" }
{ "line_start_idx": [ 0, 59, 60, 171, 172, 185, 186, 250, 251, 258, 259, 309, 350, 419, 464, 526, 527, 528 ], "line_end_idx": [ 59, 60, 171, 172, 185, 186, 250, 251, 258, 259, 309, 350, 419, 464, 526, 527, 528, 561 ] }
{ "red_pajama_v2": { "ccnet_original_length": 561, "ccnet_original_nlines": 17, "rps_doc_curly_bracket": 0, "rps_doc_ldnoobw_words": 0, "rps_doc_lorem_ipsum": 0, "rps_doc_stop_word_fraction": 0.24271844327449799, "rps_doc_ut1_blacklist": 0, "rps_doc_frac_all_caps_words": 0.029126210138201714, "rps_doc_frac_lines_end_with_ellipsis": 0, "rps_doc_frac_no_alph_words": 0.27184465527534485, "rps_doc_frac_unique_words": 0.8500000238418579, "rps_doc_mean_word_length": 5.512499809265137, "rps_doc_num_sentences": 10, "rps_doc_symbol_to_word_ratio": 0, "rps_doc_unigram_entropy": 4.154460906982422, "rps_doc_word_count": 80, "rps_doc_frac_chars_dupe_10grams": 0, "rps_doc_frac_chars_dupe_5grams": 0, "rps_doc_frac_chars_dupe_6grams": 0, "rps_doc_frac_chars_dupe_7grams": 0, "rps_doc_frac_chars_dupe_8grams": 0, "rps_doc_frac_chars_dupe_9grams": 0, "rps_doc_frac_chars_top_2gram": 0.0634920597076416, "rps_doc_frac_chars_top_3gram": 0, "rps_doc_frac_chars_top_4gram": 0, "rps_doc_books_importance": -46.059200286865234, "rps_doc_books_importance_length_correction": -60.22682571411133, "rps_doc_openwebtext_importance": -20.323320388793945, "rps_doc_openwebtext_importance_length_correction": -34.490943908691406, "rps_doc_wikipedia_importance": -21.428688049316406, "rps_doc_wikipedia_importance_length_correction": -35.5963134765625 }, "fasttext": { "dclm": 0.22082436084747314, "english": 0.716722309589386, "fineweb_edu_approx": 1.3829318284988403, "eai_general_math": 0.032573580741882324, "eai_open_web_math": 0.052179399877786636, "eai_web_code": 0.0030489598866552114 } }
{ "free_decimal_correspondence": { "primary": { "code": "005.82", "labels": { "level_1": "General works, books and libraries, information sciences", "level_2": "", "level_3": "Computer programming" } }, "secondary": { "code": "005.138", "labels": { "level_1": "General works, books and libraries, information sciences", "level_2": "", "level_3": "Computer programming" } } }, "bloom_cognitive_process": { "primary": { "code": "3", "label": "Apply" }, "secondary": { "code": "2", "label": "Understand" } }, "bloom_knowledge_domain": { "primary": { "code": "3", "label": "Procedural" }, "secondary": { "code": "2", "label": "Conceptual" } }, "document_type_v1": { "primary": { "code": "3", "label": "Reference/Encyclopedic/Educational" }, "secondary": { "code": "4", "label": "Code/Software" } }, "extraction_artifacts": { "primary": { "code": "0", "label": "No Artifacts" }, "secondary": { "code": "3", "label": "Irrelevant Content" } }, "missing_content": { "primary": { "code": "0", "label": "No missing content" }, "secondary": { "code": "-1", "label": "Abstain" } }, "document_type_v2": { "primary": { "code": "23", "label": "Tutorial" }, "secondary": { "code": "8", "label": "Documentation" } }, "reasoning_depth": { "primary": { "code": "2", "label": "Basic Reasoning" }, "secondary": { "code": "1", "label": "No Reasoning" } }, "technical_correctness": { "primary": { "code": "4", "label": "Highly Correct" }, "secondary": { "code": "3", "label": "Mostly Correct" } }, "education_level": { "primary": { "code": "3", "label": "Undergraduate Level" }, "secondary": { "code": "2", "label": "High School Level" } } }
672f1e42c33a7f9846924a2431ea77df
-2,007,909,010,696,932,000
Take the 2-minute tour × TeX - LaTeX Stack Exchange is a question and answer site for users of TeX, LaTeX, ConTeXt, and related typesetting systems. It's 100% free, no registration required. This question already has an answer here: I have the following formula in my LaTeX code: $ \mu_{x_{t}} = E(x_{t}) = \int_{a}^{b} $ This gives me the follwing in my document enter image description here Now I tried to use \bigint, but that gives me too thick integral... what do I need to do to get the same thickness but with more height? share|improve this question marked as duplicate by barbara beeton, Papiro, Gonzalo Medina, Sean Allred, egreg Jul 4 at 23:10 This question has been asked before and already has an answer. If those answers do not fully address your question, please ask a new question. 3   Like what happens when you use \displaysize? Or may be you want to use \[\int…\] instead of $\int…$? –  Khaled Hosny Jan 25 '13 at 17:44      Oh I didn't know that \displaysize, I will try thnx! I tried \[\int..\] but then my formula moved horizontally much more right and vertically more down?! x) –  jjepsuomi Jan 25 '13 at 17:48 1   For future reference, have a look at the Not So Short Introduction to LaTeX –  cmhughes Jan 25 '13 at 17:49 4   @KhaledHosny Shouldn't it be \displaystyle? However, a big integral in an inline formula will irremediably spoil the page. –  egreg Jan 25 '13 at 17:49      @egreg: you are right of course :) –  Khaled Hosny Jan 25 '13 at 17:52 2 Answers 2 up vote 9 down vote accepted More height comes with display mode: \[ \mu_{x_{t}} = E(x_{t}) = \int_{a}^{b} \] Display mode Or you can only make the integral larger: $ \mu_{x_{t}} = E(x_{t}) = {\displaystyle \int_{a}^{b} } $ Displaystyle share|improve this answer      Exactly what I was looking for! Thank you! =) –  jjepsuomi Jan 25 '13 at 17:55 There are a few ways to change the appearance- \displaystyle is one of them, and you can also use \limits, or even a combination: enter image description here \documentclass{article} \begin{document} \begin{itemize} \item $ \mu_{x_{t}} = E(x_{t}) = \int_{a}^{b}$ \item $\mu_{x_{t}} = E(x_{t}) = \int\limits_{a}^{b} $ \item $ \mu_{x_{t}} = E(x_{t}) = \displaystyle\int_{a}^{b} $ \item $ \mu_{x_{t}} = E(x_{t}) = \displaystyle\int\limits_{a}^{b} $ \end{itemize} \end{document} For future reference, have a look at the Not so short Introduction to LaTeX share|improve this answer      Thank you! I will =) It's you know sometimes just easier to ask 5 second thing from someone than to read whole book. Sometimes you do find the 5 second thing right away and sometimes you don't and when you don't it usually can scale up to hours, because when you look at one thing then you need to reference another thing and then another and that takes too much time for every single character in thesis ;) –  jjepsuomi Jan 25 '13 at 18:03 Not the answer you're looking for? Browse other questions tagged or ask your own question.
{ "url": "http://tex.stackexchange.com/questions/95359/how-do-i-make-an-integral-symbol-larger-in-in-line-math-mode", "source_domain": "tex.stackexchange.com", "snapshot_id": "crawl=CC-MAIN-2015-35", "warc_metadata": { "Content-Length": "73525", "Content-Type": "application/http; msgtype=response", "WARC-Block-Digest": "sha1:SA53JPMRY5335XHUHVIR4OZJPD6MUTPQ", "WARC-Concurrent-To": "<urn:uuid:3a8b8fd7-2e86-467d-94ee-59beea2dc07e>", "WARC-Date": "2015-09-01T08:03:42Z", "WARC-IP-Address": "104.16.16.128", "WARC-Identified-Payload-Type": null, "WARC-Payload-Digest": "sha1:3OKI5NVGIAQPYYPZH56CW5IATKBUET3P", "WARC-Record-ID": "<urn:uuid:8d9193dc-9eb1-4039-882e-89851df71ff9>", "WARC-Target-URI": "http://tex.stackexchange.com/questions/95359/how-do-i-make-an-integral-symbol-larger-in-in-line-math-mode", "WARC-Truncated": "length", "WARC-Type": "response", "WARC-Warcinfo-ID": "<urn:uuid:65af077d-fac4-4aad-b327-49885d484665>" }, "warc_info": "robots: classic\r\nhostname: ip-10-171-96-226.ec2.internal\r\nsoftware: Nutch 1.6 (CC)/CC WarcExport 1.0\r\nisPartOf: CC-MAIN-2015-35\r\noperator: CommonCrawl Admin\r\ndescription: Wide crawl of the web for August 2015\r\npublisher: CommonCrawl\r\nformat: WARC File Format 1.0\r\nconformsTo: http://bibnum.bnf.fr/WARC/WARC_ISO_28500_version1_latestdraft.pdf" }
{ "line_start_idx": [ 0, 25, 191, 192, 234, 235, 282, 283, 325, 326, 368, 369, 398, 399, 536, 537, 565, 566, 663, 664, 807, 808, 812, 949, 954, 1144, 1148, 1256, 1260, 1412, 1417, 1488, 1489, 1501, 1502, 1531, 1532, 1569, 1570, 1614, 1615, 1628, 1629, 1671, 1672, 1731, 1732, 1745, 1746, 1772, 1777, 1856, 1857, 1987, 1988, 2017, 2018, 2042, 2043, 2060, 2061, 2077, 2128, 2186, 2251, 2323, 2337, 2338, 2353, 2354, 2430, 2431, 2457, 2462, 2903, 2904 ], "line_end_idx": [ 25, 191, 192, 234, 235, 282, 283, 325, 326, 368, 369, 398, 399, 536, 537, 565, 566, 663, 664, 807, 808, 812, 949, 954, 1144, 1148, 1256, 1260, 1412, 1417, 1488, 1489, 1501, 1502, 1531, 1532, 1569, 1570, 1614, 1615, 1628, 1629, 1671, 1672, 1731, 1732, 1745, 1746, 1772, 1777, 1856, 1857, 1987, 1988, 2017, 2018, 2042, 2043, 2060, 2061, 2077, 2128, 2186, 2251, 2323, 2337, 2338, 2353, 2354, 2430, 2431, 2457, 2462, 2903, 2904, 2994 ] }
{ "red_pajama_v2": { "ccnet_original_length": 2994, "ccnet_original_nlines": 75, "rps_doc_curly_bracket": 0.027388110756874084, "rps_doc_ldnoobw_words": 0, "rps_doc_lorem_ipsum": 0, "rps_doc_stop_word_fraction": 0.331134557723999, "rps_doc_ut1_blacklist": 0, "rps_doc_frac_all_caps_words": 0.019788920879364014, "rps_doc_frac_lines_end_with_ellipsis": 0, "rps_doc_frac_no_alph_words": 0.34960421919822693, "rps_doc_frac_unique_words": 0.48093220591545105, "rps_doc_mean_word_length": 4.491525650024414, "rps_doc_num_sentences": 19, "rps_doc_symbol_to_word_ratio": 0.003957780078053474, "rps_doc_unigram_entropy": 5.113213062286377, "rps_doc_word_count": 472, "rps_doc_frac_chars_dupe_10grams": 0.05754717066884041, "rps_doc_frac_chars_dupe_5grams": 0.11650943011045456, "rps_doc_frac_chars_dupe_6grams": 0.10424528270959854, "rps_doc_frac_chars_dupe_7grams": 0.07735849171876907, "rps_doc_frac_chars_dupe_8grams": 0.05754717066884041, "rps_doc_frac_chars_dupe_9grams": 0.05754717066884041, "rps_doc_frac_chars_top_2gram": 0.023113209754228592, "rps_doc_frac_chars_top_3gram": 0.023113209754228592, "rps_doc_frac_chars_top_4gram": 0.02971697971224785, "rps_doc_books_importance": -379.02618408203125, "rps_doc_books_importance_length_correction": -379.02618408203125, "rps_doc_openwebtext_importance": -188.1312713623047, "rps_doc_openwebtext_importance_length_correction": -188.1312713623047, "rps_doc_wikipedia_importance": -103.7966537475586, "rps_doc_wikipedia_importance_length_correction": -103.7966537475586 }, "fasttext": { "dclm": 0.06053448095917702, "english": 0.8656782507896423, "fineweb_edu_approx": 1.692034125328064, "eai_general_math": 0.17954248189926147, "eai_open_web_math": 0.7868462800979614, "eai_web_code": 0.00031447000219486654 } }
{ "free_decimal_correspondence": { "primary": { "code": "005.1", "labels": { "level_1": "General works, books and libraries, information sciences", "level_2": "", "level_3": "Computer programming" } }, "secondary": { "code": "510", "labels": { "level_1": "Science and Natural history", "level_2": "Mathematics", "level_3": "" } } }, "bloom_cognitive_process": { "primary": { "code": "3", "label": "Apply" }, "secondary": { "code": "2", "label": "Understand" } }, "bloom_knowledge_domain": { "primary": { "code": "3", "label": "Procedural" }, "secondary": { "code": "2", "label": "Conceptual" } }, "document_type_v1": { "primary": { "code": "5", "label": "Social/Forum" }, "secondary": { "code": "3", "label": "Reference/Encyclopedic/Educational" } }, "extraction_artifacts": { "primary": { "code": "3", "label": "Irrelevant Content" }, "secondary": { "code": "-1", "label": "Abstain" } }, "missing_content": { "primary": { "code": "4", "label": "Missing Images or Figures" }, "secondary": { "code": "-1", "label": "Abstain" } }, "document_type_v2": { "primary": { "code": "18", "label": "Q&A Forum" }, "secondary": { "code": "23", "label": "Tutorial" } }, "reasoning_depth": { "primary": { "code": "2", "label": "Basic Reasoning" }, "secondary": { "code": "3", "label": "Intermediate Reasoning" } }, "technical_correctness": { "primary": { "code": "4", "label": "Highly Correct" }, "secondary": { "code": "3", "label": "Mostly Correct" } }, "education_level": { "primary": { "code": "3", "label": "Undergraduate Level" }, "secondary": { "code": "2", "label": "High School Level" } } }
672f1e42c33a7f9846924a2431ea77df